pharmacology pre test self assessment and review (14th ed.)

287

Upload: kj-chathura-d-senarathna

Post on 13-May-2015

5.205 views

Category:

Education


20 download

TRANSCRIPT

Page 1: Pharmacology pre test self assessment and review (14th ed.)
Page 2: Pharmacology pre test self assessment and review (14th ed.)

Notice Medicine i s an ever-changing science. As new research and cl inica l experience broaden our knowledge, changes in treatment and drug therapy arerequired. The authors and the publ i sher of this work have checked with sources bel ieved to be rel iable in thei r efforts to provide information thatis complete and genera l ly in accord with the s tandards accepted at the time of publ ication. However, in view of the poss ibi l i ty of human error orchanges in medica l sciences , nei ther the authors nor the publ i sher nor any other party who has been involved in the preparation or publ ication ofthis work warrants that the information conta ined herein i s in every respect accurate or complete, and they discla im a l l respons ibi l i ty for anyerrors or omiss ions or for the resul ts obta ined from use of the information conta ined in this work. Readers are encouraged to confi rm theinformation conta ined herein with other sources . For example and in particular, readers are advised to check the product information sheetincluded in the package of each drug they plan to adminis ter to be certa in that the information conta ined in this work i s accurate and thatchanges have not been made in the recommended dose or in the contra indications for adminis tration. This recommendation i s of particularimportance in connection with new or infrequently used drugs .

Page 3: Pharmacology pre test self assessment and review (14th ed.)
Page 4: Pharmacology pre test self assessment and review (14th ed.)
Page 5: Pharmacology pre test self assessment and review (14th ed.)

Copyright © 2013 by McGraw-Hi l l Education, LLC. Al l rights reserved. Except as permitted under the United States Copyright Act of 1976, no part of thispubl ication may be reproduced or dis tributed in any form or by any means , or s tored in a database or retrieva l system, without the prior wri ttenpermiss ion of the publ i sher.

ISBN: 978-0-07-179147-2MHID: 0-07-179147-7

The materia l in this eBook a lso appears in the print vers ion of this ti tle: ISBN: 978-0-07-179146-5, MHID: 0-07-179146-9.

Al l trademarks are trademarks of thei r respective owners . Rather than put a trademark symbol after every occurrence of a trademarked name, weuse names in an edi toria l fashion only, and to the benefi t of the trademark owner, with no intention of infringement of the trademark. Wheresuch des ignations appear in this book, they have been printed with ini tia l caps .

McGraw-Hi l l Education eBooks are ava i lable at specia l quanti ty discounts to use as premiums and sa les promotions , or for use in corporatetra ining programs. To contact a representative please e-mai l us at bulksa les@mcgraw-hi l l .com.

PreTest™ is a trademark of McGraw-Hi l l Education LLC.

TERMS OF USE

This i s a copyrighted work and McGraw-Hi l l Education and i ts l i censors reserve a l l rights in and to the work. Use of this work i s subject to theseterms. Except as permitted under the Copyright Act of 1976 and the right to s tore and retrieve one copy of the work, you may not decompi le,disassemble, reverse engineer, reproduce, modi fy, create derivative works based upon, transmit, dis tribute, disseminate, sel l , publ i sh orsubl icense the work or any part of i t wi thout McGraw-Hi l l Education’s prior consent. You may use the work for your own noncommercia l andpersonal use; any other use of the work i s s trictly prohibi ted. Your right to use the work may be terminated i f you fa i l to comply with these terms.

THE WORK IS PROVIDED “AS IS.” McGRAW-HILL EDUCATION AND ITS LICENSORS MAKE NO GUARANTEES OR WARRANTIES AS TO THE ACCURACY, ADEQUACYOR COMPLETENESS OF OR RESULTS TO BE OBTAINED FROM USING THE WORK, INCLUDING ANY INFORMATION THAT CAN BE ACCESSED THROUGH THEWORK VIA HYPERLINK OR OTHERWISE, AND EXPRESSLY DISCLAIM ANY WARRANTY, EXPRESS OR IMPLIED, INCLUDING BUT NOT LIMITED TO IMPLIEDWARRANTIES OF MERCHANTABILITY OR FITNESS FOR A PARTICULAR PURPOSE. McGraw-Hi l l Education and i ts l i censors do not warrant or guaranteethat the functions conta ined in the work wi l l meet your requirements or that i ts operation wi l l be uninterrupted or error free. Nei ther McGraw-Hi l lEducation nor i ts l i censors sha l l be l iable to you or anyone else for any inaccuracy, error or omiss ion, regardless of cause, in the work or for anydamages resul ting therefrom. McGraw-Hi l l Education has no respons ibi l i ty for the content of any information accessed through the work. Under noci rcumstances sha l l McGraw-Hi l l Education and/or i ts l i censors be l iable for any indirect, incidenta l , specia l , puni tive, consequentia l or s imi lardamages that resul t from the use of or inabi l i ty to use the work, even i f any of them has been advised of the poss ibi l i ty of such damages . Thisl imitation of l iabi l i ty sha l l apply to any cla im or cause whatsoever whether such cla im or cause arises in contract, tort or otherwise.

Page 6: Pharmacology pre test self assessment and review (14th ed.)

Reviewers

David Jonathan KadouriUnivers i ty of Medicine and Heal th Sciences—St. Ki tts

Medica l StudentClass of 2013

Matthew KlairmontRush Medica l Col lege

Medica l StudentClass of 2013

Nikul Patel, MDUnivers i ty of Medicine and Dentis try of New Jersey

School of Medicine

Page 7: Pharmacology pre test self assessment and review (14th ed.)

Contents

Preface

Introduction

Cross-References (for Answers) to Selected Pharmacology Texts

Generic Drug Name Recognition Guide

List of Abbreviations

General Principles of PharmacologyQuestionsAnswers

The Peripheral Nervous Systems: Autonomic and Somatic Nervous System PharmacologyQuestionsAnswers

Central Nervous System PharmacologyQuestionsAnswers

Cardiovascular PharmacologyQuestionsAnswers

Renal System and Diuretic PharmacologyQuestionsAnswers

Respiratory System Pharmacology: Asthma and COPDQuestionsAnswers

Autacoids and Anti-inflammatory Drug PharmacologyQuestionsAnswers

Gastrointestinal and Urinary Tract Pharmacology, Nutrition (Vitamins)QuestionsAnswers

Endocrine and Reproductive PharmacologyQuestionsAnswers

Antimicrobial and Antiviral PharmacologyQuestionsAnswers

Cancer and Immune System PharmacologyQuestionsAnswers

Toxicology, Bioterrorism, and Chemical Warfare AgentsQuestionsAnswers

Appendices

Index

Page 8: Pharmacology pre test self assessment and review (14th ed.)

Preface Welcome to this , the 14th edi tion, of Pharmacology: PreTest™ Self-Assessment and Review. I ’m pleased to have been invi ted back to wri te this edi tionafter doing the previous three. Whether you’re s tudying for Step 1 of the USMLE, or for a course exam that includes pharmacology content, I thinkyou’l l find this helpful .

I bel ieve this PreTest™ Self-Assessment and Review wi l l help you eva luate and review your intens ive and extens ive knowledge of pharmacologyand therapeutics—your knowledge of bas ic facts and principles , and your abi l i ty to apply that knowledge to some common cl inica l s i tuations .

Among the changes here you’l l find in “14/e” for pharmacology are:

• Many new or extens ively revised questions , most based on cl inica l vignettes or scenarios , most a l ready tested on hundreds of fi rs t- and second-year medica l s tudents , and a l l but a select few in a format you’l l l i kely see on Step 1. Many changes were necess i tated by the approval of newdrugs that you need to know about in some way. Many were made in response to excel lent comments from medica l s tudent and house officerreviewers .

• More integration of content between the various areas of pharmacology and therapeutics , with many questions that encourage you to integratenew materia l with content presented earl ier.

• A better blend of questions that integrate bas ic pharmacology content with bas ic information from other precl inica l discipl ines .• Clearer and more complete explanations for why correct answers are correct, and the others are not.• Updated cross -references to the latest edi tions of two widely used pharmacology texts , in the answers , so you can find additional information or

explanations i f you wish.• An updated resource that lets you look at suffixes of generic drug names and deduce with reasonable (or better) certa inty the chemica l or

pharmacologic group or class to which a drug belongs .

Page 9: Pharmacology pre test self assessment and review (14th ed.)

Introduction Even though your profs may tell you otherwise, pharmacology is pure memorization … the ultimate challenge in medical memorization … some remedy to dull thepain of the subject is needed.

That was the admonition to s tudents in a popular exam study a id.“Baloney,” I say. Being a rea l i s t, there may be some truth to that. But, read on, and you’l l see why I don’t think that’s an absolute truth, and

certa inly i sn’t the best way to learn pharmacology.

My Perspectives on and Approaches to Learning “Pharm”

Before you get on with your s tudying, I thought I ’d give you a l i ttle ins ight into who another otherwise faceless author might be, because i t mightexpla in where I’m “coming from” when I prepared the questions , answers , and explanations you’l l find here. I ’ve had about 35 years of teachingmany areas of pharmacology to more Univers i ty of Michigan Medica l School s tudents than I can count, and i t’s a l l been a great pleasure andprivi lege for me. (Yup, I ’m an “old guy,” but for what i t’s worth I drive a bright yel low 2012 Camaro convertible with a s ix-speed manualtransmiss ion! Chronologic age i s a relative thing.)

I know that things perta ining to pharmacology and therapeutics are overwhelming in terms of breadth, deta i l , number, and consequences . Ageneration or so ago the pharmacologic armamentarium (and what you ostens ibly needed to know) was a smal l fraction of what i t i s now. We’vepassed the 10,000 drug mark, with new drugs coming at a mind-boggl ing frequency. Do you need to know about them a l l? Can you poss ibly betaught about, or be expected to learn, everything?

In my opinion, no.There’s s imply too much information presented to you in the precl inica l years , and even though a host of drugs “exis t” there’s l i ttle point in

knowing about them a l l expl ici tly. Trying to do so would be a futi le and needless task. But no matter how wel l you think you know yourinformation (pharmacologic or otherwise), no matter how completely or comprehens ively you’ve been taught, a l l the content tends to becomejumbled and incomprehens ible when you’re faced with the task of “knowing i t a l l ” a l l at once; that i s , when you hi t your fi rs t of severa l “Step”exams, and get on the wards . You may lose s ight of the proverbia l forest. Knowing too much about trees and too l i ttle about the forest they’re inmay not be sufficient once you get into a cl inica l s i tuation. It’s good to know many important facts , and even to be tested on them, but fact-basedknowledge a lone i sn’t sufficient. You can’t put yoursel f in the pos i tion of knowing so much about the deta i l s that you can’t think in broader terms.

Borrowing from the l i terature, you are expected to be l ike the cheerful Major Genera l in Gi lbert and Sul l ivan’s Pirates of Penzance. You seeminglyneed to know “a l l the facts” and be able to spi t them back a lmost without having to s tra in to think.

It i s rewarding to answer an ostens ibly compl icated or deta i led question correctly (you possess the main pos i tive attributes of Gi lbert andSul l ivan’s happy Major Genera l ), but you don’t want to find yoursel f so bogged down in knowing the deta i l s that you miss seeing the moreimportant big picture, or how the facts apply or relate to one another (the Major Genera l ’s main flaw). The s implest or most bas ic concepts can beoverlooked with teaching or learning that i s too deta i led in terms of fact and focus . You have had an abundant (i f not excess ive) amount ofinformation about pharmacology presented to you, but that’s only the foundation of a broad knowledge and experience base upon which you’l lbui ld over the coming years . (I ’l l a l so go out on a l imb and s tate that you’ve a lso been taught, and expected to learn, a ton of information that i strivia l or, at best, not necessary for your understanding and ul timate appl ication of bas ic pharmacology to cl inica l rea l i ty.)

One example that comes to mind (in fact, i t’s indel ibly etched in my memory) involves a fa i rly common cl inica l problem, gout. A doc in the ER,where I showed up one hot summer day with a bad gout attack, asked me what I did for a l iving. He then went on, a fter hearing I taught pharm, toci te every metabol ic intermediate and enzyme in the biosynthes is of uric acid by the so-ca l led purine (ATP) degradation pathway. This i s , ofcourse, the metabol ic crux of the problems in hyperuricemia and gout. This new doc (he just graduated from medica l school , wi th a Ph.D. degree toboot) correctly s tated that a l lopurinol inhibi ts the “las t two s teps” in uric acid synthes is by inhibi ting xanthine oxidase. He a lso correctly s tatedthat xanthine oxidase generates oxygen free radica ls and H2O2, which contributed to the pathophys iology. Inhibi t xanthine oxidase, and none ofthose nasty oxidants are produced, he sa id. That’s great. I gave him an A for his intens ive knowledge.

Unfortunately, that young phys ician then dogmatica l ly s tated what seemed so mechanis tica l ly rational but in actua l i ty was tota l ly wrong: Giventhe role of uric acid in the pathophys iology of gout, and the abi l i ty to inhibi t urate synthes is with a l lopurinol , his fi rs t choice therapy for my acutegout would be a l lopurinol . (Now, of course, we have the new a l lopurinol -l ike drug, febuxostat.) Oops . Wrong choice. He flunked my cri tica l test. Iknew he made a serious error, and I pol i tely decl ined his prescription as I checked mysel f out of the hospi ta l AMA (Against Medica l Advice) as heheaded off to fetch a polarizing microscope that he’d use after aspi rating my “hot toe.” But what i f I didn’t know anything about gout, orpharmacology? I’d be getting a decidedly inappropriate treatment from a decidedly smart phys ician, but after days on his selected drug I’dprobably be hurting and l imping worse than when I came to the ER. Knowing too much doesn’t mean knowing the right s tuff. Si tuations such asthis , whether they apply to gout or to any of hundreds of common cl inica l scenarios , wi l l apply to your patients… and to your fami ly, friends , andeven to you!

What Does Epinephrine Do to Blood Pressure?

Many years ago I learned about autonomic pharmacology from Professor Raymond P. Ahlquis t. You probably haven’t heard about him. He i s theperson who “invented” the concept of adrenergic receptors , way back in 1948, and coined the terms a lpha- and beta- as prefixes to adrenergicreceptors that we now so gl ibly refer to and have eventual ly characterized on the molecular level . That Lasker Award winner (oh, so close to aNobel Prize) has probably caused more consternation for medica l and other heal th care profess ions s tudents , given the fact that now we havemultiple a lpha- and beta-adrenergic receptors , and you need to learn about them, the drugs that activate them, and the increas ing number ofdrugs that block them. Dr Ahlquis t used to laugh loudly, not only about how his concept developed, but a lso about how there was (and s ti l l i s ) somuch more for you to learn about, understand, and apply. If he were s ti l l a l ive he’d s ti l l be chuckl ing.

Ahlquis t taught many s tudents about autonomic pharmacology, and there i s much to be taken away and appl ied to the bas ic yet essentia lknowledge base more than 70 years later; what YOU need to know about.

What i s the effect of epinephrine on blood pressure? he would ask. His s tudents would answer that epinephrine ra ises blood pressure. Thatanswer was incorrect. Next we’d say that epinephrine lowers blood pressure. That, too, was wrong. Ahlquis t’s answer was that the effects ofepinephrine on blood pressure—and indeed the responses to any other drug—“depends .” It depends on the dose, the adminis tration route, thecondition of the patient (eg, whether he or she has any other conditions , or i s taking any other drugs), and a whole bunch of other factors .

Learn this “i t depends” answer when you learn about a l l drugs . But remember, the questions in this text, and those you’l l read on your medschool and Step exams, are wri tten to el ici t the “most l ikely” correct answer.

And So, A Disclaimer of Sorts

Page 10: Pharmacology pre test self assessment and review (14th ed.)

Your experiences from the courses and exams you’ve taken may be qui te di fferent from those of s tudents in other medica l schools , the medica ls tudents I ’ve taught, or the very same s tudents here or elsewhere taught by someone else. And so the focus of questions you’l l see here may bedi fferent from those you’ve encountered on your exams, and the explanations may di ffer too. There i s no one “s tandard” pharmacology curriculumfor a l l medica l schools . Some have separate precl inica l courses for the bas ic sciences (anatomy, phys iology, pharm, and so on). Others , such asours , have a discipl ine-based curriculum in which, for example, bas ic science information appl icable to cardiovascular diseases are a l l in onesection (we ca l l them “sequences”).

And when we think about individual facul ty i t’s obvious that points emphas ized by a particular instructor may di ffer (sometimes markedly) inscope and orientation from those made by others . Some facul ty place cons iderable emphas is on deta i led or complex mechanisms of action,perhaps replete with such things as speci fic pathways of drug metabol i sm; chemica l s tructures , and perhaps s tructure–activi ty relationships ;mathematica l approaches to pharmacokinetics ; and so on. Students sometimes leave lecture wondering what the cl inica l impl ications are.Conversely, other facul ty address the cl inica l “relevance” of certa in drugs , but do l i ttle more than say “Your 50-year-old male patient has recentlybeen diagnosed with Type 2 diabetes mel l i tus . Metformin i s a good s tarting drug.” As Homer Simpson might say, “duh-oh, ok.” But why? When?How does i t work?

The same appl ies to pharmacology texts . Some may devote a page or more to a particular drug or topic, and in others that information may notappear at a l l .

Next there i s the i ssue of “newness” or timel iness . Drugs come and go a l l the time—although i t’s clear that new drugs are being approved at afaster rate than those that are disappearing. This presents somewhat of a predicament for you, and so, for me. What to learn, what to askquestions about. For decades s tudents have learned about d-tubocurarine (“curare”) as the prototype nondepolarizing skeleta l neuromuscularblocker: a competi tive antagonis t of acetylchol ine at NM (nicotinic-skeleta l muscle) receptors . It’s no longer used in the United States . Indeed, arelatively new curare-l ike drug, metocurine, i s no longer ava i lable here ei ther. If you were to look at not-so-old edi tions of a pharmacology text—or even the previous edi tion of this book—you’d find content related to the “gl i tazones” (thiazol idinediones , to be more precise), and learn or betested about a variety of drugs in this class of ora l drugs for Type 2 diabetes mel l i tus . How things have changed. Owing to various serious s ideeffects and adverse responses , some fata l , only one gl i tazone remains : piogl i tazone.

Then there i s the i ssue of insti tutional preferences in terms of drug use which, inferentia l ly, may affect what you’re taught and expected tolearn. Perhaps you’l l learn the bas ic pharmacology of ephedrine. For years i t has been an outmoded drug for caus ing modest bronchodi lation inasthma, and has ga ined more of a reputation as a CNS s timulant and weight-loss a id that was often abused or misprescribed. Nonetheless ,because of a variety of factors ephedrine gets relatively l i ttle attention, i f any attention at a l l , in many pharmacology lectures and texts . But at ourinsti tution, ephedrine i s used frequently in the operating rooms when the goal i s to cause a l i ttle upward “bl ip” of blood pressure that may havefa l len too much in response to certa in anesthetic agents .

You’l l a l so learn about revers ing skeleta l muscle para lys is caused by the nondepolarizing (curare-l ike) drugs : reverse the blockade by giving achol inesterase inhibi tor (mainly us ing neostigmine), but fi rs t give atropine to prevent unwanted s timulation of muscarinic receptors when thechol inesterase inhibi tor does i ts many things . Wel l , here and in many other insti tutions , atropine i s not used for that purpose. Instead, theantimuscarinic agent that’s used i s glycopyrrolate—yet another drug that may not be mentioned in lecture or given much, i f any, discuss ion in atext.

Now, I want to a l lay your concerns and give you confidence that you can learn this materia l , and apply what you probably have learned aboutfami l iar (taught) drugs to those you haven’t expl ici tly learned or read about. For example, you’l l find, in this book, at least one question aboutglycopyrrolate. But i t wi l l be described in such a way in terms of i ts actions and uses (eg, given right before an acetylchol inesterase inhibi tor i sadminis tered to reverse neuromuscular blockade, causes effects and s ide effects X, Y, and Z) that i f you “know” the bas ics about atropine, you’l lbe able to answer the question about glycopyrrolate with no problems.

For better or worse, I try to a im for the middle ground in teaching and testing. My focus usual ly i s on the “whys” of things probably more than the“whats ,” and I try to reduce the number of what our s tudents refer to as “rat facts” to a minimum. I don’t spend time teaching about a l l theangiotens in-converting enzyme inhibi tors when I can teach the essentia ls by focus ing on captopri l . I prune the teaching of β-blockers with a focuson propranolol as the prototype (most representative drug), and then spend some time ta lking about the so-ca l led cardioselective β-blockers(atenolol , metoprolol ), those that have vasodi lator activi ty (whether by vi rtue of α-blockade, l ike labeta lol , or ni tric oxide generation, as withbisoprolol ), and try to show how or why those drugs are di fferent (yet in key ways , such as adverse effects or contra indications) and importantcl inica l ly. No discuss ion or testing on the two dozen or so other β-blockers .

I a l so provide you, in one of the appendices , with a “name recognition guide”—for example, i f the drug’s generic name ends in “-pri l ” i t’s anangiotens in-converting enzyme inhibi tor; i f the generic name ends in “-olol” i t’s some sort of β-adrenergic blocker. Use i t! You’l l find in thisPreTest™ severa l questions about a drug you may not have learned about expl ici tly. But i f you “know your word s tems” you’l l be a l l set to answerthe question correctly. As one example: There i s a question or two about the most l ikely s ide effects of tamsulos in, a drug used for managingbenign prostatic hypertrophy. Haven’t learned about tamsulos in expl ici tly? No problem. Look at the generic name: tamsulosin. Same as prazosin, aselective α1-receptor blocker that I ’m sure you have learned about. Al l the selective α-blockers are l ike prazos in in most (but clearly a l l ) importantways .

There i s no universa l ly adopted or “officia l” medica l pharmacology course or course content (even though the National Board of Medica lExaminers , who prepare your Step exams, have relatively focused learning and testing objectives ). If you’re curious , look at the text reference pagenumbers in the explanations to answers in this book. For some drugs or related materia l you may see severa l pages devoted to the topic in onebook, and just one (or perhaps just one figure or table) in another. In one text the page references may come from early parts of the book, and forthe other i t wi l l be pretty obvious that the content i s put toward the end. It’s not that one text i s better than the other. It s imply reflects di fferentbiases and preferences by the authors or edi tors .

I have a certa in job to do, respons ibi l i ties to ful fi l l , as I deem appropriate: pick and choose materia l I think i s important; teach i t clearly and ina manner that excludes what I deem unnecessary (eg, drug s tructures ; as a phys ician you’l l probably never have to look at the chemica l s tructure ofa drug and figure out what i t i s , what i t i s used for, i ts s ide effects , and so on). I try to s impl i fy that which can be unnecessari ly compl icated. I try tobridge learning of bas ic facts and concepts and applying them cl inica l ly, and largely ignore content I think i s , for lack of a better phrase, too overthe top or deta i led to be of use for more than just something else to learn. My background i s largely in systems-based phys iology andpathophys iology (as opposed to, say, biochem and molecular biology), and so many of my questions and explanations focus on that. This i s what Ido in class and on exams, and i t i s reflected in my main approach to wri ting questions and expla ining the answers in this book.

Breadth and Depth of Questions and Answers: Low Yield, High Yield?

Most s tudents who reviewed previous edi tions of PreTest™ Pharmacology found the book to be extremely useful . However, some questions wereci ted by a few reviewers as being “low-yield,” “too bas ic,” “too cl inica l ,” and the l ike.

I’ve tried to address these cri tici sms in every edi tion of this book that I ’ve wri tten, but let me pol i tely opine that at this point in your medica leducation you’re not in the best pos i tion to make judgment ca l l s on such matters , and just because you were (or weren’t) taught or tested on a

Page 11: Pharmacology pre test self assessment and review (14th ed.)

particular point by your profs doesn’t mean that other s tudents have or haven’t had the same expectations . And what s tudents sometimes ci te as alow yield question i s actua l ly bas ic and “must know” information, even though the correct answer may be rather obvious to most s tudents . Thatdoesn’t mean the information i s unimportant, or that your abi l i ty to recognize i t shouldn’t be eva luated. It could wel l mean that you’ve learned theessentia l bas ics , and that’s good. In addition, some s tudents have ca l led certa in questions “low yield” s imply because they haven’t learned aboutthe facts or concepts addressed in the question. It’s easy to attribute l i ttle importance to things one doesn’t know or understand, and say thequestion i s trivia l .

For some of you i t’s tempting to view some of my questions as “too cl inica l”; others may find things “too bas ic or mechanis tic.” I have beenchided for asking some questions about in vi tro observations , when the responses deemed most important and appropriate seemingly are thosethat occur in humans . That’s not necessari ly true in my opinion.

Answering a l l the questions in this book i s relatively s imple i f you think about the bas ic information you should have acquired; i f you integratei t with what you should have learned in other courses (eg, in a phys iology or cel l and molecular biology course that may incorporate a lot ofknowledge based on in vi tro s tudies ); and i f you s tart doing what you wi l l have to do soon—make reasoned judgments based on applying yourknowledge to a poss ibly new cl inica l picture: which i s “most l ikely?”

Breadth and Depth of Questions and Answers: Empiric Data or Mechanistically Certain Information?

Some reviewers have noted that some questions focus on empiric information: findings , such as certa in s ide effects , for which we don’t have good(let a lone defini tive) information on mechanisms of action. The premise of those cri tici sms i s “Why ask questions about these i ssues , for which[I] can offer no proven mechanis tic explanations , when there i s so much other information to ask questions about, so much other informationabout which we need to know?” Some examples include the increased ri sk of as thma-related deaths from long-acting β-agonis ts ; the constipatingeffect of verapami l ; the facia l flushing associated with immediate-acting niacin; the skeleta l muscle damage caused by s tatins ; and many more.The reason why I include such questions i s that these and many other so-far mechanis tica l ly unexpla ined s ide effects , adverse responses , andrelated i ssues are cl inica l ly important and sometimes common. We may not be able to expla in a l l the “whys” in biochemica l terms (I mayspeculate about some publ i shed mechanisms), but when these untoward responses happen and cause adverse consequences , as they often do,i t’s important for you to know.

Suggestions on How to Use This Book

Prepare yoursel f to answer the questions in each chapter by fi rs t reviewing the corresponding materia l from your lecture notes and favori te (or, atleast, ass igned) text. This volume that you hold in your hands i s , a fter a l l , a review and sel f-assessment tool , not an origina l source of learninginformation.

Before you work on the questions and your s tudying overa l l , try to do the fol lowing:Be able to identify main drug classes, recognizing that sometimes we use more than one classification scheme (eg, chemical; by main mechanism(s) or site(s)

of action; by clinical use); and be able to cite a prototype drug for each. Conversely, given a named prototype or otherwise representative drug, be able to workbackward and know the rest of the most relevant information, including the class(es) to which it belongs.

For example, you should be able to identi fy a group of drugs that are ca l led dihydropyridines as a large and main chemica l class of ca lciumchannel blockers (CCBs) that are mainly vasodi lators and are used for such indications as hypertens ion or other conditions in which vasodi lationis des i red. You should know that ni fedipine can be cons idered a prototype of the rather large dihydropyridine CCB class .

You should be able to take the reverse approach by identi fying ni fedipine as the prototype dihydropyridine CCB; identi fying the main actions ofthe drug and i ts overa l l class ; and recognizing the main uses and adverse effects .

And, you should a lso know how other CCBs , not class i fied as dihydropyridines (eg, verapami l , di l tiazem), di ffer and work. You certa inly don’tneed to look at chemica l s tructures to deduce the di fferent activi ty profi les , but you certa inly do need to know how, for example, the actions ofni fedipine di ffer from those of, say, verapami l .

It’s a l so important to know about what might be described as “subprototypes”—for example, what’s specia l about atenolol or metoprolol , orlabeta lol , compared with the overa l l and prototypic β-adrenergic blocker, propranolol? In this book I’l l tel l you how you can never forget thatlabeta lol , a widely used drug, blocks both α- and β-adrenergic receptors . And I’l l give you lots of other memory-enhancing tips , too.

Be able to identify the class to which a drug belongs by looking at its generic name or other name.You should have some knowledge of a drug’s class by looking at i ts generic name—for example, a drug that ends in the suffix “-s tigmine” i s a

revers ible acetylchol inesterase inhibi tor; one that ends in “-pri l ” i s an angiotens in-converting enzyme inhibi tor; a “-sartan” i s an angiotens inreceptor blocker; and so on. Al though this “look at the generic name and you’l l know the drug group” technique doesn’t apply to a l l drugs , i t doesapply to probably hundreds . To that end, I ’ve included a table at the back of the book to help you do that. You a lso need to be able to give areasonable working defini tion for certa in drug class i fi cation terms, such as catecholamine, SSRI, or reverse transcriptase inhibi tor. You get thepoint, I hope. This learning trick doesn’t work a l l the time, of course, s ince for many drugs there i sn’t any rhyme or reason behind the names.

Be able to state the main expected effects or side effects of major drugs or drug classes. This should give you a good idea of what the relevant precautions orcontraindications are, even if you haven’t been taught about the latter, even if your learning focus hasn’t been too clinical.

For example, you know that a l l β-adrenergic blockers can reduce cardiac rate, contracti l i ty, and electrica l impulse conduction veloci ty (especia l lythrough the AV node), and sometimes these drugs are used speci fica l ly to cause one or more of those effects . You should then rea l i ze thatexcess ive doses may cause unwanted degrees of suppress ion of those cardiac parameters . And you should rea l i ze that the effects of these drugswarrant extra caution (or contra indicate a l together) the use of any β-blocker in patients who a l ready have bradycardia , s igni ficantly reducedventricular contracti l i ty/cardiac output, or some degree of heart block. Making these associations or extrapolations i s not rocket science that youmust have been taught about expl ici tly. You should be able to use your bas ic knowledge of pharmacology and drug action, and of phys iology andpathophys iology, to piece things together and get the correct (or most logica l or l ikely) answer.

Be able to state the most important (eg, common, serious, or life-threatening) unwanted side effects, adverse responses, and clinically relevant druginteractions for the main drugs or drug classes.

Such “must know” adverse responses to certa in drugs aren’t necessari ly common ones , but you need to know about them. For example,myopathy and potentia l rhabdomyolys is from statins i s relatively rare, but you certa inly need to know about these rather unique class -relatedadverse effects , and their cl inica l consequences . (“Rare” i s an interesting word. Statin-induced myopathy affects far fewer than 1% of patientstaking the drug, but when you cons ider the fact that mi l l ions of patients are taking this medication, the impl ication i s that you’l l probablyencounter the problem and need to be able to recognize and properly deal with i t.) The same appl ies to gingiva l hyperplas ia from phenytoin whenadminis tered to (mainly) chi ldren; paradoxica l thrombocytopenia from unfractionated heparin; coagulopathies (including paradoxica l thrombos is )from loading doses of warfarin, and embryopathy when even otherwise proper doses are adminis tered during the fi rs t 12 weeks of pregnancy; andso on. And, as one fina l example, you need to understand the potentia l ri sks of gl ibly saying “take acetaminophen” to your patients who aretaking warfarin.

Page 12: Pharmacology pre test self assessment and review (14th ed.)

Get a decent pharmacology book, read it, and learn what not to focus on when filling up your memory bank.The explanations for a l l the answers I provide are cross -referenced to two excel lent texts . Goodman and Gi lman’s The Pharmacological Basis of

Therapeutics (12th edi tion) i s without equal as an authori tative, current, and complete book. Dr Larry Brunton has done an outstanding job at theeditor’s helm for a whi le now, and the book has been the defini tive “bible of pharmacology” for decades . The other ci tations are for Basic andClinical Pharmacology, by Drs Katzung, Masters , and Trevor. To me “Katzung” s trikes the best ba lance between bas ic pharmacology and drug actionswith current therapeutic principles . And you don’t have to read an excess ive number of pages to get the sa l ient information. Theserecommendations are not merely paying homage to this books ’ publ i sher, McGraw-Hi l l . They are unbiased and profess ional ly honest.

Many drugs come and go. We used to teach about ri todrine, a predominately β2 agonis t, as a uterine relaxant drug to help s low prematurelabor. It’s gone, at least here in the United States . Cromolyn and nedocromi l were sometimes used (and your predecessors were tested about) ascontrol meds for asthma. Gone. It was easy to describe and ask questions about tubocurarine, the prototypic nondepolarizing skeleta l muscularblocker. Gone, and now we address such drugs as vecuronium, mivacurium, and others , and i t’s rea l ly imposs ible to pick one of those drugs as aprototype. The drugs class i fied as cyclooxygenase II (COX-2) inhibi tors have been winnowed down to one drug, celecoxib. The gl i tazones , arelatively smal l group of drugs for Type 2 diabetes , but important nonetheless , have only one drug remaining: piogl i tazone.

There are other i s sues I think I need to point out to you, and they relate to what you may or may have not learned about. After reviewing lots ofanonymous patient records from our insti tution, I ’ve noted the fol lowing, for example. Ephedrine, normal ly cons idered a “minor drug” and so notoften taught, i s a mainstay in our operating rooms when the goal i s to cause a l i ttle upward “bl ip” of a too low blood pressure. Ephedrine i sdescribed in most texts very briefly, and the comments usual ly describe the l imitations of the drug and relegate i t to “seldom used” s tatus . Not soin the rea l world.

In our operating rooms, many patients get para lyzed with a nondepolarizing neuromuscular blocker (usual ly one of the “curines ,” but not ourlong-s tanding prototype, tubocurarine, s ince i t’s gone), and of course their neuromuscular blockade usual ly needs to be reversed post-op. Youhave no doubt learned that reversa l of such muscle para lys is involves treatment with atropine, then a chol inesterase inhibi tor, usual lyneostigmine. The di ffi cul ty here i s that, at our insti tution and at many others , atropine i sn’t used. We use glycopyrolate instead of atropine nearlya l l the time. Have you been taught expl ici tly, or read expl ici tly, about glycopyrrolate? Bet you the answer i s no. But you’l l find here, in this edi tionof PreTest™ Pharmacology a l l you’l l need to know to guide you to the correct answers .

Using this PreTest Book

The majori ty of questions are wri tten to el ici t the one “best” or “most l ikely” correct response. Mark your answer by each question, or download,print, and use the “answer sheets” I ’ve posted on my personal teaching web s i te. (The URL i s www.umich.edu/~mshlafer/pharm.html . When youget there, cl i ck the PreTest™ button at the top of the page.) Try to a l low yoursel f a minute or so for each question, but don’t rush. There are nopenalties for going through this review and answering incorrectly, and no rewards for speed. Then your time wi l l be spent best going through theexplanations for the answers .

Many s tudents who have used previous edi tions of PreTest™ Pharmacology have sa id that even though they may have answered questionscorrectly, they’ve s ti l l learned or fina l ly understood something va luable and perhaps even indel ible by reading the explanations . After you finisha l l the questions in a chapter, spend as much time as you need veri fying your answers and careful ly reading the explanations provided. This i sparticularly important i f you chose a wrong answer, didn’t have a rea l clue about what the right answer might be, or just made a lucky guess ,perhaps because of your test-savvy ski l l s . So, try to read every explanation, especia l ly the more lengthy ones . I wrote most of the explanations toreinforce and supplement the information sought by the questions , and sometimes gently to encourage you to look at (usual ly earl ier) parts ofthis review book. Do revis i t your class notes and a decent pharmacology text (see above and the references to key texts l i s ted for each answerexplanation) for further clari fi cation too.

I urge you not to do one rather tempting thing: read, or even peek, at the answers to individual questions in a chapter before you’ve answeredal l the questions . I know this may be pa inful in a variety of ways (but, “no pa in, no ga in” they say). Nonetheless , explanations for the answer toone question may give you a tip-off (i f not the outright correct answer) to another question that might address the same drug or drug class , butfrom a di fferent perspective. In this sel f-s tudy and review process—for assess ing your knowledge, understanding, and synthes is—such tip-offsmay lul l you arriving at a correct answer by a short-ci rcui ted approach, having just read a s imi lar or related answer to another question. There i s“planned redundancy” in some of the questions and the answers I ’ve wri tten for you.

Some Acknowledged, Up-front Caveats

As I mentioned above, and for reasons I bel ieve are defens ible, I ’ve omitted drug s tructures , or deta i led chemica l reactions that describe thesynthes is or degradation of drugs . And, whi le many s tudents have pra ised that the various chapters helped them learn materia l for speci fic areasof pharmacology and therapeutics , and do wel l on their exams, I do things a l i ttle di fferently here.

Look at questions in most other “review” books or texts , or look at the questions on your own exams, and you’l l probably find that the scope ofquestions i s qui te l imited. Questions assess ing your knowledge about, for example, antihypertens ive drugs , have answer choices that are ratherl imited to knowledge about a particular antihypertens ive drug or drug class : “What i s the most l ikely s ide effect of this particular ca lcium channelblocker?” Some wi l l divers i fy assessment of your knowledge a bi t more: “Which antihypertens ive drug most l ikely caused this s tated adverseresponse?” Going further, a question’s answer choices may include drugs representing a host of diverse cardiovascular drugs used, for example,for angina, hypertens ion, hypercholesterolemia, and so on. Sti l l , that’s a relatively narrow focus .

Such questions don’t ful ly address cl inica l rea l i ty—your abi l i ty to integrate and apply knowledge across what might seem l ike independentareas of pharmacology, therapeutics , and medicine. For example, certa in “respiratory” drugs may have a negative impact on your patient’scardiovascular disease. An ostens ibly fundamenta l autonomic nervous system drug may have important impl ications to your patient with a certa incardiovascular or ophthalmic disease, respi ratory disease, or any of severa l diseases of the centra l nervous system. And so, many questions inthis book “encourage” you to integrate and apply your knowledge across severa l seemingly discrete areas of drug therapy. Doing so i s what mostof you wi l l have to do, at least early on in your careers . It’s s imply good, and necessary, hol i s tic integration of your knowledge.

And so now, as you turn the pages and s tart your sel f-assessment and review, do as one of my favori te comics (Larry the Cable Guy) wouldencourage you to do: “Git er done!”

Good luck!

Marshal ShlaferProfessor, Department of PharmacologyUnivers i ty of Michigan Medica l SchoolAnn Arbor, MI [email protected]

Page 13: Pharmacology pre test self assessment and review (14th ed.)

2013

Page 14: Pharmacology pre test self assessment and review (14th ed.)

Cross-References (for Answers) to Selected Pharmacology Texts

“Brunton”—Brunton L, Chabner B, Knol lman B, eds . Goodman & Gilman’s The Pharmacological Basis of Therapeutics, 12th ed. McGraw-Hi l l ,2011.

“Katzung”—Katzung B, Masters S, Trevor, A eds . Basic and Clinical Pharmacology, 12th ed. McGraw-Hi l l , 2012.

Explanations for the answers provided in this edi tion of PreTest™—Pharmacology are cross -referenced to one or both of the above pharmacologytexts .

Each text excels in certa in respects , yet they di ffer in terms of actua l content and how i t i s presented. Look at the text cross -references in each ofmy answers and you’l l see the di ffering focus . One text, or a particular chapter in that text, may be more mechanis tic or more deta i led; another maybe more cl inica l ; one may pa int a discuss ion about certa in drugs or drug groups , or a particular medica l condition, with broader brush s trokes thananother. One text may address a particular point on severa l pages , the other on one or two, or may have no speci fic coverage at a l l . This i s notsurpris ing, and i t para l lels the way you were probably taught pharmacology: no one s tandard precl inica l pharmacology curriculum for a l l medica lschools , nor any s tandard or cons is tency in how the content may be presented at one school , or by one prof, compared with another.

Page 15: Pharmacology pre test self assessment and review (14th ed.)

Generic Drug Name Recognition Guide I ’ve compi led this l i s t with the hope that i t may help you look at a drug’s generic name and figure out the group or class to which i t belongs . I amsure there are some omiss ions , but the l i s t i s nonetheless qui te complete. Most of the entries are arranged a lphabetica l ly by suffixes , but in acouple of instances the l i s ting i s for a prefix (eg, ceph- for cephalosporins ) or some sequence of letters cons is tently found in the drugs ’ names (eg,quin in quinidine, quinine, chloroquine). Drug names in bold are ones I cons ider to be a prototype or most important example(s ).

Where and how do generic drugs get thei r “officia l” names , at least in the United States? They are ass igned by the United States AdoptedNames Counci l (USANC). Their purpose i s to provide “s imple, informative, and unique nonproprietary names for drugs”—the USAN. They do this by“establ i shing logica l nomenclature class i fi cations” based on drugs ’ pharmacologic relationships with others (of the same or s imi lar type) orbased on chemica l s imi lari ties (eg, the core chemica l s tructure of drugs). In the United States the FDA has the fina l say on whether a drug name isacceptable and, therefore, “officia l .”

A s ide note: a l though many generic names are identica l worldwide, there are a lso some glaring incons is tencies , some of which you’re apt toencounter. For example, the Bri ti sh (and others ) ca l l acetaminophen paracetamol , and meperidine i s pethidine.

Where can you get more information about drug names? Point your browser to:http://www.ama-assn.org/ama/pub/about-ama/our-people/coal i tionsconsortiums/united-s tates -adopted-names-counci l .shtml

Page 16: Pharmacology pre test self assessment and review (14th ed.)
Page 17: Pharmacology pre test self assessment and review (14th ed.)
Page 18: Pharmacology pre test self assessment and review (14th ed.)
Page 19: Pharmacology pre test self assessment and review (14th ed.)
Page 20: Pharmacology pre test self assessment and review (14th ed.)
Page 21: Pharmacology pre test self assessment and review (14th ed.)
Page 22: Pharmacology pre test self assessment and review (14th ed.)
Page 23: Pharmacology pre test self assessment and review (14th ed.)
Page 24: Pharmacology pre test self assessment and review (14th ed.)
Page 25: Pharmacology pre test self assessment and review (14th ed.)
Page 26: Pharmacology pre test self assessment and review (14th ed.)
Page 27: Pharmacology pre test self assessment and review (14th ed.)
Page 28: Pharmacology pre test self assessment and review (14th ed.)
Page 29: Pharmacology pre test self assessment and review (14th ed.)
Page 30: Pharmacology pre test self assessment and review (14th ed.)

List of Abbreviations

Here are some common abbreviations you are l ikely to encounter in this edi tion of PreTest™ Pharmacology or elsewhere when abbreviations relatedto pharmacology and therapeutics come up. I ’ve tried to spel l out the ful l word(s ), and give the abbreviation parenthetica l ly, at the fi rs t occurrencein each question or answer in this book. Nonetheless , there are other common abbreviations that don’t appear in this book.

I’ve omitted symbols for chemica l elements or thei r cationic or anionic forms (eg, Ca 2+, Cl –), chemica l formulae (eg, NaCl ), abbreviations ofcommon biochemica ls (ATP, ADP, DNA, etc), Greek letters , most uni ts of measure (volume, weight, time), and abbreviations for most common labtests (but there are some exceptions).

I want to note that by l i s ting abbreviations here I’m not condoning or encouraging their use when, for example, entering information into apatient’s chart or medication order or record. Indeed, use of many of these abbreviations in patient-related documents should be avoided, inlarge part to help avoid errors in interpreting them or thei r meanings or intent.

[X]i—intracel lular concentration, where X i s an anion or a cation

[X]O—extracel lular (“outs ide”) concentration, where X i s an anion or a cation

1°—firs t degree (eg, 1° heart block)2°—second degree (eg, 2° heart block)3°—third degree (eg, 3° heart block, a lso ca l led complete heart block)5-FU—5-fluorouraci l5-HT—5-hydroxytryptamine; serotonin6-MP—6-mercaptopurine (active metabol i te of azathioprine)

AA-II—angiotens in I IAA—arachidonic acid; amino acidAAPMC—antibiotic-associated pseudomembranous col i ti sAb—antibodya.c.—before meal (s ) or, s imply, before eatingAC—adenylyl cyclaseACE—angiotens in-converting enzyme (a lso known as bradykininase, kininase II)ACh—acetylchol ineAChE—acetylchol inesteraseAChEI—acetylchol inesterase inhibi torACS—acute coronary syndromeACTH—adrenocorticotropic hormone; corticotropinADD/ADHD—attention-defici t (/hyperactivi ty) disorderADH—antidiuretic hormone (vasopress in [VP]); a ldehyde dehydrogenasead l ib—(take) freely as des i red, as much as needed, as much as you’d l ikeADR—adverse drug reaction(s )AED—antiepi leptic (anticonvulsant) drugAF (or AFIB)—atria l fibri l lationAFL—atria l flutterAg—antigenAIDS—acquired immunodeficiency syndromeALL—acute lymphocytic leukemiaALS—amyotrophic latera l scleros is (Lou Gehrig disease)ALT—alanine aminotransferaseAMI—acute myocardia l infarctionAML—acute myelogenous leukemiaANA—antinuclear antibodiesANS—autonomic nervous systemAP—action potentia lAPAP—N-acetyl para-aminophenol (acetaminophen)APD—action potentia l durationAPTT—activated partia l thromboplastin time (eg, a measurement of anticoagulation, eg, as caused by unfractionated heparin)ARB—angiotens in receptor blockerARC—AIDS-related complexASA—acetylsa l icyl i c acid (aspi rin); Anesthes iology Society of AmericaAST—aspartate aminotransferaseAUC—area under the (blood) concentration versus time “curve” (of a drug)AV(N)—atrioventricular (node)A-V—arteriovenous (as in A-V shunt; or di fferences in concentrations of a substance in arteria l vs . venous blood, as used to ca lculate clearance of

a drug)AZT—azidothymidine (zidovudine)

B

Page 31: Pharmacology pre test self assessment and review (14th ed.)

BAL—Bri ti sh anti -Lewis i te (dimercaprol ; chelator, antidote for arsenic poisoning)BBB—bundle branch blockbid—twice da i ly; spel l ing out twice da i ly i s preferredBMI—body mass index (body weight/body surface area in m2)BMR—basal metabol ic rateBP—blood pressure (cardiac output × tota l periphera l res is tance)BPH—benign prostatic hypertrophyBPM—beats per minuteBUN—blood urea ni trogenBZD—benzodiazepine

CCav—average (mean) plasma concentration of a drug

Cmax—maximum plasma concentration

Cmin—minimum plasma concentration

Css—steady-state plasma concentration

CABG—coronary artery bypass graft(ing)CAD—coronary artery disease; coronary heart disease (CHD)CAI—carbonic anhydrase inhibi torCBC—complete blood countCCB—calcium channel blockerCDC—(US) Center for Disease Control and PreventionCF—cystic fibros isCGL—chronic granulocytic leukemiaCHD—coronary heart disease; coronary artery disease (CAD)CHF—congestive heart fa i lureCHI—closed head injuryCI—cardiac index (cardiac output normal ized to body surface area)CK—creatine kinaseCl—clearance (of drug)CML—chronic myelogenous leukemiaCMV—cytomegalovi rusCNS—centra l nervous systemCO—cardiac output (heart rate × s troke volume)COLD—chronic obstructive lung diseaseCOMT—catechol -O-methyl transferase (catecholamine-degrading enzyme)COPD—chronic obstructive pulmonary disease (eg, emphysema, chronic bronchi ti s )COX—cyclooxygenase(s ); COX-1 and/or COX-2CPZ—chlorpromazine; a lso used as abbreviation of a brand name product of prochlorperazine; avoid use of this abbreviation to avoid improper

substi tution of one drug for anotherCRE—carbapenem-res is tant enterobacteriaceaeCRF—corticotropin-releas ing factorCRP—c-reactive proteinCSF—cerebrospina l fluidCss—steady-state drug concentration

CTZ—chemoreceptor trigger zone (vomiting center) in the bra instemCVA—cerebrovascular accident (s troke)CVP—centra l venous pressureCYP—cytochrome P450 (system or member of i t)

DD1 (D2)—dopamine D1 (or D2) receptor

DA—dopamineDBH—dopamine β-hydroxylase (enzyme involved in catecholamine synthes is )DBP—diastol ic blood pressureDC—discontinue (a lso D/C)DDI—drug–drug interactionDFX—deferoxamine (chelator used mainly for i ron poisoning)DHT—dihydrotestosteroneDIG (or dig)—digoxinDisp—dispenseDKA—diabetic ketoacidos isDM—diabetes mel l i tusDMARD—disease-modi fying anti rheumatic drug (eg, methotrexate, many others , including corticosteroids ); sometimes referred to as SAARD (s low-

acting anti rheumatic drug)

Page 32: Pharmacology pre test self assessment and review (14th ed.)

DOPA—dihydroxyphenyla lanineDPH—diphenylhydantoin (phenytoin)DPI—dry powder inhaler adminis tration system for ora l inhalation of certa in (mainly pulmonary) drugs ; a lso see MDI, SPIdP/dt—rate of pressure change (eg, left ventricular pressure versus time; δP/δt)DPP-4—dipeptidyl peptidase-4, the target of ora l antidiabetic drugs known as the gl iptins (eg, saxagl iptin, s i tagl iptin)DU—duodenal ulcerDVT—deep venous thrombos isDx—diagnos is

EECG—electrocardiogram; EKGED—effective dose; emergency departmentED50—median effective dose

EDRF—endothel ium-derived relaxing factor (ni tric oxide)EEG—electroencephalogramEKG—electrocardiogram; ECGEPI—epinephrineERP—effective refractory period (eg, of a nerve); see a lso RP, RRPEtOH—ethanol (a lso ETOH)

FFAB—antibody fragmentFDA—(US) Food and Drug Adminis trationFdUMP—5-fluoro-2′-dexoyuridine-5′-monophosphate, active anticancer (anti )metabol i te of fluorouraci lFEV—forced expiratory volume; FEV1, forced expiratory volume in 1 secondFFA—free fatty acidsFH2—7,8-dihydrofol ic acid

FH4—5,6,7,8-tetrahydrofol ic acid

FSH—fol l i cle-s timulating hormoneFx—function

GG6PD—glucose 6-phosphate dehydrogenaseGABA—γ-aminobutyric acidGAD—genera l i zed anxiety disorderG-CSF—granulocyte colony-s timulating factorGERD—gastroesophageal reflux diseaseGFR—glomerular fi l tration rateGH—growth hormoneGI—gastrointestina lGLP—glucagon-l ike peptide (eg, GLP-1), an incretin-l ike agentGCSF—granulocyte colony-s timulating factorGM-CSF—granulocyte macrophage colony-s timulating factorG protein—guanine nucleotide-binding proteinGp IIb/II Ia—platelet membrane glycoprotein receptor respons ible for l inking (via fibrinogen) activated plateletsGSH, GSSG—glutathione (reduced, oxidized)GTT—glucose tolerance test (involving ei ther ora l [OGTT] or parentera l glucose adminis tration); drop(s ) (as in dose or rate of drug infus ion)gX—membrane conductance of an ion, where X i s potass ium, ca lcium, etc (eg, gK)GU—genitourinary

HH1—histamine H1 receptor

H2—histamine H2 receptor

H2RA—histamine H2 receptor antagonis t

HAART—highly active anti retrovi ra l therapyHb—hemoglobin (a lso HGB)HbA1c—glycosylated (or glycated) hemoglobin used to monitor glycemic control long-term

hCG—human chorionic gonadotropinHct—hematocri tHCTZ—hydrochlorothiazideHDL—high-dens i ty l ipoprotein(s ) (cholesterol )HEENT—head, eyes , ears , nose, and throat (eg, as parts of a phys ica l exam)HF—heart fa i lure (or CHF, heart fa i lure with s igns and symptoms of venous congestion)HGB—hemoglobin (a lso Hb)HHNKS—hyperosmolar hyperglycemic nonketotic syndrome; a lso abbreviated HHNSHIT—heparin-induced thrombocytopenia

Page 33: Pharmacology pre test self assessment and review (14th ed.)

HIV—human immunodeficiency vi rusH+,K+,ATPase—proton (hydrogen)–potass ium ATPase (eg, the gastric parieta l cel l “proton pump”)HLBI—Heart, Lung and Blood Insti tute (of the National Insti tutes of Heal th)HMG—CoA-β-hydroxy-β-methylglutaryl -coenzyme AHPA—hypothalamic–pi tui tary axish.s .—at bedtimeHSV—herpes s implex vi rusHTN—hypertens ionHx—history

IIBD—inflammatory bowel diseaseICP—intracrania l pressureIDDM—insul in-dependent diabetes mel l i tus ; usual ly means Type 1 diabetes mel l i tus , however many patients with Type 2 diabetes need (are

“dependent” on) insul in, so IDDM is not a sui table term nor one that automatica l ly impl ies Type 1IgE, G (etc)—immunoglobul in E, G, etcIGF—insul in-l ike growth factorIHSS—idiopathic hypertrophic subaortic s tenos isIL (-1, -2, etc)—interleukin(s )IM—intramuscular(ly)INH—isoniazidINR—international normal ized ratio (for normal izing resul ts of prothrombin time tests during warfarin therapy)I&O—intake and output (eg, of foods , fluids )INR—international normal ized ratio, derived from and used to s tandardize resul ts of prothrombin time measurements (eg, when monitoring

warfarin therapy)IOM—(The) Insti tute of Medicine, a part of the National Academy of Sciences , s tudies and advises on many aspects of medicine and medica l care;

publ i shed a landmark s tudy “To Err Is Human,” which showed us the magnitude, scope, and consequences of medica l errors—many of whichinvolve drugs .

IOP—intraocular pressureIP3—inos i tol tri sphosphate

ISA—intrins ic sympathomimetic activi ty; subclass of β-adrenergic blockers that a lso have weak agonis t activi ty, eg, pindololISMP—Insti tute for Safe Medica l Practice (heal th care/safety and monitoring organization)IT—intratheca l (adminis tration route)ITP—idiopathic thrombocytopenic purpuraIV—intravenous(ly)IVP—IV push (rapid, bolus injection of a drug); intravenous pyelogram

JJCAHO—Joint Commiss ion on Accredi tation of Heal thcare Organizations (now known s imply as the “Joint Commiss ion”)JNC—Joint National Committee (of the National Insti tutes of Heal th), as in JNC-7—more properly known as the Joint National Committee on

Prevention, Detection, Eva luation, and Treatment of High Blood Pressure

Kke—el imination rate constant

LLA—left atrium; loca l anestheticL-DOPA—levodopaLD—lethal doseLD50—median letha l dose

LDL—low-dens i ty l ipoprotein(s )LE—lupus erythematos is (see a lso SLE)LH—luteinizing hormoneLHRH—luteinizing hormone-releas ing hormone (hypothalamic)LMWH—low molecular weight heparin (enoxaparin)L&R—norepinephrine + phentolamine (occas ional ly used as IV a l ternative to dobutamine for short-term cardiac inotropic support, enables

norepinephrine to s timulate heart without caus ing vasoconstriction)LR—lactated Ringer solutionLSD—lysergic acid diethylamideLT—leukotrieneLV—left ventricle/ventricularLVEDP—left ventricular end-diastol ic pressureLVH—left ventricular hypertrophy

MMAC—minimum a lveolar concentration (eg, of inhalation anesthetic/analges ic) expressed percent of agent in tota l gas mixture needed to abol i sh

response to certa in pa inful s timul i in 50% of patients

Page 34: Pharmacology pre test self assessment and review (14th ed.)

MAP—mean arteria l pressureMAR—medication adminis tration recordMBC—minimum bactericida l concentrationMAO—monoamine oxidaseMAO-A, -B—monoamine oxidase type A, type BMAOI—monoamine oxidase inhibi tormcg—microgram; noted here because i t i s preferred abbreviation, instead of μg (which might be mis interpreted as mg), when wri ting

prescriptions , making notes in the medication adminis tration or patient chart, etcMDI—metered-dose inhaler (del ivery system for some drugs , usual ly pulmonary)MEC—minimum effective concentrationMH—mal ignant hyperthermiaMHC—major his tocompatibi l i ty complexMI—myocardia l infarctionMIC—minimum inhibi tory concentration (usual ly appl ied to antimicrobia ls )mmol—mil l imoleMMR—meas les , mumps, rubel laMOPP—chemotherapy combination used to treat Hodgkin disease, derived from generic or trade names of four drugs : mechlorethamine, Oncovin®

(vincris tine), prednisone, and procarbazinemOsmol—mil l iosmoleMP—[6-]mercaptopurineMRSA—methici l l in-res is tant S. aureusMS—morphine sul fate; multiple scleros isMTX—methotrexate

NNa-K ATPase—the sodium pump (eg, on nerve, muscle cel l s , and some other exci table cel l s )NADH—nicotinamide adenine dinucleotideNADPH—nicotinamide adenine dinucleotide phosphateNa +,K+,ATPase—sodium–potass ium–adenos ine triphosphatase (“sodium pump”)NAPA—N-acetylproca inamide, metabol i te of proca inamide; type of cabbage that i s a main ingredient in kim cheeNCI—National Cancer Insti tuteNE—norepinephrineNET—norepinephrine transporterNIDDM—noninsul in-dependent diabetes mel l i tus ; more properly ca l led Type 2 diabetes mel l i tus , a l though some patients with Type 2 diabetes

may require (depend on) supplementa l insul in (see IDDM above)NM—nicotinic-skeleta l muscle receptors (ie, those found postsynaptica l ly at the skeleta l -somatic neuromuscular junction)

NN—nicotinic-neura l receptors (those found postsynaptica l ly in autonomic gangl ia and on cel l s of the adrenal [suprarenal ] medul la )

NMDA —N- methyl -D-aspartate (glutamate channel receptor, agonis t, antagonis t, etc)NMS—neuroleptic mal ignant syndromeNNRTI—nonnucleos ide reverse transcriptase inhibi torNO—nitric oxideNOS—nitric oxidase synthaseNPH—neutra l protamine Hagedorn (type of intermediate-acting insul in); i sophane insul in suspens ionNPO—nothing by mouth (ni l per os )NRTI—nucleotide reverse transcriptase inhibi torNS—normal sa l ine (ie, 0.9% NaCl in s teri le water)NSAID—nonsteroida l anti -inflammatory drug. See a lso t-NSAIDNSTMI—non-ST (elevation) myocardia l infarctionNTG—nitroglycerin

OO2 SAT—oxygen saturation (eg, of blood sample)

OA—osteoarthri ti sOC—oral contraceptiveOCD—obsess ive–compuls ive disorderOD—overdose; right eyeOS—left eyeOTC—over-the-counter (nonprescription), eg, OTC drug

PP450—cytochrome P450 mixed-function oxidase systemPA—pulmonary artery; phys ician-ass is tantPAO2—arteria l partia l pressure of oxygen

PABA—p-aminobenzoic acid (ingredient in some sunscreens , common preservative in some multiuse parentera l medication conta iners )PAC—premature atria l contractionPAD—periphera l arteria l disease

Page 35: Pharmacology pre test self assessment and review (14th ed.)

PAF—platelet-aggregating factor(s )PAM (2-PAM)—pral idoxime (chol inesterase reactivator)PAS—para-aminosa l icyl ic acidPAT—paroxysmal atria l tachycardia (see a lso PSVT)PBP—penici l l in-binding protein(s )p.c.—after meal (s )PCA—percutaneous angioplasty; a lso used to denote patient-control led analges ia , in which patients requiring pa in control , usual ly with morphine

or another opioid, sel f-adminis ter thei r medicationPCO2—partia l pressure of CO2

PCN—penici l l inPCOS—polycystic ovarian syndromePCP—Pneumocystis pneumonia; phencycl idine (“angel dust”)PCWP—pulmonary capi l lary wedge pressure (eg, as measured with Swan-Ganz catheter)PDGF—platelet-derived growth factorPFT—pulmonary function test(s )PG—prostaglandinPGE1—prostaglandin E1 (a lprostadi l )

PGE2—prostaglandin E2 (dinoprostone)

PGI2—prostaglandin I2 (prostacycl in)

P-gp—P-glycoprotein(s ); permeabi l i ty glycoprotein(s )PI—protease inhibi torPICC—periphera l ly inserted centra l (venous) catheter (eg, used for drug del ivery, often referred to as a “PICC l ine”)PID—pelvic inflammatory diseasePKC—protein kinase cPKU—phenylkenonuriaPLC—phosphol ipase CPMS—premenstrua l syndromePNS—parasympathetic nervous system (not to be confused with periphera l nervous system, for which the abbreviation i s sometimes used)PO—(adminis tration route) by mouth (per os )PO2—partia l pressure (tens ion) of oxygen

PPAR—peroxisome prol i ferator-activated receptor(s ), eg, PPAR-γ, the cel lular target (in nuclei ) of insul in-sens i ti zing effects of thiazol idinediones(“gl i tazones ,” ie, piogl i tazone)

PPI—proton-pump inhibi tor (gastric acid-antisecretory drug, esomeprazole, etc)PR—(adminis ter) via the rectum (per rectum)PRN—as needed (pro re nata)PRP—pressure-rate product (ie, peak LV systol ic pressure multipl ied by heart rate); platelet-rich plasmaPSVT—paroxysmal supraventricular tachycardiaPT—prothrombin timePUD—peptic ulcer diseasePt—patientPT—prothrombin time (a lso see INR); phys ica l therapyPTH—parathyroid hormonePTSD—posttraumatic s tress disorderPUD—peptic ulcer diseasePVB or PVC—premature ventricular beat, premature ventricular contraction

Qq4h—every 4 hours (or other number inserted)qd—each day; best to spel l out “da i ly” than use abbreviationqid—4 times da i ly, spel l ing out 4 times a day i s preferredQTc—corrected QT interva l (on electrocardiogram)

RRA—rheumatoid arthri ti s ; right atriumRBC—red blood cel l ; erythrocytesRBF—renal blood flowRDA—recommended da i ly a l lowance (eg, of a nutrient)rDNA—recombinant DNA, for example, molecular biologic technology used to synthes ize or modi fy protein drugs such as many insul insRP—refractory period, for example, of a nerve (a lso see ERP, RRP)RR—respiratory rateRRP—relative refractory periodRx—prescription; abbreviation for Latin word meaning take or take thou

SSAARD—slow-acting anti rheumatic drug; a lso see DMARDSA(N)—sinoatria l (node)

Page 36: Pharmacology pre test self assessment and review (14th ed.)

SBP—systol ic blood pressureSC—see subQSERM—selective estrogen receptor modi fierSERT—serotonin transporterSH—sul fhydrylSIADH—syndrome of inappropriate ADH (arginine vasopress in) secretion, mani fest as hypervolemia, hyponatremia, and reduced blood osmola l i tys ig—write on prescription labelSK—streptokinaseSL—subl ingual (adminis tration route)SLE—systemic lupus erythematos isSMBG—sel f-monitoring (by the patient) of blood glucoseSPI—sol id powder inhaler (eg, for certa in respiratory drugs)SR—sarcoplasmic reticulum (in myocytes )SRS-A—slow-reacting substance of anaphylaxisSSRI—selective serotonin reuptake inhibi torSTAT—immediatelySTD—sexual ly transmitted diseaseSTE—ST elevation (as in STE myocardia l infarction)SubQ—subcutaneous (eg, injection type), preferred to the abbreviation SCSuCh—succinylchol ineSVR—systemic vascular res is tance; tota l periphera l res is tanceSVT—supraventricular tachycardia , or susta ined ventricular tachycardiaSx—symptom(s)

TT1/2—half-l i fe (eg, biologic, el imination, plasma ha l f-l i fe) of a drug

T3—tri iodothyronine

T4—thyroxine, levothyroxine, tetra iodothyronine

TB—tuberculos isTCA—tricycl ic antidepressantTDP—torsades de pointes (from French, “twis ting of the points”)TG—triglyceride(s )TI—therapeutic index (LD50/ED50)

TIA—trans ient i schemic attacktid—3 times da i ly, spel l ing out 3 times a day i s preferredTKO—to keep open (as an IV infus ion at a rate just sufficient to keep venous access open)TNF—tumor necros is factort-NSAID—tradi tional nonsteroida l anti -inflammatory drug, speci fica l ly COX-1/COX-2 inhibi tors (aspi rin, ibuprofen, many others ), as opposed to

disease-modi fying anti rheumatic drugs (DMARDs) that are not COX inhibi tors or corticosteroids . See a lso COX, DMARD, and NSAIDtPA—tissue plasminogen activatorTPR—tota l periphera l res is tance (systemic vascular res is tance)TRH—thyrotropin-releas ing hormoneTSH—thyroid s timulating hormoneTTP—thrombotic thrombocytopenic purpura (and ITTP: idiopathic TTP)TXA2—thromboxane A2

TZD—thiazol idinedione (class of ora l antidiabetic drugs , ie, piogl i tazone)

UUSAN—United States Adopted Name (officia l naming system for generic drugs)UFH—unfractionated heparin (see a lso LMWH)UTI—urinary tract infection

VVd—volume of dis tribution (eg, of a drug)

VIP—vasoactive intestina l peptideVLDL—very-low-dens i ty l ipoproteinVP—vasopress in (antidiuretic hormone [ADH])V/Q—venti lation/perfus ion (as used in pulmonary function tests )VRE—vancomycin-res is tant enterococcusVT (VTACH)—ventricular tachycardia

WWBC—white blood cel lWPW—Wolff–Parkinson–White (cardiac electrophys iologic anomaly)

X

Page 37: Pharmacology pre test self assessment and review (14th ed.)

XD—xanthine dehydrogenaseXO—xanthine oxidase

ZZES—Zol l inger–El l i son syndrome

Page 38: Pharmacology pre test self assessment and review (14th ed.)

General Principles of Pharmacology Dosage regimens and their consequencesDose-response relationshipsDrug-receptor interactions , s igna l transductionEl imination of drugsFactors affecting drug dosagePharmacodynamics (mechanisms of drug action)Pharmacokinetics (time-, dose-, and concentration-related phenomena)Regulation of drug approval and use

Questions

1. You want to estimate, fol lowing drug adminis tration, some measure that rel iably reflects the tota l amount of drug reaching target ti s sue(s ) overtime. The drug i s being given ora l ly. What would you assess to get the des i red information?

a. Area under the blood concentration-time curve (AUC)b. Peak (maximum) blood concentrationc. Product of the apparent volume of dis tribution (Vd) and the fi rs t-order rate constant

d. Time-to-peak blood concentratione. Vd

2. The FDA ass igns the letters A, B, C, D, and X to drugs i t approves for human use. To what does this class i fi cation refer or apply?

a. Amount of dosage reduction needed as plasma creatinine clearances fa l lb. Amount of dosage reduction needed in the presence of l iver dys functionc. Feta l ri sk when given to pregnant womend. Relative margins of safety (or therapeutic index)e. The number of unlabeled uses for a drug

3. You ora l ly adminis ter a weak acid drug (A) with a pKa of 3.4. Gut pH i s 1.4 and blood pH i s 7.4. Assume the drug crosses membranes by s implepass ive di ffus ion (eg, no transporters are involved). Which observation would be true?

a. Only the ionized form of the drug, A, wi l l be absorbed from the gutb. The concentration ratio of tota l drug (A + HA–) would be 10,000:1 (gut:plasma)c. The drug wi l l be hydrolyzed by reaction with HCl , and so cannot be absorbedd. The drug wi l l not be absorbed unless we ra ise gastric pH to equal pKa, as might be done with an antacid

e. The drug would be absorbed, and at equi l ibrium the plasma concentration of the nonionized moiety (HA) would be 104 times higher than theplasma concentration of A–.

4. Experiments show that 95% of an ora l 80-mg dose of verapami l i s absorbed in a 70-kg test subject. However, because of extens ivebiotransformation during i ts fi rs t pass through the hepatic porta l ci rculation, the bioava i labi l i ty was only 0.25 (25%). Assuming a l iver blood flowof 1500 mL/min, what i s the hepatic clearance of verapami l in this s i tuation?

a. 60 mL/minb. 375 mL/minc. 740 mL/mind. 1110 mL/mine. 1425 mL/min

5. Such substances as inos i tol tri sphosphate (IP3), diacetyl glycerol (DAG), cycl ic AMP (cAMP), cAMP-dependent protein kinases , and changes in suchfunctions as membrane ion conductance (eg, of gK+ and gCa 2+) are important with respect to the effects of l igands on cel l responses . Which bestsummarizes where they are found or what they do?

a. Are present in nerve cel l s , but not the various types of muscles or glandsb. Are the receptors for various agonis ts and antagonis t drugsc. Interact with neurotransmitters or hormones (endogenous l igands), but not to chemica ls that are not found natura l ly in the bodyd. Mediate exci tatory, but not inhibi tory, responses in various target s tructurese. Transduce a chemica l “s ignal” from a l igand into the fina l cel l response(s )

6. Azi thromycin, an antibiotic, has an apparent volume of dis tribution (Vd) of approximately 30 L/kg. What i s the main interpretation of thisinformation?

a. Effective only when given intravenous lyb. El iminated mainly by renal excretion, without prior metabol i smc. Extens ively dis tributed to s i tes outs ide the vascular and inters ti tia l spacesd. Not extens ively bound to plasma proteinse. Unable to cross the blood-bra in or placenta l “barriers”

Page 39: Pharmacology pre test self assessment and review (14th ed.)

7. A cardiovascular pharmacologis t i s assess ing the inotropic (contracti le) responses of cardiac muscle to a variety of drugs . She uses a smal lanimal i solated papi l lary muscle preparation to ga in her data . The experimenta l setup i s shown here:

The papi l lary muscle has been excised from the animal ’s heart and put into a “phys iologic” solution that wi l l keep the muscle a l ive and ful lyfunctional for severa l hours . The muscle i s electrica l ly s timulated at a constant rate so that there are no usual rate-dependent effects oncontracti le force development, and a l l neura l influences on papi l lary muscle function or drug responses are absent.

A reference s tandard pos i tive inotropic drug i s adminis tered, in varying concentrations , to determine the maximum increase of contracti le force(the inotropic response; maximum is set at 100% as caused by the s tandard drug). Thereafter, three other drugs—X, Y, and Z—are tested and theirabi l i ty to increase contracti le force development (compared with the s tandard or reference drug) i s measured. The dose-response curves to X, Y,and Z are shown here:

Page 40: Pharmacology pre test self assessment and review (14th ed.)

Which s tatement describes the findings of this experimenta l s tudy involving drugs X, Y, and Z?

a. Drug X i s the most efficacious because i ts ED50 i s lowest

b. Drug Y i s the least potent drug among the three drugs shownc. Drug Z i s the most potent cardiac inotroped. Drug Y i s more potent than drug Z, and more efficacious than drug Xe. Drug X i s more potent than drug Y, and more efficacious than drug Z

8. A patient needs a drug that has an apparent volume of dis tribution of 20 L. The plan i s to adminis ter a loading dose to reach a target plasmalevel of 5 mcg/mL. This plasma concentration i s the target s teady-s tate concentration (CSS), and i t wi l l be mainta ined by subsequent ora lmaintenance doses . What IV loading dose would be needed to yield that 5 mcg/mL target?

a. 1 mgb. 5 mgc. 10 mgd. 20 mge. 100 mg

9. We are repeatedly adminis tering a drug ora l ly. Every dose i s 50 mg; the interva l between doses i s 8 h, which i s identica l to the drug’s plasma(overa l l el imination) ha l f-l i fe. The bioava i labi l i ty i s 0.5. For as long as the drug i s adminis tered no interacting drugs are added or s topped, andthere are no appl icable factors affecting such things as absorption or el imination that might change the drug’s pharmacokinetics .

Which formula gives the best estimate of how long i t wi l l take for the drug to reach s teady-s tate plasma concentrations (CSS)?

Abbreviations:Cl = clearance (mL/min)D = dose (mg)F = bioava i labi l i tyke = el imination rate constant

Page 41: Pharmacology pre test self assessment and review (14th ed.)

t1/2 = ha l f-l i fe (h)

Vd = volume of dis tribution

a. (0.693 × Vd)/Cl

b. 1/ke

c. 4.5 × t1/2

d. (t1/2) × (ke)

e. D/(F × t1/2)

10. A classmate i s doing summer research, in your school ’s cl inica l pharmacology divis ion, between her fi rs t and second years of medica l school .Her topic involves new investigations into the roles of P-glycoprotein. Which s tatement accurately summarizes the biologic role of P-glycoproteins ,particularly as i t a ffects drugs and their actions?

a. Conjugates a variety of drugs , or thei r metabol i tes , to faci l i tate thei r ul timate renal el imination from the bodyb. Mainta ins s tructura l integri ty of cel l membranes and the surface receptors found on themc. Phosphorylates certa in substances , such as in the convers ion of adenos ine monophosphate (AMP) into the diphosphate and triphosphate forms

(ADP, ATP)d. Transports certa in drugs and xenobiotics across cel l membranese. Works , in conjunction with various G-proteins , to transduce (convert) interactions between drugs and their receptors into biologic responses

11. You are conducting pharmacokinetic s tudies on a new drug that we hope wi l l be approved for cl inica l use. We insert a venous catheter tosample blood at various times after drug adminis tration, and a lso take a sample for the immediate pre-drug blood concentrations of the drug(which should be zero). After assaying the blood samples for the drug we make a graph that plots plasma drug concentrations over time,continuing unti l tests reveal undetectable blood levels . What can you ca lculate or estimate based on the ratio of the area under the curve (AUC)obta ined by ora l adminis tration versus the AUC for intravenous adminis tration of the same drug?

a. Absorptionb. Bioava i labi l i tyc. Clearanced. El imination rate constante. Extraction ratiof. Volume of dis tribution

12. Identica l doses of a drug are given ora l ly and intravenous ly. We sample blood at various times , measure blood concentrations of the drug, andplot the data (shown below).

Page 42: Pharmacology pre test self assessment and review (14th ed.)

Further analys is of only these data wi l l a l low you to determine which of the fol lowing?

a. El imination route(s )b. Extent of plasma protein bindingc. Ora l bioava i labi l i tyd. Potencye. Therapeutic effectiveness

13. During your career you may have patients in age ranges from very young to very old, and adjustments in drug dosages or interva ls betweenrepeated doses may be required based on age. Assume you cons ider only the effects of one drug; no interacting drugs or comorbidi ties areinvolved. What s tatement best describes the general relationship between chronologic age and the overa l l el imination plasma ha l f-l ives of drugs?

a. Depends on chemica l class of drug (eg, benzodiazepine, catecholamine, and dihydropyridine)b. Depends on renal function (eg, creatinine clearance), not age per sec. Linear: ha l f-l i fe lengthens in di rect proportion to aged. No genera l ly appl icable relationship because el imination ha l f-l i fe depends on the drug, not i ts classe. Peaks at around age 18 to 20 years , with ha l f-l ives being much longer at younger or older ages

14. The el imination of a drug and i ts numerous metabol i tes i s described as being heavi ly dependent on Phase II metabol ic reactions . Which ofthe fol lowing i s a Phase II reaction?

Page 43: Pharmacology pre test self assessment and review (14th ed.)

a. Glucuronidationb. Decarboxylationc. Es ter hydrolys isd. Ni tro reductione. Sul foxide formation

15. The hospi ta l pharmacy sends up a solution of a drug that i s to be infused intravenous ly at a constant, speci fied rate. It has been di luted to theproper concentration in a sui table IV adminis tration fluid. Assuming that no patient- or drug-related variables change during the adminis trationperiod, which one of the fol lowing wi l l be the main determinant of how long i t wi l l take for blood concentrations of this drug to reach s teady s tate(plateau; no change of mean blood levels over time)?

a. Bioava i labi l i ty of the drugb. Concentration of the drug in the solution that wi l l be infusedc. Hal f-l i fe of the drugd. Presence or absence of cardiovascular or renal diseasee. Plasma creatinine concentration (or creatinine clearance)f. Tota l dose per 24 hoursg. Volume of drug adminis tered per minute

16. Yee et a l . (Effect of grapefrui t juice on blood cyclosporine concentration. Lancet 1995;345:955–956) examined severa l pharmacokinetic variablesrelated to ora l cyclosporine adminis tration with water, grapefrui t juice, and orange juice:

The numbers l i s ted are ari thmetic means ± one s tandard deviation of the mean. Cmax i s the peak blood concentration, and Tmax i s the time afteradminis tration at which peak plasma concentrations of the drug are reached. The p va lues are based on analys is of variance (ANOVA) corrected forrepeated measures .

These data , and what you should have learned from your bas ic pharmacology s tudies , are most cons is tent with an hypothes is that grapefrui tjuice does which of the fol lowing?

a. Acidi fies the urine, favoring cyclosporine’s tubular reabsorption via a pH-dependent effectb. Activates an intestina l wal l transporter for cyclosporinec. Al ters the route(s ) of el imination for cyclosporined. Inhibi ts metabol i sm of cyclosporinee. Reduces binding of cyclosporine to plasma proteins , thereby ra is ing free (active) drug levels in the ci rculation

17. A 60-year-old man with rheumatoid arthri ti s wi l l be s tarted on a nonsteroida l anti -inflammatory drug (NSAID) to suppress the jointinflammation. Publ i shed pharmacokinetic data for this drug include:

Bioavai labi l i ty (F): 1.0 (100%)Plasma ha l f-l i fe (t1/2) = 0.5 hour

Apparent volume of dis tribution (Vd): 45 L

For this drug i t i s important to mainta in an average s teady-s tate concentration of 2.0 mcg/mL in order to ensure adequate and continued anti -inflammatory activi ty.

The drug wi l l be given (taken) every 4 hours .What dose wi l l be needed to obta in this 2 mcg/mL average s teady-s tate drug concentration?

a. 5 mg

Page 44: Pharmacology pre test self assessment and review (14th ed.)

b. 100 mgc. 325 mgd. 500 mge. 625 mg

18. We take a blood sample from a patient (basel ine measurement) and then adminis ter drug A intravenous ly. We take additional blood samplesperiodica l ly thereafter and measure drug concentration in each sample. We repeat the experiment, this time giving the same drug ora l ly. Then weplot the logari thm of drug concentration versus time with data from both adminis tration routes to find comparable el imination “curves” indicativeof fi rs t-order el imination. What do the s lopes of the resul ting concentration versus time curves indicate about the pharmacokinetics of drug A?

a. Area under the curve (AUC)b. Bioava i labi l i tyc. El imination rate constantd. Extraction ratioe. Volume of dis tribution

19. The officia l “package insert” for prescription drugs identi fies (among many other things ) the speci fic use(s ) or indication(s ) for the drugapproved by the FDA. It i s a l so l ikely to note one or severa l more so-ca l led “off-label” uses : uses that have not been sanctioned officia l ly by theFDA, but for which there i s reasonable evidence that the drug i s both safe and effective. For which other uses can you, as a l i censed phys ician,adminis ter or prescribe these FDA-approved drugs?

a. Anything you wish, provided no speci fic laws (eg, control led-substance laws) prohibi t i tb. None—only FDA approved and wri tten off-label usesc. Only approved and off-label uses for older drugs in the same chemica l class (eg, another thiazide diuretic, benzodiazepine, etc)d. Only approved and off-label uses of older drugs used for the same purpose (eg, another antihypertens ive, antidepressant, or cancer

chemotherapeutic agent, etc)e. Only uses for which there i s some evidence of drug efficacy and safety in the experimenta l l i terature

20. When new drugs undergo precl inica l testing, one of many things to know is whether i t’s largely confined to the vascular compartment ordis tributed more widely, perhaps to speci fic ti s sues such as the l ipid-rich CNS. This i s , of course, ul timately cl inica l ly important i f the drugeventual ly gets FDA approval . One way to get a handle on that i s to ca lculate the apparent volume of dis tribution (Vd).

The table and graph below show some data concerning a new amino-glycos ide antibiotic. We give an IV dose (5 mg/kg) of the drug to a 70-kg 21-year-old volunteer, who i s hea l thy and taking no other drugs . After a l lowing time for redis tribution and equi l ibration of the drug in various bodycompartments , we measure plasma concentrations at various times .

Page 45: Pharmacology pre test self assessment and review (14th ed.)

Which va lue comes closest to the apparent Vd for this drug? (By a l l means , feel free to use your favori te ca lculator.)

a . 0.62 Lb. 19 Lc. 50 Ld. 110 Le. 350 L

21. You are reviewing the data from severa l meta-analyses that addressed the most common causes of adverse or otherwise excess ive effects ofprescription drugs in young adults and in the elderly (>60 years of age). Interactions between multiple drugs were not cons idered. Which variablewould you find to be decreased, and be the most common genera l cause of these problems, in the elders?

a. Body fat contentb. Lean body mass

Page 46: Pharmacology pre test self assessment and review (14th ed.)

c. Liver functiond. Renal function/clearancee. Plasma a lbumin levels

22. Some texts s tate that the abi l i ty of one drug (sometimes ca l led the “object drug”) to displace molecules of another drug (the “target drug”) fromplasma protein binding s i tes , thereby ra is ing the concentration and activi ty of free target drug in the blood, i s not cl inica l ly relevant. The cla imedreason? “Drug molecules that are displaced are rapidly el iminated and the equi l ibrium between bound and free drug i s rapidly reestabl i shed.”This interpretation may be correct for some drugs , but not for others .

Cons idering only the pharmacokinetics or other relevant properties of the target drug, upon which drug-related variable does the speed ofel imination and reestabl i shment of the bound versus free equi l ibrium—indeed, whether the interaction i s apt to be cl inica l ly relevant—dependthe most?

a. Bioava i labi l i tyb. Overa l l el imination t1/2

c. pK (acid or base)d. Renal clearancee. Whether the drug i s metabol i zed, or excreted without prior metabol i sm

23. Upon eva luating the effects of certa in sympathomimetic drugs in a variety of in vi tro and in vivo models , you find that the responses exhibi t thephenomenon of tachyphylaxis. What does the term tachyphylaxis mean?

a. An increase in the rate of the response, for example, an increase of the rate of muscle contractionb. Immediate hypersens i tivi ty reactions (ie, anaphylaxis )c. Prompt conformational changes of the receptor such that agonis ts , but not antagonis ts , are able to bind and cause a responsed. Quick and progress ive ri ses in the intens i ty of drug response, with repeated adminis tration, even when the doses are unchangede. Rapid development of tolerance to the drug’s effects

24. A postoperative patient wi l l require prolonged analges ia . We choose a drug that has the fol lowing pharmacokinetic properties :

Hal f-l i fe: 12 hoursClearance: 0.08 L/minVolume of dis tribution: 60 L

The patient has an indwel l ing venous catheter with a s low drip of 0.9% NaCl , and we wi l l use this to adminis ter intermittent injections of thedrug every 4 hours . The target blood level of the drug, fol lowing each injection, i s 8 mcg/mL.

With this plan in mind, and us ing no loading dose of the drug, which one of the fol lowing comes closest to the dose that should beadminis tered every 4 hours?

a. 0.960 mg (or 1 mg)b. 6.4 mg (or 6 mg)c. 25.6 mg (or 25 mg)d. 150 mge. 550 mg

25. A patient i s experiencing severe postoperative pa in, and we need to give a loading dose of an analges ic drug for prompt rel ief of discomfort.The drug we choose has the same pharmacokinetic properties as the one described in Question 24:

Hal f-l i fe: 12 hoursClearance: 0.08 L/minVolume of dis tribution: 60 L

Our target plasma concentration for the drug i s 8 mcg/mL. What number comes closest to the correct loading dose?

a. 0.48 mg (rounded to 0.5 mg)b. 150 mgc. 320 mgd. 480 mge. 640 mg

26. We adminis ter a highly l ipid-soluble drug and monitor i ts el imination in vivo and in vi tro. Al l the data indicate that i t i s transformed to avariety of more polar and oxidized metabol i tes by a group of heme proteins that activate molecular oxygen to a form that i s capable of interactingwith organic substrates such as our test drug. What enzyme or enzyme system is most l ikely involved in the ini tia l metabol i sm of this drug?

a. Cyclooxygenaseb. Cytochrome P450s (CYP system, mixed-function oxidases)c. Monoamine oxidase (MAO)d. Nicotinamide adenine dinucleotide phosphate (NADPH)e. UDP-glucuronosyl transferase

27. We measure the heart rate of a heal thy subject under the conditions noted below, a l lowing ample time for return to basel ine conditions andful l el imination of drugs between each s tep:

1. At rest

Page 47: Pharmacology pre test self assessment and review (14th ed.)

2. During treadmi l l exercise sufficient to activate the sympathetic nervous system at a time when maximum heart rate i s reached3. After adminis tration of acebutolol , a drug with affini ty for β-adrenergic receptors4. After giving acebutolol , fol lowed by exercise at the same level used in condition 2

Acebutolol given at rest causes a s l ight but cons is tent increase in heart rate. Give a bigger dose at rest and heart rate ri ses a bi t more.When the patient exercises after receiving a low-dose acebutolol , heart rate ri ses s igni ficantly less than i t did in the absence of acebutolol .

With exercise after the higher dose of acebutolol , the tachycardia i s blunted even more.The figure below summarizes the main findings .

Which s tatement best summarizes the actions of acebutolol?

a . Has higher affini ty for adrenergic receptors than the endogenous agonis ts , epinephrine, and norepinephrineb. Is a partia l agonis t for β-adrenergic receptorsc. Is activating spare receptors on myocardia l cel l sd. Is an i rrevers ible or noncompeti tive β-blockere. Is changing conformation of the adrenergic receptors

28. We are planning to infuse a drug intravenous ly at a constant amount per uni t time (rate). It has a fi rs t-order el imination rate constant (kel) of0.35/h. No loading dose wi l l be given. Approximately how long wi l l i t take for blood levels to reach s teady s tate after the infus ion begins?

a. 0.7 hourb. 1.2 hoursc. 3.5 hoursd. 9 hours

Page 48: Pharmacology pre test self assessment and review (14th ed.)

e. 24 hours

29. A patient who i s supposed to be taking a drug once a day gets confused and for a couple of days takes excess ive da i ly doses , leading totoxici ty. The drug has a mean plasma ha l f-l i fe of 40 hours .

Right now the patient’s plasma concentration of the drug i s 6 mcg/mL. Al though what to do next wi l l depend on actua l blood tests for druglevels , the usual plan in this case i s to have the patient skip one or severa l da i ly doses of the drug unti l blood levels fi rs t enter the therapeuticand nontoxic range, which in this case i s 0.8 mcg/mL. Assume the drug i s el iminated by fi rs t-order kinetics . How many da i ly doses should bewithheld?

a. 1b. 2c. 3d. 4e. 5

30. We want to ca lculate the apparent volume of dis tribution (Vd) for a hypothetica l drug (drug A) that has a ha l f-l i fe of 4 hours . Al l (100%) of anabsorbed dose of this drug undergoes Phase I oxidation, fol lowed by conjugation (Phase II reaction) and then renal excretion.

We rapidly inject a known dose, and 30 minutes later begin taking seria l blood samples (30 min apart) and quanti fying drug concentration ineach sample. What one other piece of information must be measured or otherwise determined to ca lculate Vd in the eas iest way?

a. Area under the drug concentration-time curve (AUC)b. Bioava i labi l i ty (F)c. Clearance (Cl )d. El imination rate constant (kel)

e. Maximum blood concentration after the bolus injection (C0)

31. A patient with a bacteria l infection requires intravenous antibiotic therapy. The chosen drug has a clearance (Cl ) of 70 mL/min. The apparentvolume of dis tribution (Vd) i s 50 L. The plan i s to adminis ter the drug intravenous ly every 6 hours and achieve a 4 mg/L s teady-s tate blood level ofthe drug. No loading dose s trategy i s to be used. What maintenance dose i s needed to achieve this?

a. 14 mgb. 24 mgc. 100 mgd. 300 mge. 1200 mg

32. A pharmacologica l ly inert but eas i ly measured substance, X, i s el iminated in a manner that fol lows l inear kinetics (fi rs t-order plot of log drugconcentration vs time during el imination i s a s tra ight l ine). The plasma ha l f-l i fe i s 30 minutes . Bolus IV doses wel l in excess of 100 mg must begiven in order to saturate the enzymes respons ible for metabol i zing the drug, which wi l l then lead to zero-order el imination kinetics .

We infuse a solution of X intravenous ly. The concentration of the solution i s 2 mg/mL; the infus ion rate i s 1 mL/min and i s kept constant at that.We continue the infus ion for 24 hours .

After a l lowing ample time for the drug to be el iminated completely, we repeat the adminis tration. This time the concentration of the solutionof X i s 4 mg/mL, and we infuse i t at a rate of 2 mL/min.

Which other variable wi l l a l so be changed as a resul t of the s tated changes to the infus ion protocol?

a. El imination rate constantb. Hal f-l i fec. Plasma concentration when CSS i s reached

d. Time to reach s teady-s tate concentration (CSS)

e. Tota l body clearancef. Volume of dis tribution

33. A new drug, drug A, undergoes a series of Phase I metabol ic reactions before i ts metabol i tes ul timately are el iminated. Which s tatement bestdescribes the characteris tics of drug A, or the role of Phase I reactions in i ts metabol i sm or actions?

a. Complete metabol i sm of drug A by Phase I reactions wi l l yield products that are less l ikely to undergo renal tubular reabsorptionb. Drug A i s a very polar substancec. Drug A wi l l be biologica l ly inactive unti l i t i s metabol i zedd. Phase I metabol i sm of drug A involves conjugation, as with glucuronic acid or sul fatee. Phase I metabol i sm of drug A wi l l increase i ts intracel lular access and actions

34. Dopamine, epinephrine (or norepinephrine), and his tamine are important neurotransmitter agonis ts . When these l igands interact with thei rcel lular receptors , how do they mainly el ici t thei r responses?

a. Activating adenylyl cyclase, leading to increased intracel lular cAMP levelsb. Activating phosphol ipase Cc. Inducing or inhibi ting synthes is of l igand-speci fic intracel lular proteinsd. Opening or clos ing l igand-gated ion channelse. Regulating intracel lular second messengers through G protein-coupled receptors

Page 49: Pharmacology pre test self assessment and review (14th ed.)

35. You have just eva luated and s tarted treatment on a 40-year-old woman in whom you have diagnosed malignant hypertens ion, based on herhis tory, her cl inica l presentation, and the blood pressure changes you measured over the relatively short time you’ve been at the beds ide. Younow go to the fami ly, in the waiting room, and expla in your diagnos is . You expla in that hypertens ion means high blood pressure. But when theyheard the word mal ignant one of the fami ly members says “Oh, her blood pressure i s high because she has cancer?” How should you best expla inthe term to the fami ly?

a. Blood pressure i s ri s ing very quickly and dangerous lyb. Cancer i s present, but i t i s not the cause of the high blood pressurec. Her high blood pressure i s , indeed, caused by a cancer (“mal ignancy”—in this case probably an adrenal cortica l tumor—a pheochromocytoma)d. Her high blood pressure did not fa l l in response to a blood pressure medication that i s effective for most patientse. The hypertens ion i s l ikely to prove fata l (eg, from a ruptured aneurysm in the bra in or elsewhere)

36. The Food and Drug Adminis tration has broad regulatory authori ty over prescription drugs , over-the-counter (OTC) drugs , and nutri tionalsupplements (herbals and other so-ca l led nutriceutica ls ). This authori ty includes approval , marketing (adverti s ing), and withdrawal of drugs fromthe market. Which s tatement summarizes an element of FDA rules or guidel ines?

a. Drugs approved for sa le OTC fi rs t received FDA approval for sa le and marketing “by prescription only”b. If a pharmaceutica l manufacturer provides data sufficient to obta in FDA approval for sa le by prescription, the manufacturer i s then a l lowed to

sel l the drug over-the-counter (OTC)c. If the FDA approves a prescription drug for sa le (prescribing), the phys ician can prescribe the drug only for the FDA-approved indication (use)d. Nutri tional supplements can be marketed without providing proof of efficacy or safety to the FDAe. Phase II I testing of prescription drugs that have been approved by the FDA gives complete information about adverse responses and pertinent

drug-drug interactions

Page 50: Pharmacology pre test self assessment and review (14th ed.)

General Principles of Pharmacology

Answers

1. The answer is a. (Brunton, pp 32, 36-39; Katzung, pp 43-44). We obta in the AUC ora l ly, by measuring drug concentrations measured in sequentia lblood samples . However, we must compare those data to measurements of AUC obta ined with giving the same drug intravenous ly (wherebioavai labi l i ty i s , by defini tion, 1.0). The fraction of a drug dose absorbed after ora l adminis tration i s a ffected by a variety of factors that canstrongly influence the peak blood levels and the time-to-peak blood concentration. The Vd and the tota l body clearance a lso are important indetermining the amount of drug that reaches a target ti s sue. Only the area under the blood concentration-time curve, however, reflects absorption,dis tribution, metabol i sm, and excretion factors , and their interactions ; i t i s the most rel iable method of eva luating bioava i labi l i ty overa l l .

2. The answer is c. (Brunton, p 1846; Katzung, pp 1039-1043.) These are FDA “pregnancy categories .” The short table presented below summarizes whateach of the five FDA pregnancy classes means . The actua l adverse feta l effect(s ), and the trimester(s ) of pregnancy at which their ri sks aregreatest, depend on the drug class or, in many instances , on the actua l drug. The adverse effects may include teratogenes is , feta l death, or suchother responses as endocrine imbalances (eg, neonata l hypothyroidism—cretinism—if mother received certa in anti thyroid drugs). You can look upinformation for speci fic drugs , and the trimester(s ) during which the feta l ri sk i s greatest or appl icable (for example, the main ri sks ofembryopathy from warfarin, a widely used ora l anticoagulant, apply to the fi rs t 12 or so weeks of gestation), as the need arises (eg, during your ob-gyn and perhaps other clerkships ).

Page 51: Pharmacology pre test self assessment and review (14th ed.)
Page 52: Pharmacology pre test self assessment and review (14th ed.)

Notes : Some drugs , and/or thei r active metabol i te(s ), are excreted in maternal mi lk and may adversely affect the nurs ing chi ld. If the motherrequires one or more of those drugs , she should be instructed not to breast-feed or a sui table a l ternative drug that i s not excreted in maternalmi lk (i f one i s ava i lable) might be prescribed instead. Providing a l i s t of such drugs i s beyond the scope of this review book; many profess ionaltexts and on-l ine resources are ava i lable for you to get more information. Drugs that pose or have feta l ri sks (from maternal use) are notnecessari ly ones that should be avoided by breast-feeding women; the converse a lso appl ies .

3. The answer is e. (Brunton, pp 17-19; Katzung, pp 10-12.) Reca l l the two Henderson-Hasselbach equations , which apply to how loca l pH affects theionization of molecules (eg, of a drug) in an aqueous envi ronment. Assume that membranes are permeable only to nonionized (and more l ipid-soluble) forms of a drug. Thus , we are making the assumption that ionized drugs tend to s tay, or concentrate, in an envi ronment that favors thatpH-dependent ionization; conversely, in an envi ronment that favors formation of nonionized drug molecules , a concentration gradient wi l l favorpass ive di ffus ion of nonionized molecules to another loca le.

Our drug was an acid with pKa = 3.4. In the s tomach (assume pH = 1.4 as noted) the ratio of nonionized to ionized molecules wi l l be about 1:0.01.The nonionized molecules wi l l di ffuse across the membrane. Once in the plasma, pH 7.4, the ratio of HA:A– wi l l become 1:10,000. And theconcentration ratio of tota l drug across the membrane wi l l be 10,000:1, but with the larger amount being in the plasma, not the gut.

Page 53: Pharmacology pre test self assessment and review (14th ed.)

You might a lso want to look at the figure, below, to get a “big picture” of how changing pH changes the ionization of acidic and bas ic drugs .

Page 54: Pharmacology pre test self assessment and review (14th ed.)

4. The answer is d. (Brunton, pp 28-30; Katzung, p 43.) Bioava i labi l i ty i s defined as the fraction or percentage of a drug that becomes ava i lable to thesystemic ci rculation fol lowing adminis tration by any route, compared with bioava i labi l i ty when the drug i s given IV (s ince IV bioava i labi l i ty i sdefined as 1.0, or 10%). This takes into cons ideration that not a l l of an ora l ly adminis tered drug i s absorbed, and that a drug can be removed fromthe plasma and metabol i zed by the l iver during i ts ini tia l passage through the porta l ci rculation (ie, a “fi rs t-pass” effect). An ora l bioava i labi l i ty of0.25 (25%) indicates that only 20 mg of the 80-mg dose (ie, 80 mg × 0.25 = 20 mg) reached the systemic ci rculation. Organ clearance can bedetermined by knowing the blood flow through the organ (Q, expressed as a volume per uni t time, eg, mL/min) and the extraction ratio (ER) for thedrug by the organ, according to the equation:

The extraction ratio i s a function of the amounts of drug entering the organ (arteria l s ide; CA) and leaving i t on the venous s ide (CV), and theratio i s ca lculated as :

In this problem, the amount of verapami l entering the l iver per uni t time was 76 mg (80 mg × 0.95) and the amount leaving was 20 mg. Therefore,

Page 55: Pharmacology pre test self assessment and review (14th ed.)

5. The answer is e. (Brunton, pp 51-52; Katzung, pp 21-30.) The substances and functions l i s ted above provide s ignal transduction between l igandinteractions with speci fic receptors , and responses that are characteris tic of the agonis t and the target cel l (s ). Sui table l igands includeneurotransmitters (ACh, catecholamines , various amino acids [γ-aminobutyric acid, glycine, glutamate, aspartate] opioids , serotonin, andhis tamine to name a few) and hormones (insul in, glucagon, thyroid hormones , etc)—or drugs with sui table s tructures . They are not, however, theactual receptors (b) for agonis ts or antagonis ts . These substances wi l l a l so participate in l igand binding-response coupl ing to sui table foreignsubstances—drugs (c). Aga in, depending on the l igand and the type of cel l , responses can be ei ther exci tatory or inhibi tory (d), and they occur invarious neura l , muscle, and gland cel l s (a ).

6. The answer is c. (Brunton, pp 28, 30-32; Katzung, p 38.) For a 70-kg individual , tota l body water i s about 40 L (0.6 L/kg); inters ti tia l plus plasma wateroccupies about 12 L (0.17 L/kg).

Azi thromycin, with a Vd of 30 L/kg, would be dis tributed in an apparent volume of about 2100 L in a typica l 70-kg person.

Use s imple logic to answer this question, but look at the answer to Question 20 i f you wish. Even i f you don’t remember what tota l body water i s(about 40 L or 0.6 L/kg), or the approximate va lue for inters ti tia l plus plasma water (about 12 L or 0.17 L/kg), do the quick math. If you take thestated 30 L/kg and compute the tota l (and very hypothetica l ) apparent volume for a 70-kg individual , you would arrive at 2100 L. That number notonly reflects dis tribution into a hypothetica l volume far in excess of vascular and inters ti tia l volumes , but i s a l so far beyond what could bephys ica l ly rea l . After a l l , 2100 L of water equal 2100 kg; you won’t find human beings weighing that much!

Without more information, you cannot make defini tive conclus ions about the other properties l i s ted as answer choices .

7. The answer is d. (Brunton, pp 44-45, 73-74; Katzung, pp 30-32.) “Our product i s the most potent pa in rel iever (or whatever) you can buy without aprescription,” the TV ads say far too often for me. What those cla ims don’t say i s how efficacious a particular drug i s , particularly compared withanother (eg, a competing product). So, let’s address some fundamenta ls .

The most bas ic defini tion of efficacy i s s imply the abi l i ty to cause an effect. More important i s the question “how big an effect?” Aspirin, forexample, wi l l a l leviate headache for many patients , but i t s imply cannot, at safe doses , a l leviate severe pa in from, for example, an abdominalincis ion. So, in the setting of severe pa in, aspi rin i s less efficacious than, say, morphine.

If rel ief of s imple headache i s the goal , then such drugs as aspi rin, acetaminophen, and ibuprofen, are equal ly efficacious . What di ffers i s howmany mi l l igrams of each are necessary to do that. Ibuprofen i s more potent than acetaminophen, which i s s l ightly more potent than aspirin: 400mg of ibuprofen, 600 mg of acetaminophen, and 650 mg of aspi rin are reasonable numbers .

In the question provided here, drugs X, Y, and Z a l l have efficacy: they a l l increase contracti le force development by the i solated cardiac musclepreparation shown and described in the question. However, i t should be obvious that drugs Y and Z are more efficacious than drug X. The ED50 ofdrug X i s lower than that of the other drugs (a), but that doesn’t mean that i t i s more efficacious : indeed, i ts maximal effect on the test muscle’sforce of contraction i s about less than ha l f of that for drugs Y and Z.

Answer b i s incorrect: Drug Y i s more potent than drug Z: i ts ED50 i s lower than that of drug Z, but Y and Z are equal ly efficacious s ince their peakeffects are identica l . This a lso el iminates answer c as a correct answer: aga in, the ED50 for drug Z i s higher than that of drug Y, and so Z i s lesspotent. Answer e cannot be correct: a l though the ED50 of drug X i s lower than the ED50s of Y and Z, the maximum intens i ty of drug X’s response i smuch lower than those of Y and Z.

8. The answer is e. (Brunton, pp 28-37, 1894-1898; Katzung, pp 38, 41-49.) Since the volume of dis tribution i s expressed in the volume uni t of l i ters , andthe target concentration i s per mL, convert one of the volume uni ts to be the same as the other. For example, the target concentration of 5 mcg/mLequals 5 mg/L. We can find the answer by us ing fol lowing s imple equations :

9. The answer is c. (Brunton, pp 17-20, 27-30, 33-34, 1802-1804; Katzung, pp 41-47.) This question, with i ts many variables and equations , was wri ttenintentional ly to see whether you would take a needless ly compl icated approach to a concept that i s , when reduced to i ts essentia l point,relatively s tra ightforward. If you give doses of a drug repeatedly at interva ls that are equal to or less than the drug’s overa l l el imination ha l f-l i fe,and keep every other pertinent variable constant (dose, route, el imination s tatus , etc), you s imply multiply the ha l f-l i fe by 4 or 5 (hence, my use of4.5 to take a middle ground) to arrive at the approximate time unti l CSS: the time at which “drug in = drug out” i s reached.

You can see in the figure below that with repeated adminis tration of a drug at interva ls equal to the drug’s ha l f-l i fe. Note that the averageplasma concentration (Cav) does not appear to “flatten-out,” or reach a plateau, unti l at least four doses (= 4 ha l f-l ives ) have been given. After that,and assuming nothing else changes (dose, dose interva l , route of adminis tration, el imination kinetics , s tarting, or s topping other drugs), a l thoughthere wi l l s ti l l be fluctuations in peak and trough drug levels around the mean (right after each dose, right before the next), the average bloodconcentration wi l l not change.

Page 56: Pharmacology pre test self assessment and review (14th ed.)

Note: The equation (0.693 × Vd)/Cl (answer a) i s the equation for ca lculating the ha l f-l i fe.

10. The answer is d. (Brunton, pp 97, 148; Katzung, pp 9, 10t.) P-glycoprotein (PGP; the P s tands for “permeabi l i ty”) i s part of a super-fami ly oftransmembrane transporter proteins , substrates for which include many xenobiotics (chemica ls not found natura l ly in the body), including manytherapeutic agents . An important member of the P-glycoprotein fami ly i s involved in an ATP-dependent (ie, active transport; this i s part of the ABC—ATP-binding cassette—fami ly) mechanism that pumps drugs and other chemica l ly diverse xenobiotics out of certa in cel l s , aga inst the usualconcentration gradient. That i s , PGP serves an ATP-driven efflux pump. PGP does not form drug conjugates (a) or participate in other Phase IIreactions ; mainta in membrane/receptor s tructure and function (b); phosphorylate any substrates (c); or play a role in s igna l transduction (e).

Here are some of the s i tes where PGP i s abundant and important, and some important effects at those s i tes :

• in hepatocytes (where i t transports substrates into the bi le);• in the intestina l epi thel ium (important for transporting a variety of drugs across membranes during absorption);• in the kidneys (certa in proximal tubule cel l s , where substrates are transported into the urine for eventual el imination); and• in capi l lary endothel ia l cel l s of the “blood-bra in barrier,” where certa in drugs that may have di ffused across the barrier, into the CNS, are

transported back out to the systemic ci rculation. This reduces exposure of the bra in to the drugs .

What are some cl inica l ly relevant phenomena related to PGP activi ty? For s tarters , PGP activi ty i s important in the development of multidrugres is tance (MRP—multi -drug res is tance-associated protein), particularly cel lular res is tance to certa in chemotherapeutic agents andantivi ra l/anti retrovi ra l drugs . PGP promptly transports those cel lular toxins out of the cel l s the drugs are meant to ki l l .

There i s broader importance. Cons ider, for example, PGP activi ty in the intestina l epi thel ia . Many drugs that are substrates for PGP di ffuse fromthe intestina l lumen into the epi thel ia l cel l s during absorption, but PGP pumps a portion of those drug molecules right back into the lumen, from

Page 57: Pharmacology pre test self assessment and review (14th ed.)

which they may not be reabsorbed to ga in access to the ci rculation. Thus , PGP has an important role in absorption and bioava i labi l i ty of manyora l ly adminis tered drugs .

Just as there are substrates , inducers , and inhibi tors for the cytochrome P450 system, so there are substrates , inducers , and inhibi tors for PGP. IfPGP activi ty i s increased by an inducer, the ora l bioava i labi l i ty of PGP substrates wi l l be reduced (by vi rtue of greater transport back into thelumen), plasma levels in response to a dose (or doses) wi l l be reduced (absolute decreases of plasma levels and of the “area under the [time-concentration] curve), and the intens i ty of drug responses wi l l be reduced. Conversely, PGP inhibi tors can increase ora l bioava i labi l i ty, plasmalevels (AUC), and response intens i ty of PGP substrates . Thus , there are both pharmacokinetic and pharmacodynamic consequences of PGP activi ty.

In some cases the cl inica l s igni ficance of an interaction involving PGP i s weak or otherwise inconsequentia l ; however, with many other drugs ,inducers or inhibi tors may cause cl inica l ly s igni ficant di fferences in bioava i labi l i ty, drug blood levels , and the magnitude of drug responses .

The short table that fol lows l i s ts some of the drugs , mentioned in this book, that are PGP substrates , inducers , or inhibi tors .

Page 58: Pharmacology pre test self assessment and review (14th ed.)
Page 59: Pharmacology pre test self assessment and review (14th ed.)

11. The answer is b. (Brunton, pp 20-24; Katzung, pp 43-44.) Among other things , knowing the AUC of a drug given intravenous ly (which, by defini tion, i sassociated with a bioava i labi l i ty of 1.0, or 100%) i s a prerequis i te for determining the bioava i labi l i ty of the same drug given by any other route;bioavai labi l i ty i s ca lculated as the ratio of AUC for any non-IV route and the AUCIV.

12. The answer is c. (Brunton, pp 20-24; Katzung, pp 39-44.) We are making these data comparison to determine the drug’s bioava i labi l i ty. We definethe bioava i labi l i ty of a drug given intravenous ly as 1.0 (100%), s ince with IV adminis tration we avoid a l l the appl icable barriers to drug absorption.But, of course, i t’s important to know how much tota l drug, over a period of time, gets into the bloodstream with other adminis tration routes thatwe might want to use cl inica l ly. Drugs given by routes other than IV must be absorbed (and be exposed to a l l the barriers that l imit or s low orotherwise affect absorption); and because they might not be absorbed from their adminis tration s i te, or might be susceptible to such processesas hepatic fi rs t-pass metabol i sm, they usual ly have a bioava i labi l i ty <1.0.

The ca lculation of bioava i labi l i ty i s based on the ratio of the area under the concentration-time curve (AUC) for the adminis tration route beingcons idered (ora l , IM, etc) and the AUC obta ined with IV adminis tration.

Measurement of blood levels of a drug at a s ingle time point wi l l not give us the information we need to determine bioava i labi l i ty.Note that with ora l absorption there i s a delay unti l there i s some detectable drug in the blood; that reflects both the time needed for

absorption of the drug and, in most cases , the sens i tivi ty of the assay to measure the drug (which may be present, but at undetectable levels ).With IV adminis tration blood levels ri se instantaneous ly. With ora l adminis tration you should a lso note that as blood levels of the drug ri setoward the peak the rates of drug entry into the blood exceed rates of el imination (whether by metabol i sm, excretion, or both, depending on whatthe drug i s ) because blood levels are ri s ing. Once blood concentrations s tart to fa l l , the amount of drug entering the system becomes less than theamount being el iminated, per uni t time. That i s , “amount in = amount out.”

Fina l ly—and a l though you can’t tel l precisely from the graph—the ha l f-l ives for the drug, measured under di fferent experimenta l conditions , areidentica l : in genera l (and i t depends on the drug and i ts blood level ), the drug wi l l be el iminated at the same rate (based on usual kineticinfluences , such as fi rs t-order kinetics ) regardless of adminis tration route.

None of the other choices in the question (ie, potency, effectiveness , or plasma protein binding) can be eva luated us ing this type ofcomparison.

13. The answer is d. (Brunton, pp 29-38; Katzung, pp 66, 1047, 1049.) How (or even whether) age or extremes of age affect the overa l l el imination ha l f-l i fe of a drug (and thus the right dose and dos ing interva l ) depends on what the drug i s . Whi le renal function, such as that reflected by thecreatinine clearance (b), wi l l change over certa in age ranges , renal function per se i s not universa l ly important to the el imination or el iminationrates of a l l drugs . Remember that many drugs are completely metabol i zed to inactive substances , and so changes of renal function are largelyunimportant in thei r el imination (and plasma ha l f-l ives ). Usual ly, too, there are expected age-related changes in such variables as body water orfat content. They a lso affect pharmacokinetics , but there i s no overarching relationship that appl ies to a l l drugs .

Cons ider some examples .

• Theophyl l ine’s ha l f-l i fe i s shortest between the ages of 6 months to about 9 years , being much longer at younger or older ages . That i s becauseof di fferences in the rate of the drug’s metabol i sm. Thus , the dose of this old bronchodi lator, normal ized to body weight, i s much higher for thatage range; indeed, i t i s l i kely to be excess ive for and toxic to younger or older individuals . This rather odd relationship certa inly does not applyto a l l (or even many) drugs , and so answer e i s not correct for most drugs .

• When comparing the ha l f-l i fe of penici l l in G in adults >60 years old to adults in the 20- to 30-year range, the ha l f-l i fe i s twice as long in the olderpatients ; i t’s nearly twice as long with l idoca ine, too; 50% longer with digoxin; and vi rtua l ly not di fferent, age-wise with warfarin.

Other drugs for which there are s igni ficant di fferences (here, in terms of hepatic clearance): most barbi turates ; imipramine; meperidine;propranolol ; and quinidine. And no s igni ficant age-related changes? Examples are aspi rin, ethanol , prazos in, and as noted above, warfarin.

The chemica l class to which a drug belongs (a) a lso does not invariably predict age-related changes . Cons ider the benzodiazepines . There ares igni ficant age-related pharmacokinetic changes with diazepam (i ts ha l f-l i fe i s more or less di rectional ly proportional to age). In contrast, thereare no s igni ficant age-related di fferences in the ha l f-l ives of lorazepam or ni trazepam.

14. The answer is a. (Brunton, pp 131-138; Katzung, pp 59-60.) Biotransformation reactions involving the oxidation, reduction, or hydrolys is of a drug areclass i fied as Phase I (or nonsynthetic) reactions ; these reactions may resul t in ei ther the activation or inactivation of a pharmacologic agent. Thereare many types of these reactions ; oxidations are the most numerous . Phase II (occas ional ly ca l led synthetic) reactions , which a lmost a lwaysresul t in the formation of an inactive product, involve conjugation of the drug (or i ts derivative) with an amino acid, carbohydrate, acetate, sul fate—or glucuronic acid as noted in the question. The conjugated form(s ) of the drug or i ts derivatives may be more eas i ly excreted than the parentcompound.

15. The answer is c. (Brunton, pp 32-33, 1804, 1807-1808; Katzung, pp 42, 44-47.) With intravenous infus ions of a drug, only the drug’s ha l fl i fe determineshow long i t wi l l take for blood levels to reach a s teady s tate (on average, nei ther ri s ing nor fa l l ing thereafter) so long as the infus ion rate i s notchanged. By defini tion, when s teady s tate i s reached, the amount of drug entering the blood per uni t time i s equal to the rate at which drug i sbeing el iminated, whether by excretion, metabol i sm, or a combination of both (depending on the drug).

The apparent volume of dis tribution has no impact on time to CSS. Bioava i labi l i ty does not, ei ther, because with intravenous drugadminis tration the bioava i labi l i ty i s 1.0 (100%). Clearance, a parameter that relates el imination rate of a drug to the drug’s concentration [Cl = rateof el imination (mg/h)/drug concentration (mg/mL)]. Because clearance cons iders a rate of drug el imination, i t a ffects the CSS, but i t i s not adeterminant of i t.

The infus ion rate clearly affects the blood concentration reached at s teady s tate, but i t does not affect the time needed to reach i t. For example,i f we had a drug with a ha l f-l i fe of 4 hours , infused i t at a rate of x mg/min, and then repeated the experiment with the same drug at an infus ionrate of 2x mg/min, blood concentrations at s teady s tate would clearly be di fferent. However, i t would s ti l l take the same amount of time (roughlyfour to five ha l f-l ives ), to reach s teady s tate. (See the answer to Question 9 for related information.)

16. The answer is d. (Brunton, pp 139, 1012; Katzung, pp 57-59.) Grapefrui t juice (but not most other ci trus juices ) and juice from Sevi l le oranges (notothers ; see notes at end of the answer) conta in compounds (eg, naringin, furanocoumarins ) that can inhibi t the metabol i sm of severa l drugs , oneof which i s cyclosporine, via inhibi tory effects on CYP3A4). The route (pathway) of el imination (c) i s not changed, however. (Other drugs involved inthis interaction include verapami l , some of the s tatin-type cholesterol -lowering medications ; most of the second-generation antihis tamines ,including fexofenadine; and severa l antidepressants and antihypertens ives .) The resul t i s increased bioava i labi l i ty of an ora l ly adminis tereddose and increased area under the curve in a concentration versus time plot. The potentia l outcome is excess ive (and, occas ional ly, truly toxic)

Page 60: Pharmacology pre test self assessment and review (14th ed.)

effects .The “grapefrui t juice effect” nei ther a l ters the main route(s ) of drug absorption or el imination, nor affects plasma protein binding capaci ty,

because those are properties related to the drug, not how—or how wel l or quickly—it enters the ci rculation. Note that these data are cons is tentwith the hypothes is that grapefrui t juice does not s tati s tica l ly s igni ficantly accelerate or s low entry of cyclosporine into the blood, such as byaffecting intestina l transporters (b).

Changes of urine pH (a) or effects that involve drug binding to plasma proteins (e) are not appl icable, nor can such changes be gleaned from thedata presented in the question.

Notes : (1) The frui t-derived interactants are found in the “fleshy” materia l under the skin and between frui t segments . (2) Sevi l le oranges (a lsoknown as blood orange) are rarely used as juice oranges because of thei r very bi tter taste (and so they are a lso ca l led bi tter oranges). However,Sevi l le/blood orange juice i s now appearing more and more in heal th food s tores and other reta i l grocery s tores that sel l organic or other“natura l” foods . (3) There are severa l books , and many web s i tes , that s tate dogmatica l ly that “ordinary” oranges or orange juices interact withmany medications just as grapefrui t juice does . This has , natura l ly, a larmed many individuals . So far, sol id research evidence points to no druginteractions involving oranges other than Sevi l le. (4) Don’t be surprised to see furanocoumarin-free grapefrui t juice on the market soon.

17. The answer is d. (Brunton, pp 28-38; Katzung, pp 37-51.) Here i s how you solve the problem. Note: It’s easy to be mis led by incons is tent use of uni tsof measurement (mcg vs . mg, mL vs . L), so be sure you fi rs t convert uni ts as necessary for cons is tency.

Ca lculate the drug’s el imination rate constant:

Then ca lculate the clearance:

Reca l l that Cave = (F/Cl ) × (Dose/t), where F i s the bioava i labi l i ty (between 0 and 1.0) and t i s the dos ing interva l (given as 4 hours ).

Rearrange to solve for the dose.

Thus , Dose = 499,000 mcg, or 499 mg (close enough to 500 mg).

18. The answer is c. (Brunton, pp 26-38; Katzung, p 41.) Regardless of which adminis tration route has been used, i f we plot the log of plasmaconcentration of a drug versus time (and assuming fi rs t-order kinetics , which appl ies to the el imination of most drugs when blood levels aretherapeutic a fter ini tia l redis tribution), we get a s tra ight l ine. It i s described by the equation:

The s lope of this l ine, k, i s the el imination rate constant.An arguably more useful (and fami l iar) measure of the rate at which a drug i s el iminated i s the ha l f-l i fe (t1/2). It i s equal to 0.693/k, and i s

defined as the time i t takes for the concentration of a drug in the blood to fa l l to precisely one-hal f of what i t i s now (or at any speci fied time).The area under the curve (AUC) i s the integration of a time versus concentration plot for a drug. It i s a l inear—not a logari thmic or semi log—plot.

One use for plots of AUC is to estimate one’s “tota l exposure” to a drug, usual ly from “time zero” (instantaneous ly upon adminis tration) unti lblood concentrations of drug are no longer rel iably detectable or further measurements are impractica l . The AUC can be used to estimate tota lbody clearance of a drug, without the need to know the drug’s volume of dis tribution or i ts ha l f-l i fe, s ince

Determining AUC a lso enables us to ca lculate a drug’s bioava i labi l i ty. Bioava i labi l i ty (F) i s a measure of the fraction of an adminis tered dosethat i s absorbed systemica l ly and i s detectable in the plasma. Note that when a drug i s given intravenous ly, bioava i labi l i ty i s , by defini tion, 1.0(100%), s ince there are no barriers that might prevent the absorption of drug from the adminis tration s i te. So by adminis tering a drugintravenous ly, and a lso giving i t by another route, we can ca lculate bioava i labi l i ty. For example, assume the other route we use i s ora l (PO).

If we do our bioava i labi l i ty determinations by giving the same dose of the drug, the dosage uni ts in the above equation cancel out, and so

The extraction ratio (ER) i s a measure of a drug’s removal from the blood as i t passes through an organ (eg, the l iver) that can metabol i ze (orotherwise extract) i t, for example, from the arteria l to the venous s ide of that organ.

The rate of drug entry to an organ i s the product of blood flow (Q) and the arteria l concentration of the drug (CA). The rate at which the drugleaves i s flow × the venous concentration (CV). If flow into and out of an organ are identica l (as i t often i s ), then the extraction ratio can beexpressed as

Page 61: Pharmacology pre test self assessment and review (14th ed.)

We can a lso use the extraction ratio to ca lculate the organ clearance of a drug—that i s , the volume per uni t time from which an organ removes adrug

The volume of dis tribution (Vd) relates the amount of drug in the body to i ts concentration in the blood (or plasma). It i s typica l ly ca lculated asthe adminis tered dose divided by the concentration of drug in the blood

To s impl i fy the assessment of the kinetics we typica l ly give the drug intravenous ly (so we know how much drug enters the system—the enti redose, s ince bioava i labi l i ty = 1.0) and measure the concentration immediately thereafter (or use a plot of the log of drug concentration vs . time),then extrapolate to find drug concentration at “time zero” (the y-axis intercept).

19. The answer is a. (Brunton, pp 7-10; Katzung, pp 72-76, 1146.) Once the FDA approves a drug, you, as a l i censed phys ician, can prescribe i t for anythingyou des i re. Of course, there should be some scienti fi c bas is for your new-found or hypothes ized use, hopeful ly supporting or other rationalevidence in sol id peer-reviewed cl inica l l i terature. (You probably won’t, for example, see a β-blocker being used for treating anal warts , but i f youwanted to use one that way you are not violating any federa l laws . Just be sure you have a good reason to do so, and a good attorney shouldsomething bad happen and you get sued.) And i f you are conducting a cl inica l tria l at your insti tution to determine new information about futurecl inica l efficacy of a drug for which there are other approved uses , and even printed off-label uses , you’l l probably need Insti tutional ReviewBoard approval and wi l l have to provide wri tten informed consent to your s tudy subjects .

Note: Let’s cons ider just one drug in one drug class : fluoxetine, the prototype of the SSRI class of antidepressants . What i s i t approved for? Wel l ,depress ion, of course. But other approved or off-label uses for this antidepressant (and the identica l chemica l enti ty, same dose as used fordepress ion but “approved” under a di fferent trade name for premenstrua l dysphoric disorder) include the fol lowing: obsess ive-compuls ivedisorder; bul imia nervosa; a lcohol i sm; anorexia nervosa; ADHD; bipolar I I a ffective disorder; borderl ine personal i ty disorder; narcolepsy;kleptomania; migra ine; chronic da i ly headaches and tens ion-type headache; post-traumatic s tress disorder; schizophrenia ; levodopa-induceddyskines ia ; socia l phobia ; chronic rheumatoid pa in; panic disorder; and diabetic periphera l neuropathy. Oh, one more: trichoti l lomania—but don’ts tart pul l ing out your own hair, or get depressed, trying to memorize this incomplete l i s t of sometimes-odd uses .

20. The answer is c. (Brunton, pp 24-26; Katzung, pp 41-51.) The apparent Vd i s defined as the volume of fluid into which a drug appears to dis tributewith a concentration equal to that of plasma, or the volume of fluid necessary to dissolve the drug and yield the same concentration as that foundin plasma. By convention, we use the va lue of the plasma concentration, based on data such as those shown here, that has been extrapolatedback to “time zero”. Plotting the drug concentration (y-axis ) on a logari thmic sca le, as shown here, a l lows us to do the extrapolation eas i ly. In thisexample, that i s about 7 mcg/mL. Therefore, the apparent Vd i s ca lculated as :

The tota l amount of drug in the body ini tia l ly i s the dose we gave (100% bioavai labi l i ty, s ince we gave i t IV), 350 mg (ie, 5 mg/kg × 70 kg). Theestimated plasma concentration at zero time i s 7 mcg/mL (0.007 mg/mL). Putting these numbers in the equation yields the apparent Vd:

21. The answer is d. (Brunton, pp 24-27; Katzung, pp 1057-1058.) Most excess ive and adverse effects from a s ingle drug occur because of decl ines inrenal excretory function. It a ffects drugs that are el iminated without prior metabol i sm, those for which metabol ic inactivation plays a relativelysmal l role in el imination, and those drugs that form one or more active metabol i tes . Decl ines of hepatic function clearly occur in advanced age,whether in the presence or absence of other factors that might impair hepatic drug metabol i sm, but that does not seem to account for the majori tyof cases of excess ive drug effects . Body fat content tends to ri se, not decrease, with age (a).

22. The answer is b. (Brunton, p 24; Katzung, pp 38-41, 50, 1044, 1052, 1149.) The speed of any redis tribution or re-equi l ibration of bound/freeconcentrations of the target drug depends on the drug’s overa l l el imination ha l f-l i fe (which, in turn, i s a compos i te of ei ther or both excretion andmetabol i sm, depending on the drug). If a drug has an overa l l short el imination ha l f-l i fe, relatively speaking, the re-equi l ibrium between boundand free molecules wi l l , indeed, be reached quickly. If the drug has a relatively long ha l f-l i fe (and especia l ly i f relatively large amounts of theinteracting drug are present and/or the interactant binds avidly to plasma proteins ), then i t wi l l take longer for equi l ibrium to be establ i shedonce aga in (i f i t occurs at a l l ). If the drugs are in the ci rculation largely unbound, the consequences are less . However, cons ider warfarin: normal ly99% of a l l warfarin molecules at any given time are bound to plasma proteins . Displacement of even a smal l fraction of those bound moleculeswi l l have a large impact on the number of free (active) molecules , and warfarin’s normal average ha l f-l i fe i s about 2.5 days .

Bioava i labi l i ty of the drug (a) i s not important or relevant. No matter whether bioava i labi l i ty i s great (at or near 1.0) or smal l , once the drug i s inthe ci rculatory system the interaction wi l l occur, and reequi l ibrium wi l l be reestabl i shed nei ther s lower nor faster than under any otherci rcumstances . Likewise, unless blood pH changes (I did not say i t did, and there i s no reason to speculate that i t would), the pK of the drugs (c) i slargely i rrelevant. Renal clearance (d) may be important, but reca l l that many drugs are el iminated mainly or completely by metabol i sm, sowhether clearance i s important depends on the drug(s ). The same reasoning appl ies to answer e: i t i s not important how the drug i s el iminated, i ti s the rate at which overa l l el imination occurs .

23. The answer is e. (Brunton, pp 199, 1806; Katzung, p 66.) Tachyphylaxis i s defined as rapidly developing tolerance to the effects of an agonis t that i sadminis tered repeatedly, even i f the dosages given after the fi rs t one are not changed—or even progress ively increased to overcome the

Page 62: Pharmacology pre test self assessment and review (14th ed.)

phenomenon. It i s sometimes a lso ca l led desens i ti zation or downregulation of the receptor(s ), but those phenomena do not expla in a l l themechanisms by which tachyphylaxis (or tolerance in genera l ) occurs . The mechanism behind the development of tachyphylaxis—or s lower-developing drug tolerance in genera l—varies depending on which drug i s being used, the biochemica l processes by which the drug exerts i tseffects , and what the effector and the response(s ) are. For example, cons ider the rapidly diminished response of a variety of cel l s /ti ssues torepeated adminis tration of amphetamine, which acts by releas ing neuronal norepinephrine. Chal lenge the system repeatedly and the amount ofintraneuronal NE ava i lable to be released—which i s essentia l for caus ing the ul timate response—goes down. Another example, with morecl inica l relevance, i s the somewhat s lower development of “tolerance” of a i rway smooth muscles , and their abi l i ty to relax (ie, causebronchodi lation) in response to repeated adminis tration of β-adrenergic agonis ts , such as a lbuterol , arguably the most widely used adrenergicbronchodi lator for as thma. [Note: There i s no “magica l” number—hours or days , for example—that dis tinguishes between tachyphylaxis and“regular” tolerance. The brevi ty with which the tolerance develops i s the key point.]

24. The answer is d. (Brunton, pp 28-38; Katzung, pp 46-49.) The dose (D) to give equals the product of the target blood concentration and the drug’sclearance.

To s impl i fy things , let’s get the uni ts of volume the same for both clearance and concentration. The clearance of 0.08 L/min = 80 mL/min.Therefore

The s tated dos ing interva l i s 4 hours , so

which (rounded) i s closest to 150 mg.

25. The answer is d. (Brunton, pp 36-37; Katzung, pp 41-47.) Here the loading dose (D) equals the product of the target blood concentration (Cdesired) andthe volume of dis tribution (Vd).

As a lways , convert uni ts to make things cons is tent. The volume of dis tribution, 60 L, i s , of course, 60,000 mL

26. The answer is b. (Brunton, pp 127-131; Katzung, pp 54-59.) There are four major components to this mixed-function oxidase sys tem: (1) cytochrome P450,(2) NADPH, or reduced nicotinamide adenine dinucleotide phosphate, (3) NADPH-cytochrome P450 reductase, and (4) molecular oxygen.

Page 63: Pharmacology pre test self assessment and review (14th ed.)

The figure shows the cata lytic cycle for the genera l reactions dependent on cytochrome P450s . (Not a l l reaction s teps or intermediates areshown.) In s imple terms, the system acts as a monooxygenase, converting a reduced form of a drug (RH) to an oxidized form (ROH). For some drugs ,those metabol ic s teps are fol lowed by Phase II reactions , such as conjugation (with glucuronic acid, sul fate, etc) that further enhance theel imination of drug metabol i tes .

Cytochrome P450s cata lyze diverse oxidative reactions involved in drug biotransformation; they undergo reduction and oxidation during thecata lytic cycle. A prosthetic group composed of Fe and protoporphyrin IX (forming heme) binds molecular oxygen and converts i t to an activatedform for interaction with the drug substrate.

NADPH gives up protons to the flavoprotein (FP) NADPH-cytochrome P450 reductase. The reduced flavoprotein transfers these reducingequiva lents to cytochrome P450. The reducing equiva lents are used to activate molecular oxygen for incorporation into the substrate, as describedabove. Thus , NADPH provides reducing equiva lents , and NADPH-cytochrome P450 reductase passes them on to the cata lytic enzymes that comprisecytochrome P450.

Cyclooxygenase (COX; answer a) i s a genera l term for two main enzymes , COX-1 and 2, that metabol i ze endogenous arachidonic acid toprostaglandins , prostacycl ins , and other eicosanoids . Its substrate speci fici ty i s largely l imited to the endogenous molecules derived fromarachidonic acid, not xenobiotics . A good example of COX-1 and COX-2 i s aspi rin. Most other nonsteroida l anti -inflammatory drugs are a lso COX-1and COX-2 inhibi tors . The exemplar of a selective COX-2 inhibi tor i s celecoxib.

Monoamine oxidase (MAO; c) i s a flavoprotein enzyme that i s found on the outer membrane of mitochondria in certa in neurons , and a lso inhepatocytes . It oxidatively deaminates short-cha in monoamines only, and i t i s not part of the drug-metabol i zing microsomal system. ATP i sinvolved in the transfer of reducing equiva lents through the mitochondria l respi ratory cha in, not the microsomal system. There are various formsof MAO, dis tinguished mainly by thei r cel lular locations , thei r substrates , and their inhibi tors . MAO-A, for example, i s found in rather diverselocations , such as in the l iver and other cel l s . MAO-B i s the predominant form in certa in parts of the bra in (hence, “B”).

UDP-glucuronosyl transferase, a long with such enzymes as glutathione-S-transferase, methyl transferases , and N-acetyl transferases , areimportant in Phase II metabol i sm of various drugs . Substrates for these enzymes genera l ly are drugs or xenobiotics that were previous lytransformed by Phase I reactions , such as the actions of the CYP450 mixed-function oxidases . These enzymes add a functional group to a substrate,rather than chemica l ly modi fying the origina l substrate via oxidation, reduction, deamination, and the l ike—al l of which are respons ibi l i ties ofthe mixed-function oxidase systems.

Note: How did the cytochrome P450 system get the “450” des ignation? Simi lar to hemoglobin, cytochrome P450 i s inhibi ted by carbon monoxide.This interaction resul ts in molecules with a spectrophotometric absorbance spectrum peak at 450 nm, hence the des ignation 450.

27. The answer is b. (Brunton, pp 44-50; Katzung, pp 18–21.) Partia l agonis ts have both the abi l i ty to bind to receptors (affini ty) and the abi l i ty to evokea response by activating those receptors (efficacy), a lbei t weakly, under basa l conditions . Thus , when acebutolol i s adminis tered at rest (acondition under which endogenous catecholamine levels are low), heart rate ri ses s l ightly due to β-1 receptor activation via weak agonis t activi ty.

Page 64: Pharmacology pre test self assessment and review (14th ed.)

However, the occupation of adrenergic receptors by this weak agonis t reduces the number of receptors ava i lable to bind and respond to s trongeragonis ts (epinephrine, norepinephrine). As a resul t, the magnitude of the response to s tronger agonis ts in the presence of the partia l agonis t i slower than in the absence of i t (a l l other things being equal ).

28. The answer is d. (Brunton, pp 34-38; Katzung, pp 37-45.) With fi rs t-order el imination of a drug, we get a s tra ight l ine i f we plot the log of drugconcentration in the blood versus time once dos ing has s topped. The s lope of the l ine i s the el imination rate constant (kel). The drug’s ha l f-l i fe—avalue we wi l l need to use momentari ly—is related to kel as fol lows: t1/2 × (kel)/0.693. So, for the drug noted in the question, t1/2 = 0.693/0.35, orapproximately 2 hours . It takes approximately four to five ha l f-l ives to reach a s teady-s tate blood concentration. Do the math and you’l l see thatthe time for this drug to reach s teady s tate i s approximately 8 to 10 hours .

29. The answer is e. (Brunton, pp 32-33, 1804, 1807-1808; Katzung, pp 42-44.) After a 40-hour (about 1.67 days ) drug-free interva l passes the concentrationof the drug wi l l fa l l , as predicted by the ha l f-l i fe, to 3 mcg/mL; to 1.5 mcg/mL 40 hours after that; and to 0.75 mcg (now in the nontoxic range) afteryet another 40 hours passes . Thus we have to wait 120 hours , or five da i ly doses skipped, to achieve the goal .

30. The answer is e. (Brunton, pp 30-38; Katzung, pp 47-48.) You should reca l l that Vd = Dose/C0. We know what the dose i s . What we must ca lculate i sthe ini tia l drug concentration (C0)—the peak drug concentration that i s reached “instantaneous ly” after giving the IV bolus dose. Unfortunately,we’ve waited 30 minutes before taking our fi rs t blood sample, but that i s not a s igni ficant problem: plot the log of drug concentration versus timeand extrapolate to where the l ine intercepts the log-concentration axis (t0). This gives us a good estimate of C0. Plug the extrapolated C0 into theequation and you have your answer.

Note that bioava i labi l i ty i s not a factor in this ins tance, because with IV adminis tration bioava i labi l i ty i s , by defini tion, 1.0 (100%). Knowing orca lculating the el imination rate constant won’t help ei ther, because i t i s inextricably l inked to the ha l f-l i fe, which we a l ready know as kel =0.693/t1/2 (and you can rearrange the equation eas i ly.)

Ca lculating the AUC (concentration vs . time integra l ) with a bolus injection wi l l give us no information more useful than what we a l ready have.Clearance (genera l ly referring to renal clearance) i s i rrelevant in this s i tuation: I ’ve s tated that the drug i s completely metabol i zed to otherproducts ; thus , there i s no drug A to measure in the urine.

31. The answer is c. (Brunton, pp 35-36; Katzung, pp 37-46.) The scenario above makes ca lculations relatively s tra ightforward, particularly because noloading dose therapy wi l l be used.

Steady-s tate blood levels occur when the rate of “drug in” equals the rate of “drug out.” The volume of dis tribution, given in the question, i si rrelevant for the ca lculations .

The rate of drug out i s given as a Cl = 70 mL/minRecal l that the dose (D) = Cl × CSS

Therefore, with a l i ttle rearranging, the dose can be computed as :

Convert the uni ts so they’re cons is tent for both variables , and don’t forget to ca lculate how many minutes there are in the dos ing interva l , 6hours . Now, the rest of the math:

The target blood level of 4 mg/L i s the same as 4 mcg/mL, and this i s a reasonable change of volume uni ts to make for subsequent ca lculations ,s ince clearance i s given in uni ts of mL/min.

And s ince the drug wi l l be given every 6 hours :

32. The answer is c. (Brunton, pp 28-38; Katzung, pp 37-45.) Only the drug concentration at s teady s tate wi l l change: i t wi l l be greater with this a l teredprotocol . Do not be mis led by the numbers . The time to reach CSS wi th a constant drug infus ion i s a function of ha l f-l i fe (or the el imination rateconstant, which i s related to i t: kel = 0.693/t1/2), and that wi l l not change under the conditions s tated. Likewise, with the vast majori ty of drugsel iminated by fi rs t-order kinetics , there wi l l be no change of tota l body clearance or of volume of dis tribution. (Note: I described substance X asbeing pharmacologica l ly inert s imply so you didn’t conjure up confounding but largely i rrelevant i s sues , such as an active drug that i s able tocause res idual or long-las ting effects on i ts el imination or el imination rate, for example, a drug that caused long-lasting induction of hepaticdrug-metabol i zing enzymes.)

33. The answer is a. (Brunton, pp 27-28, 127-138; Katzung, pp 54-59.) Phase I metabol ic reactions genera l ly convert (via addi tion or unmasking of suchpolar functional groups as –NH2 or –OH through, say, oxidations , reductions , or deamination) very nonpolar (ie, very l ipid-soluble) drugs into morepolar (more water-soluble) metabol i tes . Among other things , polar metabol i tes of drugs in genera l are less l ikely to undergo tubularreabsorption. We can genera l i ze by saying that Phase I reactions play a role in forming metabol i tes that are “more eas i ly excreted.” If drug A, theparent drug, was a l ready very polar (answer b), there would be l i ttle need for Phase I metabol i sm. There i s no reason to assume that drug A wi l llack intrins ic biologica l activi ty (answer c); and because i t i s qui te l ipid-soluble to begin with, once in the ci rculation i t should have good abi l i ty todi ffuse across membranes and reach intracel lular s i tes (hence, e i s incorrect). Fina l ly, note that those reactions described as Phase II are theones that further increase polari ty (water-solubi l i ty) of some drugs via forming conjugates with glucuronic acid or sul fate.

34. The answer is e. (Brunton, pp 51-52, 55-58; Katzung, pp 21-29.) The genera l i s sue of l igand-receptor-response coupl ing involves s igna l transduction,and i s addressed in Question 5. The speci fic agonis ts noted in the question transduce their s igna ls and eventual ly cause a response by processesinvolving G proteins—a fami ly of guanine nucleotide-binding proteins . These l igands bind to the extracel lular face of the transmembrane protein.The various G proteins (eg, Gi, Gq, and Gs) bind to intracel lular portions of the receptor. They then couple the ini tia l l igand interaction to the

Page 65: Pharmacology pre test self assessment and review (14th ed.)

eventual response through a series of effector enzymes or enzyme systems that are G protein-regulated.For example, adenylyl cyclase can be activated, cata lyzing the formation of cAMP that then activates a particular kinase that phosphorylates

speci fic intracel lular proteins . However, the actua l s teps that occur after l igand binding depend on what the l igand i s , what speci fic G protein i sinvolved, which kinases are activated, and what proteins they phosphory-late. And what happens (ie, what the response i s ) depends on a l l of theabove and, of course, which cel l type i s being affected.

Activation of adenylyl cyclase and increased cAMP levels may occur in one system, but the oppos i te may occur in another. Some s ignaltransduction pathways involve phosphol ipase C, others do not. A ca lcium channel may be affected in one system and a potass ium channel (or noion channel ) in others .

By way of review, reca l l that there are three other main mechanisms or pathways for s igna l transduction about which we have reasonableknowledge.

One uses a receptor protein that spans the cel l membrane, but G proteins are not involved. On the inner membrane face the protein possessesenzymatic activi ty that i s regulated by the presence or absence of l igand bound to the extracel lular face. The tyros ine kinase pathway i s anexample, and the overa l l pathway i s respons ible for the activi ty of various growth factors and insul in (which i s a lso a growth-regulating hormone).

Another mechanism is used by very l ipid-soluble l igands that cross cel l membranes eas i ly and act on some intracel lular receptor. For example,glucocorticosteroids ul timately act in the nucleus and, through interaction with heat-shock protein (hsp90), eventual ly a l ter transcription ofspeci fic genes .

The thi rd involves transmembrane ion channels , the “open” or “closed” s tates of which are control led by l igand binding to the channel . Thisprocess appl ies to some of the important neurotransmitters , especia l ly those in the bra in (GABA, the main inhibi tory neurotransmitter) and suchamino acids as glycine, which exert “exci tatory” actions . The nicotinic receptor for ACh fi ts in this category too.

35. The answer is a. You may not find the defini tion of malignant in a pharmacology book, but you ought to know what i t i s . Many people think“mal ignant” invariably means cancer, but that i sn’t true. Remember mal ignant hypertens ion, mal ignant hyperthermia, neuroleptic mal ignantsyndrome? The term s imply means a condition i s rapidly getting worse. Nothing necessari ly l inked to cancer; no impl ication that the condition wi l lnot respond to usual ly effective (or even second- or thi rd-l ine) therapies and medications (refractory i s the term for res is tance to intervention); noguarantee, or even a l ikel ihood, that the condition wi l l be fata l i f i t i s treated properly.

36. The answer is d. (Brunton, pp 7-11; Katzung, pp 73-76, 1134-1136.) One of the most controvers ia l aspects of marketing herbals , nutri tionalsupplements , and other “nutriceutica ls” i s that manufacturers and sel lers have to provide no prior or scienti fi c proof of safety or efficacy to theFDA—something that must be provided, and with abundant and scienti fi ca l ly sound data , before the FDA wi l l approve a drug for sa le byprescription. Herbals , for example, came under the umbrel la of the Dietary Supplement Heal th and Education Act (DSHEA) of 1994.

Such products , you may have noted, come with labels that s tate “serving s i ze,” rather than dose, as i f we are going to be ingesting some carrotsinstead of a potentia l ly active and dangerous drug.

Sel lers of such products as herbals cannot make expl ici t cla ims that thei r products wi l l prevent, diagnose, treat, or cure any disease. DSHEAal lows a sel ler of a nutri tional supplement to say such obtuse things as “boosts your memory,” or “improves urinary tract heal th,” but they cannotstate “helps prevent or treat Alzheimer disease” or “reduces urinary frequency and nocturia i f you have benign prostatic hypertrophy.” In fact,DSHEA s imply requires that the label on the product s tates , in essence, that the FDA hasn’t eva luated any of the cla ims the manufacturer hasmade, pla inly or impl ici tly, on the label .

Moreover, whi le a drug company must provide and pay for the obl igatory precl inica l and cl inica l s tudies before a prescription drug gets the OK,i f there i s a reason or attempt to pul l a nutriceutica l from the market the FDA must do the work and pay the costs to prove that the product i sunsafe or ineffective.

Drugs needn’t be approved by the FDA as prescription drugs (answer a) before they can be sold OTC. Whi le such prior approval i s the trend, i fnot the norm (for example, cons ider many antihis tamines for a l lergy or insomnia; or drugs that inhibi t gastric acid secretion being marketed for“heartburn” or “acid indigestion), cons ider aspi rin. It’s been ava i lable OTC for decades and for i ts common uses (pa in, fever, inflammation) i t hasnever gotten an FDA bless ing as a prescription drug. Indeed, some say that i f aspi rin were being cons idered for approval as a prescription drugtoday, i t might not be approved because of the ri sks of serious adverse responses (eg, bleeding tendencies from antiplatelet effects ;bronchospasm in many asthmatics ) and drug-drug interactions . Nonetheless , we have many OTC drugs nowadays that fi rs t got approval for sa le “byprescription only,” for example, antihis tamines , inhibi tors of gastric acid secretion (noted above), and others . Approval for OTC sa le waspeti tioned by the manufacturer, and then approved only after reviews by and recommendations from FDA’s expert advisory panels .

Answer c i s incorrect. Once a drug gets approval for sa le and use as a prescription drug, and for a given indication (or more, depending on datasubmitted and approved), the phys ician can prescribe that drug for any (reasonable) purpose that he or she thinks i s appropriate, safe, andeffective. Phys icians and scientis ts learn or hypothes ize about new uses for a drug, conduct cl inica l s tudies , and overa l l can lega l ly prescribedrugs for the off-label or unlabeled uses .

Phase II I testing of drugs i s conducted in usual ly no more than 5000 patients with a disease for which the drug i s to be marketed. The main goali s to assess effectiveness for the s tated use, and for overa l l safety. These are rather tightly control led s tudies . The number of subjects , and theconditions in which the drug i s given, are qui te di fferent from what happens in the “rea l world”—including Phase IV, which i s the postmarketingsurvei l lance phase of drug approval , where tens or hundreds of thousands of patients may get the drug in largely uncontrol led ci rcumstances .Many drugs have come (been FDA-approved) and gone (pul led from the market because of adverse reactions or interactions , including thoseleading to death) because of what we’ve learned after the drug was approved by the FDA, and was approved for use by phys icians in conditionsthat weren’t nearly as wel l control led as those involved in pre-approval s tudies .

Page 66: Pharmacology pre test self assessment and review (14th ed.)

The Peripheral Nervous Systems: Autonomic and Somatic Nervous System Pharmacology Acetylchol ine synthes is , fates , and actionsAcetylchol inesterase and i ts inhibi torsAdrenergic agonis ts and antagonis tsAdrenergic receptor subtypesCatecholamine synthes is , fates , and actionsChol inergic receptor subtypesChol inergic agonis ts and antagonis tsGangl ionic s timulants and blockersPeriphera l nervous system structure and functionSkeleta l muscle relaxants (centra l ly acting)Skeleta l neuromuscular blockers

QuestionsNote: The fi rs t severa l questions in this chapter are based on a schematic diagram of the periphera l nervous system. I think this i s a sui table andefficient way to help you assess , or clari fy, some fundamenta l (and essentia l ) aspects of periphera l nervous system anatomy, (neuro)phys iology,and pharmacology based on a few main “rules” and key exceptions to them. I hope this approach wi l l be beneficia l .

The diagram below shows the main efferent pathways in the periphera l nervous systems—the autonomic (parasympathetic and sympathetic;PNS, SNS, respectively) and somatic nervous systems—from the CNS/spina l cord (at the far left, not labeled) out to the periphera l target (effectors ).The sweat glands shown in the figure represent eccrine sweat glands . The innervation of arrector pi l i muscles i s bas ica l ly the same.

Those nerves (and answer choices ) are:

a . Pregangl ionic parasympatheticb. Postgangl ionic parasympatheticc. Pregangl ionic sympatheticd. Postgangl ionic sympathetic to s tructures other than sweat glandse. “Pregangl ionic” (functional equiva lence) sympathetic to the adrenal (suprarenal ) medul laf. Pregangl ionic sympathetic, sweat gland (eccrine) innervationg. Postgangl ionic sympathetic to eccrine sweat glandsh. Motor nerve, somatic nervous system

Page 67: Pharmacology pre test self assessment and review (14th ed.)
Page 68: Pharmacology pre test self assessment and review (14th ed.)

37. Anatomic, neurochemica l , and pharmacologic s tudies of nerves a, c, e, f, and h indicate that they share one common property. Which s tatementbelow correctly summarizes what that i s?

a . Are chol inergic, activate postsynaptic muscarinic receptorsb. Are chol inergic, activate postsynaptic nicotinic receptorsc. Cannot release their neurotransmitter(s ) in the presence of atropined. Have the abi l i ty to activate a l l the adrenergic and a l l the chol inergic nervese. Recycle thei r neurotransmitter after each action potentia l , rather than synthes izing new transmitter de novo

38. Multidiscipl inary assessments of nerve d, a “typica l” postgangl ionic sympathetic nerve, indicate that i t i s qui te di fferent from a l l the othernerves shown in the periphera l nervous system schematic. Which s tatement describes that di fference?

a. Atropine selectively blocks activation of receptors by the neurotransmitter released from nerve db. It causes bronchodi lation (a i rway smooth muscle relaxation) when i t i s activatedc. It i s adrenergic (or noradrenergic i f you wish to use that term instead)d. The primary neurotransmitter synthes ized by nerve d i s epinephrinee. When nerve d i s phys iologica l ly activated by an action potentia l , actions of i ts released neurotransmitter are terminated mainly by hydrolys is in

the synaptic cleft

39. Reuptake (into the nerve) i s the main phys iologic process for terminating the postsynaptic activi ty of a periphera l nervous systemneurotransmitter. To which nerve does this process apply?

a. Nerve ab. Nerve bc. Nerve cd. Nerve de. Nerve ef. Nerve fg. Nerve gh. Nerve h

40. Nerve d, the more typica l postgangl ionic sympathetic nerve, i s activated by a normal ly generated action potentia l fol lowed by neurotransmitterrelease into the synapse. On which receptor type does the neurotransmitter i t releases act? Remember: You can select only one answer.

a . α1 adrenergic

b. α2 adrenergic

c. β1 adrenergic

d. β2 adrenergic

e. Muscarinicf. Nicotinicg. It depends on the target ti s sue (effector) type

41. Which s tatement correctly describes what i s rather unique about nerve g, the postgangl ionic fibers that innervate eccrine sweat glands ,compared with vi rtua l ly a l l other postgangl ionic sympathetic nerves?

a. Coca ine blocks release of i ts neurotransmitterb. Is chol inergicc. Is s timulated by pregangl ionic adrenergic nerves (nerve f)d. Its released neurotransmitter acts on nicotinic receptorse. Uses epinephrine as i ts neurotransmitter

42. Nerve g, the postgangl ionic nerve innervating eccrine sweat glands and arrector pi l i muscles , i s activated by a normal ly generated actionpotentia l and subsequent release of neurotransmitter from nerve f. On which receptor type does the neurotransmitter released by nerve g act?

a. α1 adrenergic

b. α2 adrenergic

c. Muscarinicd. Nicotinice. β1 adrenergic

f. β2 adrenergic

43. Assume that a l l the efferent autonomic pathways in the schematic are tonica l ly active (a reasonable assumption), even i f at low andquanti tatively di fferent levels . We add vecuronium (or any of severa l related drugs) to the system and, as expected, i t blocks neurotransmitteractivation of certa in s tructures . Which nerve innervates those s tructures and normal ly would activate them in the absence of pancuronium or i tsrelated drugs?

a. Nerve ab. Nerve bc. Nerve cd. Nerve de. Nerve e

Page 69: Pharmacology pre test self assessment and review (14th ed.)

f. Nerve fg. Nerve gh. Nerve h

44. Nearly every s tructure that i s influenced by sympathetic nervous system activi ty has postgangl ionic sympathetic (adrenergic) nerves innervatingi t. Which one of the fol lowing s tructures i s most defini tely affected by sympathetic nervous system influences , and responds to epinephrine, butlacks innervation by postgangl ionic adrenergic fibers and so i s not affected by norepinephrine released from adrenergic nerves?

a. Ai rway (eg, bronchiolar) smooth muscleb. Arteriolar smooth musclec. Iri s di lator muscles of the eyesd. Sinoatria l cel l s (pacemaker) of the hearte. Ventricular myocytes

Questions 45 to 49

The figure below shows some of the main elements of norepinephrine (NE) synthes is , release, actions , and other s teps in adrenergicneurotransmiss ion. You do not need the diagram to answer this series of questions , but seeing i t may a id your answering or reviewing. Note thatthe effector (target) cel l on the far right has ei ther an α-adrenergic receptor or a β-adrenergic receptor.

Page 70: Pharmacology pre test self assessment and review (14th ed.)

45. Mitochondria in the terminus of adrenergic nerve “endings” conta in an abundance of monoamine oxidase (MAO). What best summarizes thebiologica l role of the MAO in these adrenergic nerves?

a. Drives s torage ves icles that conta in norepinephrine to the nerve “ending” so that exocytotic norepinephrine release can occur in response to anaction potentia l

b. Metabol ica l ly degrades NE that i s free (not s tored in ves icles ) in the nerve terminalc. Metabol i zes dopamine to norepinephrined. Provides metabol ic energy for nonexocytotic release of norepinephrine in response to amphetamines and other catecholamine-releas ing drugse. Synthes izes ATP that i s required to transport free intraneuronal norepinephrine into the s torage granules/ves icles

Page 71: Pharmacology pre test self assessment and review (14th ed.)

46. Many s tudies have shown that a large fraction of norepinephrine (NE) in the normal resting adrenergic neuron i s s tored in membrane-boundves icles or granules . We adminis ter a drug that, over time, depletes this supply of neurotransmitter and decreases the intens i ty of responses tosympathetic nerve activation. In vi tro s tudies reveal that the drug acts by inhibi ting uptake of intraneuronal NE into the ves icles ; i t has no di recteffect on catecholamine synthes is , release, or interactions with i ts receptors . Which drug fi ts this description best?

a. Pargyl ineb. Prazos inc. Propranolold. Reserpinee. Tyramine

47. A substance abuser sel f-adminis ters coca ine and experiences a variety of s igni ficant changes in cardiovascular function, in addition to theCNS-stimulating effects for which the drug was used. What s tatement describes the mechanism by which the coca ine caused i ts main periphera land CNS effects?

a. Activates α2-adrenergic receptors leading to increased NE release

b. Blocks NE (and dopamine, in the CNS) reuptake via the amine pumpc. Di rectly activates postsynaptic α- and β-adrenergic receptors , leading to sym-pathomimetic (adrenomimetic) responsesd. Inhibi ts MAO, leading to increased intraneuronal NE levelse. Prevents NE exocytos is

48. We adminis ter a drug that i s a selective antagonis t at the presynaptic α-receptors (α2) in the periphera l nervous systems. It has no effect on α1

receptors , β receptors , or any other l igand receptors that are important in periphera l nervous system function. What i s the main response that i sl ikely to occur fol lowing adminis tration of the α2 -blocker?

a. Activation of the amine pump, s timulation of norepinephrine reuptakeb. Inhibi tion of dopamine β-hydroxylase, the enzyme that converts intraneuronal dopamine to norepinephrinec. Increased norepinephrine release in response to each action potentia ld. Inhibi tion of norepinephrine exocytos ise. Stimulation of intraneuronal monoamine oxidase activi ty

49. Not shown in the diagram is an enzyme that has the abi l i ty to metabol ica l ly inactivate NE that has di ffused away from the synapse. It i s a l sopresent in the l iver (as i s MAO) and in the intestina l wal l s . Among other things , the rapidi ty with which this enzyme catabol i zes i ts substratesaccounts for why norepinephrine, dopamine, and dobutamine have extraordinari ly short ha l f-l ives , must be given intravenous ly in order to causemeaningful effects , have negl igible effects when given by other parentera l routes , and are ineffective when given by mouth. An inhibi tor of thisdrug i s used therapeutica l ly, but not for i ts autonomic effects . What i s the name of this enzyme?

a. Aromatic L-amino acid decarboxylaseb. Catechol O-methyl transferase (COMT)c. Dopamine β-hydroxylased. Phenylethanolamine N-methyl transferasee. Tyros ine hydroxylase

50. We adminis ter a therapeutic dose of a drug that selectively and competi tively blocks the postsynaptic α-adrenergic (α1) receptors . It has noeffects on presynaptic α-adrenergic receptors (α2) or β-adrenergic receptors found anywhere in the periphery, whether as an agonis t or antagonis t.What i s the most l ikely drug?

a. Clonidineb. Phentolaminec. Phenoxybenzamined. Phenylephrinee. Prazos in

51. Festoterodine i s a relatively new drug that was heavi ly marketed to prescribers and di rectly to consumers . It i s indicated for the treatment ofan overactive urinary bladder, reducing the symptoms of urge incontinence, urinary urgency, and urinary frequency. It prevents phys iologicactivation of the bladder’s detrusor and s imultaneous ly prevents relaxation of the sphincter. Side effects include constipation, dry mouth, blurredvis ion, photophobia , urinary retention, and s l ight increases in heart rate. Another advisory for the drug: “Heat s troke and fever due to decreasedsweating in hot temperatures have been reported.” Festoterodine has no di rect effects on blood vessels that might change blood pressure. Basedon this information, what prototype drug i s most l ike festoterodine?

a. Atropineb. β-adrenergic blockers (eg, propranolol )c. Isoproterenold. Neostigminee. Phentolamine

52. A morbidly obese person vis i ts the loca l bariatric (weight loss ) cl inic seeking a pi l l that wi l l help shed weight. The phys ician prescribesdextroamphetamine. In addition to caus ing i ts expected centra l ly mediated anorexigenic (appeti te-suppressant) and cortica l -s timulating effects i tcauses a host of periphera l adrenergic effects that, for some patients , can prove fata l . What best summarizes the main mechanism by whichdextroamphetamine, or amphetamines in genera l , cause their periphera l autonomic effects?

a. Activates MAO

Page 72: Pharmacology pre test self assessment and review (14th ed.)

b. Blocks NE reuptake via the amine pump/transporterc. Displaces , releases , intraneuronal NEd. Enhances NE synthes is , leading to mass ive neurotransmitter overproductione. Stabi l i zes the adrenergic nerve ending by di rectly activating α2 receptors

53. We adminis ter a pharmacologic dose of epinephrine and observe (among other responses) a di rect increase of cardiac rate, contracti l i ty, andelectrica l impulse conduction rates . Which adrenergic receptor was respons ible for these di rect cardiac effects?

a. α1

b. α2

c. β1

d. β2

e. β3a

54. A patient with a his tory of narrow-angle (angle-closure) glaucoma experiences a sudden ri se of intraocular pressure that i s sufficiently severe.In addition to pa in there i s an imminent ri sk of permanent vis ion loss . They require immediate treatment, one element of which i s adminis trationof echothiophate. What enzyme is a ffected by this autonomic drug?

a. Tyros ine hydroxylase—stimulatedb. Acetylchol inesterase (AChE)—inhibi tedc. Catechol -O-methyl transferase (COMT)—inhibi tedd. Monoamine oxidase (MAO)—stimulatede. DOPA decarboxylase—stimulated

55. A patient has essentia l hypertens ion, and lab tests would show that thei r ci rculating catecholamine and plasma renin levels are unusual lyhigh. The chosen therapeutic approach for this patient i s to give a s ingle drug that blocks both α- and β-adrenergic receptors , thereby reducingblood pressure by reducing both cardiac output and tota l periphera l res is tance (systemic vascular res is tance). What drug i s most capable of doingthat?

a. Labeta lolb. Metoprololc. Nadolold. Pindolole. Timolol

56. A man who has been “surfing the Web” in search of an aphrodis iac or some other agent to enhance “sexual prowess and performance”discovers yohimbine. He consumes the drug in excess and develops symptoms of toxici ty that require your intervention. You consul t your preferreddrug reference and learn that yohimbine i s a selective α2-adrenergic antagonis t that acts mainly in the bra in’s medul lary “cardiovascular controlcenter.” What would you expect as a response to this drug?

a. Bradycardiab. Bronchoconstrictionc. Excess ive secretions by exocrine glands (sa l ivary, lacrimal , etc)d. Reduced cardiac output from reduced left ventricular contracti l i tye. Rise of blood pressure

57. You plan to prescribe scopolamine, as a transdermal drug del ivery system (skin patch), for a patient who wi l l be leaving for an expens ive cruiseand i s very susceptible to motion s ickness . What comorbidi ty would weigh aga inst prescribing the drug because i t i s most l ikely to pose adverseeffects—or be truly contra indicated?

a. Angle-closure (narrow-angle) glaucomab. Bradycardiac. His tory of a l lergic reactions to uncooked shel l fi shd. Resting blood pressure of 112/70e. Hypothyroidism, mi ldf. Parkinson disease (early onset, not currently treated)

58. A chi ld overdoses on a drug that affects both the autonomic and somatic nervous systems. As blood levels of the drug ri se he experienceshypertens ion and tachycardia , accompanied by skeleta l muscle tremor. Further elevations of blood levels of the drug cause a l l the expected s ignsand symptoms of autonomic gangl ionic blockade, plus weakness and eventual para lys is of skeleta l muscle. Which drug did the chi ld most l ikelyingest?

a. Bethanecholb. Nicotinec. Pi locarpined. Propranolol (or another β-adrenergic blocker)e. Scopolamine

59. Many cl inica l s tudies have shown that propranolol wi l l lower blood pressure to varying degrees in nearly every patient with essentia lhypertens ion (ie, high BP of unknown etiology or identi fiable causes , such as pheochromocytoma or vasopressor drug overdose). What i s the mostlikely and phys iologica l ly most important mechanism for propranolol ’s pressure-lowering effect?

Page 73: Pharmacology pre test self assessment and review (14th ed.)

a. Induced a baroreceptor reflex that reduces vasoconstriction usual ly caused by activation of the sympathetic nervous systemb. Inhibi ted catecholamine release from adrenergic nerves and the adrenal medul la (suprarenal medul la )c. Reduced heart rate and left ventricular contracti l i tyd. Reduced tota l periphera l res is tance via di rect vasodi lator actions involving ni tric oxide synthes is in endothel ia l cel l se. Stimulated renin release, ul timately leading to enhanced synthes is of vasodi lator chemica ls such as bradykinin

60. A 35-year-old man who weighs 150 pounds and i s 5 feet 10 inches ta l l i s transported to the emergency department in severe dis tress . Hecompla ins of episodes of severe, throbbing headaches , profuse diaphores is , and pa lpi tations . Eighteen months ago his phys ician told him he i sheal thy except for essentia l hypertens ion, but he refused medication and has not seen a heal th care provider for the las t year and a ha l f. Hedenies use of any drugs , whether prescription or over-the-counter, lega l or otherwise.

Assessment reveals that he i s tachycardic and has an i rregular pulse (occas ional premature ventricular beats are noted on his ECG). Heart rateat rest i s approximately 130 beats/min, sometimes more. His resting blood pressure i s 200/140 mm Hg. These cardiovascular findings are shown inthe figure below.

The fi rs t year house officer who i s caring for this patient knows that a l l the ora l ly effective β-adrenergic blockers are approved for use to treatessentia l hypertens ion, and concludes that prompt lowering of blood pressure i s essentia l for this patient. Therefore, he orders intravenousadminis tration of propranolol (at the arrow, above), and a large dose of the drug s ince the symptoms seem severe. Unknown to the phys ician i sthe fact that the patient’s s igns and symptoms are due to a pheochromocytoma (epinephrine-secreting tumor of the adrenal/suprarenal medul la ).

What i s the most l ikely ul timate outcome of adminis tering this β-blocker (or any other β-blocker that lacks α-blocking or other vasodi latoractivi ty), supplemented with no other medication?

Page 74: Pharmacology pre test self assessment and review (14th ed.)

a. Heart fa i lure, cardiogenic shock, deathb. Long-lasting normal ization of heart rate, contracti l i ty, and blood pressurec. Normal ization of blood pressures but pers is tence of tachycardiad. Restoration of normal s inus rate and rhythm, but no change of blood pressure from predrug levelse. Sudden and s igni ficant ri se of systol ic blood pressure and heart rate

Page 75: Pharmacology pre test self assessment and review (14th ed.)

61. A patient with chronic obstructive pulmonary disease (COPD, eg, emphysema, chronic bronchi ti s ) i s receiving an ora l ly inhaled muscarinicreceptor-blocking drug to mainta in bronchodi lation. What drug belongs to that class?

a. Albuterolb. Diphenhydraminec. Ipratropium (or tiotropium)d. Pi locarpinee. Vecuronium

62. You give an “effective dose” of atropine to a person who was poisoned with an AChE inhibi tor. What s tructure wi l l continue to be overactivatedby the excess ACh after the atropine i s given?

a. Ai rway smooth muscleb. S-A node of the heartc. Sa l ivary and lacrimal glandsd. Skeleta l musclee. Vascular smooth muscle

63. The house officer cons iders prescribing nadolol for a 53-year-old patient. What preexis ting condition (comorbidi ty) would most l ikelycontra indicate safe use of this drug?

a. Angina pectoris , chronic-s table (effort-induced)b. Asthmac. Essentia l hypertens iond. Heart fa i lure, mi lde. Sinus tachycardia

64. A variety of ophthalmic drugs , working by severa l main mechanisms of action, are useful for managing chronic open-angle glaucoma. Whichone reduces intraocular pressure by decreas ing the formation of aqueous humor, rather than by changing the s i ze of the pupi l (s )?

a . Echothiophateb. Isoflurophatec. Neostigmined. Pi locarpinee. Timolol

65. It’s fa i r to say that epinephrine, norepinephrine, and acetylchol ine play the most important roles as endogenous agonis ts for the variousreceptors under control of the periphera l nervous systems. However, dopamine a lso plays a smal l but important therapeutic role, particularlywhen adminis tered IV at low doses. What other periphera l effects can dopamine cause at usual therapeutic doses?

a. Bronchodi lation via relaxation of a i rway smooth musclesb. Di rect activation of pressure receptors (eg, baroreceptors ) in response to blood pressure changes triggered by other agonis tsc. Di rect activation of the juxtaglomerular apparatus , release of a ldosteroned. Inhibi tion of epinephrine release from chromaffin cel l s (eg, cel l s of the adrenal/suprarenal medul la )e. Regulation of renal blood flow via control of renal arteria l tone

66. Guanadrel i s an antihypertens ive drug: i t reduces arteriolar constriction, and in doing so lowers BP by reducing the amount of NE in periphera ladrenergic nerves . Lowered blood pressure i s not accompanied by reflex tachycardia , and in fact a reduction of heart rate from predrug levels i sthe more common outcome. The main ocular effect of guanadrel i s mios is . The widespread abol i tion of sympathetic influences throughout thebody often causes diarrhea and urinary frequency. Based on this description, to which prototypic drug i s guanadrel most s imi lar in terms of i tsul timate qual i tative autonomic effects , and in terms of overa l l mechanism of action?

a. Acetylchol ineb. Atropinec. Epinephrined. Isoproterenole. Norepinephrinef. Propranololg. Reserpine

67. An “effective” dose of a drug i s given and the fol lowing responses occur:

• Stimulates heart rate and, apparently, left ventricular s troke volume• Di lates some blood vessels but constricts none• Di lates the bronchi (relaxes a i rway smooth muscles )• Ra ises blood glucose levels• Neither di lates nor constricts the pupi l of the eye

What drug i s capable of caus ing a l l these responses?

a. Atropineb. Epinephrine

Page 76: Pharmacology pre test self assessment and review (14th ed.)

c. Isoproterenold. Norepinephrinee. Phenylephrine

68. It i s common to include smal l amounts of epinephrine (EPI) in solutions of loca l anesthetics that wi l l be adminis tered by infi l tration (injectionaround sensory nerve endings), as when a skin laceration needs suturing. What i s the most l ikely reason for, or outcome of, including the EPI?

a. To antagonize the otherwise intense and common vasoconstrictor and hypertens ive effects of the anestheticb. To counteract cardiac depress ion caused by the anestheticc. To prevent anaphylaxis in patients who are a l lergic to the anestheticd. To reduce the ri sk of toxici ty caused by systemic absorption of the anesthetice. To shorten the duration of anesthetic action

69. In your thi rd year of medica l school , you are conducting a s tudy on s imi lari ties and di fferences between two prototypic drugs , phentolamineand prazos in, as they affect cardiovascular responses . You selected two identica l twins to ensure their pharmacogenetic make-up, and so theirresponses to drugs , i s as nearly identica l as poss ible. One twin wi l l get the phentolamine, the other gets prazos in.

Their cardiovascular parameters at basel ine (rest) are identica l and normal , and everything else that might have an impact on their drugresponses i s normal and, wel l , identica l . They aren’t taking any other drugs .

One twin gets an IV injection of phentolamine. His mean blood pressure fa l l s 20 mm Hg in 20 seconds .The other gets an IV injection of prazosin a t an equal ly effective dose in terms of the blood pressure response. His mean BP fa l l s by an identica l

amount, 20 mm Hg, over the same time period, 20 seconds .Which is the most likely difference you wi l l find in the responses of these two lads , one who got phentolamine and the other who got an

equiva lent pressure-lowering dose of prazos in?

a. Phentolamine wi l l trigger a greater baroreceptor-mediated reflex increase of heart rate and contracti l i ty than the prazos in wi l l .b. Prazos in wi l l lead to a bigger reflex pos i tive inotropic and chronotropic response than phentolamine.c. Prazos in wi l l block a l l reflex cardiac responses because i t a l so has s trong β-blocking activi ty.d. Prazos in wi l l lead to a greater ri se of cardiac output than phentolamine by selectively blocking norepinephrine-mediated vasodi lation.e. Phentolamine wi l l reduce left ventricular afterload, prazos in wi l l ra ise i t.

70. A 48-year-old woman has a his tory of myasthenia gravis . She has been treated with an ora l acetylchol inesterase (AChE) inhibi tor for severa lyears , and has done wel l unti l now. Today she presents in your cl inic with muscle weakness and other s igns and symptoms that could reflectei ther a chol inergic cri s i s (excess dosages of her maintenance drug) or a myasthenic cri s i s (insufficient treatment). You wi l l use a rapidly actingparentera l acetylchol inesterase inhibi tor to help make the di fferentia l diagnos is . Which drug would, therefore, be most appropriate to use?

a. Edrophoniumb. Malathionc. Physostigmined. Pra l idoximee. Pyridostigmine

71. A patient presents with an anaphylactic reaction fol lowing a wasp s ting. What i s the drug of choice for treating the multiple cardiovascular andpulmonary problems that, i f not promptly corrected, could lead to the patient’s death?

a. Atropineb. Diphenhydraminec. Epinephrined. Isoproterenole. Norepinephrine

72. Cardiac output improves when dobutamine i s given, by IV infus ion, to a 60-year-old man with acute, symptomatic heart fa i lure. By whatadrenergic receptor-mediated actions , and through which ul timate effects of i t, do therapeutic doses of dobutamine mainly ra ise cardiac output?

a. α-adrenergic agonis tb. α-adrenergic antagonis tc. β1-adrenergic agonis t

d. β1-adrenergic antagonis t

e. Mixed α and β agonis tf. Mixed α and β antagonis t

73. We are us ing novel in vi tro methods to investigate the fate and post-synaptic actions of norepinephrine, released upon an action potentia lgenerated in an adrenergic nerve. An action potentia l i s generated and the postsynaptic effector briefly responds . Almost instantaneous ly theresponse i s over. What process mainly accounted for the brevi ty of the response, and termination of the released NE’s actions?

a. Metabol i sm by enzyme(s ) located near the postsynaptic receptor(s ) and/or in the synaptic cleftb. Reuptake into the nerve endingc. Metabol i sm by catechol -O-methyl transferase (COMT)d. Degradation by mitochondria l monoamine oxidase (MAO)e. Convers ion to a “fa lse neurotransmitter” in the nerve ending

74. We are contemplating adminis tration of a nonselective β-adrenergic blocker to a patient. In which of the fol lowing conditions i s this

Page 77: Pharmacology pre test self assessment and review (14th ed.)

cons idered genera l ly acceptable, appropriate, and safe?

a. Angina, vasospastic (“variant”; Prinzmeta l )b. Asthmac. Bradycardiad. Diabetes mel l i tus , insul in-dependent and poorly control lede. Heart block (second degree or greater)f. Hyperthyroidism, symptomatic and acuteg. Severe congestive heart fa i lure

75. A patient presents in the Emergency Department in great dis tress and with the fol lowing s igns and symptoms:

Which drug most l ikely caused these s igns and symptoms?

a. AChE inhibi torb. α-adrenergic blockerc. Antimuscarinicd. β-adrenergic blockere. Parasympathomimetic (muscarinic agonis t)f. Periphera l ly acting (neuronal ) catecholamine depletor

76. Acebutolol and pindolol are class i fied as β-adrenergic blockers with intrins ic sympathomimetic activi ty (ISA). In a practica l sense, what doesISA mean apropos the actions of these two drugs?

a. Are partia l agonis ts (mixed agonis t/antagonis ts )b. Cause norepinephrine and epinephrine releasec. Induce catecholamine synthes isd. Potentiate the actions of norepinephrine on α-adrenergic receptorse. Useful when cardiac pos i tive inotropic and chronotropic effects are wanted

77. A 10-year-old boy i s diagnosed with Attention Defici t/Attention Defici t-Hyperactivi ty disorder (ADD/ADHD). Which drug i s most l ikely to proveeffective for rel ieving the boy’s main symptoms?

a. Dobutamineb. Methylphenidatec. Pancuroniumd. Prazos ine. Scopolaminef. Terbuta l ine

Questions 78 to 84

The table below shows the perioperative medication adminis tration record (MAR) for an otherwise heal thy 50-year-old woman. (To make thingsinteresting—and wri te some additional questions—I’l l deviate from her actua l MAR and ask a few questions about relevant but hypothetica lscenarios .) She i s a nonsmoker, consumes no more than four glasses of wine per week, and has no personal or fami l ia l ri sk factors for heartdisease or diabetes . She i s taking no other medications . This patient had two scheduled (nonemergent) laparoscopic abdominal surgeries doneback-to-back in the OR. The fi rs t was to remove an adnexal (ovary and/or fa l lopian tube) mass that had been present for a couple of years andgrew s lowly and i rregularly over a couple of years , and to get a pathology report to see whether i t i s cancerous . Her pathology affected one ovary.Once the ovary was removed a di fferent surgeon performed a second procedure, through the same bel ly incis ions , to a l leviate some inconvenienturinary bladder problems.

Her MAR is very typica l of what you would find for many otherwise heal thy patients undergoing the same or s imi lar surgeries . The fi rs t drugl i s ted, midazolam, was given in the preop area right before she was transported to the OR (and I wi l l address i ts use in the CNS questions ; theabbreviation IVP next to midazolam means “IV push,” that i s , a bolus injection). Al l the rest were given in the OR by a CRNA (certi fied regis terednurse anesthetis t).

Use the table’s data , or what you should be able to infer from i t, to answer the next seven questions .

Page 78: Pharmacology pre test self assessment and review (14th ed.)

78. Ephedrine was the fi rs t drug adminis tered after anesthes ia premedication and induction with severa l parentera l drugs . What best describesephedrine’s mechanism of action?

a. Di rectly, selectively, s trongly s timulates α-adrenergic receptors in the periphera l vasculatureb. Enhances norepinephrine release by blocking presynaptic α-receptorsc. Increases norepinephrine synthes isd. Inhibi ts norepinephrine’s intraneuronal metabol ic inactivatione. Releases intraneuronal norepinephrine into the synapse, a lso weakly but di rectly activates a l l adrenergic receptors

79. Ephedrine and drugs that act in s imi lar ways or cause s imi lar effects have a variety of therapeutic uses . Given the background informationprovided for this patient, and the timing of ephedrine adminis tration on the medication adminis tration record, what i s the most l ikely reason whythe ephedrine was given?

a. Cause bronchodi lation so mechanica l support of venti lation would be eas ierb. Counteract CNS depress ion caused by the induction agentsc. Inhibi t s inoatria l and atrioventricular nodal automatici ty to terminate or prevent anesthes ia-related cardiac arrhythmiasd. Lower heart rate that was ra ised excess ively by the anesthetic drugse. Ra ise blood pressure that was lowered excess ively by the induction agents

80. Assume, hypothetica l ly, that the surgery had to be done emergently. The patient had been taking another drug—sti l l in her ci rculation atusual ly effective concentrations—that weakened but did not completely el iminate a l l of ephedrine’s periphera l autonomic effects . Which drug(s )would be most l ikely to do that?

a. Atenolol (or metoprolol )b. Coca ine or a tricycl ic antidepressant (eg, imipramine)c. Trimethaphan (or hexamethonium)

Page 79: Pharmacology pre test self assessment and review (14th ed.)

d. Pargyl ine (or a s imi lar nonselective MAO inhibi tor)e. Propranolol

81. Shortly a fter the surgica l wounds were closed and dressed, and right before the patient was transferred to the postanesthes ia care uni t (PACU),she received neostigmine. What was the reason for which i t was given?

a. Ra ise and support blood pressure during recoveryb. Ra ise or otherwise control bradycardia upon recovery from anesthes iac. Restore normal neurotransmiss ion in the bra in, s ince i t had been suppressed by the induction and anesthetic agentsd. Reverse skeleta l neuromuscular blockade/para lys ise. Suppress urinary bladder function to reduce the ri sk of postoperative bladder incontinence

82. The patient got her dose of glycopyrrolate mere seconds before the neostigmine was injected. This i s done routinely in thousands of surgeriesin which the drugs used are identica l or at least very s imi lar to those used here. To which pharmacologic class does glycopyrrolate most l ikelybelong?

a. α-Adrenergic agonis tb. Antimuscarinic (atropine-l ike)c. Chol inesterase inhibi tord. Muscarinic receptor agonis te. Nicotinic receptor (skeleta l muscle; NM) agonis t

83. The patient in the case presented here was not facing any emergent or imminently l i fe-threatening problems, so skeleta l muscle para lys is andintubation were relatively s tra ightforward procedures , not at a l l rushed. Now assume, hypothetica l ly, that emergency intubation was required, andsuccinylchol ine (SuCh) was used because of i ts very rapid onset of action. Assume further that a s low infus ion of SuCh was used to mainta inpara lys is throughout the procedure; blood levels of the drug were kept wel l in the therapeutic range; no curare-l ike (nondepolarizing) blocker wasused; and the patient had no genetic or other factors that would affect the drug’s pharmacokinetics or action. When the surgery i s done, what drugwould be given to reverse succinylchol ine’s effects?

a. Atropineb. Bethanecholc. Neostigmined. Physostigminee. Nothing

84. Now assume this patient received bethanechol severa l hours after her abdominal surgery, a fter effects of a l l the drugs used intraoperatively,except for morphine and ketorolac (analges ics ) had worn off. Her heart rate fel l s l ightly and she experienced some wheezing. Which word orphrase most l ikely accounts for or describes these cardiac and pulmonary responses?

a. Expected s ide effectsb. Idiosyncrasyc. Parasympathetic gangl ionic activationd. Reflex (baroreceptor) suppress ion of cardiac ratee. Undiagnosed asthma

85. Unti l recently the drug ri todrine has been used to suppress uterine contractions in some women who are in premature labor. Side effects ofthe drug include ri ses of heart rate and increases in the force of contraction of the heart. High-output heart fa i lure (caused by excess ivelyincreased cardiac output) and pulmonary edema have occurred, and in some cases the outcome has been fata l . Ri todrine a lso causesbronchodi lation. It has no vasoconstrictor effects , nor any effects on the s i ze of the pupi l (s ) of the eye(s ). The description most closely fi ts thecharacteris tics of what class of drugs?

a. α-Adrenergic agonis tb. Atropine-l ike/antimuscarinic drugc. β-Adrenergic agonis td. β-Adrenergic blockere. Muscarinic receptor agonis t (parasympathomimetic)

86. A patient with a his tory of as thma experiences s igni ficant bronchoconstriction and urticaria , and drug-induced his tamine release i s a maincontributor to these responses . Which drug i s most l ikely to have caused these problems—not because i t has any bronchoconstrictor or his tamineagonis t effects in i ts own right, but because i t qui te effectively releases his tamine from mast cel l s?

a . Atropineb. Morphinec. Neostigmined. Pancuroniume. Propranolol

87. During surgery the anesthes iologis t adminis ters trimethaphan, class i fied as an autonomic gangl ionic blocker to an anesthetized patient. Whatwould you expect in response to this drug?

a. Bradycardia mediated by activation of the baroreceptor reflexb. Increased GI tract moti l i ty, poss ible spontaneous defecationc. Increased sa l ivary secretions

Page 80: Pharmacology pre test self assessment and review (14th ed.)

d. Mios ise. Vasodi lation

88. “Fi rs t-generation” (older) his tamine H1 blockers such as diphenhydramine, phenothiazine antipsychotic drugs (eg, chlorpromazine), and tri -cycl ic antidepressants (eg, imipramine) have pharmacologic actions , s ide effects , toxici ties , and contra indications that are very s imi lar to those ofwhich other drug?

a. Atropineb. Bethanecholc. Isoproterenold. Neostigminee. Propranolol

89. A 6-year-old i s transported to the Emergency Department by a parent, who says the boy took a large amount of an a l lergy medication. The soleactive ingredient in the product the parent mentions i s diphenhydramine, and i t i s clear the chi ld i s experiencing toxici ty from an overdose. Theintern, fresh out of medica l school , orders parentera l adminis tration of neostigmine, and the medication order i s not questioned. What adverseeffect of the diphenhydramine wi l l pers is t fol lowing neostigmine adminis tration?

a. Bronchoconstriction and wheezingb. Del i rium, ha l lucinations , and other CNS mani festations of toxici tyc. Profuse secretions from lacrimal , mucus , and sweat glandsd. Skeleta l muscle tremor or fasciculationse. Tachycardia

90. In between your M1 and M2 years you are volunteering in a hospi ta l in a very poor part of the world. Their drug selection i s l imited. A patientpresents with acute cardiac fa i lure, for which your preferred drug i s dobutamine, given intravenous ly. However, there i s none ava i lable. Whichother drug, or combination of drugs , would be a sui table a l ternative, giving the pharmacologic equiva lent of what you want the dobutamine to do?(Al l these drugs are ava i lable in parentera l formulations .)

a . Dopamine (at a very high dose)b. Ephedrinec. Ephedrine plus propranolold. Norepinephrine plus phentolaminee. Phenylephrine plus atropine

91. In genera l , s tructures that are affected by sympathetic influences respond to both sympathetic neura l activation and to the hormonalcomponent, epinephrine released from the adrenal medul la . Which s tructure/function i s unique in that i t responds to epinephrine, but notnorepinephrine, and has no di rect sympathetic neural control?

a . Ai rway (tracheal , bronchiolar) smooth muscle: relaxationb. Atrioventricular node: increased automatici ty and conduction veloci tyc. Coronary arteries : constrictiond. Iri s of the eye: di lation (mydrias is )e. Renal juxtaglomerular apparatus : renin release

92. Adrenergic nerves to the heart are activated, leading to a reflex increase of heart rate and cardiac contracti l i ty, in response to a sudden ands igni ficant fa l l of blood pressure. Those sympathetic nerves release norepinephrine (NE). What i s the main phys iologic mechanism by which theactions of the released NE are terminated?

a. Di ffus ion away from postsynaptic receptorsb. Hydrolys is by nonspeci fic deaminasesc. Metabol ic inactivation by MAOd. Metabol ic inactivation by catechol -O-methyl transferasee. Reuptake into the adrenergic nerve from which the NE was released

93. The figure below shows severa l responses measured in a subject (heal thy; receiving no other drugs) at rest (before) and after receiving a doseof an unknown drug. Note: The blood pressures shown can be cons idered mean blood pressure; the fa l l caused by the unknown was mainly due toa fa l l of diastol ic pressure.

Page 81: Pharmacology pre test self assessment and review (14th ed.)

Which drug most l ikely caused the observed responses?

a. Atropineb. Isoproterenolc. Neostigmined. Phenylephrinee. Propranolol

94. A 59-year-old man has a his tory of emphysema from 40 years of cigarette smoking; hypercholesterolemia that i s being managed withatorvastatin; and Stage 2 essentia l hypertens ion for which he i s taking metolazone. He presents in cl inic today with his main new compla ints :nocturia , urinary frequency, and an inabi l i ty to urinate forceful ly and empty his bladder. Fol lowing a complete work-up, the phys ician arrives at adiagnos is of benign prostatic hypertrophy (BPH). We s tart da i ly therapy with tamsulos in. What i s the most l ikely s ide effect the patient mayexperience from the tamsulos in, and about which he should be forewarned?

a. Bradycardiab. Increased ri sk of s tatin-induced skeleta l muscle pathologyc. Orthostatic hypotens iond. Photophobia and other pa inful responses to bright l ightse. Wheezing or other exacerbations of the emphysema

95. A patient walks out of the ophthalmologis t’s office and into bright sunl ight after a comprehens ive eye exam, for which he received a topica lophthalmic drug. The drug has not only di lated his pupi l s , but a lso impaired his abi l i ty to focus his eyes up-close.

The drug this patient received was most l ikely class i fied as , or worked most s imi lar to, which prototype?

a. Acetylchol ineb. Epinephrinec. Homatropined. Isoproterenol

Page 82: Pharmacology pre test self assessment and review (14th ed.)

e. Pi locarpinef. Propranolol

96. You’ve received approval from the Insti tutional Review Board to s tudy the in vi tro (ti s sue bath) respons iveness of i solated human arteriolarsegments (obta ined during surgery) to a variety of pharmacologic and other interventions . The ti ssue samples are 1 cm-long “cyl inders” ofotherwise-normal (but now denervated) arterioles obta ined from the lower legs of patients undergoing amputation surgery.

The setup a l lows you to perfuse the vessels with a solution that wi l l keep the ti ssue functional ly and s tructura l ly intact for many hours ; tomonitor and change perfus ion pressure (mm Hg; analogous to “blood pressure” in the intact organism) and perfusate flow (mL/min); and toassess the effects of various vasoactive drugs on the system.

You add ACh to the perfusate to give a concentration identica l to the plasma concentration of ACh that causes “expected responses” in an intacthuman.

Under this experimenta l setup, adding ACh causes a ri se of perfus ion pressure and a decrease of flow, both of which bas ica l ly reflectvasoconstriction.

What i s the most l ikely explanation for these findings?

a. ACh released norepinephrine from the endothel ium, which caused vasoconstrictionb. Atropine was added to the ti ssue bath before adding the AChc. Botul inum toxin was added to the bath before adding the AChd. The vascular endothel ium has been damaged or removed (denuded)e. This response i s precisely what we’d expect with injection of ACh into the intact human

97. A 33-year-old woman becomes poisoned after receiving an injection of i l l i ci tly prepared and overly concentrated botul inum toxin. What i s themain neurochemica l mechanism by which this Clostridium toxin causes i ts effects?

a. Di rectly activates a l l muscarinic and nicotinic receptorsb. Inhibi ts ACh release from a l l chol inergic nervesc. Prevents neuronal norepinephrine reuptaked. Releases norepinephrine via a nonexocytotic processe. Selectively and competi tively blocks nicotinic receptors

98. A 43-year-old woman with diagnosed myasthenia gravis , and taking pyridostigmine da i ly, presents in the neurology cl inic with profoundskeleta l muscle weakness . You are unsure whether she i s experiencing a chol inergic cri s i s or a myasthenic cri s i s , so you adminis ter a usual lyappropriate diagnostic dose of parentera l edrophonium. Assume the patient was actua l ly experiencing a chol inergic cri s i s . What i s the most l ikelyresponse to the edrophonium?

a. Hypertens ive cri s i s from periphera l vasoconstrictionb. Myocardia l i schemia, and angina, from drug-induced tachycardia and coronary vasoconstrictionc. Premature ventricular contractions from increased ventricular automatici tyd. Prompt improvement of skeleta l muscle tone and functione. Venti latory dis tress or fa i lure

99. A patient takes a mass ive overdose of diphenhydramine, suffering not only s igni ficant CNS depress ion but a lso numerous and seriousperiphera l autonomic s ide effects . By what mechanism did diphenhydramine exert i ts untoward periphera l autonomic actions?

a. Activation of both β1 and β2 adrenoceptors

b. Blockade of α-adrenergic receptorsc. Competi tive antagonism of ACh actions on muscarinic receptorsd. Mass ive, di rect overactivation of gangl ionic nicotinic receptorse. Sudden release of epinephrine from the adrenal medul la (suprarenal medul la )

100. A patient with a recent drug poisoning i s transported to the emergency department. The phys ician correctly orders adminis tration ofpra l idoxime as part of the comprehens ive emergency treatment plan. Which best describes who the patient was?

a. A 13-year-old boy who to took an overdose of methylphenidate for his ADD/ADHD.b. A 43-year-old who took an overdose of neostigmine, prescribed for her myasthenia gravis , in a suicide attempt.c. A 6-year-old who got into the fami ly medicine cabinet and took 10 “adult doses” of her dad’s prazos in.d. A farm/field worker accidenta l ly doused with insecticide from an overflying crop-duster plane.e. An asthma patient who accidenta l ly gave himsel f an intravenous injection of epinephrine in an attempt to sel f-treat a developing anaphylactic

reaction.

101. To faci l i tate a certa in eye exam you want to cause mydrias is , but not a l ter normal control of accommodation. Al l of the fol lowing drugs areavai lable as topica l ophthalmic formulations . Which one wi l l di late the pupi l wi thout a l tering accommodation?

a. Atropineb. Epinephrinec. Homatropined. Isoproterenole. Pi locarpinef. Timolol

102. A 26-year-old woman has rhinorrhea, excess ive lacrimation, and ocular congestion from a bout with the common cold. Diphenhydramineprovides symptomatic rel ief. What i s the most l ikely mechanism by which this drug rel ieved her symptoms?

Page 83: Pharmacology pre test self assessment and review (14th ed.)

a. α-adrenergic activation (agonis t, vasoconstrictor)b. β-adrenergic blockadec. Ca lcium channel blockaded. His tamine (H2) receptor blockade

e. Muscarinic receptor blockade

103. You have treated dozens of patients with acute hypotens ion from various causes , including overdoses of antihypertens ive drugs , a l l of whichcan cause hypotens ion. Your usual approach to restoring blood pressure, and one that has worked wel l every time before, i s to inject x mg ofphenylephrine intravenous ly. Today a patient with severe drug-induced hypotens ion presents in the Emergency Department. He has been takingthis drug for many months . He i s not volume-depleted, nor hemorrhaging. You give the phenylephrine at the same dose and by the same route asyou a lways have. It causes no change of blood pressure. Which drug did the patient most l ikely take and overdose on?

a. Atenololb. Bethanecholc. Prazos ind. Propranolole. Reserpine

104. This i s a s trange day for you in the ED. Now you have to treat another normovolemic patient with acute drug-induced hypotens ion, and give theusual ly correct and effective dose of phenylephrine. This time the drug causes a vasopressor response that i s far greater than you’ve everencountered when giving the very same dose: systol ic pressure ri ses dramatica l ly, i f not dangerous ly. What drug did the patient most l ikely take?

a. Atenololb. Bethanecholc. Prazos ind. Propranolole. Reserpine

105. A patient who wi l l be new to your practice makes an appointment for thei r fi rs t vi s i t. Your nurse asks them to bring in a l l thei r currentprescribed medications and any OTC drugs they take regularly. They comply. One OTC drug i s a popular brand-name product that conta insdoxylamine as i ts sole active ingredient. Which property or characteris tic best describes this drug?

a. Likely to lower heart excess ivelyb. May be a cause of diarrhea that the patient says he often getsc. Should not be used by patients with prostatic hypertrophy or angle-closure glaucomad. Tends to ra ise blood pressure through a typica l catecholamine-l ike vasoconstrictor (α-adrenergic) mechanisme. Used during the day, helps them keep awake and a lert via a weak amphetamine-l ike action in the CNS

Page 84: Pharmacology pre test self assessment and review (14th ed.)

The Peripheral Nervous Systems: Autonomic and Somatic Nervous System Pharmacology

AnswersI ’ve reproduced below the figure you saw in the questions section, but modi fied i t to show the nerve types and locations , and the receptors , tosave you a bi t of page-fl ipping when you review your answers .

37. The answer is b. (Brunton, pp 171-177; Katzung, pp 80-87.) Here’s a s imple rule: in the periphera l nervous systems—that i s , the somatic nervoussystem and both branches of the autonomic nervous system—“the fi rs t nerve ‘out’ of the CNS (in this diagram, a, c, e, f, and h) i s a lways chol inergicand the ACh released from those nerves a lways activates the nicotinic subtype of chol inergic receptor on the postsynaptic target cel l (s ).”

38. The answer is c. (Brunton, pp 171-177; Katzung, pp 80-87) Another rule, with one important exception: “a l l the efferents in the periphera l nervoussystems are chol inergic except postgangl ionic sympathetics going to s tructures other than sweat glands” (and the arrector pili muscles too). There areeight nerves in the schematic. Seven of them—al l except the majori ty of postgangl ionic sympathetics (nerve d)—are chol inergic (synthes ize and

Page 85: Pharmacology pre test self assessment and review (14th ed.)

release ACh as thei r neurotransmitter). The main postgangl ionic sympathetic fibers (except those innervating sweat glands) synthes ize andrelease NE (not epinephrine; answer d) as thei r neurotransmitter, and so are adrenergic (or noradrenergic i f you prefer) nerves . What else can youcal l nerve d? A postgangl ionic sympathetic nerve to certa in smooth muscles , to cardiac muscle, and certa in exocrine glands except sweat glands .(Be sure to see the explanation for Questions 38 and 40.)

Atropine (a) i s incorrect. It selectively and competi tively blocks the effects of ACh (and other muscarinic agonis ts ) on muscarinic receptors . In thediagram above, those receptors are found on s tructures innervated by nerves b and g.

Answer b i s incorrect. NE, nerve d’s neurotransmitter, i s an effective agonis t for α-adrenergic receptors (α1 and α2) and for β1 receptors .Bronchodi lation caused by sympathetic activation requires activation of β2 receptors ; NE cannot do that but EPI, released from the adrenal(suprarenal ) medul la , certa inly can. And once NE has been released from i ts neurons and activates i ts postsynaptic receptors , i ts actions arepromptly terminated by reuptake (via an “amine pump” that can be blocked by coca ine or tricycl ic antidepressants ). Hydrolys is in the synaptic cleft(e) i s the mechanism by which the actions of ACh, released from chol inergic nerves , i s terminated.

39. The answer is d. (Brunton, pp 194-200; Katzung, pp 85-86.) The actions of norepinephrine (NE), released from adrenergic nerves , are terminated byneuronal reuptake. (Don’t forget that this reuptake process i s inhibi ted by coca ine and tricycl ic antidepressants , and the outcome is increased andmore prolonged adrenergic effects of NE because NE l ingers longer and accumulates in the synapse, exposed to i ts postsynaptic targets .) Al l theother nerves shown in the diagram are chol inergic; the actions of the ACh they release are terminated promptly by hydrolys is (viaacetylchol inesterase).

40. The answer is g. (Brunton, pp 119, 196-198, 201, 400-401; Katzung, pp 84-87.) The neurotransmitter released from nerve d, the postgangl ionicsympathetic fibers (to s tructures other than sweat glands and arrector pi l i muscles ), i s norepinephrine. NE can activate α-adrenergic receptors(both α1 and α2), and β1 receptors (not β2). Of course, di fferent s tructures have di fferent subtypes of these receptors : s tructures such as arteriolesand the i ri s di lator muscle have α1 receptors ; β1 receptors are found in the heart and in the juxtaglomerular apparatus (kidneys ; they help regulaterenin release there), whi le β2 receptors (not activated by NE) are found on various smooth muscles , mainly in the a i rways . So, the only correctresponse to the question “which receptors are activated?” rea l ly depends on which s tructure i s being innervated. As a fina l note, NE cannotactivate chol inergic receptors , and so answers e and f are defini tely incorrect.

41. The answer is b. (Brunton, pp 173-180; Katzung, pp 80f, 90t, 120.) The postgangl ionic sympathetic fibers innervating sweat glands and arrector pi l imuscles are chol inergic. That i s the exception to the rule that “a l l postgangl ionic sympathetic fibers are adrenergic.” How do you know i t’schol inergic? A variety of biochemica l and his tochemica l methods can prove that ACh i s the neurotransmitter. They a lso show that there i sabundant acetylchol inesterase (AChE), which hydrolyzes ACh, in the synaptic cleft. But we’re going to make the assessment pharmacologica l ly: Wecan prevent release of neurotransmitter from nerve g wi th botul inum toxin, which affects only (and all) chol inergic nerves ; and we can prevent theresponse of the sweat glands innervated by nerve g wi th atropine, the prototype muscarinic receptor antagonis t—a drug that has no effects onnicotinic (or other) receptors (eg, answer d) at usual doses . Likewise, nicotinic receptor blockers (or nicotine i tsel f) have no effect at this s i te—another reason why answer d i s incorrect. And how you do know i t’s part of the SNS? Sweat glands are activated (secretions are increased) andarrector pi l i muscles contract (the ha i r on our skin “s tands on end”) when the enti re SNS i s activated, such as in the “fight or fl ight response;” andi f you trace the origins of the pregangl ionic nerves that activate the postgangl ionic ones , they emanate from the same regions of the spina l cordfrom which a l l other sympathetic pregangl ionic fibers ari se—the thoracic and lumbar regions .

Coca ine (a) i s incorrect. It, and tricycl ic antidepressants (eg, amitriptyl ine, imipramine, etc), block neuronal reuptake of NE. That i s , i ts s i te ofaction i s at the neuroeffector junction of postgangl ionic sympathetic neurons (nerve d)—al l of them except those that innervate most sweat glandsand arrector pi l i muscles , of course.

42. The answer is c. (Brunton, pp 173-180; Katzung, pp 80f, 90t.) The nerve i s chol inergic, so the neurotransmitter i t acts on, postsynaptica l ly, must beei ther nicotinic or muscarinic. Nicotinic receptors are found on cel l bodies of a l l postgangl ionic nerves (in both SNS and PNS; NN receptors ), on theadrenal medul la (a lso NN), and on skeleta l muscle (somatic nervous system, NM)—at the “fi rs t synapses out of the CNS.” Chol inergic receptors ata l l other s i tes are muscarinic, “defined” by the fact that those receptors are competi tively blocked by atropine. Be sure to see the explanation forQuestion 37, above.

43. The answer is h. (Brunton, pp 177, 258-262; Katzung, pp 80f, 466-479, 474t.) Vecuronium and severa l related drugs (a l l of which have “curonium” or, atleast, “cur” in thei r generic names) are nondepolarizing skeleta l neuromuscular blockers (qui te di fferent, mechanis tica l ly, from succinylcho-l ine,the depolarizing blocker). They speci fica l ly and competi tively block activation of nicotinic receptors (NM) on skeleta l muscle by ACh, therebypreventing skeleta l muscle depolarization and subsequent contraction; and have no effect on muscarinic receptors or any of the adrenergicreceptor subtypes . Remember something about the natives of the Amazon obta ining a poisonous substance from the skin of certa in species offrogs , putting i t on the tips of thei r darts , and us ing that to ki l l thei r dinner with a blow-gun? That was curare, d-tubocurarine, the very oldprototype nondepolarizing skeleta l neuromuscular blocker, as we’ve come to know i t.

44. The answer is a. (Brunton, pp 173f, 194-200; Katzung, pp 80f, 84-87.) The neurotransmitter of postgangl ionic sympathetic nerves i s norepinephrine(unless the target cel l s are sweat glands or arrector pi l i muscles , in which case the fina l neurotransmitter i s acetylchol ine). Norepinephrine i s a“good” agonis t for α- and β1-adrenergic receptors , but not for β2. Ai rway smooth muscle cel l s are not innervated by the sympathetic nervous system.In terms of sympathetic influences , bronchodi lation (a i rway smooth muscle relaxation) i s caused only by epinephrine released from the adrenal(suprarenal ) medul la .

Here aga in i s the diagram to which Questions 45 to 49 referred.

Page 86: Pharmacology pre test self assessment and review (14th ed.)

45. The answer is b. (Brunton, pp 172, 194-210; Katzung, pp 87, 88f.) Whi le mitochondria in vi rtua l ly a l l cel l s in which they are found are important foroxidative phosphorylation and ATP synthes is , I asked about the mitochondria l MAO that i s ri ch in adrenergic neurons . There MAO wi l l degrade NEthat i s free (ie, not safely s tored away) in the s torage ves icles . If that intraves icular uptake i s inhibi ted, NE s tores wi l l be depleted.

None of the s tructures or processes shown in the diagram are respons ible for intraneuronal movement of NE-conta ining s torage ves icles to, andultimate fus ion with, the nerve “ending” (a). Metabol i sm of dopamine to NE (c) occurs in the s torage ves icles via the enzyme, dopamine β-hydroxylase. MAO does not provide “energy” for nonexocytotic NE release (d), and plays no role in ATP synthes is (e).

46. The answer is d. (Brunton, pp 211, 400-401, 783; Katzung, pp 83f, 178.) Reserpine blocks intraneuronal synthes is and s torage of norepinephrine (NE),and the abi l i ty of the reuptake process to re-s tore norepinephrine that has a l ready been released from an activated adrenergic nerve (see

Page 87: Pharmacology pre test self assessment and review (14th ed.)

Question 39), thereby expos ing the free intraneuronal NE to degradation by intraneuronal MAO. This i s important for other reasons : the fina lsynthes is of NE from i ts precursor, dopamine, occurs in the ves icles . If dopamine entry i s blocked, as i t i s by reserpine, NE synthes is i s decreased.Reserpine a lso blocks ves icular uptake of dopamine in parts of the CNS.

Pargyl ine (a) i s a nonselective MAO inhibi tor. Note that unl ike reserpine, MAO inhibi tors do not inhibi t intraneuronal s torage of NE. Rather, theyinhibi t metabol ic inactivation of NE in adrenergic nerve endings (or, at other s i tes , other monoamines). This leads to a “bui ld-up” ofnorepinephrine in adrenergic nerve endings . Prazos in (b) and propranolol (c) are adrenergic receptor blockers (α1 and β1-2, respectively) and haveno di rect effect on NE s torage. Tyramine (e) i s an indirect-acting sympathomimetic that displaces and releases neuronal NE via a process that doesnot involve exocytos is .

47. The answer is b. (Brunton, pp 116, 198; Katzung, pp 143, 451t, 460, 463.) Coca ine (and tricycl ic antidepressants such as imipramine) are class icexamples of drugs that inhibi t catecholamine reuptake by the “amine pump,” which i s the main process by which released NE (or dopamine) re-enters the neuron and i ts receptor-mediated effects are terminated phys iologica l ly. In the presence of coca ine or a tricycl ic, releasedneurotransmitter l ingers and accumulates in the synapse (neuroeffector junction), and so pertinent adrenergic responses appear heightened ormore intense, and prolonged. The pump is a lso important for the neuronal uptake, and ul timate effects , of such catecholamine-releas ing drugs asephedrine, pseudo-ephedrine, amphetamines , methylphenidate, and tyramine.

As long as an adrenergic synapse i s present (as i t i s in the diagram) and exposed to coca ine or a tricycl ic antidepressant, the drugs ’ effects arenot dependent on whether the effector (target) i s smooth muscle, cardiac muscle, or an exocrine gland of any sort.

Coca ine has no di rect effect on α2-adrenergic receptors (a), nor on any of the postsynaptic adrenergic receptors (c). Likewise, the drug has noeffect on MAO (d) nor on the exocytotic release of NE in response to an action potentia l (e).

There i s no di rect functional l ink between the amine pump and such processes as NE release (exocytotica l ly or otherwise) or activation of pre-synaptic (α2) adrenergic receptors .

48. The answer is c. (Brunton, pp 201-210, 212; Katzung, pp 84-87.) The adrenergic neuronal α2 (presynaptic) receptor, l ike a l l other adrenergic receptors ,i s G-protein-coupled. When the receptor i s activated by a sui table agonis t, i t s igna ls the neuron to s top further NE release fol lowing an actionpotentia l . Norepinephrine i tsel f i s one such agonis t; i ts activation of the presynaptic α2 receptor, which occurs concomitant with activation ofpostsynaptic (α1 or β1) adrenergic receptors (depending on what the dis ta l target cel l i s ), provides a phys iologic “feedback” s ignal that ha l tsfurther NE release. That i s , released NE regulates the release of more NE from the very neuron from which the neurotransmitter came.

Al though activation of both the presynaptic α2 receptors and NE reup-take by the amine pump/transporter occurs a lmost s imultaneous ly, thereis no di rect functional or biochemica l l inkage between the two. That i s , blocking (or s timulating) the α2 receptor wi l l not di rectly a ffect NE reuptake(a), nor wi l l drugs that affect the amine pump necessari ly have any di rect effect on the α2 receptors and the function they serve. There i s no effecton norepinephrine synthes is (b, whether by dopamine β-hydroxylase or other enzymes involved in neurotransmitter synthes is ), nor on MAO (d).

Note: Clonidine, which you typica l ly (and correctly) think of as a centra l ly acting antihypertens ive drug, acts as an α2 agonis t in the periphery. It’s“turning off” of NE release, then, can contribute to reduced vasoconstriction (and other adrenergic processes mediated by NE), which in turn helpslower blood pressure. However, clonidine i s a very l ipophi l i c drug. When i t i s given in usual doses , by the usual route (ora l or transdermal ), thedrug enters the CNS wel l , and rather promptly, and i t acts there (in the cardiovascular control center of the bra in’s medul la ) to inhibi t sympatheticoutflow. It i s that centra l effect that accounts for the drug’s main antihypertens ive mechanism.

49. The answer is b. (Brunton, pp 138, 200; Katzung, pp 87, 88f, 490-491.) Only COMT has the properties noted in the question. Entacapone i s an exampleof a COMT inhibi tor that i s used therapeutica l ly. It i s given mainly to manage Parkinson disease, in conjunction with levodopa. It inhibi ts theperiphera l (eg, intestina l ) convers ion of levodopa to dopamine (reca l l that dopamine i tsel f does not cross the blood-bra in barrier), and i s ana l ternative to carbidopa (which inhibi ts carboxylases ). (See Questions 113, 114, 147, 166, and 174-176, in the CNS chapter, for more information onthese and other dopaminergic drugs for parkinsonism.)

The other enzymes l i s ted in the question are involved in catecholamine synthes is . Aromatic L-amino acid decarboxylase (a) converts DOPA todopamine, which i s then converted to norepinephrine by dopamine β-hydroxylase (c). In the adrenal medul la , phenylethanolamine N-methyl transferase (d) converts some norepinephrine to epinephrine. Tyros ine hydroxylase (e) converts tyros ine to DOPA.

50. The answer is e. (Brunton, pp 206t, 212, 305-307; Katzung, pp 152f, 154, 167-180, 189.) Prazos in selectively and competi tively blocks α1 adrenergicreceptors and, unl ike many other α-blockers (phentolamine, phenoxybenzamine), has vi rtua l ly no presynaptic (α2) effects at usual doses .

None of the other drugs fi t the bi l l : Clonidine (a) i s a centra l ly acting α-adrenergic agonis t used mainly as an antihypertens ive drug. The mainconsequence of that effect i s a reduction of “sympathetic outflow” that reduces a l l of the three main determinants of blood pressure: heart rate,s troke volume, and tota l periphera l res is tance. Phentolamine i s a competi tive α-blocker, but i t blocks both presynaptic (α2) and postsynaptic (α1)receptors more or less equal ly wel l . Phenoxybenzamine (c) i s s imi lar to phentolamine in terms of i ts receptor targets . However,phenoxybenzamine’s actions las t far longer than those of phentolamine, and they arise from noncompetitive/irreversable α-blockade. (Rather thanmerely occupying the receptors , as i s the case with most antagonis ts , phenoxybenzamine a lkylates the receptors , “permanently” impairing thei rabi l i ty to interact with sui table agonis ts ). Phenylephrine (d) i s a s trong agonis t for a l l the α-adrenergic receptors , has no α-antagonis t activi ty, andexerts no di rect effects of any type on β-receptors .

51. The answer is a. (Brunton, pp 225-232; Katzung, pp 120-124). You probably never learned or read about festoterodine (i t’s s ti l l relatively new), butthat makes no di fference in terms of being able to answer this question.

The description of this drug i s , in rea l i ty, a l so an excel lent description of many of the properties of atropine, the prototype muscarinic receptorblocking drug, and of nearly a dozen other antimuscarinics marketed for “overactive bladder.” In terms of bladder function, parasympatheticinfluences tend to promote urine flow; antimuscarinics , then, tend to suppress that effect. (Conversely, sympathetic influences , via α-adrenergicreceptor activation, tend to suppress urine flow and α-adrenergic blockers cause the oppos i te.)

You should be able to el iminate incorrect answers by a s imple process of el imination. For example, do β-blockers (b) cause the visua l , urinarytract, sweat gland, or cardiovascular responses noted in the question? Not at a l l , nor do i soproterenol (c; β1/2 agonis t), neostigmine (d;chol inesterase inhibi tor, which produces precisely the oppos i te responses that atropine, festoterodine, and a l l the other antimuscarinics used forbladder overactivi ty do), or phentolamine (e; nonselective α-blocker).

Note: The mechanism of action of festoterodine, i ts indications , s ide effects , and adverse responses—and just about any other relevantproperty you can imagine, including the dose—are exactly the same as those of an older and widely adverti sed brand name drug, i ts sole activegeneric ingredient being tol terodine. Coincidenta l? Not at a l l . Both drugs have the same active metabol i te.

Page 88: Pharmacology pre test self assessment and review (14th ed.)

52. The answer is c. (Brunton, pp 116, 206t, 297-299; Katzung, pp 136f, 142.) Amphetamines (dextroamphetamine, methamphetamine, amphetamine, andsevera l related drugs) can be class i fied as indi rect-acting sympathomimetics (adrenomimetics ). They are transported into the adrenergic nerveending by the amine pump, displace NE from i ts s torage ves icles (via processes that are not dependent on an action potentia l ), and cause thestored neurotransmitter to be released into the synaptic space. At that point, a l l the expected effects of NE on i ts receptors and effectors occur.These drugs have no di rect effects , whether as an agonis t or antagonis t, on the adrenergic receptors . Their actions are whol ly dependent onintraneuronal catecholamine s tores .

Amphetamines and related drugs have no abi l i ty to di rectly activate (or inhibi t) MAO (a); to affect intraneuronal NE s tores (c); or s timulate (orinhibi t) intraneuronal NE synthes is (d). The amine pump upon which coca ine and related drugs mainly act i s located on the presynaptic adrenergicnerve “ending,” as are the main populations of α2 receptors ; however, these drugs do not di rectly affect, ei ther by s timulating or blocking, those α2

receptors (e).

53. The answer is c. (Brunton, pp 177-180, 283-286; Katzung, pp 89t, 90t.) The inotropic (contracti l i ty), chronotropic (rate), and dromotropic (conductionveloci ty-related) effects of epinephrine on the heart are mediated through activation of β1-adrenergic receptors . These receptor s i tes mediate anepinephrine-induced increased fi ring rate of the SA node (spontaneous , or Phase 4, depolarization), increased conduction veloci ty through the AVnode and the His -Purkinje system, and increased contracti l i ty and conduction veloci ty of atria l and ventricular muscle. Epinephrine activation of αadrenoceptors does not affect cardiac function in any phys iologica l ly or therapeutica l ly important way—except for the crucia l role of αadrenoceptors as mediators of coronary artery vasoconstriction (clearly a vascular smooth muscle, not cardiac muscle, phenomenon). The β2-adrenergic receptors play vi rtua l ly no di rect role in cardiac s timulation. They are more important in the relaxation of tracheobronchia l smoothmuscle, di lation of arterioles that serve skeleta l muscles , increased secretion of insul in by the pancreas , and to a lesser degree relaxation of thedetrusor of the urinary bladder. (Lipolys is in fat cel l s and melatonin secretion by the pineal gland appear to involve s timulation of β3-adrenergicreceptors . However, we do not have any cl inica l ly useful drugs that selectively activate or block the β3 receptors , and so you might want to questionhow much you learn about the β3s .)

54. The answer is b. (Brunton, pp 239-242, 245, 1785t, 1788; Katzung, pp 105-111.) Echothiophate iodide i s a long-acting (“i rrevers ible,” which bas ica l lymeans “very long acting” in terms of the duration of effects ) acetylchol inesterase inhibi tor. It i s used topica l ly on the eye for the treatment ofvarious types of glaucoma. By increas ing synaptic levels of acetylchol ine, i t increases contraction of the ci rcular muscles of the eye, caus ing thepupi l s to constrict (mios is ). That reduces mechanica l obstruction of aqueous humor outflow via the Canal of Schlemm and the trabecular“meshwork” in the angle of the anterior chamber. (In contrast, sympathetic activation causes mydrias is by activating the radia l muscles through α-agonis t effects , and can lead to increased intraocular pressure.) Maximum reduction of intraocular pressure occurs within 24 hours , and the effectmay pers is t for severa l days . The drug i s water-soluble, which affords a practica l advantage over the l ipid-soluble i soflurophate (anotherchol inesterase inhibi tor used to treat glaucoma).

Note: Had I l i s ted a drug with a generic name ending in the suffix “-s tigmine,” you no doubt would have answered correctly and immediately.However, echothiophate i s one of those chol inesterase inhibi tors that i s not a s tigmine. If you’re unfami l iar with the drug—this one or any one asyou work your way through the questions—by a l l means look i t up!

Tyros ine hydroxylase (a) i s involved in the biosynthes is of catecholamines . COMT (c) i s involved in extraneuronal metabol i sm ofcatecholamines , whi le MAO (d) i s the major intraneuronal enzyme that degrades NE in adrenergic nerves . Forms of MAO are a lso found in, andimportant in, the l iver and parts of the CNS. DOPA decarboxylase (e) cata lyzes the metabol i sm of DOPA (dihydroxyphenyla lanine) to dopamine aspart of the catecholamine biosynthetic pathway.

55. The answer is a. (Brunton, pp 328-329, 780-781; Katzung, pp 151-162.) Labeta lol i s a competi tive antagonis t at both α- and β-adrenergic (both β1 andβ2) receptors . [This “trick” might help you remember that labeta lol blocks both main types of adrenergic receptors : take the fi rs t two letters in“labeta lol” and reverse them—la tranposes to al, as in alpha—and then add the next four letters—beta. You’l l s imply have to memorize the name ofanother important α-/β-blocker, carvedi lol .) Note, too, that the generic names of a l l the drugs l i s ted end in—olol—a great tip-off that a drug has β-blocking activi ty. In fact, every drug with a generic name that ends in -olol i s a β-blocker.

Labeta lol ’s α-blocking actions are weak compared with i ts actions at the β-receptors , but they are cl inica l ly important in terms of bloodpressure control nonetheless . This di fference i s somewhat concentration-dependent: at relatively low blood levels , as might be achieved withtypica l ora l doses , i t i s about three times more potent as a β-blocker than as an α-blocker (do not commit this to memory!). With higherconcentrations , such as those often achieved with parentera l (eg, IV) dos ing, the intens i ty of β-blockade increases cons iderably with l i ttleincrease in α-blocking efficacy (another point that’s not priori ty knowledge now!). One of labeta lol ’s main uses i s for managing essentia lhypertens ion (a l l the ora l ly adminis tered β-blockers are indicated for essentia l hypertens ion, and for chronic-s table angina). It lowers pressure inthree ways : reduces heart rate, reduces contracti l i ty, and a lso reduces renin release, thereby reducing the formation of angiotens in I I and,indirectly, release of a ldosterone. Al l these effects are mediated by β1 blockade. Given parentera l ly (IV), labeta lol i s often a fi rs t-choice agent formanaging urgent hypertens ive cri ses when more efficacious (and potentia l ly more dangerous) IV drugs aren’t indicated or cannot be given safely.

Metoprolol (b; and atenolol and acebutolol , not l i s ted) have a preferentia l (but certa inly not absolute!) and dose-dependent effect on β1

receptors (“cardioselectivi ty”) versus β2, and have no α-blocking actions . Relatively high doses of those drugs can (and often do) block β2 receptors ,and any dose may pose serious ri sks of bronchoconstriction or broncho-spasm for patients with asthma. Nadolol (c) and timolol (d) arenonselective β-blockers . (One property to remember for nadolol i s i ts relatively long ha l f-l i fe; for timolol you might want to reca l l that a topica lophthalmic dosage form is often used for chronic openangle glaucoma.) Pindolol (d) i s a nonselective β-blocker and a lso exerts s trong intrins icsympathomimetic activi ty (ISA; ie, partia l agonis t-antagonis t activi ty).

56. The answer is e. (Brunton, pp 207t, 212, 309, 383t; Katzung, p 155.) Whether you memorize that yohimbine i s a selective α2 antagonis t i s up to you,but you should know what the main effects of a selective α2 antagonis t are. That, of course, depends on knowing what α2 receptors—at least inthe periphera l autonomic nervous system—do in the phys iologica l sense. Reca l l that the preponderance of phys iologica l ly important α2 receptorsare located on adrenergic nerve terminals (or nerve “endings”). When s timulated by a sui table agonis t, the response i s a turning-off of further NErelease. Because NE i s the neurotransmitter released from adrenergic nerves and i t i s an excel lent α agonis t, the presynaptic α2 receptors uponwhich NE acts serve as the main phys iologic mechanism for regulating neurotransmitter release.

So, when we block those receptors with yohimbine, we enhance the apparent overa l l activi ty of the sympathetic nervous system on i ts effectorsby interfering with norepinephrine’s abi l i ty to turn-off i ts own release. Of the responses l i s ted, only hypertens ion (owing to the vasoconstrictoreffects of NE on postsynaptic α-adrenergic receptors ) occurs as a resul t of yohimbine (or of NE excess ). The other main effects you shouldanticipate would be cardiac s timulation (rate, contracti l i ty, electrica l impulse conduction rates ; a l l β1-mediated effects ) and, usual ly of lesscl inica l consequence, mydrias is (α).

Page 89: Pharmacology pre test self assessment and review (14th ed.)

In terms of the aphrodis iac effects , the drug probably increases arousa l via an action in the CNS, but the mechanism isn’t known for sure; and i tincreases the vigor of ejaculation, which i s predominantly an α-mediated effect. (Reca l l that erection primari ly involves muscarinic-chol inergicvascular effects , mediated by enhanced production of endothel ium-derived relaxing factor, ie, ni tric oxide.)

57. The answer is a. (Brunton, pp 230-233, 920, 923, 1785t; Katzung, pp 99-105.) With any antimuscarinic drug—and scopolamine certa inly i s one—narrow-angle glaucoma (which accounts for about 10% of a l l glaucomas) i s the biggest concern (assuming no other relevant comorbidi ties ). Thedrug might provoke s igni ficant ri ses of intraocular pressure as i t further reduces aqueous humor dra inage, caus ing not only pa in but vis ionproblems that might be severe or permanent. Bradycardia (b) i s not a concern; i f anything, usual doses of scopolamine may increase heart rate abi t. Should the patient eat some shel l fi sh or other a l lergy-provoking food (c), the incidence or severi ty diarrhea might be reduced or preventedal together by the scopolamine, due to i ts effects on longi tudina l muscles in the gut (inhibi ted) and on sphincters (activated). A resting bloodpressure of 112/70 or thereabouts (d) i s not at a l l uncommon or worrisome, and not l ikely to be changed at a l l by the drug. Hypothyroidism (e)typica l ly i s associated with s l ight bradycardia ; aga in, no l ikely problem with scopolamine. And i f our patient had mi ld Parkinson disease (f), wemight actua l ly, eventual ly, see a l i ttle improvement with this drug. Reca l l , one s trategy to manage parkinsonism—which i s bas ica l ly a centra limbalance between dopamine and ACh, the latter appearing to be acting in relative excess—is to block centra l muscarinic receptors (with suchdrugs as benztropine or trihexyphenidyl ; see CNS chapter).

58. The answer is b. (Brunton, pp 189, 206t, 255-258, 270-272; Katzung, pp 24, 87-89, 101f, 109-110) Nicotine ini tia l ly s timulates and then blocks nicotinic-skeleta l muscular (NM) and nicotinic-neura l (NN; in autonomic gangl ia and on adrenal medul lary cel l s ) chol inergic receptors . (Both effects involvedepolarization of the target cel l s , and in many ways i t i s comparable to the depolarizing blockade of skeleta l muscle activation by succinylchol ine—ini tia l s timulation fol lowed by inhibi tion.) Ini tia l gangl ionic s timulation leads to vasoconstriction and hypertens ion, both of which aremani festations of sympathetic activation. Bradycardia may or may not occur; parasympathetic gangl ionic activation i s l ikely to increase thepredominant parasympathetic (bradycardic) tone on heart rate, but sympathetic activation may cause the oppos i te effect. Ini tia l s timulation ofskeleta l muscle NM receptors would account for the tremor. As nicotine’s blood levels ri se we get autonomic gangl ionic blockade, leading tohypotens ion and bradycardia . Subsequent blockade at the skeleta l neuromuscular junction leads to muscle weakness and respiratory depress ioncaused by interference with the function of the diaphragm and intercosta ls . Bethanechol (a ) and pi locarpine (c) are chol inomimetics that, at usualblood levels , exert thei r primary effects as di rect agonis ts on muscarinic receptors for ACh, not on nicotinic receptors . β-Blockers (d) would notcause hypertens ion or tachycardia . Indeed, overdoses would change blood pressure and heart rate in the other di rection. Moreover, propranololi tsel f i s sometimes use to manage skeleta l muscle tremor of certa in etiologies . Scopolamine (e) i s a muscarinic blocker with vi rtua l ly no effectson skeleta l muscle (nicotinic responses).

59. The answer is c. (Brunton, pp 313-319, 774t; Katzung, pp 161-164, 178-179.) Reca l l that the bas ic “equation” for blood pressure i s :

and s ince CO = heart rate × s troke volume (HR × SV), then

Propranolol i s , of course, a nonselective (β1 and β2) β-adrenergic blocker that lacks any α-blocking or other vasodi lator effects (eg, due to ni tricoxide generation), regardless of the dose. So to answer the question, given the above equation, you s imply ask which one or more of the variables—HR, SV, or TPR—is reduced by a nonselective β-blocker. The answer, of course, i s both HR and SV.

Rapid parentera l adminis tration of propranolol or another β-blocker in the same class wi l l lower blood pressure, and i f that occurs quicklyenough (eg, with rapid- and brief-acting β-blocker esmolol ) and blood pressure fa l l s by a sufficient magnitude, a baroreceptor reflex (answer a)wi l l be activated. This reflex sympathetic activation wi l l lead to increased periphera l vasoconstriction, s ince α-adrenergic receptors are notblocked by propranolol or any other β-blockers except for labeta lol or carvedi lol (therefore answer a i s incorrect), but this wi l l not lead to reflex-mediated increases of ei ther HR or SV. No β-blocker has the abi l i ty to inhibi t catecholamine release, whether from adrenergic nerves or from theadrenal (suprarenal ) medul la (b). As noted above, there i s no mechanism by which a β-blocker such as propranolol wi l l lower TPR (d). (Labeta loland carvedi lol can, by a-blockade; and nebivolol does by a ni tric-oxide mediated mechanism.) Fina l ly, β-blockers reduce renin release (a β1-mediated effect) from the kidneys ’ juxtaglomerular apparatus , and so answer e i s incorrect.

60. The answer is a. (Brunton, p 744t; Katzung, pp 155-156.) The figure, below, approximates what i s l ikely to happen fol lowing adminis tration of a β-blocker to a patient with a pheochromocytoma.

Page 90: Pharmacology pre test self assessment and review (14th ed.)

To answer this question correctly you must integrate your bas ic knowledge of both autonomic pharmacology and cardiovascular phys iology. Witha pheochromocytoma we have what might be described as “mass ive” amounts of catecholamines—mainly epinephrine—being released from thetumor into the bloodstream. Germane to our problem, then, i s excess ive s timulation of cardiac rate, contracti l i ty, impulse conduction, andautomatici ty (β1); intense vasoconstriction (α); and vasodi lation in some vascular beds (β2).

Nonetheless , whi le the patient I ’ve described i s not at a l l hea l thy, he i s s ti l l a l ive before we give the β-blocker. But what happens after we givethe β-blocker i s cri ti ca l . The β-blocker does nothing to block the α-mediated vasoconstriction, so in terms of only vascular effects BP wi l l remainvery high. If you wish to opine that blocking β2-mediated vasodi lation wi l l ra ise BP a bi t, that’s fine; I ’ve shown that in the figure above.(Remember: Usual ly effective doses of a β-blocker wi l l ei ther lower BP [most common response], or not affect i t at a l l , in patients with essentia lhypertens ion. If pressure ri ses in response to a β-blocker, suspect pheochromocytoma.)

Nonetheless , there wi l l not be a sudden and s igni ficant ri se or normal ization of BP and/or cardiac function (b, c, d). Likewise, heart rate wi l l notri se s igni ficantly (e); i t can’t: the β1 receptors necessary for that to occur are blocked.

Next you must reca l l that the inotropic s tate of the heart (ie, of the left ventricle, LV) i s cri ti ca l . This i s cri ti ca l because LV peak systol ic pressuremust exceed aortic pressure in order to establ i sh the LV-aortic pressure gradient to expel blood into the aorta , and propel blood throughout theci rculation. If aortic diastol ic pressure exceeds LV peak systol ic pressure, blood wi l l not flow out of the heart. But unti l we give the β-blocker, theheart can mainta in (for a whi le) i ts function thanks to catecholamine-mediated s timulation.

But when we give just a β-blocker to the pheochromocytoma patient we have done l i ttle i f anything to lower the a l ready high aortic pressure,and s imultaneous ly have inhibi ted cardiac contracti l i ty, rate, and other key parameters . The function of the heart as a pump is suppressed. It nowfaces a very high aortic pressure, and ul timately i t fa i l s . Cardiac output fa l l s , the patient develops cardiogenic shock, and i s then l ikely to die.

This i s why, when treating hypertens ion associated with a pheochromocytoma, we give an α-blocker fi rs t. Blood pressure wi l l fa l l . We then dealwith the excess ive cardiac s timulation (which may be intens i fied, via the baroreceptor reflex, in response to a sudden and s igni ficant BP fa l l ) bygiving a sui table β-blocker immediately thereafter.

61. The answer is c. (Brunton, pp 226-227, 230-231; Katzung, pp 120, 127t, 339-357.) Ipratropium, a quaternary (and so poorly absorbed) antimuscarinicdrug, i s FDA-approved for use as an inhaled bronchodi lator for COPD. Its action involves blockade/antagonism of ACh-mediatedbronchoconstriction, and i t i s often used adjunctively with a lbuterol or other β2 agonis t bronchodi lators . A related drug i s tiotropium.

Albuterol (a ) i s an inhaled bronchodi lator for as thma or COPD, but i t works , of course, as a β2-adrenergic agonis t. Diphenhydramine (b) has

Page 91: Pharmacology pre test self assessment and review (14th ed.)

bronchodi lator activi ty (by blocking both his tamine H1 and muscarinic receptors ), but i t i s not given by inhalation; moreover, for ambulatorypatients with asthma the mucus-thickening effects of muscarinic receptor blockade can do more harm than good. Vecuronium (e) i s a curare-l ikeskeleta l neuromuscular blocker (competi tive antagonis t of ACh at NM receptors ) and i s used mainly for caus ing intentional skeleta l musclepara lys is during surgry to suppress spontaneous venti lation in some ICU patients on a venti lator. Pi locarpine (d) i s a muscarinic agonis t, usedmainly for caus ing mios is (constricting the pupi l [s ] of the eye[s ]) in some patients with angle-closure glaucoma. Pi locarpine wi l l causebronchoconstriction—an effect that may be harmful , i f not fata l , for patients with COPD or asthma.

62. The answer is d. (Brunton, pp 189-194, 225-230; Katzung, pp 80f, 105-1111, 1034-1036.) In “chol inesterase pois ioning” we are deal ing withoverstimulation (from accumulated ACh) of both periphera l muscarinic and nicotinic receptors . Reca l l that atropine i s a speci fic muscarinicreceptor blocker, and the muscarinic receptors are the ones found on such s tructures as smooth muscle, cardiac nodal ti s sue, and exocrine glands .In contrast, the chol inergic receptor on skeleta l muscle i s nicotinic, so skeleta l muscle i sn’t a ffected by atropine. If one receives a letha l dose of achol inesterase inhibi tor, he or she may be a l i ttle more comfortable (less defecation, urination, respiratory tract mucus hypersecretion andbronchoconstriction, and a l l that, a fter the atropine i s given), but they are s ti l l l i kely to die from skeleta l muscle (NM nicotinic) overstimulationand then para lys is . Para lys is of the diaphragm and intercosta ls muscles are the most letha l consequences .

63. The answer is b. (Brunton, pp 316-317; Katzung, pp 157-165.) You probably learned that nadolol i s a β-blocker, and perhaps that i t’s a nonselective(β1/β2) blocker with a rather long duration of action and ha l f-l i fe. If not, you should have been able to recognize that s ince the generic name endsin “-olol” i t i s , indeed, some kind of β-blocker. You should a lso know that asthma is one of the main contra indications for β-blockeradminis tration. β-Adrenergic blockade resul ts in an increase in a i rway res is tance (by antagonism of bronchodi lation mediated by epinephrine)that can be fata l in some asthmatic patients . Nadolol , and a l l the other ora l ly effective β-blockers , are approved for treating chronic-s table angina(a) and essentia l hypertens ion (c). They are cri ti ca l elements in contemporary therapy of heart fa i lure (d; except when i t i s severe and cardiacoutput i s profoundly depressed); and have beneficia l effects to control heart rate in such conditions as s inus tachycardia (e).

64. The answer is e. (Brunton, pp 318, 1787; Katzung, pp 160, 163-164.) When appl ied topica l ly to the eye, both the di rect-acting chol inomimetic agents(eg, pi locarpine) and those that act by inhibi tion of AChE (eg, echothiophate, i soflurophate, and physostigmine) cause mios is by constricting thepupi l . In patients with narrow-angle (angle closure) glaucoma, this effect provides greater dra inage of the aqueous humor through the trabecularmeshwork and the canal of Schlemm and a reduction in res is tance to outflow of the aqueous humor. Certa in β-adrenergic blocking agents (eg,timolol and levobunolol ), appl ied to the eye(s ), are a lso very useful in treating chronic wide-angle glaucoma. These drugs appear to act bydecreas ing the secretion (or formation) of aqueous humor by antagonizing the effect of ci rculating catecholamines on β-adrenergic receptors inthe ci l iary epi thel ium. Echothiophate (a), i soflurophate (b), and neostigmine (e) are acetylchol inesterase inhibi tors and a l l constrict the pupi l .Pi locarpine (d) i s muscarinic receptor agonis t; i t, too, causes mios is .

65. The answer is e. (Brunton, pp 288, 351-359; Katzung, pp 84t, 136f, 138t, 141, 148, 225.) Infus ing low doses of dopamine rather preferentia l ly activatesD1 receptors , ra ises vascular smooth muscle cAMP levels , and causes vasodi lation in the kidneys and mesentery. (Cl inicians often refer to these as“renal doses” because, in such conditions as low cardiac output the drug i s given not only as a cardiac s timulant but a lso to increase renal bloodflow and urine output.) The outcome of the increased renal blood flow is increased glomerular fi l tration and sodium excretion. A s imi lar D1/cAMP-mediated effect a lso occurs in the renal tubules (particularly the proximal tubules and the medul lary region of the thick ascending l imb in theLoop of Henle). This inhibi ts a Na + -K+ -ATPase, which further increases renal sodium loss by inhibi ting sodium reabsorption.

66. The answer is g. (Brunton, pp 206t, 210, 782-783; Katzung, pp 84-87, 178, 189.) The qual i tative and mechanis tic effects of guanadrel are very s imi lar tothose of reserpine. Both drugs cause what has been described as a “chemica l sympathectomy,” gradual ly but eventual ly markedly reducingsympathetic (adrenergic) influences on a l l the usual target s tructures and thereby expos ing or unmasking oppos ing parasympathetic influences .(The resul ting syndrome has been ca l led “parasympathetic predominance” s ince adrenergic influences are removed, leaving PNS tone on a host ofs tructures unopposed and now exposed.) For example, in the presence of guanadrel mios is , diarrhea and urination develop; and arterioles di lateand blood pressure fa l l s (because there’s l i ttle/no norepinephrine to be released, s ince i t has been depleted from adrenergic nerve endings).Heart rate does not increase reflexly (via the baroreceptor reflex mechanism) for two main reasons . Fi rs t, heart rate cannot ri se because after asufficient duration of treatment there i s l i ttle or no NE to be released. The second reason for no baroreceptor reflex i s that guanadrel (andreserpine) cause BP to fa l l so s lowly that the baroreceptors s imply do not respond to any BP changes . Indeed, heart rate fa l l s markedly duringtreatment. Reserpine causes i ts effects mainly by interfering with intraneuronal s torage of NE, expos ing the neurotransmitter to metabol icinactivation by MAO and ul timately depleting the NE. Guanadrel releases NE di rectly, and gradual ly, eventual ly leading to a s tate of intraneuronalNE depletion with s igns and symptoms very s imi lar to those caused by reserpine.

None of the other drugs l i s ted wi l l have any effects on the levels of NE in adrenergic nerves . Acetylchol ine (a), the phys iologic agonis t for bothmuscarinic and nicotinic receptors , wi l l cause many of the responses described for guanadrel , but, aga in, does not do so by reducingcatecholamine levels in adrenergic neurons . It’s a l so worth noting that ACh i s not class i fied—and certa inly not used—as an antihypertens ive drug.The periphera l autonomic effects of atropine (b) adminis tration would be the oppos i te of those described for guanadrel . Usual parentera l dosesof epinephrine (c; α and β1 and β2 agonis t) or norepinephrine (e; α and β1 agonis t) tend to ra ise blood pressure and periphera l res is tance (theeffects clearly are dose-dependent), and i f there were any changes in pupi l s i ze i t would be mani fest as mydrias is due to activation of α-receptorson i ri s di lator muscle cel l s . Isoproterenol (d; β1 and β2 agonis t with no di rect effects on α receptors ) may ra ise BP by increas ing cardiac output (viaincreased heart rate and s troke volume). Propranolol (f), the prototypic β1 and β2 antagonis t, certa inly i s an antihypertens ive drug, but i t and otherβ-blockers do not affect intraneuronal NE levels ; do not cause any change in pupi l s i ze (ie, nei ther mydrias is nor mios is ); and do not typica l lyunmask parasympathetic influences on such s tructures as the gut and urinary tract musculature.

67. The answer is c. (Brunton, pp 283f, 289-290; Katzung, pp 136-139, 141-142, 148, 343.) Piece things together. Some tip-offs to help arrive at the correctanswer: no effect on the s i ze of the pupi l , so rule out any drug that has effects on the “parasympathetic” s ide, whether as an agonis t or antagonis t(here, atropine), and rule out any drug with α effects . (As noted above, no β-receptors control the s i ze of the pupi l of the eye.)

Does i soproterenol fi t a l l the other cri teria/properties? Yes . And none of the other choices have a l l the s tated properties , only some of them.

68. The answer is d. (Brunton, pp 575-576; Katzung, p 457.) There are severa l reasons for and outcomes of adding a vasoconstrictor (usual lyepinephrine) to some loca l anesthetics given by infi l tration. The vasoconstrictor confines the loca l anesthetic to the des i red s i te of action(bas ica l ly the s i te of adminis tration) by reducing loca l blood flow and the rate of anesthetic entry into the bloodstream. It i s the bloodstream (or,more precisely, i t i s the presence of anesthetic in the systemic ci rculation) that del ivers the drug to s i tes where s igns and symptoms of toxici tyoccur. Slow down anesthetic absorption rates and the drug wi l l enter the ci rculation s lowly enough that i t can be metabol i zed (inactivated) fast

Page 92: Pharmacology pre test self assessment and review (14th ed.)

enough to prevent accumulation to toxic levels . That i s , we reduce the ri sk of systemic toxici ty with added vasoconstrictor.Another outcome of including a vasoconstrictor i s to prolong (and certa inly not shorten) the duration of loca l anesthetic effect. That occurs a lso

because reduced loca l blood flow keeps the anesthetic in the vicini ty of sensory nerves longer (s ince the anesthetic i s not being removed asquickly by blood flow).

More on this i s sue wi l l be presented in the CNS questions . But for now: Loca l anesthetics (except coca ine) can cause vasodi lation (notvasoconstriction; a ), but unless the loca l anesthetic dosages are qui te high (toxic), the vasodi lation i s not intense; hypotens ion i s uncommon withusual dosages . Likewise, cardiac depress ion can occur, but aga in that i s a mani festation of overdose. We don’t routinely include epinephrine in aloca l anesthetic to combat or prevent these cardiac-depressant problems (b), and the amount of epinephrine found in these preparations i s fartoo low to do anything meaningful to remedy these adverse responses should they occur. Likewise, the amounts of vasoconstrictor are far too lowto prevent (or treat) anesthetic-induced anaphylaxis (c)—a reaction that requires only a few molecules of antigen to occur.

So what the vasoconstrictor does i s essentia l ly cause a pharmacologic tourniquet, reducing regional blood flow that otherwise would quickly“wash away” the anesthetic. This essentia l ly confines the anesthetic to the des i red s i te longer (not shorter; e) than otherwise, and decreases thepotentia l sys temic reactions (d). Some loca l anesthetics cause vasodi lation, which a l lows more compound to escape the ti ssue and enter theblood.

Two fina l notes : In plastic surgery, otolaryngologic surgery (nose, throat) and many other types of surgery, the di rect vasoconstrictor effects ofEPI, adminis tered as part of a mixture of the anesthetic and epinephrine, not only antagonize the loca l vasodi lator effects of a loca l anesthetic,but a lso cause di rect vasoconstriction in the region—a loca l hemostatic effect that l imits blood loss in yet another way. It i s a l so important tonote that epinephrine (or other vasoconstrictors ) should not be used in or as a supplement to parentera l loca l anesthetics being used to rel ieveor prevent pa in in “end organs”—fingers , toes , penis , tip of the nose, earlobes , etc. Such s tructures have relatively discrete blood suppl ies , andblocking blood flow to dis ta l ti s sues can lead to i schemia and ti ssue damage.

69. The answer is a. (Brunton, pp 305-307, 309, 774t, 779-780, 793; Katzung, pp 152-154, 167, 180, 189.) Fi rs t you need to know that prazos in andphentolamine are α-adrenergic blockers . Then you need to know upon which α-adrenergic receptors they act, and what responses they el ici t.Phentolamine i s a nonselective α-adrenergic blocker, acting at both α1 (postsynaptic) and α2 (presynaptic) receptors . Prazos in selectively blocks α1

receptors . Activating the postsynaptic receptors , on vascular smooth muscle cel l s , causes contraction (vasoconstriction); block those receptors andvasodi lation (ie, the fa l l of blood pressure we noted) occurs . Both phentolamine and prazos in do this .

The blood pressure fa l l upon drug adminis tration i s of a sufficient magnitude (20 mm Hg), and occurs sufficiently quickly (20 seconds or less ),that the baroreceptor reflex i s activated. This leads to increased sympathetic “outflow” to the periphery as part of the body’s attempt to mainta inthe cardiovascular s tatus quo. Sympathetic s timulation of vascular smooth muscle normal ly wi l l cause vasoconstriction, but s ince the α-adrenergicreceptors that must be activated to cause i t are blocked (by phentolamine or prazos in), vasoconstriction wi l l not occur (or be very much reduced).

Increased baroreceptor-mediated sympathetic s timulation of the heart a lso occurs , and nei ther phentolamine nor prazos in blocks the β1

receptors on which released NE exerts i ts cardiac-s timulating effects .But what else occurs at the adrenergic neuroeffector junctions , such as at the heart? Normal ly, some of the released NE acts on the presynaptic

α-receptors . The resul t of that interaction i s a phys iologic “turning off” or feedback-inhibi tion of further NE release in response to each actionpotentia l . Phentolamine blocks the presynaptic α-receptors , thereby blocking what amounts to the abi l i ty of released NE to suppress further NErelease. The consequence of this? Signi ficant increases in the amount of NE released into the synapse, and prolonged presence of NE in thesynapse, and s igni ficantly increased cardiac s timulation. Prazos in, however, does not block the presynaptic α-receptors , and in doing so does notinterfere with the normal feedback-inhibi tory effects of released NE. And the obvious di fference? Much less intense reflex cardiac s timulationwith prazos in than that with phentolamine—not the oppos i te (b).

Prazos in and phentolamine have no β-blocking activi ty (c). Nei ther drug blocks NE-mediated vasodi lation (d). And nei ther wi l l reduce LVafterload (e): as noted in the question, both drugs lowered arteria l pressure and so afterload i s obvious ly decreased too.

70. The answer is a. (Brunton, pp 234, 251-252; Katzung, pp 105-112.) Al though severa l of the l i s ted drugs inhibi t the activi ty of AChE, only edrophoniumis used in the diagnos is of myasthenia gravis . The drug has a more rapid onset of action (1 to 3 minutes fol lowing intravenous adminis tration)and a shorter duration of action (approximately 5 to 10 minutes ) than pyridostigmine. This fas t acting/short duration profi le i s precisely what wewant in this s i tuation. We can quickly get our diagnostic answer, yet not have to deal too long with adverse responses (such as venti latorypara lys is ) i f the patient was experiencing a chol inergic cri s i s (excess ive doses of thei r ora l chol inesterase inhibi tor) and we’ve now worsened thes i tuation by inhibi ting the metabol ic inactivation of ACh even more with our diagnostic medication. (The short duration and the need forparentera l adminis tration preclude use of edrophonium as a practica l drug for long-term treatment of myasthenia gravis .)

Malathion (b) i s used topica l ly to treat head l i ce and i s never used internal ly (intentional ly). Pyridostigmine (e) i s used ora l ly for maintenancetherapy of myasthenia gravis . Physostigmine (c) i s indicated for treatment of glaucoma (given topica l ly), and i s a lso a va luable parentera l drug fortreating toxici ty of antichol inergic drugs such as atropine. They are a l l chol inesterase inhibi tors . Pra l idoxime (d) i s a “chol inesterase reactivator”and i s used adjunctively (with atropine) in the treatment of poisonings caused by “i rrevers ible” chol inesterase inhibi tors , such as the “nervegases” used as bioweapons and some commercia l insecticides .

71. The answer is c. (Brunton, pp 282-287, 302; Katzung, p 145.) Epi -nephrine, an excel lent agonis t for a l l the adrenergic receptors , i s the drug of choicefor managing the symptoms of an acute, systemic, immediate hypersens i tivi ty reaction to an a l lergen (anaphylactic shock), a l l of which resul t fromthe release or production of various substances (eg, his tamine, kinins ) that contribute to cardiac, vascular, and pulmonary dys function. Death canoccur because of severa l main adverse responses : profound hypotens ion and angioedema (which can be reversed by epinephrine’s α-agonis t/vasoconstrictor effects ); cardiac depress ion (reversed by epinephrine’s β1 actions); and laryngospasm and bronchospasm (reversed by theβ2 effects of epinephrine).

Atropine (a), the prototype muscarinic receptor blocker, wi l l have modest effects to counteract a i rway smooth muscle contraction caused byacetylchol ine, but no effect on the s trong adverse cardiovascular or a i rway effects of his tamine, kinins , and other mediators that play a cri ti ca l rolein anaphylaxis . Diphenhydramine (b) has s trong his tamine H1 and muscarinic receptor blocking actions , but i t, too, cannot block a l l the adverseand potentia l ly letha l cardiovascular and pulmonary effects of anaphylaxis -associated mediators . (Diphenhydramine clearly can a l leviatebothersome urticaria and other cutaneous mani festations of a l lergic or anaphylactic reactions , but those actions are rather trivia l compared withother l i fe-threatening responses that i t cannot beneficia l ly a ffect.) No antihis tamine can be cons idered a l i fe-saving substi tute for epinephrine.Isoproterenol (d), the prototype β1 and β2 agonis t, wi l l counteract a i rway smooth muscle contraction and help restore cardiac function, but becausei t lacks any vasoconstrictor (ie, α-agonis t) activi ty i t wi l l not counteract widespread vasodi lation and the hemodynamic consequences of i t.Norepinephrine (e) has beneficia l cardiac and periphera l vasoconstrictor activi ty, but s ince i t lacks β2-bronchodi lator activi ty i t i s not anacceptable a l ternative to epinephrine. Its bronchodi lator effects are relatively weak, and i t lacks any vasopressor effects that are needed to helprestore blood pressure.

Page 93: Pharmacology pre test self assessment and review (14th ed.)

72. The answer is c. (Brunton, pp 206t, 290; Katzung, pp 141-142, 148, 225.) Dobutamine i s mainly a β1-selective agonis t that causes a pos i tive inotropiceffect and in turn ra ises s troke volume, leading to increased cardiac output. Dobutamine has some pos i tive chronotropic activi ty (a lso β1-mediated) that a lso contributes to a ri se of cardiac output, but the drug’s effects on the heart’s inotropic s tate predominate.

Note: The best way to characterize the receptor-mediated effects of dobutamine i s as a “selective β1 agonis t,” but dobutamine’s actions (in theform ava i lable for therapeutic use) are a bi t more complex. It i s actua l ly a racemic mixture of two i somers : the (+) i somer i s the potent β1 agonis tthat causes the predominant cardiac and hemodynamic effects , but i t a l so has some weak α1 antagonis t (vasodi lator) effects ; the (–) i somer i s anefficacious and potent α1 agonis t (vasoconstrictor). The oppos ing vasoconstrictor effects of one i somer essentia l ly cancel -out the vasodi latoreffects of the other. Regardless , i t would not be correct to characterize the drug’s predominant cardiovascular actions as involving α-agonis t orantagonis t effects (a , b, e, f), and the drug certa inly has no β-blocking actions (d).

73. The answer is b. (Brunton, pp 116, 196-198; Katzung, pp 85-87, 134.) Norepinephrine (and other monoamine neurotransmitters such as dopa-mine) i sremoved from i ts receptors by an “amine pump” located in the neuronal membrane. Reca l l that this reuptake process can be blocked by (amongother drugs) coca ine and tricycl ic antidepressants . (You might ask “what about ‘s timulation’ of presynaptic α2 receptors?” Wel l , that answer mightwork, but i t wasn’t one of the choices here. Moreover, presynaptic α2 receptor activation only inhibi ts release of addi tional NE; i t does not s top theeffects of NE that has a l ready been released.)

Monoamine oxidase (MAO; answer d), and to a lesser degree catechol -O-methyl transferase (COMT; answer c), are enzymes respons ible formetabol ic degradation of NE (the former intraneuronal ly as far as monoaminergic nerves go, the other extraneuronal ). However, they are notimportant in terminating the immediate actions of released NE, which i s a process dependent mainly on neuronal reuptake fol lowing each actionpotentia l and which i s not affected by MAO or COMT inhibi tors .

Answer a , which ci tes metabol ic inactivation of neurotransmitter in the synaptic cleft, i s a good description of what happens to neuronal lyreleased ACh. Answer e, which mentions formation of a “fa lse neurotransmitter”, does not expla in the phys iologic termination of releasednorepinephrine’s actions . That concept, however, has been mentioned as a potentia l mechanism by which monoamine oxidase inhibi tors a l terthe adrenergic responses when MAO inhibi tors are adminis tered.

74. The answer is f. (Brunton, pp 316-317; Katzung, pp 162-165, 691-694.) The tachycardia associated with symptomatic hyperthyroidism reflects , to agreat degree, thyroid hormone-related hyperreactivi ty of β-adrenergic receptors to drugs that have β-agonis t activi ty. Of most concern in thiss i tuation i s hyperreactivi ty of β1 receptors , which accounts for the untoward and potentia l ly dangerous cardiac responses that usual ly occur. Thesecan be managed, symptomatica l ly, wi th a β-blocker. Indeed, during an acute and l i fe-threatening episode of hyperthyroidism (thyrotoxicos is , or“thyroid s torm”) a β-blocker i s not only indicated but a lso may be l i fesaving. Of a l l the conditions l i s ted, this i s the only indication for propranololor vi rtua l ly any other β-blocker; and the only one that i s not l ikely to worsen some aspect of the current cl inica l presentation.

β-Blockers are genera l ly safe and effective for chronic-s table angina (chronic-s table or “s tress -induced” angina), and they are often usedprophylactica l ly in that condition. However they may worsen, and so are genera l ly contra indicated in patients with vasospastic (variant, orPrinzmeta l ) angina (a). In s i tuations of coronary vasospasm, α-mediated vasoconstrictor/spasm-provoking influences tend to be opposed by β2-mediated vasodi lator influences . Block the β2 effects and the constrictor or spasm-favoring α-influences are unmasked, leading to increasedfrequency and severi ty of angina and myocardia l i schemia.

Asthma (b) i s an important and common condition that relatively or absolutely contra indicates the use of a β-blocker, regardless of i ts spectrumof activi ty or adminis tration route. Even very smal l doses of a β-blocker—even a topica l β-blocker that might be used for glaucoma, and even theso-ca l led selective β1-blockers such as atenolol or metoprolol—can prove letha l for some asthmatics . That i s because the a i rways of as thmaticindividuals depend greatly on β2 activation (and other influences) to prevent bronchoconstriction or bronchospasm. Block those effects , andserious or even fata l consequences may arise.

β-Adrenergic blockers lower heart rate and s low A-V nodal conduction veloci ty. Thus , pre-exis ting bradycardia (c) or heart (A-V nodal ) block (e)tends to weigh aga inst, i f not contra indicate, safe use of any β-blocker. A working defini tion of heart block i s , of course, an abnormal ly prolongedP-R interva l , which can be detected eas i ly on an electrocardiogram. If the patient has fi rs t-degree heart block (P-R interva ls are prolonged, but a l lsupraventricular electrica l activi ty passes through the A-V node to activate the ventricles ), a β-blocker may cause second-degree heart block (somesupraventricular impulses are not conducted through the A-V node). If the patient has second-degree heart block, a β-blocker may cause complete(thi rd degree) block, which can have di re consequences because the ventricular myocardium loses a l l control by impulses pass ing through the A-Vnode, and so ventricular activi ty i s l ikely to fa l l to dangerous levels .

β-Blockers may pose s igni ficant problems for some patients with severe, poorly control led diabetes mel l i tus (d). These drugs can, for example,prevent tachycardia that i s one s ignal to the patient that blood glucose levels are too low, and they can delay the recovery of blood glucose levelsfol lowing an episode of hypoglycemia. However, for many patients with mi ld and wel l -control led diabetes (especia l ly type 2), in such conditionsas mi ld-to-moderate heart fa i lure (and others ), the judicious use of a β-blocker probably provides more benefi t than harm.

75. The answer is c. (Brunton, pp 234-235; Katzung, pp 116-124, 1034.) These are among the class ic s igns and symptoms of atropine poisoning, which i sa lso more genera l ly known as the antimuscarinic or antichol inergic syndrome. The antimuscarinic drug-poisoned patient often can be describedas :

• red as a beet (characteris tic facia l flushing; a so-ca l led “atropine flush”);• dry as a bone (no exocrine gland secretions , no feca l or urinary output because bowel and bladder moti l i ty are inhibi ted);• hot as a furnace (profound fever; a CNS “problem” compounded by a lack of body heat loss normal ly afforded by sweating);• bl ind as a bat (para lys is of accommodation and di lated pupi l s do not respond to even very bright l ight);• mad as a hatter (as in the Mad Hatter from Lewis Carrol l ’s Alice in Wonderland: CNS problems, including del i rium and ha l lucinations).

You may never see true atropine poisoning. As you know, that prototype antimuscarinic drug i s not used cl inica l ly a l l that often, except in someparticular specia l ties or s i tuations . However, you should rea l i ze that many common groups of drugs (see Question 88), some of which areavai lable over-the-counter (speci fica l ly, diphenhydramine), exert s trong antimuscarinic effects . The s igns and symptoms of “atropine poisoning”are an important component of thei r overdose syndromes, and you should be able to recognize them.

76. The answer is a. (Brunton, pp 310-316; Katzung, pp 151-152.) The β-blockers with intrins ic sympathomimetic activi ty (ISA) are partia l agonis ts—theyact s imultaneous ly as both β-agonis ts and competi tive β-blockers . How can that be? At usual doses , and in the presence of low (eg, resting)sympathetic tone, they act as weak agonis ts for β-adrenergic receptors . Under these conditions , then, they may actua l ly but s l ightly increase suchβ-mediated responses as heart rate. However, whi le these drugs are occupying the β-adrenergic receptors , they s imultaneous ly block (antagonize)

Page 94: Pharmacology pre test self assessment and review (14th ed.)

the effects of more efficacious (“s tronger”) β agonis ts , eg, epinephrine and norepinephrine, or such exogenous agents (drugs) as i soproterenol .Thus , a l though they weakly increase resting heart rate, when catecholamine levels are high (as with s tress , or exercise), such β-mediatedresponses as acceleration of heart rate and contracti l i ty are less intense than they would be had these drugs with ISA not been present. SeeQuestion 27 (in the Genera l Principles chapter) for more information, because i t was based on the effects of a β-blocker with ISA/partia l agonis tactivi ty.

Acebutolol and pindolol—indeed, a l l drugs class i fied as β-blockers—have no abi l i ty to cause release of norepinephrine (from adrenergicnerves) or epinephrine (from the adrenal medul la ), and so answer b i s incorrect. Likewise, these drugs do not induce (s timulate; nor inhibi t)catecholamine synthes is (c); do not potentiate the actions of norepinephrine on α- (or on β-) adrenergic receptors (d); and cause negative inotropicand chronotropic effects (not pos i tive; answer e).

77. The answer is b. (Brunton, pp 299-300, 303; Katzung, pp 136f, 142, 703t, 1032-1034.) Methylphenidate i s pharmacologica l ly s imi lar to amphetamine. Inthe context of ADD/ADHD therapy i t works as a CNS s timulant, with more pronounced effects on menta l than on motor activi ties . It does so byreleas ing neuronal norepinephrine and dopamine via a nonexocytotic (not dependent on action potentia ls ) mechanism. It i s a l so used to treatnarcolepsy. Dobutamine (a) i s a β1-adrenergic agonis t. Pancuronium (c) i s a nondepolarizing skeleta l neuromuscular blocker (nicotinic receptorcompeti tive antagonis t). Prazos in (d) i s a competi tive and selective α1-adrenergic blocker. Scopolamine (e) i s an antimuscarinic (atropine-l ike)drug with greater efficacy for caus ing sedation and antimotion s ickness effects . Terbuta l ine (f) i s an agonis t predominantly selective for β2-adrenergic receptors at “low” doses , but i t can exert nonselective (i soproterenol -l ike) β1- and β2-agonis t effects at high doses , including dosesthat may commonly be given therapeutica l ly.

78. The answer is e. (Brunton, pp 198, 300, 323t; Katzung, pp 136f, 141-142.) Ephedrine (and pseudoephedrine) are class i fied as mixed-actingsympathomimetics , meaning that thei r overa l l actions are a mixture or combination of two mechanisms. They mainly release intraneuronalnorepinephrine, and so the predominant effects you see in the periphery are qui te s imi lar to those of NE i tsel f (α- and β1 activation) because theyare caused by NE. They a lso di rectly, but relatively very weakly, activate a l l the adrenergic receptors—alphas and the betas . Ephedrine andpseudoephedrine a lso exert s imi lar effects in the CNS, leading to such effects as greater a lertness (and di ffi cul ty fa l l ing as leep) and appeti te-suppress ion (anorexigenic effect). The intens i ty of these effects i s greater than those of typica l di rect-acting adrenergic agonis ts (EPI, NE, etc) butmuch weaker than those of indi rect-acting sympathomimetics such as the amphetamines .

Ephedrine’s actions are not at a l l selective for α-receptors on vascular smooth muscle, or anywhere else. So, answer a i s incorrect. The mixed-acting (and indirect-acting) sympathomimetics have no α- (or β- or chol inergic) blocking activi ty (b); do not enhance NE synthes is (c); and do notinhibi t NE’s metabol i sm in any other way (d).

Note: Some texts and lecturers “downplay” discuss ions of ephedrine. After a l l , i t was commonly abused as a genera l CNS s timulant (to keep oneawake) and as an anorexigenic (appeti te suppress ing/weight loss ) drug. Such uses have largely been reduced by FDA mandate. It i s a l so anoutmoded drug for as thma, because i t has bronchodi lator activi ty, but that i s very weak. Al though the drug’s vasoconstrictor effects are a lsorelatively weak (compared with, say, epinephrine, norepinephrine, or phenylephrine), i t i s “just enough” for many cl inica l s i tuations , and the drugis used precisely for that purpose qui te frequently intraoperatively, as shown in this patient’s operative drug adminis tration chart.

79. The answer is e. (Brunton, pp 198, 300; Katzung, pp 136f, 141-142.) Ephedrine’s mechanisms of action were addressed in a previous answer. There Inoted that ephedrine’s di rect vasoconstrictor effects are “weak,” and they are not at a l l l imited to α-adrenergic receptors (vascular or elsewhere).Nonetheless , the weak di rect vasoconstrictor effect combined with the predominant NE-releas ing action leads to cl inica l ly useful vasopressoreffects . The preanesthetic and induction drugs given to this patient (and many others ) are l ikely to produce some cardiodepressant effects(centra l , cardiac, and vascular), one mani festation of which i s an excess ive fa l l of blood pressure. Restoring blood pressure i s the main purposefor which the ephedrine i s given here.

Ephedrine does cause CNS-stimulating effects (b), but i t i s genera l ly weak and certa inly insufficient to overcome (let a lone affect) the CNSdepress ion caused by severa l other drugs the patient received (midazolam, propofol , and morphine). Moreover, the patient i s supposed to beanesthetized; why would we want to give a drug to counteract this so early into surgery? Ephedrine does not inhibi t SA and AV nodalelectrophys iology (c), nor heart rate (d) or contracti l i ty. If anything, through i ts indi rect and NE-releas ing effects i t would be much more l ikely tostimulate the heart.

Many years ago ephedrine was used as an ora l bronchodi lator, mainly for patients with asthma. Having no ora l or inhaled “selective” β2

agonis ts as we have today, i t was bas ica l ly the “only game in town” for chronic management of a i rway smooth muscle constriction. Itsbronchodi lator effects were, at best, weak. The di rect actions on β2 receptors are intrins ica l ly weak; and s ince NE does not activate β2 receptors , NErelease that occurs elsewhere does nothing in the way of caus ing bronchodi lation. Aga in, there was nothing in the patient’s his tory ofbronchoconstriction, nor did she receive any drugs that were l ikely to cause bronchoconstriction. (Yes , morphine can release his tamine, which cancause bronchoconstriction, but this patient was described as otherwise heal thy; and she was not a smoker, which might have put her at s l ightlygreater ri sk of bronchoconstriction.)

80. The answer is b. (Brunton, p 408; Katzung, pp 84-87, 136f, 148, 355.) In order for ephedrine to exert i ts predominant NE-releas ing effects , i t must betaken up from the synapse, into the adrenergic nerve “ending.” Coca ine and the tricycl ic antidepressants inhibi t ephedrine transport, jus t as theyinhibi t reuptake of NE that has been released from adrenergic nerves in response to an action potentia l (or in response to other NE-releas ingdrugs).

Atenolol and metoprolol (a ) relatively selectively block β1 receptors (at usual therapeutic doses ; at high doses they a lso block β2 receptors—important i f the patient has asthma, for example). They have no α-blocking activi ty at a l l . Thus , they don’t weaken all the effects of ephedrine. (If Ihad l i s ted labeta lol or carvedi lol , which block a l l the α- and a l l the β-receptors , I ’d give you your “two points” i f you picked that answer.)Hexamethonium and trimethaphan (c) are autonomic (PNS and SNS) gangl ionic (NN receptor) blocking drugs . They have no effect on ephedrine’sdi rect receptor-mediated actions , nor on ephedrine’s NE-releas ing effects , both of which occur dis ta l to the gangl ia . (They would, of course, reduceor el iminate baroreceptor reflex responses that might ari se i f ephedrine suddenly and markedly ra ised blood pressure.) Pargyl ine (d), arguablythe prototypic nonselective monoamine oxidase inhibi tor, would intens i fy the responses to ephedrine or other sympathomimetics that work inpart or in whole by releas ing intraneuronal NE. Remember that MAO inhibi tors cause a gradual but eventual ly dramatic ri se in intraneuronal NElevels by inhibi ting NE degradation by the enzyme; and a l though that NE i s not released by a normal action potentia l , catecholamine-releas ingsympathomimetics (ephedrine, pseudoephedrine, amphetamines , and even dietary tyramine) certa inly, suddenly, and mass ively do so.Propranolol (e), of course, blocks a l l β-adrenergic receptors , but has no α-blocking activi ty and so does not affect any α-mediated responses toephedrine.

81. The answer is d. (Brunton, pp 241-242, 268; Katzung, pp 106-109.). In the vast majori ty of s i tuations in which we give a patient a nondepolarizing

Page 95: Pharmacology pre test self assessment and review (14th ed.)

(curare-l ike) neuromuscular blocking drug (vecuronium in this case), we wi l l a l so be giving the patient two other drugs . One, the focus of thisquestion, i s an acetylchol inesterase (AChE) inhibi tor (neostigmine, l i s ted here, i s the most common one). It i s given to reverse the competi tiveskeleta l neuromuscular blockade: cause a bui ld-up of ACh at the synapse by inhibi ting i ts enzymatic hydrolys is by AChE), and the ACh can nowovercome the blockade of the NM receptors on skeleta l muscle.

As expla ined more ful ly in other questions , the AChE inhibi tor wi l l cause bui ld-up of ACh at a l l periphera l chol inergic synapses by inhibi tingACh’s metabol ic inactivation. This leads to a variety of usual ly unwanted muscarinic effects . [Neostigmine does not enter the CNS, as doesphysostigmine (which i s not at a l l appropriate for this indication), and so answer c i s incorrect.] If the neostigmine (a lone) does anything to bloodpressure i t wi l l cause a fa l l (not a ri se, a ) due to the drug’s indi rect (via increased ACh) negative chronotropic effect on the heart (thus , b i s a l soincorrect). The secondary ri se of ACh wi l l a l so s timulate the bladder’s detrusor and trigone, and relax the sphincter, and so there wi l l be noreduced ri sk or severi ty of urinary incontinence (e).

82. The answer is b. (Brunton, pp 265-268; Katzung, pp 106-109.) The patient wi l l receive neostigmine, and as expla ined in the previous question i t wi l lact at a l l chol inergic synapses in the periphera l nervous system—the NM receptors on skeleta l muscle that we want to reactivate, and themuscarinic receptors that we don’t. If we didn’t block the muscarinic responses fi rs t, the patient i s l ikely to experience ACh-mediated suppress ionof heart rate and nodal automatici ty and conduction; s timulation of various gut and urinary tract muscles and relaxation of sphincters ;bronchoconstriction; and increases in the secretory activi ty of a host of exocrine glands (sa l ivary, lacrimal , mucous , gastric acid/parieta l cel l ). As aresul t, and routinely, we give an antimuscarinic drug right before giving the s tigmine, and that i s what you need to know (and should have beenable to deduce, even though you might not be fami l iar at a l l wi th glycopyrrolate.)

Note: Glycopyrrolate i s yet another drug that seems to get relegated to miscel laneous or unimportant s tatus in texts and lectures (in which i tmight not be mentioned at a l l ). However, i t i s used routinely in many hospi ta ls as the go-to antimuscarinic in s i tuations such as described here.Why not atropine, which you obvious ly know about? The effects of atropine tend to las t much longer than what i s needed here. Glycopyrrolate’sonset and duration of action are shorter than atropine’s—indeed, a rather close match to the pharmacokinetics of neostigmine, and so i t i s anexcel lent choice for this use.

83. The answer is e. (Brunton, pp 263-268; Katzung, pp 466-467, 475-476, 481.) At usual therapeutic (ie, nontoxic) doses succinylchol ine (SuCh) causesrapid, brief (with a s ingle dose), and noncompeti tive depolarizing blockade of NM receptors on skeleta l muscle. Even i f skeleta l muscle para lys isi s mainta ined longer by continuous infus ion of the drug (i t i s sometimes done), skeleta l muscle function wi l l return spontaneous ly—due to rapidmetabol ic inactivation of the drug by chol inesterases—without the need to adminis ter any “reversa l” drug. Indeed, for a l l practica l purposes thereis no cl inica l ly useful SuCh reversa l drug (but see the note below). Succinylchol ine has no effect on muscarinic receptors , and so atropine (a) orany other antimuscarinic drug wi l l do nothing in terms of skeleta l muscle function. Bethanechol (b) activates only muscarinic receptors (at a l l butextraordinari ly high doses). Stigmines (neostigmine, physostigmine, or any other AChE inhibi tors ) wi l l theoretica l ly add to skeleta l musclepara lys is , not reverse i t. (I say theoretica l ly because i f skeleta l muscle i s now para lyzed by SuCh, i t’s not l ikely that making more ACh ava i lable atthe skeleta l neuromuscular junction wi l l intens i fy an a l ready “maximum” response.)

Note: You may have learned about so-ca l led Phase-2 neuromuscular blockade caused by SuCh, and that i t can be reversed with such drugs asatropine. This “Phase 2 block” i s not a part of the “normal pharmacology” of SuCh unless a frank overdose occurs and toxici ty develops . Phase 2block can be partia l ly reversed by adminis tration of an antichol inergic drug (eg, glycopyrrolate or atropine i tsel f), but adding yet another drug to adrug-overdosed patient i sn’t the way to go. If Phase 2 block from succinylchol ine does occur a l l one needs to do i s keep the patient on mechanica lventi lation for as long as necessary, s imply wait for the drug to be el iminated (yes , i t may take a whi le), and be as sure as poss ible beforeextubating the patient and/or s topping mechanica l venti lation that spontaneous respiration i s adequate and not l ikely to be compromised aga in.

84. The answer is a. (Brunton, pp 227-234; Katzung, pp 99-104.) These are expected s ide effects (not at a l l idiosyncratic, b) from bethanechol , amuscarinic agonis t (parasympathomimetic drug). They occur even though the effects of the drug—the ones for which i t i s given—are“predominately” on the bladder musculature: contraction of the detrusor and relaxation of the sphincter to enable (i f not cause) urination. Thes ide effects noted in the question nonetheless are common and dose- (and patient-) dependent, and not at a l l idiosyncratic reactions . Moreover,a l though we could inhibi t those s ide effects with an atropine-l ike (antimuscarinic) drug, doing so would s imultaneous ly inhibi t the drug’sintended effects on the bladder. Bethanechol has no sympathetic (or parasympathetic) gangl ionic activating (c) or blocking actions at usual doses .

The potentia l cardiac/cardiovascular effects of bethanechol used at usual doses are not so s imple to expla in, but certa inly would not involvebaroreceptor reflex-mediated suppress ion of cardiac rate (d). In order for the baroreceptors to reflexly suppress heart rate, blood pressure wouldhave to go up qui te a bi t, and qui te quickly. Bethanechol i s a vasodi lator, and so would lower BP (quickly, i f injected too fast). The expected baro-receptor response to that would be increased sympathetic “outflow” from the CNS to the periphery, and that in turn would favor reflex tachycardia .However, bethanechol s imultaneous ly wi l l di rectly suppress spontaneous depolarization (Phase 4 of the action potentia l ) of the S-A node,attempting to s low heart rate via di rect muscarinic receptor agonis t activi ty. So, what happens to heart rate? So the chronotropic effects depend onwhether the di rect bradycardic effect “wins out” or the reflex tachycardic influences do.

If the patient had undiagnosed asthma (e; the presence of as thma would contra indicate use of the drug), the pulmonary response would bemuch more s igni ficant than “some wheezing.” Potentia l ly fata l broncho-spasm could occur because the a i rway smooth muscles of as thmaticindividuals are exquis i tely (hyper)sens i tive to muscarinic agonis ts , wi th even smal l doses of those drugs capable of caus ing a letha l outcome.

85. The answer is c. (Brunton, pp 178-180t, 290; Katzung, pp 90t, 131-132, 136-140, 145-147.) Ri todrine may no longer be used for suppress ing prematureuterine contractions (a tocolytic effect), but the description of this drug’s actions , I think, helps you work through your understanding of bas icautonomic function and drugs . Ri todrine i s a relatively selective β2-adrenergic agonis t that relaxes uterine smooth muscle. Note the word“relatively;” the preferentia l β2 effects (vs β1) are not absolute, and they are dose-dependent. Yes , ri todrine, the uterine-relaxing drug i s class i fiedmechanis tica l ly the same as , for example, a lbuterol , sa lmeterol , or terbuta l ine, and severa l other drugs typica l ly used as adrenergicbronchodi lators for as thma. Wi l l , or can, ri todrine cause bronchodi lation? Absolutely, even at the relatively low doses used for uterine-relaxingeffects ! And when blood levels become sufficiently high (and perhaps even s ti l l in the therapeutic range for uterine effects ) the preferentia lactivation of β2 receptors i s los t, and the drug behaves just l ike i soproterenol , the prototype β1/β2 agonis t, to cause not only bronchodi lation buta lso cardiac s timulation, enhanced renin secretion, hyperglycemia, and more. This dose-dependent loss of β2-adrenergic agonis t “selectivi ty” i s aproperty of a l l the drugs class i fied, qui te imprecisely, as selective β2 agonis ts or (even less precisely) as selective adrenergic bronchodi lators .

86. The answer is b. (Brunton, pp 154-158, 172t, 220-226; Katzung, p 460.) Morphine i s one of severa l drugs that can cause a “di rect” degranulating effecton mast cel l s , caus ing the release of not only his tamine but a lso of many other mast cel l -derived mediators . The his tamine-releas ing effect i s notcl inica l ly s igni ficant for patients who do not have asthma, but for many who do the bronchoconstriction can be intense and problematic. You’l lprobably see the mani festations of this , even i f the response i s “mi ld.” They typica l ly appear in the form of cutaneous flushing (largely his tamine-

Page 96: Pharmacology pre test self assessment and review (14th ed.)

mediated), particularly in the neck and face regions .Atropine (a) causes bronchodi lation by blocking muscarinic receptors on a i rway smooth muscle cel l s . Pancuronium (d) does not cause

s igni ficant his tamine release, nor do i ts related nondepolarizing skeleta l neuromuscular blockers (eg, vecuronium). (D-tubocurarine, which usedto be the prototype of this class of neuromuscular blockers , did cause appreciable his tamine release, but i t i s no longer used.) Neostigmine (c)can cause s igni ficant bronchoconstriction in asthmatics , but that occurs because the drug inhibi ts metabol ic inactivation (by AChE) of acetylchol ine—another bronchoconstrictor agonis t to which the a i rways of as thmatics are exquis i tely sens i tive. Propranolol (e) can provoke severe andsometimes fata l a i rway smooth muscle contraction in asthmatics , but that i s due to blockade of epinephrine’s agonis t (bronchodi lator) actions onβ2 receptors . It does not involve his tamine release from mast cel l s .

87. The answer is e. (Brunton, pp 142-145, 233-235; Katzung, pp 122-123, 174.) Answering this question requires your knowledge of two things : howtrimethaphan i s class i fied (what i t does); and which branch of the autonomic nervous system, parasympathetic or sympathetic, exerts“predominant resting tone” over various s tructures and their functions . Trimethaphan i s an autonomic gangl ionic blocking drug, preventingactivation of NN receptors found on a l l postgangl ionic nerves (and on the adrenal/suprarenal medul la ). Upon giving this drug we essentia l lydenervate dis ta l s tructures by a pharmacologic means . (If you have not speci fica l ly learned or read about trimethaphan, perhaps you know aboutother and very old gangl ionic blocking drugs such as hexamethonium. It’s OK to cons ider these gangl ionic blockers equiva lent for the purpose ofanswering this question.) Now, we observe how things change.

For a l l the s tructures and functions l i s ted (and severa l others that weren’t), with the exception of controlling vascular smooth muscle and peripheralresistance (vasomotor tone), i t i s the parasympathetic nervous system that exerts the predominant resting tone. In contrast, control of vascularsmooth muscle tone (and, therefore, of periphera l res is tance and blood pressure) i s primari ly regulated by the sympathetic nervous system. Blocka l l gangl ionic transmiss ion and vasodi lation wi l l occur as predominant SNS influences on the vessels are removed.

As far as the other s tructures go—al l of which, I sa id, have predominant PNS tone at rest—you would observe a ri se of heart rate (notbradycardia , a ); decreases of bladder and gut tone (eg, reduced tone of the bladder detrusor and longi tudina l muscles of the gut), not increases(b); contraction of sphincters in the GI and urinary tracts ; mydrias is ; and reduced sa l ivary secretions (xerostomia), not increases (c). Eccrine sweatgland secretions , which are mainly under sympathetic-chol inergic influences , would decrease too. In terms of pupi l s i ze, gangl ionic blockadewould lead to mydrias is , not mios is (d).

Fina l ly, rea l i ze that in the presence of an autonomic gangl ionic blocking drug no autonomic reflexes (eg, baroreceptor reflexes ; pupi l laryconstriction in response to bright l ight) can occur. Al though the afferent pathways that are important in el ici ting those reflexes are unimpaired,responses such as changes in heart rate or pupi l s i ze depend on intact efferent pathways , and gangl ionic blockade interrupts those pathways andthe reflex responses that otherwise might occur.

88. The answer is a. (Brunton, pp 918-926, 1313-1314; Katzung, pp 115-124, 278f, 292, 518, 525-528, 534-536.) These drugs a l l possess antimuscarinic(atropine-l ike) actions that often are sufficiently s trong to cause a l l the s ide effects and adverse responses associated with atropine i tsel f.Likewise, i t i s prudent to assume they share a l l the contra indications and precautions associated with atropine and that managing severeoverdoses of those drugs or drug groups wi l l resemble (and need to be treated) in qui te the same way as those of the prototype antimuscarinic.That would include the potentia l need to adminis ter physostigmine, the acetylchol inesterase inhibi tor that plays an important role in managingatropine poisoning.

Note that of a l l the drugs with atropine-l ike actions , diphenhydramine (and, to a somewhat lesser extent the other fi rs t-generationantihis tamines) are ava i lable over-the-counter. Al though you may do your best to avoid prescribing drugs with atropine-l ike actions for patientswho should not receive them, atropine-l ike problems can arise in patients who sel f-prescribe these nonprescription medications .

Clearly, the effects of bethanechol (muscarinic agonis t) and neostigmine (acetylchol inesterase inhibi tor) are the oppos i te of those you’d expectto see with atropine (muscarinic receptor blocker). You might argue that i soproterenol (β1 and β2 agonis t) causes some effects (eg, tachycardia)that you would expect with atropine. True. However, despi te any s imi lari ties in appearance, the mechanism is qui te di fferent, the spectrum of a l leffects caused by the drug i s di fferent, and certa inly most contra indications and toxic mani festations are very di fferent. Propranolol (prototypenonselective β-blocker) i s radica l ly di fferent from atropine in nearly every important way.

89. The answer is b. (Brunton, pp 86t, 250, 918-924, 1313-1314, 1851-1816; Katzung, pp 122-125, 1034.) The major s igns and symptoms of diphenhydraminetoxici ty are attributable to i ts s trong antimuscarinic (atropine-l ike) effects in both the periphera l and centra l nervous systems. Neostigmine—unl ike the preferred antidote, physostigmine—is a quaternary compound and does not enter the CNS. Thus , del i rium, ha l lucinations , and otherCNS mani festations of diphenhydramine poisoning wi l l pers is t. Physostigmine i s the preferred antidote speci fica l ly because i t i s not a quaternary(ionized) at phys iologic pH, and wi l l readi ly cause des i red antidota l effects in the CNS as wel l as in the periphery.

Bronchoconstriction and wheezing (a) are not expected outcomes of diphenhydramine toxici ty; the antimuscarinic effects of the drug wi l l causebronchodi lation by blocking the bronchoconstrictor influences of endogenous ACh. Likewise, toxic (or even therapeutic) blood levels ofdiphenhydramine wi l l inhibi t—not s timulate—exocrine gland secretions (c). Excesses of diphenhydramine (or atropine i tsel f) wi l l have nos igni ficant effects on skeleta l muscle activation (d) because these drugs only affect muscarinic receptors ; skeleta l muscle activation by AChinvolves nicotinic receptors . Overdoses of diphenhydramine or any other drugs with s trong periphera l muscarinic receptor-blocking effectscerta inly wi l l cause tachycardia by blocking muscarinic receptor activation of the s ino-atria l node. That effect can be reversed byacetylchol inesterase inhibi tors , even those that work only in the periphera l nervous systems, such as neostigmine.

You won’t encounter too many patients overdosed on atropine i tsel f, but you’l l see many people suffering toxici ty from older antihis tamines(eg, diphenhydramine), older (tricycl ic or tetracycl ic) antidepressants (eg, imipramine), some of the centra l ly acting antimuscarinics that are usedfor parkinsonism (eg, benztropine and trihexyphenidyl ); scopolamine (used for motion s ickness ), and most of the phenothiazine antipsychotics(eg, chlorpromazine). Owing to the often s trong antimuscarinic s ide effects of these drugs , treating overdoses of any of them probably wi l l involvemanaging what amounts to “atropine poisoning”—and many other problems too.

(You may not know this , but perhaps i t wi l l have some meaning or help jog your memory: the trade name for physostigmine i s Anti l i rium. Reca l lthat one of the ha l lmark CNS s igns of atropine/antimuscarinic poisoning i s del i rium. Hence, “anti-lirium” makes sense.)

90. The answer is d. (Brunton, pp 206t, 290, 301, 321t, 804-805; Katzung, pp 141-142, 144, 148, 225.) Dobutamine behaves , for a l l practica l purposes , as aselective β1 agonis t. Norepinephrine i s a β1 agonis t that a lso activates α-adrenergic receptors effectively. However, when i t i s adminis tered withphentolamine (prototype α-blocker) i ts spectrum of activi ty i s , qual i tatively, identica l to that of dobutamine.

High doses of dopamine (a) cause pos i tive inotropic and chronotropic effects , but a lso release neuronal norepinephrine and probably activateα-adrenergic receptors di rectly (caus ing unwanted vasoconstriction). These vasoconstrictor effects would negate vasodi lator effects due tostimulation of dopamine D1 receptors found in some arterioles ; and of D2 receptors found on some gangl ia , and in the cardiovascular controlcenter of the CNS.

Page 97: Pharmacology pre test self assessment and review (14th ed.)

Ephedrine (b) weakly activates a l l adrenergic receptors and a lso leads to norepinephrine release. Overa l l , i ts effects are qui te s imi lar to thoseproduced by norepinephrine i tsel f. Regardless , the intens i ty periphera l adrenergic responses for a usual therapeutic dose of ephedrine usual lyare less intense than those norepinephrine. And i f one adminis ters ephedrine with propranolol (c), the prototypic nonselective (β1 and β2)blocker, ephedrine’s remaining actions amount to selective α-adrenergic activation (ie, phenylephrine-l ike)—not at a l l l i ke dobutamine, and notat a l l what we want in this s i tuation.

Phenylephrine (α-agonis t) plus atropine (muscarinic antagonis t) (e) causes effects that in no way resemble those of dobutamine or thenorepinephrine-phentolamine combination. From a cardiovascular perspective, this combination would give us a ri se of blood pressure due toperiphera l vasoconstriction. The vasoconstriction would el ici t a baroreceptor reflex tantamount to withdrawing sympathetic tone and increas ingoppos ing parasympathetic tone. However, the presence of atropine would blunt parasympathetic-mediated cardiac s lowing. The overa l l andcombined effect on heart rate of reduced sympathetic tone to the SA node, and concomitant blockade of ACh-mediated cardiac s lowing (a lso aneffect on the SA node) can vary. However, i t would be reasonable to predict a s l ight increase of rate over predrug rates . Nonetheless , there wouldnot be a pos i tive inotropic response, which i s what would occur with ei ther dobutamine or a norepinephrine-phentolamine combination.

91. The answer is a. (Brunton, pp 178-179; Katzung, pp 88-90, 138-145.) With only one major exception (eccrine sweat glands), the neurotransmitterreleased by postgangl ionic sympathetic nerves (adrenergic nerves) to activate thei r targets i s norepinephrine (NE). NE i s a “good” agonis t for onlyα- and β1-adrenergic receptors , but not β2. In contrast, epinephrine (EPI; from the adrenal medul la ) i s a very efficacious agonis t for both classes ofadrenergic receptors and their main subtypes , including β2. Of the responses l i s ted in the question, only a i rway smooth-muscle relaxation i s aprocess that involves (depends on) autonomic activation of β2-adrenergic receptors . There i s no innervation of these muscles , and so no NE to bereleased. Even i f NE were injected, i ts lack of β2-agonis t activi ty would render i t ineffective as a bronchodi lator.

A-V nodal automatici ty and conduction (b) are increased, in terms of sympathetic influences , by β1 receptors , which can be activated by ei ther orboth NE and EPI. The same appl ies to the coronary vasculature, which constricts in response to α-adrenergic receptor activation by ei ther or both NEor EPI. (If the answer choice were coronary vasodi lation involving sympathetic influences , then only EPI would be a correct answer, s incesympathetic coronary vasodi lation involves β2 receptors .) Ei ther NE or EPI wi l l cause mydrias is (d), which i s an α-mediated response. Fina l ly,sympathetica l ly influenced renin release from the juxtaglomerular apparatus involves β1 activation, which can be caused by ei ther NE or EPI.

Note: You may have learned something l ike “i f i t’s a sympathetic response involving the heart, the receptor involved i s β1.” Wel l , that’s usual lytrue, but now you see that sympathetic mediated coronary vascular smooth muscle relaxation—a response that occurs “in the heart”—involves β2

receptors . (α-Adrenergic receptor activation in the coronaries causes vasoconstriction, just as occurs in the periphery.) You may have a lso learned“i f a β1 receptor i s involved, the response i s occurring (only) in the heart.” Aga in, the s tatement i s usual ly true, but juxtaglomerular cel l reninrelease i s obvious ly occurring outs ide the heart, but i t i s a β1-mediated response nonetheless .

92. The answer is e. (Brunton, pp 200-201; Katzung, pp 84-86.) In short, the answer i s “reuptake.” A norepinephrine transporter plays the main role interminating the activi ty of neuronal ly (phys iologica l ly) released NE. This transporter i s blocked by coca ine and tricycl ic antidepressants (eg,imipra-mine). Di ffus ion of released NE from the synaptic space, leading to metabol i sm by such enzymes as COMT (d), i s overa l l a minor process interms of the phys iologic inactivation of the effects of NE. Metabol ic inactivation by MAO (c) requires reuptake of the catecholamines into theneuron, s ince the MAO that degrades catecholamines i s located intraneuronal ly (and intramitochondria l ly).

93. The answer is b. (Brunton, pp 289-290; Katzung, pp 90t, 108, 118, 136-145.) To me, the tip-off that helps get to the correct answer, i soproterenol , i s tofocus (no pun intended) on the fact that the unknown drug did not change the s i ze of the pupi l of the eye. Of a l l the main autonomic receptortypes , adrenergic and chol inergic, only the β-receptors play no role in regulating the s i ze of the pupi l . That narrows things down to i soproterenol orpropranolol . Propranolol might lower BP (particularly in a hypertens ive patient), but i t would not ra ise heart rate or di late the bronchi . Onlyisoproterenol fi ts the bi l l .

And the other answers? Atropine (a) might ra ise heart rate, i t i s not l ikely to lower BP, and i t wi l l relax a i rway smooth muscle cel l s and increasebronchiolar diameter, but i t would cause mydrias is , which we don’t see in the resul ts presented in the diagram. Neostigmine (c) would s low heartrate, perhaps lower BP (probably not), and constrict the a i rways and pupi l s . Phenylephrine (d) would ra ise BP, and i f the BP ri se i s sufficiently highand quick, reflexly lower heart rate (baroreceptors ). It would do nothing to a i rway diameter, but would cause mydrias is .

94. The answer is c. (Brunton, pp 206t, 212, 305-307; Katzung, pp 151-156.) Tamsulos in i s a selective α1-adrenergic blocker, much l ike prazos in (note the“-os in” at the end of the generic names—that’s a certa in tip-off to the drug’s genera l class ), and presumably i ts a ffini ty i s greater for α-receptorson smooth muscles in the prostate (hence, i ts use for BPH, due to relaxation of smooth muscles there) than for those in the periphera lvasculature. Nonetheless , i ts pharmacologic profi le i s most s imi lar to that of prazos in, which can be cons idered the prototypic α1-selectiveadrenergic blocker that i s mainly used to treat hypertens ion because i t competi tively blocks vasoconstriction caused by α-agonis ts such asepinephrine and norepinephrine. (Remember: Drugs with a generic name that ends in “-os in” or “-zos in” are selective α1-adrenergic blockers : eg,tamsulos in, prazos in, terazos in, doxazos in.)

So, of the answers l i s ted above, orthostatic hypotens ion i s the most l ikely s ide effect. Among the many classes of adrenergic drugs , bradycardia(a) would most l ikely be caused by a β-blocker (or lesser prescribed drugs such as reserpine, a catecholamine depletor), or a muscarinic agonis t orchol inesterase inhibi tor, and certa inly not by a drug that has no di rect cardiac effects and i s more l ikely to el ici t reflex cardiac s timulationsecondary to reduced blood pressure. There are no known interactions between α-blockers and atorvastatin or related drugs , whether involvingskeleta l muscle pathology and dys function (b) or other adverse responses that a s tatin might cause. Photophobia (d) i s an exceedingly unl ikelyproblem: i t i s usual ly caused by drugs that cause mydrias is (eg, α-adrenergic agonis ts or antimuscarinics ), and i f anything tamsulos in i s l ikely toprevent the mydrias is that i s a common cause of photophobia . Exacerbations of emphysema (e), whether due to bronchoconstriction or othercauses , i s unl ikely too. From an autonomic perspective, epinephrine i s the main bronchodi lator substance (via β2 activation), ACh i s the mainautonomic bronchoconstrictor (via muscarinic activation). Tamsulos in affects nei ther.

95. The answer is c. (Brunton, pp 226-227, 234-237; Katzung, p 118.) There are a couple of ways you could have arrived at this answer, even i f you did notlearn speci fica l ly about homatropine being a shorter-acting, semisynthetic derivative of atropine, the prototypic antimuscarinic drug. For onething, the drug’s generic name, homatropine, should immediately lead you to thinking about an antimuscarinic drug.

Now, cons ider major autonomic innervation of the eye. The sympathetic nervous system, via α-adrenergic receptors , controls mainly the s i ze ofthe pupi l of the eye; there i s very l i ttle influence on the ci l iary muscle or other s tructures that control accommodation. (β-Adrenergic influencesplay no role in control l ing pupi l s i ze or tone of the ci l iary muscle.) The parasympathetic branch of the ANS, and i ts influences on muscarinicreceptors , can a l ter both pupi l s i ze and accommodation—precisely what we have here. So, we can rule out any “sympathetic” drug, agonis t orantagonis t, as a correct answer. Thus , we reject epinephrine (b), an α- and β-agonis t that would cause mydrias is ; i soproterenol (d), the β1/2 agonis t

Page 98: Pharmacology pre test self assessment and review (14th ed.)

and propranolol (f), the β1/2 blocker, that affect nei ther pupi l s i ze nor accommodation. We reject ACh (a); this muscarinic agonis t would causemios is and faci l i tates accommodation. The same appl ies to pi locarpine (e). This leaves us with homatropine (c). So you could arrive at this correctanswer ei ther by the process of el imination, or by making an educated guess (correctly, of course) that homatropine i s an atropine-l ike(antimuscarinic) drug.

96. The answer is d. (Brunton, pp 178-179, 214; Katzung, pp 102-103, 331-337.) ACh, in vivo or in vi tro, causes i ts vasodi lator response via an action on theendothel ium, where the muscarinic receptors are located. The vasodi lator response ul timately depends on endothel ia l cel l formation of ni tricoxide (NO), which in the case of ACh i s a process that requires endothel ia l cel l integri ty. When the endothel ium is damaged or removed (d; themost l ikely case described here), the di rect effect of ACh on vascular smooth muscle i s contraction (constriction of arterioles ). Atropine, theprototype muscarinic antagonis t, wi l l prevent ACh-mediated vasodi lation of intact arterioles (b), wi th intact endothel ium, by blocking theinteraction between ACh and endothel ia l cel l muscarinic receptors , but i t wi l l not cause vasoconstriction when the endothel ium is intact (i t i simportant here to dis tinguish between preventing vasodi lation and caus ing vasoconstriction). Botul inum toxin (c) prevents ACh release fromchol inergic nerves , but in our experimenta l setup that i s i rrelevant because the in vi tro preparation i s devoid of innervation, and ACh i s addeddirectly to the ti ssue bath. Moreover, even i f the smooth muscle preparation were innervated, botul inum toxin would not convert the usualvasodi lator response to ACh into a vasoconstrictor one.

97. The answer is b. (Brunton, pp 186f, 206t, 1792; Katzung, pp 84, 479, 481.) Botul inum toxin, the cause of botul i sm, inhibi ts release of ACh from a l lchol inergic nerves , particularly those in the autonomic and somatic/motor nervous systems. (Take a look at the schematic of the periphera lnervous systems on pages 50 and 81 to refresh your memory about where these s i tes are.) It has no agonis t (a ) activi ty on any chol inergicreceptors , nor an abi l i ty to block nicotinic (e) or muscarinic receptors . The toxin has no di rect effects on adrenergic nerves (other than preventingtheir phys iologic activation by ACh, normal ly released by pregangl ionic sympathetic nerves . Thus , NE reuptake i s not affected (c; reca l l that coca ineand tricycl ic antidepressants do inhibi t NE reuptake); nor i s NE released (d) as occurs with, say, amphetamines .

98. The answer is e. (Brunton, pp 207t, 234, 242-243f, 251-252; Katzung, pp 105-107, 112.) Edrophonium is a fas t- and short-acting parentera lacetylchol inesterase inhibi tor—one of the relatively few therapeutic AChE inhibi tors with a generic name that doesn’t end in “-s tigmine.”

The question s tates that we are deal ing with a patient experiencing a chol inergic cri s i s . That means , of course, that she has received ei therrelatively or absolutely excess ive doses of the ora l chol inesterase inhibi tor (eg, pyridostigmine, neostigmine, etc) used to manage her myastheniagravis . Because of the dosage excess , and the excess ACh accumulating at the skeleta l neuromuscular junctions (owing to reduced metabol icinactivation of the neurotransmitter), her skeleta l muscles are weakened and fatigued from what amounts to overstimulation of nicotinicreceptors by the ACh. This skeleta l muscle hypofunction involves not only muscles in the extremities , but a lso the diaphragm and intercosta ls thatmust function normal ly for adequate venti lation. So, when we give a chol inesterase inhibi tor to patients l ike the one described above, who i sa l ready experiencing excess ive chol inergic effects , further (over)activation of the diaphragm and intercosta ls may be sufficient to cause venti latorydis tress or tota l venti latory fa i lure. This i s a main reason why the “edrophonium test” i s so dangerous should the underlying problem actual ly bethe chol inergic cri s i s , and why edrophonium is preferred to other ACh esterase inhibi tors because i t has a very short duration of action (highlydes i rable should adverse effects occur).

Edrophonium is not at a l l l i kely to trigger a hypertens ive cri s i s (a ). It and other drugs in i ts class , or di rect muscarinic agonis ts (eg, bethanechol )for that matter, have no vasoconstrictor activi ty (when vascular endothel ia l cel l s are intact). Chol inesterase inhibi tors are far more l ikely to lowerheart rate (via indi rect muscarinic receptor activation secondary to inhibi ted ACh hydrolys is at the S-A node) than s timulate i t (b); and just as thedrug causes no periphera l vasoconstriction, i t causes no coronary vasoconstriction. Ventricular automatici ty i s l ikely to decrease further, notincrease (c), in response to increased muscarinic activation. Edrophonium wi l l not improve skeleta l muscle tone and function (d) i f the problem isa chol inergic cri s i s ; that i s the expected response, however, i f the patient has a myasthenic cri s i s (reflecting inadequate dosages of thechol inesterase inhibi tor used for maintenance therapy).

Note: For many patients with myasthenia gravis , and who are being treated with an ora l chol inesterase inhibi tor, you should be able todi fferentiate between a myasthenic and a chol inergic cri s i s without resorting to the “edrophonium test.” You s imply need to assess your patientcareful ly, and apply some bas ic autonomic pharmacology knowledge. In ei ther cri s i s , the main common features are skeleta l muscle fatigabi l i ty,weakness , or para lys is due to inadequate (myasthenic cri s i s ) or excess ive (chol inergic cri s i s ) activation of nicotinic receptors on skeleta l muscle.With a myasthenic cri s i s , the primary problems are confined to skeletal muscle function (and secondary consequences of that, such as inadequateventi lation).

Things usual ly present qui te di fferently, however, i f the underlying problem is a chol inergic cri s i s . Remember that chol inesterase inhibi tors donot act only at skeleta l neuromuscular junctions . They a lso act at a l l chol inergic synapses in the periphera l nervous systems: at a l l thepostgangl ionic parasympathetic nerve “endings” and at the synapses between postgangl ionic sympathetic fibers and sweat glands—inhibi tingACh metabol i sm there, and leading to s igns and symptoms of muscarinic activation in addition to nicotinic/skeleta l muscle effects .

Thus , in patients who are experiencing a chol inergic cri s i s many or most of the s igns and symptoms typica l ly associated with parasympathetic(muscarinic) activation wi l l occur, and they wi l l be useful to you in making the di fferentia l diagnos is i f you look and l i s ten careful ly: mios is andexcess ive lacrimation; sa l ivation; bradycardia (and poss ibly A-V block determined by an ECG); wheezing or other mani festations ofbronchoconstriction and/or increased a i rway mucus secretions ; gut and bladder hypermoti l i ty (eg, defecation, urinary frequency or incontinence,increased bowel sounds); and diaphores is . See some of these in addition to skeleta l muscle dys function and you should be thinking chol inergiccri s i s and placing myasthenic cri s i s further down on your l i s t of poss ible diagnoses .

99. The answer is c. (Brunton, pp 225-235, 918-919, 921t, 923, 1816; Katzung, pp 115-122, 124-125, 278f, 292.) The periphera l autonomic actions ofdiphenhydramine (an ethanolamine-type “fi rs t-generation” his tamine H1 antagonis t) are very s imi lar to those of atropine, the prototypiccompeti tive muscarinic receptor antagonis t. These effects are seen with both therapeutic doses (eg, dry mouth, blurred vis ion, and occas ional lyurinary retention or constipation) and with toxici ty. In l i fe-threatening cases physostigmine, the “antidote for atropine poisoning,” may be ava luable adjunct.

Diphenhydramine, and other H1 blockers for that matter, have no di rect effects ei ther as agonis t (a ) or antagonis t (b) on adrenergic receptors ;and no nicotinic receptor-activating activi ty, whether in autonomic gangl ia (d) or at the adrenal medul la (suprarenal gland; e).

100. The answer is d. (Brunton, p 250; Katzung, pp 123f, 127, 1033t.) Pra l idoxime i s class i fied as a “chol inesterase reactivator” and i s used speci fica l ly,and adjunctively, for managing poisoning with the long-acting (“i rrevers ible”) organophosphate chol inesterase inhibi tors that are found in someinsecticides ; and “nerve gases” such as soman, sarin, and VX. Pra l idoxime, an oxime, has very high affini ty for phosphorus in the organo-phosphates . If given early enough in the poisoning i t wi l l prevent or reverse the binding of the organophosphate to, and long-term inhibi tion of,the active binding s i te on the chol inesterase. The need for early pra l idoxime adminis tration i s cri ti ca l : i f i t i s delayed too long the enzyme wi l l“age,” yielding a largely permanent and i rrevers ible enzyme configuration.

Page 99: Pharmacology pre test self assessment and review (14th ed.)

Methylphenidate (a) i s an indirect-acting (norepinephrine-releas ing or amphetamine-l ike) sympathomimetic; pra l idoxime has no effect on i tsactions , nor on the actions of epinephrine (b) or prazos in (e), a selective α1 adrenergic blocker. Al though neostigmine (d) i s a chol inesteraseinhibi tor i t i s not an organophosphate (i t i s a carbamate), and i t i s spontaneous ly and relatively quickly hydrolyzed leading to loss ofchol inesterase-inhibi tory activi ty. Thus , use of pra l idoxime for excesses of neostigmine (or other carbamate) doses would be both ineffective(mechanis tica l ly) and unnecessary.

101. The answer is b. (Brunton, pp 228, 233, 286, 1776, 1785t-1786t; Katzung, pp 88t, 89t, 93.) Start with a fundamenta l review of autonomic innervation ofthe eye insofar as control of pupi l s i ze, and effects on the ci l iary muscle (which affects accommodation—the abi l i ty of the lens of the eye tothicken so we can see close-up). The sympathetic nervous system, via α-adrenergic receptor activation, has the abi l i ty to cause mydrias is . Activatethose receptors and the pupi l di lates ; or block the oppos ing parasym-pathetic influences (via muscarinic receptor blockade) and the same thinghappens , s ince there’s a relatively comparable influence of both branches of the autonomic nervous system on pupi l s i ze. However, there i s l i ttleresting influence of the sympathetic nervous system (or adrenergic receptors ) on the tone of the ci l iary muscle. That i s mainly a parasympathetic-dependent phenomenon that indi rectly affects the shape of the lens of the eyes and, so, the point of sharp focus . And, for a l l practica l purposes ,β-receptor activation has no influence on pupi l s i ze or accommodation.

Now, class i fy the drugs by thei r s i tes and mechanisms of action.El iminate atropine (a) as a poss ible answer choice. By blocking a l l ocular effects of normal parasympathetic innervation (muscarinic blockade),

i t wi l l both di late the pupi l and impair accommodation. The same appl ies to homatropine (c), a drug that i s , for a l l practica l purposes , atropine-l ike. Pi locarpine (e), a muscarinic agonis t, wi l l constrict the pupi l of the eye and impair chol inergic mechanisms involved in focus ing of the lens .Isoproterenol (d) activates a l l β-adrenergic receptors , timolol (f) blocks them a l l ; and as I sa id, β-receptors are not relevant in terms of pupi l s i zeor accommodation. This leaves us with epinephrine: i t has both α-adrenergic and β-adrenergic activating activi ty, and no chol inergic/muscarinicactivi ty. It wi l l di late the pupi l and not s igni ficantly affect accommodation.

102. The answer is e. (Brunton, pp 920-922, 1816; Katzung, pp 278f, 292.) You may have chosen d (his tamine H2 receptor blocker) as your answer to thisquestion. However, H2 receptors are mainly involved in regulating gastric acid secretion, not typica l responses to seasonal a l lergies . What i f I hadgiven “His tamine H1 receptor blockade” as an answer choice? Al though diphenhydramine blocks H1 receptors qui te wel l , his tamine plays arelatively minor role in the symptoms of rhinovirus infections (in contrast with a much more important role in, say, seasonal a l lergies ). As a resul t,blocking the effects of his tamine on H1 receptors per se does not account for most of the symptom rel ief in the common cold. The drying up ofnasa l secretions afforded by diphenhydramine in this instance i s due to the drug’s rather intense muscarinic receptor-blocking (atropine-l ike)actions .

The antihis tamines (fi rs t-generation drugs such as diphenhydramine or second-generation ones such as loratadine) lack any antagonis t oragonis t activi ty on α-adrenergic receptors (a) or on β-adrenergic receptors (b); and have no di rect or major effect on ca lcium channels (d).

103. The answer is c. (Brunton, pp 305-307, 324t, 383t; Katzung, pp 152f, 154, 167, 180, 189.) Phenylephrine ra ises blood pressure by activating α-adrenergic receptors on vascular smooth-muscle cel l s . Prazos in competi tively blocks those receptors , and when present at sufficiently high doses(as might occur with an acute, severe overdose) may el iminate the vasopressor response to even higher than usual doses of phenylephrine. β-Blockers (atenolol , propranolol ) have no α-blocking activi ty and so won’t reduce the vasopressor response to phenylephrine. Nor wi l l bethanechol ,which causes vasodi lation by activating muscarinic receptors for ACh. And reserpine? See the answer to Question 104.

104. The answer is e. (Brunton, pp 68, 783; Katzung, pp 83f, 178.) Reserpine treatment of sufficient duration (because i ts effects develop s lowly)depletes neuronal norepinephrine. This accounts for the lowered blood pressure with therapeutic doses or frank hypotens ion with overdoses .One consequence of this , long-term, i s development of adrenergic receptor supersens i tivi ty because the receptors just aren’t being activated asthey should. And a consequence of that, in turn, there wi l l be heightened (and sometimes extreme) responses to a given dose of adrenergicagonis ts that can activate their receptors (α or β) di rectly, including the phenylephrine you adminis tered. (That i s , the dose-response curve fordi rect-acting adrenergic agonis ts i s “shi fted to the left” after long-term reserpine treatment.)

Atenolol (a ) and propranolol (d) are β-blockers . Al though atenolol preferentia l ly blocks β1 receptors , whi le propranolol (and most other β-blockers ) a ffect both β1 and β2, nei ther drug wi l l a l ter the expected vascular responses to an α-agonis t such as phenylephrine. Prazos in (c)competi tively antagonizes the effects of an α-agonis t. Bethanechol (b) i s a muscarinic agonis t; i f effective concentrations of the drug were in theci rculation at the same time as phenylephrine was given, i ts effects would phys iologica l ly antagonize the pressor responses to thevasoconstrictor.

105. The answer is c. (Brunton, pp 918-923, 1816; Katzung, pp 278f, 292.) Fi rs t off, I think you should know that the main active ingredient in OTC s leepaids (other than those adverti sed as “natura l” supplements such as melatonin, vi tamins , etc) i s a sedating antihis tamine: doxylamine or, morel ikely, diphenhydramine. Doxylamine i s extremely s imi lar to diphenhydramine, the prototype fi rs t-generation (older) “antihis tamine”—acompeti tive H1 receptor antagonis t. Chemica l ly, they are in the ethanolamine class and are used not only for a l lergy rel ief, but a lso for sedationand s leep. If one wanted to s tay awake and a lert (e) these certa inly would not be drugs that achieve that goal ; moreover, they don’t have anyamphetamine-l ike catecholamine-releas ing activi ty. These drugs a lso block muscarinic receptors—and qui te effectively to boot. So the answer tothis question boi l s -down to “What best describes the actions or effects of atropine?” As you should know, atropine or drugs with s imi larantimuscarinic actions should not be used by patients with prostatism or angle-closure (narrowangle) glaucoma.

Given the antimuscarinic effects , i f therapeutic doses do anything to heart rate i t wi l l be an increase, not a decrease (a) s ince chronotropiceffects of ACh on the S-A node are inhibi ted. ACh normal ly increases gut moti l i ty, faci l i tates micturi tion by s timulating the bladder’s trigone anddetrusor, and relaxes sphincters in the GI and urinary tracts . Blocking those influences on the gut would tend to cause constipation, not diarrhea(b). Blood pressure wi l l not ri se because of catecholamine-l ike vasoconstriction; these drugs have no α-adrenergic (or any other) agonis t activi ty.

Note: Once aga in I wrote a question about an ostens ibly “miscel laneous” drug, so why did I ask about doxylamine? I did so not to be picky, butbecause doxylamine (and more so diphenhydramine) are widely used; can cause problems for certa in people; and are ava i lable OTC, which meansyour patient can take them without your advice, consent, or knowledge.

Page 100: Pharmacology pre test self assessment and review (14th ed.)

Central Nervous System Pharmacology Alzheimer drugsAntidepressants and other mood-stabi l i zing drugsAntiepi leptics/anticonvulsantsAntiparkinson drugsAntipsychoticsAnxiolyticsCentra l nervous system stimulants and anorexigenic agentsEthanolGenera l anestheticsIntravenous anesthetics and anesthes ia adjunctsLoca l anestheticsOpioid analges ics and antagonis tsPsychotomimeticsSedatives , hypnotics

Questions

106. Guanfacine has recently been FDA-approved for managing attention-defici t hyperactivi ty disorder (ADHD) in patients between the ages of 6and 17 years . In addition to i ts benefi ts for ADHD, i t appears to provide symptom rel ief in genera l i zed anxiety disorder and post-traumatic s tressdisorder.

Al though the newly approved indications for guanfacine focus on the centra l nervous system, the drug can cause many important periphera lautonomic effects . They are as fol lows: • lowers tota l periphera l res is tance (systemic vascular res is tance);• at usual doses i t does not block vasopressor responses to such drugs as phenylephrine;• resting heart rate and ventricular s troke volume decl ine as effects of guanfacine bui ld during long-term therapy;• sympathetic-mediated increases of heart rate and s troke volume normal ly associated with such activi ties as exercise, or as a baroreceptor reflex

response to a sudden fa l l of blood pressure, are blunted;• mios is may occur; and• guanfacine has no di rect or indi rect effects on a i rway smooth muscle.• Based on this description, which one of the fol lowing drugs i s most s imi lar to guanfacine?

a. Atropineb. Clonidinec. Phentolamined. Prazos ine. Propranolol

Questions 107 to 111

The next five questions refer to the table below (shown earl ier for Questions 78 to 84 in the chapter “Periphera l Nervous Systems/Autonomics”),which i s the anesthes ia record from a patient undergoing two operative procedures during laparoscopic abdominal surgery. Here I ask you aboutsome drugs not addressed before. Remember: This medication adminis tration plan i s very typica l of what you’l l see with many s imi lar operativeprocedures .

Page 101: Pharmacology pre test self assessment and review (14th ed.)

107. You see that midazolam was the fi rs t drug given to the patient—actual ly right before she was transported into the OR. (The notation IVP nextto midazolam means “IV push,” ie, a rapid bolus injection.) In addition to caus ing sedation, anxiety rel ief, and genera l ly “smoothing” theinduction of anesthes ia , which of the fol lowing i s the most l ikely effect you would expect to achieve as a resul t of premedicating with midazolam?

a. Potentiating the analges ic effects of the morphineb. Preventing an intraoperative fa l l of blood pressurec. Preventing seizures l ikely to be caused by the propofold. Prophylaxis of cardiac arrhythmiase. Providing amnestic effects (suppress ing reca l l of perioperative events )

108. The record shows that propofol was given for induction. What i s /are the most l ikely adverse response(s ) associated with the adminis tration

Page 102: Pharmacology pre test self assessment and review (14th ed.)

of this widely used drug?

a. Hypotens ion and respiratory (venti latory) depress ionb. Laryngospasmc. Long-term memory lossd. Mal ignant hyperthermiae. Seizuresf. Unintended or excess ively prolonged skeleta l muscle para lys is

109. Assume that instead of inducing anesthes ia with propofol , the nurse anesthetis t used a parentera l barbi turate (eg, methohexi ta l , thiamyla l ,and thiopenta l ) with an extremely fast onset of action and short duration (sometimes ca l led “ul trashort”). These barbi turate induction agents areeffective, but are associated with a fa i rly high incidence of a certa in and rather unique adverse effect. What best describes what that adverseeffect i s?

a . Hypera lges ia (heightened perception of pa in)b. Hypertens ive cri s i sc. Laryngospasmd. Mal ignant hyperthermiae. Seizures , typica l ly monocl inic

110. The patient received an intravenous dose of ketorolac shortly before the wounds were closed and surgery was done. What i s the most l ikelypurpose for which i t was given?

a. Control of anticipated postoperative nausea and vomitingb. Postoperative pa in controlc. Reversa l of CNS depress ion caused by multiple depressant drugsd. Reversa l of drug-induced neuromuscular blockade (para lys is )e. Suppress ion of wound inflammation

111. Ondansetron was adminis tered about an hour before the anticipated end of surgery. What was the most l ikely reason for giving this to thepatient?

a. Hasten recovery of consciousness whi le effects of anesthetics wear offb. Intens i fy pa in-rel ieving effects of other analges ics given to the patientc. Lessen ri sks of postop urinary retention through effects on bladder musculatured. Prevent or reduce the ri sk of postoperative para lytic i leuse. Reduce the ri sk of postoperative nausea and vomiting

112. Two inhaled genera l anesthetics , A and B, have the fol lowing MAC va lues :

Based only on this information (note that we have not named any drugs), which s tatement i s true?

a. Drug A has a longer duration of action than Drug Bb. Drug A i s more soluble in the blood than Drug Bc. Drug B causes greater analges ia and skeleta l muscle relaxation than Drug Ad. The concentration of drug in inspi red a i r that i s needed to cause adequate surgica l anesthes ia i s higher for Drug B than for Drug Ae. The time to onset of adequate genera l anesthes ia i s 50 times longer for Drug B than for Drug A

113. A patient with Parkinson disease has s igns and symptoms that can be cons idered “moderate” now, but they are worsening and notresponding wel l to current drug therapy. The phys ician decides to empirica l ly assess an antiparkinson drug that i s a selective inhibi tor ofmonoamine oxidase type B (MAO-B). What drug would that be?

a. Bromocriptineb. Carbidopac. Phenelzined. Selegi l inee. Tranylcypromine

114. A patient who has been treated with levodopa i s switched to a regimen with a product that conta ins levodopa plus carbidopa. What i s themain action of carbidopa that provides the rationale for us ing i t in this dual -drug approach?

a. Blocks ACh release in the CNS, thereby faci l i tating levodopa’s abi l i ty to restore a dopamine-ACh ba lanceb. Helps activate dietary vi tamin B6, a deficiency of which occurs during levodopa therapy

c. Increases permeabi l i ty of the blood-bra in barrier to levodopa, giving levodopa better access to the CNSd. Inhibi ts metabol ic convers ion of levodopa to dopamine outs ide the CNSe. Reduces levodopa-induced hypotens ion by blocking vascular dopamine receptors

115. A 34-year-old man with mi ld anxiety and depress ion symptoms has heard about buspirone on TV and asks whether i t might be sui table for

Page 103: Pharmacology pre test self assessment and review (14th ed.)

him. According to the most diagnostic and treatment cri teria , the drug would be appropriate, particularly for short-term symptom control . Whichphrase correctly describes an important property of buspirone?

a. Associated with a withdrawal syndrome that, i f unsupervised or control led, may be fata lb. Has a s igni ficant potentia l for abusec. Is l ikely to potentiate the CNS depressant effects of a lcohol , benzodiazepines , and sedative antihis tamines (eg, diphenhydramine), so such

interactants must be avoided at a l l costd. Requires a lmost da i ly dosage ti trations in order to optimize the responsee. Seldom causes drows iness

116. A phys ician cons iders placing a patient on long-term (months , years ) phenobarbi ta l for control of a relatively common medica l condition. Formost of these indications , newer and arguably more efficacious drugs , which participate in fewer drug interactions , are ava i lable and preferred.For which one of the fol lowing uses , nonetheless , i s this barbi turate s ti l l cons idered reasonable and appropriate?

a. Alcohol withdrawal s igns/symptomsb. Anxiety managementc. Certa in epi leps iesd. Endogenous depress ion (adjunct to SSRIs )e. Sleep disorders such as insomnia

117. One reason for the decl ining use of tricycl ic antidepressants such as imipramine, and the growing use of newer classes , i s the preva lence ofcommon tricycl ic-induced s ide effects or adverse responses . What s ide effect or adverse response, l i s ted below, i s most l ikely to occur with usualtherapeutic doses of a tricycl ic?

a . Antichol inergic (antimuscarinic) effectsb. Arrhythmiasc. Hepatotoxici tyd. Nephrotoxici tye. Seizures

118. A 42-year-old woman develops akathis ias , parkinsonian-l ike dyskines ias , ga lactorrhea, and amenorrhea, as a consequence of psychotropicdrug therapy. What drug-receptor-based mechanism, occurring in the centra l nervous system, most l ikely caused these responses?

a. Blockade of α-adrenergic receptorsb. Blockade of dopamine receptorsc. Blockade of muscarinic receptorsd. Supersens i tivi ty of dopamine receptorse. Stimulation of nicotinic receptors

119. A patient on the trauma-burn uni t received a drug to ease the pa in of debridement and dress ing changes for severa l severe burns . Heexperiences good, prompt analges ia , but despi te the absence of pa in sensation during the procedure his heart rate and blood pressure ri secons iderably, cons is tent with sympathetic nervous system activation by the pa in and not affected by the analges ic drug. As the effects of the drugdevelop his skeleta l muscle tone progress ively increases . He appears awake at times because his eyes periodica l ly open. As drug effects wear offhe ha l lucinates and behaves in a very agi tated fashion. Hal lucinations , “bad dreams,” and periods of del i rium recur over severa l days afterreceiving the drug. What drug was most l ikely given?

a. Fentanylb. Ketaminec. Midazolamd. Succinylchol inee. Thiopenta l

120. A 17-year-old male was diagnosed with epi lepsy after developing repeated episodes of genera l i zed tonic-clonic seizures fol lowing a motorvehicle accident in which he received a closed-head injury. After treating acute seizures with the proper injectable drugs , he i s s tarted on aregimen of ora l phenytoin, the da i ly dose ti trated upward unti l symptom control and a therapeutic plasma concentration were reached. Theel imination ha l f-l i fe of the drug during ini tia l treatment was measured to be 24 hours , a va lue that i s qui te typica l for otherwise heal thy adultstaking no other drugs .

Today he presents in the neurology cl inic with nystagmus, ataxia , diplopia , cogni tive impairment, and other s igns and symptoms cons is tent withphenytoin toxici ty. A blood sample, drawn at noon, has a plasma phenytoin concentration of 30 mcg/mL. That va lue i s 50% higher than typica l peaktherapeutic plasma concentrations , and twice the usual minimum effective blood level . These va lues are summarized in the figure below.

Page 104: Pharmacology pre test self assessment and review (14th ed.)

The attending orders discontinuation of further doses of phenytoin unti l plasma levels fa l l into the therapeutic range, and the patient i s largelyfree of s igns and symptoms of phenytoin toxici ty. In the interim, what would you do or expect to occur next?

a. Adminis ter flumazeni l , which wi l l quickly reverse s igns and symptoms of phenytoin toxici ty but may cause seizures to recurb. Anticipate that el imination of phenytoin from the plasma wi l l fol low zero-order kinetics for severa l daysc. Give an amphetamine or other CNS s timulant to reverse genera l i zed CNS depress ion due to the phenytoin excessd. Give phenobarbi ta l to induce the P450 system, thereby hastening phenytoin’s metabol ic el iminatione. Plasma phenytoin concentrations wi l l fa l l to 15 mcg/mL, in the middle of the therapeutic range, by noon tomorrow (24 hours later, per the usual

ha l f-l i fe)

121. A 12-year-old boy has been treated with methylphenidate for the las t 3 years . His younger s i s ter finds the bottle of pi l l s and consumesenough to cause s igni ficant toxici ty. Which of the fol lowing findings would you most l ikely expect?

a. Hypertens ion, tachycardia , seizuresb. Hypotens ion, bronchospasmc. Drows iness , obtunded reflexes , diarrhead. Mios is , bradycardia , profuse sa l ivation, sweatinge. Hypothermia, skeleta l muscle weakness or para lys is , pupi l s that are not respons ive to l ight

Page 105: Pharmacology pre test self assessment and review (14th ed.)

122. Meperidine i s s imi lar to morphine in many ways , but has some decided di fferences that are cl inica l ly relevant: with very high blood levels orwith true overdoses , meperidine can cause s igni ficant adverse responses that s imply aren’t seen with morphine or most other opioid analges ics .What i s that rather unique effect of meperidine?

a. Constipation leading to para lytic i leusb. Heightened response to pa in (paradoxica l hypera lges ia)c. Intense bi l iary tract spasmd. Psychos is -l ike s tate, poss ibly seizurese. Respiratory depress ion, apnea, venti latory arrest

123. Chlorpromazine and ha loperidol can be cons idered prototypes of two relatively old but s ti l l -used antipsychotic drug classes : thephenothiazines and the butyrophenones , respectively. Whi le many of the actions and s ide effects of these drugs are qual i tatively s imi lar, they aredi fferent quanti tatively: that i s , in terms of incidence and severi ty. Which effect or s ide effect typica l ly occurs more frequently, i s usual ly moresevere, and has a relatively rapid onset, with ha loperidol?

a . Extrapyramidal reactionsb. Intense atropine-l ike s ide effectsc. Letha l blood dyscras iasd. Orthostatic hypotens ione. Urinary retention necess i tating bladder catheterization

124. A patient i s transported to the emergency department. A friend who accompanies the patient says “he was experimenting with ‘angel dust’”.What best describes the actions or other characteris tics of this recreational drug, more properly know as phencycl idine?

a. Causes i ts periphera l and centra l effects via antimuscarinic propertiesb. Causes s igni ficant withdrawal symptomsc. Has s trong opioid receptor-activating activi tyd. Has amphetamine-l ike properties and i s an ha l lucinogene. Overdoses should be treated with flumazeni l

125. Package inserts for a drug caution aga inst adminis tering i t concurrent with any other drug that can ra ise or lower sodium concentrations . Therisks are inadequate or excess ive effects of the drug, depending on the di rection in which sodium concentrations change. This , of course, requirescautious use or avoidance (i f poss ible) of the common diuretics . To which of the fol lowing drugs does this caution or warning apply?

a. Cholestyramineb. Li thium (eg, l i thium carbonate)c. Ni fedipined. Phenylephrinee. Statin-type cholesterol -lowering drugs

126. A 31-year-old woman has been treated with fluoxetine for 5 months . She i s diagnosed with another medica l problem and receives one ormore drugs that, otherwise, would be sui table and probably problem-free. She i s rushed to the ED with unstable vi ta l s igns , muscle rigidi ty,myoclonus , CNS i rri tabi l i ty and a l tered consciousness , and shivering. What add-on drug(s ) most l ikely caused these responses?

a. Codeine for coughb. Loratadine for seasonal a l lergiesc. Midazolam and fentanyl , used to ease discomfort from endoscopyd. Sumatriptan for migra inee. Zolpidem for short-term insomnia

127. A 72-year-old woman with a long his tory of anxiety that has been treated with diazepam decides to triple her da i ly dose because of increas ingfearfulness about “environmenta l noises .” Two days after her attempt at sel f-prescribing, she i s found extremely lethargic and nonrespons ive,with markedly obtunded reflexes and reactions to pa inful s timul i . Respirations are 8/min and sha l low. What drug should be given to reversethese s igns and symptoms?

a. Dextroamphetamineb. Flumazeni lc. Nal trexoned. Physostigminee. Pra l idoxime

128. A patient who has been treated for Parkinson disease for about a year presents with purpl i sh, mottled changes to her skin. What drug i s themost l ikely cause of this cutaneous response?

a. Amantadineb. Bromocriptinec. Levodopa (a lone)d. Levodopa combined with carbidopae. Pramipexole

129. A young boy who has been treated for epi lepsy for a year i s referred to a periodontis t for eva luation and probable treatment of mass iveovergrowth of his gingiva l ti s sues . Some teeth are a lmost completely covered with hyperplastic ti s sue. Which drug was the most l ikely cause of theora l pathology?

Page 106: Pharmacology pre test self assessment and review (14th ed.)

a. Carbamazepineb. Lorazepamc. Phenobarbi ta ld. Phenytoine. Va lproic acid

130. A patient with undiagnosed coronary artery disease i s given a medication. Shortly thereafter she develops intense tightness and “crushingdiscomfort” of her chest. An ECG reveals ST-segment changes indicative of acute myocardia l i schemia. Which drug most l ikely caused this reaction?

a. Clozapineb. Pentazocinec. Phenytoind. Sumatriptane. Zolpidem

131. Ni trous oxide i s a common component in the technique of ba lanced anesthes ia . It i s used in conjunction with such other drugs as ahalogenated hydrocarbon volati le l iquid anesthetic, and usual ly included as 80% of the tota l inspi red gas mixture. Which phrase best summarizeswhy ni trous oxide cannot be used a lone for genera l anesthes ia?

a. Almost tota l lack of analges ic activi ty, regardless of concentrationb. Inspi red concentrations >10% tend to profound cardiac negative inotropic effectsc. MAC (minimum a lveolar concentration) i s >100%d. Methemoglobinemia occurs even with low inspired concentrationse. Such great solubi l i ty in blood that i ts effects take an extraordinari ly long time to developf. Very high frequency of bronchospasm

132. A patient develops a severe and rapidly worsening adverse response to a drug. The phys ician orders prompt adminis tration of antipyretics , IVhydration, and bromocriptine or dantrolene to manage symptoms and hopeful ly to prevent a fata l outcome. Which drug or drug group most l ikelycaused these adverse responses?

a. Benzodiazepines , especia l ly those used as hypnoticsb. Chlorpromazinec. Levodopad. Phenytoine. SSRIs

133. Ropini role i s a relatively new drug that recently was approved to treat what’s commonly ca l led restless leg syndrome (a lso known as Ekbomsyndrome). The drug works as a dopamine receptor agonis t in certa in parts of the bra in. Given this mechanism of action, what other disorder i s ,most l ikely, another indication for this drug?

a. Daytime anxietyb. Hypersomnia (excess ive s leepiness )c. Parkinson diseased. Schizophreniae. Status epi lepticusf. Treatment of severe pa in

134. A patient in the neurology uni t at your hospi ta l develops s tatus epi lepticus , and at the time there i s no good information about the etiology.What drug should be given fi rs t for the fastest suppress ion of the seizures?

a. Carbamazepineb. Lorazepamc. Phenobarbi ta ld. Phenytoine. Va lproic acid

135. A patient has had a documented severe a l lergic reaction to ester-type loca l anesthetics . What other drug i s a member of the ester class , andso would be the most l ikely to provoke an a l lergic or anaphylactic reaction i f this patient received i t?

a . Bupivaca ineb. Lidoca inec. Mepivaca ined. Pri loca inee. Tetraca ine

136. A 66-year-old woman is diagnosed with Alzheimer disease, with symptoms being described as mi ld-to-moderate. What pharmacologicapproach i s genera l ly cons idered the most frui tful in terms of a l leviating symptoms of early Alzheimer and probably s lowing the course of theunderlying bra in pathology?

a. Activate a population of serotonin receptorsb. Block dopamine release or receptor activationc. Inhibi t acetylchol inesterase

Page 107: Pharmacology pre test self assessment and review (14th ed.)

d. Inhibi t MAOe. Dissolve cerebra l vascular thrombi

137. Trihexyphenidyl i s prescribed as an adjunct to other drugs being used to manage a patient with Parkinson disease. What i s the most l ikelypurpose or action of this drug as part of the overa l l drug treatment plan?

a. To counteract sedation that i s l ikely to be caused by the other medicationsb. To help correct further the dopamine-ACh imbalance that accounts for parkinsonian s igns and symptomsc. To manage cutaneous a l lergic responses that are so common with “typica l” anti -parkinson drugsd. To prevent the development of manic/hypomanic responses to other antiparkinson drugse. To reverse tardive dyskines ias i f the parkinsonism was induced by an antipsychotic drug

138. A few years ago the FDA granted approval to market a new prescription drug (“drug X”) that wi l l be adminis tered in the form of a dermal patch(apply the patch to intact skin, the drug i s absorbed from there).

Drug X belongs to a very old class of drugs that, when given by i ts usual route, ora l ly, can interact with foods such as cheese and processedmeats (and certa in breads , other foods , and a lcohol ic beverages) leading to an interaction that can elevate blood pressure to severe andsometimes fata l levels . After more than a decade of testing, the FDA approved i ts use for adul ts . In i ts lowest dose, no dietary restriction(s ) arerequired.

Based on this information, how is drug X most l ikely class i fied and what i s i ts most l ikely cl inica l use?

a. Amphetamine-l ike agent for ADD/ADHDb. Barbi turates used for daytime anxietyc. Benzodiazepine for anxiety and s leepd. MAO inhibi tor for depress ione. Morphine-l ike analges ic for severe/chronic pa in

139. The pediatrician wri tes a prescription for a combination (of severa l drugs) product that conta ins dextromethorphan, which i s an i somer of acodeine analog. The patient i s a 12-year-old boy. What i s the most l ikely purpose for which the drug was prescribed?

a. Control mi ld-moderate pa in after the lad broke his wris t playing soccerb. Manage diarrhea caused by food-borne bacteriac. Provide sedation because the chi ld has ADD/ADHDd. Suppress severe cough associated with a bout of influenzae. Treat nocturnal bed-wetting

140. Many lega l juri sdictions have imposed various restrictions on over-the-counter sa le of products , mainly ora l decongestants , that conta inpseudoephedrine. That i s because pseudoephedrine can be rather eas i ly used to synthes ize which highly psychoactive and abuse-prone drug?

a. Methamphetamineb. Morphinec. Oxycodoned. Pentazocinee. Phencycl idine (PCP)

141. The anesthes iologis t prepares to adminis ter severa l drugs to a patient as part of normal pre- and intraoperative care. What drug lacks, as i tsnormal spectrum of action, the abi l i ty to cause genera l i zed CNS depress ion or the patient’s level of consciousness , or lacks any intrins ic analges iceffects?

a. Droperidolb. Midazolamc. Vecuroniumd. Propofole. Thiopenta l

142. A 26-year-old woman has been on antidepressant therapy for severa l months . Today she compla ins of miss ing her period and having ga lactor-rhea, and your careful assessment suggests that she has developed some dyskines ias (mi ld tremors , for example) not unl ike those you wouldtypica l ly associate with a phenothiazine or butyrophenone (eg, ha loperidol ) antipsychotic drug. Pregnancy tests are negative. What drug mostl ikely to have caused these findings?

a. Amoxapineb. Ci ta lopramc. Fluoxetined. Sertra l inee. Tranylcypromine

143. A patient has been taking an ora l monoamine oxidase inhibi tor (MAOI), but that fact i s unknown to the heal th team who is now taking care ofher, for unrelated medica l conditions , in the hospi ta l . The patient receives a drug that leads to a fata l response characterized by profound fever,del i rium, psychotic behavior, and s tatus epi lepticus . It was found to have occurred because of an interaction with the MAOI. Which, most l ikely,was this second drug or the drug class to which i t belongs?

a. Barbi turateb. Diazepam

Page 108: Pharmacology pre test self assessment and review (14th ed.)

c. Meperidined. Morphinee. Phenytoin

144. A young woman is taken to the emergency department by some of her friends . It seems they were out on “bar night” and someone s l ippedsomething into her a lcohol ic beverage, the fi rs t and only one she consumed that night. She i s now extraordinari ly drowsy and has l i ttle reca l l ofwhat happened between the time she s ipped her drink and now. Someone overheard another bar patron ta lking about “roofies .” You suspect herdrink was spiked with Rohypnol , the lesser-known generic name of which i s fluni trazepam. A pos i tive response (ie, symptom improvement) towhat drug that you give would confi rm your suspicion?

a. Diazepamb. Flumazeni lc. Ketamined. Nal trexonee. Triazolam

145. In deciding on pharmacotherapy for many patients you’ve diagnosed with depress ion, you’ve usual ly cons idered s tarting with an SSRI or, insome cases , a tricycl ic. Today you assess a patient and suspect endogenous depress ion. Whi le discuss ing treatment options they refer to a drugby name and ask you about i t; they’ve seen many adverti sements for i t in magazines and on TV. The drug (generic name) i s bupropion. In whatmain way does bupropion di ffer from ei ther or both the SSRIs or tricycl ics?

a. Higher incidence of CNS depress ion, drows inessb. Higher incidence of weight ga inc. Less drug-induced sexual dys functiond. Much more common and severe fa l l s of resting blood pressure and orthostatic hypotens ione. More severe and more frequent periphera l antichol inergic (atropine-l ike) s ide effectsf. Stronger inhibi tion of monoamine oxidase

146. A 33-year-old woman patient treated with ha loperidol i s seen in the emergency department (ED). Her husband describes compla ints of rapidlyworsening fever, muscle s ti ffness , and tremor. Her level of consciousness i s diminishing. Her temperature i s 104°F, and her blood creatine kinase(CK) level i s elevated. What i s the most l ikely explanation for these findings?

a. Al lergic response to her medicationb. Neuroleptic mal ignant syndrome (NMS)c. Overdosed. Parkinsonisme. Tardive dyskines ia

147. Nearly a l l the drugs used as primary therapy, or as adjuncts , for the treatment of Parkinson disease or drug-induced parkinsonism exert thei rdes i red effects di rectly in the bra in’s s triatum. Which one exerts i ts main effects in the gut, not in the bra in?

a. Amantadineb. Benztropinec. Bromocriptined. Carbidopae. Selegi l ine

148. You have a patient with severe postoperative pa in who i s not getting adequate analges ia from usual ly effective doses of morphine. Thephys ician orders an immediate switch to pentazocine (at usual ly effective analges ic doses). What i s the most l ikely outcome of s topping themorphine and immediately s tarting the pentazocine?

a. Abrupt, added respiratory depress ionb. Acute development of phys ica l dependencec. Comad. Seizurese. Worsening of pa in

149. The chosen pharmacologic approach to managing a patient with mi ld and recently diagnosed parkinsonism wi l l be to enhance speci fica l lythe activi ty of endogenous bra in dopamine by inhibi ting i ts metabol ic inactivation. What drug works primari ly by that mechanism?

a. Benztropineb. Selegi l inec. Trihexyphenidyld. Bromocriptinee. Chlorpromazine

150. Chlorpromazine has been prescribed for a patient with schizophrenia , and the patient has been taking the drug, at usual ly effective doses , forabout 6 months . Today he comes to the hospi ta l with other medica l conditions that require surgery and the adminis tration of other drugs , and wedecide i t i s unwise to s top the chlorpromazine and run the ri sk of psychotic behavior whi le we perform other interventions . What others igns/symptoms that the patient may a lso have or acquire as the resul t of surgery and drug therapy are most l ikely to be affected beneficia l ly bythe continued use of chlorpromazine?

a. Epi lepsy and the ri sk of seizures

Page 109: Pharmacology pre test self assessment and review (14th ed.)

b. Hypotens ionc. Nausea and vomitingd. Urinary retention caused by abdominal surgerye. Xerostomia (dry mouth) caused by antimuscarinic drugs used to prevent intra-operative bradycardia

151. There are, rightful ly, concerns about coca ine abuse, and too many deaths have occurred from smoking “crack” coca ine or injecting or nasa l lyinhal ing the drug. What s tatement best describes the main mechanism by which coca ine exerts i ts deleterious effects in the centra l nervoussystem or in the periphery?

a. Di rectly activates , as an agonis t, both α- and β1-adrenergic receptors

b. Enhances neuronal ly mediated adrenergic receptor activation by inhibi ting neuronal norepinephrine reuptakec. Inhibi ts catecholamine inactivation by inhibi ting MAO and catechol -O-methyl transferased. Produces bradycardia and vasodi lation, leading to hypotens ion and acute heart fa i lure, by blocking neuronal NE releasee. Stimulates autonomic nerve conduction effectively, leading to increased neuronal norepinephrine release

152. One approach to managing hyperprolactinemia i s to adminis ter a drug that has relative selectivi ty, as an agonis t, for centra l (as an agonis t)dopamine D2 receptors . What drug works in that manner?

a. Bromocriptineb. Chlorpromazinec. Fluphenazined. Haloperidole. Promethazine

153. We perform a meta-analys is on the abi l i ty of various antipsychotic drugs to cause constipation, urinary retention, blurred vis ion, and drymouth—al l of which reflect s igni ficant blockade of muscarinic receptors in the periphera l nervous systems. What drug most l ikely caused theseunwanted effects?

a. Chlorpromazineb. Clozapinec. Haloperidold. Olanzapinee. Sertra l ine

154. It i s obvious that morphine should a lways be adminis tered “with care.” However, one of i ts expected effects that occur only whi le the drug i sbeing given makes i t particularly dangerous to adminis ter to certa in patients unless “specia l measures” can be taken to prevent this adverseeffect. Which comorbidi ty weighs aga inst us ing morphine, unless you fi rs t take precautions to prevent adverse consequences from occurring?

a. Acute pulmonary edemab. Closed-head injuryc. His tory of epi lepsyd. Hypertens ione. Recent/evolving myocardia l infarction

155. A 66-year-old woman has terminal cancer, and i s in hospice. She i s receiving round-the-clock opioids , at rather high doses , but s ti l l reportswhat she describes as s igni ficant burning, shooting pa in. The phys ician bel ieves i t i s neuropathic. Increas ing the dose of opioids may be helpful ,but that option i s ruled-out at this time because doing so i s l ikely to suppress venti lation excess ively. In addition, the patient does not want theexcess ive grogginess that i s apt to occur with more opioid on board. Al though the patient i s not at a l l hypertens ive, the phys ician prescribes as ananalges ic adjunct that i s far more widely used as an antihypertens ive drug. What i s the most l ikely adjunctive drug she prescribed?

a. Captopri lb. Clonidinec. Hydrochlorothiazided. Labeta lole. Prazos in

156. A patient i s on long-term methadone therapy as part of a hol i s tic plan to curb their opioid addiction and abuse. What phrase best describes acharacteris tic of this drug?

a. Causes pentazocine-l ike activation of κ receptors and blockade of μ receptorsb. Has greater ora l bioava i labi l i ty than morphine, especia l ly when ora l adminis tration i s s tartedc. Remarkably devoid of such typica l opioid analges ic s ide effects as constipation and respiratory depress iond. Useful for maintenance therapy in opioid- (eg, heroin-) dependent individuals , but lacks cl inica l ly useful ana lges ic effectse. When abruptly s topped after long-term adminis tration, causes a withdrawal syndrome that i s more intense, but briefer, than that associated

with morphine or heroin withdrawal

157. A mom gets a note from her 10-year-old daughter’s teacher that the chi ld recently s tarted experiencing numerous a lbei t brief episodes of “justs taring into space” throughout the school day. The mother rea l i zes she’s noticed the same in her gi rl at home. After a trip to the pediatrician, andreferra l to a neurologis t, a diagnos is of absence epi lepsy i s made. What drug i s genera l ly cons idered the preferred s tarting drug for this type ofepi lepsy in an otherwise heal thy chi ld?

a. Diazepam

Page 110: Pharmacology pre test self assessment and review (14th ed.)

b. Ethosuximidec. Lorazepamd. Methylphenidatee. Phenytoin

158. A 43-year-old woman becomes hypertens ive and suffers a fata l acute coronary syndrome shortly after s tarting therapy on a drug. Autopsyshows l i ttle in the way of coronary atheroscleros is , but ECG changes noted just before her death revealed s igni ficant myocardia l i schemia in themyocardium served by the left anterior descending and ci rcumflex coronary arteries . The cause of death i s thought to involve coronary vasospasm.What drug most l ikely precipi tated this event?

a. Bromocriptine for Parkinson diseaseb. Ergotamine given to abort a migra ine attackc. Morphine for post-trauma analges iad. Phenoxybenzamine used for carcinoid syndromee. Phenytoin to manage genera l i zed tonic-clonic seizures

159. Promethazine, a phenothiazine derivative with substantia l anti -emetic, anti tuss ive, and H1-his tamine receptor blocking activi ty, has a cl inica lprofi le qui te s imi lar to diphenhydramine. Recently the FDA mandated a “black box warning” for this widely used drug. The FDA now warns aga instuse of the drug, in a l l doses and forms, for chi ldren aged 2 years or younger. Fata l i ties have occurred in these young patients , even in response todosages that previous ly were cons idered therapeutic and safe. What i s the most l ikely cause of death from promethazine in these patients?

a. Complete (thi rd degree) heart block fol lowed by asystoleb. Hypertens ive cri s i s , intracrania l hemorrhagec. Parkinsonian-l ike dyskines ias , including tardive dyskines iasd. Severe and refractory diarrhea leading to fluid and electrolyte losse. Venti latory depress ion, apnea, excess ive CNS depress ion

160. A 55-year-old undergoes surgery. She receives severa l drugs for pre-anesthes ia care, intubation, and intraoperative skeleta l muscle para lys is ;and a mixture of inhaled and parentera l agents to provide ba lanced anesthes ia . Toward the end of the procedure she develops a rapidlyprogress ing fever, hypertens ion, hyperka lemia, tachycardia , muscle rigidi ty, and metabol ic acidos is . Which drug combination i s most l ikely to haveel ici ted this reaction?

a. Fentanyl and midazolamb. Midazolam and morphinec. Ni trous oxide and etomidated. Propofol and midazolame. Succinylchol ine and i soflurane

161. A 30-year-old woman with partia l seizures i s treated with vigabatrin. What i s the speci fic mechanism of action of this GABA-relatedanticonvulsant?

a. Blocked neuronal reuptake of released GABAb. Di rectly activated postsynaptic GABA receptorsc. Increased neuronal GABA released. Inhibi ted catabol i sm of released GABAe. Stimulated neuronal GABA synthes is by acting as a metabol ic precursor

162. A 24-year-old woman has a his tory of epi lepsy that i s being treated with phenytoin. She i s hea l thy otherwise. She becomes pregnant. Whatwould you do throughout the remainder of her pregnancy, in addi tion to providing otherwise proper perinata l care?

a. Add va lproic acidb. Discontinue a l l anticonvulsant medicationc. Increase da i ly dietary i ron intaked. Prescribe da i ly fol ic acid supplementse. Switch from the phenytoin to phenobarbi ta l

163. A patient i s transported to the emergency department by ambulance after repeated episodes of fa inting. The cause was attributed to severedrug-induced orthostatic hypotens ion due to α-adrenergic blockade from one of the drug’s main s ide effects . What drug was the most l ikely causeof this problem?

a. Buspironeb. Chlorpromazinec. Diphenhydramined. Haloperidole. Zolpidem

164. Clozapine, as an example of the “atypica l antipsychotics ,” seldom is used as fi rs t-l ine (ini tia l ) therapy of schizophrenia . Compared with theolder “tradi tional” antipsychotics , i t i s associated with a much higher ri sk of a serious adverse response. What i s that greater ri sk?

a. Agranulocytos isb. Extrapyramidal s ide effects (parkinsonism)c. Hypoglycemia

Page 111: Pharmacology pre test self assessment and review (14th ed.)

d. Hypotens ion, severee. Venti latory depress ion or arrest

165. You’re at the end of the fi rs t week of your M3 ob-gyn clerkship. You are about to go into the del ivery room to see your fi rs t chi ldbirth. You’vereviewed her chart and see that she was taking, for the duration of pregnancy, a drug that i s not absolutely contra indicated in pregnancy (ie, notcategory X). You note that about 1 month before she was prescribed da i ly ora l vi tamin K supplements . The baby i s born and promptly gets aninjection of vi tamin K. Knowing what you do about vi tamin K, you correctly reason that these measures were taken to reduce the ri sks of excess iveor abnormal bleeding, caused by drug-induced impairments of hepatic vi tamin K-dependent clotting factors , in the newborn. What drug did themother most l ikely receive during pregnancy, assuming that i t was not in pregnancy category X?

a. Bupropionb. Diazepamc. Methadoned. Phenytoine. Warfarin

166. When carbidopa i s adminis tered a long with levodopa for Parkinson disease, we increase the bioava i labi l i ty of levodopa by inhibi ting theformation of dopamine in the gut. However, the carbidopa-induced inhibi tion of dopa decarboxylase favors the periphera l metabol i sm oflevodopa to another metabol i te that competes with levodopa for transport across the blood-bra in barrier. This i s cata lyzed by catechol -O-methyl transferase (COMT). If your goal i s to inhibi t COMT, and so increase the centra l bio-ava i labi l i ty and effects of levodopa, what drug would youchoose?

a. Donepezi lb. Entacaponec. Selegi l ined. Tacrinee. Trihexyphenidyl

167. A patient has a long his tory of excess ive a lcohol consumption. He was arrested severa l times for drunk driving and was referred to a phys icianfor therapy. The phys ician prescribed a drug to s ti fle further a lcohol ingestion, to be used a long with other interventions . The doctor properlyinstructed the patient not to consume any a lcohol , not to use a lcohol -conta ining mouthwashes , nor even apply a lcohol -based toi letries , becausealcohol may cause a dis turbing, i f not dangerous , interaction with his medication. The patient ignored the advice and decided to have a beer.Within minutes he develops flushing, a throbbing headache, nausea, and vomiting. Which drug was he most l ikely taking to curb his a lcohol use?

a. Nal trexoneb. Diazepamc. Disul fi ramd. Phenobarbi ta le. Tranylcypromine

168. A patient with endogenous depress ion i s s tarted on a drug that selectively inhibi ts neuronal serotonin (5-HT) reuptake and has minimaleffect on the reuptake of norepinephrine or dopamine. What drug best fi ts this description?

a. Amitryptyl ineb. Bupropionc. Fluoxetined. Imipraminee. Venlafaxine

169. A 29-year-old man uses an ora l benzodiazepine and a lcohol to sati s fy his addiction to CNS depressants . During the past week, he has beenincarcerated and i s not able to obta in the drugs . He i s brought to the medica l ward because of the onset of severe anxiety, increased sens i tivi ty tol ight, dizziness , and genera l i zed tremors due to drug withdrawal . On phys ica l examination, he i s hyperreflexic. Which drug would be the bestchoice to diminish his withdrawal symptoms?

a. Buspironeb. Chlora l hydratec. Chlorpromazined. Lorazepame. Trazodone

170. A 50-year-old man has been consuming large amounts of ethanol on an a lmost da i ly bas is for many years . One day, unable to find anyethanol , he ingests a large amount of methanol (wood a lcohol ) that he had bought for his camp lantern. What drug would adminis ter to best treatunderlying biochemica l consequences of the methanol poisoning?

a. Diazepamb. Ethanolc. Flumazeni ld. Phenobarbi ta le. Phenytoin

171. Many news reports have told of a large number of deaths of opioid abusers who purchased and sel f-adminis tered i l l i ci t drugs that conta inedletha l amounts of fentanyl . One patient who received this fentanyl -laced drug presents in your emergency department, barely a l ive. What drugwould you adminis ter fi rs t, wi th the best hope that i t can promptly reverse the letha l effects of the fentanyl?

Page 112: Pharmacology pre test self assessment and review (14th ed.)

a. Diazepamb. Flumazeni lc. Naloxoned. Nal trexonee. Phenytoin

172. A 10-year-old boy has nocturnal enures is . His parents take him to a cl inic that specia l i zes in management of this condition. The phys icianwri tes an order for a low dose of imipramine. After a couple of weeks on the drug, the episodes of bed-wetting decrease dramatica l ly. What i s themost l ikely mechanism by which the imipramine provided benefi t?

a . Al leviates depress ion s igns and symptoms by increas ing neuronal catecholamine reuptakeb. Blocks muscarinic receptors in the bladder musculaturec. Causes sedation such that the boy s leeps through the night without voidingd. Reduces renal blood flow, glomerular fi l tration, and urine outpute. Releases antidiuretic hormone (ADH)

173. A patient i s transported to your emergency department because of a seizure. A review of his his tory reveals that he has been treated bydi fferent phys icians for di fferent medica l conditions , and there has been no dia log between them in terms of what they’ve prescribed. Onephys ician prescribed a drug for short-term management of depress ion. Another prescribed the very same drug, marketed under a di fferent tradename, to help the patient qui t smoking cigarettes . What drug was most l ikely prescribed by both doctors , and was the most l ikely cause of theseizures?

a. Bupropionb. Chlordiazepoxidec. Fluoxetined. Imipraminee. Li thium

174. About 1 year ago you diagnosed schizophrenia s igns and symptoms in a 23-year-old otherwise heal thy man. As a resul t of intens ivepsychotherapy, careful ti tration of chlorpromazine dosages , and remarkably good compl iance on the patient’s part, he i s wel l enough to return towork. Severa l months later, at a scheduled vis i t, you observe numerous s igns and symptoms of drug-induced parkinsonism, and the patient reportsrather dis tress ing symptoms of akathis ias (inner restlessness , ji tteriness , etc). However, typica l mani festations of schizophrenia seem to be wel lcontrol led. Which approach i s most l ikely to a l leviate the motor and subjective parkinsonian responses , and pose the lowest ri sk of caus ingschizophrenia s igns and symptoms to reappear?

a. Add a catechol -O-methyl transferase inhibi tor (eg, tolcapone)b. Add a centra l ly acting chol inesterase inhibi tor (eg, donepezi l or tacrine)c. Add benztropined. Add levodopa or levodopa plus carbidopae. Switch from chlorpromazine to ha loperidol

175. A patient with Parkinson disease s tarts therapy with a drug that acts in the CNS as an agonis t for dopamine receptors . It has no di rect effectson dopamine synthes is , neuronal reuptake, or metabol ic inactivation. What drug fi ts this description the best?

a. Amantadineb. Apomorphinec. Bel ladonnad. Bromocriptinee. Selegi l ine

176. A patient develops profound fever, skeleta l muscle rigidi ty, and autonomic and systemic electrolyte imbalances as part of a severe adverseresponse to a psychoactive drug. The working diagnos is i s neuroleptic mal ignant syndrome. In addition to adminis tering dantrolene in an attemptto restore some semblance of normal skeleta l muscle function, what other drug i s most l ikely to be given to help provide additional symptomrel ief?

a . Benztropineb. Bromocriptinec. Diazepamd. Flumazeni le. Naloxonef. Propranolol

177. A patient diagnosed with depress ion had been taking amitriptyl ine for severa l months . During a vis i t to his phys ician he reported di ffi cul tys leeping, so the MD prescribed trazodone, to be taken once da i ly, at bedtime. The dosages of both the amitriptyl ine and the trazodone wereclearly within the therapeutic range. Severa l weeks later the patient died. The attorney hi red by the deceased man’s patient, as part of a wrongfuldeath sui t, contacted severa l “expert witnesses” who s tated that the amitriptyl inetrazodone combination caused a letha l serotonin syndrome.Based only on the information presented here, what i s your most reasonable conclus ion?

a. An interaction between trazodone and an SSRI, each at proper dosages , i s unl ikely to cause the serotonin syndrome.b. The trazodone-SSRI combination i s absolutely contra indicated, and should never have been prescribed.c. Trazodone would be appropriate only i f the patient were taking a monoamine oxidase (MAO) inhibi tor or for depress ion.d. Sumatriptan should have been prescribed to prevent the development of the serotonin syndrome.

Page 113: Pharmacology pre test self assessment and review (14th ed.)

e. Trazodone should never be prescribed to help patients go to s leep, whether or not they are depressed or are taking antidepressants .

Page 114: Pharmacology pre test self assessment and review (14th ed.)

Central Nervous System Pharmacology

Answers

106. The answer is b. Apologies for s tarting the chapter on CNS pharmacology with a question that defini tely focuses on the autonomic nervoussystem. However, the question i s helpful as yet another way to recognize the connections between ostens ibly di fferent areas of pharmacology anddrug therapy. The description of guanfacine’s autonomic effects are extraordinari ly s imi lar to those of clonidine, which i s genera l ly cons idered theprototype of the “centra l ly acting antihypertens ive drugs .” Clonidine and guanfacine act as postsynaptic α-adrenergic agonis ts (speci fica l ly, α2A; inthe CNS, probably on prefronta l cortica l neurons in terms of the effects on ADHD and anxiety; and in the medul lary “cardiovascular control center”in terms of periphera l autonomic effects . Indeed, the overa l l “dia l ing-down” of sympathetic outflow, which accounts for guanfacine’s mainautonomic action, a lso accounts for i ts much older use as an antihypertens ive drug.

None of the responses described for guanfacine are qual i tatively or mechanis tica l ly s imi lar to those expected with atropine (a) or any otherdrug with s trong muscarinic receptor blocking activi ty. Phentolamine (c), the prototype periphera l ly acting α1- and α2-adrenergic blocker, wouldcause mios is (by unmasking oppos ing parasympathetic influences on pupi l s i ze), a l though as described in the question i t would have no effect onairway smooth muscle (which i s a ffected by drugs that ei ther work di rectly on β2 adrenergic, or muscarinic-chol inergic, receptors ). And, unl ike thes i tuation with guanfacine or clonidine, phentolamine wi l l markedly antagonize vasopressor effects of α-agonis ts such as phenylephrine. Fina l ly,phentolamine would not cause fa l l s of resting heart rate or s troke volume, s ince both of those functions are regulated by activation of β1-adrenergic receptors in various parts of the heart.

Prazos in (d) i s a periphera l ly acting and selective α1 antagonis t. Some of the effects of guanfacine are s imi lar to those of prazos in, but there arethree effects that were noted and that would argue aga inst prazos in being the correct answer: (1) prazos in will indeed block the vasopressorresponses to phenylephrine or any other drug that activates α-adrenergic receptors ; (2, 3) i t wi l l not interfere with β-mediated increases of heartrate or s troke volume, whether mediated by the baroreceptor reflex or otherwise. In contrast, propranolol (d; or any other β-blocker that lacksvasodi lator activi ty) would lower resting heart rate and a lso blunt baroreceptor-mediated increases of heart rate and s troke volume.

107. The answer is e. (Brunton, pp 466t, 527-560; Katzung, pp 40t, 373-381.) Benzodiazepines as a class tend to cause short-term memory loss fol lowingadminis tration. The intens i ty of the effect depends on the unique drug (common and intense with midazolam, which has a fast onset and shortha l f-l i fe; and fluni trazepam [Rohypnol , known on the s treet mainly as “roofies”], for example), the drug’s pharmacokinetics , and the dose andadminis tration route.

The memory loss does not affect reca l l of prior events but rather events occurring shortly a fter the drug i s given. This antegrade amnesia i sva luable in settings such as described in the scenario, where many unpleasant and anxiety-provoking events may occur between preopmedication, the trip to the operating room, and other unpleasantries that might occur before the patient i s anesthetized. (The patient here hadabsolutely no reca l l of events occurring within about 30 seconds after the midazolam was given, up to about an hour after she rega inedconsciousness in the OR. Many patients who receive midazolam for endoscopic procedures such as colonoscopy or bronchoscopy a lso lack reca l l .)

Midazolam intens i fies the overa l l sedative and menta l clouding effects of opioids , but does not potentiate analges ia (a). It does not prevent orreverse hypotens ion (b), and i f the dose i s too large or i t i s adminis tered too quickly, the drug may actua l ly lower blood pressure in i ts own right.Midazolam has some anticonvulsant effects but i t i s seldom used for that purpose (lorazepam is one benzodiazepine that i s routinely used forcerta in seizures , particularly s tatus epi lepticus ). More importantly the question s tated “…seizures l ikely to be caused by propofol ,” but propofolrarely causes seizures (c). Benzodiazepines are not arrhythmogenic nor do they have antiarrhythmic activi ty (d).

108. The answer is a. (Brunton, pp 538t, 541-542; Katzung, pp 438-441.) Hypotens ion and respiratory/venti latory depress ion i s a common s ide effect withpropofol and, indeed, with most other classes of CNS depressants injected for ei ther anesthes ia induction or maintenance. Laryngospasm (b)sometimes and rather uniquely appl ies to injection of rapidly acting barbi turate induction agents (see the next question). Memory loss (c)—speci fica l ly short-term antegrade amnes ia—is associated with certa in benzodiazepines (here, the midazolam, previous question).

Propofol does not cause mal ignant hyperthermia (d), or interact with other medications (those l i s ted in the MAR, or others ) to cause i t.Mal ignant hyperthermia i s mainly l inked, in the surgica l setting, to the combined use of a ha logenated hydrocarbon inhaled anesthetic(particularly ha lothane, the use of which i s decl ining) and a neuromuscular blocker (typica l ly succinylchol ine more so than a curare-l ikenondepolarizing agents ). Propofol does not cause seizures (e); of the injectable induction agents , seizures are mainly a problem with etomidate.Fina l ly, as ide from genera l i zed suppress ion of the CNS, propofol causes no s igni ficant or cl inica l ly relevant effects on skeleta l muscle, and tendsnot to cause nausea or emes is (when used a lone) that i s common with other reasonable a l ternatives .

109. The answer is c. (Brunton, pp 539-540; Katzung, pp 441-442.) Laryngospasm is associated with the barbi turate IV induction agents , whether the drugis an oxybarbi turate (methohexi ta l ) or a thiobarbi turate (thiamyla l , thiopenta l , others ; “thio” refers to a sul fur atom replacing an oxygen atom at acerta in point in the molecule). The mechanism is not known for sure, but the phenomenon is , and the laryngospasm may be sufficient in a smal lnumber of cases to make a i rway intubation di ffi cul t. The barbi turates a lso cause venti latory depressant effects (centra l ly mediated), but in thes i tuation described here that i s not worrisome, because the patient i s receiving mechanica l venti lation.

Barbi turates lack analges ic effects (at blood levels not far lower than letha l ). They can actua l ly increase the sensation of pa in (hypera lges iceffect, a ) when therapeutic doses of ora l agents (mainly phenobarbi ta l nowadays) are given (inappropriately) to patients with pa in and who arenot being treated with true analges ic drugs sui table for the degree of pa in the patient has . This i s one of the reasons why barbi turates—andbenzodiazepines for that matter—should never be used as analges ics . Note, however, that when the fast-acting injectable barbi turates are givenat usual induction/anesthetic doses the hypera lges ic effect does not occur; in addi tion, this patient got other proper analges ics (morphine,ketorolac).

Barbi turates , whether given parentera l ly or ora l ly, and regardless of thei r pharmacokinetics , tend to lower blood pressure (not ra ise i t, answerb; the probable s i tes of action are both centra l and di rectly on the heart). Barbi turates used as IV induction agents tend to lower blood pressures igni ficantly and sometimes excess ively. Mal ignant hyperthermia (d) i s a serious adverse perioperative response that i s associated with thecombined adminis tration of skeleta l neuromuscular blockers (typica l ly succinylcho-l ine) and certa in inhaled ha logenated hydrocarbonanesthetics (eg, ha lothane). Barbi turates do not interact or cause mal ignant hyperthermia in thei r own right. Al l the barbi turates haveanticonvulsant activi ty (dose-dependent and a lso dependent on which barbi turate i s being used), and they do not cause myoclonus or other typesof seizures (e). The higher ri sk of myoclonic seizures i s mainly associated with etomidate, which i s another IV induction agent. (Etomidate-induced seizure ri sk can be reduced by premedication with an opioid or a benzodiazepine.)

Page 115: Pharmacology pre test self assessment and review (14th ed.)

110. The answer is b. (Brunton, pp 529, 967t, 986; Katzung, pp 638t, 642, 655.) Al though ketorolac i s usual ly presented in a chapter on NSAIDs , i t has veryweak (and not cl inica l ly useful ) anti -inflammatory activi ty. However, i t provides excel lent analges ia that has been described as equiva lent to thatprovided by usual s tarting doses of an opioid (eg, morphine) when pa in i s “moderate” and relatively short l ived. The mechanism probably involvesinhibi ted prostaglandin synthes is (as with a l l NSAIDs) and does not involve activation of any opioid receptors . The drug has no antiemetic effects(a); does not affect venti latory rate or depth (c); has no effect on skeleta l neuromuscular transmiss ion or the effects of neuromuscular blockers (d).Unl ike most opioids , ketorolac a lso lacks adverse effects on the bi l iary tract, usual ly making i t a good choice for patients with nephrol i thias is orother conditions that trigger ga l l bladder/bi le duct spasm and pa in.

Ketorolac i s not sui table, a lone, for severe pa in (when an opioid truly i s indicated) nor for chronic pa in. In surgica l settings such as the onedescribed here i t i s used as a supplement to an opioid. What are the advantages of ketorolac, whether used a lone or as a supplement, comparedwith an opioid? Ketorolac’s analges ia i s not accompanied by typica l opioid-l ike venti latory depress ion; hypotens ion; sedation, menta l clouding oreuphoria ; nor a potentia l for dependence or abuse. It i s often used as an opioid supplement precisely because the main a l ternative approach—simply giving a higher dose of morphine—could lead to excess ive venti latory depress ion and hypotens ion.

Despi te the relative lack of anti -inflammatory activi ty, the main adverse responses to ketorolac are s imi lar to those of tradi tional NSAIDs on theGI tract: the potentia l for gastric ulceration, potentia l ly with GI bleeding or hemorrhage; for renal impairment; and for prolonged bleeding due toantiplatelet effects (weak and not therapeutica l ly useful ). These adverse effects are most l ikely to occur with long-term use, which i s whyketorolac should be used only for short-term pain control (no more than 5 days ). Most of the contra indications that apply to tradi tional NSAIDsapply to ketorolac, for example, peptic ulcer disease (active or a ri sk thereof); renal dys function; active bleeding (especia l ly intracrania l );pregnancy; and prior hypersens i tivi ty.

Notes : You should know about ketorolac because i t i s used widely in genera l surgica l settings such as described in the question; inorthopedics ; and in ora l surgery and other settings where “good” short-term analges ia i s needed and i t i s best to minimize or avoid otherunwanted effects of opioids .

When ketorolac was fi rs t approved, many practi tioners used i t frequently and prescribed i t for chronic pa in control . (Some described prescribingthe drug “l ike candy.” After a l l , i t provided excel lent pa in control with no opioid-l ike problems.) Not long thereafter prescribers rea l i zed the s trongulcerogenic actions of the drug, and so new guidel ines and warnings were made to restrict use of the drug to no more than severa l days in a row.

111. The answer is e. (Brunton, pp 1341-1343; Katzung, pp 287, 1098-1099.) Ondansetron, l ike a related drug, dronabinol , i s used to manage drug-induced nausea and vomiting that can occur postoperatively or in response to emetogenic drugs (eg, many anticancer drugs). It i s a serotoninreceptor (5-HT3) blocker that acts mainly in the medul lary chemoreceptor trigger zone and on vagal a fferents (inhibi ted) to parts of the upper GItract.

Ondansetron has no genera l CNS or cortica l s timulating effects that would help restore consciousness , a lertness , or awareness of time andplace (a); no analges ic-intens i fying effects (b); no s igni ficant effects on the bladder (c); and does not reduce the ri sk of para lytic i leus (d) becausevagal tone to intestina l smooth muscles i s suppressed.

112. The answer is d. (Brunton, pp 540-542, 546-547; Katzung, pp 431-436.) MAC (minimum a lveolar concentration) i s an express ion of inhaled anesthetic“potency.” It i s defined as the minimum inspired concentration needed to abol i sh a speci fied pa inful response in 50% of treated patients . (Thus ,i t i s much l ike the ED50, measured in a population dose-response curve, for most other drugs .) Obvious ly, giving a drug at a dose that suppresses aresponse in only ha l f the treated patients i s not des i rable, so inhaled anesthetics are typica l ly given at a dose more than the MAC. (Reca l l , too,that MAC is not absolute: i t can change depending on the use of other anesthes ia adjuncts and such other factors as body temperature, venti latoryrate, presence of other diseases , etc.)

A drug’s MAC gives us no useful and invariable information about onsets or durations of action (a , e). You cannot s tate correctly that drug Y (MAC≈ 100%) causes greater analges ia and/or skeleta l muscle relaxation than another drug any more than we can say that a drug with an ED50 of 5 mgcauses a greater response than one with an ED50 of 1 mg. It a l l depends on the dose given, not the MAC or ED50.

113. The answer is d. (Brunton, pp 398, 408, 614t, 618; Katzung, pp 488f, 490, 499.) Two of the important types of MAO are: (1) MAO-A, which metabol i zesnorepinephrine and serotonin and other biogenic amines , and i s the predominant hepatic form of the enzyme; and (2) MAO-B, which metabol i zesdopamine and other monoamines in the bra in. Selegi l ine i s a selective inhibi tor of MAO-B. It therefore inhibi ts the breakdown of dopa-mine andprolongs the therapeutic effectiveness of levodopa (endogenous or that provided pharmacologica l ly) in parkinsonism. The ri sks of serious druginteractions in patients taking nonselective MAO inhibi tors (eg, phenelzine, tranylcypromine), such as hypertens ive cri s i s in response to mixed-and indirect-acting sympathomimetics (tyramine, pseudoephedrine, and amphetamines) are much less (but s ti l l poss ible) with selegi l ine.

Bromocriptine (a) i s a dopamine receptor agonis t. Carbidopa (b) inhibi ts the periphera l metabol i sm of levodopa by DOPA decarboxylase. Bothare useful in the treatment of some cases of idiopathic (but not antipsychotic drug-induced) parkinsonism. Phenelzine (c) and tranylcypromine (e)are nonselective MAOIs . Combining them with L-dopa may lead to a potentia l ly fata l hypertens ive cri s i s , and thus they are not used in the therapyof parkinsonism. A s imi lar interaction may occur with tyramine-rich foods and beverages ; and with catecholamine-releas ing sympathomimetics(ephedrine, pseudoephedrine, amphetamines , and amphetamine-l ike drugs). As a resul t of these common and potentia l ly letha l interactions ,non-selective MAO inhibi tors are rarely used for anything unless no other usual ly effective therapies work adequately or are otherwisecontra indicated.

114. The answer is d. (Brunton, pp 536-537, 614t; Katzung, pp 485, 499.) When levodopa i s adminis tered ora l ly, the vast majori ty of the adminis tereddose (about 90%) i s metabol i zed in the gut to dopamine by DOPA decarboxylase. However dopamine cannot cross the blood-bra in barrier, and soonly a fraction of the parent drug gets into the CNS, to be metabol i zed and cause i ts des i red effects there. Carbidopa inhibi ts DOPA decarboxylasein the periphery, reducing periphera l metabol i sm of levodopa to dopamine and “sparing” a bigger fraction of the adminis tered levodopa dose soi t can be metabol i zed in the nigrostriatum. By reducing periphera l convers ion of levodopa to dopamine, the adjunctive use of carbidopa oftenenables management of parkinsonian s igns and symptoms with doses of levodopa much lower than would be needed in the absence ofcarbidopa. One additional benefi t of that i s a reduction in the number and severi ty of periphera l s ide effects of the levodopa (or, more precisely,i ts metabol i te, dopamine).

Adding carbidopa to a regimen involving levodopa only may a lso help (at least trans iently) combat such problems as dopamine’s “on-offphenomenon” and “end-of-dosefailure.” The on-off phenomenon i s characterized by frequent, abrupt, and dramatic changes between what appears tobe good symptom control to episodes that look l ike parkinsonian s igns and symptoms weren’t improved at a l l (much l ike what happens whenswitching the l ights on and off). With end-of-dose fa i lure, symptom control between doses gets progress ively less progress ively sooner. Forexample, a patient who had good symptom control up to an hour or so before the next scheduled levodopa dose may gradual ly experience loss ofthat control 2 hours , 3 hours , or even much more, before they were to take their next dose.

Carbidopa, whether cons idered in i ts own right or in the context of coadminis tration with levodopa has no effects (inhibi tory or s timulatory) on

Page 116: Pharmacology pre test self assessment and review (14th ed.)

ACh release (a). Vi tamin B6 deficiencies of the vi tamin do not occur in response to therapy with ei ther or both drugs (b); and i t i s important to notethat ingesting large amounts of that vi tamin, whether as part of the diet or in the form of supplements , wi l l antagonize the des i red effects oflevodopa with or without added carbidopa. Carbidopa does give “better access” of levodopa to the CNS, but that effect has nothing to do withincreas ing blood-bra in barrier permeabi l i ty to dopamine (c). It i s due, as noted, to increased concentrations of levodopa in the ci rculation due toinhibi ted periphera l convers ion to dopamine. Carbidopa has no beneficia l effects on dopamine-induced blood pressure changes , and has noabi l i ty to block dopamine receptors in the vasculature (e) or anywhere else.

115. The answer is e. (Brunton, p 349; Katzung, pp 283-285, 374, 376, 387.) Buspirone i s an attractive drug for managing mi ld short-term anxiety. Amongthe reasons (and especia l ly when compared with more tradi tional anxiolytics , such as benzodiazepines) are a lack of sedation (buspirone i s not aCNS depressant); very l i ttle (i f any) potentiation of the effects of other CNS depressants , including a lcohol ; no known abuse potentia l (i t i s notregulated by the Control led Substances Act) or tendency for development of tolerance; and no major withdrawal syndrome. One major drawback i sa s low onset of symptom rel ief (a week or two), and typica l ly i t takes about a month from the onset of therapy for antianxiety effects to s tabi l i ze.(Knowing this s low onset, one should res is t the temptation to ti trate the dosage upward, to hasten or increase the drug’s effects , prematurely.)

You should reca l l that long-term benzodiazepine adminis tration i s associated with withdrawal phenomena (and, depending on the use, dose,exact drug, and other patient-related factors , the syndrome can be severe). Thus , one can envisage a switch from a benzodiazepine to buspirone.Because buspirone lacks CNS depressant effects and i ts effects take some time to develop, one should s tart the buspirone severa l weeks beforestopping the benzodiazepine and then taper the benzodiazepine dose once i t’s time to s top the drug.

116. The answer is c. (Brunton, pp 593-594; Katzung, p 411.) Phenobarbi ta l ’s role in the primary management of a l l of the named conditions has waneddramatica l ly, and appropriately, as newer drugs , with fewer ri sks of drug interactions and tolerance development, have come on the market. Theonly use in the options provided in the question, for which phenobarbi ta l i s s ti l l cons idered reasonable for long-term therapy in some patients , i sfor managing certa in epi leps ies (mainly as an a l ternative to phenytoin). The seizure types include, mainly, partia l seizures and genera l i zed tonic-clonic seizures . Benzodiazepines are a lmost a lways preferred, and used, for managing a lcohol withdrawal (a ) and therapy of anxiety (b).Benzodiazepines , or the benzodiazepine-l ike agents za leplon or zolpidem, are a lmost a lways turned to for insomnia (e). Phenobarbi ta l—andother barbi turates for that matter—have no beneficia l effects on depress ion s igns and symptoms (d), and they are not properly used a lone or incombination with an SSRI (or tricycl ic or atypica l antidepressant).

Major reasons for selecting a benzodiazepine over a barbi turate include fewer drug-drug interactions (phenobarbi ta l i s a class ic P450 inducerthat hastens the el imination of many other drugs that are metabol i zed by the P450 system); a lesser ri sk of genera l i zed ri sk of CNS depress ion(somnolence, etc); lower ri sk of dependence; withdrawal syndromes that are typica l ly less severe or dangerous ; lower ri sk of fata l venti latorydepress ion with ora l overdoses (and the ava i labi l i ty of flumazeni l , the speci fic benzodiaze-pine antagonis t, to treat them); and less narrowingbetween letha l and effective doses (better therapeutic index or margin of safety) as use continues .

Note that even though benzodiazepines may be preferred, treatment for such problems as anxiety and insomnia should be kept as short asposs ible.

117. The answer is a. (Brunton, pp 225-235, 400-403; Katzung, pp 526-527, 531-534, 536-537.) The most common s ide effects associated with tri -cycl icantidepressants are thei r antimuscarinic effects , which may occur and often be prominent in over 50% of patients . Cl inica l ly, these effects maymani fest as dry mouth, blurred vis ion, constipation, tachycardia , dizziness , and urinary retention. These are, of course, some of the typica l andcommon s ide effects associated with atropine or any other drug with substantia l antimuscarinic effects . At therapeutic plasma concentrationsthese drugs usual ly do not cause changes in the ECG (b)—but with severe overdoses letha l arrhythmias often do occur. These drugs are notparticularly hepatotoxic or nephrotoxic (c, d); seizures are not l ikely to occur unless the patient has a his tory of seizure disorders or mass iveoverdoses have been taken.

Note: In terms of tricycl ic toxici ty, ie, wi th overdoses , you might want to reca l l what some students learn as the “Three Cs :” coma, cardiotoxici ty(mainly arrhythmias that are often di ffi cul t to treat), and convuls ions . In addition, a l l the other mani festations of “atropine poisoning” wi l ldevelop and usual ly be severe. This i s an extremely di ffi cul t s i tuation to treat, and to treat success ful ly, when the cause i s a tricycl icantidepressant.

118. The answer is b. (Brunton, pp 356-357, 611, 614t; Katzung, pp 501-520.) Unwanted extrapyramidal s ide effects produced by antipsychotic drugs (eg,chlorpromazine, as the exemplar of the phenothiazines ; and, more so, by ha loperidol as the prototype of the butyrophenones) include Parkinson-l ike syndrome, akathis ia , dystonias , ga lactorrhea, amenorrhea, and inferti l i ty. These s ide effects are due to the abi l i ty of these agents to blockdopamine receptors . The phenothiazines a lso block muscarinic and α-adrenergic receptors , which are respons ible for other effects in both the CNSand (especia l ly) in the periphery. The incidence and severi ty of these autonomic s ide effects i s much greater with low-potency antipsychotics (eg,chlorpromazine and other phenothiazines) than with the high-potency butyrophenones (eg, ha loperidol ).

119. The answer is b. (Brunton, pp 542-544; Katzung, pp 444-446, 550.) The scenario describes most of the class ic responses to ketamine, a “dissociativeanesthetic”: ana lges ia ; an ostens ibly l ight s leep-l ike s tate; a trancel ike and cataplectic s tate (including increased muscle tone); and activation ofmost cardiovascular/hemodynamic parameters . The various psychos is -l ike emergence reactions are the main disadvantages to us ing a drug that,otherwise, causes many of the des i red elements of ba lanced anesthes ia , usual ly without the need for compl icated and expens ive anesthes iaadminis tration devices or personnel . Ketamine undergoes s igni ficant metabol i sm in humans , with about 20% of the absorbed dose recovered asmetabol i tes . The only other drug l i s ted that provides adequate analges ia i s fentanyl (a ). Midazolam (c; benzodiazepine), succinylchol ine (d;depolarizing neuro-muscular blocker), and thiopenta l (e; thiobarbi turate) lack analges ic activi ty; moreover, i f any cardiovascular or autonomicchanges were to occur in response to any of those drugs , they would be better characterized as depress ion, not activation.

120. The answer is b. (Brunton, pp 591-593, 845, 1012, 1524, 1528, 1873t; Katzung, pp 40t, 407-409, 427.) The hepatic enzymes respons ible for phenytoinmetabol i sm (mainly CYP2C9) become saturated at plasma drug concentrations above approximately 10 to 15 mcg/mL, which are clearly within thetypica l therapeutic range yet wel l below maximum or peak therapeutic levels . At da i ly dosages associated with or below those 10 to 15 mcg/mLvalues , dose increases give relatively proportional increases in plasma drug concentrations , and phenytoin el imination fol lows usual fi rs t-orderkinetics (a constant fraction of drug i s el iminated with the pass ing of each ha l f-l i fe). Once the metabol ic capaci ty i s exceeded (as i t has in ourpatient, which may arise with intentional or inadvertent increases in the da i ly dose), smal l increases in dosage lead to disproportionately largeincreases of plasma concentrations (and effects ) because, in essence, “drug in greatly exceeds drug out:” zero-order kinetics now describes thedrug’s el imination, such that a constant amount (not fraction) of drug i s el iminated per uni t time. Therefore, the fi rs t-order ha l f-l i fe does notapply; el imination i s s lower, and the plasma concentration wi l l be much greater than 15 mcg/mL (ha l f of 30 mcg/mL; ans . e) 24 hours after theini tia l blood sample i s taken.

The figure below shows an approximation of the relationship between plasma phenytoin levels and da i ly doses (the placement of the curve can

Page 117: Pharmacology pre test self assessment and review (14th ed.)

vary from patient to patient a long the x-axis ). From the rapid ri se in the curve at dosages above about 300 mg/day you can deduce a s igni ficantreduction of clearance rates owing to s lowed metabol i sm (s ince metabol i sm is the main pathway for phenytoin’s el imination).

Answer a i s incorrect; flumazeni l i s a benzodiazepine receptor antagonis t (competi tive blocker) that has no effects (beneficia l ly or not;pharmacokinetic or otherwise) on the el imination or effects of phenytoin. It would be inappropriate—and dangerous—to give an amphetamine orany other CNS s timulant to counteract the excess CNS depress ion. Ti trating upward the dose of a CNS s timulant to combat CNS depress ion (whethercaused by a drug or from another cause) i s a ri sky endeavor, due to the chance of inducing seizures from excess ive CNS s timulation, and the ri sksare far greater in a person with a his tory of seizure disorders .

Phenobarbi ta l (d) i s a class ic example of a P450 inducer, and indeed i t i s metabol i zed mainly by CYP2C9, on which phenytoin’s el imination i smainly dependent. In theory, giving phenobarbi ta l might increase phenytoin’s metabol i sm via P450 induction. In rea l i ty, the pharmacokineticoutcomes of a phenobarbi ta l -phenytoin interaction are qui te variable (and dependent on blood levels of each drug), but the more l ikelyconsequence i s inhibi ted phenytoin el imination as the barbi turate competes with i t for convers ion by the same cytochromes. Regardless , addingphenobarbi ta l i s l i kely to add to the genera l i zed CNS depress ion caused by the phenytoin, and compl icate both the cl inica l picture and i tsmanagement. Therefore, this approach, too, would be inappropriate.

121. The answer is a. (Brunton, pp 299-300; Katzung, pp 142, 146, 1032-1034.) Methylphenidate, which i s widely used to manage ADHD, hasamphetamine-l ike sympathetic and CNS-stimulating effects . Indeed, amphetamine i tsel f (and derivatives or mixtures of amphetamine) i s used fortreatment of ADHD.

Periphera l sympathetic (adrenergic) effects ari se from the drug’s abi l i ty to release neuronal norepinephrine. Related findings would includeincreased vasoconstriction (greater α1-receptor activation of vascular smooth muscle), and therefore increased blood pressure; and tachycardia(plus increased cardiac contracti l i ty and electrica l impulse conduction rates , probably accompanied by tachyarrhythmias , from β1 activation).

At toxic doses , the CNS effect of the drug i s l ikely to cause seizures .There i s no mechanism by which di rect effects of methylphenidate or amphetamine (whether at normal or toxic doses) would cause

bronchospasm, bronchoconstriction, or even bronchodi lation (because norepinephrine that i s released by those drugs has no abi l i ty to activate orblock β2 receptors on a i rway smooth muscle). Mios is (which occurs with ei ther muscarinic activation or α blockade) would be the oppos i te of what

Page 118: Pharmacology pre test self assessment and review (14th ed.)

you would predict in terms of ocular effects .In terms of autonomic effects , skeleta l muscle weakness or para lys is are findings you would expect from drugs that ei ther blocked nicotinic

chol inergic receptors (eg, tubocurarine [no longer on the market in the United States ], but related agents such as vecuronium are) or s timulatedthem in an excess ive and prolonged fashion (eg, succinylchol ine).

122. The answer is d. (Brunton, pp 503-506; Katzung, pp 551-552, 557, 562.) High or frankly toxic plasma levels of meperidine can cause seizures ,hypertens ion, and a psychos is -l ike s tate, in addi tion to typica l morphine-l ike effects (eg, ana lges ia and venti latory depress ion). Interestingly, i tappears that adminis tration of na loxone to combat excess ive effects of meperidine may increase the ri sk of seizures . This i s something that i sextraordinari ly rare when na loxone i s adminis tered to counteract excess ive effects of morphine and most other opioids .

The rather unique adverse effects associated with meperidine are probably due to a major metabol i te, normeperidine. A weak opioidanalges ic, propoxyphene (s tructura l ly related to methadone), can produce toxic reactions s imi lar to those of meperidine. It i s off the United Statesmarket now, largely because of the ri sks and the drug’s overa l l poor efficacy. Nonetheless , the ri sks described here account for why meperidinedoses should be kept as low as poss ible, and the duration of treatment with i t should be kept short (usual ly no more than a day or two).

Meperidine, l ike other opioids , can cause constipation (a) through i ts actions on longi tudina l muscles and sphincters in the GI tract. Hyper-a lges ia (b), an increased perception of pa in, does not occur. Bi l iary tract/sphincter of Oddi spasm (c) i s much less l ikely to occur with meperidinethan with most other opioids (say, the prototype, morphine). Indeed, the ri sks of bi l iary tract/ga l l bladder problems are sa id to be less withmeperidine than any other opioid analges ic. And so, meperidine can be cons idered a preferred opioid (for short-term use) for patients withmoderate-to-severe pa in and ga l l bladder disease.

Respiratory depress ion (e), poss ibly leading to venti latory arrest, i s a property shared by a l l the opioid analges ics , but of course to varyingdegrees based on their efficacy and dose.

Note: Before the unusual ri sks of meperidine were ful ly appreciated, the drug was often prescribed for days or weeks on end. That should neverbe done anymore.

123. The answer is a. (Brunton, pp 359, 422, 424t, 436-442, 920, 924; Katzung, pp 504-509, 511-512.) In genera l , autonomic s ide effects are fa i rly commonwith phenothiazines such as chlorpromazine. These include muscarinic receptor blockade leading to dry mouth, blurred vis ion; and a l l the rest ofthe unwanted effects caused by atropine, or a drug with s trong atropine-l ike actions . Ha loperidol , the butyrophenone antipsychotic drug, tends tocause extrapyramidal s ide effects more often, more s trongly, than autonomic s ide effects noted above. As a resul t, extrapyramidal (parkinsonian)s ide effects are more common. Indeed, when ha loperidol i s given by injection to help treat acute psychos is many treated patients develop(temporari ly) muscle tremors and other mani festations of Parkinson disease.

124. The answer is d. (Brunton, pp 664-665; Katzung, pp 569t, 575-577.) “Angel dust” i s phencycl idine (PCP), an ha l lucinogen or psychotomimetic with anamphetamine-l ike mechanism of action. Thus , the problems that ari se are due to blockade of neuronal reuptake of such monoamines asdopamine, norepinephrine, and serotonin. The drug does not have centra l or periphera l muscarinic receptor antagonis t activi ty (a ), nor opioidagonis t actions (c). A phencycl idine withdrawal syndrome (b) has not been documented cons is tently humans , largely because i t i s usedepisodica l ly and not chronica l ly present in the ci rculation. In overdose, the treatment of choice for mani festations of drug-induced psychos is i sha loperidol , a l though rapidly acting and highly sedating benzodiazepines may be used instead. Flumazeni l (e) i s a benzodiazepine receptorantagonis t with no use or efficacy in phencycl idine toxici ty.

125. The answer is b. (Brunton, pp 446-447, 448-450, 976; Katzung, pp 514-517.) In essence, the intens i ty of effects from any given plasma concentrationof l i thium is inversely related to sodium concentrations . When sodium concentration fa l l s , the effects of l i thium can be intens i fied to the point ofcaus ing toxici ty. By way of thei r actions to increase renal sodium loss , vi rtua l ly a l l the common diuretics a lso reduce renal l i thium excretion. It i sprobably of greatest concern with thiazides or thiazide-l ike diuretics , which have the greatest potentia l of a l l the common diuretics to causehyponatremia. Cholestyramine (a) i s a cholesterol -binding res in. Given concomitantly with an ora l diuretic, the most l ikely outcome would bereduced bioava i labi l i ty of the diuretic, not increased or decreased cholestyramine effects . Ni fedipine (c), a dihydropyridine ca lcium channelblocker, may cause excess ive antihypertens ive effects when adminis tered with any diuretic (a l though combined adminis tration i s common andusual ly problem-free). If excess ive effects of ni fedipine occur in conjunction with a diuretic, they are most l ikely blood volume-related, notspeci fica l ly due to changes of sodium concentrations . Phenylephrine’s (d) vasoconstrictor effects are not l ikely to be a l tered appreciably in thepresence of a diuretic. The same lack of a cl inica l ly s igni ficant interaction appl ies to the s tatins (e; HMG Co-A reductase inhibi tors , eg,atorvastatin).

126. The answer is d. (Brunton, pp 405, 408, 412, 1538; Katzung, pp 285-286, 1162t.) This patient probably has the serotonin syndrome. Serotonin i sa l ready present in increased amounts in synapses because of blockade of i ts reuptake by the SSRIs . When sumatriptan (or other triptans used formigra ine therapy; they are 5-HT1B/2D agonis ts ) i s added, rapid accumulation of serotonin and/or the triptan in the bra in can occur.

The ri sk of the serotonin syndrome in SSRI-treated patients i s much higher when MAO inhibi tors are used concomitantly. Nonetheless , suchsevere reactions from an SSRI-triptan interaction have been reported. In addition, do not forget that MAO inhibi tors can a lso cause an acute andpotentia l ly fata l hypertens ive cri s i s when co-adminis tered with tricycl ic/tetracycl ic antidepressants (eg, imipramine); such combined use shouldbe avoided.

None of the other drugs l i s ted (codeine, a ; loratadine or another second-generation antihis tamine, b; midazolam or fentanyl—or otherbenzodiazepines or opioids—c; or zolpidem or the related drug za leplon, e) interact with fluoxetine or other SSRIs in terms of speci fica l ly ra is ingsystemic or synaptic serotonin levels .

127. The answer is b. (Brunton, pp 86t, 468-469, 1059; Katzung, pp 381-382, 1033t, 1037.) Flumazeni l i s a competi tive antagonis t of benzodiazepines atthe GABA receptor. Repeated adminis tration i s necessary because of i ts short ha l f-l i fe relative to that of most benzodiazepines—especia l lydiazepam, which forms many long-lasting active metabol i tes .

Dextroamphetamine (a) i s a CNS s timulant that wi l l increase the patient’s level of consciousness , but i t i s not indicated for this use. High dosesprobably be needed to overcome the intense benzodiazepine effects , and they may cause dangerous CNS and cardiovascular responses ratherthan s imply “normal izing” variables depressed by the diazepam. Nal trexone (c) speci fica l ly antagonizes the effects of opioid agonis ts on μ and κreceptors and i s used to diagnose or treat opioid overdoses ; i t has no benzodiazepine receptor-blocking activi ty. Physostigmine (d) i s a l ipophi l i cacetylchol inesterase inhibi tor that may be a l i fe-saving intervention for severe poisoning with many drugs (a key exception being tricycl icantidepressants ) having s igni ficant antimuscarinic (atropine-l ike) actions . Pra l idoxime (e) i s a chol inesterase reactivator used in the adjunctivemanagement of poisoning with such “i rrevers ible” chol inesterase inhibi tors as the organo-phosphate insecticides and many “nerve gases”(soman, sarin, VX).

Page 119: Pharmacology pre test self assessment and review (14th ed.)

128. The answer is a. (Brunton, pp 438, 614t, 619; Katzung, p 491.) This cutaneous response, ca l led l ivedo reticulari s , i s characteris tica l ly associatedwith amantadine. Reca l l that this seldom-used antiparkinson drug probably works by releas ing endogenous dopamine and blocking i ts neuronalreuptake. Livedo reticulari s i s not associated with levodopa (used a lone or with carbidopa; c or d), nor with the dopamine agonis ts bromocriptine(d) or pramipexole (e; a newer and genera l ly preferred drug for s tarting treatment of mi ld parkinsonian s igns and symptoms). (You might a lsoreca l l that amantadine i s a lso used for prophylaxis of some stra ins of influenza vi rus infections . The mechanism of action in this setting i s notwel l understood.)

129. The answer is d. (Brunton, pp 591-593; Katzung, pp 407-409.) For many years , phenytoin has been ci ted as a class ic example of a drug that cancause gingiva l hyperplas ia . (Among ora l surgeons and dentis ts , i t i s often referred to as Di lantin® hyperplas ia , in recognition of phenytoin’scommon proprietary name.) The mechanism is unknown, but we suspect the drug a l ters col lagen metabol i sm in the gingiva l ti s sues . Al thoughphenytoin i sn’t the only drug associated with potentia l gingiva l hyperplas ia (verapami l i s another common one), i t i s the only one l i s ted as achoice. Note that phenobarbi ta l , which i s s ti l l used as an a l ternative to phenytoin for seizures , does not cause gingiva l hyperplas ia . It i s preciselybecause of the low gingiva l hyperplas ia ri sk that phenobarbi ta l i s sometimes prescribed for chi ldren with respons ive seizure disorders in l ieu ofphenytoin.

130. The answer is d. (Brunton, pp 344-347, 832; Katzung, pp 285, 292.) The serotonin receptor agonis t actions of the “triptans ,” including the prototype,sumatriptan, can trigger intense vasoconstriction in various vascular beds . The cerebra l vasoconstrictor effects of these drugs contributeimportantly to the rel ief they afford in migra ine headaches . However, coronary vasospasm can occur a lso; i t may be particularly intense andpotentia l ly l i fe-threatening in patients with coronary vasospastic disease (ie, variant or Prinzmeta l angina), for whom the triptans arecontra indicated.

None of the other drugs l i s ted have any s igni ficant coronary (or other) vasoconstrictor effects . Indeed, some such as phenytoin and zolpidem (abenzodiazepine-l ike hypnotic) may cause s l ight cardiovascular changes (eg, a fterload reduction) that would actua l ly reduce the degree of myocar-dia l i schemia and the ri sk of angina.

131. The answer is c. (Brunton, pp 551-552; Katzung, pp 430-438.) Reca l l that ni trous oxide has a MAC (minimum a lveolar concentration) of about 105%.Achieving that concentration in an inspi red gas mixture i s phys ica l ly imposs ible at normal atmospheric pressure. Even i f we could safely give pureni trous oxide (we obvious ly cannot), and we rounded-off the MAC to 100%, the drug would (1) cause abol i tion of a pa inful response in only 50% ofsubjects (this i s one element of the defini tion of MAC), and (2) be letha l , s ince 100% ni trous oxide means no (0%) oxygen at normal atmosphericpressures . This very effective analges ic gas i s poorly soluble in the blood, so equi l ibration between a lveolar and blood concentration and theonset of i ts centra l effects are qui te rapid.

Even at the usual concentrations (typica l ly 80% of the inspi red gas mixture), and whether used only with a i r (as in many aspects of denta lpractice) or with other inhaled agents (common in surgery), ni trous oxide causes l i ttle or no bronchospasm nor s igni ficant cardiac depress ion.

132. The answer is b. (Brunton, pp 86t, 437t; Katzung, pp 503-512.) The question described common and necessary interventions for managingneuroleptic mal ignant syndrome, which i s characteris tica l ly associated with older antipsychotics—chlorpromazine and other “low potency” anti -psychotics to a degree, but occurs more so with ha loperidol (butyrophe-none). Signs and symptoms include muscle tetany/rigidi ty, profound fever,rapid swings of heart rate, rhythm, blood pressure (autonomic instabi l i ty), electrolyte abnormal i ties , and dehydration. None of the other agentsare associated with this syndrome, whether acutely or with long-term therapy or at therapeutic or toxic plasma levels , nor would the interventionsdescribed be indicated.

133. The answer is c. (Brunton, pp 356-357, 614; Katzung, pp 487, 489f, 496, 499.) This i s a question about a drug you may not have learned aboutexpl ici tly, but given the description of i ts mechanism, which should be qui te fami l iar, you should have no problem arriving at the right answer.Ropini role (a related drug i s pramipexole) di rectly activates s triata l dopamine receptors . It i s one of severa l “dopaminergic” drugs that can beused to ini tiate therapy of early Parkinson disease. (With the ava i labi l i ty of ropini role, pramipexole, and some related drugs , we now hold-off onprescribing such older drugs as levodopa when parkinsonian s igns and symptoms are in the early s tages). Dyskines ias and hypotens ion arerelatively common s ide effects , as a lso appl ies to a l ternatives such as levodopa and bromocriptine. Ropini role a lso causes drows iness ors leepiness ; ei ther would render the drug not idea l for managing daytime anxiety (a), but anxiety management i s not an indication for the drug, noris hypersomnia (b). Ropini role’s dopaminergic action may exacerbate s igns and symptoms of schizophrenia (d); l i ke many other antiparkinsondrugs i t may lower the seizure threshold in patients with epi lepsy, and so would be i rrational for managing s tatus epi lepticus (e). It has nocl inica l ly useful ana lges ic activi ty (f).

134. The answer is b. (Brunton, pp 583-606, 1859t; Katzung, pp 377t, 420, 423, 439t.) Intravenous lorazepam is genera l ly regarded as the drug of choice forini tia l treatment of s tatus epi lepticus . (It has surpassed diazepam because of a faster onset of action and less venous i rri tation, among otherthings .) Lorazepam, l ike the benzodiazepines in genera l , increases the apparent affini ty of the inhibi tory neurotransmitter GABA for binding s i teson bra in cel l membranes . This increases the frequency of chloride channel opening, which then leads to inhibi ted target nerve activi ty.

It i s essentia l to note that IV lorazepam’s anticonvulsant effects are relatively short l ived; i f only lorazepam was given for s tatus the seizureswould s top quickly (a lmost immediate onset of action), but there i s a great ri sk of seizure reappearance as the drug i s el iminated quickly andblood concentrations fa l l (too short a duration of action). Therefore, immediately after giving the lorazepam ei ther phenytoin or fosphenytoinshould be given to provide longer seizure suppress ion and “coverage” because lorazepam’s effect wanes so quickly. Compared with lorazepam,phenytoin’s onset of action i s too s low, but the duration of action i s much longer.

Why not just give phenytoin or fosphenytoin and skip the lorazepam? Reca l l that the longer s tatus epi lepticus pers is ts , the greater the ri sk ofbra in damage or death. Our goal i s to terminate the seizures as fast as poss ible. As noted above, phenytoin or fosphenytoin have onsets of actionthat aren’t fas t enough to achieve the goal of prompt seizure suppress ion. Lorazepam works quickly, and that’s why we give i t fi rs t.

None of the other drugs l i s ted in the question are appropriate for ini tia l therapy of s tatus epi lepticus , despi te thei r widespread use for ora ltherapy of seizure disorders long-term.

135. The answer is e. (Brunton, pp 564-565, 571, 574; Katzung, pp 449-464.) Fi rs t, a learning “trick” for how to tel l whether a loca l anesthetic (one you’velearned about, or not) i s an ester or amide. Look at the drug’s generic name; i f there are two occurrences of the letter “i” in i t, i t’s an amide; i f not,i t’s an ester. Thus , we have such esters as coca ine, chlor(o)proca ine, proca ine i tsel f, and tetraca ine, which i s the correct answer for this question.The others I l i s ted are amides .

This may seem l ike a picky question, or perhaps a tricky one, i f you’ve been taught the main properties of only one or two prototypic loca lanesthetics—eg, l idoca ine as the prototype amide and prototype loca l anesthetic overa l l , proca ine as the prototype of the ester class—andperhaps a thi rd drug, benzoca ine, an ester that i s sui table only for topica l adminis tration because i t i s largely insoluble and can’t be given safely

Page 120: Pharmacology pre test self assessment and review (14th ed.)

or effectively by other routes . Nonetheless , there are a couple of important points here, and they revolve around a fundamenta l di fferencebetween amides and esters , and so knowing the class to which a particular loca l anesthetic belongs : (1) a l lergic reactions of various severi tiesare more common with esters than with amides ; (2) there i s classe-based cross -reactivi ty, meaning that i f a patient has had a true immunologicreaction to any ester, he or she i s at ri sk for a s imi lar reaction from subsequent exposure to any ester; and (3) there i s no cross -reactivi ty betweenesters and amides , such that use of an amide in an “ester-sens i tive” patient i s not l ikely to pose problems of a l lergenici ty.

As a fina l as ide, i f you ask a group of people “how many of you have received Novocain before?” you’l l get a lot yes responses . And most ofthem wi l l be incorrect. Novocain i s the most widely known brand name for the generic drug, proca ine. Truth i s , proca ine i s such a poor loca lanesthetic (low potency, short duration, a l lergenici ty, etc) in comparison with drugs l ike l idoca ine that i t’s a lmost never used nowadays (notava i lable in the United States ), in fact hasn’t seen much use for decades . Before then, however, i t was bas ica l ly the only choice when aninjectable loca l anesthetic was needed, and the name Novocain pers is ted as a ubiqui tous (but incorrectly appl ied) name for many loca lanesthetics that came out later. If a patient younger than about 40 years old reports a Novocain a l lergy, be careful how you interpret thatinformation. Chances are good that someone inappropriately told them they were getting Novocain when the drug was actua l ly something else.And be sure that i f you’re going to give someone a dose of a loca l anesthetic you don’t ca l l i t Novocain when, in fact, i t’l l surely be something else.Names matter.

136. The answer is c. (Brunton, pp 252-253, 621-622; Katzung, pp 509, 1054-1055.) Ini tia l recommended therapy includes use of acetylchol inesteraseinhibi tors (donepezi l , tacrine, ga lantamine, rivastigmine; a l l are revers ible chol inesterase inhibi tors ). These drugs are not cures , but they appearto s low symptoms of bra in neurodegeneration in some patients . None appears to be invariably superior to any other. Their actions a lmostcerta inly involve activation of centra l muscarinic receptors—indirectly, by inhibi ting metabol i sm of ACh released from viable nerves , of course—inpart because drugs with centra l antimuscarinic actions reduce their efficacy. Tacrine i s noteworthy, compared with the other drugs noted in theanswer, because i t causes a higher incidence of chol inesterase inhibi tion in the periphery: more and more dis turbing symptoms typica lcharacteris tic of increased (and indirect, due to chol inesterase inhibi tion) muscarinic effects on the gut, bladder, a i rways , etc.

Activating serotonin receptors (a) or blocking dopaminergic effects (b) are not sui table or effective therapies for Alzheimer. Selegi l ine (MAO-Binhibi tor, d) and high doses of vi tamin E (antioxidant) apparently s low neurodegenerative processes , but so far there are no convincing data thatthey s low cognitive decl ine, and they are not fi rs t-l ine therapies . Thrombolytic drugs (e) are not indicated, as the s igns and symptoms of Alzheimerare not due to clots or other vaso-obstructive causes .

137. The answer is b. (Brunton, p 234; Katzung, pp 492t, 499.) Reca l l that in idiopathic parkinsonism the s igns and symptoms genera l ly reflect animbalance between the oppos ing effects of dopamine and of ACh in the bra in’s s triatum. More speci fica l ly, dopaminergic influences are, or atleast appear to be, outweighing the effects of muscarinic chol inergic activi ty. There are two bas ic ways to manage this : (1) enhance dopaminergicactivi ty; and/or (2) diminish the centra l muscarinic influences . Trihexyphenidyl (and the related drug benztropine) do the latter. They causes igni ficant antimuscarinic (atropinel ike) actions in the CNS—hence, these drugs have been class i fied as “centra l ly acting antimuscarinics .” Thesedrugs are qui te l ipophi l i c and enter the CNS wel l (this accounts for the drug’s marked sedative effects , too) to block muscarinic receptors there.That helps adjust the dopamine-ACh imbalance that appears to be a main biochemica l underpinning of parkinsonism. Diphenhydramine can beused instead of trihexyphenidyl or benztropine, but i t causes more drows iness in most patients . Nonetheless , none of these drugs i s used tocounteract CNS depress ion caused by other antiparkinson drugs .

Diphenhydramine does help a l leviate cutaneous a l lergy symptoms (eg, urticaria ; d), but those are rare with any of the common antiparkinsondrugs (I did not speci fy which “other” drug was given). None of these drugs affect manic/hypomanic episodes that might occur with, say, high dosesof levodopa. They do not reverse antipsychotic-induced tardive dyskines ias (e); no drug does that effectively.

138. The answer is d. (Brunton, pp 404, 411-412; Katzung, pp 490, 527-528, 1034.) Many drugs (but not amphetamines , a ; barbi turates , b;benzodiazepines , c; or morphine or related opioid analges ics , e) participate in s igni ficant interactions with certa in foods or beverages . However,when you read phrases a long the l ines of those noted in the question—“severe and sometimes fata l hypertens ion from consuming such foods ascheeses and processed meats”—you should automatica l ly be thinking of the older, nonselective (MAO A/B) monoamine oxidase inhibi tors . Thesedrugs have been used for severe hypertens ion or depress ion, but due to the potentia l ly letha l interactions with tyramine-conta ining foods orbeverages , thei r use has dwindled because the ri sks were too great. (Nonetheless , you need to know your pharmacology for such drugs aspargyl ine, phenelzine, and tranylcypromine.) Reca l l that tyramine i s a catecholamine-releas ing drug with sympathomimetic actions in i ts ownright. When we consume tyramine-rich foods or beverages much of that sympathomimetic i s metabol i zed by hepatic MAO before i t reaches thesystemic ci rculation (fi rs t-pass metabol i sm), and so i ts effects are s l ight. In the presence of a nonselective MAO inhibi tor, however, the metabol icinactivation does not occur and s igni ficant amounts of the drug reach the ci rculation to cause more intense and potentia l ly very intense effectsthat involve adrenergic receptor activation. In addition, catecholaminergic (eg, adrenergic) nerves in the MAO-treated patient can be cons idered“loaded” with an abundance of the neurotransmitter due to inhibi ted intraneuronal metabol i sm of the neurotransmitter. Al though theneurotransmitter may not be released phys iologica l ly, in response to an action potentia l (at least in the periphera l sympathetic nervous system),i t can be released by such drugs as tyramine or by such drugs as pseudoephedrine and ephedrine (mixed-acting sympathomimetics ) oramphetamines (indi rect-acting sympathomimetics ), which can then trigger poss ibly fata l hypertens ive cri s i s , profound cardiac s timulation, and theconsequences thereof. These problems led to the development of MAO inhibi tors (eg, selegi l ine) that have preferentia l effects on MAO in thebra in (MAO-B) and far less an effect on MAO forms in the l iver and periphera l nervous system (MAO-A). Nonetheless , as s tated in the question,there i s a new nonselective MAO inhibi tor that has just been approved for use, for depress ion, in the form of a transdermal del ivery system (skinpatch).

139. The answer is d. (Brunton, p 516; Katzung, p 559.) Dextromethorphan i s , indeed, chemica l ly related to codeine, but i t lacks many effects youprobably associate with codeine or other opioids , except one: i t has excel lent anti tuss ive activi ty. This cough-suppressant effect presumablyoccurs via some i l l -defined centra l mechanism, but does not involve agonis t actions on opioid receptors . (Some have speculated that themechanism involves suppress ion of the afferent l imb of the 10th crania l nerve, but don’t memorize that!)

Dextromethorphan does not have analges ic effects (a ); a l ter gut moti l i ty (b); or have effects on the bladder musculature that might rel ievenocturia . Very high doses of dextromethorphan can cause CNS depress ion, but regardless of the dose i t i s not used as a sedative or other agent forcontrol l ing symptoms of ADD/ADHD. Fina l ly, unl ike codeine and most other opioids , dextromethorphan lacks s igni ficant addictive properties orpotentia l .

140. The answer is a. (Brunton, pp 297t, 299, 302; Katzung, pp 142, 148.) Whether you have learned or read about this speci fica l ly in a pharmacologyclass or text, for medicolega l and societa l (and board-related?) reasons you should be aware of the growing problem of i l l i ci t “meth labs” that usepseudoephedrine (“pseudo”) as the s tarting point for a rather easy chemica l synthes is of this highly addictive and dangerous CNS s timulant. Noneof the other drugs l i s ted can be synthes ized us ing this common indirect-acting sympathomimetic as a s tarting point.

Page 121: Pharmacology pre test self assessment and review (14th ed.)

141. The answer is c. (Brunton, pp 191t, 206t, 262-263; Katzung, pp 470-474.) Vecuronium is a curare-l ike nondepolarizing neuromuscular blocker that i sused to induce or mainta in skeleta l muscle para lys is for surgery. (Your learning may have focused on vecuronium; the two drugs , and indeed a l lother drugs with “curonium” or “curine” in the generic name are s imi lar.) It has no effects on bra in functions such as levels of consciousness orsensations such as of pa in. The drug prevents activation of nicotinic receptors on skeleta l muscle (i t i s a competi tive antagonis t), therebypreventing myocyte depolarization and other elements of normal skeleta l muscle activation and ul timately caus ing flaccid para lys is . The onsetand duration of this effect depends on the actua l drug being adminis tered.

Droperidol (a ) i s a ha loperidol -l ike neuroleptic drug. It causes drows iness and sedation in i ts own right. In some instances i t i s adminis teredwith fentanyl to cause neurolept analges ia (a ca lming effect, indi fference to pa in and surroundings); or with fentanyl and ni trous oxide for greaterpain control (neurolept anesthes ia). However, droperidol does not have any cl inica l ly useful intrins ic analges ic effects . A l imitation to us ingdroperidol for any purpose i s i ts tendency to prolong ventricular repolarization, which renders patients with “long QT syndrome”) at ri sk of seriousarrhythmias (especia l ly torsades).

Midazolam (b), propofol (d), and thiopenta l (e), when given intravenous ly, cause prompt sedation or loss of consciousness (dose-dependent)and can be used for such purposes as induction of anesthes ia or sedation; some are used for a variety of purposes (patients on venti lators , thoseundergoing endoscopy, or other noninvas ive procedures). These drugs lack intrins ic analges ic effects , and should not be used a lone for pa incontrol .

142. The answer is a. (Brunton, pp 398-400, 406t, 409t; Katzung, pp 527-530, 539.) The mention of menstrua l i rregulari ties , ga lactorrhea, and whatappears to be extrapyramidal s ide effects should be a tip-off that we are deal ing with a drug that interacts with dopamine receptors—speci fica l lyD2—by blocking them. Amoxapine, the correct answer, i s a tricycl ic anti -depressant (secondary amine). It i s rather unique among a l l the tricycl ics(other secondary amines , and tertiary amines such as the more fami l iar amitriptyl ine and imipramine) in severa l respects . For one thing i t inhibi tsneuronal dopamine and norepinephrine reuptake (the others affect mainly norepinephrine and serotonin). That, however, does not readi lyexpla in the amenorrhea-ga lactorrhea, and extrapyramidal s ide effects . What provides the explanation or mechanism relates to another ratherunique property of amoxapine: one of i ts metabol i tes i s a s trong dopamine receptor antagonis t, an action not shared by any of the other tricycl ics ;the SSRIs (eg, ci ta lopram, fluoxetine, sertra l ine; answers b, c, and d); and the monoamine oxidase inhibi tors (eg, tranylcypromine, e; orphenelzine).

You should reca l l that such antipsychotic drugs as the phenothiazines and ha loperidol may a lso cause amenorrhea-ga lactorrhea, andextrapyramidal s ide effects , by the same dopaminergic receptor-blocking mechanism. Conversely, drugs that activate dopamine receptors in oneway or another (eg, bromocriptine) can be used to manage these endocrine dys function. Amoxapine’s dopamine receptor blockade a lso seems toaccount for why the drug exerts some antipsychotic properties , theoretica l ly making i t useful for patients with both psychos is and depress ion.

143. The answer is c. (Brunton, pp 404, 412, 913; Katzung, pp 551-552, 557, 562.) Meperidine appears to be rather unusual among the common opioidanalges ics in terms of i ts abi l i ty to inhibi t neuronal reuptake of serotonin. The MAO inhibi tor has increased synaptic levels of serotonin (andother endogenous monoamines) in the centra l synapses , and so the presence of the opioid has enabled mass ive overstimulation of serotoninreceptors . We have caused the “serotonin syndrome.” None of the other drugs l i s ted participate in such an interaction. However, other importantdrugs not l i s ted in the question certa inly do. They include, especia l ly, both tricycl ic and SSRI-type antidepressants ; and the serotonin agonis tscommonly referred to as “triptans” (eg, sumatriptan).

Note: Compared with other opioid analges ics , meperidine causes s trong antimuscarinic s ide effects , which may be problematic or dangerousfor some patients . The concerns over interactions with MAOIs , the s trong antimuscarinic activi ty, and the genera l preva lence of toxici ty owing toready accumulation of the drug’s toxic metabol i te (normeperidine; especia l ly when the drug i s used for long-term pain management) have ledsome to s tate that i ts use as a “fi rs t-l ine analges ic i s (becoming) increas ingly rare.” That may be true for long-term pain control , but for s ingledose or other short-term management of moderate-to-severe pa in, meperidine i s s ti l l used often. See the explanation for the answer to Question122 for more information.

144. The answer is b. (Brunton, pp 86t, 468-469, 1059; Katzung, pp 379, 381-382, 1033.) The generic name for Rohypnol (“roofies ,” on the s treet) i sfluni trazepam, a very l ipophi l i c and potent benzodiazepine that mani fests a typica l but unusual ly s trong benzodiazepine-ethanol interaction.Given the unfortunate frequency with which this drug i s used i l lega l ly (there are some lega l uses outs ide the United States ), you need to knowabout i t, and we suspect you know that flumazeni l i s a competi tive benzodiazepine receptor antagonis t used to reverse (or diagnose)benzodiazepine overdoses . If you sa id that flumazeni l has no effect on ethanol -induced CNS depress ion you would be correct, but our patient hasconsumed l i ttle a lcohol ; her problems are due to the excess ive benzodiazepine effects and the interaction with the a lcohol . Diazepam (a) andtriazolam (e) being in the same class as the offending drug in this s i tuation, are l ikely to exacerbate the cl inica l problems. Ketamine, adissociative anesthetic (b; see Question 119 for more information) would be inappropriate. Nal trexone (d) i s an opioid antagonis t, much l ikenaloxone, but i s used ora l ly for uses somewhat di fferent than those for na loxone. Naloxone’s speci fici ty i s for opioid receptors , and so i t wouldhave no good or bad impact on our patient’s symptoms or vi ta l s igns .

145. The answer is c. (Brunton, pp 404, 407, 411; Katzung, pp 527-531, 539, 575.) One of the most noteworthy actions of bupropion, compared withtricycl ics or other common antidepressants , i s arguably the lowest incidence of sexual dys function. In males there i s no antimuscarinic action (e)that might interfere with erection, and no periphera l α-adrenergic blockade (d) that might interfere with ejaculation or cause orthostatichypotens ion. In fact, the drug seems to enhance sexual drive and performance. This latter effect has been cl inica l ly useful when the drug hasbeen used to counteract the suppressed sexual des i re caused by many of the SSRI antidepressants , and most of the tricycl ics . Women withdepress ion who report a decreased interest in intercourse a lso seem to benefi t from this drug more than others . Bupropion has s tructura ls imi lari ties with amphetamine and so i s largely nonsedating (and so a i s incorrect); and tends to suppress , rather than increase, appeti te (b). Thedrug lacks any effects on monoamine oxidase (f). However, bupropion’s metabol i sm is dependent on MAO, and i t should not be given within about2 to 3 weeks of s topping MAO inhibi tor therapy to minimize ri sks of toxici ty—the main and most serious of which i s a predispos i tion to seizures .

You may a lso remember that bupropion i s marketed as a smoking cessation a id, under a di fferent trade name than the one labeled for use fordepress ion. It i s in s i tuations such as this , where the same active ingredient i s sold under two di fferent trade names for decidedly di fferent uses ,that accidenta l overdoses can occur i f both products are inadvertently prescribed for the same patients .

146. The answer is b. (Brunton, pp 86t, 359, 423-424, 437t; Katzung, pp 504-512.) Neuroleptic mal ignant syndrome is thought to be a severe form of anextrapyramidal syndrome that can occur at any time with any dose of a neuroleptic agent. However, the ri sk i s higher when so-ca l led “high-potencyantipsychotics” are used in high doses , especia l ly i f given parentera l ly (You’l l find more information on this in Questions 118, 132, and 153).Morta l i ty from NMS is greater than 10%. Al lergic reactions (a) to ha loperidol are rare. Whi le ha loperidol overdoses (c) may trigger NMS, that i s nota prerequis i te for the development of the syndrome. Haloperidol clearly causes a parkinsonian-l ike cl inica l presentation (d) and tardivedyskines ias (e); but the fever, elevations of CK, and diminishing level of consciousness would be incons is tent with such findings .

Page 122: Pharmacology pre test self assessment and review (14th ed.)

147. The answer is d. (Brunton, pp 234, 614; Katzung, pp 485-486.) Carbidopa i s used as an adjunct to levodopa. It works by inhibi ting dopaminedecarboxylation, and subsequent metabol i sm of levodopa to dopamine, in the gut. This i s important because the majori ty of an ora l lyadminis tered dose of levodopa i s metabol i zed to dopamine in the gut, and dopamine in the systemic ci rculation does not cross the blood-bra inbarrier (and so never reaches the s triatum to cause des i red antiparkinson effects ). Amantadine (a) works in the CNS to promote dopamine releasefrom functional dopaminergic nerves . Benztropine (b) works centra l ly to block muscarinic receptors , thereby helping to “normal ize a dopamine-AChimbalance” that seems to be important in parkinsonism. Bromocriptine (c) works centra l ly as a di rect dopamine receptor agonis t. Selegi l ine (e) i sa centra l ly acting MAO-B inhibi tor that works by reducing metabol ic inactivation of dopamine.

148. The answer is e. (Brunton, pp 491-493, 521-522; Katzung, pp 545t, 559.) Pentazocine i s a partia l agonis t: i t acts as an agonis t on κ receptors , andblocks μ receptors . Adding i t to an analges ic regimen involving morphine or another “pure” opioid agonis t wi l l counteract key effects of morphine.In this case, the patient’s pa in wi l l grow worse, not become less ; and such other effects as venti latory depress ion wi l l be counteracted a lso.Under the ci rcumstances described in the question, the worsening of pa in i s far more l ikely to occur than seizures , which have been reported“occas ional ly.”

To answer this question you need to remember that pentazocine i s class i fied as a partia l μ agonis t (or mixed agonis t-antagonis t); and thatmorphine causes the fol lowing effects by acting as an agonis t on μ receptors : ana lges ia , respi ratory depress ion, euphoria , sedation, phys ica ldependence, and decreased gut moti l i ty. Pentazocine, given a lone, causes analges ia , sedation, and decreased gut moti l i ty by acting as an agonis ton κ receptors . However, i t i s a weak agonis t, yet i t i s ful ly capable of antagonizing the actions of morphine on μ receptors in a concentration-dependent fashion, and so pa in returns in this patient.

Concomitant with antagonism of a s trong opioid antagonis t’s ana lges ic effects , adminis tration of pentazocine wi l l antagonize (not intens i fy, a )respiratory depress ion. It wi l l not promote, accelerate, or intens i fy s trong opioid-related phys ica l dependence (b); induce coma (c), nor induceseizures (d).

149. The answer is b. (Brunton, pp 356-357, 614t; Katzung, pp 488f, 490, 499.) Selegi l ine inhibi ts MAO-B, thus inhibi ting the metabol ic breakdown ofdopamine and making more neuronal ly released dopamine ava i lable for i ts postsynaptic receptor activation. It increases the effectiveness ofboth endogenous and pharmacologica l ly adminis tered L-dopa. Benztropine (a) and trihexyphenidyl (c) are chol inergic muscarinic antagonis ts thatmainly act in the CNS. They have no effect on dopamine metabol i sm, release, or di rect receptor-mediated agonis t actions . Bromocriptine (d) i s adopamine receptor agonis t. Chlorpromazine (e) i s an antipsychotic drug with anti -adrenergic properties , and i t a l so competi tively blocksdopamine receptors in both the CNS and in the periphery.

150. The answer is c. (Brunton, pp 436, 1344t; Katzung, pp 508-510.) Chlorpromazine, the prototype phenothiazine antipsychotic drug, i s a l so indicatedfor managing nausea and vomiting, in both adults and chi ldren, from a number of causes . The drug can be adminis tered ora l ly, recta l ly, orintramuscularly for this very purpose. (Some phenothiazines with better antiemetic activi ty, such as prochlorperazine or promethazine, are usual lyused instead.) Regardless , the antiemetic mechanism appears to involve blockade of dopaminergic receptors in the chemoreceptor trigger zone ofthe bra in’s medul la .

Chlorpromazine (and most other antipsychotics ) can lower the bra in’s “seizure threshold,” thereby potentia l ly increas ing the ri sk of seizures (a)in susceptible patients . (Cons ider the seizure threshold to be the point at which further CNS s timulation leads to seizures ; the lower i t i s , thegreater the l ikel ihood that a drug such as a phenothiazine, a dopaminergic drug, etc, can cause epi lepti form bra in changes .) Chlorpromazine, andmost phenothiazines , tend to lower blood pressure (b; by blocking α-adrenergic receptors in the vasculature) and so probably would aggravatepreexis ting hypotens ion. Most of the phenothiazines a lso cause s igni ficant antimuscarinic effects , which would aggravate bladder hypomoti l i ty(d) and xerostomia (e; and other conditions involving reduced exocrine gland secretions). The antimuscarinic effects would inhibi t activation ofthe bladder’s detrusor muscle and inhibi t relaxation of the sphincter, thereby aggravating problems with micturi tion in such patients as theelderly man with an enlarged prostate.

Note: In genera l , phenothiazines are not used often for managing emes is or nausea. That i s , in part, because of the ri sk of excess ive sedation,extrapyramidal reactions , orthostatic hypotens ion, and occas ional cholestatic jaundice (hepati ti s ) or blood dyscras ias . Nowadays we tend to turnto other dopamine antagonis ts (eg, metoclopramide), a cannabinoid (dronabinol ), or a serotonin receptor blocker (ondansetron; a 5-HT3-selectiveblocker).

151. The answer is b. (Brunton, pp 198t, 661-663; Katzung, pp 143, 451t, 460.) Coca ine, an ester of benzoic acid, has loca l anesthetic properties ; i t canblock the ini tiation or conduction of a nerve impulse. It i s metabol i zed by plasma esterases to inactive products . Most important, coca ine (andtricycl ic antidepressants ) blocks the neuronal reuptake of norepinephrine and other monoamines (eg, dopamine). This action produces CNSstimulant effects including euphoria , exci tement, and restlessness . Periphera l ly, the blocked norepinephrine reuptake cause sympathomimeticeffects including tachycardia and vasoconstriction. Death from acute overdose can be from cardiac fa i lure, acute coronary vasospasm, s troke(hypertens ive cri s i s ), seizures , or apnea during the seizures . Coca ine does not di rectly activate adrenergic (a ) or chol inergic receptors ; i t has noeffect on catecholamine metabol i sm (c); tachycardia and vasoconstriction, not the oppos i te (d) are expected responses ; and coca ine has loca lanesthetic activi ty and so actua l ly suppresses the conduction of a variety of neuron types .

152. The answer is a. (Brunton, pp 357, 641, 1114.) The main subclasses of dopamine receptors in the CNS are D1 and D2 receptors . Bromocriptine i s aselective D2 agonis t, and i s useful to treat parkinsonism or hyperprolactinemia. Chlorpromazine (b), fluphenazine (c), ha loperidol (d), andpromethazine (e) are competi tive D2 receptor antagonis ts , and are used mainly as antipsychotic drugs .

153. The answer is a. (Brunton, pp 424t, 427t; Katzung, pp 507-508, 511-512.) Of a l l the antipsychotics , the incidence and severi ty of these anti -muscarinic(and other periphera l autonomic) s ide effects are highest with the phenothiazines , of which chlorpromazine can be cons idered the prototype.Clozapine (b) blocks α-adrenergic, his tamine (H1), ACh (muscarinic) receptors , but the affini ty for those receptors i s very low in comparison withdopamine and serotonin receptor blockade. Haloperidol has cons iderable dopamine receptor-blocking activi ty, and l i ttle effect on muscarinicreceptors that might account for the s ide effects described here. (See Questions 118, 123, and 132 for more on fundamenta l di fferences between“high potency” and “low potency” antipsychotics , especia l ly as they predict the relative incidence of periphera l autonomic and centra l(extrapyramidal ) s ide effects .) Olanzapine (d) i s pharmacologica l ly most s imi lar to clozapine, except for the fact that the ri sk of agranulocytos is i squi te low. Sertra l ine (e) i s an SSRI, and you should reca l l that SSRIs (eg, the prototype, fluoxetine) lack cl inica l ly s igni ficant antimuscarinic effects .

154. The answer is b. (Brunton, pp 502-503; Katzung, pp 549-552.) Morphine and other s trong opioids cause venti latory depress ion. That, in turn, ra isesblood p CO2 (most importantly) and lowers p O2 a l so. Cerebra l blood vessels respond by autoregulating (di lating) to increase cerebra l perfus ion.This , in turn, increases intracrania l pressure (ICP)—which i s a l ready l ikely to be high with a closed-head injury. If ICP ri ses sufficiently high frombrain swel l ing (the combined effects of drug-induced cerebra l vasodi lation and trauma-induced edema), then cerebra l blood flow can be reduced

Page 123: Pharmacology pre test self assessment and review (14th ed.)

to dangerous or even fata l levels (ie, i schemic s troke).Now let’s cons ider a scenario that’s germane to the question. Assume that the patient suffered the CHI in a motor vehicle accident, a long with

numerous and severe lacerations , fractures , and internal injuries . They hurt. Wi l l we let them hurt by avoiding use of morphine or any other s trongopioid analges ic—which a lso suppresses venti lation and ra ises ICP? Of course not, so what can we do? (1) Mechanica l ly venti late the patient tocontrol and mainta in reasonably normal blood gases ; (2) adminis ter mannitol , the osmotic diuretic, to help counteract bra in swel l ing; (3) perhapsgive a corticosteroid with s trong anti -inflammatory activi ty, such as dexamethasone; (4) dri l l a burr hole in the skul l to help reduce pressure; andso on.

And the other answer choices? Morphine i s indicated as adjunctive therapy for acute pulmonary edema (a ; improves pulmonary hemodynamics ,helps normal ize venti lation, helps a l lay anxiety); and a lso does s imi lar things for a recent or evolving MI or acute coronary syndrome (e). A his toryof epi lepsy (c) or hypertens ion (d) does not pose any cl inica l ly relevant problems in terms of morphine adminis tration.

155. The answer is b. (Brunton, pp 295-297; Katzung, pp 146, 176-177, 189.) In addi tion to being approved (and widely used) as an antihypertens ive drug,clonidine i s a lso approved for managing severe pa in in s i tuations such as the one described in the question. For analges ia i t i s mainly used asan adjunct to an opioid, and i t seems to provide s igni ficant added rel ief from pain or other unpleasant sensations that are neuropathic in origin.The drug i s given by continuous epidura l infus ion at rates and dosages lower than those reached when the drug i s given ora l ly or transdermal lyfor blood pressure control . The receptor-based analges ic mechanism of this very l ipophi l i c drug i s probably s imi lar to i ts centra l antihypertens iveeffects . Whereas antihypertens ive doses act as an agonis t for a population of α2-adrenergic receptors in the cardiovascular control center of thebra in (medul la ), ana lges ic doses of clonidine primari ly act in parts of the spina l cord. Once those adrenergic receptors are activatedneurotransmiss ion by afferent sensory nerves i s inhibi ted. (The drug acts only in those parts of the spina l cord that have sensory afferent pa inpathways from periphera l s tructures .) Clonidine-mediated analges ia i s not suppressed or abol i shed by tradi tional opioid receptor antagonis ts(na loxone, na l trexone). Nonetheless , when clonidine i s used for pa in (or to manage drug withdrawal s igns and symptoms), you s ti l l shouldanticipate a l l the drug’s expected s ide effects , including a fa l l of blood pressure (reduced sympathetic vasoconstriction) and of heart rate andstroke volume (reduced β1 activation). Also, even when used as an analges ic, when the drug i s to be s topped the da i ly doses need to be taperedgradual ly to reduce the ri sk of rebound hypertens ive episodes .

None of the other drugs l i s ted as poss ible answers , or members of thei r class—ACE inhibi tors , diuretics , β-blockers (of any sort or selectivi ty), orperiphera l ly acting α-blockers—share clonidine’s analges ic actions or use. Indeed, of a l l the antihypertens ive drugs clonidine i s the only one withthis approved analges ic use.

156. The answer is b. (Brunton, pp 508-509; Katzung, pp 557, 562.) Methadone i s a s low-onset, long-acting, opioid receptor (μ and κ) agonis t. It i s usedas an analges ic (for long-term pain control when an opioid i s indicated) and for maintenance therapy of individuals dependent on opioids . It hasgreater ora l bioava i labi l i ty than morphine, especia l ly when ora l therapy i s s tarted. (Reca l l that morphine i s subject to extens ive fi rs t-pass hepaticmetabol i sm, and only with repeated ora l adminis tration can we saturate i ts drug-metabol i zing enzymes such that bioava i labi l i ty i s improvedenough to get good analges ia . That’s why i f our analges ic of choice i s morphine, we s tart treatment with parentera l adminis tration.) Methadone’slong biologic ha l f-l i fe accounts for the mi lder but more protracted abstinence syndrome when the drug i s s topped. Methadone does not possessopioid antagonis t properties (whether μ or κ) and, thus , would not precipi tate withdrawal symptoms in a heroin addict, as would na loxone ornal trexone, so long as we adminis ter sufficient doses of the methadone, and that i s precisely what we do with methadone maintenance therapyor when we switch a patient to i t from another opioid. Excess ive doses of methadone can cause typica l opioid-related venti latory depress ion.

157. The answer is b. (Brunton, pp 584t, 596; Katzung, pp 417-418, 422-423.) Ethosuximide i s very effective (and, according to many, the drug of choice) forabsence seizures . Phenytoin (e) may aggravate absence seizures , despi te i ts efficacy and proven track record for other chronic seizure types . Noneof the other drugs l i s ted (a , diazepam; c, lorazepam; d, methylphenidate; e, phenytoin) are effective or indicated for long-term therapy of epi lepsy.

Severa l other things to reca l l : Diazepam used to be the drug of choice for ini tia l therapy of s tatus epi lepticus . No longer. For s tatus epi lepticuswe give IV lorazepam fi rs t, for prompt seizure control ; fol lowed immediately by IV phenytoin (or fosphenytoin) for a s lower-onset but longer-las ting control of the seizures . Methylphenidate, an amphetamine-l ike drug, i s one of severa l s imi lar agents indicated for managing ADD-ADHD. Itwi l l not help absence seizures , and owing to i ts CNS-stimulating effects may aggravate other seizure types .

158. The answer is b. (Brunton, pp 310, 347-349; Katzung, pp 155, 287-290.) Ergotamine has for years been cons idered a mainstay of aborting migra ineheadaches , a l though nowadays for efficacious and arguably safer drugs such as sumatriptan are used. Whether the drug i s an ergot a lka loid or atriptan, the l ikely migra ine-aborting mechanism involves antagonizing cerebra l vasodi lation. It activates 5-HT3 receptors , which in turn causescerebra l vasoconstriction. Ergotamine in particular a lso can cause s igni ficant and long-las ting systemic vasoconstriction via α-adrenergic receptoractivation. Whi le normal coronary vessels aren’t uniquely sens i tive to this effect, those of patients who are prone to developing coronaryvasospasm (Prinzmeta l or variant angina) are at greater ri sk, and that i s l ikely what this patient had. [Perhaps you’ve heard of “St. Anthony Fi re”—what we now ca l l ergotism—that occurred in the Middle Ages . It was characterized by gangrene and “bloodless” loss of the fingers and toes(among other problems), and was due to intense vasoconstriction caused by eating spoi led gra ins laden with a fungus that produced largeamounts of ergot a lka loids .]

Bromocriptine (a), another ergot a lka loid, has l i ttle effect on 5-HT or adrenergic receptors . However i t i s an efficacious dopamine D2 agonis t,and that provides the bas is for i ts use for parkinsonism, hyperprolactinemia, and treatment of some prolactin-secreting pi tui tary tumors .Morphine (c) i s not l ikely to cause hypertens ion, and would produce myocardia l i schemia (of a global nature) only when such high doses are giventhat hypotens ion (and venti latory depress ion) ensue.

Phenoxybenzamine (d) blocks 5-HT receptors (and ACh and his tamine receptors too), which expla ins i ts use (infrequently, nowadays) forcarcinoid syndrome—a tumor that secretes large amounts of serotonin and peptides that s timulate smooth muscles (primari ly in the a i rways andGI tract). However, phenoxybenzamine’s main pharmacologic effect i s long-lasting and noncompeti tive blockade of α-adrenergic receptors , and therelated use i s adjunctive management of catecholamine-secreting tumors (ie, pheochromocytoma). Thus , i t lowers blood pressure and a lso wouldblock coronary vasoconstriction or spasm mediated by α-adrenergic activation. Dosages needed to block 5-HT receptors are much higher than thoseneeded to block serotonin receptors .

Phenytoin (e), an important anticonvulsant, has no abi l i ty to cause hypertens ion (nor any other s igni ficant increase of blood pressure), nor tocause or exacerbate coronary vasospasm.

159. The answer is e. (Brunton, pp 920-921; Katzung, pp 279, 1100.) Promethazine, l ike most other phenothiazines , tends to cause dose-dependent CNSdepress ion. Of the answers given here, the most l ikely outcome of that effect i s venti latory depress ion and apnea, and that i s precisely the reasonwhy the FDA bols tered the warnings for chi ldren. Like most other phenothiazines , promethazine has s igni ficant antimuscarinic activi ty, and thatwould not cause heart block (a) or diarrhea (d). It has α-adrenergic blocking activi ty, which would lower, rather than ra ise, blood pressure (b).Promethazine may, indeed, cause parkinsonian-l ike s ide effects (c), but they are not l ikely to be a cause of death unless the patient develops

Page 124: Pharmacology pre test self assessment and review (14th ed.)

neuroleptic mal ignant syndrome that goes undiagnosed and improperly treated unti l i t i s too late.

160. The answer is e. (Brunton, pp 86t, 268; Katzung, pp 284, 437, 475, 479.) The syndrome I described i s that of mal ignant hyperthermia (MH). (Somefolks l ike to use the mnemonic FEVER—fever, encephalopathy, vi ta ls unstable, elevated enzymes such as creatine kinase, and rigidi ty of muscles .)For decades i t has been attributed to an interaction between ha logenated hydrocarbon volati le l iquid anesthetics—primari ly ha lothane, but s ti l la poss ibi l i ty with others , including i soflurane; and skeleta l neuro-muscular blockers , particularly succinylchol ine. It can occur intraoperatively orin the immediate postoperative period. The overa l l incidence i s about 1 in 50,000 anesthes ias , and 1 in about 15,000 pediatric anesthes ias .Worldwide, MH is fata l about 80% of the time, but only about 10% to 15% of the time in the United States . There i s a lmost certa inly a genetic bas isfor MH, involving an autosomal dominant tra i t that accounts for about ha l f of a l l MH cases . On a molecular bas is i t i s characterized by aberrantfunction of the so-ca l led ryanodine-sens i tive ca lcium channel in the sarcoplasmic reticulum. In essence, i t impairs the abi l i ty of the sarcoplasmicreticulum to take up (via an ATPase) and reta in Ca 2+ as i s necessary for skeleta l muscle to relax, a lbei t briefly, between contractions . The resul tingand more or less perpetual ri se of free intracel lular Ca 2+ leads to tonic skeleta l muscle activation (hence, rigidi ty). The heightened ATPconsumption that occurs to mainta in cross -bridge formation uses an abundant amount of oxygen, and heat i s generated (the fever). As musclefunction deteriorates and ATP i s depleted, metabol i sm switches to glycolys is and lactic acid accumulation (acidos is ). Damaged muscle cel l s leakK+ that in turn causes hyperka lemia, tachycardia , and cardiac arrhythmias . Myoglobin and large intracel lular enzymes such as creatine kinase a lsoleak out of damaged myocytes . Unless treated promptly (dantrolene, phys ica l means to cool the body, attempts to correct blood pH and electrolyteprofi les ), the consequences can be fata l . See Questions 132, 146, 159, and 176 for more information on the i ssue of neuroleptic mal ignantsyndrome or the somewhat related syndrome, mal ignant hyperthermia.

None of the other drugs , individual ly or in the combinations given, are associated with mal ignant hyperthermia.

161. The answer is c. (Brunton, pp 583-606, 1792; Katzung, pp 416-417.) Vigabatrin (γ-vinyl GABA) i s , indeed, useful in partia l seizures . Before address ingthe speci fic mechanism: If you encounter a question about vigabatrin, you should immediately see that the drug works , in one way or another,because the letters gaba are in the drug’s generic name. And the speci fic mechanism? Vigabatrin i s an i rrevers ible inhibi tor of GABAaminotransferase, an enzyme respons ible for the termination of GABA action once the neurotransmitter has been released into the synapse. Thisresul ts in accumulation of GABA at synaptic s i tes , thereby enhancing i ts effect. (Note the many para l lels between pharmacologic modi fication of“GAB-Aergic” neurotransmiss ion and those that can be used to modi fy chol inergic and adrenergic neurotransmiss ion. For example, just asvigabatrin impairs inactivation of released GABA, thereby “boosting” GABA’s postsynaptic effects , so do acetylchol inesterase inhibi tors (eg,neostigmine) with respect to the actions of released ACh.

162. The answer is d. (Brunton, pp 509-510; Katzung, pp 407-408, 423-424.) Fol ic acid supplementation during pregnancy i s recommended during normaluncompl icated pregnancy, and i t i s cri ti ca l when the mother i s receiving anticonvulsant drugs (AEDs) during gestation. The purpose i s to reduce therisk of spina bi fida and other neura l tube defects (and some other teratogenic consequences). The condition i s known as the “feta l hydantoinsyndrome.” The question’s scenario describes no reason to add va lproic acid (or add or switch to any other AED, including phenobarbi ta l ) i f themother i s kept seizure-free during pregnancy and gets proper perinata l care. Va lproic acid i s in pregnancy class D; most others are C, and so i tappears to carry the highest ri sk of teratogenic effects of a l l the common AEDs . Discontinuing any or a l l AEDs during pregnancy (b) i s ri sky in thisand most ci rcumstances , as i t carries the ri sk of seizure recurrence that can be more dangerous to the mother and the fetus than continuingeffective therapy and providing good prenata l care—and fol ic acid supplementation.

Notes : Current recommendations (Fol ic acid for the prevention of neura l tube defects : US Preventive Services Task Force recommendationstatement. Ann. Intern. Med. 2009;150:626) are that women “(even) planning or capable of pregnancy should take a da i ly supplement (in addition toany folate in the diet) conta ining between 0.4 and 0.8 mg of fol ic acid.” These recommendations apply even to women who are otherwise heal thyand taking no teratogenic drugs , AEDs , or any others . As an as ide, the overa l l ri sk of having a chi ld born with neura l tube defects or otheranomal ies i s about 1%. With s ingle-dose AED therapy throughout pregnancy, and with proper perinata l care, the ri sk (overa l l ) ri ses to about 2%.The ri sk goes up cons iderably, however, when the mother i s taking more than one AED during pregnancy, whether that drug i s va lproic acid (a) orany other agent.

Fina l ly, judicious supplementation of the mother’s diet with i ron (c) i s often recommended. That i s to reduce the ri sk or severi ty of hematologicabnormal i ties , and has nothing to do with prevention of teratogenes is or other s imi lar AED-related problems.

163. The answer is b. (Brunton, pp 436-443, 920; Katzung, pp 506-508, 511-512.) Chlorpromazine, of a l l the drugs l i s ted, has s trong α-adrenergic receptor-blocking activi ty. This accounts for a relatively high incidence of orthostatic hypotens ion, and expla ins why measuring the patient’s blood pressurein both the supine and s tanding pos i tions i s important. At cl inica l ly relevant dosages , buspirone (a) has no s igni ficant autonomic orcardiovascular effects . Diphenhydramine s trongly blocks both muscarinic and H1 hi s tamine receptors , and nei ther action would l ikely causehypotens ion. Haloperidol (d), a butyrophenone antipsychotic, causes few periphera l autonomic s ide effects , including those noted in thequestion. Zolpidem (e) i s a benzodiazepine-l ike hypnotic that lacks appreciable periphera l autonomic s ide effects .

164. The answer is a. (Brunton, pp 350-351, 359; Katzung, pp 504f, 509t, 519.) Clozapine causes agranulocytos is in 1% to 2% of treated patients , which i sfar more common and potentia l ly serious than with other, and older, antipsychotics that might be used in l ieu of i t. This potentia l ly letha l blooddyscras ia i s genera l ly revers ible on discontinuation of the drug, but this of course depends on frequent blood tests to detect the problem early on.Monitoring for this adverse response i s so cri ti ca l that the ini tia l prescription for the drug, and refi l l s for continued therapy, cannot be fi l ledwithout proof of blood counts that are within “acceptable” levels . (The incidence of blood dyscras ias with the clozapine-l ike atypica l antipsychoticolanzapine i s lower than that with clozapine, but nonetheless higher than with any of the older agents .) Other concerns with clozapine, for whichmonitoring i s important, include the development of seizures ; weight ga in (i t i s common and the patient may put on an extra 10 or 20 pounds);ri ses of blood glucose levels (perhaps with the development of new-onset diabetes , and so answer c i s incorrect); and myocardi ti s . Hypotens ion(d) and venti latory depress ion (e) are not at a l l common with the atypica l antipsychotics .

Another cri ti ca l di fference between any of the atypica l antipsychotics and any of the older agents i s this : Extrapyramidal s ide effects (EPS; b) arerare with any of the atypica l agents , in absolute numbers and in comparison with the older agents—phenothiazines or, especia l ly, ha loperidol .The ri sk of EPS with clozapine or related drugs i s low because of the drugs ’ low affini ties for blocking dopamine D2 receptors , a property that i sprobably the key factor in drug-induced EPS. Given the sheer number of patients who are treated with a phenothiazine antipsychotic, orha loperidol , and the associated ri sks of parkinsonism or tardive dyskines ias , why i sn’t a drug l ike clozapine or clozapine i tsel f used more often?One reason i s cost, but the others include the s ide effects caused by the atypica l antipsychotics , especia l ly the ri sk of blood dyscras ias that can befata l i f not detected early, and managed properly and promptly.

165. The answer is d. (Brunton, pp 591-593, 845, 1012, 1524, 1529; Katzung, pp 409, 425t.) Phenytoin, given throughout pregnancy, can inhibi t hepaticsynthes is (activation) of vi tamin K-dependent clotting factors in utero. (Phenobarbi ta l and severa l other anticonvulsants can do the same, and so

Page 125: Pharmacology pre test self assessment and review (14th ed.)

vi tamin K prophylaxis appl ies to thei r use as wel l .) None of the other drugs l i s ted i s associated with such an effect. Warfarin (e), arguably themost widely used ora l anticoagulant in the world i s absolutely contra indicated (category X) during pregnancy because of the ri sks of so-ca l led“warfarin embryopathy.” See the explanation for answers to Question 199 for more information.

Notes : Bupropion i s pregnancy category B (low ri sk in humans). Most of the benzodiazepines , including diazepam, are in pregnancy category D(defini te ri sks , not absolutely contra indicated, but best avoided). Es tazolam, flurazepam, quazepam, temazepam, and triazolam are category X.Opioids are genera l ly category B or C (methadone, answer c).

166. The answer is b. (Brunton, pp 138, 207t, 212, 617-618; Katzung, pp 488f, 490, 499) Entacapone i s a catechol O-methyl transferase (COMT) inhibi tor.COMT normal ly i s a minor player in the metabol i sm of levodopa, but when the major pathway for levodopa metabol i sm to dopamine (via dopadecarboxylase) i s inhibi ted, as i t i s with adminis tration of carbidopa, COMT plays a more important role. It helps form 3-O-methyldopa, whichcompetes with levodopa for uptake into the bra in. So, when we have poor responses to levodopa (a lone or in combination with carbidopa), orwhen levodopa therapy has been ongoing for so long that the patient develops acute or chronic refractoriness to i ts effects (eg, the “on-off” or the“wearing-off” phenomena), adding entacapone (or a s imi lar drug, tolcapone) might be reasonable.

Donepezi l (a ) and tacrine (d) are centra l ly acting chol inesterase inhibi tors with no effect on dopamine metabol i sm. Selegi l ine (c) i s a selectiveinhibi tor of dopamine metabol i sm by MAO in the bra in (MAO-B). Trihexyphenidyl (e) i s a centra l ly acting muscarinic receptor blocking drug, mainlyused for parkinsonism. Related antimuscarinic drugs for parkinsonism are benztropine and diphenhydramine (the latter of which a lso exertss trong periphera l muscarinic-blocking effects ).

167. The answer is c. (Brunton, pp 643-644; Katzung, pp 379, 400, 610t, 1156t.) This i s a class ic description of an a lcohol -disul fi ram interaction.Disul fi ram is sometimes used in control l ing a lcohol abuse. It acts by inhibi tion of a ldehyde dehydrogenase, resul ting in the accumulation ofaceta ldehyde. This “aceta ldehyde syndrome” i s characterized by the s igns and symptoms described in the question. The onset of symptoms i sa lmost immediately fol lowing ingestion of a lcohol and may las t for severa l hours in some patients . None of the other drugs l i s ted in the answerscan cause these outcomes. Nal trexone (a), an opioid antagonis t, would have no effect on the responses to ethanol . Diazepam (b) andphenobarbi ta l (d), both CNS depressants that clearly have their depressant effects potentiated by a lcohol , would not lead to the described effects .Tranylcypromine (e) i s an older nonselective MAO inhibi tor. MAO plays no role in the metabol i sm or responses to a lcohol .

Note: Severa l other drugs or drug classes can cause cl inica l ly s igni ficant disul fi ram-l ike reactions . They include the older sul fonylureas(tolbutamide, chlorpropamide) used for ora l therapy of type 2 diabetes mel l i tus ; and some of the cephalosporin antibiotics .

168. The answer is c. (Brunton, pp 398-413; Katzung, pp 528-529, 539.) Fluoxetine—and related SSRI antidepressants such as sertra l ine, fluvoxamine,ci ta lopram, and esci ta lopram—selectively inhibi t neuronal serotonin uptake, with minimal effects on other monoamines . The tricycl ics(imipramine) and venlafaxine inhibi t serotonin and norepinephrine reuptake (and, apparently to a smal l degree, dopamine). Bupropion affectsreuptake of both norepinephrine and dopamine.

169. The answer is d. (Brunton, pp 458-468, 642-644; Katzung, pp 377t, 420, 439t.) Benzodiazepines (particularly those with prompt and s igni ficantsedating activi ty) are cons idered good choices for a l leviating benzodiazepine and a lcohol withdrawal symptoms. The anxiolytic effects ofbuspirone (a) take severa l days to develop, rendering i t unsui table for acute, severe withdrawal s igns and symptoms. In addition, i t causes l i ttleor no CNS depress ion in most patients . Chlora l hydrate (b) and ethanol interact by inhibi ting the metabol i sm of each other, with the main resul tbeing profound CNS depress ion. We would not want to give chlora l hydrate to any patient who s ti l l may have a lcohol in the ci rculation.Chlorpromazine (c) i s used mainly for psychos is , and i s not l ikely to beneficia l ly a ffect the s igns and symptoms that our patient has . Trazodone (e)i s an atypica l antidepressant with a fa i r amount of genera l i zed CNS-depressant effects , which makes i t often prescribed for patients who haveinsomnia as a major depress ion symptom. It weakly inhibi ts neuronal serotonin reuptake and seems to increase serotonin release fromserotoninergic nerves . It would not be sui table for the immediate management of this patient.

170. The answer is b. (Brunton, pp 86t, 631-632; Katzung, pp 397-398, 1037.) Methanol i s metabol i zed by the same enzymes that metabol i ze ethanol , butthe products are di fferent: formaldehyde and formic acid in the case of methanol . Headache, vertigo, vomiting, abdominal pa in, dyspnea, andblurred vis ion can occur from accumulation of these metabol ic intermediates . However, the most dangerous (or at least permanently disabl ing)consequence in severe cases i s hyperemia of the optic disc, which can lead to bl indness . The rationale for adminis tering ethanol to treatmethanol poisoning i s fa i rly s imple. Ethanol has a high affini ty for a lcohol and a ldehyde dehydrogenases and competes with methanol as asubstrate for those enzymes , reducing metabol i sm of methanol to i ts more toxic products . Important adjunctive treatments include hemodia lys isto enhance removal of methanol and i ts products ; and adminis tration of systemic a lka l inizing sa l ts (eg, sodium bicarbonate) to counteractmetabol ic acidos is . None of the other drugs l i s ted would be appropriate. They may a l leviate some of the s igns and symptoms resul ting frommethanol poisoning, but they would have no beneficia l effects on the crux of the problem—the metabol i sm of the methanol .

171. The answer is c. (Brunton, p 484t; Katzung, pp 559-560, 572, 1033t.) Na loxone i s given parentera l ly. It speci fica l ly blocks opioid receptors that areactivated by such agonis ts as morphine, heroin, and fentanyl (among many others ). Diazepam (a) and other benzodiazepines (eg, lorazepam) haveno effects on opioid receptors . Flumazeni l (b) i s a speci fic benzodiazepine antagonis t. It may be l i fe-saving in cases of benzodiazepineoverdoses , but has no effects on the responses to opioids . Nal trexone (d) i s an opioid receptor antagonis t (as i s na loxone), but i t i s given ora l ly,has a s lower onset of action, and so wi l l be of l i ttle benefi t in this acute, l i fe-threatening s i tuation. Fentanyl i s unl ikely to cause seizures .Phenytoin (e) i s , of course, an anticonvulsant; i t would be inappropriate to adminis ter, and i f the adminis tered dose was sufficiently high i t couldcontribute to the CNS and venti latory depress ion caused by the fentanyl .

172. The answer is b. (Brunton, pp 404-405, 411, 923; Katzung, pp 525-530.) Imipramine, a tricycl ic antidepressants , causes s trong periphera lantimuscarinic effects . In the context of our patient, reca l l that muscarinic receptor activation contracts the detrusor muscle of the bladder, andrelaxes the bladder sphincter. Imipramine blocks those effects , and that seems to expla in i ts effects in managing nocturnal enures is—especia l lyin chi ldren. The drug and other members of i ts class do rel ieve depress ion s igns and symptoms. However, they are due to inhibi tion, not increasesof (a ) monoamine neurotransmitter reuptake via the so-ca l led norepinephrine transporter at the “endings” of catecholaminergic neurons .Imipramine and i ts group members tend to cause sedation (c), but that i s not l ikely to reduce the frequency of enures is . These drugs a lso have α-adrenergic blocking activi ty, and one consequence of that would most l ikely be an increase, not a decrease, of renal blood flow and GFR. Tricycl icantidepressants , nor other major classes of antidepressants , have any important effects on the release of antidiuretic hormone or the responsesof the renal tubules to ADH.

173. The answer is a. (Brunton, pp 404, 407, 411; Katzung, pp 527-531, 1034.) Bupropion i s marketed, under di fferent trade names , for two main purposes :anxiety rel ief and suppress ion of the cravings for nicotine. Since too many phys icians prescribe by brand-name products , and ignore the activegeneric drug, such problems as the one I described in our scenario, due to accidenta l dupl ication and overdoses of a drug, are more common than

Page 126: Pharmacology pre test self assessment and review (14th ed.)

you’d l ike to bel ieve. One of the main consequences of bupropion overdose (or even sudden discontinuation of the drug, especia l ly in patientswith epi lepsy) i s seizures . Chlordiazepoxide (b) i s a benzodiazepine that i s not indicated for depress ion or as a s top-smoking a id. If anything, thedrug wi l l suppress , rather than cause, seizures . Fluoxetine (c) and imipramine (d) are antidepressants , but they are not prescribed as smoking-cessation a ids , nor are they marketed in products approved for that use. Li thium (e) i s a mood-stabi l i zing drug used in bipolar i l lness . It i s notused for managing depress ion or for smoking cessation.

174. The answer is c. (Brunton, p 234; Katzung, pp 117f, 492t, 499) At fi rs t blush i t would seem that choos ing any “antiparkinson” drug would beappropriate for this man. However, i f you think about the etiology of schizophrenia and of parkinsonism (especia l ly drug-induced), and themechanisms of action of the drugs l i s ted, you would select benztropine (or a related drug, trihexyphenidyl )—or diphenhydramine, a β-blocker, or abenzodiazepine, none of which was l i s ted as a choice).

Phenothiazines (chlorpromazine), and especia l ly butyrophenones (ha loperidol , e), rel ieve schizophrenia s igns and symptoms largely byblockade of centra l dopamine (D2) receptors , particularly in the mesol imbic-mesocortica l sys tem. Simultaneous ly, D2 blockade in thenigrostriatum, and concomitant unmasking of oppos ing muscarinic-chol inergic effects , seems to account for the extrapyramidal/parkinsonian s ideeffects that have surfaced in patients l ike the one I describe. Reca l l that despi te the imprecis ion of the terminology, “high potency” antipsychoticssuch as ha loperidol are associated with a much higher incidence of extrapyramidal s ide effects than are the “low potency” antipsychotics (ie, thephenothiazines), due to thei r s tronger centra l D2 blockade, and so switching from chlorpromazine to ha loperidol would not at a l l a l leviate thispatient’s parkinsonian s ide effects : i t would l ikely worsen them.

In idiopathic Parkinson disease there i s deterioration of dopaminergic neurons and, inferentia l ly, a decrease of centra l dopamine content orrelease. That pathophys iology provides a rationale for pharmacologica l ly boosting dopaminergic influences : increas ing centra l dopaminesynthes is (levodopa, a lone or with carbidopa; d); inhibi ting dopamine’s metabol ic inactivation with ei ther a COMT inhibi tor (tolcapone, a ; orentacapone) or an MAO inhibi tor (eg, selegi l ine, not l i s ted); or us ing such other s trategies as dopamine-releas ing or -mimetic agents(amantadine, bromocriptine).

In this s i tuation of drug-induced parkinsonism, however, dopamine receptors are blocked by the antipsychotic. Whi le we could try to overcomethat blockade (s ince i t i s surmountable and competi tive antagonism we are deal ing with), that approach i s not l ikely to be effective nor tolerableto the patient, s ince we would have to try very large doses of a dopaminergic drug—doses sufficient to cause, in a l l l i kel ihood, s igni ficant s ideeffects .

So what we do, therefore, i s attack the dopamine-ACh imbalance from the “other s ide”—adminis ter a centra l ly acting antimuscarinic drug. Notethat s ince we are deal ing with symptoms aris ing from a relative excess of chol inergic influences , a centra l ly acting chol inesterase inhibi tor(donepezi l or tacrine, b) would worsen the problems by making even more ACh ava i lable at centra l muscarinic synapses .

175. The answer is d. (Brunton, p 357; Katzung, pp 489, 499, 672.) Bromocriptine mimics the action of dopamine in the bra in, but i s not as readi lymetabol i zed and inactivated as the endogenous neurotransmitter. It i s especia l ly useful in parkinsonism that i s unrespons ive to L-dopa.Apomorphine (b) i s a l so a dopamine receptor agonis t, but i ts s ide effects (mainly emes is ) preclude i ts use for parkinsonism. Selegi l ine (e) i s anMAO-B inhibi tor. Bel ladonna preparations (c) are atropine and atropine-l ike antimuscarincs . Centra l ly acting antimuscarinics such as benztropineand trihexyphenidyl are important drugs for managing some patients with parkinsonism, but they work not by enhancing the effects of dopaminebut rather by blocking the centra l effects of ACh. Amantadine (a) i s an antivi ra l agent that a lso i s useful in some cases of parkinsonism. It ei ther(or both) enhances the synthes is or inhibi ts neuronal reuptake of dopamine. It i s not a dopamine receptor agonis t.

176. The answer is b. (Brunton, p 357; Katzung, pp 489, 499, 672.) Bromocriptine, an ergot derivative, i s a di rect-acting dopamine receptor agonis t. Itsmain uses are for adjunctive management of Parkinson disease, and for management of amenorrhea and inferti l i ty (and, i f present, ga lactorrhea)due to hyperprolactinemia. It i s a l so used, long-term, for management of some pi tui tary adenomas. However, i t has ga ined acceptance as animportant adjunct (usual ly a long with dantrolene, as noted in the question) for management of neuroleptic mal ignant syndrome (NMS). Reca l lthat NMS is a rare but potentia l ly fata l response to tradi tional neuroleptic/antipsychotic drugs (phenothiazines such as chlorpromazine, andbutyrophenones such as ha loperidol ). The newer, atypica l antipsychotics (clozapine, olanzapine, and ri speridone) may a lso cause NMS, but thecl inica l presentation seems not to include muscle rigidi ty that i s usual ly an accompaniment of NMS caused by phenothiazines or butyrophenones .

177. The answer is d. (Brunton, pp 344-350, 403t, 406t, 410t; Katzung, pp 155, 284-285, 529, 536-537, 1034.) Many patients , particularly those with s igns andsymptoms of depress ion, and whether or not they are treated with a tricycl ic antidepressant (or, for that matter, an SSRI), are prescribed trazodoneto help them go to s leep at night. Trazodone has not only genera l CNS depressant effects (helping to induce s leep), but a lso intrins icantidepressant properties . It i s a common and largely safe drug for patients such as the one described here.

Trazodone has many effects , one of which i s potentiating the postsynaptic effects of serotonin. As such, i t should not be prescribed for patientstaking a nonselective monoamine oxidase (MAO) inhibi tor for depress ion. (Thus , answer a i s incorrect. And, because of a host of adverse effectsand drug-drug interactions that may prove fata l , nonselective MAO inhibi tors are, at best, rarely prescribed for anyone.) Sumatriptan (c), which“adds” serotonin to synapses and i s often prescribed to abort migra ine headaches , i s contra indicated for patients taking antidepressants—particularly selective serotonin reuptake inhibi tors (SSRIs , eg, fluoxetine, sertra l ine, esci ta lopram, severa l others ) or MAO inhibi tors . Use oftrazodone and an SSRI carries a s l ightly increased ri sk of serotonin syndrome. Giving both trazo-done and a nonselective MAO inhibi tor to apatient i s associated with a greatly increased ri sk of serotonin syndrome.

Page 127: Pharmacology pre test self assessment and review (14th ed.)

Cardiovascular Pharmacology Antiangina lsAntiarrhythmicsAnticoagulantsAntihyperl ipidemicsAntihypertens ivesAntiplatelet drugsHeart fa i lure therapyThrombolytics (fibrinolytics )Vasodi lators

Questions

178. Your patient i s a 50-year-old man with Type 1 (insul in-requiring) diabetes , normal renal function, and no microa lbuminuria . Al though hisHbA1c levels are acceptable, because of his l i fes tyle and eating habi ts he has experienced more than a few episodes of symptomatichypoglycemia fol lowing insul in injections . He currently has asymptomatic hyperuricemia, but he has had severa l attacks of acute gout over the las t5 years . Today you diagnose hypertens ion that must be treated. Which antihypertens ive drug would be the most rational fi rs t choice for s tartinghis antihypertens ive therapy?

a. Angiotens in-converting enzyme (ACE) inhibi tor or angiotens in receptor blockerb. β-adrenergic blockerc. Ni fedipined. Thiazide diuretice. Verapami l or di l tiazem

179. A patient i s hospi ta l i zed and waiting for coronary angiography. His his tory includes angina pectoris that i s brought on by “modest” exercise,and i s accompanied by trans ient electrocardiographic changes cons is tent with myocardia l i schemia. There i s no evidence of coronary vasospasm.In the hospi ta l , he i s receiving ni troglycerin and morphine (both by s low IV infus ion), plus oxygen via nasa l cannula .

He suddenly develops episodes of chest discomfort. Heart rate during these episodes ri ses to 170 to 190 beats/min; blood pressure reaches 180to 200/110 to 120 mm Hg, and prominent findings on the ECG are runs of ventricular ectopic beats that terminate spontaneous ly, plus ST-segmentelevation.

Al though there are severa l things that need to be done for immediate care, what drug would you adminis ter to remedy (at least temporari ly) themajori ty of these s igns and symptoms and pose the lowest ri sk of doing further harm?

a. Aspirinb. Captopri lc. Furosemided. Labeta lole. Lidoca inef. Ni troglycerin (increased dose as a bolus )g. Prazos in

180. A 55-year-old woman has just been diagnosed with Stage 2 essentia l/primary hypertens ion, and you conclude that i t i s time to s tart drugtherapy for i t. She a lso tends to be tachycardic. Notes wri tten by her ophthalmologis t indicate that she has chronic open-angle glaucoma. Whichone of the fol lowing drugs would be the most rational choice for this woman, given only the information presented in this question, because i tmight help control both the hypertens ion and the increased intraocular pressure?

a. Captopri lb. Di l tiazemc. Hydrochlorothiazided. Timolole. Verapami l

181. Nebivolol i s a fa i rly new and rather selective β1-adrenergic blocker that i s indicated for managing essentia l hypertens ion. The drug a l legedlycauses fewer s ide effects than most other β-blockers , and orthostatic hypotens ion does not occur even with ful l therapeutic doses . The drug hasno effects on changes of tota l periphera l res is tance induced by infus ion of phenylephrine or adminis tration an amphetamine. Nebivolol ’santihypertens ive mechanism of action clearly involves those common to a l l β-blockers , but another mechanism contributes to the overa l l effect.Given your genera l knowledge of blood pressure control and of auto-nomic/cardiovascular pharmacology, which one of the fol lowing i s the othermost l ikely mechanism by which nebivolol helps lower TPR and blood pressure?

a. Competi tively blocks postsynaptic (α1) adrenergic receptors

b. Competi tively blocks presynaptic (α2) adrenergic receptors

c. Increases ni tric oxide formationd. Increases synthes is of thromboxane A2

e. Inhibi ts catechol O-methyl transferase (COMT)

182. A 44-year-old woman is transported to the emergency department after consuming what might be a letha l dose of a β-adrenergic blocker thathas no intrins ic vasodi lator activi ty. Despi te adminis tration of large doses of i soproterenol to overcome the blockade, her cardiac output i s

Page 128: Pharmacology pre test self assessment and review (14th ed.)

dangerous ly low because heart rate and s troke volume are profoundly depressed. Which one of the fol lowing approaches i s most l ikely to provebeneficia l , i f not l i fesaving?

a. Adminis ter glucagonb. Adminis ter phenylephrinec. Adminis ter phentolamined. Switch from isoproterenol to ephedrinee. Switch from isoproterenol to epinephrine

183. At high (but not necessari ly toxic) blood levels , a cardiovascular drug causes many s igns and symptoms that resemble what you see with “low-grade” aspi rin toxici ty (sa l icyl i sm): l ight-headedness , tinni tus , and visua l dis turbances such as diplopia . What drug most l ikely caused theseresponses?

a. Atropineb. Captopri lc. Dobutamined. Propranolole. Quinidine

184. A patient has periodic episodes of paroxysmal supraventricular tachycardia (PSVT). What drug would be most sui table for outpatient prophylaxisof these worrisome electrophys iologic events?

a. Adenos ineb. Lidoca inec. Ni fedipined. Ni troglycerine. Verapami l

185. You and a col league are discuss ing which α-blocker to use, adjunctively, to control blood pressure in a pheochromocytoma patient beforesurgery for an adrenalectomy can be performed. Your col league correctly s tates that phenoxybenzamine i s the “preferred” drug. You s tate thatprazos in would be a better choice. Which s tatement about prazos in i s correct in comparison with phenoxybenzamine, and might actua l ly supportyour proposa l that i t would be a better choice?

a. Causes not only periphera l α-blockade but a lso suppresses adrenal epinephrine releaseb. Has a longer duration of action, which enables less frequent dos ingc. Has good intrins ic β-blocking activi ty, phenoxybenzamine does notd. Overdoses , and the hypotens ion i t may cause, are eas ier to manage pharmacologica l lye. Wi l l not cause orthostatic hypotens ion, which i s a common consequence of phenoxybenzamine

186. A patient with a his tory of atria l fibri l lation develops acute coronary syndrome and i s admitted to the hospi ta l ’s coronary care uni t. As oftoday, he has been receiving otherwise “proper” doses of a drug for 5 days s tra ight. Dos ing was done correctly, s tarting with usual maintenancedoses ; no loading dose s trategy was used. Then, and rather precipi tous ly, the patient develops s igns and symptoms of widespread thromboticevents , and platelet counts decl ine s igni ficantly concomitant with the thrombos is . The patient dies within 24 hours of the onset of s igns andsymptoms. What drug most l ikely caused these ul timately fata l responses?

a. Abciximabb. Clopidogrelc. Heparin (unfractionated)d. Ni fedipinee. Warfarin

187. A 65-year-old man with heart fa i lure i s unable to cl imb a fl ight of s ta i rs without experiencing dyspnea. After severa l years of therapy withcarvedi lol , captopri l , low doses of labeta lol , and furosemide, the therapeutic plan probably needs to change now. You empirica l ly add digoxin toimprove cardiac muscle contracti l i ty. Within 4 weeks , he has a marked improvement in his symptoms. What best describes the main cel lular actionof digoxin that accounts for i ts abi l i ty to improve his cardiovascular function and overa l l hemodynamic s tatus?

a. Activates β1-adrenergic receptors

b. Faci l i tates GTP binding to speci fic G proteinsc. Increases mitochondria l ca lcium (Ca 2+) released. Inhibi ts sarcolemmal Na +-K+-ATPasee. Stimulates cycl ic adenos ine 5′-monophosphate (cAMP) synthes is

188. A patient with newly diagnosed essentia l (primary) hypertens ion s tarts treatment with a commonly used antihypertens ive drug at a dose thatis cons idered to be therapeutic for the vast majori ty of patients . Soon after s tarting therapy the patient experiences crushing chest discomfort. ECGchanges show myocardia l i schemia. Studies in the cardiac cath lab show episodes of coronary vasospasm, and i t i s l i kely the antihypertens ivedrug provoked the vasospasm. Which antihypertens ive drug or drug class most l ikely caused the i schemia and the angina?

a. Atenololb. Di l tiazemc. Hydrochlorothiazided. Losartane. Metolazone

Page 129: Pharmacology pre test self assessment and review (14th ed.)

189. A patient with chronic-s table (“effort-induced”) angina i s on prophylactic β-blocker (propranolol ) therapy, with subl ingual ni troglycerin (NTG)used as needed for managing acute angina. One day he experiences particularly severe angina and takes the usual ly recommended dose ofsubl ingual NTG. His discomfort i s not reduced at a l l . Seeking rel ief, he repeats the usual recommended NTG dose frequently over a period ofabout 10 minutes , and now has taken far too much of the ni trovasodi lator. An electrocardiogram taken by the paramedics , who were ca l led for thepatient’s emergency, shows changes cons is tent with severe myocardia l i schemia. The patient goes into cardiac arrest and cannot be resusci tated.What i s the most l ikely cause of or contributing factor to the patient’s responses to the excess ive dosage of NTG?

a. Cyanide, a toxic metabol i te of NTG, accumulatedb. Ni troglycerin di rectly induced coronary vasoconstrictionc. The NTG lowered arteria l pressure excess ivelyd. The patient has vasospastic (variant or Prinzmeta l ) angina, not chronic-s tablee. The β-blocker’s intrins ic vasodi lator activi ty potentiated that of the NTG

190. A 50-year-old man has a long his tory of asymptomatic hyperuricemia (elevated uric acid levels ), and you are about to s tart therapy for newlydiagnosed essentia l hypertens ion (BP 136/90 mm Hg, based on repeated measurements with the patient supine and at rest). Whichantihypertens ive drug i s most l ikely to increase his uric acid levels further, and in doing so, be most l ikely to cause the most common cl inica lpresentation of the hyperuricemia—gout?

a. Hydrochlorothiazideb. Labeta lolc. Losartand. Ramipri le. Verapami l

191. We conduct an experiment to assess the effects of other drugs on the anti -platelet aggregatory effects of aspi rin. In each case we samplevenous blood from an otherwise heal thy patient, centri fuge the blood to obta in a platelet-rich fraction, and put samples of the platelets into aninstrument that accurately measures platelet aggregation in vitro. We then activate the platelets by adding ADP (or col lagen; makes no di fference).Resul ts are graphed below.

The control (top l ine) condition reflects resul ts from the patients who has been taking no drugs at a l l . The bottom trace shows aggregation ofplatelets i solated from the patient after they have taken aspirin (ASA; 81 mg/day) for 14 days in a row. We s top aspirin for 2 weeks to ensure thatthe patient has no aspi rin in thei r blood. Then we restart 81 mg/day aspirin plus an unknown drug taken with each aspirin tablet for fina l 2 weeks .The platelets taken at the end of the 14 days show only s l ight inhibi tion of platelet aggregation—resul ts not s igni ficantly from control .

The resul ts are shown here:

Page 130: Pharmacology pre test self assessment and review (14th ed.)

Which drug was taken with the aspi rin and inhibi ted i ts aggregatory effects in response to ADP or col lagen?

a. Acetaminophenb. Clopidogrelc. Dabigatrand. Ibuprofene. Warfarin

192. A 57-year-old patient compla ins of muscle aches , pa in, and tenderness . These affect the legs and trunk. There i s no fever, bruis ing, or anyrecent his tory of muscle trauma or s tra ins (as from excess ive exercise). He has myoglobinuria , a cl inica l ly s igni ficant fa l l of creatinine clearance,and a ri se of plasma creatine kinase (CK) to levels nearly ten times the upper l imit of normal . Which drug i s the most l ikely cause of thesefindings?

a. Aspirin (low dose) for i ts cardioprotective/antiplatelet effectsb. Captopri l for hypertens ion and heart fa i lurec. Carvedi lol for hypertens ion, heart fa i lure, and angina prophylaxisd. Furosemide as adjunctive management of his heart fa i luree. Rosuvastatin to control his hypercholesterolemia and the associated ri sks

193. A coronary artery sample was removed from a heal thy animal , put ins ide a sui table oxygenated sa l t and nutrient solution, and connected to atransducer that measured increases (contraction) or decreases (relaxation) of smooth muscle tens ion (force). ACh was then added to give thecumulative concentrations shown below.

As expected, in the control setting (left) ACh caused concentration-dependent vasorelaxation (equiva lent to vasodi lation in the intact animal ).The ACh was washed out severa l times , control conditions returned. The conditions were manipulated, and then the ACh dose-responseexperiment was repeated (right). Now the data show increased tens ion developed by the muscle sample (vasoconstriction in vivo) in response toACh and incrementa l increases of i ts concentration.

Page 131: Pharmacology pre test self assessment and review (14th ed.)

Which one of the fol lowing summarizes what was most l ikely done to the vessel under the experimenta l conditions , before retesting theresponses to added ACh? Assume that the vascular responses in this animal model are identica l to those that would occur in a human.

a . Endothel ium was removed mechanica l lyb. Isoproterenol was added right before AChc. Muscarinic receptors were blocked with atropined. Sample was pretreated with prazos ine. Tissue was pretreated with botul inum toxin

194. A 28-year-old woman is receiving drug therapy for essentia l (primary) hypertens ion. She subsequently becomes pregnant. You rea l i ze that thedrug she’s been taking for her high blood pressure can have serious , i f not fata l , effects on the fetus (i t i s in pregnancy category X). As a resul t, youstop the current antihypertens ive drug and substi tute another that i s deemed to be equiefficacious in terms of her blood pressure, and safer forthe fetus . Which drug was she most l ikely taking before she became pregnant?

a. α-Methyldopab. Captopri l

Page 132: Pharmacology pre test self assessment and review (14th ed.)

c. Furosemided. Labeta lole. Verapami l

195. We treat a patient with a drug that affects the clotting-thrombolytic systems for a time sufficient to let the drug’s effects and blood levelss tabi l i ze at a therapeutic level . We then i solate platelets from a blood sample and test thei r in vi tro aggregatory responses to ADP, col lagen, PAF,and thromboxane A2. Aggregatory responses to ADP are inhibi ted; responses to the other platelet proaggregatory agonis ts are unaffected. Whichdrug exhibi ts these properties?

a. Aspirinb. Biva l i rudinc. Clopidogreld. Heparine. Warfarin

196. Your newly diagnosed hypertens ive patient has vasospastic angina. Which drug or drug class would be the most rational for s tarting anti -hypertens ive therapy because i t exerts not only antihypertens ive effects , but a lso di rectly lowers myocardia l oxygen demand and consumption andtends to inhibi t cel lular processes that otherwise favor coronary vaso-spasm? Assume there are no other speci fic contra indications to the drug youchoose.

a . Angiotens in-converting enzyme (ACE) inhibi tor or angiotens in receptor blockerb. β-Adrenergic blockerc. Ni fedipined. Thiazide diuretice. Verapami l (or di l tiazem)

197. You are reviewing the medication his tory of a 59-year-old man. He has been taking ramipri l and pravastatin for the las t 5 years . Othermedications include metformin for type 2 diabetes mel l i tus , and esci ta lopram to help manage his depress ion. At his las t cl inic vi s i t, a year ago,he was told to continue his current medications but he was a lso s tarted on s low-release niacin because diet, exercise and other l i fes tylemodi fications , and his current medications were not adequate. What was the most l ikely reason for adding the niacin?

a. Counteract deficiencies of B-vi tamin absorption caused by the antidepressantb. Counteract polyphagia , and over-eating, caused by the metformin.c. Lower HDL and triglyceride levels that did not respond adequately to the s tatind. Prevent s tatin-induced neuropathye. Slow the progress ion of diabetic nephropathy caused by the ACE inhibi tor

198. Quinidine i s ordered for a patient with recurrent atria l fibri l lation and who refuses any interventions other than drugs in an attempt toterminate and control the arrhythmia. He has some pulmonary fibros is and a thyroid disorder—both leading you to conclude that amiodaronetherapy might not be the best approach. Which s tatement appl ies to the quinidine?

a. Decreases SA nodal automatici ty due to a s trong antichol inergic/vagolytic effectb. Is l ikely to increase blood pressure via a di rect vasoconstrictor effectc. Is contra indicated i f the patient a lso requires anticoagulant therapyd. Tends to increase electrica l impulse conduction veloci ty through the AV nodee. Wi l l increase cardiac contracti l i ty (pos i tive inotropic effect) independent of i ts antiarrhythmic effects

199. A woman has received a cardiovascular drug that i s absolutely contra indicated (category X) in pregnancy, but in the absence of pregnancywould have been deemed beneficia l because of a particular cardiovascular disorder she has . Unfortunately, nei ther she nor her phys ician (whomshe rarely vis i ted) knew she was pregnant unti l she had just s tarted the second trimester. The drug i s s topped (and a sui table and safera l ternative i s s tarted), but i t i s too late. Her baby i s del ivered prematurely, and s ti l lborn. It i s obvious from examination of the baby that there i s anasa l deformity: the nose i s flattened into the face, with no apparent bridge. Standard x-rays reveal s tippl ing of a l l the epiphyses . Theseresponses are characteris tic of which drug?

a. Clonidineb. Heparin, low molecular weight (eg, enoxaparin)c. Hydrochlorothiazided. Ni troglycerine. Warfarin

200. A patient who has been taking an ora l antihypertens ive drug for about a year develops a pos i tive Coombs’ test, and now you are worriedabout the poss ibi l i ty (a l though low) that hemolytic anemia may develop i f the drug i s continued. Which drug was the most l ikely cause?

a. Captopri lb. Clonidinec. Labeta lold. Methyldopae. Prazos in

201. A patient presents with severe hypertens ion and tachycardia . Blood chemistry resul ts , radiologic s tudies , and the overa l l cl inica l presentationpoint to pheochromocytoma. The tumor appears operable, but the patient wi l l have to wait a couple of weeks for the adrenalectomy. We prescribephenoxybenzamine in the interim, with the goal of suppress ing some of the major s igns and symptoms caused by the tumor and the mass ive

Page 133: Pharmacology pre test self assessment and review (14th ed.)

amounts of epinephrine i t i s releas ing. Which of the fol lowing best summarizes what phenoxybenzamine does , or how i t acts?

a. Controls blood pressure by blocking α-adrenergic receptors in the periphera l vasculatureb. Controls heart rate by selectively blocking β1-adrenergic receptors

c. Inhibi ts catecholamine synthes is in the adrenal (suprarenal ) medul lad. Lowers blood pressure by inhibi ting angiotens in converting enzymee. Stimulates catechol -O-methyl transferase, thereby faci l i tating epinephrine’s metabol ic inactivation

202. A 30-year-old man who has a his tory of as thma has just been diagnosed Stage 1 essentia l hypertens ion. He regularly uses an inhaledcorticosteroid, which seems to work wel l as a control medication, but a lso needs to use an a lbuterol inhaler about once every 3 weeks forsuppress ion of as thma attacks (rescue therapy). Which antihypertens ive drug or drug class poses the greatest ri sk of exacerbating the patient’sasthma and counteracting the des i red pulmonary effects of the a lbuterol , even though i t might control his blood pressure wel l?

a . Di l tiazemb. Hydrochlorothiazidec. Labeta lold. Ramipri le. Verapami l

203. Digoxin affects a host of cardiac electrophys iologic properties . Some of i ts effects are caused di rectly by the drug. Others are indi rect: they mayinvolve increas ing “vagal tone” to the heart or other compensations that ari se when cardiac output i s improved in a patient with heart fa i lure. Forsome parameters the di rect and indirect effects may be qual i tatively (but not quanti tatively) oppos ing, but one wi l l predominate over the other.What i s an expected and usual ly predominant effect of the drug?

a. Increased rate of SA nodal depolarizationb. Reduced atria l automatici tyc. Reduced ventricular automatici tyd. Slowed AV nodal conduction veloci tye. Slowed conduction veloci ty through the atria l myocardium and His -Purkinje system

204. A patient has Stage 2 essentia l hypertens ion and heart fa i lure. After eva luating the responses to many other antihypertens ive drugs , a loneand in combination, the phys ician concludes that i t would be reasonable to try hydra lazine. Which drug(s ) i s /are l ikely to be needed, as add-ons(adjuncts ), to manage the expected and unwanted cardiovascular and renal s ide effects of the hydra lazine?

a. Captopri l plus ni fedipineb. Digoxin plus spi ronolactonec. Digoxin plus vi tamin Kd. Hydrochlorothiazide and a β-blockere. Ni troglycerinf. Triamterene plus ami loride

205. A heal thy adult subject participating in a cl inica l tria l i s given an intravenous injection of a test drug. Both blood pressure and tota lperiphera l res is tance ri se promptly. This i s fol lowed immediately by a reduction of heart rate. In repeated experiments we find that thevasopressor response i s not affected by pretreatment with prazos in. However, pretreatment with atropine prevents the cardiac chronotropicresponse. The test drug was most l ikely which of the fol lowing?

a. Angiotens in I Ib. Dobutaminec. Isoproterenold. Norepinephrinee. Phenylephrine

206. We are adminis tering ni tropruss ide intravenous ly for control of severe hypertens ion during surgery. The dose has gotten too high, and thedrug has been adminis tered too long. Refractoriness to the antihypertens ive effects has occurred. Blood pressure i s ri s ing, and other s igns andsymptoms of potentia l ly severe toxici ty develop. What ni tropruss ide metabol i te accounts for or at least contributes to these problems?

a. A highly efficacious α-adrenergic agonis tb. An extraordinari ly potent and i rrevers ible Na-K-ATPase inhibi torc. An i rrevers ible antagonis t for angiotens in at the A-II receptorsd. Cyanidee. Ni tric oxide

207. A 66-year-old man who l ives in a smal l rura l town, and who has been treated by his fami ly doctor for decades , presents at your medica l center.He has coronary atheroscleros is and “mi ld” heart fa i lure that has been treated for the las t 10 years with digoxin and severa l other drugs . His chiefcompla ints are nausea, vomiting, and diarrhea, which have not resolved despi te a recommendation from his phys ician to take prescriptionmedications for those conditions . His ECG reveals a bigeminal rhythm and second-degree heart block. A drug-drug interaction i s suspected. Whatcoadminis tered drug most l ikely provoked the problem?

a. Captopri lb. Cholestyraminec. Furosemided. Lovastatin

Page 134: Pharmacology pre test self assessment and review (14th ed.)

e. Ni troglycerin

208. We have a patient who i s diagnosed with variant (vasospastic) angina. Which drug would be most appropriate, and genera l ly regarded asmost effective, for long-term therapy a imed at reducing the incidence or severi ty of the coronary vasospasm?

a. Aspirinb. Atorvastatinc. Di l tiazemd. Ni troglycerine. Propranolol

209. A 56-year-old man has heart fa i lure. His fami ly doctor, who has been treating him s ince he was a young lad, has been treating him withdigoxin, furosemide, and triamterene for severa l years . The patient now develops atria l fibri l lation, and so his doctor s tarts quinidine andclopidogrel . What i s the most l ikely outcome of adding the quinidine?

a. Development of s igns and symptoms of quinidine toxici ty (cinchonism)b. Hyponatremia due to quinidine’s abi l i ty to enhance diuretic-induced sodium lossc. Onset of s igns and symptoms of digoxin toxici tyd. Precipi tous development of hypokalemiae. Prompt suppress ion of cardiac contracti l i ty, onset of acute heart fa i lure

210. Fleca inide and propafenone are in Vaughan-Wi l l iams (antiarrhythmic) Class I-C. What i s the cl inica l ly relevant “take home” message aboutthis class of drugs?

a. Are only given for arrhythmias during acute myocardia l infarctionb. Are particularly sui ted for patients with low ejection fractions or cardiac outputc. Are preferred drugs (drugs of choice) for relatively innocuous ventricular arrhythmiasd. Cause pulmonary fibros is and a hypothyroid-l ike syndrome when given long terme. Have a s igni ficant pro-arrhythmic effect (induction of letha l arrhythmias )

211. You want to compare and contrast the cardiac and hemodynamic profi les of immediate-acting dihydropyridine-type ca lcium channel blockers(CCBs) and the nondihydropyridine, verapami l (or di l tiazem). Which of the fol lowing best summarizes how, in genera l , a nondihydropyridine CCBdi ffers from ni fedipine?

a. Causes a much higher incidence of reflex tachycardiab. Causes s igni ficant dose-dependent s lowing of AV nodal conduction veloci tyc. Causes s igni ficant venodi lation, leading to profound orthostatic hypotens iond. Has s igni ficant and di rect pos i tive inotropic effectse. Is best used in conjunction with a β-blocker or digoxin

212. A patient has received excess ive doses of ni tropruss ide, and toxic mani festations are developing in response to a metabol i te. Which of thefol lowing drugs or drug groups would be most effective in l imiting and ul timately counteracting the effects of the toxic product?

a. Aminocaproic acidb. Hydroxycobalaminc. Protamine sul fated. Thrombine. Vi tamin K

213. A patient with Stage 2 essentia l hypertens ion i s treated with usual ly effective doses of an ACE inhibi tor. After a sui table period of time, bloodpressure has not been lowered satis factori ly. The patient has been compl iant with drug therapy and other recommendations (eg, weightreduction, exercise). A thiazide i s added to the ACE inhibi tor regimen. What i s the most l ikely and earl ies t (and probably trans ient) untowardoutcome of this drug add-on, for which you should monitor closely?

a. Fa l l of blood pressure sufficient to cause syncopeb. Hypokalemia due to synergis tic effects of the ACE inhibi tor and the thiazide on renal potass ium excretionc. Onset of acute heart fa i lure from depress ion of ventricular contracti l i tyd. Paradoxica l hypertens ive cri s i se. Sudden prolongation of the P-R interva l and increas ing degrees of heart block

214. A 45-year-old man postmyocardia l infarction (MI) i s being treated with severa l drugs , including intravenous unfractionated heparin. Stoolguaiac on admiss ion was negative, but i s now 4+, and he has had an episode of hematemes is . What would be the best drug to adminis ter tocounteract the effects of excess ive heparin remaining in the ci rculation?

a. Aminocaproic acidb. Dipyridamolec. Factor IXd. Protamine sul fatee. Vi tamin K

215. A 45-year-old man asks his phys ician for a prescription for s i ldenafi l to improve his sexual performance. Because of ri sks from a serious druginteraction, this drug should not be prescribed, and the patient should be urged not to try to obta in i t from other sources , i f he i s a l so taking whichof the fol lowing drugs?

Page 135: Pharmacology pre test self assessment and review (14th ed.)

a. Angiotens in-converting enzyme inhibi torb. β-Adrenergic blockerc. Ni trovasodi lator (eg, ni troglycerin)d. Statin-type antihypercholesterolemic druge. Thiazide or loop diuretic

216. A phys ician i s preparing to adminis ter a drug for which there i s a label warning: “do not adminis ter this drug to patients with second-degreeor greater heart block, or give with other drugs that may cause heart block.” Which finding would be speci fica l ly indicative of heart block, andsecond-degree heart block in particular?

a. Auscul tation of the precordium reveals an i rregular rhythmb. Blood pressure i s lowc. Heart rate i s abnormal ly low (bradycardia), but there i s normal s inus rhythmd. ECG reveals ventricular ectopic beatse. ECG shows an excess ively prolonged PR interva l , and some P waves are not fol lowed by a normal QRS complexf. ECG shows abnormal ly widened QRS complexes

217. A 52-year-old woman with essentia l hypertens ion, hypercholesterolemia, and chronic-s table angina develops severe constipation. It i sattributed to one of her medications . What was the most l ikely cause?

a. Atorvastatinb. Captopri lc. Labeta lold. Ni troglycerine. Verapami l

218. We use s tandard invas ive hemodynamic techniques to measure or ca lculate the effects of various drugs on such parameters as arteria lpressure, tota l periphera l res is tance, and centra l venous (right atria l ) pressures . Our goal i s to eva luate whether the drug primari ly causesarteriolar or venular di lation, or affects both s ides of the ci rculation. Which drug exerts vasodi lator effects only in the arteria l s ide of theci rculation?

a. Hydra lazineb. Losartanc. Ni fedipined. Ni troglycerine. Prazos in

219. A 20-year-old col legiate vars i ty hockey player i s referred to you by his coach. The young athlete has excess ive bruis ing after a very phys ica lmatch 2 days before. His knee had been bothering him, so he took two 325 mg aspirin tablets severa l hours before the contest. He got checkedhard into the boards many times during the game, but denies any excess ive or unusual trauma. As you ponder the s i tuation you order severa lblood tests . Which test or finding do you most l ikely expect to be abnormal as a resul t of the prior aspi rin use?

a. Activated partia l thromboplastin time (APTT)b. Bleeding timec. INR (International Normal ized Ratio)d. Platelet counte. Prothrombin time

220. A patient in the coronary care uni t develops episodes of paroxysmal AV nodal reentrant tachycardia (PSVT). What drug would genera l ly becons idered fi rs t-l ine for promptly s topping the arrhythmia?

a. Adenos ineb. Digoxinc. Edrophoniumd. Phenylephrinee. Propranolol

221. A 60-year-old man, hospi ta l i zed for an acute myocardia l infarction, i s treated with warfarin (among other drugs). What i s the main mechanismby which warfarin i s caus ing the effects for which i t i s given?

a. Increase in the plasma level of Factor IXb. Inhibi tion of thrombin and early coagulation s tepsc. Inhibi tion of synthes is/activation of prothrombin and Factors VII, IX, and Xd. Inhibi tion of platelet aggregatione. Activation of plasminogenf. Binding of Ca 2+ ion cofactor in some coagulation s teps

222. A 42-year-old man with an acute MI i s treated with a l teplase, and electrocardiographic and hemodynamic s tatus improve quickly. By whatmechanism did the a l teplase cause i ts beneficia l effects?

a. Blocked platelet ADP receptorsb. Inhibi ted platelet thromboxane production

Page 136: Pharmacology pre test self assessment and review (14th ed.)

c. Inhibi ted synthes is of vi tamin K-dependent coagulation factorsd. Prevented aggregation of adjacent platelets by blocking glycoprotein IIb/II Ia receptorse. Promoted convers ion of plasminogen to plasmin

223. A patient with atria l fibri l lation i s placed on long-term arrhythmia control with amiodarone. In addition to “s tandard” monitoring, whatshould be assessed periodica l ly in order to detect adverse effects that are rather unique to this drug among vi rtua l ly a l l the antiarrhythmics?

a. Blood glucose, triglyceride, cholesterol , and sodium concentrationsb. Hearing thresholds (audiometry) and plasma a lbumin concentrationc. Prothrombin time and antinuclear antibody (ANA) ti tersd. Pulmonary function and thyroid hormone s tatuse. White cel l counts and blood urate concentration

224. A 64-year-old woman has had severa l episodes of trans ient i schemic attacks (TIAs ). Aspirin would be a preferred treatment for prevention ofthrombos is , but she has a his tory of severe “aspirin sens i tivi ty” mani fest as intense bronchoconstriction and urticaria . What would you cons ider tobe the best a l ternative to the aspi rin?

a. Acetaminophenb. Aminocaproic acidc. Clopidogreld. Dipyridamolee. Streptokinase

225. A patient who has excess ively s low AV nodal conduction rates , that unfortunately haven’t been recognized, i s s tarted on a drug. As soon asblood levels cl imb toward the usual therapeutic range the patient goes into complete heart block. Which drug most l ikely provoked this furtherprolongation of the P-R interva l , ul timately leading to the cessation of a l l AV nodal conduction?

a. Captopri lb. Losartanc. Ni fedipined. Ni troglycerine. Prazos inf. Verapami l

226. A patient with heart fa i lure, Stage 1 essentia l hypertens ion, and hyperl ipidemia (elevated LDL cholesterol and abnormal ly low HDL-C) i staking furosemide, captopri l , atenolol , and s imvastatin (an HMG-CoA reductase inhibi tor).

During a scheduled phys ica l exam, about a month after s tarting a l l the above drugs , the patient reports a severe, hacking, and relentless cough.Other vi ta l s igns , and the overa l l phys ica l assessment, are cons is tent with good control of both the heart fa i lure and blood pressure and indicateno other underlying disease or abnormal i ties . Resul ts of blood tests are not yet ava i lable.

Which of the fol lowing i s the most l ikely cause of the cough?

a. An expected s ide effect of the captopri lb. An a l lergic reaction to the s tatinc. Dyspnea due to captopri l ’s known and powerful bronchoconstrictor actiond. Excess ive doses of the bumetanide, which led to hypovolemiae. Hyperka lemia caused by an interaction between bumetanide and captopri lf. Pulmonary edema from the bumetanide

227. A patient with a his tory of hypertens ion, heart fa i lure, and periphera l vascular disease has been on ora l therapy, with drugs sui table for each,for about 3 months . He runs out of the medication and plans to have the prescriptions refi l led in a week or so.

Within a day or two after s topping his medications he experiences an episode of severe tachycardia accompanied by tachyarrhythmias , and anabrupt ri se of blood pressure to 240/140 mm Hg—wel l above pretreatment levels . He compla ins of chest pa in, anxiety, and a pounding headache.Soon thereafter he suffers a hemorrhagic s troke.

Which drug or drug group that the man was taking and s topped taking suddenly most l ikely caused these responses?

a. ACE inhibi torb. Clonidinec. Digoxind. Furosemidee. Ni fedipine (a long-acting formulation)f. Warfarin

228. Your class just completed a mini review of the phys iology and pathophys iology of blood pressure control and fina l ly dawned on a classmate:bradykinin, an endogenous vasodi lator, contributes to keeping blood pressure low; and i ts metabol i te, formed by an enzyme cleverly ca l led brady-kininase, lacks vasodi lator activi ty. A col league says “wouldn’t i t be great i f we had a drug that could inhibi t bradykinin’s metabol ic inactivation?We could probably use i t as an antihypertens ive drug!” You reply that we a l ready have a drug—severa l , in fact—that does that very thing. Whatdrug or drug group would that be?

a. Atenolol or metoprololb. Captopri l or other “pri l s”c. Hydrochlorothiazide or s imi lar diuretic-antihypertens ives

Page 137: Pharmacology pre test self assessment and review (14th ed.)

d. Labeta lol or carvedi lole. Losartan or other “sartans”

229. A 70-year-old woman is treated with subl ingual ni troglycerin for occas ional bouts of effort-induced angina. What best describes part of themechanisms by which ni troglycerin causes i ts des i red antiangina l effects?

a. Blocks α-adrenergic receptorsb. Forms cyanide, much l ike the metabol i sm of ni tropruss ide doesc. Increases loca l synthes is and release of adenos ined. Ra ises intracel lular cGMP levelse. Selectively di lates/relaxes coronary arteries

230. Your patient has bipolar i l lness , hypercholesterolemia, chronic-s table angina, and Stage 1 essentia l hypertens ion. He has been takingl i thium and an SSRI for the bipolar i l lness . Cardiovascular drugs include atorvastatin, di l tiazem, subl ingual ni troglycerin, captopri l , andhydrochlorothiazide. What outcome, due to interactions involving these drugs , should you most l ikely expect?

a. Development of acute psychos is from an ACE inhibi tor-antipsychotic interactionb. Development of a hypomanic s tate from antagonism of l i thium’s action by the ni troglycerinc. Li thium toxici ty because of hyponatremia caused by the hydrochlorothiazided. Loss of cholesterol control from antagonism of the HMG Co-A reductase inhibi tor by the antipsychotice. Worsening of angina because the antipsychotic counteracts the effects of the ca lcium channel blockerf. Worsening of angina because the l i thium antagonizes the effects of the ni troglycerin

231. A fi rs t-year house officer notices that a patient i s experiencing s igni ficant and rapidly ri s ing blood pressure (currently 180/120 mm Hg). One ofthe medications the patient had been taking i s immediate-acting ni fedipine ora l capsules . There i s a dose of this ni fedipine formulation at thebeds ide, so the phys ician pricks the capsule open and squirts the contents into the patient’s mouth. This technique avoids “fi rs t-pass”metabol i sm of the drug and causes rapid absorption and a l l the effects associated with this ca lcium channel blocker. What i s the most l ikelyoutcome of giving ni fedipine as described here?

a. AV nodal blockb. Further ri se of heart rate, worsening of the ventricular arrhythmiac. Hypotens ion and bradycardiad. Normal ization of blood pressure and heart ratee. Return of blood pressure toward normal , no s igni ficant effect on heart rate or the ECG

232. A 55-year-old patient with multiple cardiovascular diseases i s being treated with digoxin, furosemide, triamterene, atorvastatin, andnitroglycerin—al l prescribed by the fami ly phys ician he’s had for decades . The patient now experiences nausea, vomiting, and anorexia , anddescribes a “yel lowish-greenish tint” to white objects and bright l ights . These s igns and symptoms are most characteris tic of toxici ty due to whichdrug?

a. Atorvastatinb. Digoxinc. Furosemided. Ni troglycerine. Triamterene

233. A 28-year-old female patient has Stage 1 essentia l hypertens ion (resting BP 144/98), tachycardia , and occas ional pa lpi tations (ventricularectopic beats ). Normal ly we might cons ider prescribing a β-blocker to control the blood pressure and cardiac responses , but our patient a lso hasasthma, and she i s trying to get pregnant. Which drug would be the best a l ternative to the β-blocker in terms of l ikely efficacy on pressure andheart rate, and in terms of relative safety?

a. Di l tiazemb. Enalapri lc. Furosemided. Phentolaminee. Prazos in

234. A patient presents with hypertens ion. The underlying cause—a pheochromocytoma—is not looked-for or detected in the ini tia l work-up. Anora l antihypertens ive drug i s prescribed. We soon find that the patient’s blood pressure has ri sen to levels above pretreatment levels—so muchso that we are worried about imminently dangerous effects from the drug-induced worsening of hypertens ion—in response to a drug. Concomitantwith the drug-induced ri se of blood pressure the patient develops s igns and symptoms of heart fa i lure. Which drug was most l ikely adminis tered?

a. Captopri lb. Hydrochlorothiazidec. Labeta lold. Losartane. Propranololf. Verapami l

235. A patient on long-term warfarin therapy has an INR that i s excess ive (4.5; normal , not anticoagulated i s 1.0; the target for this patient was 2.5).She reports episodes of epis taxis over the las t 2 days and now there i s a great ri sk of serious bleeding episodes . In addition to s topping warfarinadminis tration for a day or more, which drug would you want to adminis ter to counteract warfarin’s excess ive effects that led to spontaneous

Page 138: Pharmacology pre test self assessment and review (14th ed.)

bleeding?

a. Aminocaproic acidb. Epoetin a l fac. Ferrous sul fated. Phytonadione (vi tamin K)e. Protamine sul fate

236. A patient with hypertens ion and heart fa i lure has been treated for 2 years with carvedi lol and l i s inopri l . He has just had hip replacementsurgery, but because he i s not ambulating he i s s tarted on unfractionated heparin, postoperatively, for prophylaxis of deep venous thrombos is .Ora l antacids and esomeprazole (gastric parieta l cel l proton pump inhibi tor) have been added for prophylaxis of acute s tress ulcers . Five dayspostop, he experiences sudden onset dyspnea and electrocardiographic and other indications of an acute MI. The patient’s platelet counts aredangerous ly low. What i s the most l ikely underlying problem?

a. Accidenta l substi tution of low-molecular-weight heparins (LMWH) for unfractionated heparinb. Accidenta l/inadvertent aspi rin adminis trationc. Hemolytic anemia from a carvedi lol -ACE inhibi tor interactiond. Heparin-induced thrombocytopeniae. Reduced heparin effects by increased metabol ic clearance (caused by rani tidine)

237. A patient with angina pectoris i s s tarted on a ni troglycerin transdermal del ivery system (“skin patch”) for prophylaxis of his angina. He wearsthe patch 24 hours a day, 7 days a week, except for the few minutes when he showers each day. What i s the main concern with “around-the-clock”adminis tration of this or other long-acting formulations of ni trovasodi lators?

a. Cyanide poisoningb. Development of tolerance to thei r vasodi lator actionsc. Gradual development of reflex bradycardia in response to success ive dosesd. Onset of delayed, characteris tic adverse responses including thrombos is and thrombocytopeniae. Paradoxica l vasoconstriction leading to hypertens ion

238. For many hypertens ive patients we can prescribe ei ther l i s inopri l (or an a l ternative in the same class ) or losartan. What s tatement correctlysummarizes how losartan di ffers from l i s inopri l other l i s inopri l -l ike drugs?

a. Li s inopri l competi tively blocks catecholamine-mediated vasoconstriction, losartan does notb. Li s inopri l effectively inhibi ts synthes is of angiotens in I I , losartan does notc. Losartan causes a higher incidence of bronchospasm and hyperuricemiad. Losartan i s preferred for managing hypertens ion during pregnancy, whereas captopri l i s contra indicatede. Losartan i s sui table for adminis tration to patients with heart fa i lure, whereas captopri l and related drugs should be avoided.

239. A 46-year-old man has Stage 1 essentia l hypertens ion (resting BP 150/98), primary hypercholesterolemia, and modestly elevated fastingglucose levels (130 mg/dL) measured on severa l occas ions . His cholesterol levels (tota l , HDL, LDL) have not been acceptably modi fied by dietarychanges and da i ly use of a “s tatin.” The phys ician adds ezetimibe to the regimen. Which s tatement summarizes ezetimibe’s actions , or whatwould be expected in response to i ts use?

a. Exerts profound cardiac negative inotropic effects that poses a ri sk of heart blockb. Frequently causes orthostatic hypotens ion that in turn triggers reflex cardiac s timulationc. More l ikely than other drugs to increase the ri sk of severe s tatin-induced myopathyd. Reduces intestina l cholesterol uptake, has no di rect hepatic effect to inhibi t cholesterol synthes ise. Signi ficantly increases ri sk of atherosclerotic plaque rupture

240. A 58-year-old man presents in the emergency department with his fi rs t episode of acute coronary syndrome (ACS) and a l l evidence points to amyocardia l infarction. Angioplasty and s tenting are not poss ible because the cardiac cath lab i s busy with other higher-priori ty patients , soadminis tration of a thrombolytic drug i s the only option. What i s the most important determinant, overa l l , of the success of thrombolytic therapy interms of sa lvaging viable cardiac muscle (or other i schemic ti ssues)?

a. Choos ing a “human” (cloned) plasminogen activator (eg, t-PA), rather than one that i s bacteria l -derived (eg, s treptokinase)b. Infarct location (ie, anterior wal l of left ventricular vs another s i te/wal l )c. Presence of col latera l blood vessels to the infarct-related coronary arteryd. Systol ic blood pressure at the time the MI i s diagnosede. Time from onset of infarction to adminis tration of the thrombolytic agent

241. A patient with an acute coronary syndrome is given a variety of cardiovascular drugs as he i s being readied for transport to the “cath lab” forposs ible placement of a s tent. One of the meds i s abciximab. What best describes the mechanism of action of this drug?

a. Blocks thrombin receptors selectivelyb. Blocks ADP receptorsc. Blocks glycoprotein IIb/II Ia receptorsd. Inhibi ts cyclooxygenasee. Inhibi ts prostacycl in production

242. A patient presents in the emergency department with acute hypotens ion that requires treatment. Hypovolemia i s ruled-out as a cause orcontributor, and information gathered from the patient and fami ly indicates that the cause i s overdose of an antihypertens ive drug.

Page 139: Pharmacology pre test self assessment and review (14th ed.)

One approach to treatment i s to adminis ter a pharmacologic (ordinari ly effective) dose of phenylephrine, an α-adrenergic agonis t. You do justthat, and blood pressure fa i l s to ri se at a l l—and a second dose doesn’t work ei ther. On which antihypertens ive drug did the patient most l ikelyoverdose?

a. Captopri l or another ACE inhibi torb. Hydra lazinec. Prazos ind. Thiazide diuretic (eg, hydrochlorothiazide)e. Verapami l

243. An elderly man who has just been referred to your practice has been taking a drug for symptomatic rel ief of benign prostatic hypertrophy. Inaddition to i ts effects on smooth muscles of the prostate and urethra , this drug can lower blood pressure in such a way that i t reflexly triggerstachycardia , pos i tive inotropy, and increased AV nodal conduction. The drug nei ther di lates nor constricts the bronchi . It causes the pupi l s of theeyes to constrict and interferes with mydrias is in dim l ight. Ini tia l ora l dosages of this drug have been associated with a high incidence ofsyncope.

Which prototype i s most s imi lar to this unnamed drug in terms of the pharmacologic profi le?

a. Captopri lb. Hydrochlorothiazide (prototype thiazide diuretic)c. Labeta lold. Ni fedipinee. Prazos inf. Propranololg. Verapami l

244. You are contemplating s tarting ACE inhibi tor therapy for a patient with essentia l hypertens ion. Which one of the fol lowing patient-relatedcondition(s ) contra indicates use of any ACE inhibi tor and so should be ruled out before you prescribe this drug?

a. Asthmab. Heart fa i lurec. Hyperl ipidemia, coronary artery diseased. Hypokalemiae. Is a woman who i s pregnant or may become pregnant

245. A patient develops s inus bradycardia . Heart rate i s dangerous ly low, and an effective and safe drug needs to be given right away. Which drugwould be the best choice for normal izing heart rate without ini tiating any other arrhythmias?

a. Atropineb. Amiodaronec. Edrophoniumd. Lidoca inee. Phentolamine

246. A patient presents in the emergency department (ED) with severe angina pectoris , and acute myocardia l i schemia i s confi rmed byelectrocardiographic and other cl inica l indicators . Unknown to the ED team is the fact that the i schemia i s due to coronary vasospasm, not tocoronary occlus ion with thrombi . Given this etiology, which drug, adminis tered in usual ly effective doses , may actua l ly make the vasospasm, andthe resul ting i schemia, worse?

a. Al teplase (t-PA)b. Aspirinc. Captopri ld. Ni troglycerine. Propranololf. Verapami l

247. Many cl inica l s tudies have investigated the benefi ts of da i ly aspi rin use in the primary prevention of coronary heart disease and suddendeath in adults . The resul ts have been somewhat incons is tent, in part because di fferent dosages were s tudied, and there were importantdi fferences in the populations that were s tudied. Nonetheless , many (i f not most) of the s tudies have revealed that for some patients aspi rinincreased the incidence of a particularly unwanted adverse response, even when dosages were kept within the range typica l ly recommended forcardioprotection (eg, 81-162 mg/day). What i s the most l ikely adverse response associated with aspi rin prophylaxis , particularly in patients whohave a low ri sk of an acute coronary syndrome or cardiovascular disease in genera l?

a . Centrolobular hepatic necros isb. Hemorrhagic s trokec. Nephropathyd. Tachycardia and hypotens ion leading to acute myocardia l i schemiae. Vasospastic angina

248. A patient with essentia l hypertens ion has been treated with a fixed-dose combination product that conta ins hydrochlorothiazide andtriamterene. Blood pressure and electrolyte profi les have been kept within acceptable l imits for the las t 18 months . Now, however, blood pressurehas ri sen to the point where the phys ician wants to add another antihypertens ive drug. The drug i s s tarted; a fter severa l weeks blood pressurefa l l s into an acceptable range, but the patient has become hyperka lemic. What drug was added and was most l ikely respons ible for the des i red

Page 140: Pharmacology pre test self assessment and review (14th ed.)

blood pressure fa l l and the unwanted ri se of potass ium levels .

a . Di l tiazemb. Prazos inc. Propranolold. Ramipri le. Verapami l

249. A patient has a supraventricular tachycardia . We inject a drug and heart rate fa l l s to a normal (or at least more acceptable) level . Al thoughthis drug caused the des i red response, i t did so without any di rect effect in or on the heart. Which drug was most l ikely used?

a. Edrophoniumb. Esmololc. Phenylephrined. Propranolole. Verapami l

250. A 69-year-old man presents with NYHA Stage II (“mi ld”) heart fa i lure. His symptoms fa i led to improve adequately in response to captopri l andcarvedi lol so the phys ician s tops the carvedi lol and adds usual therapeutic doses of digoxin and furosemide. At a fol low-up exam 3 months laterwe find good symptomatic rel ief of the heart fa i lure. Blood electrolytes and a l l other lab tests are within normal l imits . At this time, whichelectrocardiographic change would you expect to see in response to the digoxin, compared with a basel ine (pretreatment) ECG?

a. P waves widened, ampl i tude increasedb. PR interva ls prolongedc. QRS complexes widenedd. RR interva ls shortenede. ST segments elevated

251. A patient presents with a blood pressure of 220/120 and a heart rate of 90 beats per minute despi te usual ly effective antihypertens ive drugtherapy. Further work-up indicates the patient has a rare cause of these and other s igns and symptoms: pheochromocytoma. You rea l i ze that β-adrenergic blockers are useful as antihypertens ive drugs , and for helping to normal ize heart rate in patients with supraventricular tachycardia . Asa resul t of the diagnos is , and your knowledge, you adminis ter a usual ly effective dose of propranolol . What i s the most l ikely outcome of doingthis?

a. Blood pressure fa l l s promptly, fol lowed by reflex tachycardiab. Epinephrine release from the tumor i s suppressed, hemodynamics normal izec. Heart rate and cardiac function ri se quickly because the β-blocker has triggered additional epinephrine release from the tumor.d. Left ventricular a fterload i s decreased, cardiac output ri ses via increases of both left ventricular s troke volume and heart ratee. Tota l periphera l res is tance ri ses , cardiac output fa l l s , the patient goes into cardiogenic shock

252. A 59-year-old man presents in the emergency department with crushing chest discomfort. An ECG indicates a smal l transmural left ventricularinfarction, and prompt cardiac catheterization and assessment of prior lab resul ts indicate s igni ficant hypercholesterolemia. The patient i s givenal l the drugs l i s ted below, for both immediate management of the i schemia and i ts symptoms and for long-term prevention of a subsequent, andpotentia l ly fata l , MI. Which drug would provide immediate rel ief of the consequences of myocardia l i schemia, but has no long-term effects toreduce the ri sk of sudden death or ventricular dys function from another MI?

a. Aspirinb. Atorvastatinc. Captopri ld. Ni troglycerine. Propranolol

253. A 65-year-old woman is transferred to the thoracic surgery ICU after cardiac surgery. She has di ffuse ra les bi latera l ly, a pulse of 90/min, anelevated centra l venous pressure, and a blood pressure of 160/98 mm Hg. The surgery res ident wants to inject an otherwise-correct dose of an IVdrug to control heart rate and blood pressure, but grabs a syringe that conta ins another drug. The patient’s heart rate increases to 150/min and herblood pressure ri ses to 180/106. Which drug did this patient most l ikely receive in error?

a. Dobutamineb. Esmololc. Neostigmined. Propranolole. Verapami l

254. A patient with chronic-s table (“effort-induced”) angina begins taking metoprolol , and once blood levels approach the therapeutic range thefrequency and severi ty of angina attacks , and the need for subl ingual ni troglycerin, were reduced. Which of the fol lowing s tates the di rectpharmacologic action by which the β-blocker provided the des i red effects?

a. Decreased myocardia l oxygen demandb. Di lated the coronary vasculaturec. Di rectly inhibi ted angiotens in I I synthes isd. Reduced tota l periphera l res is tancee. Slowed AV nodal conduction veloci ty

Page 141: Pharmacology pre test self assessment and review (14th ed.)

255. A 50-year-old man i s aware of the benefi ts of aspi rin in terms of reducing the ri sk of death from an acute myocardia l infarction, mainlybecause he has seen many of the ads and internet posts about this . He notices that the usual recommended dose of aspi rin for cardioprotectionis 81 mg/day, but reasons that the bigger the dose, the bigger and better the protective effect. He has taken “at least” 1,000 mg of aspi rin (3“regular s trength” aspi rin tablets ) twice a day for the las t 6 months . Whi le he i s fortunate in terms of having no apparent gastrointestina l adverseeffects that are associated with long-term, high-dose aspi rin use, he suffers an MI. Autopsy resul ts show cons iderable platelet occlus ion ofsevera l coronary vessels . What expla ins the most l ikely mechanism by which high dose aspi rin use these adverse events?

a. Inhibi ted thromboxane A2 synthes is in platelets

b. Favored adhes ion of platelets to the vascular (coronary) endothel iumc. Ruptured atherosclerotic plaque in the coronaries , expos ing platelets to col lagend. Suppressed hepatic synthes is of vi tamin K-dependent clotting factorse. Triggered excess ive activation of platelets by ADP

256. In cl inic you meet a 55-year-old man who i s described by the attending as having “metabol ic syndrome,” including high LDL and low HDLcholesterol levels , essentia l hypertens ion, type 2 diabetes mel l i tus , and angina l attacks upon s tress about once every 2 months . He currently hasasymptomatic hyperuricemia, but has a gout attack about once a year. The patient i s obese (92 kg), 6 feet ta l l , and has a body mass index(kg/square meter of body surface area) of 40 (normal or des i rable no more than 24.9 kg/m2). He has a 20 year his tory of smoking a ha l f pack ofcigarettes a day, and both parents died in thei r late 50s—the father from an acute MI, the mother from hemorrhagic s troke. The gentleman i staking medications deemed appropriate for each of the conditions noted above. One i s colesevelam. What i s the probably reason why thecolesevelam was given?

a. Counteracts hypokalemia caused by a thiazide diureticb. Lowers LDL-cholesterol levelsc. Lowers plasma urate levels , prophylaxis of goutd. Prevents myocardia l i schemia, angina, by reducing myocardia l oxygen demande. Provides antihypertens ive and natriuretic effects

257. A man has an aneurysm in the aortic root, a consequence of Marfan syndrome. He experiences a hypertens ive cri s i s that requires promptblood pressure control . Ni tropruss ide wi l l be infused for i ts immediate antihypertens ive effects . What drug would we adminis ter a long with thenitropruss ide to minimize the ri sk of aneurysm rupture due to increases of left ventricular dP/dt (ΔP/Δt; change in pressure/change in time) asblood pressure fa l l s?

a . Atropineb. Diazoxidec. Furosemided. Phentolaminee. Propranolol

258. Nicotinic acid (niacin), in the relatively large doses that are used to treat certa in and common hyperl ipidemias , often causes a cutaneousflush and pruri tus . These can be accompanied by widely dis tributed and sharp “pins and needles” or burning sensation on the skin. The responsecan be attenuated by severa l means , one of which involves pretreatment with aspi rin. What mechanism or action most l ikely contributes to thevasodi latory response and the flushing?

a. Activation of α-adrenergic receptors on vascular smooth muscleb. Ca lcium channel blockade in vascular smooth musclec. Loca l production of prostaglandinsd. Release of angiotens in I Ie. Release of his tamine

259. The figure below shows typica l cardiovascular responses to the s low IV injection of four adrenergic drugs into a normal , resting subject.Assume the doses of each are sufficient to cause the effects seen here, but not so high that toxic effects occur. No other drugs are present, andsufficient time has been a l lowed to enable complete diss ipation of the effects of any prior drugs . The dashed l ine between the systol ic anddiastol ic pressure traces approximates mean arteria l pressure.

Page 142: Pharmacology pre test self assessment and review (14th ed.)

Abbreviations used, and answer choices , are

EPI, epinephrineISO, i soproterenolNE, norepinephrinePHE, phenylephrinePHN, phentolaminePRO, propranolol

Select the letter that indicates the drugs that are ordered in the sequence shown (1, 2, 3, 4).

a . EPI, NE, PHE, ISOb. ISO, EPI, NE, PHEc. ISO, PHE, EPI, NEd. NE, ISO, PHE, EPIe. PHE, EPI, NE, PROf. PHE, ISO, NE, EPIg. PRO, PHN, PHE, ISO

Questions 260 to 262

Look at the ECG below and answer the fol lowing three questions .

Page 143: Pharmacology pre test self assessment and review (14th ed.)

Here you see a continuous (uninterrupted) tracing of lead V1, before and after carotid s inus massage (at arrow).

260. What i s the mechanism by which carotid massage exerted i ts effect?

a. Activated what i s tantamount to the baroreceptor reflex, increas ing vaga l tone and acetylchol ine release cons iderablyb. Caused catecholamine releasec. Induced atria l fibri l lation (atria l rate >> 300/min)d. Occluded venous return to the heart, thereby interfering with fi l l ing and contraction of a l l heart chambers “downstream” of the right atrium

261. Based on the outcome of carotid s inus massage, what can you say about the origin of the aberrant electrica l activi ty that leads to thetachycardia you see before the massage?

a. Bundle of Kent (ie, anomalous or accessory pathway for AV conduction) with retrograde and antegrade conductionb. Left bundle branchc. Multiple ectopic ventricular focid. Supraventricular

262. What drug, given as an intravenous bolus , might be used as an a l ternative to carotid massage, caus ing essentia l ly the same outcome and,therefore, the same interpretation of the origin of the ventricular tachycardia?

a. Adenos ineb. Atropinec. Epinephrined. Isoproterenole. Lidoca ine

263. A 23-year-old nonpregnant woman has been us ing a preparation of ora l ergotamine to manage her frequent migra ine headaches . Sheconsumes an excess ive dose of the drug whi le trying to abort a particularly severe and refractory attack. What adverse cardiac or cardiovascularconsequences are most l ikely to occur as a resul t of the ergot overdose?

a. Myocardia l and periphera l (eg, l imb) i schemia due to intense vasoconstrictionb. Renal fa i lure secondary to rhabdomyolys isc. Spontaneous bleeding due to di rect inhibi tion of platelet activation/aggregationd. Syncope secondary to acute hypotens ione. Tachycardia , tachyarrhythmias from β-1 adrenergic receptor activation

Page 144: Pharmacology pre test self assessment and review (14th ed.)

Cardiovascular Pharmacology

Answers

178. The answer is a. (Brunton, pp 730-736; Katzung, pp 184-185, 219-220, 295-300.) ACE inhibi tors are often selected fi rs t for hypertens ive patients whoalso have diabetes mel l i tus (type 1 or type 2)—provided their renal function i s sati s factory (speci fica l ly, no severe bi latera l renal arteria l s tenos is ,or no inadequate blood flow to one kidney i f the other was removed; no a lbuminuria). Reca l l that angiotens in I I constricts the efferent arteriolesin the kidneys , which increases glomerular fi l tration. So, by reducing angiotens in II synthes is an ACE inhibi tor wi l l block this effect and reducefi l tration even further. Angiotens in receptor blockers (ARBs , eg, losartan) wi l l do the same but by blocking angiotens in receptors rather than byinhibi ting angiotens in synthes is .

ACE inhibi tors (and ARBs) do not cause any problems with glycemic control or the responses to antidiabetic drugs , and they seem to exert someprotective effect that s lows or delays diabetes-related nephropathy. Note that ei ther class of angiotens in modi fier i s contra indicated for patientswith severe bi latera l renal arteria l s tenos is , including that which might occur with long-s tanding and severe diabetes mel l i tus . In such patients ,maintenance on some reasonable degree of renal perfus ion and renal function overa l l depends on the renovascular/postglomerularvasoconstrictor effects of angiotens in. Block angiotens in synthes is (ACE inhibi tors ) or angiotens in’s abi l i ty to activate i ts receptors (ARBs), andacute renal fa i lure may ensue.

β-Adrenergic blockers would not be a good choice i f hypertens ion i s accompanied by diabetes mel l i tus , particularly when insul in i s used forglycemic control . Should the diabetic patient experience an episode of hypoglycemia, a β-blocker may delay recovery of blood glucose levels , masktachycardia that i s one symptom of hypoglycemia development, and interact with some antidiabetic drugs (even those used for type 2; to causeexcess ive blood glucose-lowering). (If the diabetes i s wel l control led, and i f the patient has other disorders for which benefi ts of β-blockade mayoutweigh potentia l problems, such as mi ld-moderate heart fa i lure or recent myocardia l infarction, then a β-blocker may be cons idered.)

Thiazides can elevate blood glucose levels and antagonize the des i red effects of antidiabetic drugs (probably by reducing the respons ivenessof skeleta l muscle cel l s and adipocytes to insul in). Nonetheless , they might not be a fi rs t-choice in the setting of diabetes—at least not for thepatient described in the scenario. (Some cl inicians may disagree, and cons ider a thiazide or thiazide-l ike diuretic (eg, metolazone) a fi rs t-choicedrug for essentia l hypertens ion in most patients .)

Verapami l , di l tiazem, and ni fedipine seem not to compl icate blood glucose regulation or interact with antidiabetic drug therapy. Nonetheless ,they lack other benefi ts offered by ACE inhibi tors (especia l ly the renal -protective effects ) in the setting of diabetes and so would not be a fi rs tchoice or of specia l va lue.

179. The answer is d. (Brunton, pp 774t, 780-781; Katzung, pp 155-162, 167, 179.) Labeta lol i s the best choice, assuming no contra indications to β-blockade, such as asthma or second degree or complete heart block. Given i ts combination of both α-and β-adrenergic (β1 and β2) blocking effect, i toffers the best approach for managing the hypertens ion, the tachycardia , the resul ting oxygen supply-demand imbalance that leads to both chestdiscomfort and the i schemic ST-changes , and the ventricular ectopy (which i s probably a reflection of excess ive catecholamine s timulation of β1-receptors ). If the patient i s having an acute myocardia l infarction, s tarting β-blocker therapy early i s a l so decidedly beneficia l short term and forthe long run. (Most any other β-blocker might be a sui table a l ternative, but labeta lol has the combined α/β-blocking actions that are l ikely to be ofgreatest benefi t. Carvedi lol has the same profi le, but i t i s given ora l ly and in this setting that would not be idea l because of s low onset of action.)

Aspirin wi l l do no harm in this s i tuation, but i t wi l l a l so do no good acutely unless there i s ongoing platelet aggregation and coronaryocclus ion. Even i f there were, the aspi rin would do l i ttle to control heart rate, blood pressure, or the ECG changes .

Nothing in the scenario suggests that this patient i s volume-overloaded or suffering acute pulmonary edema. Therefore, adminis tering thefurosemide (c) in such a s i tuation i s not appropriate. Moreover, giving i t i s l i kely to cause prompt reductions of blood volume and, a long with i t, ofblood pressure. The latter effect i s l ikely to lead to further—and unwanted—reflex sympathetic activation that would make matters worse.

Lidoca ine (e) might be sui table for the ventricular ectopy. However, I have identi fied severa l other important s igns and symptoms that wouldnot be rel ieved by this antiarrhythmic drug. As noted above, the profi le of labeta lol offers the greatest l ikel ihood of managing multiple problemswith one drug.

Increas ing the dose of ni troglycerin (f; and especia l ly giving i t as a bolus ) i s l ikely to drop blood pressure acutely, triggering reflex(baroreceptor) s timulation of the heart. The usual “anti -i schemic” effects of the drug would be counteracted by such “pro-ischemic” changes asfurther ri ses of heart rate and a probable worsening of the premature ventricular beats .

Prazos in (g) would lower blood pressure nicely. However, once aga in we have to worry about excess ive pressure lowering, triggering thebaroreceptor reflex, and worsening many of the a l ready worrisome findings (eg, heart rate, PVCs).

180. The answer is d. (Brunton, pp 313t, 326, 767, 774t, 831; Katzung, pp 159t, 161, 167, 189.) Timolol i s a nonselective β-adrenergic blocker. Ora l dosageforms are approved for managing essentia l hypertens ion (and angina). A topica l ophthalmic dosage form is indicated for managing some cases ofchronic open-angle glaucoma. β-Blockers are not only l ikely to control blood pressure, and help lower intraocular pressure, but a lso to reducecatecholamine-induced cardiac s timulation (the tachycardia).

Verapami l or di l tiazem (b, e), both ca lcium channel blockers (CCBs) of the nondihydropyridine class , wi l l not only lower blood pressure but a lsotend to modulate the tachycardia (through di rect and β-receptor-independent processes ). However, they don’t lower intraocular pressure. Al l otherthings being equal , then, the β-blocker would s ti l l be a better choice for the hypertens ive, tachycardic, glaucomatous patient. Thus , we get threepotentia l benefi ts from β-blockade and only two with verapami l or di l tiazem.

ACE inhibi tors , such as captopri l (a ) and thiazide diuretics (c), have no actions that would make them particularly sui table for hypertens ivepatients who are tachycardic, have glaucoma, or both.

181. The answer is c. (Brunton, pp 330, 780-781; Katzung, pp 159t, 161, 179-180.) Even i f you know nothing speci fica l ly about nebivolol you should be ableto conclude that increased formation or activi ty of ni tric oxide (a vasodi lator, among other things ) i s the only reasonable answer. A drug thatcompeti tively blocks α1 receptors (a ; for example phentolamine or prazos in) would, indeed, lower blood pressure (BP). However, the questionstated that nebivolol had no effect on the vasoconstrictor effects of ei ther phenylephrine (di rect α-agonis t) or amphetamines (indi rect-actingsympathomimetics ; release neuronal norepinephrine). (Reca l l that labeta lol and carvedi lol are the β-blockers that a lso block α1 adrenergicreceptors .) Competi tive blockade of α2 receptors (b) cannot be correct. Blocking the presynaptic α-receptors would inhibi t releasednorepinephrine’s usual abi l i ty to “feedback inhibi t” further NE release upon an action potentia l . Had nebivolol blocked the α2 receptors , then, themost l ikely response would be a ri se, not a fa l l , of periphera l res is tance and BP. Thromboxane A2 (d) i s a vasoconstrictor (among other things ).

Page 145: Pharmacology pre test self assessment and review (14th ed.)

Increas ing i ts synthes is would tend to ra ise BP/periphera l res is tance a lso. COMT (e) inhibi ts the metabol ic inactivation of catecholamines in theGI tract, l i ver, and elsewhere. In theory, COMT inhibi tors a lso would ra ise BP caused by i ts substrates . Fina l ly, β-blockers are not catecholamines ,and so are not substrates for COMT.

182. The answer is a. (Brunton, pp 86t, 1271-1272; Katzung, 743, 762-763, 1033t.) β1 activation, whether in contracti le, nodal , or conducting ti ssues of theheart, leads to and requires increased production of cycl ic AMP. Those responses wi l l be inhibi ted in the presence of excess ive doses of a β-blocker. Glucagon, a parentera l drug normal ly used for managing severe and acute hypoglycemia, can di rectly increase c-AMP levels in the heart,even in the presence of β-blockade. It has the best chance of helping to restore (or at least improve) cardiac function—even in the presence ofprofound β-blockade.

Phenylephrine (b), the prototype α-agonis t, would ei ther not affect the a l ready high TPR, or might increase i t further. Giving i t in this case wouldnot only be i rrational , but a lso dangerous . Phentolamine (c), the prototype α-antagonis t, would lower TPR, but i t i s s ti l l unl ikely that the heart cangenerate sufficient CO to perfuse the ti ssues adequately. Ephedrine (d), a drug that works in part by releas ing norepinephrine, would be of l i ttle i fany benefi t. If i t were given CO would s ti l l be depressed owing to the β-blockade, and α-receptor activation would cause phenylephrine-l ikeeffects on the periphera l vasculature. As noted above, those periphera l vasoconstrictor effects are precisely what we don’t need or want to cause.Epinephrine (e) would a lso cause not-needed/unwanted periphera l vasoconstriction; i t i s not l ikely to do better at activating cardiac β-receptors ,and improving CO, than i soproterenol a l ready has .

183. The answer is e. (Brunton, pp 844, 1405-1407; Katzung, pp 238, 921-922.) Many of the s igns and symptoms of sa l icyl i sm are s imi lar to those causedby high blood levels of quinidine (antiarrhythmic, no longer a fi rs t-l ine antiarrhythmic) or quinine (mainly used as an antimalaria l ). Quini -dine,quinine, and related drugs are ca l led cinchona a lka loids , and the low grade toxici ty syndrome caused by these drugs i s ca l led cinchonism. (Thesedrugs were origina l ly obta ined from a plant known, generica l ly, as Cinchona.) Aspirin and the cinchona a lka loids are chemica l ly s imi lar in someimportant chemica l and pharmacologic ways . The common s igns and symptoms include l ight-headedness , tinni tus , and visua l dis turbances suchas diplopia .

184. The answer is e. (Brunton, pp 761, 765-767, 829-831; Katzung, pp 201-205, 245.) Verapami l , a nondihydropyridine ca lcium channel blocker (CCB),depresses both the SA node and the AV node and would be effective for prophylaxis of paroxysmal atria l or supraventricular tachycardia .Ni fedipine (c), the prototypic dihydropyridine CCB, has l i ttle effect on SVT because i t and the other dihydropyridines lack cardiac depressanteffects . Moreover, i f we chose a fast-/immediate-acting dosage form of ni fedipine, we would probably trigger substantia l reflex cardiacstimulation. The increased sympathetic tone to the heart could worsen the PSVT. Ni troglycerin (d) i s mainly a venodi lator, but i t can cause fa l l s ofarteria l blood pressure sufficient to trigger reflex cardiac activation that would exacerbate the tachycardia . Adenos ine (a) may be useful indiagnos ing whether ventricular tachycardia i s of supraventricular or ventricular origin, because i t effectively s lows AV nodal conduction. It i s a l soused as acute therapy for PSVT. However, i t i s a fas t- and short-acting parentera l drug, which renders i t unsui table for the condition I s tated:outpatient prophylaxis . Lidoca ine (b) i s mainly used for ventricular tachyarrhythmias , not those of supraventricular origin. Both adenos ine andl idoca ine are parentera l drugs with short ha l f-l ives ; nei ther property makes them sui table for outpatient prophylactic use.

185. The answer is d. (Brunton, pp 206t, 212, 305-307, 309; Katzung, pp 151-152, 154, 167, 180, 189.) Prazos in causes competi tive α-blockade. This i s ins tark contrast with phenoxybenzamine, which causes insurmountable (noncompeti tive) and long-lasting blockade of α-receptors by a lkylatingthem. Phenoxybenzamine i s class i fied as a ha loa lkylamine. You probably don’t need to remember that, but perhaps i t wi l l help you rea l i ze thatthe drug a lkylates the α-adrenergic receptors . This cova lent interaction, in comparison with typica l weak ionic interactions between most drugsand their receptors , accounts for phenoxybenzamine’s long-lasting (not shorter-acting; a ) and noncompeti tive blockade. As a resul t of a l teringreceptor confi rmation, not merely occupying them as prazos in and most other α-blockers do, phenoxybenzamine renders largely ineffective α-agonis ts that ordinari ly would be used to counteract effects of excess ive receptor blockade. Thus , i f the patient became hypotens ive in responseto phenoxybenzamine, ra is ing pressure with typica l and usual ly effective drugs may not succeed. Nei ther drug has intrins ic β-blocking activi ty (b),and so regardless of which of these drugs we use for the pheochromocytoma patient, a β-blocker wi l l need to be used adjunctively to control(primari ly) heart rate and contracti l i ty. Nei ther drug suppresses epinephrine release from the adrenal/suprarenal medul la (c); both tend to causeorthostatic hypotens ion by blocking periphera l α-mediated vasoconstriction that occurs upon s tanding up suddenly.

186. The answer is c. (Brunton, pp 859, 864-865; Katzung, pp 604-607.) This brief scenario describes heparin-induced thrombocytopenia (HIT). It i s animmune-mediated thrombocytopenia that i s accompanied by a paradoxica l increase in thrombotic events (eg, in l imbs , bra in, lungs , and heart). Ita ffects about 1% to 3% of patients receiving heparin for more than about 4 days in a row. The cause appears to involve formation of antibodiesthat develop to heparin-platelet complexes . This leads to substantia l increases of platelet activation that, in turn, leads to thrombos is , vasculardamage, and eventual ly s igni ficant decl ines in the number of functional ci rculating platelets (s ince platelets have been consumed in widespreadthrombotic events ). This phenomenon i s not associated with any of the other drugs l i s ted in the question.

Important note: Warfarin can a lso cause a paradoxica l prothrombotic s tate, probably due to suppressed protein C activi ty. It i s most l ikely tooccur i f loading doses are used, rather than s tarting with the anticipated maintenance dose. If you do some checking on the web or in medicinetexts , you wi l l probably find severa l a lgori thms for ca lculating warfarin loading doses . The safest advice i s not to use any loading doses : a lwaysget the ini tia l anticoagulation with a heparin, s tart warfarin s imultaneous ly, then continue the heparin for at least 5 days and unti l the INRreaches the des i red range. Us ing warfarin loading dose s trategies does not hasten warfarin’s anti thrombotic effect.

187. The answer is d. (Brunton, pp 801-804, 837-839; Katzung, pp 215-218, 220.) Digoxin inhibi ts the sarcolemmal Na +, K+-ATPase (“sodium pump”). Thisreduces the active (ATP-dependent) extrus ion of intracel lular Na +. The relative excess of intracel lular Na + competes with intracel lular Ca 2+ for s i teson a sarcolemmal 2Na-Ca exchange di ffus ion carrier, such that less Ca 2+ i s extruded from the cel l s . The net resul ts include a ri se of free [Ca 2+]i andgreater actin-myos in interactions (ie, a pos i tive inotropic effect that increases cardiac output through an increase of s troke volume).

188. The answer is a. (Brunton, pp 752, 767-768; Katzung, pp 205-207.) β-Blockers (represented in this question by atenolol ) should not be adminis tered(especia l ly by a systemic route) to patients with vasospastic angina unless for a medica l emergency that requires β-blockade as a l i fe-savingmeasure. Reca l l the dual roles of adrenergic receptors in the coronary vasculature. Activation of β2-adrenergic receptors causes vasodi lation.Activation of α-adrenergic receptors in the coronary (and other) vasculature favors vasoconstriction or—in the setting of variant angina—vasospasm. Normal ly these receptors are exposed to ci rculating epinephrine, which causes the oppos ing vasodi lator (β2) and vasoconstrictor (α)effects . Norepinephrine i s a lso activating α-adrenergic receptors . Block only the β2 (vasodi lator) effects in the coronaries and the constrictor (andspasm-favoring) effects of the α receptors are left unopposed.

Di l tiazem (b), a nondihydropyridine ca lcium channel blocker, would not only lower blood pressure but a lso suppress the tendency for coronaryvasoconstriction or spasm by blocking vascular smooth muscle ca lcium influx. Hydrochlorothiazide (c) i s the prototype thiazide diuretic, and

Page 146: Pharmacology pre test self assessment and review (14th ed.)

metolazone (e) i s thiazide-l ike in terms of most of i ts pharmacologic profi les . Nei ther i s at a l l l i kely to cause or favor vasoconstriction in thecoronary vessels or elsewhere. Losartan (d) i s an angiotens in receptor blocker. It not only has good antihypertens ive activi ty, but a lso i s apt tosuppress any coronary vasoconstrictor influences of ci rculating angiotens in.

189. The answer is c. (Brunton, pp 753-755, 760; Katzung, pp 195-201.) At usual therapeutic doses , ni troglycerin (NTG) acts primari ly as a periphera lvenodi lator. At higher doses arteria l di lation a lso occurs . The net effect i s that, particularly with excess ive doses , periphera l vasodi lation occursand blood pressure (BP) fa l l s . Cri tica l among these pressure changes i s a fa l l of diastol ic BP. I say cri ti ca l because DBP i s the main driving force forblood flow through the coronary vasculature. Up to a point autoregulation of coronary flow may help mainta in cardiac perfus ion, but once DBPreaches a certa in lower level coronary blood flow is inadequate to meet the oxygen demands of the myocardium (ie, i schemia has developed).Add to the underperfus ion of the heart by reduced DBP the fact that i f BP fa l l s by a sufficient amount, and sufficiently fas t, the baroreceptor reflexis activated. That, in turn, increases heart rate and contracti l i ty—an increase of myocar-dia l oxygen demand s imultaneous with reduced oxygensupply due to the decreased coronary perfus ion.

Nitroglycerin i s not metabol i zed to cyanide (a). If you selected that poss ibi l i ty as your preferred answer you are confus ing NTG with anotherni trovasodi lator that clearly does form cyanide: ni tropruss ide. NTG does not, cannot, induce coronary vasoconstriction or spasm directly (b). NTG isoften used, with success and no extra ri sk, for patients with coronary vasospasm (d). For such individuals NTG is mainly used for acutemanagement of an ongoing attack (as appl ies to vi rtua l ly a l l angina patients who take the drug), with long-term suppress ion of vasospasmprovided by a ca lcium channel blocker. Fina l ly, for patients with angina that does not involve vasospasm, NTG is frequently prescribed with a β-adrenergic blocker (e). Here aga in, the NTG is used mainly for acute symptom control and the β-blocker for prophylaxis .

As an as ide, i f you indeed chose answer e, that the β-blocker’s “intrins ic vasodi lator activi ty potentiated that of the NTG,” I think you should beprepared to ci te a speci fic β-blocker with that property. The β-blocker named in the question, propranolol , has absolutely no intrins ic vasodi latoractivi ty. Indeed, of the long l i s t of β-blockers on the market, only three have the abi l i ty to cause vasodi lation. Labeta lol and carvedi lol have someα-blocking activi ty that causes vasodi lation, but the effect i s weak. Then there i s nebivolol , which causes some vasodi lation through i tsmetabol i sm to ni tric oxide.

Note: There i s an interesting and somewhat controvers ia l phenomenon ascribed to NTG and some other vasodi lator drugs : coronary s tea l . Inessence, vasodi lation shunts or diverts (s tea ls ) blood flow from areas of low perfus ion (eg, a s tenotic coronary vessel ) to areas of high perfus ion.The postulated outcome is that poorly perfused vessels (and the ti ssues served by them) become even less wel l perfused, thereby caus ing orworsening dis ta l ti s sue i schemia.

190. The answer is a. (Brunton, pp 688t, 774-775, 791; Katzung, pp 260-262, 270.) Thiazide diuretics tend to ra ise uric acid concentration in the blood,probably by inhibi ting tubular secretion of urate. The elevations of urate may be of l i ttle concern for patients with no his tory of hyperuricemia orgout, but for those with such a his tory i t can be a problem that i s not associated with any of the other answer choices given. Thiazides can beadminis tered to hyperuricemic/gouty patients , but that may require another drug (a l lopurinol or febuxostat, which are xanthine oxidase inhibi tors )to counteract diureticinduced ri ses of urate levels . If one can avoid the problems by avoiding the thiazide, and the poss ible need for adding asecond drug to counteract the hyperuricemia, why not do just that? That i s one of many reasons why we might use another class ofantihypertens ive drugs (eg, angiotens in modi fiers , ca lcium channel blockers , many more) that don’t have negative effects on urate levels .

None of the other answer choices have any appreciable des i red or untoward effects on urate levels , renal handl ing of urate, or the incidence orseveri ty of gout.

191. The answer is d. (Brunton, pp 849, 946, 949, 977-982; Katzung, pp 320, 612.) These in vi tro responses , showing that ibuprofen (or another NSAID,naproxen) can s igni ficantly antagonize the antiplatelet effects of aspi rin, are cl inica l ly relevant. Both ibuprofen and aspirin compete for the samebinding s i tes on both COX-1 and COX-2. Aspirin’s interaction i s cova lent, and i rrevers ible. Ibuprofen, however, binds to the same COX s i tes asaspirin. Its antiplatelet effects are comparatively weak and revers ible. More importantly, the bound ibuprofen prevents the abi l i ty of aspi rin tobind to same cri tica l enzyme s i te. If aspi rin therapy i s s topped, COX-1 activi ty wi l l increase by about 10% per day, and just 20% of basel ine COXactivi ty—that i s , skipping aspi rin for more than 2 to 3 days—may be enough to enable sufficient COX activi ty to be restored and the protectiveantiplatelet effects lost. Us ing ibuprofen for more than a few days in a row is l ikely to do the same. In short, the interaction should be avoided.And remember: ibuprofen (and naproxen) i s ava i lable OTC, and unless you expl ici tly tel l your patients to avoid i t, your pharmacologic efforts toafford cardioprotection may be for naught. Such NSAIDs as diclofenac, and many others , do not interfere with aspi rin’s antiplatelet effects .

Acetaminophen (a), in dosages of about 1 g/day or more, taken for more than 7 days in a row, may potentiate warfarin’s anticoagulant effects .However, i t does not interfere with aspi rin’s antiplatelet effects ; and i t has no anticoagulant or anti thrombotic effects in i ts own right. Clopidogrel(b), a platelet ADP receptor blocker that i s used as an antiplatelet drug in l ieu of aspi rin (for some patients ), does not block aspi rin’s antiplateleteffects , because the mechanisms of action of clopidogrel and aspirin are synergis tic and di fferent. Dabigatran (c) i s an ora l anticoagulant,sometimes used as an a l ternative to warfarin (e). It i s class i fied as a di rect-acting thrombin inhibi tor. Nei ther dabigatran nor warfarin counteractsaspirin’s antiplatelet effects .

192. The answer is e. (Brunton, pp 752-753, 885, 892-898; Katzung, pp 625-627.) The findings are cons is tent with s tatin-induced myos i ti s and myopathy,which seem to have progressed to rhabdomyolys is and renal fa i lure—both potentia l ly fata l . This syndrome (and hepatotoxici ty) i s the mostserious adverse response to the s tatins . It i s more preva lent in older patients , those with multiple i l lnesses , and especia l ly those with renal orl iver disease. Coadminis tration of most other l ipid-lowering drugs (none of which are in the above l i s t) increases the ri sk of rhabdomyolys is ,hepatotoxici ty, or both. As an as ide, the ri sk of rhabdomyolys is (or lesser skeleta l muscle changes) i s not too much di fferent between the currentlyava i lable s tatins . However, i t was (a l legedly) such a problem with one relatively recent drug, cerivastatin, that the drug was pul led from themarket.

Note: Never been taught speci fica l ly about rosuvastatin? Wel l , the drug name ends in “-s tatin” so you should automatica l ly know quite a bi tabout i t.

193. The answer is a. (Brunton, pp 178-179, 214t; Katzung, pp 98, 103, 331.) ACh causes vasodi lation only when the vascular endothel ium is intactfunctional ly and anatomica l ly. The normal response, ini tiated by ACh acting as an agonis t on muscarinic receptors , involves endothel ium-derivedrelaxing factor/ni tric oxide (EDRF/NO) that i s generated in and released from the endothel ium (that i s why i t’s ca l led “endothel ium-derived”). TheNO di ffuses into the adjacent vascular smooth muscle cel l s and increases cGMP formation. That, in turn, causes extrus ion of Ca 2+ and relaxation ofthe muscle (vasodi lation in the intact animal ). When the endothel ium is removed (actua l ly qui te easy to do experimenta l ly) or damaged, AChcauses concentration-dependent increases of smooth muscle tens ion.

Isoproterenol (b) i s a β1/β2 agonis t. Added to an in vi tro preparation such as the one used here, i t would only cause smooth muscle relaxation(β2 effect) unless the β-receptors were blocked; i t does not a l ter the contracti le responses to added ACh. Blockade of muscarinic receptors (c)cannot be correct. If that were the case, particularly under control conditions , contracti le response to ACh would be diminished or abol i shed, and

Page 147: Pharmacology pre test self assessment and review (14th ed.)

contraction shown under the experimenta l conditions would not be changed from what i s shown in the figure. Prazos in (d) cannot be correct. It i san α1 receptor blocker, and pretreating with i t would not cause vasoconstriction with or without added ACh. Botul inum toxin (e) blocks ACh releasefrom chol inergic nerves . Al though there may be some postgangl ionic para-sympathetic (and sympathetic) nerve “endings” in the ti ssue sample,the responses shown above were caused by adding ACh di rectly to the ti ssue bath and so they are independent of any res idual autonomic neura linfluences .

194. The answer is b. (Brunton, pp 732-733; Katzung, pp 183-185, 219-220, 299, 1042t.) ACE inhibi tors (and angiotens in receptor blockers , ARBs , eg,losartan) are contra indicated in pregnancy (category X), and should not be adminis tered to “women of chi ldbearing potentia l”—not just womenwho are pregnant—in the fi rs t place. Normal in utero development of the kidneys and other urogenita l s tructures seems to be angiotens in-dependent. ACE inhibi tors , or ARBs , given during the second or thi rd trimesters (not the fi rs t) have been associated with severe and sometimesfata l developmenta l anomal ies of these s tructures . Crania l hypoplas ia , and neonata l hyperka lemia and hypotens ion, have a lso been reported.

α-Methyldopa (a) i s cons idered one of the preferred antihypertens ive drugs for pregnant women. It not only controls maternal blood pressurewel l , but a lso i s remarkably free of adverse effects on the fetus . (Indeed, i t i s the most l ikely drug that we would prescribe for this hypertens iveand now pregnant woman.)

β-Blockers such as labeta lol (d) are sometimes used during pregnancy, pos ing no speci fic or s igni ficant ri sks to the mother or the fetus ,provided that adequate perinata l care i s given and blood pressure doesn’t fa l l excess ively. None of the ca lcium channel blockers (includingverapami l , e) seem to have any s igni ficant benefi ts or ri sks during pregnancy, compared with other drugs . (Verapami l , for example, i s in pregnancycategory C.) One theoretica l concern with verapami l and di l tiazem is a prolongation of labor as parturi tion draws near, due to suppress ion ofuterine contracti l i ty.

There are concerns with us ing diuretics during pregnancy. Furosemide (c) or other loop diuretics pose relatively s igni ficant feta l ri sks . However,that i s related to potentia l maternal hypovolemia and hypotens ion that may lead to placenta l underperfus ion. There i s no teratogenic orembryopathic ri sk on par with that associated with the angiotens in modi fiers (ACE inhibi tors or ARBs). Moreover, when the goal i s treatingessentia l hypertens ion in the absence of hypervolemia, edema, asci tes , and so on, loop diuretics seldom are used. Instead, thiazide or thiazide-l ike diuretics are the ones usual ly chosen. Nevertheless , the thiazides a lso pose a ri sk of placenta l underperfus ion, and so they are not preferredor idea l antihypertens ives for the pregnant woman.

Note: It i s not necessary (now) for you to memorize the pregnancy class i fi cations of the nonteratogenic drugs l i s ted above. What you need toknow is that ACE inhibi tors and ARBs are absolutely contra indicated. So i s warfarin, at least during the fi rs t 12 weeks of gestation, which i saddressed in Question 199.

195. The answer is c. (Brunton, pp 868-870; Katzung, pp 613, 617.) Clopidogrel (and the much lesser-used drug ticlopidine) i s a noncompeti tiveantagonis t of ADP. This prodrug (i t must be metabol ica l ly activated) causes largely i rrevers ible (ie, for the l i fetime of the platelet) inhibi tion ofplatelet aggregation by blocking ADP binding to the Gi coupled P2Y(AC) receptor. It has no effect on platelet activation and ampl i fi cation caused bysuch other proaggregatory agonis ts as col lagen, thromboxane A2, thrombin, PAF, serotonin, or epinephrine.

Aspirin (a) inhibi ts platelet aggregation caused by thromboxane A2 (TXA2) only, and does so only by inhibi ting TXA2 synthes is via cyclooxygenase,not by blocking TXA2 receptors .

Biva l i rudin (b) i s a synthetic hi rudin derivative (you may reca l l that hi rudin i s produced by the medicina l leach, Hirudo medicinalis). It i s class i fiedas an anticoagulant, not as an antiplatelet drug. It i s a di rect-acting inhibi tor of free and clot-bound thrombin, which leads to two main effects : (1)decreased convers ion of fibrinogen to fibrin; and (2) reduced activation of Factor XIIIa , which in turn decreases convers ion of soluble fibrinmonomers to insoluble (polymerized) fibrin.

Biva l i rudin i s given IV as an a l ternative to heparin (both drugs bind to free thrombin, but biva l i rudin a lso interacts with clot-bound thrombin),mainly a long with aspi rin for patients with unstable angina who are undergoing angioplasty. It i s a l so used to help treat heparin-inducedthrombocytopenia ; and seems to be more effective than heparin when given postmyocardia l infarction. A related drug i s argatroban.

Heparin and warfarin (d, e) have no di rect effects on platelets . (You should a lso reca l l that s ince warfarin’s s i te of action i s the l iver, i t has noanticoagulant effects when tested in vi tro (ie, added to a tube of whole blood).

Note: I mentioned that ticlopidine i s much lesser-used than clopidogrel . Actua l ly, i t’s seldom used. The reason? A greater ri sk of thromboticthrombocytopenic purpura (TTP) and/or neutropenia .

196. The answer is e. (Brunton, pp 765-767; Katzung, pp 193-208.) The vascular ca lcium channel -blocking actions of verapami l or di l tiazem(nondihydropyridine class ) wi l l not only lower systemic blood pressure, but a lso tend to counter coronary arteria l ca lcium influx that favorsvasospasm. Thus , in this setting we can expect both antihypertens ive and antiangina l effects from one drug.

One might argue that ni fedipine (answer c) would be a reasonable a l ternative. However, reca l l that the dihydropyridines (which i s the class towhich ni fedipine belongs) lack cardiac depressant actions . Rapidly acting formulations of a dihydropyridine, even i f given ora l ly, are l ikely tolower blood pressure (the des i red antihypertens ive effect), but lacking cardiac-depressant actions are l iable to trigger reflex(sympathetic/baroreceptor) cardiac s timulation. The resul ting increases of ei ther or both cardiac rate and contracti l i ty (mediated, reflexly, bysympathetic activation) necessari ly may ra ise myocardia l oxygen demand sufficient to cause myocardia l i schemia, and/or may trigger coronaryvasospasm. (The degree to which a dihydropyridine wi l l cause unwanted reflex cardiac s timulation depends on the drug, the dose, and even thedosage form [eg, immediate-acting vs extended-acting].) Nonetheless , a nondihydropyridine would have a much better overa l l profi le ofcardiac/vascular actions .

ACE inhibi tors or an ARB (a), or a thiazide (d), might cause no particular problems for a patient with vasospastic angina, but they a lso have noactions that would help that comorbidi ty. Why select one of them when a nondihydropyridine ca lcium channel blocker might be beneficia l for bothproblems?

β-Blockers (b) would be a poor choice. Reca l l that the coronary vasculature constricts in response to α-adrenergic influences , whi le vascular β2

receptor activation tends to cause coronary vasodi lation. Patients with vasospastic angina depend on β-mediated vasodi latory influences , and areat greater ri sk of i schemic events i f the β-receptors are blocked.

197. The answer is c. (Brunton, pp 899-901, 972; Katzung, pp 627-628.) Answering this question correctly indeed depends on your knowledge of severa ldrugs , and their prototype or representative agents , that goes beyond “cardiovascular” (s ince I included a diabetes drug [metformin] and anantidepressant [esci ta lopram] as poss ible answer choices ).

Nonetheless , niacin (nicotinic acid; vi tamin B3) i s often added-on—in dosages higher than those recommended for vi tamin supplementation—mainly to lower HDL and triglyceride levels , particularly in patients with the class ic mixed hyperl ipidemia characterized by elevated cholesteroland triglyceride levels (Type IIb) and who aren’t adequately managed with a s tatin. The primary effect of niacin i s to decrease production of VLDLs

Page 148: Pharmacology pre test self assessment and review (14th ed.)

(mainly by inhibi ting l ipolys is in adipocytes ). Since LDLs are by-products of VLDL degradation, the fa l l in VLDL levels causes LDL levels to fa l l aswel l . Lastly, reca l l that VLDLs del iver triglycerides to and increase their s torage in adipose ti ssue. By decreas ing VLDL production, then, niacinindirectly counters triglyceride accumulation and favors triglyceride clearance (el imination). See the answer for Question 258 for more informationabout niacin’s more common s ide effects .

Pravastatin and other s tatins (HMG CoA reductase inhibi tors ), whi le mainly used for hypercholesterolemia (to lower LDL-cholesterol levels ),often concomitantly ra ise HDL and lower triglyceride levels adequately. When s tatins and l i fes tyle modi fications fa i l to control triglyceride levelsadequately, adding niacin i s usual ly a reasonable next s tep.

Ci ta lopram (a), esci ta lopram, and other SSRI-antidepressants have no known effects on a l tered vi tamin absorption or on the cardiovascularsystem. Metformin (b), the prototype biguanide often used for managing type 2 diabetes mel l i tus , typica l ly suppresses (not increases) appeti te.Statins do not cause periphera l neuropathy (d); and ACE inhibi tors (ramipri l and others ; e) do not cause diabetic nephropathy, a l though they areclearly contra indicated for patients with severe bi latera l renal arteria l s tenos is , such as that associated with severe and poorly control leddiabetes mel l i tus .

198. The answer is d. (Brunton, pp 828, 844, 1405-1407; Katzung, pp 238, 921-922.) Quinidine i s becoming an outmoded drug, but i t i s , of course, s ti l lbeing used. It may be described as a broad-spectrum antiarrhythmic (that does not mean i t i s indicated or safe for a l l arrhythmias and a l letiologies ). It i s mainly used for long-term management of supraventricular and ventricular arrhythmias : atria l flutter and fibri l lation; andsusta ined ventricular tachycardia . The drug’s effects reflect di rect ion channel -blocking actions plus a variety of receptor-blocking effects that maycounteract the di rect effects .

Direct blockade of sodium channels (a “major” action of Class I antiarrhythmics ) s lows impulse conduction in the atria , ventricles , and His -Purkinje system. Repolarization at those s i tes tends to be delayed, presumably by blockade of potass ium channels . Among other changes , the ECGshows widened QRS complexes and prolonged QT interva ls (which may predispose patients , particularly those with long QT syndrome, to someserious arrhythmias , including torsades).

Quinidine has s trong antichol inergic (vagolytic) effects that are particularly apparent at the SA node (increased automatici ty, tends topredominate over di rect depressant effects from sodium channel blockade) and the AV node (increased conduction veloci ty, correct answer d).(Reca l l that vaga l tone on the SA node s lows spontaneous depolarization and heart rate; the vagolytic effects of quinidine wi l l therefore do theoppos i te.) The effects on AV conduction are a main reason why, when quinidine therapy i s to be s tarted and atria l rates are s ti l l high, we pretreatthe patient with a dose of a drug that “blocks down” the AV node: digoxin, sometimes verapami l , and occas ional ly a β-blocker. The main reasonwhy ventricular rates aren’t identica l (or close to) atria l rates during atria l fibri l lation i s because the AV node cannot transmit impulses at suchhigh rates . This protective effect depends on AV nodal refractoriness and a relative inabi l i ty to transmit too many impulses per time. If we did notsuppress the AV node before giving quinidine, the AV nodal “conduction-s timulating” effects of the quinidine might increase AV nodaltransmiss ion; ventricular rates might ri se to dangerous levels as the atria l rate s lows in response to the quinidine.

Quinidine i s not l ikely to increase blood pressure (b), or worsen preexis ting hypertens ion. Quite the contrary: the predominant vascular effect ofthe drug i s di lation, and blood pressure fa l l s , due to periphera l vascular α-adrenergic blocking activi ty. Likewise, quinidine exerts a negativeinotropic effect—not a pos i tive one (e)—on ventricular muscle. That, too, can contribute to a fa l l of blood pressure. For long-term management of apatient with atria l fibri l lation, anticoagulants are important (not contra indicated, c) for prophylaxis of venous thrombos is ; the use of quinidinedoes not contra indicate that.

199. The answer is e. (Brunton, pp 857, 865; Katzung, pp 608-611, 1043t.) The scenario describes the class ic findings with “warfarin embryopathy”: nasa ldeformities (mainly a flattening of the nose) and s tippl ing (based on s tandard x-ray images) at the epiphyses of the long bones . The drug i s , asnoted, absolutely contra indicated during pregnancy. The feta l ri sks are greatest i f warfarin i s taken at any time during the fi rs t 12 or so weeks ofgestation, but adverse feta l effects can occur later. They are i rrevers ible, sometimes fata l ; there i s no drug or vi tamin (eg, vi tamin K, folate) therapythat wi l l be of benefi t. Al l the other drugs l i s ted pose some pregnancy-related ri sks (but lower and with less supporting data than with warfarin),and they do not share the characteris tics of warfarin embryopathy. Clonidine (a) i s in pregnancy category C. Its antihypertens ive mechanism islargely identica l to that of methyldopa, which i s in category B and usual ly i s a preferred drug for hypertens ion during pregnancy. Low molecularweight heparin (b) i s a l so in pregnancy category B. If anticoagulation i s required during pregnancy, i t i s the drug that most l ikely wi l l be used.Hydrochlorothiazide (or thiazide diuretics in genera l ) has no known human teratogenic effects (category B). Ni troglycerin i s not contra indicatedbecause of teratogenes is (category C).

Note: I ’l l kindly ask you to review the answers and explanations for Question 194, and reca l l that even though a cardiovascular drug may not beteratogenic nor absolutely contra indicated in pregnancy, i t can cause adverse (potentia l ly fata l ) feta l effects i f, for example, i t causes hypotens ionand/or hypovolemia, which can s igni ficantly reduce placenta l blood flow. Reca l l that warfarin, ACE inhibi tors , and angiotens in receptor blockersare contra indicated in pregnancy.

200. The answer is d. (Brunton, pp 297, 780-781; Katzung, pp 175-176.) Among a l l the common ora l antihypertens ives , a Coombs-pos i tive test i sassociated only with methyldopa. It occurs in up to about 20% of patients taking this drug long term. It may progress to hemolytic anemia, but thatseldom occurs . The probable cause i s an immune reaction (IgG antibodies ) di rected aga inst and potentia l ly lys ing the red cel l membrane. Otherdrugs with the potentia l to cause an immunohemolytic anemia are penici l l ins , quinidine, proca inamide, and sul fonamides .

201. The answer is a. (Brunton, pp 206t, 306f, 309, 774; Katzung, pp 151-152, 154-156.) Phenoxybenzamine i s a long-acting and noncompetitive α-adrenergicblocker. It works by a lkylating (rather than merely occupying and “blocking”) α-adrenergic receptors , thereby rendering them incapable ofinteracting with α-adrenergic agonis ts such as epinephrine (which i s being released in abundance by the pheochromocytoma). The drug has noabi l i ty to block β-adrenergic receptors (b), inhibi t catecholamine synthes is (c), inhibi t angiotens in converting enzyme (d), or a ffect any of the majorenzymes involved in catecholamine degradation (COMT, answer e; or MAO).

202. The answer is c. (Brunton, pp 328-329; Katzung, pp 157-161, 165-166.) No β-blocker ordinari ly should be adminis tered to patients with asthma,because of a great ri sk of severe and potentia l ly fata l bronchoconstriction or bronchospasm (unless the β-blocker i s being given for a medica lemergency that requires β-blockade, and l ikely benefi ts outweigh l ikely ri sks ). This appl ies to a l l classes of β-blockers : nonselective, l ikepropranolol ; β1 “selective,” that i s , atenolol or metoprolol ; those with intrins ic sympathomimetic/partia l agonis t activi ty, such as pindolol ; andthose that a lso have α-blocking activi ty, that i s , labeta lol and carvedi lol . The contra indication appl ies to a l l adminis tration routes , includingtopica l (ophthalmic); there are cl inica l reports of fata l bronchospasm induced by topica l ophthalmic adminis tration of “just one drop” of β-blockers .

The reason? Airways of persons with asthma are exquis i tely sens i tive to a host of bronchoconstrictor s timul i and exquis i tely dependent on thebronchodi lator effects of ci rculating epinephrine (β2-receptors ). Block those β-receptors , and that sets the s tage for bronchoconstriction orbronchospasm. Another important factor i s that this patient needs a lbuterol for occas ional rel ief of bronchospasm. The drug works , of course, by

Page 149: Pharmacology pre test self assessment and review (14th ed.)

activating β2 adrenergic receptors in the a i rways . Any or a l l of the ava i lable β-blockers wi l l antagonize a lbuterol ’s des i red effects , the so-ca l ledcardioselective agents included, because their selectivi ty for β1-receptors i s only relative, not absolute; and i t i s highly dose-dependent.

Di l tiazem (a) and particularly verapami l (e), both nondihydropyridine ca lcium channel blockers , might be a good choice for this patient. They notonly lower blood pressure by blocking ca lcium influx into vascular smooth muscle, but a lso (theoretica l ly, at least) do the same in a i rway smoothmuscle, thereby preventing or at least reducing bronchoconstriction. Hydrochlorothiazide poss ibly could exacerbate the problems by caus ingexcess ive fluid loss (which could favor bronchoconstriction), but the chances of this are low. (Loop diuretics such as furosemide would be of moreconcern, owing to thei r greater efficacy in terms of caus ing loss of ci rculating fluid volume). There are no speci fic concerns with us ing ramipri l , anangiotens in converting enzyme inhibi tor. (Remember: Drugs with generic names ending in “-pri l ” are ACE inhibi tors , so even i f you learned aboutonly captopri l as an ACE inhibi tor, you should have rea l i zed that ramipri l i s in the same class .)

Note: It’s fa i r to say that the “Big 4” classes of antihypertens ive drugs can be summarized as A, B, C, D: ACE inhibi tors or angiotens in receptorblockers ; Beta-blockers ; Ca lcium channel blockers ; and Diuretics (thiazides or thiazide-l ike agents such as metolazone). Which i s “best” for aparticular patient depends on a host of patient-related factors that may weigh in favor of selecting a drug from a particular class (A, B, C, D) orweigh aga inst a particular class . There has been a resurgence in the use and recommendation of thiazides or thiazide-l ike drugs for ini tiatinganti -hypertens ive therapy. Likewise, much more recently, and for a variety of reasons , some experts are s tating that β-blockers should be droppedfrom the Big 4 l i s t for most patients .

203. The answer is d. (Brunton, pp 801-804, 829t, 835t, 837-839; Katzung, pp 215-218, 221, 1036-1037.) Of the effects l i s ted here, you would expect to finds lowed AV nodal conduction veloci ty in response to digoxin. This i s a common and, in many s i tuations useful , effect. For example, when we givethe drug as part of the pharmacologic management of atria l fibri l lation or flutter, the main des i red response i s not suppress ion of the arrhythmiaper se, but rather to “block down” the AV node so that as atria l rates fa l l (but are s ti l l high), the AV node wi l l be unable to transmit the samefrequency of impulses to the ventricles . That i s , we “protect” the ventricles from excess ive acceleration by inducing a degree of AV block. In termsof more speci fic effects on the AV node, digoxin s lows conduction veloci ty and increases AV nodal refractory periods .

Other “predominant” effects on the heart—al l concentration-dependent—include increases of both atria l and ventricular automatici ty and ofconduction veloci ty through those s tructures and the His -Purkinje system.

204. The answer is d. (Brunton, pp 767t, 779-781; Katzung, pp 181, 189, 220.) Hydra lazine predominately di lates arterioles , with negl igible effects onvenous capaci tance. It typica l ly lowers blood pressure “so wel l” and quickly that i t can trigger two unwanted cardiovascular responses that usual lyneed to be deal t with:

1. Reflex cardiac stimulation (involving the baroreceptor reflex) i s common, and i t i s typica l ly managed with a β-adrenergic blocker (unless i t i scontra indicated). An a l ternative approach would be to use ei ther verapami l or di l tiazem (but not a dihydropyridine-type ca lcium channelblocker such as ni fedipine, which would not suppress—and, in fact might aggravate—the reflex cardiac s timulation).

2. Activation of the renin-angiotensin-aldosterone system to compensate for what the body bel ieves i s an excess ive and unwanted fa l l of bloodpressure. One consequence of this unwanted response i s increased renal sodium retention that would expand ci rculating fluid volume andcounteract hydra lazine’s pressure-lowering effects . This i s typica l ly managed with a diuretic. A thiazide may be sufficient to combat the sodiumretention, but a more efficacious diuretic (loop diuretic) may be necessary and i s usual ly selected.

Captopri l (or another ACE inhibi tor, or an angiotens in receptor blocker such as losartan) might be a sui table add-on (i t would cause synergis ticantihypertens ive effects and prevent a ldosterone-mediated renal effects ). However, combining i t wi th ni fedipine (dihydropyridine ca lciumchannel blocker) i s i rrational . As noted above, given the “pure” vasodi lator actions of ni fedipine and the lack of cardiac-depress ing activi ty (asoccurs with verapami l ), the net effect on heart rate would be ei ther no suppress ion of the tachycardia or a worsening of i t.

Digoxin, a lone or with vi rtua l ly any other drug (c), i s not rational because i t would not meet the goals of blunting reflex cardiac s timulation andefficiently counteracting the consequences of renin-angiotens in-a ldosterone system activation. There i s no indication that there i s need forinotropic support in this patient.

Spi ronolactone, a lone or with digoxin, would be of l i ttle benefi t. One could argue that by vi rtue of the spi ronolactone’s abi l i ty to inducediures is by blocking a ldosterone’s renal tubular effects , i t would counteract hydra lazine’s abi l i ty to lead to renal sodium retention. That may betrue, but spi ronolactone (with or without digoxin) wi l l do nothing des i rable to blunt the unwanted reflex cardiac activation.

Nitroglycerin (e) would add to hydra lazine’s antihypertens ive effects , but i t would probably aggravate (and certa inly not counteract) the reflexcardiac s timulation, and may a lso increase the unwanted renal response (via a hemodynamic mechanism).

Both triamterene and ami loride are potass ium-sparing diuretics . The combination of two diuretics in this class i s a lmost a lways i rrational (i tcan lead to hyperka lemia). Ei ther might beneficia l ly combat a propens i ty for renal sodium retention in response to hydra lazine. But, as with anydiuretic a lone, ei ther or both would do l i ttle i f anything to control the unwanted cardiac response.

Vi tamin K (included in answer c) was included as a foi l . If you are associating hydra lazine with some vi tamin-related problem, you should bethinking of vi tamin B6 (pyridoxine): hydra lazine can interfere with B6 metabol i sm, caus ing such symptoms as periphera l neuri ti s , and soprophylactic B6 supplementation i s often used a long with long-term hydra lazine therapy. Hydra lazine i s a lso known to cause a lupus-l ikesyndrome. Whi le prophylaxis with pyridoxine helps with the potentia l neuri ti s i t does nothing for the lupus-l ike syndrome.

Note: Nowadays many phys icians cons ider hydra lazine to be (at best) a fi fth or s ixth l ine agent for chronic hypertens ion. Regardless , the drug i snot a “go to” drug unless , essentia l ly, a l l other reasonable options have been tried and shown to be ineffective.

205. The answer is a. (Brunton, p 725; Katzung, pp 295-300.) The bradycardia caused by the unknown drug i s reflex-mediated by baroreceptor activationin response to the ri se of blood pressure. Angiotens in I I , by activating vascular A-II receptors , ra ises blood pressure and periphera l res is tance,and that response would not be inhibi ted by pretreatment with prazos in (selective α1-adrenergic antagonis t), nor any other adrenergic blocker(antagonis t) for that matter.

The bradycardia , of course, involves reflex parasympathetic activation (and relative and s imultaneous withdrawal of sympathetic tone to theheart), release of ACh from the vagus and activation of muscarinic receptors on the SA node. Atropine pretreatment indeed prevents that response,as described in the question—and regardless of whether the origina l pressor response was caused by A-II , or any other vasopressor drug for thatmatter.

Dobutamine (b) doesn’t fi t the bi l l ; i t i s largely a selective β1 agonis t. For a l l practica l purposes i t causes no vasoconstriction (an α1 response i fwe l imit things to adrenergic drugs), and via the β1 activation wi l l di rectly increase heart rate, contracti l i ty, automatici ty, and electrica l impulseconduction (eg, through the AV node). No α-activation occurs (ri ses of blood pressure, i f they occur in response to dobutamine, are due to thepos i tive inotropic effect), and so prazos in would have no di rect effect on responses to dobutamine. Isoproterenol (c) can be ruled-out for largely

Page 150: Pharmacology pre test self assessment and review (14th ed.)

s imi lar reasons ; i t activates both β1 and β2 receptors ; and lacks vasoconstrictor activi ty, whether due to activation of α-adrenergic receptors or byother mechanisms.

Could norepinephrine (d) be a reasonable answer? It certa inly causes a vasopressor response and reflex bradycardia (at usual doses).However, the question s tated that the unknown drug’s pressor response i s not inhibi ted by α-blockade. Since a major element of NE’s pressoreffect depends on α activation, i t’s not a reasonable choice. The same appl ies to phenylephrine (e): this nonselective α-agonis t a lso causes apressor response that would be reduced or blocked a l together by prazos in pretreatment.

206. The answer is d. (Brunton, pp 390, 793; Katzung, pp 182, 189, 334.) Cyanide i s the ul timate toxic metabol i te of ni tropruss ide sodium. The drug i sini tia l ly metabol ica l ly reduced to ni tric oxide, which i s respons ible for the arteriolar and venular di lation. However, i t i s another metabol i te, CN–

(and to a lesser extent the next metabol i te, thiocyanate) that i s the main cause of or contributor to toxici ty.When CN– accumulates to sufficiently high levels (as from excess ive or excess ively prolonged adminis tration of the drug), the vasculature

develops what amounts to a tolerance to the drug’s vasodi lator effects , and so blood pressure usual ly s tarts to ri se despi te the presence of highdrug levels . Toxic cyanide accumulation can a lso lead to severe lactic acidos is : the CN– reacts with Fe3+ in mitochondria l cytochrome oxidase,inhibi ting oxidative phosphorylation. Other characteris tic s igns and symptoms of the toxic syndrome include a cherry red skin (becausemitochondria l oxygen consumption i s blocked, venous blood remains oxygenated and as “bright red” as normal arteria l blood), hypoxia , and,ul timately, hypoxic seizures and venti latory arrest. Note: Ni troglycerin and other s imi lar ni trovasodi lators are not metabol i zed to CN– and somani festations of CN– poisoning are not elements of thei r toxici ty profi le.

207. The answer is c. (Brunton, pp 682-684, 777, 790-791; Katzung, pp 189, 201, 334.) Hypokalemia due to the effects of potass ium-wasting diuretics suchas furosemide increase susceptibi l i ty to digoxin toxici ty, and they are probably the most common cause of i t. How does this occur? Digoxin bindsto a K+-binding s i te on the sodium pump (ATPase). That i s , there i s competi tion between digoxin molecules and K+ for the same binding s i tes .When potass ium levels are low (as can occur with any potass ium-wasting diuretic, ie, a loop, thiazide, or thiazide-l ike diuretic), more digoxinmolecules are able to bind to the ATPase—even though the actua l digoxin concentration in the bloodstream hasn’t changed. More binding leadsto greater ATPase inhibi tion: an intens i fied digoxin effect that, qui te often, can lead to adverse responses rather than better therapeutic effects .

Captopri l (a ) has no di rect effects on digoxin’s actions . (We might add that an ACE inhibi tor such as captopri l , a long with a β-blocker and adiuretic—not digoxin—are now cons idered the preferred therapy for most patients with recent onset and mi ld heart fa i lure.) Cholestyramine (b), acholesterol -binding res in, interacts with concomitantly adminis tered (ora l ) digoxin to reduce digoxin absorption. It would not increase the ri sk ofdigoxin toxici ty; qui te the oppos i te, i t would reduce digoxin’s therapeutic effectiveness . Lovastatin (d; an HMG-Co-A reductase inhibi tor/“statin”)and ni troglycerin (e) are not l ikely to cause the observed toxici ty ei ther.

208. The answer is c. (Brunton, pp 830, 831, 764-765; Katzung, pp 202t, 238t.) The etiology involves coronary vasospasm, and that can be blocked wel lwith a ca lcium channel blocker (CCB): di l tiazem, verapami l , or dihydropyridines (eg, ni fedipine, for which s low-/extended-acting ora l formulationsare used). The CCBs block coronary vascular smooth muscle influx of ca lcium, which i s a cri ti ca l process in triggering vasospasm.

Nitroglycerin seems to be margina l ly effective in terms of long-term symptom rel ief, a l though i t may be the only rapidly acting drug that wi l l beefficacious for acute angina and sel f-medication.

Aspirin, through i ts antiplatelet aggregatory effects , would be beneficia l , prophylactica l ly, i f coronary thrombos is were part of the etiology ofvariant angina, but thrombos is i s not the main problem with coronary vasospasm.

Atorvastatin (or other s tatins ) are useful for primary prevention of coronary heart disease, but coronaries that undergo spasm may beremarkably free of atherosclerotic plaque, and the s tatins have no antispasmodic effects per se.

The β-blockers , which are important drugs for many patients with i schemic heart disease, can do more harm than good in vasospastic angina. Inessence, β-receptor activation in the coronaries tends to cause vasodi lation, an effect that to a degree counteracts s imultaneous α-mediatedconstriction. Block only the β-receptors and the α-mediated constrictor effects—vasospasm-favoring effects—are left unopposed. Variant angina,then, i s l ikely to be made worse, not better, with β-blockers . (You might ask whether a combined α-/β-blocker l ike labeta lol or carvedi lol might bebetter than a nonselective or cardioselective β-blocker. Perhaps in theory, but not in practice. Remember that the α-blocking effects of these drugsare comparatively weak; thei r β-blocking, spasm-favoring effects wi l l predominate.)

209. The answer is c. (Brunton, pp 821t, 1405-1407; Katzung, pp 238, 921-922, 1160t.) Digoxin toxici ty i s l ikely to occur within 24 to 48 hours unless thedigoxin dose i s adjusted down. The reason i s that quinidine wi l l reduce the renal excretion of digoxin (digoxin’s main el imination route). This i sprobably due to some mechanism by which quinidine inhibi ts P-glycoprotein transport of digoxin in the kidneys . (See the answer for question 10,on page 28, for a short table l i s ting some drugs that are substrates , inducers , and inhibi tors of P-glycoprotein.)

There i s no “reverse interaction”—that i s , an abi l i ty of digoxin to cause s igns and symptoms of quinidine toxici ty (a ). Quinidine has nos igni ficant impact on the renal actions of any diuretics , whether these actions are expressed in terms of urine output (volume or concentration) orrenal handl ing of sodium or potass ium or other electrolytes or solutes (b, d).

Quinidine-induced digoxin toxici ty may suppress cardiac contracti l i ty, but that would not necessari ly be a di rect effect of an interaction on theinotropic s tate of the myocardium. Rather, i t would be secondary to potentia l digoxin-induced arrhythmias , and i t would not occur “promptly.”

Quinidine does cause some drug-drug interactions by pharmacokinetic mechanisms. It i s a potent inhibi tor of CYP2D6, and can (for example)inhibi t the analges ic effects of codeine by inhibi ting i ts metabol i sm to morphine. However, this mechanism does not apply to thequinidinedigoxin interaction; digoxin i s el iminated completely by the kidneys , with no prior metabol i sm.

210. The answer is e. (Brunton, pp 828, 835t, 840-841; Katzung, pp 241-242, 246-248.) The class I-C antiarrhythmics are associated with a higher incidenceof severe proarrhythmic events than vi rtua l ly any other antiarrhythmics in other classes . This ri sk partia l ly expla ins why, when these drugs werefi rs t approved, they were indicated only for l i fe-threatening ventricular arrhythmias that fa i led to respond to a l l other reasonable (and safer)a l ternatives . (This ri sk a lso contributed to why another I-C agent, enca inide, was withdrawn from the market.)

Nowadays , these I-C agents are s ti l l used for serious (l i fe-threatening) and refractory ventricular arrhythmias , thei r efficacy ari s ing fromsigni ficant sodium channel blockade. However, they a lso block some potass ium channels , which accounts for modestly growing interest in anduse of these drugs for some atria l arrhythmias . Regardless of whether the use i s for an atria l or ventricular arrhythmia, the proarrhythmic effectsshould be of concern.

Pulmonary fibros is and a l terations of thyroid hormone s tatus (typica l ly, a hypothyroid-l ike s tate) are uniquely associated with amiodarone(among a l l the antiarrhythmics ), and amiodarone was not one of the answer choices .

Note: I ’l l opine that memorizing which antiarrhythmic agents are in which Vaughn-Wi l l iams class may not be profi table (except for answeringni t-picky and cl inica l ly i rrelevant exam questions). Among the reasons why (1) this class i fi cation i s based largely on electrophys iologic effects ofthe drugs in largely normal , i solated cardiac cel l s , not in diseased intact human hearts ; (2) some antiarrhythmic drugs have

Page 151: Pharmacology pre test self assessment and review (14th ed.)

electrophys iologic/ionic mechanisms of action that would reasonably place them in more than one Vaughn-Wi l l iams class (eg, amiodarone); (3)belonging to a particular Vaughn-Wi l l iams class does not necessari ly predict cl inica l use of the antiarrhythmic; and (4) s ide effects profi les andtoxici ties of drugs in the same Vaughn-Wi l l iams class—both cardiac and extracardiac—can di ffer substantia l ly.

211. The answer is b. (Brunton, pp 752, 766-767, 830-831; Katzung, pp 183, 194, 201-205, 245.) In a nutshel l , this question summarizes some of the mainand important di fferences between the dihydropyridine CCBs (eg, ni fedipine, and especia l ly in immediate-release formulations , plus manyothers ) and the nondihydropyridines (particularly verapami l , less so with di l tiazem). The dihydropyridines are qui te “selective” for thei r vasculareffects . They cause s igni ficant arteriolar di lation, which usual ly activates the baroreceptor reflex that increases sympathetic influences on theheart: pos i tive inotropy, pos i tive chronotropy (reflex tachycardia can be severe), and increased automatici ty and conduction veloci ty (dromotropiceffects ).

In contrast, and importantly, verapami l and di l tiazem cause not only arteriolar di lation (via ca lcium channel blockade in vascular smoothmuscle), but a lso di rect cardiac depressant effects (due to ca lcium channel blockade in cardiac ti s sues). These cardiac effects oppose, or blunt,reflex sympathetic cardiac activation that otherwise would occur in response to only periphera l vasodi lation: such problems as reflex tachycardiaand pos i tive inotropy are much less—often nonexis tent—with the nondihydropyridines . In fact, when reflex cardiac s timulation caused by otherdrugs (eg, ni troglycerin) i s problematic and must be control led, ei ther vera-pami l (probably preferred) or di l tiazem may be a reasonablea l ternative to the tradi tional agents for blocking the unwanted cardiac responses : the β-adrenergic blockers . Moreover, a nondihydropyridine CCBis usual ly the drug of choice for control l ing cardiac s timulatory responses when a β-blocker i s contra indicated (eg, in asthma). Be aware thatcombined use of di l tiazem or verapami l with a β-blocker i s ri sky due to poss ibi l i ty of addi tive or greater inhibi tory effects on the cardiac rate,contracti l i ty, and especia l ly AV nodal function. And, whether used a lone or in combination, β-blockers and the nondihydropyridine CCBs(verapami l , di l tiazem) should not be adminis tered to patients with severe heart fa i lure due to the added ri sks from further suppress ion of cardiacoutput.

Notes : The vasodi lator effects of verapami l are weaker than those of di l tiazem, but the periphera l vascular (vasodi lating) effects of ei ther aresti l l far weaker than those of a dihydropyridine. Di l tiazem is class i fied, chemica l ly, as a benzothiazepine (s imi lar to but not to be confused withbenzodiazepines—diazepam, midazolam, and many other drugs used for thei r CNS effects .) Verapami l i s , chemica l ly, a phenyla lkylamine.However, memorizing the names of these chemica l groups i s probably a waste of your time: these drugs are a lmost a lways referred to s imply asnondihydropyridines .

212. The answer is b. (Brunton, pp 86t, 390, 793; Katzung, pp 182, 189, 334.) Ni tropruss ide’s main toxic metabol i te i s cyanide. Nowadays , a preferred andeffective approach to managing cyanide toxici ty from ni troprus-s ide overdose i s to adminis ter hydroxycobalamin. It reacts with cyanide, formingcyanocobalamin, which i s largely nontoxic (and i s cons idered a form of vi tamin B12).

A more tradi tional (older) approach to managing cyanide toxici ty—one that i s s ti l l occas ional ly used, i s based on the fact that cyanide normal lyi s metabol i zed to the less toxic and more readi ly excreted thiocyanate by an enzymatic process (rhodanese i s the key enzyme) that usesthiosul fate as a cofactor. Endogenous thiosul fate s tores are rather l imited, and with more serious degrees of cyanide (or ni tropruss ide) toxici tythe thiosul fate i s readi ly depleted. Other endogenous sul fur-conta ining substrates are depleted too. This tradi tional treatment plan, therefore,involves adminis tering sodium thiosul fate to restore thiocyanate production. Ni tri tes (amyl and sodium ni tri tes ) are adminis tered to inducemethemoglobinemia. The methemoglobin then binds free cyanide, forming less toxic cyan(o) methemoglobin.

None of the other drugs l i s ted play a role in the management of cyanide poisoning, whether from ni tropruss ide excess or from other causes (eg,suicide attempts , envi ronmenta l exposure to cyanide). Aminocaproic acid (a) i s an anti thrombolytic (anti fibrinolytic) drug used, for example, whensuch drugs as a l teplase (t-PA; plasminogen activators ) cause excess ive thrombolys is that needs to be counteracted pharmacologica l ly. The druginhibi ts the actions of not only therapeutic plasminogen activators , but a lso those of endogenous activators . It a l so weakly inhibi ts plasminactivi ty. Protamine sul fate (c) should be remembered as the antidote for excess ive anticoagulation caused by heparin (which occurs mainly withunfractionated, rather than low molecular weight, heparin). Thrombin (d) i s a prothrombotic agent that plays no role in ni tropruss ide (or cyanide)toxici ty. Vi tamin K (e) i s used to antagonize excess ive effects of warfarin.

213. The answer is a. (Brunton, pp 730-736, 753, 784-785; Katzung, pp 170-174, 185-188.) Al though combined use of an ACE inhibi tor (or angiotens inreceptor blocker [ARB], eg, losartan) and a diuretic i s qui te common, great care must be taken when adding one of the drugs to therapy that hasbeen s tarted with the other. The reason i s that some patients develop a sudden fa l l of blood pressure that may be sufficient to cause syncope orother compl ications . Volume and sodium depletion seem to be among severa l probable causative factors .

Answer b i s incorrect. The effects of ACE inhibi tors (or ARBs) and thiazides on renal handl ing of potass ium are the oppos i te of one another, notsynergis tic. ACE inhibi tors tend to elevate potass ium levels (in part, by lowering a ldosterone levels ); the thiazides (and loop diuretics , eg,furosemide) are potass ium-wasting.

There i s no evidence that adding one of these drugs to therapy with the other can cause acute (or chronic) heart fa i lure (c); indeed, such acombination i s often an essentia l component in managing chronic heart fa i lure. Blood pressure wi l l fa l l , not ri se, and certa inly not causehypertens ive cri s i s (d); and s lowed AV nodal conduction rates (e) due to this drug combination do not occur.

214. The answer is d. (Brunton, pp 853-859; Katzung, pp 604-607.) Adminis tration of protamine sul fate wi l l quickly reverse the bleeding caused byexcess ive effects of heparin. Protamine binds to heparin, forming a s table complex that abol i shes heparin’s anticoagulant activi ty. It may a lsohave i ts own anticoagulant effect by binding with platelets and fibrinogen. Aminocaproic acid (a) binds avidly to plasmin and plasminogen, andso i s an effective inhibi tor of fibrinolys is and an antagonis t of fibrinolytic/thrombolytic drugs . Dipyridamole weakly blocks platelet ADP receptors ,and has a long but uninspiring his tory as an antiplatelet drug. Factor IX wi l l not reverse the excess ive effects of heparin. Vi tamin K i s used as anantidote for excess ive effects of such drugs as warfarin, and does nothing for heparin overdoses .

215. The answer is c. (Brunton, pp 758-759; Katzung, p 199.) Ni troglycerin causes i ts vasodi lator effects via a ni tric oxide (NO)- and cycl ic GMP-dependentmechanism. The NO activates guanylyl cyclase which forms cGMP from GMP. The cGMP, in turn, dephosphorylates myos in l ight cha ins , leading toreduced actin-myos in interactions and relaxation of the smooth muscle cel l s . Normal ly, this vascular effect i s modulated by cGMP degradation (toGMP) by cGMP-speci fic phosphodiesterases (PDEs). Si ldenafi l (and the related drugs tadalafi l and vardenafi l ) inhibi t the activi ty of those PDEs ,thereby mainta ining cGMP levels and potentiating the vasodi lator effects . This interaction may lead to severe excess ive effects , including l i fe-threatening hypotens ion and myocardia l and cerebra l i schemia. Si ldenafi l and related drugs a lso increase the ri sk of symptomatic hypotens ion i ftaken by people who are a lso being treated with α-adrenergic blockers , including those that selectively block α1 receptors (eg, prazos in anddoxazos in) and are used for managing hypertens ion.

Note the important mechanis tic l inks between vasodi lation/hypotens ion, sexual intercourse, and potentia l ly fata l cardiac responses . Sexualarousa l—and, especia l ly orgasm—causes a mass ive activation of the sympathetic nervous system. One consequence of that, α-mediatedvasoconstriction that tends to keep blood pressure up, i s too feeble to overcome the hypotens ive effects of the s i ldenafi l -ni troglycerin

Page 152: Pharmacology pre test self assessment and review (14th ed.)

combination. Along with a fa l l of blood pressure i s a fa l l of coronary perfus ion pressure (diastol ic blood pressure), that i s , reduced myocardia lblood flow/oxygen supply. Yet the sympathetic activation concomitantly causes s igni ficant increases of cardiac rate and contracti l i ty, that i s ,increased myocardia l oxygen demand. Oxygen demand ri ses , supply fa l l s , and the s tage i s set for acute myocardia l i schemia.

A fina l point to cons ider. It i s reasonable to assume that i f the patient i s taking any or even severa l of the drugs l i s ted in the question, he hasunderlying cardiac disease. However, and in contrast with what some s tudents have suggested, underlying cardiac disease per se does notautomatica l ly contra indicate or even excess ively increase the ri sks from sexual intercourse and orgasm. (How severe i s the patient’scardiovascular disease? It i s wel l control led? What are the speci fic diseases and ri sks he has?) Simi larly, none of the drugs l i s ted other than thenitrovasodi lators (and some α-blockers mentioned in the previous paragraph) contra indicate the use of any of the ED drugs .

216. The answer is e. (Brunton, pp 818-819, 823; Katzung, pp 233-235.) Al though you may not cons ider this question a pharmacology question, being ableto answer i t correctly i s important to the rational and safe use of many drugs .

In genera l , heart block refers to excess ively s lowed AV nodal conduction, which you assess by measuring the PR interva l on the ECG. Reca l l thatthis interva l gives information on how long i t takes for electrica l impulses that originate with SA nodal depolarization to pass through the atria lmyocardia and the AV node, ul timately leading to ventricular activation (mani fest as the QRS complex). Impulse conduction through the atria i snormal ly quick; i t i s the AV node that has the s lowest intrins ic impulse conduction rate anywhere in the heart, and i t i s arguably the s tructure thatis most susceptible to drug- (or disease-) induced changes of supraventricular conduction that lead to the diagnos is of AV block.

In fi rs t-degree heart block, the main mani festation i s a prolonged PR interva l , but each P wave i s fol lowed by a normal ly generated QRScomplex. In second-degree heart block, the PR interva l i s prolonged and some P waves are not conducted through the AV node, and so are notfol lowed by a QRS triggered by the prior atria l activation (eg, 2:1 block). In thi rd-degree (complete) heart block, no P waves are conducted normal lythrough the AV node, and ventricular activation i s solely dependent on intrins ic automatici ty of the ventricles (or conducting ti ssue therein).

Auscul tation of the heart, the presence of blood pressures that are above or below what i s genera l ly regarded as normal , or the presence of as low s inus rhythm, are not rel iable indicators of heart block.

217. The answer is e. (Brunton, pp 83t, 765-767; Katzung, pp 201-205, 245.) Constipation i s a fa i rly common, sometimes very bothersome, andoccas ional ly dangerous response to verapami l (more so than with any other ca lcium channel blocker). It i s a widely used drug. The mechanismunderlying this s ide effect i sn’t known for sure, but probably l ies in vera-pami l ’s rather unique abi l i ty to block ca lcium channels on longi tudina land sphincter smooth muscle cel l s in the GI tract. Regardless of whether we know the mechanism for sure, i t i s important to know that verapami ltends to cause constipation in more than a handful of patients . Statins (a), ACE inhibi tors (b), β-blockers of any class or organic ni trovasodi latorssuch as ni troglycerin (d) do not cause constipation as a s ide effect.

218. The answer is a. (Brunton, pp 779-781; Katzung, pp 181, 220.) Hydra lazine’s primary vasodi lator actions occur on arterioles , not on venules . Theothers have both arteriolar and venular di lator activi ty, thereby lowering both afterload and centra l venous pressure (venous capaci tance; rightatria l pressure). Hydra lazine’s probable mechanism of action involves a ni tric oxide (NO)-mediated process . (There i s a lso some evidence that thedrug opens vol tage-gated potass ium channels on vascular smooth muscle cel l s , caus ing muscle cel l hyperpolarization and ul timate vasodi lation.)

The other drugs l i s ted cause vascular smooth muscle relaxation on both arterioles and venules : losartan (b), by blocking angiotens in I Ireceptors ; ni fedipine (c) via ca lcium channel blockade; ni troglycerin (d) by NO as an intermediate; and prazos in (e) by selective α1-adrenergicblockade. Here i s a short table l i s ting the main s i tes where various drugs or drug classes exert thei r main or sole vasodi lator actions .

Page 153: Pharmacology pre test self assessment and review (14th ed.)

219. The answer is b. (Brunton, pp 849, 977-982; Katzung, p 320.) Aspi rin inhibi ts cyclooxygenases I and II . In terms of clotting, the main effect wi l l beinhibi tion of platelet aggregation by reduced formation of thromboxane A2. Bleeding time wi l l be prolonged and wi l l remain that way unti lsufficient numbers of new platelets have been synthes ized and released into the bloodstream, because aggregation of those platelets a l readyexposed to the drug wi l l be inhibi ted for thei r l i fetime. The APTT, which should not be affected by aspi rin, i s used to monitor effects and adjust thedose of unfractionated heparin (such monitoring i s not required with low molecular weight heparin, eg, enoxaparin). The prothrombin time and i tsnormal ized va lue, the INR, are used with warfarin. Platelet counts , a l so not affected by aspi rin, are used to assess for the development ofthrombocytopenic purpura, which may rarely occur during therapy with, for example, the clopidogrel -l ike drug ticlopidine, or with heparin whenthrombocytopenia i s anticipated or suspected.

220. The answer is a. (Brunton, pp 824-825, 829t, 834-837; Katzung, pp 238t, 245-246.) Nowadays , IV injection of adenos ine i s genera l ly regarded as thefi rs t choice for terminating PSVT in which reentry phenomena play an important pathophys iologic role. (If the ECG shows a wide QRS complex,

Page 154: Pharmacology pre test self assessment and review (14th ed.)

adenos ine would be contra indicated.) Among other reasons , i t i s preferred over another reasonable a l ternative, verapami l , because of a fas teronset of action. Adenos ine i s a lso a primary drug for managing episodes of ventricular tachycardia , provided that there are no cardiac s tructura ldefects (aneurysms, damage to papi l lary muscles or the chordae tendinae, etc).

Digoxin (b), edrophonium (c; rapidly acting ACh esterase inhibi tor), phenylephrine (d; nonselective α-adrenergic agonis t), and propranolol orother β-blockers (e) are older therapies fa l l ing into relative disuse. In one way or another thei r effects revolve around caus ing or unmaskingincreased parasympathetic influences on the SA and/or AV nodes : digoxin via i ts predominant effects to s low AV nodal conduction; phenylephrineby increas ing blood pressure, which triggers a baroreceptor reflex that reduces sympathetic drive and essentia l ly increases or unmasksparasympathetic tone; propranolol , by blocking β1-mediated sympathetic influences ; and edrophonium, which quickly but briefly ra isesparasympathetic influences on nodal ti s sues .

221. The answer is c. (Brunton, pp 857, 861f-862; Katzung, pp 608-611.) Warfarin i s a coumarin derivative that i s genera l ly used for long-termanticoagulation, and i s whol ly unsui table for immediate anticoagulation because i t takes at least 5 days of adminis tration for meaningfulinhibi tion of prothrombin time (reported as the International Normal ized Ratio [INR]) to develop and s tabi l i ze. It antagonizes the gammacarboxylation of severa l glutamate res idues in prothrombin and the coagulation factors I I , VII , IX, and X. This process i s coupled to the oxidativedeactivation of vi tamin K. The reduced form of vi tamin K i s essentia l for susta ined carboxylation and synthes is of the coagulation proteins . Itappears that warfarin inhibi ts the action of the reductase(s ) that regenerate the reduced form of vi tamin K. The prevention of the inactive vi taminK epoxide from being reduced to the active form of vi tamin K resul ts in decreased carboxylation of the proteins involved in the coagulationcascade.

222. The answer is e. (Brunton, p 771; Katzung, pp 611-612.) Al teplase, a thrombolytic drug, i s an unmodified ti ssue plasminogen activator (t-PA). Itactivates plasminogen that i s bound to fibrin (ie, i t i s “clot-speci fic,” unl ike s treptokinase which acts throughout the ci rculatory system). Theplasmin that i s formed in response to the drug acts di rectly on fibrin. This resul ts in dissolving the fibrin into fibrin-spl i t products , fol lowed bylys is of the clot. Clopidogrel exerts antiplatelet effects by blocking platelet ADP receptors (a); aspi rin exerts antiplatelet effects by inhibi tingthromboxane production (b) via cyclooxygenase; abciximab i s an example of an antiplatelet drug that blocks the platelet Gp IIb/II Ia receptors—the“fina l common step” in platelet aggregation, regardless of which l igand ini tiated platelet activation (eg, col lagen, ADP, TXA2). Warfarin, nei ther athrombolytic nor an antiplatelet agent, inhibi ts hepatic synthes is (activation) of vi tamin K-dependent clotting factors (c).

223. The answer is d. (Brunton, pp 834-837, 1153t; Katzung, pp 242-243.) Pulmonary fibros is has been reported with long-term amiodarone therapy, butnot in response to other antiarrhythmics . Pulmonary function tests may remain normal for months and then decl ine quickly and to s igni ficantdegrees as i rrevers ible fibros is develops . Changes of thyroid hormone s tatus—occas ional ly reflecting hypothyroidism and, for more patients(about 3% of a l l who take amiodarone long-term), hyperthyroidism—are a lso uniquely associated with amiodarone: this drug i s s tructura l lyrelated to the thyroid hormones and i s rich in iodine. Changes of thyroid hormone s tatus may be subcl inica l and detectable only with sui tableblood tests or may lead to typica l s igns and symptoms of hyper- or hypothyroidism. Note that the newer antiarrhythmic s imi lar to amiodarone,namely dronedarone, does not share the s ide effects profi le associated with amiodarone.

Keep in mind that some adverse responses that are unique to other antiarrhythmic drugs were l i s ted as poss ible answers . For example, low-grade quinidine toxici ty (cinchonism) often includes tinni tus (but that mani festation of ototoxici ty cannot be detected with audiometry), andproca inamide commonly causes a lupus-l ike syndrome, for which monitoring of ANA ti ters i s important.

Of course, i t’s l ikely that a patient with atria l fibri l lation wi l l be placed on warfarin (at least for a whi le), and so monitoring the prothrombintime (reported as the INR) would be essentia l in that case. And, s ince this patient may have multiple cardiovascular “ri sk factors ,” periodicmonitoring of l ipid profi les would be essentia l too. Nonetheless , these do not apply speci fica l ly or uniquely to amiodarone.

224. The answer is c. (Brunton, pp 868-870; Katzung, pp 613, 617.) Clopidogrel (ti clopidine i s a related drug) decreases platelet aggregation by blockinga population of platelet ADP receptors (which dipyridamole, d, a l so seems to do, but very weakly), thereby inhibi ting ADP-induced plateletactivation. It has no di rect platelet effects involving activation by thromboxane A2, col lagen, or other mediators , nor does i t inhibi t plateletaggregation by any actions on the platelet glycoprotein IIb/II Ia receptors . Acetaminophen (a) has no antiplatelet effects . Aminocaproic acid (b)prevents activation of plasminogen and inhibi ts plasmin di rectly. It may be used to counteract the effects of thrombolytic drugs given in relative orabsolute overdoses . Streptokinase (e) i s a bacteria l derived, “non-clot-speci fic” thrombolytic drug.

225. The answer is f. (Brunton, pp 761, 756-767, 830-831; Katzung, pp 201-205, 245.) In essence, we are asking “which drug can suppress AV nodalconduction veloci ty?” Verapami l and the very s imi lar nondihydropyridine ca lcium channel blocker, di l tiazem, do that. Reca l l the profi le ofverapami l and di l tiazem: a vasodi lator effect plus a di rect cardiac “depressant” effect that includes s lowing of AV conduction (and potentia ldepress ion of other cardiac contracti le and electrophys iologic phenomena).

Be sure you can contrast this dual vasodi lator/cardiac depressant profi le for verapami l and di l tiazem with that of the dihydropyridines (eg,ni fedipine; answer c). The dihydropyridines cause vasodi lation, but lack any cardiac depressant actions . Indeed, with dihydropyridine dosagessufficient to lower blood pressure enough, and quick enough, there wi l l be reflex (baroreceptor) activation of the sympathetic nervous system. Oneconsequence of increased norepinephrine release at the heart would be increased (faster) AV nodal conduction veloci ty, which could beconstrued as an “unblocking” of the AV node—precisely the oppos i te of what may happen with verapami l or di l tiazem.

Captopri l (ACE inhibi tor) and losartan (angiotens in receptor blocker) have no s igni ficant effects on AV nodal conduction.Nitroglycerin (ni trovasodi lator) and prazos in (vasodi lator that acts by competi tive α1 blockade) a lso have no di rect effect on the AV node, but are

l ikely to lead to an indirect quickening of AV conduction via baroreceptor activation.On a fina l and important note, be sure you understand that a l l β-adrenergic blockers (including those with some α-blocking activi ty, eg,

labeta lol and carvedi lol ) can s low AV nodal conduction and can cause or worsen heart block.

226. The answer is a. (Brunton, pp 730-736, 798; Katzung, pp 183-185.) Captopri l , and some of the other ACE inhibi tors , may cause severe, hacking, andrelentless cough in some patients . One theory i s that i t i s due to increased levels of bradykinin in smooth muscles in the throat. (Reca l l thatangiotens in-converting enzyme, which forms angiotens in I I , i s the same enzyme as bradykininase, which metabol ica l ly inactivates bradykinin.)Many patients receiving captopri l (or other ACE inhibi tors ) experience no problems of this sort. Sti l l other patients , mainly taking other ACEinhibi tors , may develop swel l ing of the oropharyngeal mucosae, and some may develop l i fe-threatening angioedema.

There are no l ikely a l lergic reactions (b) triggered by any of the drugs , s tatins included.Captopri l i s not a bronchoconstrictor (c). Hyperka lemia i s not a l ikely explanation. Note that by indi rectly lowering a ldosterone levels

(angiotens in I I i s the main s timulus for a ldosterone release, and we have inhibi ted angiotens in I I synthes is ), the main renal effects would beincreased sodium excretion and increased potass ium retention. However, we are adminis tering furosemide (a loop diuretic), which causes renal

Page 155: Pharmacology pre test self assessment and review (14th ed.)

potass ium-wasting. Thus , we would not expect hyperka lemia from this combination of drugs (as we would i f the diuretic was a potass ium-sparingone such as ami loride, triamterene, or spi ronolactone).

Cough i s not part of the main s ide effect/toxici ty profi le of s imvastatin or other HMG-CoA reductase inhibi tors . Reca l l that thei r main toxici tiesinclude myos i ti s , myopathy, rhabdomyolys is , renal damage (from the rhabdomyolys is ), and hepatotoxici ty.

227. The answer is b. (Brunton, pp 296-297, 322t, 782; Katzung, pp 176-177.) Abrupt discontinuation of clonidine has been associated with a rapidlydeveloping and severe “rebound” phenomenon that includes excess ive cardiac s timulation and a spike of blood pressure that may be sufficientlygreat as to cause s troke or other s imi lar compl ications . Reca l l that clonidine i s a “centra l ly acting α2-adrenergic agonis t.” Through i ts centra leffects i t reduces sympathetic nervous system tone. This , in turn, appears to cause supersens i tivi ty of periphera l adrenergic receptors to di rect-acting adrenergic agonis ts , including endogenous norepinephrine and epinephrine. Once, and soon after, the drug i s s topped, endogenouscatecholamines trigger hyperrespons iveness of a l l s tructures under sympathetic control . When ACE inhibi tors (or angiotens in receptor blockers ),furosemide, or ni fedipine (long-acting or otherwise) are abruptly s topped, blood pressure (and blood volume, depending on the drug) wi l l beginto ri se from treatment levels , but there wi l l be no sudden “spike” of pressure nor an “overshoot” of i t.

Digoxin discontinuation i s not associated with the symptoms noted in the question. Bes ides , the ha l f-l i fe of digoxin (about 36-40 hours i f renalfunction i s normal ) i s such that s topping the drug abruptly would not in a l l l i kel ihood lead to any s igni ficant “withdrawal” events occurring withina day or two of discontinuation.

There i s no reason to predict that suddenly s topping warfarin would cause tachyarrhythmias , hypertens ion, or hemorrhagic s troke—andcerta inly not within 24 to 48 hours .

Important note: A s imi lar, and dangerous , rebound phenomenon occurs when β-blockers are s topped after continuous use for more than acouple of weeks . If the drug blocks only β-receptors , then excess ive cardiac s timulation wi l l be the main excess ive response. If α-receptors area lso blocked, rebound vasoconstriction may occur too. The take-home messages : (1) don’t s top β-blockers abruptly unless you have a readi lyava i lable means to control the potentia l ly l i fe-threatening responses ; (2) make sure your patients understand that they must not abruptly s top β-blockers or clonidine or related antihypertens ive drugs , without contacting you fi rs t.

228. The answer is b. (Brunton, pp 911, 926-932; Katzung, pp 183-185, 295-300.) Angiotens in converting enzyme (ACE) and bradykininase (a lso known askininase II) are exactly the same enzymes. We use the former nomenclature when the substrate i s angiotens in I (the product, of course, i sangiotens in I I); we use the name bradykininase when the substrate i s bradykinin. None of the other drugs l i s ted inhibi t the enzymatic convers ionof ei ther A-I to A-II , or of bradykinin to i ts inactive products .

229. The answer is d. (Brunton, pp 752-753; Katzung, pp 195-197.) Ni tric oxide i s thought to be enzymatica l ly derived from ni troglycerin. It then reactswith and activates guanylyl cyclase to increase GMP, which in turn dephosphorylates myos in l ight cha in kinase, causes ca lcium extrus ion, andsuppresses smooth muscle tone. Tolerance may develop in part from a decrease in ava i lable sul fhydryl groups . Autonomic receptors are notinvolved in the primary response of ni troglycerin, but compensatory mechanisms may counter the primary actions .

230. The answer is c. (Brunton, pp 446-450, 976; Katzung, pp 513-517, 1158t.) There i s a cl inica l ly important relationship between blood (or extracel lular)sodium concentrations and the concentration-dependent effects of l i thium. In essence, Li + and Na + compete with one another, such that in thepresence of hyponatremia the effects of the l i thium may be increased to the point of caus ing toxici ty. (Conversely, hypernatremia can counteractl i thium’s therapeutic effects .) Of the drugs l i s ted, hydrochlorothiazide (and other thiazides and such thiazide-l ike agents as metolazone) posesthe greatest ri sk of caus ing hyponatremia. (And you should cons ider the ul timate renal effects of ACE inhibi tion to lower Na + levels further whenused with a diuretic. ACE inhibi tors used without a diuretic are not at a l l as l ikely to cause hyponatremia.)

There are no cl inica l ly s igni ficant pharmacodynamic or pharmacokinetic interactions between ni troglycerin or HMG CoA reductase inhibi tors andthe SSRIs (fluoxetine, sertra l ine, others ) or l i thium.

231. The answer is b. (Brunton, pp 774t, 784; Katzung, pp 190, 203f, 209.) A good way to arrive at the answer i s to remember the rather narrowcardiovascular profi le of ni fedipine, the prototype dihydropyridine ca lcium channel blocker, and perhaps to compare i t wi th the two mainnondihydropyridines , di l tiazem and verapami l .

The nondihydropyridines block vascular smooth-muscle ca lcium channels , and so cause vasodi lation. Any of these drugs , therefore, would helplower this patient’s blood pressure.

However, ni fedipine (and other dihydropyridines) lack any cardiac-depressant effects . The impl ication i s that as the ni fedipine drives bloodpressure down (and i t wi l l , qui te promptly and in an uncontrol led fashion with this sometimes-used but whol ly inappropriate and unsafeadminis tration method), there wi l l be intense baroreceptor activation and resul ting cardiac s timulation. There are no drug-induced negativeinotropic, chronotropic, or dromotropic (conduction veloci ty) effects to counteract the excess ive cardiac s timulation as there would be i f we hadused ei ther di l tiazem or verapami l .

So, in the presence of reflex-mediated increases of catecholamines affecting the heart, AV conduction would increase (not be s lowed orblocked); there would be further increases of heart rate (at least; certa inly, no fa l l ) to accompany lowering of blood pressure (poss ibly tohypotens ive levels ); and the current episodes of ventricular ectopy might convert to longer runs , or to ventricular tachycardia or fibri l lation.

232. The answer is b. (Brunton, pp 803-804, 834-839, 844; Katzung, pp 215-218, 221.) This col lection of s igns and symptoms i s characteris tic (ie, asyndrome) of digoxin toxici ty, regardless of the cause (eg, frank overdose or the development of hypokalemia, which increases the ri sk of digoxintoxici ty). Al though a probable cause of the hypokalemia i s the furosemide, i t i s not correct to say that furosemide per se i s the cause, becauses igns and symptoms of furosemide toxici ty, whether acute or chronic, are not s imi lar at a l l to those described here. Note, too, that we haveadminis tered triamterene. The expected effect of that potass ium-sparing diuretic i s to counteract renal potass ium loss from the furosemide(potass ium-wasting).

Do reca l l that the digoxin-induced visua l changes described in the question are ca l led chromatops ia .

233. The answer is a. (Brunton, pp 752, 766-767; Katzung, pp 202t, 245.) The main goals are to safely lower blood pressure and help normal ize heartrate. If we cannot use a β-blocker for that, a nondihydropyridine ca lcium channel blocker, ei ther di l tiazem or verapami l , would be an excel lentchoice. In addition to control l ing the cardiovascular problems they are unl ikely to exacerbate the asthma.

Enalapri l (b), one of many ACE inhibi tors , might nicely and gradual ly lower blood pressure, and should have no adverse effects on a i rwayfunction. Nonetheless i t i s unl ikely that i t or any other ACE inhibi tor would have much of a beneficia l impact on the tachycardia . More important i sthe fact that ACE inhibi tors , and angiotens in receptor blockers such as losartan, should not be adminis tered to women who are pregnant or l ikelyto become pregnant. Furosemide (c) would trans iently lower blood pressure, but our patient has essentia l hypertens ion: she i s not volume-

Page 156: Pharmacology pre test self assessment and review (14th ed.)

overloaded, and so this diuretic would not be a good choice. Moreover, i f ci rculating fluid volume fel l sufficiently, and sufficiently fast, thebaroreceptor reflex might be activated, caus ing even further ri ses of heart rate.

Phentolamine (d) and prazos in (e) are both α-adrenergic blocker. Phentolamine i s rapidly acting, given parentera l ly, and nonselectively blocksboth α1 and α2 (presynaptic) receptors . Even with smal l doses , the blood pressure fa l l usual ly i s sufficient to reflexly increase heart rate (andcontracti l i ty) even more. A parentera l drug such as this s imply i s not appropriate therapy in this s i tuation. Prazos in i s given ora l ly, works relativelys lowly, and blocks only α1 receptors . Whi le i ts effects are more gradual , perhaps s low enough that the baroreceptors are not activated, this drugwi l l do nothing to control the patient’s tachycardia .

234. The answer is e. (Brunton, p 210; Katzung, pp 155-156.) Pheochromocytomas—rare causes of hypertens ion—genera l ly involve excess ive levels ofci rculating catecholamines from tumors of the adrenal (suprarenal ) medul la . (In adults , only about 10% of a l l cases of pheochromocytoma are dueto catecholamines released from nonadrenal s i tes .) Regardless of the s i te(s ) of the tumor(s ), the main factor in leading to an increase of bloodpressure and heart rate in this condition i s α-mediated vasoconstriction aris ing from excess ive levels of epinephrine, norepinephrine, or both.Epinephrine (but not norepinephrine) wi l l cause β2-mediated di lation of some vascular beds . However, that vasodi lator effect, which might beconstrued as a mechanism to keep blood pressure from ris ing too much, i s extraordinari ly s l ight in comparison with the oppos ing vasoconstrictor(α-mediated) influences of the catecholamines elsewhere in the periphera l vasculature. Even i f you cons ider the vasodi lator influences to bes l ight, they wi l l be blocked by propranolol or any other β-adrenergic blocker, such that diastol ic and mean blood pressures wi l l ri se further, atleast ini tia l ly. (The so-ca l led cardioselective/β1 blockers such as atenolol and metoprolol wi l l block β2 receptors in the vasculature, andelsewhere, at blood levels that are not too far above the usual “therapeutic” range.) In essence, blockade of β2 receptors in the vasculature wi l lleave α-mediated constrictor effects unopposed, and blood pressure wi l l ri se (concomitant with suppress ion of cardiac contracti l i ty and rate). Theproblem is not l ikely to happen with labeta lol ; i t i s , indeed, a β-blocker, but i t a l so has intrins ic α-blocking activi ty that should blunt to someuseful degree the vasoconstrictor influences of ci rculating epinephrine and neuronal ly released norepinephrine. None of the other drug wouldra ise blood pressure further, and whi le none of them is indicated as primary therapy for pheochromocytomas , ul timately any of them are morel ikely to lower blood pressure.

Aminocaproic acid (a) i s a backup (to whole blood, packed red cel l s , or fresh-frozen plasma) for managing bleeding in response to excess iveeffects of thrombolytic drugs (eg, a l teplase [tPA]). It i s not indicated for warfarin-related bleeding. Epoetin a l fa (b) i s a hematopoietic growthfactor that s timulates erythrocyte production in peri tubular cel l s in the proximal tubules of the kidney. Its uses include management of anemiasassociated with chronic renal fa i lure, chemotherapy (of nonmyeloid mal ignancies ), or zidovudine therapy in patients with acquiredimmunodeficiency syndrome. It i s inappropriate for this patient. Ferrous sul fate (b; or fumarate or gluconate) i s indicated for prevention ortreatment of i ron-deficiency anemias . It wi l l do nothing to lower the patient’s INR or a l leviate related symptoms. Protamine sul fate (e) i s theantidote for heparin overdoses . It acts electrostatica l ly with heparin, in the blood, to form a complex that lacks anticoagulant activi ty. It doesnothing to the hepatic vi tamin K-related problems that are at the root of excess ive warfarin effects .

235. The answer is d. (Brunton, pp 35, 857, 865; Katzung, pp 609f, 614-615.) Phytonadione (vi tamin K1) i s the antidote. It overcomes (reverses ,antagonizes ) warfarin’s hepatic anticoagulant effects , which involve inhibi ted vi tamin K-dependent synthes is/activation of clotting factors (I I , VII ,IX, X, and prothrombin).

236. The answer is d. (Brunton, p 859; Katzung, pp 604-607.) This i s a fa i rly typica l presentation of heparin-induced thrombocytopenia (HIT) in terms ofboth phys ica l findings and time-course of onset. If heparin adminis tration las ts for more than about a week, platelet counts should be checkedsevera l times a week for the fi rs t month or so and then monthly thereafter, because this i s a potentia l ly fata l response. It i s immune-mediated:antibodies form against a heparin-platelet complex; platelets are activated (thus , the thrombos is and such compl ications as pulmonary and/orcoronary occlus ion, as I described); the vascular endothel ia are damaged; and ul timately the number of platelets that can be detected in a venousblood sample decl ines markedly (and so, the thrombocytopenia). The overa l l incidence of HIT i s about 10 times higher with unfractionatedheparin than with LMW heparin. Should HIT occur, or be suspected, the approach includes s topping the heparin and substi tuting otheranticoagulants . A good choice would be a di rect thrombin inhibi tor (eg, biva l i rudin).

Aspirin, by vi rtue of i ts antiplatelet effects , should not induce thrombos is . ACE inhibi tors and β blockers (carvedi lol , any others ) do not interactto cause hemolytic anemia or other blood dyscras ias .

237. The answer is b. (Brunton, pp 757-758, 795; Katzung, pp 195-201.) Us ing long-acting ni trovasodi lators “24-7” i s not at a l l a good idea, because i tul timately leads to tolerance to the des i red vasodi lator effects . This i s of concern in part because, by defini tion, when a system or functiondevelops tolerance to a drug, we must increase the dose in order to achieve effects of the same intens i ty as occurred before. With ni troglycerinpatches , or other long-acting ni trovasodi lators , increas ing the dose to restore the response ul timately accelerates or otherwise intens i fies thetolerance, such that further dosage increases seem warranted. More importantly, with the development of tolerance, the vasodi lator/antiangina lefficacy of subl ingual (or transmucosa l ) ni trate i s diminished too. So, should the patient develop angina, usual dosages of immediate-actingdrugs may not provide rel ief of ei ther the symptoms or the underlying myocardia l i schemia.

If you thought cyanide poisoning (a) was the correct answer you were probably thinking of ni tropruss ide toxici ty. Cyanide i s not a metabol i te ofni troglycerin or other drugs tradi tional ly used for angina. Doses of ni trates that are too high, or that otherwise lower periphera l res is tance tooquickly, may trigger reflex tachycardia . This i s , for some patients , a problem early-on in therapy with rapidly acting ni trates . It usual ly becomes lessproblematic as therapy continues , of i f the dosage i s ti trated down a bi t. It i s rarely a problem with long-acting ni trates . We do not seebradycardia (c), reflex or otherwise, in response to these drugs . Ni trovasodi lators , short- or long-acting, and regardless of the adminis tration route,do not cause thrombos is or affect any other aspects of the coagulation-thrombolys is processes (d). Paradoxica l vasoconstriction leading tohypertens ion (e) does not occur.

238. The answer is b. (Brunton, pp 736-739; Katzung, pp 184-185, 219.) Losartan, an angiotens in receptor blocker (ARB), has no effect on angiotens in I Isynthes is (as do the ACE inhibi tors such as l i s inopri l , captopri l , and others ). Its main antihypertens ive actions , therefore, include only blockade ofangiotens in I I-mediated a ldosterone release from the adrenal cortex, and of angiotens in I I ’s vasoconstrictor effects .

Li s inopri l and the other ACE inhibi tors inhibi t angiotens in I I synthes is , and that effect accounts (indi rectly) for reduced a ldosterone release andAII-mediated vasoconstriction. They a lso inhibi t metabol ic inactivation of bradykinin, an endogenous vasodi lator (bradykininase, angiotens in-converting enzyme, and kininase II are synonyms for essentia l ly the same enzyme).

Neither an ACE inhibi tor nor an ARB i s associated with an increased incidence of bronchospasm (b; a l though some ACE inhibi tors may causecough, presumably from loca l ly increased bradykinin levels , and angioedema, perhaps by the same mechanism). Nei ther elevates urate levels .

There i s some evidence that angiotens in I I enhances (not inhibi ts ) sympathetic-mediated vasoconstriction (by increas ing neuronalnorepinephrine release and/or blocking neuronal norepinephrine reuptake). Ei ther an ACE inhibi tor or an ARB should be equiva lent in

Page 157: Pharmacology pre test self assessment and review (14th ed.)

attenuating that blood pressure-elevating effect.ACE inhibi tors are cons idered to be an “essentia l” part of therapy (a long with β-blockers and a diuretic) for most patients with heart fa i lure.

Angiotens in receptor blockers such as losartan may prove to be equal ly effective a l ternatives .By the way: You didn’t learn expl ici tly about l i s inopri l and so didn’t recognize i ts class i fi cation? Remember: drugs with generic names that end

in “-pri l ” are ACE inhibi tors .

239. The answer is d. (Brunton, pp 959-960; Katzung, p 616.) Ezetimibe inhibi ts absorption of dietary cholesterol through the gut. It has no hepatic orother di rect effects that contribute to i ts cl inica l ly des i red effect. The drug lacks cardiac effects (a ) or vascular effects (b). There i s no goodevidence that ezetimibe increases the ri sks of plaque rupture, and the subsequent and often devastating consequences of that. Other l ipid-lowering drugs , such as niacin, the fibrates , and perhaps some of the cholesterol -binding res ins , seem to increase the ri sks of s tatin-inducedmyopathy, rhabdomyolys is , or l iver dys function. Such increased ri sks are the major problems to be faced when s tatins are used in combination ofother l ipid-lowering agents . However, this seems not to be a problem with ezetimibe. It i s a rational drug to combine with a s tatin, and oneproprietary fixed-dose combination that conta ins ezetimibe and a s tatin i s ava i lable and qui te widely used.

240. The answer is e. (Brunton, p 771; Katzung, pp 611-612.) “Time i s muscle,” some say when address ing the i ssue of thrombolys is for coronaryocclus ion. (Those who use a lytic drug for i schemic s troke wi l l s imply say “time i s ti s sue.”) Idea l ly, the “door to needle” time should be 90 minutesor (preferably) much less . It i s true that choos ing a l teplase (a) or some other t-PA variant may be associated with a lower ri sk of bleeding, owing totheir clot-speci fic s i te of action; and that i t i s relatively safer to use them with heparin. In contrast, s treptokinase adminis tration once may lead toantibodies that, i f the patient i s retreated with the same drug (within about 5 days to 2 years ), may ei ther or both neutra l i ze the drug (rendering i tineffective) or trigger an a l lergic reaction. However, I s tated this was our patient’s fi rs t MI. (You could argue that the patient may a l ready have anti -SK antibodies from a prior s treptococca l infection, but you didn’t think of that, did you?)

The key point, then, i s s tarting thrombolys is as quickly as poss ible, not which drug i s chosen. There seems to be maximum benefi t (sa lvagingtissue, reducing ul timate infarct s i ze, mainta ining reasonable cardiac function, and reducing morta l i ty) i f a thrombolytic—any thrombolytic—isadminis tered within 1 hour of symptom onset. According to many s tudies , benefi ts of thrombolys is become comparable to placebo i f the drug i sn’ts tarted unti l around 6 hours after symptom onset, and in terms of morta l i ty thrombolytics may actua l ly increase the ri sk of death i f not given afterabout 12 hours or so.

241. The answer is c. (Brunton, pp 752, 868f, 871t; Katzung, pp 613, 617, 994, 999.) Abciximab, and such related drugs as ti rofiban and epti fibatide, blockthe glycoprotein IIb/II Ia receptor on platelets . Abciximab i s an antibody ra ised aga inst the IIb/II Ia receptor. It binds to the receptor, and in doingso blocks the formation of fibrinogen bridges between the IIb/II Ia receptors on adjacent platelets . That i s , otherwise, the fina l cri ti ca l s tep incaus ing platelets to aggregate with thei r neighbors .

It i s a l so noteworthy that platelet activation can be triggered by thromboxane A2, thrombin, col lagen, ADP, and platelet-activating factor (PAF)(among other agonis ts ). Block the activating effects of just one of those factors (eg, those of ADP with clopidogrel ), and the proaggregatory effectsof the other activators are left unchecked. Platelet activation ensues ; some platelet-derived activators are released, thereby recrui ting theactivation of more platelets—the ampl i fi cation process .

Abciximab, in contrast, blocks the “fina l s tep” in aggregation, regardless of the presence or absence of influences from “upstream” activators .

242. The answer is c. (Brunton, pp 212, 305-307; Katzung, pp 152f, 154, 167, 180, 189.) Phenylephrine, as noted, causes vasoconstriction through i tsagonis t activi ty on α-adrenergic receptors . Prazos in competi tively blocks those receptors , and s ince we have a patient who has taken an excess ivedose of the antagonis t i t should be no surprise that usual ly effective (or even higher) doses of phenylephrine wi l l not be able to overcome thatblockade. None of the other drugs l i s ted have any di rect effects on an α-adrenergic receptors , nor the abi l i ty of phenylephrine to activate them.(Thiazides probably exert thei r antihypertens ive effects by caus ing some net sodium depletion, which reduces vascular respons iveness toadrenergic agonis ts , angiotens in I I , and any other vasoconstrictors you may conjure. However, i t i s highly unl ikely that a thiazide would completelyel iminate any vasopressor effects of a usual dose of phenylephrine—let a lone repeated doses .)

243. The answer is e. (Brunton, pp 305-307; Katzung, pp 152f, 154, 167, 180, 189.) This i s an apt description of an α-adrenergic blocker, and not at a l ldescriptive of the actions or effects of any other drug l i s ted. Prazos in can be cons idered the prototype of the α-adrenergic blockers , at least thosethat selectively block α1 receptors (in comparison with phentolamine and severa l other drugs , which block both α1 and α2 receptors ). It’s not l ikelyyour patient was taking prazos in i tsel f (or a related drug l ike doxazos in or terazos in). That i s because those drugs not only exert s igni ficantinhibi tory effects on smooth muscle of the prostate capsule and urethra , but a lso on the periphera l vasculature. They could be (and sometimesare) used for benign prostatic hypertrophy. If the patient i s a l so hypertens ive, one drug may help both conditions . If blood pressure i s normal , orcontrol led wel l wi th other antihypertens ives , then prazos in or doxazos in may lower pressure too much (unless we change dosages or drugs). Morel ikely your patient i s taking tamsulos in. It’s clearly in the same class as prazos in, but seems to have more selectivi ty for smooth muscles in theurinary tract, and fewer or mi lder periphera l vascular actions that would tend to lower blood pressure and trigger the other responses I noted.

244. The answer is e. (Brunton, pp 730-736; Katzung, pp 184-185, 1054t.) ACE inhibi tors (and angiotens in receptor blockers , ARBs eg, losartan) arecontra indicated in women who are pregnant or of chi ld-bearing potentia l . ACE inhibi tors and ARBs do not exacerbate asthma (a); they arecons idered fi rs t-l ine adjunctive drugs for managing most patients with heart fa i lure (b), which may or may not be an accompaniment ofhyperl ipidemias or coronary artery disease (c). We would want to normal ize potass ium levels i f the patient were hypokalemic (d; but i t dependson what the absolute potass ium levels are, and whether the hypokalemia i s symptomatic), before we gave an ACE inhibi tor, but one would wantto be very careful in terms of how to go about doing that. ACE inhibi tors and ARBs ul timately lower ci rculating a ldosterone levels , which in turnfavors more renal potass ium retention. One of these drugs , a lone, might correct potass ium levels . Giving an ACE inhibi tor or ARB a long with apotass ium-sparing diuretic may not merely correct the hypokalemia but, eventual ly and more l ikely, cause hyperka lemia.

245. The answer is a. (Brunton, pp 225-235; Katzung, pp 115-124.) Sinus bradycardia can be conceptual ly viewed as an imbalance of two oppos ingfactors on the SA node: (a) too great an influence of the parasympathetic nervous system, speci fica l ly, of ACh acting on muscarinic receptors ; or (b)too l i ttle activation of β1 adrenergic receptors . We can address ei ther, or both, imbalances pharmacologica l ly. According to current ACLS guidel ines ,us ing atropine to block excess ive parasympathetic influences , a l lowing heart rate to go up, i s the recommended fi rs t approach.

An a l ternative approach would be to activate β1 adrenergic receptors , eg, with i soproterenol . Amiodarone (b) i s nei ther effective nor indicatedfor s inus bradycardia . It i s used as a fi rs t-l ine drug for cardiac arrest; i s indicated for managing some serious and l i fe-threatening ventriculararrhythmias ; and has gotten growing attention for i ts apparent efficacy to manage atria l fibri l lation—a rather unusual use given the seriousnessof the other arrhythmias for which the drug i s given. Edrophonium (c) would be among the worse drugs you could give to this bradycardic patient.This rapidly acting ACh esterase inhibi tor wi l l cause accumulation of even more ACh at the SA node (and elsewhere in the autonomic nervous

Page 158: Pharmacology pre test self assessment and review (14th ed.)

system where ACh i s the neurotransmitter activating smooth muscles and various exocrine glands . If the patient’s heart doesn’t s top in responseto edrophonium, assuming we give a usual dose, i t i s mere luck. Lidoca ine (d) i s effective for a host of ventricular arrhythmias , including thoseassociated with an MI. It wi l l do nothing good for the bradycardia . You might argue that phentolamine (e), the prototype α-adrenergic blocker,would work. It might, but probably at great cost to this patient’s hemodynamic s tatus . When we inject “usual” doses of phentolamine heart rateindeed goes up. That i s mediated by the baroreceptor reflex: we have given enough drug to rapidly and markedly drop blood pressure via thedrug’s vasodi lator actions . That, in turn, “withdraws” centra l parasympathetic tone and causes a relative increase in sympathetic outflow. Botheffects would increase the heart rate. Unfortunately, our bradycardic patient probably a l ready has a blood pressure and cardiac output that are onthe low s ide of conducive to a long and happy l i fe. Even i f he didn’t, caus ing blood pressure to promptly and markedly fa l l often doesn’t lead to afavorable outcome.

246. The answer is e. (Brunton, pp 752, 767-768; Katzung, pp 193-208.) In terms of autonomic control of coronary vascular tone, envisage a “ba lance”between the vasoconstrictor influences of α-adrenergic activation and the oppos ing vasodi lator influences of β2-receptor activation. Spasm-pronecoronaries appear to be very dependent on the β-mediated vasodi lator influences to reduce the incidence and severi ty of spasm. Any β-blockerwi l l remove those favorable influences , and tend to provoke (or intens i fy or prolong) spasm and the resul ting i schemia in dis ta l ti s sues . Thisvascular effect i s l ikely to overshadow any beneficia l effects attributed to reductions of myocardia l oxygen demand via suppress ion of heart rateand contracti l i ty.

You could correctly argue that in the absence of demonstrated coronary occlus ion the adminis tration of t-PA (a) would be inappropriate.However, that drug i s not l ikely to make matters worse. Captopri l (c), the prototype ACE inhibi tor, should have no negative or pos i tive effects onvasospasm. Ni troglycerin (d) might reduce the incidence or severi ty of spasm through ni tric-oxide-mediated vasodi lation. Verapami l (f) wouldprobably be one of the most rational drugs to give, s ince i ts vascular ca lcium channel blocking activi ty would help suppress spasm; i ts negativeinotropic and chrono-tropic effects would, addi tional ly, reduce myocardia l oxygen demand and be beneficia l .

247. The answer is b. (Brunton, pp 767, 971-972; Katzung, pp 612, 638-640.) That aspi rin might increase the ri sk of s troke should not be surpris ing. Thesame mechanism by which the drug exerts protective effects aga inst MI, by inhibi ting platelet aggregation, expla ins why some patients whootherwise might develop a cerebra l bleed are at increased ri sk of doing so. There i s no good evidence that aspi rin ingestion causes hepatotoxici tyof any sort (a ); tachycardia and/or hypotens ion (d; other than via blood loss and hypovolemia due to excess ive bleeding); nor causes or provokescoronary vasospasm (e). Long-term use of many nonsteroida l anti -inflammatory drugs may cause nephropathy, but that i s typica l ly associated withhigh-dose use, and rarely with aspi rin i tsel f.

248. The answer is d. (Brunton, pp 113, 732-733, 738-739, 784-785; Katzung, pp 183-185, 219-220.) The patient has been taking two diuretics , onepotass ium-wasting (the thiazide), the other potass ium-sparing. Ramipri l i s an ACE inhibi tor (reca l l : drugs with generic names ending in “-pri l ” aremembers of that class ) that ul timately wi l l lower a ldosterone levels . This wi l l , in essence, counteract the sodium-reta ining effects of a ldosterone,and a lso i ts potass ium-wasting effects . So by adding the ACE inhibi tor to the regimen we wi l l now be treating the patient with two drugs that tendto elevate potass ium levels to a degree that cannot be compensated for by the thiazide.

Adding an ACE inhibi tor to your patient’s regimen would be a good idea (provided there are no contra indications to doing so) in terms of gettingadditional blood pressure control , but i t would probably require el iminating the triamterene at some point (i f not from the s tart) to avoid thehyperka lemia.

Prazos in (b), a selective α1-adrenergic blocker, i s not at a l l l i kely to have el ici ted the hyperka lemia. Di l tiazem (a) and verapami l (e), both non-dihydropyridine ca lcium channel blockers , are not l ikely to affect potass ium levels in a measurable way. β-Adrenergic blockers (eg, propranolol , c)have been reported to lower potass ium levels , presumably by enhancing K+ uptake into skeleta l muscle. These drugs are not, therefore, a l ikelycause of the hypokalemia, even when used in combination with the diuretics .

249. The answer is c. (Brunton, pp 206t, 211, 302t, 824t; Katzung, pp 136-138, 139f.) Phenylephrine i s the prototypic α-adrenergic agonis t. It terminated thearrhythmia reflexly, via the baroreceptors , in response to a vasopressor effect. (Ra is ing blood pressure quickly and markedly i s a ri sky way ofterminating this tachycardia , of course, due to such ri sks as caus ing a hemorrhagic s troke.) Al l the other drugs would a lso terminate thearrhythmia, but by actions in the heart. Edrophonium (a) i s an acetylchol inesterase inhibi tor with a fast onset of action and a brief duration. Itwould s low heart rate by increas ing the effects of ACh on the SA node, s lowing the spontaneous rate of phase 4 depolarization. Esmolol i s a β-blocker (nonselective) that a lso has a fast onset and short duration of action. It and propranolol (d) would s low heart rate via the di rect β-blockingeffects on the SA node. Verapami l (e) wi l l do the same by blocking AV nodal ca lcium channels .

250. The answer is b. (Brunton, pp 801-804, 835t; Katzung, pp 216-217.) Answering this question, of course, requires that you not only know the expectedeffects of digoxin, but a lso that you can “trans late” those effects into interpretations of findings from a most useful diagnostic tool , theelectrocardiogram.

An expected and important effect of digoxin i s s lowed atrioventricular nodal conduction veloci ty, mani fest as prolongation of the PR interva l .The effect, which can lead to increas ing degrees of heart block, depends on the plasma concentration of digoxin and on the concentrations ofsevera l ions , potass ium arguably the most important.

Digoxin tends to speed electrica l impulse veloci ty through the atria l myocardium, and so widening of P waves (a) would be the oppos i te of theexpected response. There i s no good reason to expect increased P-wave ampl i tude due to the digoxin. (Atria l enlargement, a resul t of the heartfa i lure and not the drug, i s l ikely.)

Digoxin a lso speeds electrica l impulse conduction veloci ty through the ventricles (eg, the His -Purkinje system), and so widened QRS complexes ,compared with basel ine, would be counter to the expected effect.

RR interva ls essentia l ly reflect ventricular rate. Before treatment of heart fa i lure there are varying degrees of compensatory “sympathetic drive”over the heart, leading to tachycardia and shortened RR interva ls (d). Once digoxin s tarts increas ing cardiac output, there i s a lessening ofsympathetic drive (and the phys iologic need for i t). Thus , compared with basel ine heart rate, post-treatment heart rate i s s lower; this i s mani festas a longer RR interva l compared with basel ine.

ST segment changes are among the mani festations of acute coronary syndrome and regional myocardia l i schemia. Based on the description ofthe patient, who appears to have normal blood profi les and no evidence of acute i schemia, ST elevation (e) i s not a reasonable answer.

251. The answer is e. (Brunton, pp 210, 828, 831-832; Katzung, pp 155-156.) Pheochromocytomas are epinephrine-secreting tumors , and so the worrisomeand dangerous consequences are due to excess ive vasoconstriction (α-mediated) and cardiac chronotropic and inotropic responses (β1). A β-adrenergic blocker i s an essentia l component of pharmacotherapy. It must be adminis tered a long with another drug that blocks α-adrenergicreceptors , but should not be given before the α-adrenergic blocker i s given. Activation of β2-adrenergic receptors in the periphera l vasculature

Page 159: Pharmacology pre test self assessment and review (14th ed.)

plays a smal l but important role in blood pressure control (s ince β2 activation in some vascular beds causes vasodi lation). Block that effect in apatient with epinephrine excess—a pheochromocytoma—and any potentia l ly blood pressure-lowering effects are lost. Blood pressure wi l l ,therefore, ei ther ri se or s tay the same in terms of periphera l vascular effects .

In the face of this s ti l l -high blood pressure and tota l periphera l res is tance the β-blocker wi l l reduce both heart rate and contracti l i ty, and socardiac output wi l l fa l l (reca l l that CO = HR × SV). The outcome may be not merely a fa l l of cardiac output, but a cons iderable fa l l in a very shorttime that might wel l put the patient into acute heart fa i lure.

Blood pressure i s not at a l l l i kely to fa l l promptly due to any periphera l vasodi lator effects ; i f i t did (and i t won’t), the β-blocker would preventreflex tachycardia (a) via di rect actions on cardiac β1 receptors . The β-blockers do not inhibi t catecholamine release (b), whether from acatecholamine-secreting tumor or from a normal sympathetic nervous system. A β-blocker wi l l not trigger catecholamine release from apheochromocytoma (c), nor wi l l i t normal ize cardiovascular s tatus in any other way i f given without an α-blocker. A β-blocker a lone, in thiss i tuation, wi l l ra ise left ventricular a fterload, and reduce cardiac output by effects on both heart rate and s troke volume. Thus , answer e i s wrong.

252. The answer is d. (Brunton, pp 760, 768-771; Katzung, pp 195-201.) Ni troglycerin and other ni trovasodi lators provide immediate symptom rel ief inmost patients with an acute coronary syndrome or MI. However, there i s no evidence that these drugs provide long-term preventative effectsaga inst sudden death from an acute MI. In contrast, aspi rin (a ; antiplatelet drug), a l ipid-lowering drug such as atorvastatin (b), ACE inhibi torssuch as captopri l (c), and β-blockers such as propranolol (e), have documented protective effects to reduce the ri sk of death from a subsequent MI.

253. The answer is a. (Brunton, pp 206t, 290, 804-805; Katzung, pp 141-142, 144, 148, 218, 225.) Intravenous infus ion of dobutamine, a β1 agonis t, was themost l ikely cause of the increased heart rate. Blood pressure probably rose secondary to a pos i tive inotropic effect of the drug; i t cannot be by anydirect effect on tota l periphera l res is tance, s ince even at high doses dobutamine has no vasoconstrictor activi ty. Esmolol (b) i s a nonselective β-blocker with a rapid onset of action, and a brief duration of action due to rapid hydrolys is by plasma esterases . Being a β-blocker, nei ther i t norpropranolol (d) would ra ise heart rate or blood pressure. Neostigmine (c) i s an ACh esterase inhibi tor. It and verapami l (e), a nondihydropyridineca lcium channel blocker, would be expected to reduce heart rate. Vera-pami l would a lso tend to lower, not ra ise, blood pressure by vi rtue of i tsperiphera l vasodi lator action.

254. The answer is a. (Brunton, pp 752, 767-768; Katzung, pp 159t, 161, 175t, 179.) Metoprolol i s primari ly a β1-adrenergic blocker at low doses , but athigh doses can block β2 receptors a lso. Thus , i t i s sometimes ca l led a “cardioselective” β-blocker, much l ike atenolol . Most of the β-blockers haveno vasodi lating activi ty (noteworthy exceptions are labeta lol and carvedi lol , which have α-blocking activi ty; and nebivolol , which vasodi lates via ani tric oxide-dependent process ).

β-Adrenergic receptor blockers s low resting heart rate and reduce contracti l i ty, both of which reduce myocardia l oxygen demand. They a lso bluntcardiac s timulatory responses that increase oxygen demand, whenever the sympathetic nervous system is activated (eg, in response to exercise ordrugs that tend to cause reflex sympathetic activation). These a l l involve blockade of β1 receptors .

Metoprolol does not cause coronary vasodi lation (and may favor constriction, which can become cl inica l ly s igni ficant in patients with variantangina, by “unmasking” α-mediated coronary vasoconstriction; see Question 188; thus answer b i s incorrect); and i t may increase (not decrease, d)tota l periphera l res is tance by blocking β2-mediated di lation in some vascular beds . This effect i s usual ly s l ight (s tronger with the vasodi lating β-blocker noted in the fi rs t paragraph). Al l the β blockers s low AV nodal conduction veloci ty, but that effect per se contributes l i ttle to the reducedoxygen demand that i s mainly derived from the drugs ’ effects on overa l l rate and contracti l i ty. Thus , answer e i s incorrect.

Al l the β-blockers reduce angiotens in I I synthes is (c), but i t i s an indirect effect, involving β1 blockade, due to reduced renin release from thejuxtaglomerular apparatus of the kidneys . Al though that effect clearly occurs , i t i s not a major mechanism by which β-blockers exert thei rantiangina l effects , nor i s the magnitude of the inhibi tion of angiotens in I I synthes is on par with that caused by ACE inhibi tors .

255. The answer is b. (Brunton, pp 941, 977-982, 849, 964; Katzung, pp 314-316, 323-324, 638-640.) At the usual low cardioprotective doses (usual ly 81mg/day) of aspi rin, the main and therapeutica l ly useful effect of the drug i s inhibi tion of thromboxane A2 (TXA2) synthes is via the COX-1 pathway.Reca l l that TXA2 i s one important—but certa inly not the only—trigger of platelet activation, ampl i fi cation, and aggregation. However, at high(er)doses , aspi rin a lso inhibi ts synthes is of other eicosanoids , of which PGI2 (prostacycl in), synthes ized in the vascular endothel ium, i s of mostimportance here. Endothel ia l prostacycl in synthes is helps prevent platelets from adhering to the vascular wal l . Suppress PGI2 in the endothel ium,and platelet adherence i s increased, despi te the fact that platelet TXA2 synthes is has a l ready been blocked. It’s important to remember here thataspirin does nothing to inhibi t platelet activation by such other agonis ts as col lagen or ADP. By lowering endothel ia l PGI2 levels we’ve increasedthe l ikel ihood that platelets activated by eicosanoid-independent agonis ts wi l l adhere to the l ining of blood vessels , and potentia l ly occludethem.

Aspirin has no effect on Gp IIb/II Ia receptors (a); i t does not rupture or otherwise damage vascular plaques (c) to expose col lagen; inhibi tsynthes is of any l iver-based clotting factors (d); or ampl i fy or otherwise enhance the effects of ADP (e) or of other platelet activators .

256. The answer is b. (Brunton, pp 898-899; Katzung, pp 629-630, 633, 760.) Colesevelam is the newest bi le-acid sequestrant, and i s the drug of choicewhen a bi le acid-sequestrant i s indicated. The older agents are cholestyramine and colestipol . These drugs , which are given ora l ly, arenonabsorbable res ins that bind (sequesters , adsorbs) many substances that are present in the GI tract (eg, bi le sa l ts ) at the same time as thedrug, prevents them from being (re)absorbed, and faci l i tates thei r el imination in the feces . None of these drugs have any effects on renalhandl ing of electrolytes , non-electrolytes , or free water (a , e); no effects on urate levels (c); have no di rect antiangina l effects , and certa inly nonegative inotropic or chronotropic effects ; and are not antihypertens ive (e).

Colesevelam may be used a lone, but usual ly i t i s in combination with a s tatin. On average, colesevelam a lone can lower LDL cholesterol byabout 20% (the typica l range i s between 15% and 30%). In contrast, combined therapy with a s tatin can reduce LDL cholesterol by up to 50%. Simi larresul ts can be obta ined by combining a sequestrant with niacin. LDL-lowering effects begin soon after dos ing s tarts , but i t may take a month or sofor effects to peak.

Al l the bi le-acid sequestrants lower LDL cholesterol through a mechanism that ul timately depends on increas ing LDL receptors on hepatocytes ,and that occurs via a sequence of related s teps . Bi le sa l ts and the cholesterol from which they are made are excreted into the intestines .Normal ly, thereafter, a good portion of the excreted bi le sa l ts and cholesterol i s reabsorbed and recycled in a process usual ly ca l ledenterohepatic reci rculation. The bi le acid sequestrants interrupt the recycl ing. They adsorb the cholesterol -rich bi le acids in the intestine andform an insoluble complex that prevents the bi le sa l ts and cholesterol from being returned to the ci rculation. As a resul t, the excretion ofcholesterol -laden bi le sa l ts increases . This increased excretion creates a demand for increased bi le acid formation in the l iver; and that, in turnrequires increased cholesterol supply to the l iver. That cholesterol comes from LDL. In order to use more LDL cholesterol the l iver cel l s increasetheir number of LDL receptors . That increases their capaci ty for LDL uptake. The resul t i s an increase in LDL uptake from plasma, and a decrease of

Page 160: Pharmacology pre test self assessment and review (14th ed.)

ci rculating LDL levels . With continued adminis tration of the sequestrant the cycle repeats i tsel f as bi le sa l t excretion continues to increase, andover time the eventual resul t i s a fa l l of LDL levels .

Bi le-acid sequestrants may increase VLDL levels in some patients . In most cases , the elevation i s temporary and mi ld. However, i f VLDL levelsare elevated before treatment, the bi le-acid sequestrants may cause further susta ined and substantia l VLDL increases . Therefore, bi le-acidsequestrants are not drugs of choice for lowering LDL cholesterol in patients with high VLDL levels .

Colesevelam is now the preferred bi le acid sequestrant because i t di ffers from cholestyramine and colestipol in at least four main ways : (1)Colesevelam is better tolerated (less constipation, flatulence, bloating, and cramping). (2) It does not reduce the absorption of fat-solublevi tamins (A, D, E, and K), whether from foods , beverages , or from vi tamin supplements . (3) It does not s igni ficantly reduce ora l absorption ofs tatins , digoxin, warfarin, and most other drugs s tudied that are present in the GI tract at the same time as the sequestrant. (4) Fina l ly, i t i s ofcourse very common for patients with CHD ri sk to have hypercholesterolemia and diabetes . Colesevelam a lso causes des i rable glucose-control l ingeffects for many patients with type 2 diabetes (ie, about 90% of a l l cases of diabetes ) when used in combination with such other antidiabeticmedications as metformin, sul fonylureas , or insul in. It i s the only bi le acid sequestrant approved for use as an antidiabetic adjunct.

257. The answer is e. (Brunton, pp 128t, 320, 831-832, 864; Katzung, pp 157-164.) Ni tropruss ide (or any other IV vasodi lator, eg, ni troglycerin) i s l ikely toel ici t baroreceptor reflex-mediated ri ses of heart rate and contracti l i ty. It may be s igni ficant and dangerous for any patient, more so for patientswith i schemic heart disease, and potentia l ly deadly for patients with aneurysms, such as our patient. For aortic aneurysm patients , the problem isnot the ri se of heart rate, but rather of left ventricular contracti l i ty (left ventricular dP/dt—the change of ventricular pressure over a speci fic changeof time). The bounding aortic pressure pulse with each systole favors rupture of the aneurysm, and use of a β-blocker such as propranolol i s aneffective and common way to minimize the ri sk (and, of course, reflex cardiac s timulation overa l l ).

Atropine (a) i s an i l logica l choice. It wi l l increase heart rate (and, indi rectly, contracti l i ty) further by removing parasympathetic tone on the SAnode. Diazoxide (b) i s a rapidly acting antihypertens ive/vasodi lator, given as an IV bolus in s i tuations where safe ni tropruss ide use (and theinvas ive monitoring i t normal ly requires ) i s not practica l . Diazoxide, l ike ni tropruss ide, triggers reflex cardiac s timulation as blood pressure fa l l s .The drug a lso has some di rect cardiac-s timulating effects (not involving antimuscarinic or adrenergic-agonis t effects ). Diazoxide a lso tends tocause hyperglycemia (an ora l dosage form of the drug i s sometimes prescribed to manage blood glucose levels in patients prone to developinghypoglycemia.) Furosemide (c) i s a loop diuretic that i s an important adjunct in such conditions as hypertens ive cri s i s , but only i f due to oraccompanied by volume overload, heart fa i lure, and acute pulmonary edema. However, the volume depletion and hypotens ion i t would causewould exacerbate the cl inica l s i tuation I have described. The prompt diures is and fa l l of blood pressure may a lso trigger unwanted baroreceptor-mediated cardiac s timulation. Phentolamine (d), an α-adrenergic blocker, would be i rrational . It i s a powerful vasodi lator (indeed, i t i s often usedfor vasoconstrictor drug-induced hypertens ive emergencies ) that would add to—unfavorably—nitropruss ide’s prompt antihypertens ive effect andthe reflex cardiac s timulation that i s so dangerous for our patient.

258. The answer is c. (Brunton, p 972; Katzung, pp 627-628.) You may argue about including this question, but there are at least two main reasons forasking i t. (1) Niacin i s s ti l l a widely used drug for managing certa in but common hyperl ipidemias , and the s ide effects described are common. (2)Al though the mechanis tic explanation i s based largely on empiric observations , i t should help you understand fundamenta l actions of a large andwidely used group of drugs , the cyclooxygenase inhibi tors (eg, nonsteroida l anti -inflammatory drugs).

(1) Al though prescribing habi ts have changed a bi t over the las t year or so, niacin i s the most common add-on to s tatin therapy when a s tatina lone does not lower HDL adequately. We are ta lking about mi l l ions of patients ! The cutaneous flushing and pruri tus , and the underlyingvasodi lation caused by niacin, are common (especia l ly with high doses of niacin and/or certa in ora l formulations). In addition, these s ide effectsare qui te dis turbing to many patients who experience i t.

(2) The problems a lmost certa inly involve prostaglandins . We can reach that conclus ion rather empirica l ly, based on the fact that theseresponses may be prevented by the prior adminis tration of aspi rin, which of course blocks prostaglandin synthes is . Al though i t’s not true 100% ofthe time, far more l ikely than not, if a response is attenuated or abolished by aspirin (or any other traditional NSAID) the mechanism will involve inhibition ofcyclooxygenase(s) and the resul ting inhibi tion of the synthes is of one or more prostaglandins or thromboxanes (eicosanoids ).

α-Adrenergic receptor activation (a) would tend to counteract, not cause or contribute, to the vasodi lation that i s involved in the flush. Ca lciumchannel blockers have no effect on the phenomenon, so we can rule out answer b. Simi larly, drugs that inhibi t angiotens in synthes is or i tsreceptor activation (d), or his tamine receptor blockers (e; whether H1 or H2) largely have no effect on the flushing and pruri tus .

259. The answer is c. (Brunton, pp 274-277, 304-316; Katzung, pp 136-138, 144-145.) ISO, PHE, EPI, NE. Yes , some responses to these drugs are variable,largely depending on the dose and the speed of adminis tration. Nonetheless , the responses shown are typica l .

ISO, a β1/β2 agonis t, lowers periphera l res is tance (and diastol ic blood pressure) via β2-mediated periphera l vasodi lator actions . That fa l l ofdiastol ic pressure i s greater than the ri se of systol ic pressure (which occurs because of a di rect cardiac pos i tive inotropic/β1-activating effect), somean pressure fa l l s a bi t. The combined reflex response to a fa l l of mean pressure (a lbei t s l ight), plus the drug’s di rect pos i tive chronotropiceffect, leads to s igni ficant tachycardia .

PHE, which activates only (and a l l ) α-adrenergic receptors , causes only a vasopressor response that accounts for the changes of pressures andperiphera l res is tance. These changes activate the baroreceptor reflex, leading to reflex bradycardia . The drug has no β-agonis t activi ty.

EPI, injected in (reasonably) low doses in normal humans , can cause the effects shown here (and certa inly no other drug l i s ted as a poss ibleanswer could do the same). The fa l l of periphera l res is tance and diastol ic pressure reflects predominant β2-mediated vasodi lation; the ri se ofsystol ic pressure i s a melding of both periphera l vasoconstriction (α) and di rect cardiac s timulation (β1). Heart rate a lso reflects di rect changes(β1), as there i s no appreciable baroreceptor influence because there i s no sudden or s igni ficant blood pressure change. Responses toepinephrine are arguably more variable than those of any other drugs l i s ted. Clearly, when large doses are given to a hypotens ive patient (eg, inanaphylaxis ), the predominant and wanted vascular effect i s a pressor response, much greater than what we see here.

NE, lacking any β2 activi ty but being qui te effective as an α and β1 agonis t, causes typica l responses l ike those shown here. Periphera l ly, there i sno vasodi lation; just constriction. Diastol ic, mean, and systol ic pressures ri se. The ri se i s sufficient to reflexly s low heart rate; that i s , i t i ssufficient to overcome NE’s di rect pos i tive chronotropic effects .

260. The answer is a. (Brunton, pp 171-175; Katzung, pp 136-141.) You should know the answer to this from a bas ic phys iology course. We have mimicked(but not caused) a sudden ri se of blood pressure, s tretching the baroreceptors and activating the baroreceptor reflex. Consequences of thisinclude a reduction in sympathetic “outflow” from the CNS (and a reduction of catecholamine release), and a s imultaneous unmasking andincrease of oppos ing parasympathetic influences . The increased muscarinic receptor activation breaks the tachycardia , mainly by s lowingspontaneous (Phase 4) depolarization of SA nodal cel l s .

261. The answer is d. (Brunton, pp 815-817; Katzung, pp 231-236, 245.) The origin of the arrhythmia i s supraventricular (any s tructure “above” the AV

Page 161: Pharmacology pre test self assessment and review (14th ed.)

node), and in this case both the expected effects of ACh and of carotid massage suggest the origin i s the SA node i tsel f. If you look at the ECG youshould be able to rule out the other (and incorrect) answer choices . The ventricular wave form shows no s lurring or “hump” that would suggestpreactivation of the ventricles by an accessory pathway (as you might see in Wolff-Parkinson-White syndrome). There i s no notching of the QRS thatwould be cons is tent with bundle branch block (right or left). And, s ince ventricular activation fol lows atria l activation (P waves), we are not deal ingwith ventricular ectopy.

262. The answer is a. (Brunton, pp 825, 833t-837; Katzung, pp 245-246.) Adenos ine interacts with G-protein-coupled receptors and increases maximumdiastol ic membrane potentia l . When given as a bolus , which was used here, the main effects are a trans ient s lowing of s inus (nodal ) rate; anincrease of AV nodal refractoriness ; and a s lowing of AV nodal conduction veloci ty. Also, with this adminis tration route, there i s no effect on theHis -Purkinje system, nor on ventricular myocardia l cel l s per se. Given the lack of ventricular effects , adenos ine would not have terminated thearrhythmia i f the origin was in the ventricles . Stated otherwise, s ince the drug does act on the AV node, and i t did terminate the origina larrhythmia, we must conclude that the arrhythmia originated in the AV node or another supraventricular s tructure.

Atropine, by blocking the electrophys iologic effects of muscarinic receptor activation (in the SA node, for example), would have unmaskedoppos ing sympathetic (β1) influences and probably worsened (but certa inly not s topped) the aberrant electrica l activi ty. Isoproterenol orepinephrine would have done the same, a lbei t by di rect activation of β1-adrenergic receptors .

We can rule out l idoca ine as an acceptable answer in severa l ways . Fi rs t, we are deal ing with an arrhythmia originating in a supraventriculars tructure. Lidoca ine has no s igni ficant effect on SA nodal rate, nor on AV nodal refractory periods . It a l so has no effects on PR interva ls (because i tlacks s igni ficant effects on AV nodal refractoriness or conduction), QT interva ls , or the duration of the QRS. Stated more pragmatica l ly, the drug i snot useful for supraventricular arrhythmias (except, perhaps , digoxin-associated atria l arrhythmias ). It would not work in the s i tuation describedor shown in the ECG.

263. The answer is a. (Brunton, pp 310, 347-349; Katzung, pp 155, 287-290, 292.) Al though ergotamine, and the ergot a lka loids in genera l , may not becons idered typica l cardiovascular drugs , they clearly can cause s igni ficant, cumulative, and sometimes dangerous cardiovascular effects .Ergotamine can cause intense and prolonged vasoconstriction in both the periphera l and coronary vasculatures . Thus , of the answer choices givenmyocardia l i schemia and i schemia of (for example) one or more extremities are the most l ikely outcomes. These vascular responses poseparticular problems for patients with i schemic heart disease, especia l ly those who have vasospastic, or variant, angina, or patients withperiphera l vascular disease. (One ergot a lka loid, ergonovine, i s such a powerful coronary vasoconstrictor that i t i s sometimes used inangiographic s tudies of the heart to diagnose vasospastic angina.) The coronary effects may be rapidly fata l ; l imb i schemia may be sufficientlyintense and prolonged to induce gangrene and require amputation of the affected l imb(s ). Rhabdomyolys is , wi th or without renal fa i lure (b), i sunl ikely, at least as a di rect consequence of ergotamine, as the ergot compounds in genera l have no appreciable effects on skeleta l musclestructure or function. There are no di rect effects on platelet activation or aggregation that would lead to spontaneous bleeding (c); however,platelet activation and thrombotic events certa inly may occur secondary to s tas is in vascular beds that have been intensely constricted by the drug.Hypertens ion, not hypotens ion with or without syncope (d), i s a l ikely consequence of periphera l vasoconstriction. Sudden ri ses in blood pressureare l ikely to cause bradycardia (not tachycardia , e) via baroreceptor reflex activation. Members of the ergot a lka loid class (eg, ergotamine,ergonovine) have variable agonis t or antagonis t effects (di rect or indi rect) on α–adrenergic, dopaminergic, and serotoninergic receptors ), but noton β-adrenergic receptors .

[For s tudy purposes in other areas of pharmacology you may want to reca l l the fol lowing: (1) Triptans (eg, sumatriptan) or methysergide tends tobe used more than ergotamine or s imi lar drugs for migra ine. (2) Ergo-novine i s sometimes used as an a l ternative or backup to oxytocin forcontrol l ing postpartum uterine bleeding or hemorrhage: i t causes intense and prolonged uterine smooth muscle contraction and relatively s l ightcardiovascular effects . (3) Another ergot drug, bromocriptine, i s used to manage hyperprolactinemia (due to i ts s trong dopaminergic effects ) suchas that which may occur with pi tui tary tumors , and i s a lso sometimes used to manage Parkinson disease.]

Page 162: Pharmacology pre test self assessment and review (14th ed.)

Renal System and Diuretic Pharmacology Aldosterone and i ts antagonis tsAntidiuretic hormone and i ts antagonis tsCarbonic anhydrase and i ts inhibi torsLoop (“high cei l ing” or Na-Cl symporter inhibi tor) diureticsOsmotic diureticsPotass ium-sparing diureticsThiazides (benzothiadiazides) and thiazide-l ike diuretics

Questions

264. After a few weeks on a drug that was prescribed by another phys ician, a patient reports fine tremors of his fingers , headache and fatigue, andtrans ient GI dis tress . More worrisome to him is that he i s constantly thi rs ty and urinates copious ly and frequently throughout the day and night. A24-hour urine col lection produces nearly 5 L of hypotonic urine. Blood tests show that levels of the causative drug are within i ts therapeutic range.Nonetheless , the cl inica l picture leads you to hypothes ize that the offending drug i s caus ing renal responses qui te s imi lar to a syndromecharacterized by reduced production or renal response to ADH. Which drug most l ikely caused or contributed to these s igns and symptoms?

a. Diazepamb. Fluoxetinec. Haloperidold. Li thiume. Phenytoin

265. A patient taking an ora l diuretic for about 6 months presents with elevated fasting and postprandia l blood glucose levels . You check thepatient’s HbA1c and find i t i s elevated compared with normal basel ine va lues obta ined 6 months ago. You suspect the glycemic problems arediuretic-induced. What was the most l ikely cause?

a. Acetazolamideb. Ami loridec. Chlorothiazided. Spi ronolactonee. Triamterene

266. A patient with severe heart fa i lure i s in the ICU. His urine output i s dangerous ly low. You begin an intravenous infus ion of dopamine at ausual therapeutic dose and urine output ri ses quickly and dramatica l ly. What i s the most l ikely mechanism by which the dopamine caused thiseffect?

a. Blocked β-adrenergic receptors in the juxtaglomerular apparatus , thereby inhibi ting renin release and susequent angiotens in-mediatedaldosterone release from the adrenal cortex.

b. Di rectly inhibi ted a renal Na +, K+, 2Cl - cotransporter in the Loop of Henle.c. Improved renal blood flow and glomerular fi l trationd. Lowered the medul lary-to-cortica l osmotic gradient, such that normal urine concentrating mechanisms were impairede. Reduced the permeabi l i ty of the ascending l imb, Loop of Henle, and of the col lecting ducts , to water

267. Usual doses of a thiazide diuretic (eg, hydrochlorothiazide) produce a urine that i s relatively rich in sodium and potass ium, but conta insrelatively l i ttle increase in urine volume (ie, comparatively l i ttle free water loss compared with the relative loss of ions). An action at what s i te inthe nephron mainly accounts for the abi l i ty of the thiazides to cause these responses?

a. Ascending l imb of the loop of Henleb. Cortica l di luting segment of the proximal nephronc. Cortica l -to-medul lary “countercurrent multipl ier”d. Descending l imb of the loop of Henlee. Principa l cel l s of the nephron

268. A patient with essentia l hypertens ion i s being treated with hydro-chlorothiazide and a ca lcium channel blocker, and i s doing wel l . He a lsotakes atorvastatin for hypercholesterolemia, and aspirin to reduce his ri sk of an acute coronary syndrome. He i s now diagnosed with a seizuredisorder. We begin therapy with one of the sui table anticonvulsants that, fortunately, does not a l ter the metabol i sm of any of the medicationsprescribed for his cardiovascular problems. We’ve a lso read that systemic adminis tration of acetazolamide may prove to be a useful adjunct to theanticonvulsant therapy: the metabol ic acidos is i t causes may help suppress seizure development or spread. So, we s tart acetazolamide therapytoo. What i s the most l ikely outcome of adding the acetazolamide?

a. Excess ive ri ses of plasma sodium concentrationb. Hypertens ive cri s i s (antagonism of both antihypertens ive drugs)c. Hypokalemia via synergis tic actions with the thiazided. Spontaneous bleeding (potentiation of aspi rin’s actions)e. Sudden ci rculating volume expans ion, onset of heart fa i lure

269. A patient has very high plasma uric acid levels , has had two acute gout attacks in the las t 8 months , and i s at imminent ri sk of developingacute uric acid nephropathy. We wi l l treat the patient with proper anti -inflammatory drugs and other agents , but feel that reducing solubi l i ty of

Page 163: Pharmacology pre test self assessment and review (14th ed.)

uric acid in the urine might help ward-off the development of renal problems. What drug i s best able to produce this des i red renal effect vi s -á-visurate solubi l i ty without appreciably increas ing systemic ri sks of the hyperuricemia?

a. Acetazolamideb. Antidiuretic hormone (ADH) (vasopress in [VP])c. Ethacrynic acidd. Furosemidee. Hydrochlorothiazide

270. A patient with heart fa i lure has been managed with digoxin and furosemide and i s doing wel l by a l l measures , for 3 years . He develops acuterheumatoid arthri ti s and i s placed on rather large doses of a very efficacious nonsteroida l anti -inflammatory drug—one that inhibi ts both the COX-1 and -2 cyclooxygenase pathways . What i s the most l ikely outcome of adding the NSAID?

a. Hyperchloremic acidos is indicative of acute diuretic toxici tyb. Dramatic increase of furosemide’s potass ium-sparing effectsc. Edema, weight ga in, and other s igns/symptoms indicative of reduced diures isd. Increased digoxin excretione. Reduced digoxin effects because the NSAID competes with digoxin for myocyte receptor-binding s i tes

271. A patient was in a recumbent pos i tion for a 45-minute ora l surgery procedure. When the surgery was completed the patient s tood up quicklyand promptly got l ight-headed and fa inted. The cause was hypotens ion due to hypovolemia from excess ive diures is , attributed to a drugprescribed by her phys ician and taken for severa l months . What drug was the most l ikely cause?

a. Acetazolamideb. Furosemidec. Hydrochlorothiazided. Spi ronolactonee. Triamterene

272. One of your cl inic patients i s being treated with spi ronolactone. Which s tatement correctly describes a property of this drug?

a. Contra indicated in heart fa i lure, especia l ly i f severeb. Inhibi ts Na + reabsorption in the proximal renal tubule of the nephronc. Interferes with a ldosterone synthes isd. Is a rational choice for a patient with an adrenal cortica l tumore. Is genera l ly preferred to a thiazide in most patients with essentia l hypertens ion

273. Chlortha l idone and torsemide are members of di fferent diuretic classes , in terms of mechanisms of action and chemica l s tructure, but theyshare the abi l i ty to cause hypokalemia. Which s tatement best describes the genera l and common mechanism by which these drugs cause theireffects that lead to net renal potass ium loss?

a. Act as a ldosterone receptor agonis ts , thereby favoring K+ lossb. Block proximal tubular ATP-dependent secretory pumps for K+

c. Increase del ivery of Na + to principa l cel l s in the dis ta l nephron, where tubular Na + i s transported into the cel l s via a sodium channel inexchange for K+, which gets el iminated in the urine

d. Stimulate a proximal tubular Na, K-ATPase such that K+ i s actively pumped into the urinee. Lower dis ta l tubular urine osmola l i ty, thereby favoring pass ive di ffus ion of K+ into the urine

274. A 52-year-old man presents to your cl inic for his fi rs t vi s i t wi th you, a fter moving from a dis tant town. His only medications are a s tatin,aspi rin, (81 mg/day) and metolazone. The pharmacis t who fi l led his prescriptions expla ined to the gentleman why he was taking the aspi rin andthe s tatin, but merely referred to the metolazone as a “water pi l l .” Thus , you’re asked about i t. What i s the most l ikely reason why the metolazonewas prescribed?

a. Adjunctive management of an adrenal cortica l tumorb. Adjunctive management of hepatic ci rrhos is from years of excess ive a lcohol consumptionc. Hypertens ion accompanied by a his tory of gout and diabetesd. Treatment of essentia l hypertens ione. Treatment of edema and asci tes from heart fa i lure

275. Urinary potass ium concentrations are measured before and after severa l weeks of adminis tering a loop diuretic (eg, furosemide, at typica lda i ly dosages). We find that post-treatment urine K+ concentrations are substantia l ly lower than those measured at basel ine. What i s the mostl ikely explanation for this observation?

a. An expected response to the drugb. Loop diuretics cause potass ium-wasting only in in vi tro experimenta l modelsc. Measurements of post-treatment urine K+ concentrations were erroneousd. The patient has hypoaldosteronism from bi latera l adrenalectomye. Potass ium secretion by principa l cel l s of the nephron are inhibi ted by loop diuretics

276. You have just completed your thi rd-year medicine clerkship, having spent some time in genera l internal medicine, cardiology, nephrology, andendocrine-metabol i sm cl inics . In each of those venues you have reviewed charts of patients taking eplerenone. What phrase best describes acharacteris tic or other property of this drug?

Page 164: Pharmacology pre test self assessment and review (14th ed.)

a. Faci l i tates renal sodium water loss and potass ium retentionb. Inhibi ts s teroidogenes is (s teroid hormone production)c. Mimics antidiuretic hormone (vasopress in)d. Speci fica l ly inhibi ts a ldosterone synthes ise. Used to counteract glycosuria in patients with type 1 or 2 diabetes mel l i tus

277. A 58-year-old man with a his tory of hypertens ion and hypercholesterolemia i s diagnosed with heart fa i lure. We s tart therapy with furosemideas one of severa l medications . Which of the fol lowing would you expect to occur a long with the increased urine volume caused by this diuretic?

a. Di lute (hypotonic) urine because normal urine concentrating mechanisms are impairedb. Hyperca lcemia due to impaired renal Ca 2+ excretionc. Reduced net excretion of Cl -

d. Metabol ic acidos is due to increased renal bicarbonate excretione. Reduced plasma uric acid (urate) concentrations because of increased urate excretion

278. A patient post-head trauma is in the neurosurgery uni t at your hospi ta l . He has become hypervolemic and hyponatremic from two maincauses : (1) trauma probably increased vasopress in release from his posterior pi tui tary gland; and (2) the s taff was not sufficiently careful whenadminis tering IV fluids , which further increased blood volume and lowered blood sodium concentrations . This patient a lso has symptomatic heartfa i lure that would be further compromised by additional ri ses of blood volume, even i f s l ight or short-l ived. He had been on a number of drugsprior to admiss ion. One was ora l potass ium-supplements to counteract diuretic-induced hypokalemia. Unfortunately, the supplement dose wasexcess ive, and the patient was borderl ine-hyperka lemic too. What drug would be the most rational to adminis ter, with the goal of normal izing orat least improving blood volume and electrolyte compos i tion and without further compromis ing the patient’s hemodynamic s tatus?

a. Conivaptanb. Mannitolc. Metolazoned. Sodium chloride 0.9% (normal sa l ine)e. Spi ronolactone

279. A 60-year-old man has multiple medica l problems, including severe hepatic ci rrhos is from hepati ti s at age 40, and over 30 years of excess ivea lcohol intake. Three days after a vis i t to a phys ician, who prescribed a drug for a condition unrelated to the l iver dys function, the man i s foundcomatose. He i s transported to the emergency department. What drug, prescribed 3 days before, most l ikely induced or contributed to the onsetcoma in this man?

a. Acetazolamideb. Eplerenonec. Furosemided. Hydrochlorothiazidee. Triamterene

280. Your patient, who l ives in Death Val ley, Ca l i fornia (240 feet below sea level ) i s planning a vacation that includes a short hike to the top ofMount Everest (approx. 29,000 feet above sea level ). You’re concerned about “a l ti tude s ickness .” He has no other s igni ficant medica l conditions ,and takes no other drugs that would interact with the drug you wi l l prescribe for his trip. Which drug would you recommend that this adventurerstart taking before his trek, and continue unti l he returns to an a l ti tude much closer to sea level?

a . Acetazolamideb. Ami loridec. Bumetanided. Furosemidee. Sprionolactonef. Triamterene

281. A patient presents with chronic open angle glaucoma. What “renal” drug, or a drug in the same chemica l and pharmacologic class , might beprescribed as an adjunct to lower intraocular pressure and help manage this condition?

a. Acetazolamideb. Ami loridec. Furosemided. Sprionolactonee. Triamterene

282. A patient has unacceptably low cardiac output and intense reflexmediated sympathetic activation of the periphera l vasculature that i sattempting, unsuccess ful ly, to keep vi ta l organ perfus ion pressure sufficiently high. The patient i s edematous because of the poor cardiac functionand renal compensations for i t. Which one of the fol lowing drugs should be avoided in this patient because i t i s most l ikely to compromisefunction of the a l ready-fa i l ing heart and the ci rculatory system overa l l?

a . Ami lorideb. Ethacrynic acidc. Hydrochlorothiazided. Mannitole. Spi ronolactone

Page 165: Pharmacology pre test self assessment and review (14th ed.)

283. A patient i s recently diagnosed with adrenal cortica l adenoma. Among the pertinent Cushingoid s igns and symptoms are hypertens ion andweight ga in from fluid retention; and hypernatremia and hypokalemia. Which drug would be the most rational to prescribe, a lone or adjunctively,to speci fica l ly antagonize both the renal and the systemic effects of the hormone excess?

a. Acetazolamideb. Ami loridec. Furosemided. Metolazonee. Spi ronolactone

284. A patient has been referred to your academic medica l center because of recent-onset ventricular ectopy, second-degree AV nodal block,chromatops ia , and other extracardiac s igns and symptoms of digoxin intoxication. His fami ly doctor, who has been treating him for a host ofcommon medica l problems over the las t 30 years , had prescribed furosemide and digoxin for this gentleman’s heart fa i lure. Blood tests show thatdigoxin levels are wel l wi thin a normal range. We bel ieve the problems are diuretic-induced. What did the diuretic most l ikely do to precipi tatethe digoxin toxici ty?

a. Caused hyperca lcemiab. Caused hypokalemiac. Caused hyponatremiad. Displaced digoxin from ti ssue binding s i tese. Inhibi ted digoxin’s metabol ic el imination

285. A 48-year-old man with bi latera l diabetic nephropathy develops acute heart fa i lure and additional , and s igni ficant, decl ines of renal function(eg, decl ines of GFR) as two of severa l consequences of seps is . You wi l l adminis ter appropriate antibiotics , vasodi lators , and cardiac inotropes ,but a lso need to adminis ter a diuretic to promptly reduce ci rculating fluid volume and “unload” the fa i l ing heart. What drug would be mostappropriate in terms of managing the hemodynamic problems for this patient?

a. Acetazolamideb. Hydrochlorothiazidec. Mannitold. Torsemidee. Metolazone (or chlortha l idone)

286. A patient with severe infectious disease i s being treated with an aminoglycos ide antibiotic. Which diuretic should be avoided, i f poss ible, forthis patient, because of the ri sk of a serious adverse effect shared by both drugs?

a. Acetazolamideb. Furosemidec. Metolazoned. Spi ronolactonee. Triamterene

287. Ami loride i s a useful drug for managing hypokalemia caused by other drugs . Which phrase best describes the mechanism by which i t causesi ts potass ium-sparing effects?

a. Blocks the agonis t effects of a ldosterone with i ts renal tubular receptorsb. Blocks dis ta l tubular sodium channels and, ul timately, Na +-K+ exchangec. Hastens metabol ic inactivation of a ldosteroned. Stimulates a proximal tubular Na, K-ATPasee. Suppresses corti sol and a ldosterone synthes is and release in the adrenal cortex

288. A patient who admits to drinking many l i ters of water each day has had recurrent episodes of symptomatic hyponatremia, and i s at great ri skof recurrences . Because of another medica l problem he now requires adminis tration of a diuretic. Which drug i s most l ikely to precipi tate anotherrecurrence of the hyponatremia, whether or not the patient’s da i ly water intake i s reduced to a more acceptable level?

a . Bumetanideb. Ethacrynic acidc. Furosemided. Hydrochlorothiazidee. Torsemide

289. A hypertens ive patient has been on long-term therapy with l i s inopri l , a long-acting ACE inhibi tor, for hypertens ion. The drug i sn’t control l ingpressure as wel l as wanted, so the phys ician decides to add ami loride. What i s a potentia l , i f not l ikely, outcome of adding this diuretic to the ACEinhibi tor regimen?

a. Blood pressure would ri se abruptlyb. Blood pressure would fa l l aga in once ami loride was added (better BP control ), but there’s a l ikel ihood that the patient would become

hyperka lemicc. Cardiac depress ion, because both drugs di rectly depress heart rate and left ventricular contracti l i tyd. Diabetes ins ipidus-l ike syndrome with production of large volumes of di lute urine, plus di lutional hyponatremiae. Hypokalemia because of the two drugs ’ addi tive or synergestic effects on renal potass ium handl ing

290. Cons ider aga in the patient described in the previous question. He now presents with diuretic-induced hyponatremia. The condition i s

Page 166: Pharmacology pre test self assessment and review (14th ed.)

symptomatic and severe enough that the excess ively low Na + concentration needs to be corrected promptly. In addition to cautious intravenousadminis tration of NaCl , adjunctive drug therapy i s indicated. Which drug or drug class would be the most rational to use adjunctively, in addi tionto IV NaCl and s topping the causative diuretic at least temporari ly, to help correct sodium concentration?

a. Captopri lb. Furosemidec. Spi ronolactoned. Thiazide-l ike diuretic (eg, metolazone)e. Triamterene

291. The table below shows the urinary electrolyte excretion patterns typica l of various prototype diuretics . These are qual i tative changes , and donot reflect the magnitude of the changes . They show whether excretion of an electrolyte (net amount) is increased or decreased; they do not reflect changes inurine concentrations of these substances.

Your answer choices are:

a . Acetazolamideb. Ami loridec. Chlortha l idoned. Furosemidee. Hydrochlorothiazide

Which drug causes effects most s imi lar, i f not identica l , to unknown drug 2, above?

Page 167: Pharmacology pre test self assessment and review (14th ed.)

Renal System and Diuretic Pharmacology

Answers

264. The answer is d. (Brunton, pp 448-450; Katzung, pp 513-517, 1158t.) Li thium treatment (for bipolar i l lness ) frequently causes polyuria and (as aconsequence of excess ive renal fluid loss ) polydips ia (increased thi rs t leading to increased fluid intake). The col lecting ducts of the kidney losethe capaci ty to conserve water via the expected actions antidiuretic hormone (arginine vasopress in). This amounts to drug-induced nephrogenicdiabetes ins ipidus , di lutional hyponatremia, and SIADH: a syndrome of inappropriate ADH secretion that tries , unsuccess ful ly, to l imit or reducefree water loss from the kidneys . Such findings are not associated with diazepam (a ; prototype benzodiazepine anxiolytic); fluoxetine (b; theprototypic SSRI antidepressant); ha loperidol (c; butyrophenone antipsychotic); or phenytoin (e; hydantoin anticonvulsant).

265. The answer is c. (Brunton, pp 686-690; Katzung, pp 260-261.) Thiazides and thiazide-l ike diuretics (eg, chlortha l idone, metolazone) tend to elevateblood glucose levels , impair glucose tolerance, and cause frank hyperglycemia.

Severa l mechanisms have been proposed to expla in the effect: decreased release of insul in from the pancreas ; increased glycogenolys is anddecreased glycogen synthes is ; a reduction in the convers ion of proinsul in to insul in; and reduced respons iveness of adipocyte and skeleta lmyocyte insul in receptor response to the hormone (insul in res is tance—the most l ikely mechanism).

(You might reca l l that diazoxide [mainly used as a parentera l drug for prompt lowering of blood pressure] can be used in i ts ora l dosage form tora ise blood glucose levels in some hypoglycemic s tates . It i s , chemica l ly, a thiazide, but i s not used as a diuretic.)

Elevations of blood glucose levels , or other mani festations of glycemic control , are rarely associated with treatment with acetazolamide (a),ami loride (b), spi ronolactone (d), or triamterene (e).

266. The answer is c. (Brunton, pp 288, 351-359; Katzung, pp 84t, 136f, 138t, 141.) The often dramatic increases in urine output caused by therapeuticdoses of dopamine (so-ca l led “renal doses”) i s mainly due to improved hemodynamics . The drug, under these conditions , not only improvescardiac contracti l i ty and cardiac output (which, in turn, improves renal perfus ion), but a lso di lates the renal arterioles ; these effects lead toimproved glomerular fi l tration, and the increased urine volume. Improved hemodynamics wi l l suppress the renin-angiotens in-a ldosteronesystem, which was activated in response to poor renal perfus ion. Weak β1 receptor activation a lso contributes to this effect (particularly with suchdrugs as dobutamine). However, dopamine does not block (a) any of the adrenergic receptors .

Dopamine a lso has renal tubular effects , much l ike tradi tional diuretics do, a l though we often overlook them (perhaps because they are lessimportant than the effects described above. The drug, via i ts D1-receptor agonis t actions , increases c-AMP formation in the proximal tubules andalso in the thick ascending l imb of the Loop of Henle. At the latter s i te, the resul t i s inhibi tion of both the Na +-H+ exchange mechanism and theNa +, K+-ATPase, but not of water (e; reca l l that the thick ascending l imb of the Loop of Henle i s impermeable to water.) Taken together, the renalhemodynamic and tubular actions of dopamine make i t particularly useful for managing heart fa i lure that i s accompanied by reduced renalfunction.

Higher doses cause pos i tive inotropy via β1-receptor activation, and causes neuronal NE release. High doses activate α1 receptors in variousvascular beds , including the renal , leading to vasoconstriction that can ra ise blood pressure and tota l periphera l res is tance, and can reduce renalblood flow, thereby reducing renal excretory function.

267. The answer is b. (Brunton, pp 686-690; Katzung, pp 251-255, 260-261.) Thiazides typica l ly produce a urine that i s concentrated (hyper-osmola l , moreconcentrated than normal ) because the urine conta ins a relative excess of sodium and potass ium (and such anions as chloride and, to a lesserextent, bicarbonate), and l i ttle in the way of extra free water loss . The main s i te at which thiazides and thiazide-l ike diuretics (eg, chlortha l idone,metolazone) act to cause this effect i s in the cortica l di luting segment. Contrast this with the loop diuretics (furosemide, torsemide, bumetanide,and ethacrynic acid), which cause the formation of a di lute urine because they inhibi t reabsorption of sodium in the descending l imb of the Loopof Henle (a), thereby inhibi ting the formation of the medul lary-to-cortica l osmotic gradient (c, d) necessary for the formation of a normal lyconcentrated urine. None of the common diuretic drugs act on the ascending l imb of the Loop of Henle (a). Al l the potass ium-wasting diuretics ,that i s , the thiazides , thiazide-l ike agents , and loop agents , such as furosemide, indi rectly act on the principa l cel l s of the nephron (e). Byinhibi ting a portion of sodium reabsorption more proximal ly they del iver extra sodium dis ta l ly to the principa l cel l s . There, the principa l cel l s takeup (reabsorb) an additional fraction of sodium in exchange for potass ium, which i s los t into the urine. This accounts for the potass ium-wastingeffects of these diuretics , but does not account for whether they produce a concentrated or a di lute urine.

268. The answer is c. (Brunton, pp 677-681; Katzung, pp 256-257, 261-262, 265.) We seldom adminis ter acetazolamide as a diuretic, because i ts effectsare “mi ld”; associated with s igni ficant changes of both urine pH (up) and blood pH (down; metabol ic acidos is ); and sel f-l imiting (once sufficientbicarbonate has been lost from the blood, into the urine, refractoriness to further diures is occurs ). More often we adminis ter acetazolamide andother carbonic anhydrase inhibi tors for nonrenal/noncardiovascular problems, such as to lower intraocular pressure in some cases of glaucoma(carbonic anhydrase inhibi tors inhibi t aqueous humor formation) or as an adjunct to anticonvulsant therapy as described here. As a resul t, we mayforget that these systemica l ly adminis tered drugs are diuretics , one common property of a l l the diuretics being increased renal sodium loss (anatriuretic effect; thus , answer a i s not correct). We may even forget that carbonic anhydrase inhibi tors , given systemica l ly, are potass ium-wastingdiuretics : they act proximal ly and del iver extra sodium dis ta l ly where, at the principa l cel l s of the nephron, some extra Na + i s taken up in exchangefor additional K+ that gets el iminated in the urine.

In this scenario the patient i s taking a thiazide, which i s obvious ly potass ium-wasting and has the potentia l in i ts own right to causehypokalemia. Add a carbonic anhydrase to the regimen and the ri sks of hypokalemia increase. Acetazolamide does not antagonize theantihypertens ive effects of thiazides or ca lcium channel blockers , nor provoke hypertens ion or a hypertens ive cri s i s (b). If there were anyinteractions between the acetazolamide and the aspi rin, i t would be antagonism, not potentiation (d) of aspi rin’s antiplatelet effects . Aspirinundergoes renal tubular reabsorption, and that i s a pH-dependent effect. Aspirin’s reabsorption i s reduced (that i s , i ts excretion increases) in analka l ine urine, which i s precisely what occurs with acetazolamide. (You should reca l l that a lka l inizing the urine i s an important adjunctivemeasure in treating severe sa l icylate poisoning, in part because i t reduces tubular reabsorption of sa l icylate.) There i s no reason to suspectsudden ri ses of blood volume, with or without concomitant heart fa i lure from that (e). Indeed, the added diures is from the acetazol -amide may, atleast trans iently, potentiate the effects of the thiazide on urine volume, blood pressure, or both.

269. The answer is a. (Brunton, pp 677-681; Katzung, pp 256-257.) Here I asked about reducing the ri sk of urate nephropathy acutely by increas ing uricacid solubi l i ty, through urinary a lka l inization. One key to answering this question correctly i s to rea l i ze that uric acid becomes more soluble (less

Page 168: Pharmacology pre test self assessment and review (14th ed.)

l i kely to precipi tate or crysta l l i ze) as loca l pH ri ses . Reca l l that normal urine i s acidic. We want to a lka l inize the urine, and that i s precisely whatacetazolamide does , by inhibi ting carbonic anhydrase in the proximal nephron. Note that acetazolamide i s only an adjunct, and in addition to (orinstead of) us ing i t, we might a lso adminis ter sodium bicarbonate, which wi l l a lka l inize the urine; and keep the patient wel l hydrated to helpform large amounts of a di lute urine. Acetazolamide does cause a metabol ic acidos is , which would seemingly favor reductions of uric acidsolubi l i ty in the blood. However, we are keeping our patient wel l hydrated (helps reduce precipi tation); and the volume of blood i s far greaterthan urine volume at any given time, and given the great s i ze of the “blood pool” we have l i ttle to worry about in terms of urate precipi tationsystemica l ly.

Antidiuretic hormone (b) would be i l logica l . It would reduce urine volume and concentrate solutes (such as uric acid) in i t. Ethacrynic acid (c)and furosemide (d) lead to the formation of copious volumes of di lute urine. In terms of renal problems, that may be beneficia l . However, theloop diuretics tend to cause such large amounts of fluid loss via the urine that the concentration of solutes (including uric acid) in the blood maygo up.

Hydrochlorothiazide (e) and related drugs tend to form a very concentrated urine by interfering with normal urine-di luting mechanisms in thekidneys . Whi le some of these drugs have carbonic anhydrase activi ty, and a lka l inize the urine, these effects are weak in comparison withacetazolamide. The most l ikely predominant renal effect i s the unwanted one: increased ri sk of urate nephropathy. In addition, thiazides tend toelevate urate levels (by interfering with tubular secretion of urate), and so these drugs are l ikely to pose additional systemic problems.

270. The answer is c. (Brunton, pp 682-684; Katzung, pp 258-260.) An important element in the renal responses to furosemide i s maintenance ofadequate renal blood flow. That i s , to a degree, prostaglandin-mediated. Prostaglandins di late the afferent arteriole (to the glomerul i ) andincrease GFR and urine production.

The NSAIDs , such as the hypothetica l one described here, inhibi t prostaglandin synthes is . That, in turn, antagonizes the des i red effects of theloop diuretic, leading to less fluid and sa l t el imination: edema, weight ga in, and other markers of heart fa i lure are l ikely to develop as a resul t.Hyperchloremic a lka los is (a ) i s incorrect: chronic or acute excess ive effects of loop diuretics are characterized by hypochloremic metabol ica lka los is . Regardless , NSAIDs are not l ikely to potentiate the effects of these diuretics . “Dramatic increases of furosemide’s potass ium-sparingeffects (b)” i s incorrect. Reca l l that loop diuretics are potass ium-wasting. Digoxin i s el iminated by renal excretion. If we accept the notion thatloop diuretics may increase excretion of digoxin, then we should accept the l ikely poss ibi l i ty that NSAID-induced reductions of diuretic actionshould reduce the glycos ide’s renal loss , not increase i t (d). The NSAIDs do not bind to and inhibi t the myocyte Na +, K+-ATPase, which i s digoxin’scel lular receptor (e). Do remember that furosemide (and thiazides) are apt to increase the ri sk or severi ty of digoxin toxici ty. The mechanismmainly involves diuretic-induced hypokalemia, not changes in ci rculating fluid volume or urine volume per se.

271. The answer is b. (Brunton, pp 682-686; Katzung, pp 258-260.) One way to s impl i fy answering this question i s merely to ask “which diuretic has thegreatest abi l i ty to cause hypovolemia?” That narrows the choice to furosemide or the other loop diuretics (bumetanide, torsemide, ethacrynicacid). In terms of extra free water loss (and the concomitant ri sk of hypovolemia) the maximal efficacy of acetazolamide (carbonic anhydraseinhibi tor; a ) i s modest at best, and sel f-l imiting to boot. Hydrochlorothiazide (c) and the two potass ium-sparing diuretics l i s ted (spi ronolactoneand triamterene; d, e) a lso have modest efficacy in terms of the peak diuretic effect, even i f unusual ly large doses were to be given. Note:Hyponatremia reduces the respons iveness of the periphera l vasculature to vasoconstrictors (eg, EPI, NE, and angiotens in I I). If I s tated that thepatient’s hypotens ion were due to diuretic-induced hyponatremia, then the most l ikely correct answer would be hydrochlorothiazide or anotherthiazide or thiazide-l ike diuretic (eg, metolazone); of a l l the diuretics classes (and most other classes of drugs), they are the most common causeof hyponatremia.

272. The answer is d. (Brunton, pp 690-693, 1841; Katzung, pp 261-263, 711, 739.) Spi ronolactone i s a potass ium-sparing diuretic. Its active metabol i teblocks a ldosterone receptors in the dis ta l nephron (thus answer b i s incorrect). Nei ther spi ronolactone nor i ts active metabol i te a l ter a ldosteronesynthes is (c). The drug i s ineffective in the absence of a ldosterone. Reca l l that a ldosterone normal ly causes renal Na + retention and K+ loss . Theeffects of a ldosterone are qual i tatively the oppos i te: Na + loss , K+ retention.

Owing to the abi l i ty of spi ronolactone to counteract the effects of a ldosterone, i t i s particularly sui ted for patients with primary or secondaryhypera ldosteronism (eg, adrenal cortica l tumor or hepatic dys function, as might occur with long-term/high-dose a lcohol consumption,respectively). There i s growing evidence that the drug i s beneficia l in heart fa i lure and probably reduces morbidi ty in severe heart fa i lure (and soa i s incorrect). Al though spi ronolactone has antihypertens ive effects , i t i s not cons idered to be a fi rs t-l ine choice (e) for most patients withessentia l hypertens ion. For those patients a thiazide (most often), an ACE inhibi tor, a β-blocker, or a ca lcium channel blocker i s usual ly chosenfi rs t.

In addition to the potentia l for caus ing hyperka lemia (especia l ly i f combined with ora l potass ium supplements , which should not be done) andhyponatremia (overa l l ri sk i s low i f spi ronolactone i s the only diuretic used), spi ronolactone may cause severa l other s ide effects . CNS s ide effectsinclude lethargy, headache, drows iness , and menta l confus ion. Other s ide effects that are fa i rly common arise from the drug’s androgen receptor-blocking actions : gynecomastia (in men and women) and erecti le dys function. It may a lso cause seborrhea, acne, and coarsening of body ha i r.(Paradoxica l ly, the drug can cause hi rsutism in some patients , but i t i s a l so used to manage hi rsutism in others .)

273. The answer is c. (Brunton, pp 676-677, 682-686; Katzung, pp 251-256, 259-261.) Thiazides (and thiazide-l ike agents such as chlortha l idone andmetolazone) and loop diuretics (furosemide, bumetanide, torsemide, and ethacrynic acid) increase del ivery of Na + to the dis ta l nephron becausethey inhibi t reabsorption of Na + a t more proximal s i tes . This extra Na + reaches the principa l cel l s in the dis ta l nephron, and some of i t i s takenfrom the tubular fluid via sodium channels . This reclamation of Na + leads to exchange of K+, which i s los t into the urine (ie, potass ium “wasting”).In essence, the more Na + del ivered (and recovered) dis ta l ly, the more K+ that i s el iminated in exchange.

The processes by which these potass ium-sparing drugs work do not involve proximal tubular ATP-dependent potass ium secretion (b), nor doesi t involve any di rect effect on proximal tubular ATPase (d). In addition, there are no agonis t effects on a ldosterone receptors (a)—an action thatwould cause an antidiuretic effect and renal potass ium loss. Indeed, no diuretic acts as an a ldosterone receptor agonis t; spi ronolactone andeplere-none (not l i s ted here) exert thei r natriuretic and potass ium-sparing effects by blocking a ldosterone receptors . Fina l ly, lowering dis ta l tubularosmola l i ty (e) does not occur, nor would such an effect (i f i t occurred) favor pass ive di ffus ion of K+ into the urine.

274. The answer is d. (Brunton, pp 688t, 690-692, 776-777; Katzung, pp 260-261, 267.) Metolazone, a l though not a thiazide (benzothiadiazide) in thechemica l sense, i s largely (hydrochloro)thiazide-l ike in terms of i ts pharmacologic properties and uses , a l though i t has a much longer duration ofaction than the prototype, hydrochlorothiazide. As far as the “main use” of thiazides goes , that would be essentia l hypertens ion (assuming nocontra indications), wi th lesser uses being “mi ld” and especia l ly trans ient edema, management of idiopathic hyperca lciuria , adjunctivemanagement of nephrogenic diabetes ins ipidus , and perhaps management of Meniere disease. Thiazides (or such thiazide-l ike drugs asmetolazone) would not be rational , nor very efficacious , for drug therapy of an adrenal cortica l tumor (a). In that instance, i f we are cons ideringonly diuretics , the proper drug would be spi ronolactone, the a ldosterone receptor blocker. Likewise, s ince thiazides can ra ise ci rculating urate and

Page 169: Pharmacology pre test self assessment and review (14th ed.)

glucose levels , they would not be good choices for the patient with hepatic ci rrhos is or other diseases characterized by poor l iver function (b; heretoo spi ronolactone would be the best choice) or gout or diabetes (e). If our goal were to manage severe edema (e) with or without asci tes (as in apatient with heart fa i lure), our best choice would be a loop diuretic.

275. The answer is a. (Brunton, pp 682-686; Katzung, pp 252f, 252t, 258-260.) One expected response to therapeutic doses of loop diuretics , which areclearly and correctly class i fied as potass ium-wasting, i s a reduction of urinary potass ium concentrations. How can this be? Note that the termconcentration reflects the amount of a substance (here, potass ium) per unit volume; i t i s not equiva lent to “amount” per se. The loop diuretics doincrease K+ excretion: because they work more proximal ly, they provide an added load of Na + del ivered to the dis ta l nephron, where sodiumchannels in the principa l cel l s take up the Na + in exchange for K+. Potass ium secretion by the principa l cel l s i s not inhibi ted (e). Loop diureticsa lso impair the abi l i ty of the kidneys to form a concentrated urine and so promote formation of a large volume of more di lute (= lessconcentrated) urine. The net loss of K+ (say, on a 24-h bas is ) i s increased, but i t’s accompanied by a disproportionate increase in free water losssuch that urine K+ concentration (but not tota l amount lost) i s decreased.

276. The answer is a. (Brunton, pp 693t, 695; Katzung, pp 225, 270, 697, 708, 711.) Eplerenone i s a relatively new spi ronolactone-l ike drug. As such, i tsmain mechanism of action involves blockade of a ldosterone receptors (eg, in the nephron). Its main uses are a lso largely identica l to those ofspi ronolactone: heart fa i lure, hypertens ion (mainly as a potass ium-sparing adjunct, and especia l ly when plasma renin levels are elevated, asthat leads to increased angiotens in I I synthes is which, in turn, increases a ldosterone release from the adrenal cortex); and in patients in whomaldosterone levels are high from other causes (eg, adrenal cortica l tumor, Cushing disease). The drug has no effect on the synthes is ofa ldosterone (d) or other s teroid hormones (b); i ts ul timate renal effects are the oppos i te of those of ADH (at least in terms of free water loss orconservation), so c. i s incorrect. Fina l ly, the drug has no di rect or cl inica l ly useful effects on glucose metabol i sm or levels , nor on glycosuria (e) andother consequences of hyperglycemia.

277. The answer is a. (Brunton, pp 677f, 682-686; Katzung, pp 251-255, 258-260.) Reca l l one of the main mechanisms by which a concentrated urine i sformed: a hypertonic mi l ieu in the renal medul la—and a medul lary-to-cortica l osmotic gradient—osmotica l ly withdraws water (but not solute)from the tubular fluid as i t passes through the col lecting ducts . What creates that hypertonic medul lary-to-cortica l gradient? Reabsorption of Na +

and Cl - as tubular fluid ascends the loop of Henle, which ul timately increases inters ti tia l osmola l i ty. However, that process of ion resorption i simpaired by loop diuretics . That reduces osmola l i ty in the medul lary inters ti tium, thereby dramatica l ly reducing the osmotic gradient that enablesthe tubular fluid to become hypertonic as water i s los t in the dis ta l nephron. Thus , in the presence of a loop diuretic the urine remains di lute andhypotonic.

Hyperca lcemia (b) i s not an expected accompaniment. Loop diuretics (in contrast with thiazides) increase renal Ca 2+ el imination.Reca l l that the main anion excreted (a long with Na +, K+, etc) in response to a loop diuretic i s chloride, and so net Cl - excretion goes up (not

down; c). Metabol ic acidos is from increased bicarbonate excretion (d) doesn’t occur. Bicarbonate tends to be reabsorbed, therefore. You shouldreca l l that hypochloremic a lka los is (a lso ca l led contraction a lka los is , because blood volume contracts as the resul t of excess ive fluid loss in theurine) i s one potentia l and qui te dangerous adverse responses caused by loop diuretics .

Loop diuretics do not lower urate concentrations (e). Rather, urate levels tend to ri se, in part, because of reduced urate excretion combined witha “concentration” of urate in the blood owing to increased free water loss via the urine. Blood uric acid concentration tends to ri se. The loopdiuretics reduce urate excretion through a di rect renal tubular action. More important, perhaps , the proportional ly large extra free water lossshrinks blood volume and tends to increase solute concentration, independent of any renal tubular effects on urate el imination.

278. The answer is a. (Brunton, pp 714, 716t-717; Katzung, pp 264, 270, 675, 678.) This patient probably has SIADH—a syndrome of inappropriatevasopress in (antidiuretic hormone; ADH) secretion—and the scenario described above (hypervolemia, di lutional hyponatremia) i s a fa i rly commoncause or accompaniment of i t. Conivaptan, a relatively new drug, i s indicated for managing SIADH. It i s a nonpeptide and competi tive antagonis t ofADH’s water-sparing effects in the nephron’s col lecting duct.

Why not mannitol (b)? Its anatomic s i te of renal action i s the same as conivaptan’s , but i t increases free water loss through an osmotic, ADH-independent, mechanism. (In fact, a compensatory response to counteract mannitol ’s effects and conserve water i s increased ADH release, butmannitol ’s effects typica l ly “win out” and diures is indeed occurs .) Reca l l that the osmotic mechanism by which mannitol causes diures is a lsocontributes to increased plasma osmola l i ty and blood volume—at least ini tia l ly and unti l sufficient mannitol has been excreted. Heart fa i lure(which this patient had) i s a contra indication, or at least an important precaution, to mannitol ’s use because even a trans ient increase ofci rculating fluid volume, which trans iently increases blood pressure, can provide a s igni ficant added hemodynamic load on the fa i l ing heart. (Therisk would be even greater i f the patient had poor renal blood flow or function that would s low mannitol ’s el imination and keep blood osmola l i tyhigh longer.)

Metolazone (c) i s a thiazide-l ike diuretic. Reca l l that, as a class , thiazides or related agents have only a modest (actua l ly s l ight) abi l i ty toreduce ci rculating fluid volume (which would be des i rable in SIADH). More importantly, thiazides and thiazide-l ike agents tend to causehyponatremia; that’s something we don’t want to cause or, in this case, worsen. Infus ing 0.9% NaCl (d) wi l l not correct sodium concentrations andi f too much i s infused too quickly may put additional fluid-overload-related hemodynamic s tresses on the fa i l ing heart. Spi ronolactone (e) i si rrational . Fi rs t, i t wi l l not correct the excess ive free water loss . More important, by antagonizing the renal tubular effects of a ldosterone i t wi l lincrease sodium loss and increase renal potass ium retention—precisely the oppos i te of what we want to do with this patient.

279. The answer is a. (Brunton, pp 85, 677-681, 1059; Katzung, pp 251-258.) Acetazolamide, the prototypic carbonic anhydrase inhibi tor, causes metabol icacidos is as i t increases renal bicarbonate excretion, which in turn a lka l inizes the urine. Al though acetazolamide i s infrequently used as adiuretic, i t i s used more for other conditions including a l ti tude s ickness , an adjunct to certa in antiepi leptic drugs , and (perhaps in this case) formanagement of glaucoma.

Patients with severe l iver dys function cannot synthes ize urea adequately. As a resul t, el imination of a variety of endogenous ni trogen-basedmetabol ic waste products becomes dependent on renal excretion of ammonium ion. In normal ly acidic urine the ammonium ions are el iminatedrather efficiently. However, and due to the acetazolamide’s urinary-a lka l inizing effects , ammonia gas forms in the urine. It then readi ly di ffusesfrom the urine to the ci rculation. This causes hyperammonemia, which i s the most l ikely cause of the coma.

Eplerenone (b) and the older related drug, spi ronolactone, tend not to a l ter blood or urine pH. Indeed, s ince patients with severe l iverdysfunction (such as this patient, with hepati ti s - and a lcohol -induced hepatic ci rrhos is ) tend to have high ci rculating a ldosterone levels , theblockade of a ldosterone receptors by eplerenone would probably be beneficia l for this patient, and not at a l l l i kely to account for the coma andrelated neurologic abnormal i ties .

Furosemide (c) a lso i s an unl ikely explanation. It i s l i kely to faci l i tate ammonium excretion, and pH-related changes (eg, urinary a lka l inization)are not apt to occur. Reca l l that one mani festation of loop diuretic excess i s hypochloremic metabol ic a lka los is (a lso ca l led contraction a lka los is

Page 170: Pharmacology pre test self assessment and review (14th ed.)

because ci rculating fluid volume shrinks—contracts ) in response to the very efficacious loop agents . Because the kidneys tend to lose largeamounts of chloride, and tend to reta in bicarbonate, urinary a lka l inization that leads to ammonia production in the urine i s not l ikely.

Some of the thiazides (eg, hydrochlorothiazide, d) and thiazide-l ike agents (eg, chlortha l idone, metolazone) have carbonic anhydrase activi tythat could lead to a lka l inization of the urine. However, this property i s genera l ly weak in comparison with acetazolamide or related agents thatare more appropriately class i fied as carbonic anhydrase inhibi tors . Triamterene (e, or the related drug ami loride), a potass ium-sparing diureticwith actions that are a ldosterone-independent (contrast with eplerenone, spi ronolac-tone), i s not l ikely to a l ter urine pH or cause other effectsthat might precipi tate coma in this patient.

280. The answer is a. (Brunton, pp 677-681, 1059; Katzung, pp 256-258.) The s igns and symptoms of a l ti tude s ickness are related to the development ofrespiratory a lka los is : At high a l ti tudes the partia l pressure of O2 in the inspi red a i r i s low. The hypoxia triggers hyperventi lation, which increasesnet venti latory loss of CO2. Blood pH ri ses . Acetazolamide effectively inhibi ts carbonic anhydrase and so increases bicarbonate loss . This , in turn,causes a metabol ic acidos is that counteracts the venti latory-induced ri se of blood pH. It i s the drug of choice for prophylaxis or management ofa l ti tude s ickness . It i s true that some of the thiazide or thiazide-l ike diuretics inhibi t carbonic anhydrase, but the effect i s very weak incomparison to that of acetazolamide. None of the other drugs l i s ted would be sui table for this patient and the a l ti tude s ickness he i s l ikely toexperience.

281. The answer is a. (Brunton, pp 680, 1775, 1787; Katzung, p 257.) Aqueous humor formation (as wel l as that of cerebrospina l fluid) involves carbonicanhydrase activi ty. Reduce aqueous humor synthes is and, a l l other factors being equal , intraocular pressure goes down. Acetazolamide i s , ofcourse, “the” carbonic anhydrase-inhibi ting diuretic.

Note that when a carbonic anhydrase inhibi tor i s used to manage glaucoma, the drug’s renal/diuretic effects , which certa inly occur i f the drug i sgiven systemica l ly, have nothing to do with the beneficia l effects that are due to an ocular (ci l iary body) s i te of action. If fact, those extraoculareffects aren’t needed for glaucoma control . That i s why, when we prescribe a carbonic anhydrase inhibi tor for glaucoma, we usual ly choose atopica l ophthalmic carbonic anhydrase inhibi tor (eg, dorzolamide or methazolamide).

282. The answer is d. (Brunton, pp 681-682; Katzung, pp 251-271.) Mannitol i s an osmotic diuretic, the prototype of that smal l class of drugs(glycerin/glycerol i s another somewhat noteworthy member of the group) with indications and potentia l s ide effects that are qui te di fferent fromthose of “typica l” diuretics l ike the thiazides or loop agents . What happens when you inject (intravenous ly) this nonmetabol i zable sugar, whichhas a s tructure s imi lar to that of glucose and the same molecular weight? Ini tia l ly, and unti l i t i s excreted by glomerular fi l tration, mannitolincreases plasma osmola l i ty. That, in turn, osmotica l ly withdraws water from the extracel lular space and, ul timately, from the parenchymal cel l s ,and into the blood. If the patient has good renal function, renal blood flow and GFR ri se, and the drug i s eventual ly excreted. If he or she hasadequate cardiac function, ci rculating that extra volume (up to a l imit) i s not a problem.

However, i f the patient has a sufficiently contracti l i ty or cardiac output to begin with, or renal perfus ion i s compromised, the increased bloodvolume and pressure may be such that the heart s imply cannot handle the added workload. Indeed, in a futi le attempt to ci rculate that additionalvolume and eject i t aga inst a higher afterload, the heart may fa i l acutely.

Ami loride (a), a potass ium-wasting diuretic with relatively low efficacy in terms of i ts abi l i ty to increase urine volume, should have no adverseeffects in this patient. The same appl ies to hydrochlorothiazide (c), which i s potass ium-wasting. Ethacrynic acid (b) i s a loop diuretic. It i s one ofsevera l loop agents we might choose to help “unload” this patient’s ventricles , provided the patient i sn’t hypovolemic a l ready. Spi ronolactone (e),a potass ium-sparing a ldosterone antagonis t, might ul timately be prescribed for this patient, particularly i f his heart fa i lure worsens .Nonetheless , at this time i t i s not a drug we should avoid.

283. The answer is e. (Brunton, pp 692-793, 693t, 1841; Katzung, pp 261-263, 711, 739.) The most rational choice for speci fica l ly antagonizing theconsequences of hormone excess—aldosterone being the hormone of most importance in terms of the renal , hemodynamic, and electrolyteproblems here—is spi ronolactone (or eplerenone). Its primary mechanism of action i s blockade of a ldosterone receptors . Remember that themain renal effects of a ldosterone are sodium (and water) retention, and increased renal loss of potass ium.

Acetazolamide (a), the carbonic anhydrase inhibi tor, exerts weak and sel f-l imiting diuretic and natriuretic effects . It i s a l so potass ium-wasting,and increased excretion of potass ium is what we do not want to do in s i tuations of preexis ting hypokalemia. Ami loride (b) would help increasesodium excretion and help normal ize potass ium levels too. However, i t does not provide the “rational” approach to hypera ldosteronism becausei t does not block a ldosterone receptors . Furosemide (c) or another loop diuretic might prove effective in terms of increas ing excretion of sodiumand reducing that of potass ium. Indeed, i f the hypera ldosteronism is sufficiently great to cause dangerous degrees of fluid retention(hypervolemia) and heart fa i lure we may need to adminis ter i t. Nonetheless , spi ronolac-tone would be a reasonable, rational , and probablynecessary element of management. Metolazone (d), a thiazide-l ike diuretic, would des i rably help increase renal sodium excretion and lowerblood pressure. Having relatively low efficacy in terms of increased urine production, i t may not do much to manage the hypervolemia and theresul ting weight ga in. More important, metolazone i s l ikely to aggravate the preexis ting hypokalemia because i t i s , of course, a potass ium-wasting diuretic.

284. The answer is b. (Brunton, pp 801-804, 837-839; Katzung, pp 251-255, 258-260.) Whi le many ionic and other factors can predispose a patient todigoxin toxici ty, hypokalemia, as can be caused by a loop diuretic (usual ly used for edema and/or asci tes , including that associated with heartfa i lure) or thiazide diuretic (mainly used for treating essentia l hypertens ion), i s arguably the most important, i f not the most common. There i s acompeti tion between extracel lular K+ and digoxin for binding to digoxin’s cel lular receptor, the Na +, K+-ATPase on the sarcolemma. Whencirculating (extracel lular) K+ concentration i s reduced (ie, when hypokalemia develops), the binding of digoxin i s enhanced and so i ts effects areincreased, even without a ri se of digoxin concentrations , and usual ly those increased effects are deleterious , as described here. Hyperca lcemiacan increase the ri sk or severi ty of digoxin toxici ty. However, loop diuretics increase renal Ca 2+ loss , and so hyperka lemia (a) i s not a l ikelyexplanation. Hyponatremia (c) i s not at a l l l i kely to have occurred with a loop diuretic, which tends to increase both renal Na + and H2O loss , withthe loss of free water being much greater than the loss of Na +. Loop diuretics do not displace digoxin from ti ssue binding s i tes (d) or inhibi tdigoxin’s el imination (which involves renal excretion, not metabol i sm).

Note that in the scenario I described the patient as having been treated by “his fami ly doctor who has been treating him for the las t 30 years .”This i s not at a l l meant to impugn fami ly doctors or genera l practi tioners . The point i s that nowadays , and for a variety of reasons (toxici ty beingone), digoxin i s no longer cons idered an appropriate drug for managing heart fa i lure, with or without s igns or symptoms of congestive fa i lure,unti l a l l other reasonable and more sui table options have been tried and the patient’s condition has deteriorated to the point that inotropes(digoxin or parentera l agents ) are warranted. Reca l l that our preferred approach for ini tia l therapy of fa i lure involves ACE inhibi tors , a β-blocker,and perhaps a loop diuretic. Nonetheless , and unfortunately, some phys icians s ti l l use the “old way,” involving digoxin. It i s these patients whoare l ikely to be seen in the hospi ta l when the a lmost inevi table problems due to digoxin develop.

Page 171: Pharmacology pre test self assessment and review (14th ed.)

285. The answer is d. (Brunton, pp 682-686; Katzung, pp 258-260.) This patient needs a diuretic that has a fast onset of action the abi l i ty to rathers igni ficantly increase renal excretion of free water to “unload” the heart. Torsemide i s very s imi lar in terms of actions and uses to the prototypeloop or “high cei l ing” diuretic, furosemide, which inhibi ts a Na +/K+/2Cl - transporter in the ascending l imb of the Loop of Henle. (Note that bothfurosemide and torsemide end in “-Semite”—a tip for name recognition.)

Acetazolamide (a), the prototype carbonic anhydrase inhibi tor, would be unsui table. Its onset of action i s s low; i ts peak effects in terms ofincreas ing free water el imination are low; and i ts abi l i ty to cause metabol ic acidos is (secondary to increased renal bicarbonate loss ) would notonly compromise the patient described in the scenario but a lso cause loss of the drug’s diuretic effects . Hydrochlorothiazide (b) and thiazide-l ikediuretics (eg, metolazone and chlortha l idone, e) would not be sui table; thei r efficacies are low in terms of managing edema; thei r effects ares low to develop; and only one thiazide, chlorothiazide, can be given parentera l ly (a point I do not think you need to commit to memory). Mannitol(c), the prototype osmotic diuretic, i s given parentera l ly; i t has a relatively prompt onset of action; and i t can s igni ficantly increase fluidel imination via the kidneys . Mannitol ’s intens i ty and duration of effects , however, depend on many variables , including basel ine renal function.Nonetheless , mannitol ini tia l ly ra ises ci rculating fluid volume—increas ing the hemodynamic load on the heart and being particularly problematicin the face of heart fa i lure—and i ts adverse volume-related effects wi l l be prolonged i f GFR i s reduced in patients such as the one described inthe question.

286. The answer is b. (Brunton, pp 682-686; Katzung, p 260.) Both the loop diuretics and the aminoglycos ides (tobramycin, s treptomycin, gentamicin,others ) are ototoxic—capable of caus ing vestibular damage (eg, ba lance problems), cochlear damage (tinni tus or sensorineura l hearing loss ), orboth. The ototoxic effects of each drug i s enhanced (often s igni ficantly) by the other’s , and so i t i s best to avoid use of both these drugs (or otherototoxins ) unless the benefi ts clearly outweigh the ri sk of perhaps permanent and tota l hearing loss . (Of course, there are instances in which suchcombinations are necessary.) There’s s trong evidence that of a l l the loop diuretics , the ri sk of ototoxici ty i s highest with ethacrynic acid (and i tsparentera l formulation, sodium ethacrynate). Nonetheless , a l l the other loop diuretics—bumetanide, furosemide, and torsemide—are defini telyon the short l i s t of ototoxic drugs .

Acetazolamide (a), metolazone (c) or other thiazide or thiazide-l ike diuretics , spi ronolactone (d), and triamterene (e) are not ototoxic, nor dothey potentiate the adverse auditory effects of other ototoxic drugs .

287. The answer is b. (Brunton, pp 690-692; Katzung, pp 261-263.) Ami loride and triamterene block dis ta l tubular Na + channels that provide for Na +

reabsorption in exchange for K+, which i s los t into the urine. This sodium channel , which i s the main s i te of action of ami loride, i s on the luminals ide of the principa l cel l membranes . (Principa l cel l s in the nephron are located in the late dis ta l tubule and col lecting ducts .) There i s an ATPaseon the inters ti tia l face of the principa l cel l s (aga in, these are in the dis ta l nephron, not proximal ; d). The ATPase i s respons ible for the extrus ionof potass ium into the urine in exchange for extra sodium taken up by the luminal sodium channels , and so i t i s essentia l in the overa l l actions ofpotass ium-sparing diuretics . Nonetheless , the ATPase i s not inhibi ted (or s timulated; d) di rectly by ami loride.

Ami loride has no di rect effects on a ldosterone synthes is or metabol i sm (c), or on a ldosterone receptors (a ; those receptors are blocked byspironolactone).

288. The answer is d. (Brunton, pp 686-690; Katzung, pp 252, 260-261, 268.) Thiazide and thiazide-l ike diuretics should be high (highest!) on your l i s t ofdrugs that are l ikely to cause hyponatremia. Was i t easy to select hydrochlorothiazide as the correct answer because i t was the only drug l i s tedthat was not a loop diuretic? Were you puzzled because you knew that furosemide i s the prototype loop agent, and weren’t sure about theclass i fi cation bumetanide, ethacrynic acid, or torsemide. Or did you not select the thiazide because you cons idered i ts diuretic effects to be mi ld,or modest, and therefore least l ikely to cause hyponatremia?

In absolute or relative terms the natriuretic (sodium-wasting) effects can be cons idered weak (especia l ly in comparison with loop diuretics ).They cause relatively s l ight increases in free water loss in the urine (compared with normal ), interfere with normal urine-di luting mechanisms,and lead to the formation of a concentrated (in terms of such solutes as sodium) urine. That i s , the amount of extra sodium lost i s , in a relativesense, much greater than the extra volume lost. This i s reflected, in the blood, as not much extra volume loss coupled with proportional ly greaterdecl ines in the amount of sodium. Remember that hyponatremia means excess ively reduced sodium concentration, and concentration i s a functionof amount per uni t volume. The thiazides have caused “di lutional hyponatremia.”

289. The answer is b. (Brunton, pp 677-701; Katzung, pp 172, 184-185, 219-220, 295-300.) Li s inoprol (a “-pri l ,” and therefore an angiotens in-convertingenzyme inhibi tor) indi rectly reduces a ldosterone release from the adrenal cortex by inhibi ting angiotens in I I synthes is . (Remember: A-II i s themajor phys iologic s timulus for a ldosterone release.) Normal ly a ldosterone causes renal Na + retention and K+ loss . When there i s lessa ldosterone, the kidneys wi l l now tend to lose Na + and reta in K+. Ami loride (l ike triamterene) blocks sodium channels on the luminal face of theprincipa l cel l s of the nephron. Those sodium channels normal ly are respons ible for Na + and K+ exchange (some extra Na + recla imed, extra K+ los tinto the urine). In the presence of ami loride the ion exchange i s inhibi ted and this consti tutes the potass ium-sparing effects that we use toclass i fy these diuretics . Al though a ldosterone plays a role in renal handl ing of Na + and K+, ami loride (l ike triamterene) wi l l work in the absence ofa ldosterone—unl ike the potass ium-sparing effects of spi ronolactone or eplerenone, which only work in the presence of a ldosterone becausetheir mechanism involves blockade of a ldosterone receptors , with no di rect effects on principa l cel l ion channels .

There i s no logica l mechanism with which to envisage an ACE inhibi tor/potass ium-sparing diuretic caus ing hypertens ion (a). Nei ther an ACEinhibi tor nor a K+-sparing diuretic, a lone or together, has di rect cardiac-depressant effects (c; unless the drugs were adminis tered at such toxicdoses that electrolyte levels changed so much that they affected the heart). Polyuria , a diabetes ins ipidus-l ike s tate (d) and a syndrome ofinappropriate ADH secretion do not occur with ei ther of these drugs or with the two combined. As noted above, both drugs exert potass ium-sparingeffects , and so hypokalemia (e) wi l l not occur.

290. The answer is b. (Brunton, pp 682-686, 696-701; Katzung, pp 258, 260.) That a loop diuretic (furosemide, bumetanide, torsemide, or perhaps evenethacrynic acid) would be a rational choice for helping to manage diuretic-induced hyponatremia seems i rrational , i f not pla in wrong. After a l l ,these drugs are cons idered to be the “big guns” of a l l diuretics , capable of caus ing a lmost l imitless dose-dependent diures is—large increases ofrenal loss of water, sodium, potass ium, and other ions . So, what i s the explanation?

When I described the actions of a thiazide or thiazide-l ike diuretic I sa id that they impair the kidney’s abi l i ty to form a normal ly di lute urine: ina relative sense they cause the loss of more Na + than of free water, and so Na + concentration in the blood fa l l s (di lutional hyponatremia). Thes i tuation i s essentia l ly the oppos i te with loop diuretics . By reducing the medul lary-to-cortica l inters ti tia l osmotic gradient (the countercurrentmultipl ier phenomenon), secondary to reduced Na + transport out of the urine in the ascending l imb of the Loop of Henle, they impair the abi l i ty ofthe kidneys to form a normal ly concentrated urine. In a relative sense, the urine formed in the presence of a loop agent therefore conta ins morewater than sodium, and so the concentration of Na + in the blood ri ses . This tends to correct the hyponatremia.

Captopri l (a ), the prototype ACE inhibi tor, tends to lower or not change blood Na + concentrations , but i t does not ra ise them. ACE inhibi tors

Page 172: Pharmacology pre test self assessment and review (14th ed.)

reduce the formation of A-II , which normal ly s timulates a ldosterone release from the adrenal cortex. Aldosterone causes the renal tubules toreta in Na + and lose K+. So when a ldosterone release i s reduced (due to less A-II ava i lable), the oppos i te occurs : Na + (and water) el imination ri ses(but Na + concentration tends not to change), and additional K+ i s los t. Spi ronolactone (c) tends to do many of the same things that an ACE inhibi tordoes to the kidneys , but i t works by blocking renal a ldosterone receptors (leading to increased Na + loss and K+ retention) and has no effects on A-II ’s synthes is or di rect effects . Thiazides or related agents (d) are clearly the wrong choice: to rei terate an important point, of a l l the diuretics thethiazides and related agents are the most common cause of hyponatremia. Triamterene (and the related drug ami loride) are K+-sparing diureticsthat act by blocking Na + channels on the luminal face of the principa l cel l s , thereby inhibi ting Na + for K+ exchange, in a manner that i s notdependent on a ldosterone. They do not ra ise plasma Na + concentrations and are not effective or used for di lutional hyponatremia.

291. The answer is d. (Brunton, pp 676-701; Katzung, pp 258-265.) Drug 2 i s furosemide, and there are severa l tip-offs to help you pick the right answer,some that are admittedly less helpful than the others . Not at a l l helpful in dis tinguishing between diuretics i s the increased renal sodium loss .Al l the common diuretics do that. Next, note the increase in renal potass ium loss . This i s a l so not too helpful , s ince acetazolamide (a), thethiazide-l ike diuretic chlortha l idone (c), and hydrochlorothiazide (e) have a potass ium-wasting effect s imi lar to that of furosemide.

How do you narrow things down from here, then? Fi rs t, the data show that Drug 2 causes no increase in renal bicarbonate loss . From that youcan infer no inhibi tion of carbonic anhydrase activi ty. That rules out, especia l ly, acetazolamide (a ; i t i s , a fter a l l , a s trong carbonic anhydraseinhibi tor). But the absence of any changes in bicarbonate loss should a lso lead you away from selecting a thiazide (e) or thiazide-l ike (c) agent,s ince most of them have at least some—but rather weak in comparison with acetazolamide—carbonic anhydrase inhibi tory activi ty.

Perhaps the best tip-off to help you dis tinguish between furosemide and a thiazide or thiazide-l ike drug? Loop diuretics increase renal ca lciumloss ; thiazides and thiazide l ike agents do the oppos i te, caus ing a ca lcium-sparing effect. Changes in renal magnes ium loss aren’t too helpful ,s ince thiazides , thiazide-l ike agents , and loop diuretics a l l increase magnes ium loss .

So, here’s the ful l set of answers :Acetazolamide (a) has a profi le that best matches drug 4 (note the markedly increased HCO3

- loss and decreased Cl - loss ). Be aware, too, thatacetazolamide i s a lso a potass ium-wasting diuretic, a point that i s often overlooked because we tend to ta lk about thiazides and loop diureticsas the “main” potass ium-wasting agents .

Ami loride’s (b) profi le fi ts that of drug 3. More so than anything else, the give-away i s the reduced renal K+ loss—not surpris ing s ince ami lorideis class i fied as a potass ium-sparing diuretic.

Chlortha l idone (c) and hydrochlorothiazide (e), which are thiazide-l ike or true thiazide diuretics (respectively), would produce profi les l ike drug1: K+ loss , a decrease of Ca 2+ excretion, and variable effects on HCO3

- loss due to variable effects (drug- and dose-dependent, but genera l ly weak)on carbonic anhydrase.

Rather than rehashing the s i tes and mechanisms by which the “main” diuretics cause their effects—that’s largely been addressed elsewhereabove—what fol lows are some tips for arriving at a correct answer more eas i ly. I ’l l add some additional comments that may not have beenemphas ized before.

Note that a l l the profi les show an increased (qual i tative) renal excretion of Na +. The essence of this i s that i t reflects the fact that a l l thecommon diuretics do that. Indeed, this natriuretic effect (sodium-excreting effect) i s part of the working defini tion of “diuretic.”

The next s tep i s to identi fy which agents increase renal potass ium excretion and which do the oppos i te, because the main diuretics are ei therpotass ium-sparing or potass ium-wasting. That wi l l narrow your choices nicely. You should be able to place a drug in the proper potass ium-relatedclass instantaneous ly (with the poss ible exception of acetazolamide, a potass ium-wasting drug, because we tend to emphas ize i ts CA-inhibi toryeffects and overload other important properties ; see the answer to Question 268 above).

Fina l ly, once you see that a drug’s profi le involves increased K+ loss , you have narrowed your choices to a thiazide or thiazide-l ike agent (hydro-chlorothiazide, chlortha l idone, metolazone, many others ), a loop agent, or a CA inhibi tor.

Look now at what happens to urinary Ca 2+ excretion. If i t’s reduced, i t’s a thiazide or a related agent (reca l l we can use these drugs foridiopathic hyperca lciuria , an effect that capi ta l i zes on reduced presence of Ca 2+ in the urine due to reduced renal Ca 2+ loss ). If Ca 2+ excretion i sincreased, i t’s a loop diuretic.

Now, how do you identi fy which agent i s a CA inhibi tor (ie, acetazol -amide)? In addition to looking at Na + and K+, note especia l ly what happensto HCO3

- and Cl -. Acetazolamide causes profound urinary a lka l inization from the HCO3- loss ; so much so that the blood’s other major anion, Cl -, i s

reta ined. Just looking at the table you would probably rule out a thiazide (a) as being acetazolamide because I indicate that the effect on thatanion i s variable (+/-).

How could you rule out triamterene, s ince we showed increased HCO3- loss with i t? (It’s a s l ight effect, by the way, and CA inhibi tion i s not i ts

main nor an important mechanism or outcome, but you can’t tel l from the table.) Look at the Cl - profi le and you wi l l see that acetazolamide i s theonly diuretic that lowers Cl - excretion. That’s because one of the blood’s major anions , HCO3

-, i s los t to such a great degree that the other mainanion, Cl -, i s conserved.

Page 173: Pharmacology pre test self assessment and review (14th ed.)

Respiratory System Pharmacology: Asthma and COPD Autonomic control of a i rway smooth muscle toneCorticosteroids and control of a i rway inflammation in asthmaDrugs that can exacerbate asthmaHistamine and his tamine receptor blockersImmunoglobul in antibodies for as thmaLeukotrienes and leukotriene modi fiers in asthmaMethylxanthine roles in asthma therapy

Questions

292. A 27-year-old patient i s seen in the outpatient cl inic for a routine checkup. She’s just moved to your town, and this i s the fi rs t time you’ve mether. She has asthma and says she’s been us ing her inhaler (a lbuterol ) most days each week (sometimes twice a day) as her only therapy. “Ibreathe a lot eas ier after I take severa l puffs ,” she says . She feels wel l otherwise. Which s tatement appl ies to this patient or her treatment plan?

a. Her treatment plan i s inappropriate and needs to be modi fiedb. If pulmonary function tests performed at your hospi ta l today are normal , i t i s l i kely that the diagnos is i s incorrect: she does not have asthmac. She i s l ikely to enjoy activi ties , such as ski ing and skating, in which she breathes cold, clear a i rd. She i s most l ikely to have trouble with her breathing when in a warm, humid atmospheree. She i s probably s l ightly bradycardic and hypertens ive from the drug’s expected actions on the heart and periphera l vasculature

293. Certa in adrenergic agonis ts clearly play a role in managing some patients with asthma, whether for prophylaxis (control medication) or forrescue therapy. Which drug i s class i fied as an adrenergic agonis t, but has no phys iologica l ly relevant or cl inica l ly useful effects on a i rway smoothmuscle tone?

a. Albuterolb. Epinephrinec. Norepinephrined. Sa lmeterole. Terbuta l inef. Theophyl l ine

294. A patient with asthma has moderate bronchospasm and wheezing about twice a week. Current medications are inhaled a lbuterol (mainly foracute symptom control ) and inhaled beclomethasone as a “control medication.” The patient continues to have occas ional and genera l ly mi ldflare-ups of his as thma. If the phys ician wishes to make sa lmeterol part of the treatment plan, how best should i t be used for this patient?

a. A replacement for the a lbuterolb. A replacement for the corticosteroidc. An add-on to current medications for addi tional prophylactic benefi tsd. Primary (sole) therapy, replacing both a lbuterol and the s teroide. The preferred agent for acute symptom control (rescue therapy)

295. The attending in the outpatient pulmonary cl inic s tates she i s going to s tart omal izumab therapy for a 19-year-old with refractory asthma.What most accurately describes the actions , uses , or adverse effects of this drug?

a. Antibody that binds and therefore inactivates endogenous ACh and his tamineb. Contra indicated i f patient i s taking an ora l or ora l ly inhaled corticosteroidc. Good a l ternative to a lbuterol or s imi lar adrenergic bronchodi lators for rescue therapyd. Immediate- or delayed-onset anaphylactic reactions pose the greatest ri ske. Novel agent l ikely to become fi rs t-l ine therapy as a control medication for mi ld but recurrent asthma

296. A 19-year-old moves from a smal l town to your ci ty, and i s now your patient. He has a his tory of as thma, and his previous primary carephys ician was managing i t wi th ora l theophyl l ine. What best summarizes the efficacy or current s tatus of theophyl l ine in particular, ormethylxanthines in genera l , in such patients as this?

a. Dos ing i s s imple and convenient, rarely needs to be adjustedb. Excel lent a l ternative to an inhaled s teroid for “rescue” therapyc. Is , at best, a second- or thi rd-l ine agent for long-term asthma controld. Possesses s trong and cl inica l ly useful anti -inflammatory activi tye. Sedation i s a major s ide effect, even with therapeutic doses or blood levels

297. An elderly man with COPD is being managed with severa l drugs , one of which i s inhaled ipratropium. What i s the main mechanism thataccounts for the beneficia l effects of this drug?

a. Blocks receptors upon which an endogenous bronchoconstrictor mediator actsb. Enhances epinephrine release from the adrenal medul lac. Inhibi ts cAMP breakdown via phosphodiesterase inhibi tiond. Prevents antigen-antibody reactions that lead to mast cel l mediator releasee. Stimulates venti latory rates (CNS effect in bra in’s medul la )

Page 174: Pharmacology pre test self assessment and review (14th ed.)

f. Suppresses synthes is and release of inflammatory mediators

298. A 16-year-old gi rl treated for asthma develops drug-induced skeleta l muscle tremors . What was the most l ikely cause?

a. Albuterolb. Beclomethasonec. Cromolynd. Ipratropiume. Montelukast

299. A 26-year-old patient with asthma is being treated with montelukast. What i s the main mechanism by which this drug works?

a. Blocks receptors for certa in proinflammatory and bronchoconstrictor arachidonic acid metabol i tesb. Enhances release of epinephrine from the adrenal (suprarenal ) medul lac. Increases a i rway β-adrenergic receptor respons iveness to endogenous norepinephrined. Inhibi ts cAMP breakdown via phosphodiesterase inhibi tione. Prevents antigen-antibody reactions that lead to mast cel l mediator releasef. Stimulates venti latory rates (CNS effect in bra in’s medul la )

300. An elderly man, in obvious respiratory dis tress due to exacerbation of his emphysema and chronic bronchi ti s (COPD), presents in theemergency department. One drug ordered by the phys ician, to be adminis tered by the respiratory therapis t, i s N-acetylcysteine. What i s the mainaction or purpose of this drug?

a. Blocks receptors for the cysteinyl leukotrienesb. Inhibi ts metabol ic inactivation of epinephrine or β2 agonis ts that were adminis tered

c. Inhibi ts leukotriene synthes isd. Promptly suppresses a i rway inflammatione. Reverses ACh-mediated bronchoconstrictionf. Thins a i rway mucus secretions for eas ier removal by suctioning or postura l dra inage

301. A 23-year-old woman with asthma has what i s described as “aspirin (hyper)sens i tivi ty” (triad asthma) and experiences severe bronchospasmin response to even smal l doses of the drug. What i s the most l ikely mechanism by which the aspi rin provoked her pulmonary problems?

a. Blocked synthes is of endogenous prostaglandins that have bronchodi lator activi tyb. Induced formation of antibodies di rected aga inst the sa l icylate on a i rway mast cel l sc. Induced hypersens i tivi ty of H1 receptors on a i rway smooth muscles

d. Induced hypersens i tivi ty of muscarinic receptors on a i rway smooth musclese. Prevented epinephrine binding to β2-adrenergic receptors (a i rways and elsewhere)

302. A young boy i s diagnosed with asthma. His primary symptom is frequent cough, not bronchospasm or wheezing. Other asthma medications arestarted, but unti l thei r effects develop ful ly we wish to suppress the cough without running a ri sk of suppress ing venti latory drive or caus ingsedation or other unwanted effects . Which drug would meet these needs the best?

a. Codeineb. Dextromethorphanc. Diphenhydramined. Hydrocodonee. Promethazine

303. A mother brings her 10-year-old son, who has a longstanding his tory of poorly control led asthma, to the Emergency Department (ED). He i s in arelatively early s tage of what wi l l prove to be a severe asthma attack. Arteria l blood gases have not been analyzed yet, but i t i s obvious that thelad i s in great dis tress . He i s panting with great effort at a rate of about 60/min.

Given the boy’s his tory and the l ikely diagnos is of acute asthma, the heal th care team adminis ters a l l the drugs l i s ted below by the s tatedroutes , and with the purposes noted. The chi ld’s condition quickly improves , and the team leaves the boy with his mother whi le they go to care forother ED patients . Within a couple of minutes , the mother comes out of her son’s cubicle frantica l ly screaming “he’s s topped breathing!” Whichdrug most l ikely precipi tated the venti latory arrest?

a. Albuterol , inhaled, given by nebul i zer for prompt bronchodi lationb. Atropine, nebul i zed and inhaledc. Midazolam, IV, to normal ize venti latory rate and a l lay anxietyd. Methylprednisolone (glucocorticosteroid), IV, for prompt suppress ion of a i rway inflammatione. Normal sa l ine, nebul i zed and inhaled, to hydrate the a i rway mucosae

304. You prescribe an ora l ly inhaled corticosteroid for a patient with asthma. Previous ly she was us ing only a rapidly acting adrenergicbronchodi lator for both prophylaxis and treatment of acute attacks . She used the s teroid as di rected for 5 days , and then s topped taking i t. What i sthe most l ikely reason why the patient qui t us ing the drug?

a. Dis turbing tachycardia and pa lpi tations occurredb. Relentless diarrhea developed after just 1 day of us ing the s teroidc. She experienced l i ttle or no obvious improvement in breathingd. The drug caused extreme drows iness that interfered with daytime activi ties

Page 175: Pharmacology pre test self assessment and review (14th ed.)

e. The drug caused her to reta in fluid and ga in weight

305. A patient consumes an excess ive dose of theophyl l ine and develops toxici ty in response to the drug. What i s the expected s ign, symptom, orother consequence of this overdose?

a. Bradycardiab. Drows iness progress ing to s leep and then comac. Hepatotoxici tyd. Paradoxica l bronchospasme. Seizures

306. Acetylchol inesterase inhibi tors , muscarinic agonis ts such as pi locarpine, and β-blockers are among the drugs used to manage patients withglaucoma. They a lso share properties that are particularly relevant to patients with asthma. Which s tatement summarizes best what thatrelevance i s?

a . Contra indicated, or pose great ri sks , for people with asthmab. Degranulate mast cel l s , cause bronchoconstrictionc. Tend to ra ise intraocular pressure in patients who have both glaucoma and asthmad. Trigger bronchoconstriction by di rectly activating H1 hi s tamine receptors on a i rway smooth muscle cel l s

e. Useful for acute asthma, not for ambulatory patients

307. A patient suffering s tatus asthmaticus presents in the emergency department. Blood gases reveal severe respiratory acidos is and hypoxia .Even large parentera l doses of a selective β2 agonis t fa i l to di late the a i rways adequately; rather, they cause dangerous degrees of tachycardia .Which pharmacologic intervention or approach i s most l ikely to control the acute symptoms and restore the bronchodi lator efficacy of theadrenergic drug?

a. Give a l ipoxygenase inhibi tor (eg, montelukast)b. Give a parentera l corticosteroidc. Give parentera l diphenhydramined. Switch to epinephrinee. Switch to i soproterenol (β1/β2 agonis t)

308. The FDA has mandated a warning about the increased ri sk of death in asthma patients taking recommended doses of sa lmeterol (or others low-/long-acting adrenergic as thma control medications) for too long a period of time. What i s the most l ikely mechanism or phenomenon bywhich this fata l adverse response, or other severe a i rway responses , occur?

a. Anaphylaxis due to gradual formation of antibodies aga inst the drugb. Hypertens ive cri s i sc. Inhibi ted synthes is , release, and anti -inflammatory actions of endogenous corti sold. Plugging of the bronchioles with viscous mucus , due to the drug’s antimuscarinic actionse. Tolerance of a i rway smooth muscle cel l s to the effects of adrenergic bronchodi lators

309. A hungry 8-year-old i s vi s i ting his grandparents ’ house and sees grandpa sprinkle some powder from a smal l foi l pack onto his applesauce.Grandpa smi les and says “Mmmm, tastes good.” The packet conta ins a theophyl l ine sa l t that was prescribed for his COPD. Liking applesauce, andthinking the drug packets conta in candy, the lad opens the applesauce jar, finds grandpa’s s tash of dozens of theophyl l ine packets , and sprinkleson lots of drug before eating the whole thing. Toxici ty ensues . What i s this chi ld most l ikely to experience, and do so before anything elses igni ficant and untoward develops?

a. Acute urinary retentionb. Bradycardia and hypotens ionc. Irri tabi l i ty, other s igns/symptoms of CNS s timulation, poss ibly seizuresd. Ischemic s troke from drug-induced platelet aggregatione. Venti latory suppress ion (centra l ly mediated) leading to respiratory acidos is and apnea

Page 176: Pharmacology pre test self assessment and review (14th ed.)

Respiratory System Pharmacology: Asthma and COPD

Answers

292. The answer is a. (Brunton, pp 302, 1035-1040; Katzung, pp 144, 339-357.) This patient certa inly seems to have recurrent episodes of as thma. Even i fthey are “mi ld” and ostens ibly respons ive to an inhaled β2 agonis t, she’s not being treated properly. She or her prior prescriber has been over-rel iant on, and overus ing, a medication that only manages symptoms (and not wel l in her case) and does not address the underlying cause, whichis a i rway inflammation. (Some phys icians define overuse of an inhaled adrenergic bronchodi lator more than once a day; the NIH says needing touse i t more than twice a week i s overuse.) Reasonable options for her would be to prescribe an inhaled corticosteroid (beclomethasone, others ),s ince she reports only mi ld symptoms. (Of course, pulmonary function tests [PFTs] should be done.) If her bouts become more severe or frequent,then an ora l s teroid (eg, prednisone) might be prescribed for a whi le, with a proper switch to an inhaled s teroid.

The PFTs might be reasonably normal (b), but that depends on such factors as when she used her inhaler las t, and whether she i s s imply havinga “good day” (eg, no exposure today to a l lergens , etc). A s ingle normal PFT in a patient who describes frequent need for a bronchodi lator does notrule out asthma. Let’s use an analogy that appl ies to many asthma patients . You are s tanding right on the edge of a thousand-foot drop-off on therim of the Grand Canyon. Only a couple of inches from a bad trip down. Take just one l i ttle s tep forward and you’re a goner. Same with asthma: youmay be “feel ing good” with current medications , but just a l i ttle provocation of a i rway inflammation and bronchoconstriction and severesymptoms, poss ibly necess i tating intubation or leading to a fata l attack, can put the patient over the proverbia l edge.

Patients with asthma tend not to do wel l in cold temperatures (c). Breathing cold a i r often provokes bronchospasm (probably mediated byincreased parasympathetic effects on a i rway smooth muscle). (Have you ever taken a deep breath when the temperatures are near or below zero?)Conversely, as thmatics tend to do wel l when i t i s warm and humid. (Ever noticed how many athletes with asthma tend to gravi tate to sports thatprovide a warm, humid envi ronment, eg, swimming?) Fina l ly, β2 agonis ts do not cause bradycardia or increases of periphera l res is tance.Muscarinic agonis ts or β-blockers would cause the former, α agonis ts the latter.

293. The answer is c. (Brunton, pp 173, 196-200, 287-288; Katzung, pp 138-148, 341-344, 352-353.) Reca l l that “adrenergic bronchodi lation” requiresactivation of β2-adrenergic receptors on a i rway smooth muscle cel l s . Norepinephrine i s an α and β1 agonis t, wi th no abi l i ty to activate β2 receptors ,and so i t has no bronchodi lator activi ty. Albuterol (a ), a β2 agonis t, can be cons idered the prototype of rapidly acting adrenergic bronchodi latorsused for asthma (prophylaxis/control , but mainly rescue therapy). Epinephrine (b) i s a very efficacious bronchodi lator, but i ts use for routinemanagement of as thma is cons iderably l imited due to i ts abi l i ty to cause vasoconstriction (α-agonis t activi ty) and cardiac s timulation (β1 agonis tactivi ty). In terms of pulmonary problems, i t i s reserved mainly for managing anaphylaxis or s tatus asthmaticus , but i t indeed causes broncho-di lation wel l and promptly. Sa lmeterol (d) i s a β agonis t a lso. Given by inhalation, i ts bronchodi lator effects are s low in onset, but i t has a longduration compared with s imi larly class i fied drugs such as a lbuterol . Thus , sa lmeterol i s an efficacious bronchodi lator, but not sui table for prompteffects (rescue therapy). Terbuta l ine (c) i s class i fied as a rapidly acting β2 agonis t, s imi lar to a lbuterol . However, i t i s seldom used any more,whether as an adrenergic bronchodi lator or for i ts other main use as a uterine relaxant to help manage premature labor. Theophyl l ine (f) hasmodest bronchodi lator activi ty, but i t i s not an adrenergic agonis t. It inhibi ts phosphodiesterase, and so inhibi ts metabol ic inactivation of c-AMP(the “second messenger” formed by adrenergic bronchodi lators in a i rway smooth muscle).

294. The answer is c. (Brunton, pp 292f, 293, 322t; Katzung, pp 341-344, 352-353.) If you were to decide to use sa lmeterol in this s i tuation, i t would bebest to cons ider i t as an add-on. Reca l l that sa lmeterol , l i ke a lbuterol , i s mainly a β2 agonis t, but sa lmeterol ’s onset of action i s much s lower, i tsduration of action much longer. The s low onset of sa lmeterol makes i t a useful prophylactic agent for some patients , but renders i t completelyinappropriate for rescue therapy (a , e). It can be adminis tered separately, or the phys ician might prescribe one of the proprietary fixed-dosecombination products that conta in both sa lmeterol and a s teroid (given by ora l inhalation). One could reasonably argue that sa lmeterol i s a l sodangerous i f used as the sole intervention for rescue, because serious venti latory compromise can develop before i ts bronchodi lator effectsdevelop. The patient described needs a corticosteroid. You might increase the dose of the inhaled s teroid as a s tart, or perhaps try a burst of anora l s teroid (eg, prednisone) for a whi le. Regardless , us ing sa lmeterol as a replacement for any corticosteroid (b) or for a l l the other currentmedications (d) i s l ikely to provoke more and more severe asthma attacks . Indeed, current guidel ines are that i f sa lmeterol or another long-actingβ agonis t i s used, i t must be used as an adjunct to a corticosteroid. Remember: The key to success in asthma therapy long-term requirescontrol l ing a i rway inflammation, and corticosteroids are the key drugs for doing just that.

295. The answer is d. (Brunton, pp 1054-1055; Katzung, pp 350, 993.) The most serious adverse response to omal izumab, a monoclonal antibody di rectedagainst IgE, i s an anaphylactic reaction (often severe, less often fata l ). These adverse reactions usual ly (not a lways!) occur after the fi rs t dose, andusual ly within 2 hours of injection. This i s one reason why patients should receive the drug only in a “control led” heal th care setting and shouldbe under observation for at least 2 hours thereafter. Nonetheless , anaphylaxis or less severe untoward immunologic reactions have been reported24 hours or longer, and so the patient should have an epinephrine autoinjector (sel f-injector) (prescribed or dispensed by the phys ician) handy forat least severa l days after an omal izumab dose. The ri sk of anaphylaxis i s a l so a major reason why the drug should be used only for patients whohave severe asthma not adequately control led with other drugs (adrenergic bronchodi lators , leukotriene receptor blockers , corticosteroids ).

As noted, the antibody i s di rected aga inst IgE. The IgE-omal izumab complex then prevents IgE from binding to mast cel l and basophi lmembranes , which otherwise would trigger a l lergen-mediated cel l activation. Omal izumab does not bind other l igands (eg, ACh or his tamine; a ),nor does i t a l ter the effects of those bronchoconstrictor agonis ts in any other way. It a l so has no intrins ic or di rect bronchodi lator activi ty.

Owing to the anaphylaxis ri sk, the drug’s s low onset of action, and i ts lack of bronchodi lator activi ty, omal izumab is not, nor ever wi l l be, fi rs t-l ine therapy for asthma (e), whether mi ld or severe, frequent or not. It wi l l never be a substi tute for a rapidly acting adrenergic bronchodi lator forrescue therapy (c). And prior or current use of any of the “tradi tional” as thma medications does not contra indicate (b) a tria l wi th omal izumab.Indeed, owing to omal izumab’s s low onset, continued and otherwise proper corticosteroid therapy to suppress a i rway inflammation i s essentia l .

296. The answer is c. (Brunton, pp 821t, 1040-1044, 1534; Katzung, pp 344-346, 1037-1038, 1161t.) Theophyl l ine, the prototype methylxanthine, i s an ora lbronchodi lator that has some beneficia l pulmonary effects in patients with asthma or COPD. We don’t know precisely how i t exerts i tsbronchodi lator activi ty, at least at concentrations found in human beings (in vi tro i t inhibi ts phosphodiesterase, preserving cAMP levels ; i t a l soblocks receptors for adenos ine, which has bronchoconstrictor activi ty).

However, there are serious problems with this old drug that relegate i t to second or thi rd s tatus in therapy; that render i t a drug that should beprescribed only by pulmonologis ts who are qui te fami l iar with the l imitations , s ide effects , and toxici ty; and that make i ts use as primary therapyfor asthma usual ly inappropriate (at best).

Page 177: Pharmacology pre test self assessment and review (14th ed.)

Theophyl l ine (and other methylxanthines) has a very low therapeutic index: i t i s a l l too easy (whether because of drug-drug interactions , ors imply improper dos ing) to have blood levels ri se into a range that can cause toxici ty (mainly excess ive cardiac and CNS s timulation, with thepotentia l for arrhythmias or seizures ). Moreover, and because of the problems s tated above, frequent blood testing i s necessary in order to get,and keep, plasma levels within an acceptable therapeutic range and hopeful ly avoid subtherapeutic or toxic blood levels . The drug depends onhepatic P450 s tatus for i ts el imination, and that makes el imination very eas i ly influenced by l iver dys function or other interacting drugs that canei ther induce or inhibi t i ts metabol i sm.

Most important i s the fact that theophyl l ine has no anti -inflammatory activi ty; as I sa id, i f we are treating asthma long-term, the “secret tosuccess” i s control l ing a i rway inflammation. For our foundation we use corticosteroids to suppress inflammation.

If we were deal ing with COPD and the goal i s bronchodi lation (i t i s ), such drugs as ipratropium (inhaled antimuscarinic) are not only moreefficacious than a methylxanthine or an adrenergic bronchodi lator but a lso associated with fewer dos ing problems, monitoring needs , and s ideeffects . In COPD the pathophys iology involves not only bronchoconstriction but a lso goblet cel l hyperplas ia , leading to increased mucusproduction. Such drugs as ipratropium are, therefore, usual ly a fi rs t choice and usual ly improve symptoms and pulmonary function tests the most.

297. The answer is a. (Brunton, pp 231, 1044-1046; Katzung, pp 121, 127, 347, 353, 355.) You should be able to deduce quickly (i f not s imply know outright)that ipratropium is an atropine-l ike drug—an antimuscarinic that blocks the bronchoconstrictor effect of ACh on a i rway smooth muscle. It i s aquaternary, inhaled antimuscarinic: that i s , i t acts loca l ly in the a i rways , and very l i ttle of the “a lways charged” molecule di ffuses into theci rculation to cause systemic effects . Ipratropium has no effects on the adrenal medul la , on mast cel l s or other elements of the inflammatoryresponse, or in the CNS. Ipratropium is approved for managing COPD, but i s often used for some asthma patients . The drug has a rapid onset ofaction. However, some s tudies indicate that i t i s not as efficacious a bronchodi lator as β2 agonis ts , and so i t i s not a sui table agent for rescuetherapy. Fina l ly, the bronchodi lator effects of ipratropium and β agonis ts are synergis tic (same effect, di fferent mechanisms), and so there’s somelogic to us ing them both.

298. The answer is a. (Brunton, pp 206t, 291, 302, 1035-1040; Katzung, pp 145, 341-344, 355.) Of the drugs l i s ted in the question, skeleta l muscle tremor i sa s ide effect most often associated with β2-adrenergic agonis ts—albuterol and others . It i s a lmost a lways a dose-dependent phenomenon. (Note:Some phys icians ti trate the dose of adrenergic bronchodi lators upward, s topping when tremors develop and equating the blood level associatedwith that dose as being therapeutic with respect to control l ing the pulmonary disease. The wisdom of that i s questionable; proper dosageadjustments or other therapy changes should be based on pulmonary symptom rel ief [subjective assessment] in genera l and, idea l ly, quanti tativepulmonary function tests .) None of the other drugs l i s ted i s associated with skeleta l muscle tremor. Beclomethasone (b) i s an inhaledcorticosteroid; cromolyn (c), a l so inhaled for asthma, reduces mast cel l degranulation; ipratropium (d) i s an inhaled antimuscarinic indicated forCOPD; montelukast (e) i s a leukotriene receptor blocker that i s a l so indicated for asthma prophylaxis .

299. The answer is a. (Brunton, pp 942, 1052-1054; Katzung, pp 328, 339-357.) Montelukast (and a related drug, zafi rlukast) blocks receptors forleukotrienes (LTs ). (When you see “leuk” or “luk” as part of these drugs ’ generic names , think leukotrienes or leukotriene modi fiers , and think ofdrugs for as thma.)

Reca l l that the LTs are proinflammatory and bronchoconstrictor mediators formed as part of normal arachidonic acid metabol i sm via the 5′-l ipoxygenase pathway. LTs a lso participate in recrui ting eos inophi l s to the inflamed regions (reca l l that loca l eos inophi l ia i s often anaccompaniment of as thma), thereby blunting the loca l inflammatory processes in yet another way.

In the context of as thma therapy leukotriene receptor blockers are indicated for prophylaxis (control therapy) only, and should not be used inl ieu of corticosteroids (ora l or ora l ly inhaled) when corticosteroids are sui table (which i s most of the time). They wi l l do vi rtua l ly no good in ashort enough time i f they were adminis tered to suppress ongoing bronchoconstriction (eg, for rescue therapy), mainly because they are too s low-acting. It’s worth noting that a l though montelukast works wel l as a control medication for many asthma patients , i t i s not a panacea. For about50% of asthma patients i ts efficacy appears to be no better than placebo.

The leukotriene modi fiers do not enhance epinephrine release from the adrenal medul la (b), increase β-adrenergic receptor respons iveness (c;note that norepinephrine, mentioned in the question, has no bronchodi lator activi ty because i t i s not a β2 agonis t), inhibi t phosphodiesterase (d),or interfere with mast cel l function or mast cel l respons iveness to antigens (e).

Note: Remember the two other main classes of drugs that affect ei ther the formation or breakdown of arachidonic acid. The cyclooxygenasepathway(s ) metabol i ze arachidonic acid to form various prostaglandins , prostacycl in (PGI2), and thromboxanes (eg, thromboxane A2). Thecyclooxygenases are inhibi ted by the tradi tional nonsteroida l anti -inflammatory drugs , of which aspi rin i s a reasonable and common example.The (gluco) corticosteroids work “upstream” of both l ipoxygenase and cyclooxygenase. They inhibi t the formation of arachidonic acid by inhibi tingphosphol ipase activi ty, which thereby removes substrate for both l ipoxygenase and cyclooxygenase metabol i sm.

300. The answer is f. (Brunton, p 1057; Katzung, p 1033t.) You should know N-acetylcysteine as an antidote for acetaminophen poisoning. When usedfor that purpose, i t i s given ora l ly or intravenous ly. The sul fhydryl groups that are part of the molecule are important, as they react with the toxicacetaminophen metabol i te and spare glutathione-depleted hepatocytes from oxidative attack. N-Acetylcysteine i s a lso a mucolytic (mucus-thinning) drug, given by inhalation in a nebul i zed solution or by intratracheal insti l lation, to reduce the viscos i ty of a i rway mucus that can then beremoved eas ier by coughing, postura l dra inage and chest percuss ion, or a i rway suctioning. Here, too, the mechanism is based on i ts -SH-richcompos i tion. N-Acetylcysteine lacks other a i rway effects such as bronchodi lation or suppress ion or inflammation.

301. The answer is a. (Brunton, pp 947, 964; Katzung, pp 339-357.) Aspirin, the prototype of the nonsteroida l anti -inflammatory drugs (NSAIDs), inhibi tscyclooxygenases (COX-1 and -2) and the “arachidonic acid cascade.” One outcome of that i s inhibi ted synthes is of PGE2. The PGE2 (a long withci rculating epinephrine) i s a phys iologica l ly important endogenous bronchodi lator that i s particularly important to mainta in a i rway patency(di lation) for most asthmatics . Inhibi t this synthes is , wi th aspi rin or another nonselective cyclooxygenase inhibi tor/NSAID, andbronchoconstriction or bronchospasm may ensue in that population of as thmatics who often are exquis i tely sens i tive to these drugs . This aspi rinhypersens i tivi ty phenomenon i s relatively uncommon in chi ldren with asthma, but more preva lent in adults . Es timates are that i t a ffects between10% and 25% of adul t as thmatics with so-ca l led “triad asthma”: as thma plus nasa l polyps , chronic urticaria , and the NSAID sens i tivi ty.

Vi rtua l ly a l l other NSAIDs that nonselectively inhibi t COX-1 and -2 may cross -react, a l though the incidence of severe pulmonary reactions seemsthe highest with aspi rin i tsel f. Selective COX-2 inhibi tors (ie, celecoxib) pose less of a problem because they are less l ikely to inhibi t thebronchodi lator prostaglandins that are formed by the COX-1 pathway, but they are not absolutely free from the potentia l problem. If one needs adrug to manage fever or mi ld pa in in an asthmatic, acetaminophen i s preferred: i t does not cross -react. Owing to negl igible effects onprostaglandin synthes is via cyclooxygenase-dependent pathways , i t i s not class i fied as an NSAID.

302. The answer is b. (Brunton, p 516; Katzung, p 559.) Dextromethorphan i s a centra l ly acting anti tuss ive drug that i s about as efficacious a cough

Page 178: Pharmacology pre test self assessment and review (14th ed.)

suppressant as codeine. It i s thought to suppress the vagal a fferent neura l pathway involved in the cough reflex. However, unl ike codeine (c) andhydrocodone (d; another useful anti tuss ive in some cases), dextromethorphan i s not an opioid and lacks analges ic effects or the potentia l forventi latory suppress ion or abuse.

Diphenhydramine (c) and promethazine (e) a lso have anti tuss ive action. However, they, too, can cause genera l i zed CNS and venti latorydepress ion. They a lso exert s igni ficant antimuscarinic effects , but nei ther i s a preferred or even frequently used drug for cough suppress ion inmost patients , and in asthma patients in genera l . Al though the atropine-l ike effects of these drugs may be beneficia l in terms of inhibi ting ACh-mediated bronchoconstriction in a patient with asthma, they may a lso cause thickening of a i rway mucus , favoring plugging of the a i rways withviscous mucus depos i ts that cannot be removed normal ly by mucoci l iary transport or by coughing. That i s , the benefi ts of bronchodi lation may beoutweighed by the ri sks of mucus plugging in some asthma patients .

303. The answer is c. (Brunton, pp 458-468, 552-553; Katzung, pp 339-357, 549-552.) As ide from the midazolam, any, i f not a l l , the other drugs l i s ted wouldbe appropriate for this patient. The term asthma derives from a Greek word that means , l i tera l ly, “to pant.” In severe asthma attacks such as theone described here the hyperpnea precedes the l ikely development of venti latory depress ion and, in some cases , venti latory arrest. Thisincreased venti latory rate i s essentia l . It i s a sometimes success ful yet too often inadequate compensatory phys iologic response el ici ted toincrease venti latory oxygen uptake and el iminate excess CO2 (a l though i t may be insufficient to ra ise arteria l O2 saturation adequately and morethan sufficient to induce metabol ic a lka los is from excess CO2 loss ). Nonetheless , i t i s a “protective” venti latory response, and breathing tooquickly, even inefficiently, i s better than not breathing at a l l .

This leads to an admonition: Never give a drug that can depress venti lation or the normal venti latory drive to an asthma patient unless he orshe has a protected a i rway and venti lation can be control led and supported mechanica l ly. IV midazolam (or the IV adminis tration of vi rtua l ly anyother benzodiazepine, or, especia l ly an opioid) wi l l a l lay anxiety, but wi l l a l so tend to suppress venti latory drive and hasten the onset ofventi latory arrest. In the scenario described here, diazepam is the wrong drug to give.

Albuterol (a ) or a s imi lar β2-agonis t bronchodi lator, whether given by inhalation or parentera l ly, would not be expected to worsen the boy’sventi latory s tatus . They should not be the only drugs rel ied-on, but they would be appropriate adjunctive treatments , as would be a l l the rest forthe s tated purposes . As an important as ide, you may reca l l that atropine (b) and other drugs with antimuscarinic activi ty have bronchodi latoractivi ty, but a lso tend to make a i rway mucus secretions more viscous , leading to a i rway mucus plugging. In the context of this scenario, with theadminis tration of other drugs that I l i s ted, and the ava i labi l i ty of a i rway suctioning devices as needed, the mucus-thickening effects of anantimuscarinic should not be a problem with which we cannot eas i ly dea l by way of a i rway suctioning and the adminis tration of mucus-thinning(mucolytic) drugs . Glucocorticoids (d) and nebul i zed sa l ine would be va luable, i f not essentia l , adjuncts to this boy’s therapy.

304. The answer is c. (Brunton, pp 1046-1052; Katzung, pp 351-353.) It usual ly takes a couple of weeks of inhaled corticosteroid use “as di rected” tosuppress a i rway inflammation, and the resul ting bronchoconstriction, wel l enough that the patient can sense cl inica l improvement (fewer ormi lder asthma symptoms). Unless the patient i s forewarned about this s low onset, and urged to remain compl iant, they are l ikely to bel ieve thedrug i s ineffective and therefore not worth taking. Clearly, inhaled corticosteroids do not cause the obvious “rush”—the prompt, dramatic, andunmistakable symptom rel ief that typica l ly occurs with adrenergic bronchodi lators . Nonetheless , given adequate time, the inhaled s teroidsusual ly prove to be the key to effective long-term control of as thma. Inhaled corticosteroids act loca l ly; l i ttle enters the systemic ci rculation; suchs ide effects as fluid retention, weight ga in (e), and hyperglycemia, which are typica l with systemic s teroids (eg, prednisone), rarely occur orbecome problematic. Inhaled s teroids do not cause cardiac s timulation (a), diarrhea (b), or drows iness (d).

305. The answer is e. (Brunton, pp 1040-1044; Katzung, pp 345-346.) Theophyl l ine, a methylxanthine, i s a caffeine-l ike drug that i s becoming outmodedas therapy for asthma in most adolescents and adults . It should probably not be prescribed by any phys ician other than a pulmonologis t who i sfami l iar with the l imitations and problems with this class of drugs . A low margin of safety, extreme dependence on adequate l iver function (formetabol i sm), susceptibi l i ty to numerous cl inica l ly s igni ficant drug interactions , and a lack of a i rway anti -inflammatory activi ty are among thereasons why. Without proper dosage adjustments and monitoring, i t i s a l l too easy, and common, for blood levels to fa l l into subtherapeuticranges or, as we see here, into toxic ranges . The earl ies t s igns and symptoms of excess involve CNS s timulation (ji tteriness , tremors , di ffi cul tys leeping, anxiety). As blood levels ri se, the CNS i s increas ingly s timulated. Seizures may occur, and when they do, the inabi l i ty to breathe duringthe seizures i s the main cause of death. Thus , answer b i s incorrect.

Theophyl l ine tends to cause tachycardia , increases of cardiac contracti l i ty, and, potentia l ly, tachyarrhythmias . This occurs mainly becausemethylxanthines inhibi t phosphodiesterase, which normal ly metabol ica l ly inactivates cAMP, the levels of which are increased in smooth andcardiac muscles by β-adrenergic agonis ts . Bradycardia (a) i s not at a l l l i kely. Theophyl l ine i s not hepatotoxic (c); i t does not cause paradoxica lbroncho-spasm (d), even when plasma levels are very high or truly toxic.

306. The answer is a. (Brunton, pp 1005, 1008, 1031-1057; Katzung, pp 108-110.) By now you should be able to reca l l the class i fi cations and actions of thedrugs l i s ted in this question. Acetylchol inesterase inhibi tors and such muscarinic agonis ts as pi locarpine are sometimes used as topica l mioticsfor managing glaucoma (mainly angle-closure/narrow angle forms of the disease). Certa in topica l β-blockers are used for open-angle glaucoma,probably working by inhibi ting aqueous humor production. However, a l l these drugs (and others in these classes , used systemica l ly for a host ofother common medica l conditions) are contra indicated for asthma patients—even i f or when they are used topica l ly on the eye(s ). An importantelement in the pathophys iology of as thma is a i rway smooth-muscle hyperrespons iveness to various bronchoconstrictor s timul i , and muscarinicreceptor activation i s clearly a phenomenon that leads to often intense bronchoconstriction.

You may reca l l that the chol ine ester, methachol ine, i s used to help diagnose asthma when the diagnos is i s otherwise questionable. The bas isof this “methachol ine cha l lenge” centers on the concept of a i rway hyperreactivi ty (in this case, to muscarinic agonis ts ) in asthmatics . Very lowdoses of methachol ine are given, by inhalation, whi le pulmonary function tests are recorded. Asthmatics experience s igni ficant (sometimesdangerous) bronchoconstrictor responses to methachol ine doses that are far lower than those that would provoke even s l ight changes innonasthmatics .

Reca l l that ACh esterase inhibi tors cause what amounts to a “bui ldup” of ACh at the neuroeffector junction, leading to heightened muscarinicreceptor activation. These chol inesterase inhibi tors , whether used for glaucoma or such other conditions as myasthenia gravis , can have letha leffects in some asthma patients . (Note, too, that chol inesterase inhibi tors are found in some insecticides , so there i s an extra and s igni ficant ri skto the asthma patient in agricul tura l or gardening/lawn care settings .) Muscarinic agonis ts , whether those used for glaucoma or for s timulatingthe gut or urinary tract (eg, bethanechol for functional urinary retention), may prove letha l too. Fina l ly, as thmatics tend to be extremely dependenton the bronchodi lator actions of ci rculating epinephrine. Block the β2 receptors that mediate that effect and the outcome can be disastrous . Eventhe so-ca l led “cardioselective” β-blockers (atenolol , metoprolol ) can cause serious problems for some asthmatics . That i s because their abi l i ty toblock β1 receptors i s relative and dose-dependent, not absolute: they may pose no pulmonary problems for persons without asthma, but may bedeadly in those who do.

Page 179: Pharmacology pre test self assessment and review (14th ed.)

307. The answer is b. (Brunton, pp 1046-1052; Katzung, pp 352-353.) When pulmonary function deteriorates so much that respi ratory acidos is ensues(because sufficient amounts of CO2 aren’t being el iminated by venti lation) and severe hypoxia develops (because of inadequate oxygen transfer),acute tolerance (in essence, desens i ti zation) develops to the bronchodi lator effects of drugs with β2-agonis t activi ty—al l of them. If this point i sforgotten, repeated adminis tration of a β2 agonis t wi l l lead to increas ing degrees of cardiac s timulation (rate, contracti l i ty, automatici ty,conduction) because under these conditions they lose their selectivi ty for β2 receptors and a lso begin activating β1 receptors very effectively. Theybecome isoproterenol -l ike in thei r profi les , and i soproterenol (e) wi l l not be efficacious .

Even epinephrine (d) won’t work as an efficacious bronchodi lator under these conditions , and repeated injections of i t wi l l do l i ttle more thancause further cardiac s timulation via β1 activation, plus vasoconstriction owing to α-adrenergic receptor activation. Through mechanisms that arenot qui te clear, adminis tering sui table doses of a parentera l s teroid under these conditions of acidos is and hypoxia “restores” a substantia ldegree of a i rway respons iveness to β agonis ts . Giving a s teroid (plus oxygen, which helps correct the underlying blood gas and pH changes) i sessentia l .

Under these conditions there i s l i ttle l ikel ihood that the severe and refractory a i rway problems involve excess ive activation of a i rwayleukotriene receptors , and so adminis tering a l ipoxygenase inhibi tor such as montelukast (a ) wi l l be of l i ttle va lue. Giving diphenhydramine (c),even though i t blocks the bronchoconstrictor effects of both ACh and his tamine, wi l l not do much good for the acute and l i fe-threatening s igns andsymptoms. Therefore, a parentera l corticosteroid, adminis tration of oxygen, and support of venti lation as needed are most l ikely to have the bestoutcome.

308. The answer is e. (Brunton, pp 211-212, 302, 1035-1040; Katzung, pp 343-344, 355.) Long-term, more or less round-the-clock, exposure of β2-adrenergicreceptors to an agonis t eventual ly causes those receptors to lose sens i tivi ty to those very drugs . The loss of respons iveness to usualbronchodi lator actions—that i s , a i rway smooth muscle tolerance—affects responses not only to sa lmeterol or the pharmacokinetica l ly s imi lardrug, formoterol , but a lso to a lbuterol or s imi lar rapidly acting adrenergic bronchodi lators typica l ly used for rescue therapy.

Of course, for most drugs overcoming tolerance and restoring effects to a certa in intens i ty can be achieved by increas ing the dose, the dosefrequency, or both. This i s precisely what asthma patients who are poorly managed do, especia l ly with thei r rapidly acting rescue inhalers(a lbuterol , etc): four or more puffs at a time, many times each 24 hours . Thus , the “solution” to the tolerance, higher and more frequent doses ,actua l ly becomes the cause. And so the usual outcome of excess ive or excess ively prolonged adrenergic bronchodi lator use i s a fas ter onset andprogress ion of tolerance and a greater degree of tolerance. That i s , increas ing dose and/or frequency only makes matters worse over the long run.

So, patients taking one or two β2 agonis ts month after month gradual ly get less and less bronchodi lation and symptom rel ief over time; thei rbreathing di ffi cul ty becomes worse, and should severe bronchoconstric-tion occur, usual or even relatively high doses of a rapidly acting (rescue)bronchodi lator wi l l fa i l to work wel l , i f at a l l . Usual ly this wi l l be assessed, cl inica l ly, by diminishing improvement of forced expiratory volume in 1second (FEV1) in response to a rapidly acting adrenergic bronchodi lator (eg, a lbuterol ). Thus , the patient gets in severe venti latory dis tress or mayeven die. Thus , what might be ca l led “antiasthma” drugs can actua l ly lead to asthma-related fata l i ties under some conditions .

Answer a i s incorrect; antibodies aga inst sa lmeterol or other adrener-gic bronchodi lators do not develop. Answer b i s not acceptable. Al thoughhigh doses of a so-ca l led “selective β2 agonis t” may increase blood pressure (at high blood levels selectivi ty for β2 receptors i s los t and β1

receptors can be activated, ra is ing blood pressure via increases of both heart rate and left ventricular contracti l i ty), a s igni ficant ri se of pressure i svery unl ikely; hypertens ive cri s i s wi l l not occur. Sa lmeterol or other adrenergic broncho-di lators have no di rect or major effects on corti solsynthes is , release, or actions (c), nor do they have antimuscarinic effects , whether on mucus viscos i ty, a i rway smooth muscle tone, or any otherprocesses mediated by muscarinic (chol inergic) receptors .

309. The answer is c. (Brunton, pp 138t, 1040-1044; Katzung, pp 344-346.) Theophyl l ine (or the related drug aminophyl l ine) i s class i fied as amethylxanthine, just l ike the caffeine and other methylxanthines in your favori te coffee, tea , cola , or chocolates . So to answer this question,s imply imagine what i s l ikely to happen i f you consumed, say, eight cups of s trong caffeinated coffee in a short time. You’re typica l ly wide awake(i f not s imply “hyper”), tachycardic, urinating (from cardiac-mediated increased renal perfus ion and GFR), and tremorous . Methylxanthines are CNSstimulants , not just bronchodi lators or cardiac s timulants . In overdoses , i rri tabi l i ty or increased a lertness and awakeness develop fi rs t. As bloodlevels ri se, ul timately seizures occur, and that i s one of the major things we would see i f this chi ld took a sufficient overdose. Methylxanthinesdon’t cause urinary retention (a); as noted above, increased GFR wi l l lead to more frequent diures is . Bradycardia and hypotens ion (b) are theoppos i te of what i s apt to occur in the cardiovascular system. Methylxanthines induce nei ther platelet aggregation (they weakly inhibi t i t via a c-AMP-dependent mechanism) nor i schemic s troke because of i t. If anything, ri ses of blood pressure from excess ive cardiac s timulation might(rarely) cause hemorrhagic s troke. Methylxanthines , of course, s timulate centra l venti latory control mechanisms, tending to cause hyperpnea.(Caffeine has been used as a venti latory s timulant, mainly in newborns .) That, in turn, would l ikely cause respiratory a lka los is , not acidos is (e).Apnea i s not l ikely.

Page 180: Pharmacology pre test self assessment and review (14th ed.)

Autacoids and Anti-inflammatory Drug Pharmacology AcetaminophenArachidonic acid metabol i smErgot a lka loidsHis tamine and i ts antagonis tsLeukotriene receptor blockersMast cel l “s tabi l i zers”Nonsteroida l anti -inflammatory drugs (cyclooxygenase inhibi tors )Prostaglandins and related eicosanoidsPurine metabol i sm, hyperuricemia, goutSerotonin agonis ts and antagonis ts

Questions

310. About 10% to 20% of asthma patients aged 20 years to 30 years develop wheezing or more severe respiratory s igns and symptoms in responseto a certa in class of drugs . The offending drugs indi rectly shunt (redirect) arachidonic acid metabol i sm to the formation of leukotrienes , whichhave bronchoconstrictor and proinflammatory properties . To which one of the fol lowing drugs or drug groups does this explanation most l ikelyapply?

a. Acetylchol inesterase inhibi tors (eg, neostigmine)b. β-Adrenergic blockers (propranolol , many others )c. His tamined. Nonsteroida l anti -inflammatory drugs (tradi tional agents such as aspi rin)e. Opioids such as morphine

311. Febuxostat i s the fi rs t new drug in i ts class in four decades . It i s indicated for managing hyperuricemia, including that which i s drug-induced,but i s not indicated for symptom-suppress ion during an acute gout attack. Its bas ic mechanism of action i s inhibi tion of purine degradation at theformation of hypoxanthine (HX), preventing further metabol i sm of HX to xanthine and uric acid. It should not be adminis tered to patients receivingpurine antimetabol i tes (eg, mercaptopurine, thioguanine, for certa in cancers ) because the metabol ic inactivation of those anticancer drugs wi l lbe inhibi ted and toxici ty may occur. Based on the description given, what drug i s most s imi lar to febuxostat?

a. Al lopurinolb. Aspirinc. Colchicined. Indomethacine. Probenecid

312. A patient receives a drug that inhibi ts the l ipoxygenase pathway of arachidonic acid metabol i sm. What natura l ly occurring metabol i te(s ) wi l lbe produced in lesser amounts in response to this drug?

a. Leukotrienesb. Prostacycl in (PGI2)

c. Prostaglandinsd. Thromboxanese. Uric acid

313. A patient has mi ld cutaneous and systemic mani festations of a seasonal a l lergic response. Before you prescribe a short course ofdiphenhydramine for symptom rel ief, you should rea l i ze that this drug has one mechanism of action resembl ing, causes many s ide effects s imi larto, and shares many contra indications , that apply to an “autonomic” drug which you should be very fami l iar. What i s that drug?

a. Atropineb. Bethanecholc. Norepinephrined. Phentolaminee. Physostigminef. Propranolol

314. A patient presents with a his tory of frequent and severe migra ine headaches . When we give one of the more commonly used drugs forabortive therapy, sumatriptan, upon which of the fol lowing “loca l control substances” i s i t mainly acting?

a. His tamineb. PGF2α

c. Prostacycl ind. Serotonine. Thromboxane A2

315. A male patient with severe arthri ti s wi l l be placed on long-term therapy with indomethacin. You recognize the ri sk of NSAID-inducedgastrointestina l ulceration, and so want to prescribe another drug for ulcer prophylaxis . Which drug would you most l ikely choose, as an add-on to

Page 181: Pharmacology pre test self assessment and review (14th ed.)

indomethacin, assuming that there are no speci fic contra indications to i ts use?

a. Celecoxibb. Cimetidinec. Diclofenacd. Diphenhydraminee. Misoprostol

316. Aspi rin causes s igni ficant bronchoconstriction and bronchospasm in a patient who was subsequently identi fied as being “aspirin-sens i tive.”Which mechanism summarizes best why or how aspirin provoked the respiratory problems in this patient?

a. Induced drug-mediated hypersens i tivi ty of H1 receptors on a i rway smooth muscles

b. Induced drug-mediated hypersens i tivi ty of muscarinic receptors on a i rway smooth musclesc. Enhanced formation of antibodies di rected aga inst the sa l icylate on a i rway mast cel l sd. Inhibi ted synthes is of endogenous prostaglandins that have bronchodi lator activi tye. Reduced (blocked) epinephrine binding to β2-adrenergic receptors on a i rway smooth muscle cel l s

317. A chi ld takes what comes close to being a letha l dose of acetaminophen. What i s the most l ikely pathology that ul timately can lead to deathinvolved in this drug overdose?

a. Acute nephropathyb. A-V conduction dis turbances , heart blockc. Liver fa i lured. Status asthmaticuse. Status epi lepticus

318. A patient with asymptomatic hyperuricemia i s s tarted on probenecid. In a couple of days , he develops acute gout. What i s the mechanism bywhich probenecid, early in the course of treatment with i t, triggered this acute gouty arthri tic episode?

a. Accelerated synthes is of uric acidb. Co-precipi tated with uric acid in the jointsc. Induced an idiosyncratic response that triggered urate precipi tationd. Induced systemic acidos is , favoring uric acid crysta l l i zatione. Reduced renal excretion of uric acid

319. Epidemiologic data publ i shed in the early 1980s found a probable relationship between aspirin and a serious adverse response when thedrug was adminis tered to chi ldren with influenza, chickenpox, and other vi ra l i l lnesses . Al though no causa l relationship between aspirin, vi ra li l lness , and this adverse effect was proven, you s ti l l caution the parents or caregivers of chi ldren to avoid giving any aspi rin or aspi rin-conta iningproduct when there i s even a remote poss ibi l i ty that a chi ld’s i l lness might involve a vi rus . Avoidance of aspi rin i s necessary to avoid whichspeci fic adverse response?

a. Asthmab. Cardiomyopathyc. Renal fa i lured. Reye syndromee. Thrombocytopenia and related bleeding disorders

320. Such drugs as methotrexate, hydroxychloroquine, or penici l lamine are often turned to for managing rheumatoid arthri ti s that i s not control ledadequately with “tradi tional” NSAIDs (eg, aspi rin, ibuprofen, diclofenac, or indomethacin). Which s tatement most correctly summarizes how thosedrugs di ffer from a typica l NSAID?

a. Activate the immune system to neutra l i ze inflammatory mediatorsb. Are primary therapies for gouty arthri ti sc. Are remarkably free from serious toxici tiesd. Provide much quicker rel ief of arthri ti s s igns , symptomse. Slow, s top, poss ibly reverse joint pathology in rheumatoid arthri ti s

321. For qui te a whi le the “coxibs” (selective COX-2 inhibi tors ) were prescribed in preference to nonselective cyclooxygenase inhibi tors formanaging such conditions as rheumatoid arthri ti s . Now we have bas ica l ly one drug in this class , celecoxib. Which s tatement best describes anaction or property of this COX-2 inhibi tor?

a. Associated with a lower ri sk of gastric or duodenal ulcerationb. Cures arthri ti s , rather than just give symptom rel iefc. Effectively inhibi ts uric acid synthes isd. Has a lower ri sk of adverse or fata l cardiac eventse. Has s igni ficantly fas ter onset of action

322. Glucocorticoids are widely used for a host of inflammatory reactions and the diseases they cause. In terms of inflammatory responses andunderlying metabol ic reactions , which enzyme or process i s the main target of these drugs when they are given at pharmacologic(supraphys iologic) doses?

a. Cyclooxygenases (COX-1 and -2)b. His tidine decarboxylase

Page 182: Pharmacology pre test self assessment and review (14th ed.)

c. 5′-l ipoxygenased. Phosphol ipase A2 (PLA2)

e. Xanthine oxidase

323. Bradykinin plays important roles in loca l responses to ti ssue damage and a variety of inflammatory processes . It a l so has vasodi lator activi ty.Which s tatement i s correct about this endogenous peptide?

a. Captopri l inhibi ts i ts metabol ic inactivationb. Drugs that are metabol i zed to, or generate, ni tric oxide, counteract bradykinin’s vascular effectsc. Increased blood pressure i s the predominant cardiovascular responsed. Newer his tamine H1 blockers (eg, fexofenadine; “second-generation” antihis tamine) a lso competi tively block bradykinin receptors

e. The main renal responses to endogenous bradykinin are arteriolar constriction and reduced GFR

324. A newborn has oxygenation and hemodynamic problems because of a patent (open) ductus arteriosus . Which drug usual ly would beadminis tered in an attempt to close the les ion?

a. Cimetidineb. Diphenhydraminec. Indomethacind. Misoprostole. Prostaglandin E1 (PGE1; a lprostadi l )

325. A 29-year-old woman develops frequent, debi l i tating migra ine headaches . Sumatriptan i s prescribed for abortive therapy. Not long aftertaking the drug she i s rushed to the hospi ta l . Her vi ta l s igns are unstable, and she has muscle rigidi ty, myoclonus , genera l i zed CNS i rri tabi l i ty andal tered consciousness , and shivering. You learn that for severa l months she had been taking another drug with which the triptan interacted. Whichadd-on drug most l ikely caused these acute and serious problems?

a. Acetaminophenb. Codeinec. Diazepamd. Fluoxetinee. Phenytoin

326. A patient (assume he or she i s taking no other drugs) has been taking doses of aspi rin that are too high for severa l weeks . Low-grade aspirintoxici ty (sa l icyl i sm) develops . What s ign or symptom would be cons is tent with sa l icyl i sm and the high sa l icylate levels that caused i t?

a . Constipationb. Coughc. Hypertens iond. Myopiae. Tinni tus

327. Aspi rin genera l ly should be avoided as an anti -inflammatory, ana lges ic, or antipyretic drug by patients with hyperuricemia or gout. That i sbecause i t counteracts the effects of one important drug the hyperuricemic patient may be taking. Which drug has i ts des i red uric acid-relatedeffects reduced or el iminated by this prototypic NSAID?

a. Acetaminophenb. Al lopurinolc. Colchicined. Indomethacine. Probenecid

328. Misoprostol , an analog of PGE1, i s sometimes used adjunctively to s timulate gastric mucus production and help reduce the incidence ofgastric ulcers associated with long-term or high-dose NSAID therapy for arthri ti s . What i s the other main use for this l ipid-derived autacoid?

a. Closure of a patent ductus arteriosus in newbornsb. Contraception in women who should not receive estrogens or progestinsc. Induction of abortion in conjunction with mifepris tone (“RU486”)d. Prophylaxis of as thma in l ieu of a corticosteroide. Suppress ion of uterine contracti l i ty in women with premature labor

329. A patient who was transported by ambulance to the Emergency Department took a potentia l ly letha l overdose of aspi rin. What drug would bea helpful , i f not l i fesaving, adjunct to manage this severe aspi rin poisoning?

a. Acetaminophenb. Amphetamines (eg, dextroamphetamine)c. N-acetylcysteined. Phenobarbi ta le. Sodium bicarbonate

330. We look at data that summarize the actions of two prototypic his tamine receptor blockers : diphenhydramine as the exemplar of the olderantihis tamines (competi tive his tamine receptor antagonis ts ); and fexofena-dine as a representative agent of the “second-generation”

Page 183: Pharmacology pre test self assessment and review (14th ed.)

antihis tamines . Which s tatement correctly describes , compares , or contrasts these drugs or the pharmacologic groups to which they belong?

a. Diphenhydramine and other drugs in i ts class (ethanolamines) tend to cause drows iness more often than fexofenadine and related second-generation antihis tamines

b. Diphenhydramine i s a preferred antihis taminic for patients with prostate hypertrophy or angle-closure glaucoma, whereas fexofenadine andrelated drugs are contra indicated

c. Diphenhydramine overdoses tend to cause bradycardia , whereas fexofenadine overdoses tend to cause s igni ficant increases of heart rated. Fexofenadine and related second-generation his tamine antagonis ts have intrins ic bronchodi lator activi ty, which makes them sui table as

primary/sole therapy for people with asthmae. Fexofenadine, and drugs in i ts class , have better efficacy in terms of suppress ing his tamine-mediated gastric acid secretion

331. A patient has been taking one of the drugs l i s ted below for about 4 months , and i s experiencing the des i red therapeutic effects from i t. Thephys ician now prescribes indomethacin to treat a particularly severe flare-up of rheumatoid arthri ti s . Within a matter of days the therapeuticeffects of the fi rs t drug wane dramatica l ly, i ts actions antagonized by the indo-methacin. Which drug was affected by the indomethacin?

a. Al lopurinol , given for prophylaxis of hyperuricemiab. Captopri l , given for essentia l hypertens ionc. Fexofenadine, given for managing seasonal a l lergy responsesd. Sumatriptan, given for abortive therapy of migra ine headachese. Warfarin, given for prophylaxis of venous thrombos is

332. A patient takes an acute, mass ive overdose of aspi rin that, without proper intervention, wi l l probably be fata l . What would you expect to occurin the advanced (late) s tages of aspi rin (sa l icylate) poisoning?

a. Hypothermiab. Metabol ic a lka los isc. Respiratory a lka los isd. Respiratory plus metabol ic acidos ise. Venti latory s timulation

333. A patient presents in the emergency department with an overdose of a drug. The phys ician knows what the drug i s , and so orders appropriate“symptomatic and supportive care,” plus multiple doses of N-acetylcysteine. Upon which drug did the patient overdose?

a. Acetaminophenb. Aspirinc. Colchicined. Diphenhydraminee. Loratadinef. Methotrexate

334. A patient has acute gout. The phys ician ini tia l ly thinks about prescribing just one or two ora l doses of colchicine, 12 hours apart, but thendecides otherwise. What i s the main and most common reason for avoiding colchicine, even with a very short ora l course, and prescribing anotherdrug for the acute gout instead?

a. Bone marrow suppress ionb. Bronchospasmc. GI dis tress that i s a lmost as bad as the acute gout discomfortd. Hepatotoxici tye. One or two ora l doses seldom rel ieve gout pa inf. Refractoriness/tolerance with just a dose or two

335. A patient with hyperuricemia i s placed on an “antigout” drug. Before s tarting the drug you measure the tota l uric acid (amount, notconcentration) in a 24-hour urine sample and then do the same severa l weeks after continued drug therapy at therapeutic doses . The post-treatment sample shows a s igni ficant reduction in urate content. There were no new pathologies developing during therapy, and the patient’sda i ly purine intake did not change at a l l . What drug most l ikely accounted for these findings?

a. Acetaminophenb. Al lopurinolc. Colchicined. Indomethacine. Probenecid

336. We adminis ter sodium bicarbonate to a patient who has severe hyperuricemia and i s at great ri sk of developing urate s tones in his urinarytract. What i s the bas is for us ing the bicarbonate?

a. Causes antidiuretic effect, reducing urate content of the urineb. Causes metabol ic a lka los is that inhibi ts cata lytic activi ty of xanthine oxidasec. Counteracts metabol ic acidos is that i s characteris tic of severe hyperuricemiad. Lowers urate el imination by lowering glomerular fi l tration ratee. Reduces solubi l i ty of urate in the urine

337. A patient with annoying hay fever (seasonal a l lergy) symptoms (mainly rhinorrhea) goes to the s tore, intent on purchas ing an ora l medication

Page 184: Pharmacology pre test self assessment and review (14th ed.)

to make them more comfortable. They see the prices for loratadine and related second-generation antihis tamines and are shocked about howhigh they are. Nearby on the shel f they see s tore-brand a l lergy rel ief pi l l s that are much less expens ive, the cheapest ones havingdiphenhydramine as the sole active ingredient. If they were to take ful l therapeutic doses of the diphenhydramine, based on label di rections ,which effects or s ide effects are they most l ikely to experience as an additional price to be pa id for thei r a l lergy rel ief?

a . Bradycardiab. Diarrheac. Drows iness , somnolenced. “Heartburn” from increased gastric acid secretione. Urinary frequency

338. Your patient has rheumatoid arthri ti s that has been largely refractory to diclofenac, ibuprofen, indomethacin, and sul indac. In addition, shehas experienced s igni ficant GI dis tress , and severa l GI bleeds , fol lowing attempts to get better anti -inflammatory effects by ra is ing the doses . Westart her on therapy with etanercept. What i s the most l ikely mechanism by which etanercept suppresses the s igns , symptoms, or underlyingpatho-phys iology of rheumatoid arthri ti s?

a . Inhibi ts eicosanoid synthes is by inhibi ting phosphol ipase A2

b. Inhibi ts leukocyte migration by blocking microtubular formationc. Neutra l i zes ci rculating tumor necros is factor (TNF-α)d. Selectively and effectively inhibi ts COX-2e. Stimulates col lagen and mucopolysaccharide synthes is in the joints

Page 185: Pharmacology pre test self assessment and review (14th ed.)

Autacoids and Anti-inflammatory Drug Pharmacology

Answers

310. The answer is d. (Brunton, pp 973t, 974, 1031-1032; Katzung, pp 320, 638, 639f.) Arachidonic acid i s a by-product of phosphol ipid metabol i sm (viaphosphol ipases) in most cel l s . Reca l l that there are two main pathways for further arachidonic acid metabol i sm: (1) via the cyclooxygenasepathway; and (2) via the l ipoxygenase pathway. Tradi tional NSAIDs , including aspi rin, inhibi t both cyclooxygenase-1 and -2. In those patients withwhat i s generica l ly ca l led “aspirin-sens i tive” asthma (a lso ca l led tra id or Sampter asthma), the working hypothes is i s that when arachidonatemetabol i sm via COX i s inhibi ted, arachidonic acid must s ti l l be metabol i zed. That occurs via increased metabol i sm through the l ipoxygenasepathway. This resul ts in greater formation of leukotrienes that cause bronchoconstriction and additional a i rway inflammation.

Acetylchol inesterase inhibi tors (a ) clearly can provoke severe bronchospasm in asthmatic patients . That i s a di rect effect, apparently involvingincreased a i rway smooth muscle reactivi ty to a host of bronchoconstrictor substances , one of which i s acetylchol ine. The response i s independentof leukotriene metabol i sm. The same appl ies to β-adrenergic blockers (b). In thei r case, a i rway problems arise from inhibi tion of important β-2-mediated bronchodi lator effects of epinephrine. His tamine’s effects (c) are di rect and do not involve arachidonic acid metabol i sm, but there i s nodoubt that his tamine i s a particularly powerful bronchoconstrictor in patients with asthma. Opioids (e) may cause excess ive bronchoconstriction insome asthma patients . That i s a lmost certa inly due to morphine’s abi l i ty to release his tamine from mast cel l s and basophi l s . It has nothing to dowith speci fic inhibi tion of COX or l ipoxygenase pathways of arachidonic acid metabol i sm.

311. The answer is a. (Brunton, pp 997-998; Katzung, pp 653-654.) The description of febuxostat in the question—actions , uses , drug interactions andmore—appl ies di rectly to a l lopurinol , which i s class i fied as a xanthine oxidase inhibi tor and has been ava i lable for decades . As you shouldreca l l , a l lopurinol (and now febuxostat) i s indicated for managing hyperuricemia, whether idiopathic, associated with certa in diseases (includingmany mal ignancies ), or drug-induced (eg, by thiazide or loop diuretics , and cancer chemotherapeutic agents ). Aspirin (b) has no effects on purinemetabol i sm in genera l or speci fica l ly on xanthine oxidase. At a l l but very high dosages , i t can ra ise plasma uric acid (and xanthine) levels byinhibi ting tubular secretion of uric acid, and i s genera l ly contra indicated for patients with hyperuricemia or gout. Colchicine (b) i s an anti -inflammatory drug used for acute gout, or for prophylaxis , but i t has no effect on endogenous purine metabol i sm or the metabol i sm of purineantimetabol i tes such as mercaptopurine. Indomethacin (d) i s a tradi tional NSAID (tNSAID) that has many indications , including treatinginflammation secondary to gout, but once aga in i t shares none of the other properties noted in the question. Probenecid (e) can be cons idered theprototype uricosuric drug: indicated for prophylaxis , but not for treatment, of gout. Therapeutic blood levels of probenecid lower plasma uric acidlevels by blocking tubular reabsorption of urate.

312. The answer is a. (Brunton, pp 939-942; Katzung, pp 349-350, 352.) The arachidonic acid (AA) “cascade” begins with AA synthes is from membranephosphol ipids by phosphol ipase A2. Once arachidonic acid i s formed, the metabol ic pathways diverge into the l ipoxygenase pathway (that formscysteinyl leukotrienes [LTs ]) and the cyclooxygenase pathways that synthes ize prostaglandins , including prostacycl in and thromboxane A2. Oneimportant intermediate early on in the LP pathway i s LTA4. From that we get LTB4, which mainly regulates chemotaxis (cytokine activi ty) andactivates phagocytos is in white blood cel l s . Also derived from LTB4 are, sequentia l ly, LTC4, LTD4, and LTE4 (cysteinyl leukotrienes , his torica l ly andcol lectively ca l led SRS-A, for s low-reacting substance of anaphylaxis ), which cause mainly bronchoconstriction. Leukotriene synthes is can beinhibi ted by zi leuton (no longer used cl inica l ly), a 5′-l ipoxygenase inhibi tor. The leukotriene receptors can be blocked by montelukast andzafi rlukast, which are used prophylactica l ly (“control meds”) to suppress a i rway inflammation and bronchoconstriction in asthma. They should notbe used in l ieu of corticosteroids (ora l or inhaled) when control of a i rway inflammation i s necessary.

Prostaglandins (c), including prostacycl in (b), and thromboxane A2 (d) are synthes ized via the cyclooxygenase pathways . Uric acid (e) i ssynthes ized by what i s genera l ly referred to as the ATP (purine) degradation pathway, in which the fina l two s teps in the pathway (convers ion ofhypoxanthine to xanthine, and convers ion of xanthine to uric acid) are cata lyzed by xanthine oxidase.

313. The answer is a. (Brunton, pp 921t, 918-924, 1816; Katzung, p 292.) Diphenhydramine, an older or “fi rs t-generation” antihis tamine, i s a competi tiveantagonis t of his tamine’s effects on H1 receptors , and a lso s trongly blocks muscarinic chol inergic receptors . Other older H1 blockers (but none ofthe second-generation H1 blockers , such as fexofenadine) possess this atropine-l ike effect, often to a lesser degree than diphenhydramine. (Ifyou wished to argue that because diphenhydramine was more l ike scopolamine, because i t causes not only antichol inergic effects but a lso acons iderable degree of sedation, you would be correct.) Thus , common s ide effects related to antimuscarinic activi ty include sedation, dry mouth,photophobia , and cycloplegia (para lys is of accommodation). Key contra indications include prostatic hypertrophy, bowel or bladder obstruction orhypomoti l i ty, tachycardia , and narrowangle (angle-closure) glaucoma.

To refresh your memory about the other drugs l i s ted, bethanechol (b) i s a muscarinic agonis t, norepinephrine (c) i s an effective agonis t for β1-and α-adrenergic receptors , phentolamine (d) i s a nonselective α blocker, physostigmine (e) i s an acetylchol inesterase inhibi tor, and propranolol(f) i s , of course, the prototypic nonselective β-adrenergic blocker.

314. The answer is d. (Brunton, pp 344-347; Katzung, pp 285, 292.) Acute migra ine therapy often involves giving a drug that mimics the effects ofendogenous serotonin (5-hydroxytryptamine), thereby revers ing the cerebra l vasodi lation that contributes s igni ficantly to migra ine s igns andsymptoms. A good example i s sumatriptan (member of a smal l group of drugs ca l led triptans). These are 5-HT1B/2D receptor agonis ts . His tamine(a), and to a lesser extent prostacycl in (PGI2, answer c), are vasodi lators , too, but they don’t seem to have appreciable roles in migra ine, nor arethey targets of antimigra ine drug activi ty. Activation of β2-adrenergic receptors in the cerebra l vasculature a lso leads to vasodi lation and migra inesymptoms. The β-adrenergic blockers (particularly propranolol ) are useful for some migra ineurs , but only for prophylaxis , not for abortive therapy.

315. The answer is e. (Brunton, pp 952, 1837, 1849; Katzung, pp 315, 317, 1090.) Misoprostol i s a long-acting synthetic analog of PGE1, and i ts only use(outs ide of reproductive medicine) i s prophylaxis of NSAID-induced gastric ulcers . Its main effects are suppress ion of gastric acid secretion(modest); and, more importantly, enhanced gastric mucus production (a so-ca l led mucotropic or cytoprotective effect). The need for the drug arises ,of course, because such drugs as indomethacin (and most other COX-nonselective inhibi tors ) inhibi t PGE1 synthes is (as wel l as that of otherprostaglandins , prostacycl in, and thromboxane A2).

Al though cimetidine (b) might help reduce gastric acid secretion (via H2 blockade) and diphenhydramine (c) may too (via antimuscarinic effects ),thei r antisecretory effects (with respect to gastric acid) are weak, and they don’t increase formation of the s tomach’s protective mucus .

Celecoxib (a), a COX-2 selective NSAID, i s associated with a lower ri sk or incidence of GI ulcers than more tradi tional COX-1/-2 NSAIDs , including

Page 186: Pharmacology pre test self assessment and review (14th ed.)

indomethacin (or diclofenac, ibuprofen, many others ). However, adding a COX-2 inhibi tor would be i rrational in terms of reducing ulcer ri sk.Diclofenac (c) i s a tradi tional NSAID that, when used long-term (eg, as an a l ternative to indomethacin or any of the other reasonable options), maycause ulcers just as indomethacin may. In fact, a fixed-dose combination product conta ining diclofenac and misoprostol i s ava i lable, the latterdrug being used to reduce the ri sk of ulcers caused by the former.

316. The answer is d. (Brunton, pp 937-952; Katzung, pp 638-640.) Inhibi ted synthes is of prostaglandins that are bronchodi lators (mainly PGE2) arethought to be respons ible for severe or fata l responses to aspi rin in some asthmatics . Note that aspi rin and other tradi tional NSAIDs inhibi t thesynthes is of PGF2α and of TXA2, both of which are bronchoconstrictors . As a resul t, one would predict reduced bronchoconstriction with aspi rin.However, in those patients with aspi rin-sens i tive asthma (about 20% of adul t as thmatics overa l l ), the adverse effects ari s ing from inhibi ted PGE2

synthes is tend to predominate, and so aspi rin and other efficacious NSAIDs are genera l ly contra indicated.Aspirin does not cause increased respons iveness of a i rway smooth muscle cel l s to the bronchoconstrictor effects of his tamine (a) or muscarinic

agonis ts (b). It does not el ici t antibody formation (c) or have any effect on the interaction between β2 agonis ts and their cel lular receptors (e).

317. The answer is c. (Brunton, pp 76, 128, 136; Katzung, p 1032.) The primary cause of death from acetaminophen overdoses i s hepatic necros is . Thedrug’s main toxic metabol i te i s N-acetyl -benzoqinoneimine. It reacts with sul fhydryl groups that are consti tuents of key macromolecules andmetabol ic cofactors (eg, glutathione) in the l iver. If acetaminophen doses are low, glutathione conjugates the metabol i te. With toxic doses ,however, glutathione i s depleted and other—SH groups on hepatocyte proteins are attacked and i rrevers ibly a l tered. Concomitant with overa l lhepatic damage we find, eventual ly, profound hypoglycemia (as the l iver’s s tores of glycogen are depleted) and coagulopathies (as hepaticclotting factor synthes is s tops and the patient begins bleeding spontaneous ly).

Nephropathy (a) i s not a primary or important consequence of acetaminophen toxici ty. If AV conduction dis turbances , heart block, or othercardiac electrophys iologic anomal ies (b) occur, they are secondary to hepatic dys function. Status asthmaticus (d) and acute seizures (e) are notl ikely or di rect consequences of acetaminophen poisoning.

Note: Let’s summarize the main events or “s tages” (they’re arbi trary) of what otherwise would be a letha l dose of acetaminophen: for anotherwise-heal thy chi ld, i t i s around 12 g taken a l l at once; for otherwise heal thy adults i t i s around 25 g. Various i l lnesses , and consuming otherdrugs (including aspi rin) can dramatica l ly lower the dose sufficient to cause serious or letha l toxici ty. A “s tandard s trength” acetaminophen tabletconta ins 300 mg. Be aware that acetaminophen toxici ty i s the most common drug cause of l iver fa i lure in the United States . And, most of thosecases are not caused by suicide attempts in which the drug i s taken a l l at once. More often, the toxici ty develops gradual ly, wi th excess iveacetaminophen use over time. A major contributor to that problem is that acetaminophen i s found in so many products , most ava i lable OTC, andthe consumer s imply i sn’t aware that he or she i s ul timately taking many doses of a product that conta ins this hepatotoxin.

Page 187: Pharmacology pre test self assessment and review (14th ed.)
Page 188: Pharmacology pre test self assessment and review (14th ed.)

318. The answer is e. (Brunton, p 999; Katzung, p 656.) It has been sa id that the ini tia l phase of uricosuric therapy i s the most worrisome period for ahyperuricemic patient. Probenecid i s a uricosuric drug (one that increases renal el imination of uric acid), but that effect i s highly dose-dependent.Des i red uricosuric actions depend on having therapeutic probenecid blood levels that are sufficient to inhibi t active tubular reabsorption of urate.At subtherapeutic blood levels , the main effect i s inhibi tion of tubular secretion of urate, which reduces net urate excretion and ra ises plasmaurate levels (sometimes to the point of caus ing cl inica l gout). It i s only once probenecid levels are therapeutic that the des i red effects to inhibi ttubular urate reabsorption (ie, increase excretion) predominate. Thus , and intui tively, once a patient s tarts probenecid therapy drug levels mustri se and pass through that s tage in which urate excretion wi l l actua l ly go down, plasma urate levels up.

Some texts suggest us ing a short course of colchicine or another (non-aspirin) NSAID that i s indicated for gout when probenecid therapy i ss tarted. That i s for prophylaxis of acute gout that might occur. Al though this may be acceptable, other rules are perhaps more important: (1) do notadminis ter a uricosuric during a gout attack; (2) i f the patient has had a gout attack recently, suppress the inflammation for 2 to 3 months with asui table anti -inflammatory; and (3) do not use uricosurics for patients with “severe hyperuricemia” and/or poor renal function. Doing otherwise i sassociated with a great ri sk of potentia l ly severe renal tubular damage as the uricosuric shi fts large amounts of uric acid from the blood (with i tslarge volume, that keeps urate relatively “di lute”) into a smal l volume of acidic urine, which concentrates urate and lowers i ts solubi l i ty via pH-dependent mechanisms.

(The patient who skips doses of probenecid a lso becomes very vulnerable to the “paradoxica l” extra ri sk, because doing this may a l low druglevels to fa l l into that subtherapeutic range in which more urate i s reta ined than el iminated.)

319. The answer is d. (Brunton, pp 975-976.) Before the warning was i ssued, over 500 chi ldren with any vi ra l i l lness and who received aspirin (even inusual recommended doses at the time) developed Reye syndrome. It i s characterized by encephalopathy and fatty l iver degeneration. Themorta l i ty rate ranged from about 20% to 30%, and many more patients developed permanent and serious CNS and/or l iver dys function. Nowadaysthere are fewer than five cases of Reye syndrome per year as a resul t of the aspi rin/vi ra l i l lness “interaction.” (Note: the age range for whichaspirin should not be adminis tered in the presence of vi ra l i l lness i s debatable; some suggest the ri sk of Reye syndrome lessens dramatica l ly bythe age of 12 years , but many experts suggest that the ri sk may pers is t up to 18-20 years of age.)

Asthma (a) i s the most common cause of hospi ta l i zations in chi ldren, and many patients have their as thma provoked or worsened by aspi rin orother NSAIDs . However, the potentia l as thma—NSAID problems are unrelated to the presence or absence of vi ra l i l lnesses . There i s no evidenceof a l inkage between aspirin and cardiomyopathies (b). Long-term high dose adminis tration of NSAIDs (other than aspirin) may cause renaldys function or fa i lure (c). Here, too, there i s no l inkage between vi ra l i l lness , and the kidney problems are not at a l l focused on the pediatricpopulation. Aspirin obvious ly has antiplatelet-aggregatory effects , but whether the patient i s a chi ld or older, thrombocytopenia (e) i s not aconcern.

320. The answer is e. (Brunton, p 960; Katzung, pp 643-648.) Methotrexate, gold sa l ts , and penici l lamine are members of a diverse group of drugs ca l ledDMARDs (disease-modi fying anti rheumatic drugs) or SAARDs (s low-acting anti rheumatic drugs). The former term derives from the abi l i ty of thesedrugs to s low, s top, or in some cases reverse joint damage associated with rheumatoid arthri ti s (RA). They do more than merely mask or rel ieve RAsymptoms, which i s mainly what the tradi tional NSAIDs do. The second acronym derives from the fact that i t may take a month (or a couple more)for meaningful symptom rel ief to develop; they are not at a l l quick-acting drugs . Their actions probably are due to suppress ion of immuneresponses that often contribute to the etiology of RA.

Their toxici ties can be serious , which i s one reason why, unti l not long ago, these agents were cons idered thi rd-l ine or even las t resorttreatments for refractory rheumatoid disease. (Methotrexate i s now being used much earl ier, and safely, for RA, now that we know better how touse i t and monitor for serious toxici ties .) Most of these drugs can cause serious blood dyscras ias ; in addi tion, penici l lamine can cause renal andpulmonary toxici ty; hydroxychloroquine i s associated with vis ion impairments/retinopathy.

321. The answer is a. (Brunton, pp 941-942, 964-965; Katzung, p 640.) Celecoxib, by vi rtue of selective COX-2 inhibi tion, does not interfere as much withsynthes is of PGE2, which normal ly suppresses a component of gastric acid secretion and s timulates gastric mucus production. Overa l l , then, therisks of gastric and duodenal ulcers are reduced. The selectivi ty a lso means that COX-2 inhibi tors do not interfere with the production of othereicosanoids , such as TXA2. That i s both good and bad, cl inica l ly. On the good s ide, this means that COX-2 inhibi tors don’t cause antiplatelet effectsand increase the ri sk of excess ive or spontaneous bleeding. On the other hand, this lack of effect renders them unsui table for caus ing des i redantiplatelet-aggregatory effects , as might be wanted when we adminis ter aspi rin. This may expla in the finding that some COX-2 inhibi tors havebeen associated with (cause?) a higher ri sk of sudden cardiac death (d) in vulnerable patients , and why some have been pul led off the market(and are subjects of cons iderable l i tigation).

Selective COX-2 inhibi tors , such as the nonselective a l ternatives , aren’t cures for arthri ti s (b); they a l leviate s igns and symptoms but seem tohave no demonstrable impact on the underlying pathophys iology. They have no effect on uric acid metabol i sm (c) or excretion. Their onsets ofaction are, overa l l , no faster (e) than those of a typica l NSAID.

322. The answer is d. (Brunton, pp 937-939; Katzung, pp 339-340, 347-348, 639f.) Anti -inflammatory doses of glucocorticosteroids inhibi t phosphol ipaseA2 activi ty. In doing so, they inhibi t arachidonic acid synthes is and, therefore, synthes is of a l l subsequent products of the cyclooxygenase andl ipoxygenase pathways , both of which originate with arachidonic acid. This action of glucocorticoids i s indi rect because their ini tia l or di rect effecti s induced synthes is of annexins (previous ly ca l led l ipocortins ), which are the moieties that di rectly inhibi t PLA2 activi ty. Glucocorticoids have nointrins ic or di rect inhibi tory effects on later s teps in AA metabol i sm, that i s , no di rect effects on cyclooxygenase or l ipoxygenase activi ty.

Cyclooxygenases (a) are inhibi ted by tradi tional NSAIDs such as aspi rin (nonselective COX-1 and -2 inhibi tors ), or the “coxibs” (celecoxib), whichrelatively selectively inhibi t COX-2. We have no cl inica l ly useful inhibi tors of his tamine synthes is (which involves his tidine decarboxylase activi ty;b). As noted elsewhere, zi leuton inhibi ts 5′-l ipoxygenase (c) and the subsequent formation of leukotrienes . Al lopurinol inhibi ts the synthes is ofxanthine and uric acid by inhibi ting xanthine oxidase (e). The same appl ies to the new a l lopurinol a l ternative, febuxostat.

323. The answer is a. (Brunton, pp 721-725, 932-933; Katzung, pp 184-185.) Bradykinin i s metabol i zed to biologica l ly inactive peptides by an enzyme thathas at least three names: angiotens in-converting enzyme (ACE; reca l l that the prototype ACE inhibi tor i s captopri l ), bradykininase, and kininase II .The name that i s used depends on the substrate. When the substrate i s angiotens in I , the enzyme is ca l led ACE. When the substrate i s bradykininwe ca l l i t ei ther bradykininase or kininase II .

Bradykinin, whether injected experimenta l ly or derived from endogenous sources (kininogens cleaved by speci fic proteases ca l led ka l l ikreins ),exerts s igni ficant vasodi lator effects that can lower systol ic and diastol ic blood pressures . Al though this may not be an important pressure-regulating mechanism in normotens ive individuals , i t probably i s in many (most?) patients with essentia l hypertens ion. Clearly, bradykinin doesnot increase blood pressure (c) or constrict the renal vasculature (e).

Reca l l that ACE inhibi tors lower blood pressure in many hypertens ive patients . One mechanism involves “preserving” bradykinin by inhibi ting

Page 189: Pharmacology pre test self assessment and review (14th ed.)

i ts enzymatic inactivation. Bradykinin a lso causes prerenal arteriolar vasodi lation and increases GFR, leading to diuretic effects .The peptide’s vascular effects are mediated by endothel ia l cel l -derived ni tric oxide, and they are enhanced (not counteracted, b) by other drugs

that cause vasodi lation by a ni tric oxide-related mechanism.Bradykinin receptor blockers prevent the peptide’s vasodi lator effects . However, no currently approved drugs , including the so-ca l led second-

generation antihis tamines such as fexofenadine (d) exert that effect.

324. The answer is c. (Brunton, p 966t; Katzung, p 641.) The ductus arteriosus in neonates may remain patent largely because of the vasodi lator effectsof endogenous PGE1, formed via the cyclooxygenase pathway. When the goal i s to close a patent ductus after bi rth we genera l ly use aprostaglandin synthes is (COX-1/-2) inhibi tor (indomethacin, his torica l ly; or ibuprofen more often nowadays). (Conversely, there are times whensurgica l procedures are required on a congenita l ly anomalous heart in newborns , and we want to keep the ductus open unti l surgery. In that case,a lprostadi l [PGE1, answer e] may be adminis tered.)

Misoprostol (d) i s a prostaglandin analog. Giving i t to a newborn with a patent ductus i s l ikely to keep the les ion open, not close i t. None of theH2 blockers (eg, cimetidine, a ) nor H1 blockers (diphenhydramine, b, and many others ) have any important effects on prostaglandin synthes is , norare they used ei ther to close a patent ductus arteriosus or to keep i t open for subsequent surgery.

325. The answer is d. (Brunton, pp 399, 405, 410-413, 1537; Katzung, pp 285f, 292, 533-537, 1161t.) This patient has what i s a lmost certa inly the serotoninsyndrome. The triptan “adds” serotonin to serotoninergic synapse, and serotonin’s neuronal reuptake wi l l be blocked by fluoxetine (or sertra l ine,others ), which i s class i fied as a selective serotonin reuptake inhibi tor (SSRI) antidepressant. When sumatriptan (or other triptans used formigra ine) i s added, rapid accumulation of serotonin and/or the triptan in the bra in synapses can occur.

The other drugs l i s ted (acetaminophen, a ; codeine, b; diazepam, c; and phenytoin, e) are not l ikely to interact with serotoninergic drugs .

326. The answer is e. (Brunton, pp 75, 83t, 977-982; Katzung, pp 62, 1034-1035.) Tinni tus , a long with a feel ing of dizziness or l ightheadedness , GI upset(including nausea and some pain or other discomfort, and diarrhea more so than constipation), and such visua l changes as blurred or diplopia(but not myopia , d), a l l are part of a low-grade aspirin “toxici ty” syndrome ca l led sa l icyl i sm. It i s not necessari ly worrisome (provided the phys icianintentional ly prescribed the drug at dosages l ikely to produce the syndrome), dangerous , or indicative of imminent and severe toxici ty. Indeed,some patients experience one or more s igns or symptoms of sa l icyl i sm in response to high (antiarthri tic) doses of aspi rin.

Constipation (a) or hypertens ion (c) are not causa l ly associated with sa l icylate adminis tration, regardless of the dose used. Cough (b) mayoccur, but that i s only l ikely to occur in patients who have “aspirin-sens i tive” asthma, and for those patients bronchoconstriction i s more l ikelythan cough to be the response.

327. The answer is e. (Brunton, p 999; Katzung, pp 656, 1160t.) Probenecid (and the related drug, sul finpyrazone) are class i fied as uricosurics : atsufficiently high (therapeutic) doses , they enhance the renal el imination of uric acid by inhibi ting tubular reabsorption of fi l tered urate. Aspirins igni ficantly impairs the uricosurics ’ actions .

Important note: Aspirin (given a lone) has blood level -dependent effects on urate el imination by the kidneys . At “low doses” perhaps up toabout 1 g/day, i t selectively inhibi ts tubular secretion of urate and so can ra ise plasma urate levels . At doses much higher than that (includingdoses sometimes prescribed for arthri ti s other than gout), the predominant effect (and the net, or overa l l , effect) i s uricosuria due to blockade oftubular reabsorption of urate (this effect i s greater than the drug’s inhibi tory effect on tubular secretion of urate). Nonetheless , aspi rin i s not usedas a uricosuric drug because the doses/blood levels needed to increase urate el imination are sufficiently high that they cause s igni ficant s ideeffects (eg, sa l icyl i sm; see Question 326) that don’t ari se with the tradi tional uricosurics such as probenecid.

Acetaminophen (a) has no appreciable or cl inica l ly useful effects on uric acid el imination or synthes is . Al lopurinol (b, and the new related drugfebuxostat) inhibi ts uric acid synthes is , but i ts xanthine oxidase-inhibi tory effects are not a l tered by aspi rin. However, s ince a l lopurinol i s used tolower plasma urate levels i t i s largely i rrational to use aspi rin at the same time. The same appl ies to colchicine (c) and indomethacin (d), both ofwhich are used for in acute gout and nei ther of which has i ts main biochemica l effects (suppress ion of inflammation) impaired by aspi rin.

328. The answer is c. (Brunton, pp 952, 1837, 1849; Katzung, pp 315, 327, 1090.) In addi tion to misoprostol ’s use as a cytoprotective/mucotropic drug forsome patients taking NSAIDs , misoprostol i s a l so used as an adjunct to mifepris tone to induce therapeutic abortion. This capi ta l i zes on theprostaglandin analog’s s trong uterine-s timulating effects (thus , answer e i s incorrect). The drug i s sometimes used to mainta in patency of anopen ductus arteriosus , not to close i t (a ). It i s not a contraceptive in the typica l sense, such as we would associate with estrogen-progestin ora lcontraceptives . Given the drug’s aborti facient effects , i t i s contra indicated for women who are pregnant, or wish to become pregnant (b).Misoprostol has bronchodi lator activi ty, but i t i s relatively weak and the drug i s not sui table as a substi tute for corticosteroids (d) or other typica lasthma medications .

329. The answer is e. (Brunton, pp 75-76, 86t, 983; Katzung, pp 1034-1035.) Adjunctive use of sodium bicarbonate (IV) can be an important adjunct tomanaging severe sa l icylate poisoning for two main reasons : (1) i t helps ra ise blood pH, which as s tated earl ier i s profoundly reduced frommetabol ic plus respi ratory acidos is ; (2) i t a lka l inizes the urine, which (via a pH-dependent mechanism, a la Henderson-Hasselbach) converts moreaspirin molecules into the ionized form in the tubules , thereby reducing tubular reabsorption of a substance we want to el iminate from the bodyas quickly as poss ible.

Acetaminophen (a), even though i t i s usual ly an effective antipyretic, i s not good for managing fever of severe aspi rin poisoning. It would addyet another drug that might compl icate the cl inica l picture, and ordinary (and ordinari ly safe) doses aren’t l ikely to do much to lower temperaturequickly or sufficiently. (Thus , we use phys ica l means to lower body temperature.) N-acetylcysteine (c), the antidote for acetaminophen poisoning,does nothing for sa l icylate poisoning. Amphetamines (b) might seem rational for managing venti latory depress ion that characterizes late s tagesof severe aspi rin poisoning. The more l ikely outcome of giving an amphetamine i s s imply to hasten the onset of seizures . Phenobarbi ta l (d), orother CNS depressants , would aggravate an a l ready bad s tate of CNS/venti latory depress ion. (However, i f seizures develop they must be managed—eg, with IV lorazepam and phenytoin, even though they cause CNS depress ion. Without them, the patient may quickly die from statusepi lepticus .)

Just as I provided a table (answer to Question 317) to summarize the genera l sequence of events in acetaminophen toxici ty, I ’l l provide onehere for aspi rin overdoses . Aga in, the “s tages” I present are arbi trary, but they should be helpful—knowing what i s l ikely to come next as toxici typrogresses helps you prepare for what you’l l probably need to do next.

Note that the average letha l dose for an otherwise heal thy chi ld i s between around 5 g and 8 g, taken a l l at once. For adults , i t i s betweenaround 10 g and 30 g. “Usual s trength” aspi rin tablets—those used for aches , pa ins , mi ld inflammation, and fever—conta in 325 mg of the drug.(“Heart dose” aspi rin tablets conta in 81 mg.) Higher doses may be prescribed for arthri ti s—severa l grams a day—and a l though these usual ly arenot sufficient to cause death they are l ikely to cause s ide effects that often are dis turbing enough that the phys ician wi l l prescribe another

Page 190: Pharmacology pre test self assessment and review (14th ed.)

antiarthri tic drug instead of pla in, cheap, aspi rin.

Page 191: Pharmacology pre test self assessment and review (14th ed.)
Page 192: Pharmacology pre test self assessment and review (14th ed.)
Page 193: Pharmacology pre test self assessment and review (14th ed.)
Page 194: Pharmacology pre test self assessment and review (14th ed.)
Page 195: Pharmacology pre test self assessment and review (14th ed.)

Fina l note: Whi le treating aspi rin poisoning may sound di ffi cul t, ask a phys ician or nurse who has had to deal with i t many times . They arel ikely to tel l you that i t i s a “piece of cake”—and more often success ful—than deal ing with acetaminophen poisoning.

330. The answer is a. (Brunton, pp 911, 920, 1313-1314, 1815-1816; Katzung, pp 277-280.) One of the main advantages of the fi rs t-generationantihis tamines (fexofenadine, loratadine, others ) over the older (second-generation) H-1 antagonis ts such as diphenhydramine i s a relative lackof CNS depressant (sedating) effects when adminis tered at usual recommended doses . The impl ication of this di fference i s that the newer agentsare much less l ikely to cause daytime drows iness or other problems that might interfere with daytime activi ties requiring a lertness . Sedation i s aparticular problem with diphenhydramine, doxylamine, and other members of the ethanolamine class of fi rs t-generation antihis tamines , yet i ta lso provides a cl inica l ly useful effect based on this action: they are useful as OTC (diphenhydramine, doxylamine) and prescription(diphenhydramine) s leep-a ids .

Diphenhydramine, and to a large degree a l l the other fi rs t-generation antihis tamines , tend to cause appreciable muscarinic receptor-blocking(atropinel ike) effects , and so they are unsui table or potentia l ly dangerous for patients with such atropine-related comorbidi ties as prostatichypertrophy, hypomoti l i ty disorders of the urinary or GI tracts , tachycardia , or angle-closure glaucoma (so answer b i s not correct). Owing toantimuscarinic effects , diphenhydramine overdoses are more l ikely to cause tachycardia (by blocking parasympathetic-ACh-muscarinic receptorinfluences on the SA node) than bradycardia (c). The second-generation antihis tamines do block his tamine-related bronchoconstriction, but in theabsence of his tamine they have no intrins ic bronchodi lator activi ty. More important i s the fact that these drugs play no role as primary therapy forasthma. They may be preferred to older antihis tamines (largely because they lack the antimuscarinic-related mucus-thickening effects of suchdrugs as diphenhydramine), but they should be used only as adjuncts to more appropriate asthma drugs such as inhaled corticosteroids (tosuppress a i rway inflammation) and inhaled β-adrenergic drugs for control or rescue therapy.

None of the H1 blockers—firs t-generation or second-generation agents—has H2 blocking activi ty that i s necessary to suppress his tamine-mediated acid secretion by gastric parieta l cel l s .

331. The answer is b. (Brunton, pp 730-736, 976; Katzung, pp 183, 299, 1160t.) A major component of the antihypertens ive actions of captopri l and otherACE inhibi tors i s prostaglandin-dependent. That element of drug action i s antagonized by indomethacin due to i ts great efficacy to inhibi tprostaglandin synthes is . Other drugs with actions that may be inhibi ted by prostaglandin synthes is inhibi tors such as indomethacin include thethiazide and loop diuretics and (a l legedly) many i f not most of the β-blockers . The actions of none of the other drugs l i s ted are antagonized byindomethacin. You should note, however, that giving indomethacin to a patient taking warfarin or any of the antiplatelet drugs (used forprophylaxis of arteria l thrombos is ) i s ri sky. One of the major toxici ties of the NSAIDs i s gastric mucosa l damage and the ri sk of GI bleeds . In thepresence of anticoagulants or antiplatelet drugs the ri sk of a serious GI bleed, perhaps progress ing to hemorrhage, goes up markedly. However,this i s not due to a typica l pharmacodynamic or pharmacokinetic interaction between the drugs , and the anticoagulants ’ effects certa inly are notinhibi ted by the NSAID.

332. The answer is d. (Brunton, pp 75, 83t; Katzung, pp 1034-1035.) Late, severe aspi rin poisoning i s characterized by a combination of respi ratory andmetabol ic acidos is . In early s tages of aspi rin poisoning (or even with high “therapeutic” doses of the drug), venti latory s timulation occurs . Thatinduces a respi ratory a lka los is (c; net CO2 loss , relative HCO3 retention). The kidneys compensate for this by increas ing HCO3 excretion to helpnormal ize blood pH and caus ing what has been ca l led “compensated respiratory a lka los is .” As plasma levels of aspi rin ri se, however, blood pHfa l l s precipi tous ly. Part of that i s due to the accumulation of acidic sa l icyl i c acid in the blood, and part i s due to inhibi ted oxidativephosphorylation that shi fts metabol i sm from oxidative to glycolytic (with lactic acid being the key end product). No longer synthes izing ATPeffectively, the mitochondria generate metabol ic heat, which contributes to fever (not hypothermia; a ). Venti latory fa i lure (not s timulation; e)ensues , leading to respiratory acidos is (from CO2 retention) a long with the metabol ic acidos is . Cardiovascular col lapse and seizures eventual lycause death. Note that hepatotoxici ty i s not a component of this : that i s the main cause of morbidi ty and morta l i ty with acetaminophen poisoning.

333. The answer is a. (Brunton, pp 75-76, 128t, 136, 982-984; Katzung, pp 60-61, 1032.) Acetaminophen’s main toxic metabol i te i s N-acetylbenzoqinoneimine. It reacts with sul fhydryl groups (particularly with glutathione) in hepatocytes , ul timately leading to hepatic necros iswhen acetaminophen levels are sufficiently high. N-acetylcysteine i s ri ch in—SH groups . It therefore reacts with the imine, hopeful ly sparingendogenous sul fhydryl compounds from further attack and, hopeful ly, a l lowing time for the hepatocytes to recover from the biochemica l s tress .This foul -smel l ing antidote must be given early after acetaminophen poisoning occurs or i s l ikely (i t i s usual ly given ora l ly), and i t must be givenrepeatedly (see specia l ty texts for more information). It i s a preventative aga inst hepatotoxici ty, not a revers ing agent. As an as ide, variousnomograms are ava i lable to give some reasonable of whether a particular acetaminophen dose (more precisely, blood level ) i s l ikely to behepatotoxic or not. They plot plasma acetaminophen levels as a function of time after drug ingestion (yielding, bas ica l ly, an el imination rate andan el imination ha l f-l i fe). If the patient’s acetaminophen level at a speci fied time fol lowing intake i s above the l ine, hepatotoxici ty i s probable.Such a finding, however, does not mean that N-acetylcysteine adminis tration wi l l be without benefi t and so should not be done. (See theexplanation for Question 317 for more information, and a look at this i s sue from a somewhat di fferent perspective.)

Al l of the other drugs l i s ted can exert toxic effects (often serious) with overdoses , but N-acetylcysteine i s not an appropriate or effectiveantidote for any of them.

334. The answer is c. (Brunton, pp 995-996; Katzung, pp 651-652.) The main reason why many phys icians are shunning ora l colchicine for acute gout i sthat many patients develop horrible GI discomfort, vomiting, diarrhea, and the l ike. For some, the “cure” i s a lmost as bad as the disorder for whichthe drug i s given. This GI dis tress can be a l leviated somewhat by giving colchicine IV, but more serious systemic responses can develop i f the IVdose i s too great or too many IV doses are given in a short period of time. (These are among the reasons why IV formulations of colchicine are nolonger ava i lable.) Indomethacin seems to have become one of the preferred a l ternatives for anti -inflammatory therapy of acute gout orprophylaxis of recurrences . (And clearly indomethacin i s not without s ide effects or toxici ties ; the ri sks and discomforts are s imply moreacceptable, for some patients , with i t.)

But when colchicine does work in acute gout, rel ief may be dramatic and occur l i tera l ly “overnight” with just a dose or two. The l ikelymechanism of action involves impaired microtubular assembly or function in leukocytes , which l imits thei r migration to the area of crysta ldepos i tion and so l imits thei r abi l i ty to ampl i fy the inflammatory reaction.

Bone marrow suppress ion can occur, but that i s mainly with long-term, high-dose ora l or parentera l colchicine adminis tration (the latter ofwhich must be avoided). The same appl ies to frank gastric damage (with the poss ibi l i ty of gastric bleeding or hemorrhage) and to blooddyscras ias (bone marrow toxici ty).

335. The answer is b. (Brunton, pp 996-997; Katzung, pp 653-654.) You may think that reductions in urate content in a 24-hour urine sample would be badfor the hyperuricemic patient, but that i s because you probably automatica l ly (and incorrectly) equate less urine urate with increased plasmaurate. Not necessari ly so. Al lopurinol inhibi ts uric acid synthes is by inhibi ting xanthine oxidase: less uric acid made, less to be excreted in the

Page 196: Pharmacology pre test self assessment and review (14th ed.)

urine, and less to be detected there. This i s one mani festation of a l lopurinol “at work.” Acetaminophen (a) has no effects on uric acid synthes is ,excretion, or solubi l i ty. Colchicine (c) i s sometimes used for prophylaxis or treatment of gout, but i ts actions derive from i ts anti -inflammatoryaction and, probably, suppress ion of neutrophi l chemotaxis by interfering with microtubular function. It has no effects on urate synthes is orel imination. The same appl ies to indomethacin (d): anti -inflammatory actions , but no effects per se on uric acid. Probenecid (e) i s a uricosuricdrug that acts by inhibi ting tubular reabsorption of urate when plasma levels of the drug are therapeutic. As a resul t, the more l ikely outcome ofthis drug i s an increase of urate concentrations in a urine sample. Probenecid does not affect uric acid synthes is .

336. The answer is e. (Brunton, pp 995-996, 1000; Katzung, pp 651-654.) The use of sodium bicarbonate (unless otherwise contra indicated) forprophylaxis of urate s tone formation in the renal tubules i s based on the fact that uric acid (sodium urate) i s poorly soluble in body fluids andsolubi l i ty i s greatly influenced by loca l pH. When loca l pH fa l l s , more urate crysta l l i zes (ie, the KSP fa l l s ). Cons idering that urine pH i s lower thanblood (or synovia l fluid) pH, and s ince relatively large amounts of urate i s excreted in the urine (even in the absence of a uricosuric drug, eg,probenecid), there i s a greater tendency for urate to crysta l l i ze in the urinary tract. Conversely, as urine pH ri ses (as occurs with sodiumbicarbonate adminis tration), urate i s more soluble (for a given concentration) and less l ikely to precipi tate. This pH-related phenomenon a lsocontributes to the “vicious cycle” that s tarts when an acute gout attack in a joint occurs .

An additional body load of sodium bicarbonate wi l l induce, through normal renal compensatory processes , a diuretic (not antidiuretic, answera) effect; this should a lso increase the amount of urate that i s los t into the urine. Al though sodium bicarbonate ra ises blood pH, that has nocl inica l ly relevant effect on formation of xanthine and uric acid by xanthine oxidase (b). There i s no metabol ic acidos is that i s characteris tic ofsevere hyperuricemia (c). Sodium bicarbonate does not lower GFR and/or urate el imination (d).

337. The answer is c. (Brunton, pp 918-926; Katzung, pp 117-124, 277-280.) Diphenhydramine, a member of the ethanolamine class of H1 antagonis ts andarguably the prototype of a l l the older (“fi rs t-generation” antihis tamines), possesses two main properties in addition to effective competi tiveblockade of H1 receptors : sedation (c), in part due to the drug’s l ipophi l i ci ty; and muscarinic (chol inergic) receptor blockade that i s a lsocompeti tive. Both effects occur commonly, and can be qui te intense. The CNS depressant effects of diphenhydramine are such that this drug i s themain (i f not only) active ingredient in OTC s leep a ids , helping one dri ft off to s leep even in the absence of a l lergy s igns and symptoms andprobably in a way that i s unrelated to the drug’s periphera l H-1 blocking activi ty.

The antimuscarinic (atropine-l ike effects ) would be mani fest by such s ide effects as tachycardia (not bradycardia ; a ); constipation (not diarrhea;b); a suppress ion of ACh-induced gastric acid secretion (not the oppos i te; d); and a tendency for urinary retention (not frequency; e). The s tronganti -muscarinic effects of diphenhydramine earn i t the same precautions and contra indications we normal ly apply to atropine i tsel f, includingprostatic hypertrophy and angle-closure glaucoma. Other s ide effects include xerostomia, blurred vis ion, para lys is of accommodation, andinhibi tion of sweating. Diphenhydramine toxici ty mani fests in ways that are markedly s imi lar to what occurs with atropine poisoning, and i smanaged in the same way, including the use of physostigmine (ACh esterase inhibi tor with actions in both the periphera l and centra l nervoussystems) for severe or l i fe-threatening toxici ty.

As an important as ide, loratadine, des loratadine, fexofenadine, and related newer (second generation) H-1 blockers (piperidine class ) areassociated with much less CNS depress ion. (they are adverti sed as “nonsedating”—usual ly with a discla imer, in fine print, “when taken asdirected.”) Unl ike diphenhydramine and most of the older H-1 blockers , the newer agents cause no antimuscarinic effects and have no atropine-related precautions or contra indications .

338. The answer is c. (Brunton, pp 1018, 1826-1827; Katzung, pp 647, 656, 992, 999.) Etanercept (and the related drug infl i ximab) binds to and neutra l i zesTNF-α, which i s one of the primary pathophys iologic mediators in rheumatoid arthri ti s and Crohn disease. It functions , then, as an antibody.Etanercept has no effect on eicosanoid synthes is (a , d) or leukocyte migration (b). It does not s timulate col lagen, mucopolysaccharide, or synovia lfluid synthes is (e).

Page 197: Pharmacology pre test self assessment and review (14th ed.)

Gastrointestinal and Urinary Tract Pharmacology, Nutrition (Vitamins) Acid secretion inhibi tors (H2 blockers , proton pump inhibi tors , others )

AntacidsAntidiarrhealsDrugs affecting GI or urinary tract musculatureEmetics , antiemeticsGal l s tone-dissolving drugsGastric mucotropic/cytoprotective protective drugsInflammatory bowel disease drugsLaxatives , catharticsPancreatic enzyme replacementVitamins

Questions

339. A 65-year-old man wi l l be undergoing elective tota l knee replacement surgery. He i s urged to s tart taking a speci fied s tool softener severa ldays before surgery, and i t wi l l continue to be adminis tered as needed for severa l days after the surgery. That i s because of one of the drugs hewi l l receive in the operative and post-op periods (l i s ted below) routinely causes severe constipation. Which drug i s that?

a. Ketorolac to supplement postoperative analges ia from morphineb. Midazolam for sedation, anxiety rel ief, and induction of short-term amnes iac. Morphine and/or a s imi lar high-efficacy opioid for pa ind. Neostigmine as part of a protocol to reverse skeleta l muscle para lys ise. Ondansetron for management of postoperative nausea and vomiting

340. A patient with multiple medica l problems i s taking severa l drugs , including theophyl l ine, warfarin, quinidine, and phenytoin. Despi te thel ikel ihood of interactions , dosages of each were adjusted careful ly so thei r plasma concentrations and effects are acceptable. However, thepatient suffers some GI dis tress and s tarts taking a drug provided by one of his “wel l -intentioned” friends . He presents with excess ive or toxiceffects from all hi s other medications , and blood tests reveal that plasma concentrations of a l l the prescribed drugs are high. Which drug did thepatient most l ikely sel f-prescribe and take?

a. Antacid (typica l magnes ium-aluminum combination)b. Cimetidinec. Famotidined. Nizatidinee. Rani tidine

341. A 53-year-old woman has been taking warfarin for prophylaxis of venous thromboembol ism for the las t 6 months . During that time the degreeof anticoagulation, based on frequent measurements of her INR (International Normal ized Ratio, an adjusted prothrombin time), has been in thedes i red therapeutic range, 2.5, and she suffered no adverse effects . Two weeks ago, she developed what she bel ieved to be heartburn, andwithout consul ting her MD began taking an over-the-counter medicine on a da i ly bas is to control i t. She presents in cl inic today, reporting frequentepis taxis , and says her gums bleed when she brushes her teeth. Her INR i s now 9, wel l above the normal therapeutic range and indicative ofexcess warfarin effects . Which of the fol lowing medicines most l ikely interacted with the warfarin to cause these s igns and symptoms?

a. Bismuth sa l ts (eg, Pepto-Bismol ®)b. Ca lcium carbonatec. Cimetidined. Esomeprazolee. Magnes ium-aluminum antacid combinationf. Nizatidine

342. Esomeprazole i s one of the medications prescribed for a patient you are seeing today in the genera l medicine cl inic. Which one of thefol lowing s tatements best describes the actions or effects of esomeprazole and other drugs in i ts class?

a. Cause s trong systemic atropine-l ike (antimuscarinic) effects that l imit thei r use in patients for whom atropine i tsel f i s contra indicatedb. Inhibi t gastric acid secretion by s imultaneous ly and competi tively blocking the actions of the agonis ts his tamine, acetylchol ine, and gastrin, on

their parieta l cel l membrane receptorsc. Neutra l i ze gastric acid faster than any other classes of drugs indicated for peptic ulcer disease or gastroesophageal reflux disease (GERD)d. Profoundly inhibi t an ATPase located on parieta l cel l membranes , thereby inhibi ting acid secretione. Tend to cause bradycardia by antagonizing the pos i tive chronotropic effects of his tamine on cardiac H1 receptors

343. A 60-year-old man has episodes of severe gastric and esophageal pa in that i s ul timately l inked to gastric acid. Which one of the fol lowingdrugs or drug classes , whether ava i lable over-the-counter or prescribed, i s most l ikely to rel ieve the acid-related symptoms the fastest when takenat usual ly effective/recommended doses?

a. Antacids (eg, magnes ium-aluminum combinations)b. Cimetidine (but not other drugs in i ts class )c. H2 blockers , any of them

Page 198: Pharmacology pre test self assessment and review (14th ed.)

d. Misoprostole. Proton pump inhibi tors

344. Five days after s tarting sel f-medication for diarrhea and other GI compla ints that arose after a trip to Centra l America , a very worried patientca l l s to report that his tongue has turned black. His s tools have darkened too, but his description doesn’t at a l l fi t wi th what would be expected i fthe patient had a bleed anywhere in the GI tract (eg, no “coffee grounds s tools” suggestive of a gastric bleed.). Which drug most l ikely causedthese s igns?

a. Bismuth sa l tsb. Ca lcium carbonatec. Cimetidined. Esomeprazolee. Magnes ium-aluminum antacid combinationf. Nizatidine

345. A patient who has been a high-dose a lcohol abuser for many years presents with hepatic porta l -systemic encephalopathy. What drug, given inrelatively high doses , would be most sui table for the rel ief of some of the s igns and symptoms of this condition, and the l ikely underlyingbiochemica l anomal ies?

a. Diphenoxylateb. Esomeprazolec. Lactulosed. Loperamidee. Ondansetron

346. Fat-soluble vi tamins , compared with thei r water-soluble counterparts , genera l ly have a greater potentia l toxici ty to the user when taken inexcess . What property of those vi tamins—A, D, E, and K—is bas ica l ly the main reason for this finding?

a. Adminis tered in larger dosesb. Avidly s tored by the bodyc. Capable of dissolving membrane phosphol ipidsd. Involved in more essentia l metabol ic pathwayse. Metabol i zed much more s lowly

347. A patient i s transported from a dis tant hospi ta l to your surgica l service by a i r ambulance. He had abdominal surgery and in the postop careunit he received a drug that was clearly not indicated. The drug caused intense contraction of the detrusor and trigone muscles of his bladder. Thefi rs t dose fa i led to cause emptying of the bladder, so a second dose was given. However, and unknown to his prior care team, he had a mass thatrather s igni ficantly obstructed his urethra , and prior anatomica l weakening in a portion of his bladder. Upon adminis tration of the drug he fi rs tsuffered retrograde urine flow (from his bladder, back to the kidneys via the ureters ), caus ing renal damage. Soon thereafter his bladder rupturedinto his peri toneum. Which drug or drug class most l ikely caused these adverse effects?

a. Albuterolb. Atropine or a s imi lar antimuscarinicc. Bethanechold. Furosemidee. Propranolol or a s imi lar β-blocker

348. A patient presents with mala ise, and skin and mucous membranes appear pa le. Among the key findings from blood work are hypochromic,microcytic red cel l s and reduced red cel l count/hematocri t; reduced reticulocyte count; and reduced tota l hemoglobin content. Which drug would beindicated, based on the presentation?

a. Cyanocobalamin (B12)

b. Fol ic acidc. Irond. Vi tamin Ce. Vi tamin D

349. A patient wi l l s tart taking one of the drugs l i s ted below. As we hand them the prescription we advise them not to take supplementa l vi taminB6 (pyridoxine), whether a lone or as part of a multivi tamin supplement, because the vi tamin i s l ikely to counteract a des i red effect of theprescribed drug. To which drug does this advice apply?

a. Captopri l for heart fa i lure or hypertens ionb. Haloperidol for Tourette syndromec. Levodopa/carbidopa for Parkinson diseased. Niacin for hypertriglyceridemiae. Phenytoin for epi lepsy

350. A patient with tuberculos is i s being treated with i soniazid. She develops paresthes ias , muscle aches , and unsteadiness . Which vi taminneeds to be given in supplementa l doses in order to reverse these symptoms—or used from the outset to prevent them in high-ri sk patients?

a. Vi tamin Ab. Vi tamin B1 (thiamine)

Page 199: Pharmacology pre test self assessment and review (14th ed.)

c. Vi tamin B6 (pyridoxine)

d. Vi tamin Ce. Vi tamin K

351. A patient has severe gastroesophageal reflux disease (GERD). In addition to providing some immediate symptom rel ief, for which you wi l lprescribe usual ly effective doses of an OTC combination antacid product, you want to suppress gastric acid completely as poss ible. Which drug i sbest sui ted for achieving that goal?

a . Atropineb. Ca lcium carbonatec. Cimetidined. Esomeprazolee. Misoprostol

352. An opioid abuser, seeking something to sel f-adminis ter for subjective responses , gets a large amount of diphenoxylate and consumes i t a l l atonce. He i s not l ikely to do this aga in because he has consumed a combination product that conta ins not only the diphenoxylate but a lso anotherdrug that causes a host of unpleasant systemic responses . What i s the other drug found in combination with the diphenoxylate?

a. Apomorphineb. Atropinec. Ipecacd. Magnes ium sul fatee. Nal trexone

353. During a regular check-up, your patient s tates “You know, doc, sometimes after eating I get heartburn … you know, acid-indigestion.” Since thesymptoms seem to be mi ld and infrequent, you suggest an empiric tria l of an OTC antacid for prompt symptom rel ief, plus an ora l anti -acidsecretory drug for long-term control . You recommend severa l antacid brands for the patient to try. Al l the products you l i s t are combinationproducts that conta in a magnes ium sa l t and an a luminum sa l t. What i s main rationale or reason why the vast majori ty of these products conta inthese two particular drugs or sa l ts?

a. Al sa l ts counteract the gastric mucosa l -i rri tating effects of Mg sa l tsb. Al sa l ts require activation by an Mg-dependent enzyme in order to inhibi t the parieta l cel l proton pumpsc. Mg sa l ts cause a diures is that helps reduce systemic accumulation of the Al sa l t by increas ing renal Al excretiond. Mg sa l ts potentiate the abi l i ty of Al sa l ts to inhibi t gastric acid secretione. Mg sa l ts tend to cause a laxative effect (increased gut moti l i ty) that counteracts the tendency of an Al sa l t to cause constipation

354. A patient with multiple GI compla ints i s receiving ursodeoxychol ic acid (ursodiol ) as part of his drug regimen. What i s the most l ikely purposefor which this drug i s being given?

a. Dissolving cholesterol s tones in the bi le ductsb. Enhancing intestina l digestion and absorption of dietary fatsc. Helping to reverse malabsorption of fat-soluble vi tamins from the dietd. Stimulating gastric acid secretion in achlorhydriae. Suppress ing s teatorrhea and i ts consequences

355. Early-on in your genera l medicine clerkship you encounter a patient who i s taking a proton pump inhibi tor, bismuth, metronidazole, andtetracycl ine. What i s the most l ikely condition for which this drug combination i s being used?

a. Antibiotic-associated pseudomembranous col i ti sb. Irri table bowel syndrome (IBS)c. Refractory or recurrent, and severe, gastric or duodenal ulcers secondary to H. pylorid. “Traveler’s diarrhea,” severe, Escherichia coli-induced, from drinking contaminated watere. Ulcers that occur in response to long-term, high-dose NSAID therapy for arthri ti s

356. Severa l brand-name and s tore-brand “pink medications ,” adminis tered ora l ly and ava i lable without prescription, are widely used to helpa l leviate occas ional and short-l ived nausea and vomiting, nonspeci fic GI dis tress , and diarrhea. They are a lso recommended for prophylaxis of“traveler’s diarrhea,” which typica l ly i s caused by ingestion of foods or beverages contaminated with E. coli. These products conta in bismuthsubsa l icylate, and because of the presence of an aspi rin-l ike compound (a sa l icylate) they should not be taken by or adminis tered to certa inpatients . Which patient-related factor or comorbidi ty contra indicates use of this drug or product?

a. Essentia l hypertens ionb. Flu, chickenpox, or other vi ra l i l lness in a chi ld or adolescentc. Hot flashes and other s igns/symptoms of menopaused. Prostatic hypertrophy or glaucoma in elderly mene. Rheumatoid or osteoarthri ti sf. Severe seasonal a l lergy s igns and symptoms

357. A patient has undiagnosed multiple gastric ulcers . Shortly a fter consuming a large meal and large amounts of a lcohol , he experiencess igni ficant GI dis tress . He takes an over-the-counter heartburn remedy. Within a minute or two he develops what he wi l l la ter describe as a “badbloated feel ing.” Severa l of the ulcers have begun to bleed and he experiences searing pa in.

The patient becomes profoundly hypotens ive from upper GI blood loss and i s transported to the hospi ta l . Endoscopy confi rms multiple bleeds ;the endoscopis t remarks that i t appears as i f the les ions had been l i tera l ly s tretched apart, caus ing additional ti s sue damage that led to the

Page 200: Pharmacology pre test self assessment and review (14th ed.)

hemorrhage. Which drug or product did the patient most l ikely take?

a. An a luminum sa l tb. An a luminum-magnes ium combination antacid productc. Magnes ium hydroxided. Rani tidinee. Sodium bicarbonate

358. You have a patient who has been consuming extraordinari ly large amounts of a lcohol for severa l years , and i s genera l ly malnourished. Heabruptly s tops his a lcohol intake and goes into acute a lcohol withdrawal . Pertinent s igns and symptoms include nystagmus and bizarre ocularmovements , and confus ion (Wernicke encephalopathy). Al though this patient’s a lcohol consumption pattern has been accompanied by poornutrient intake overa l l , you need to manage the encephalopathy. Which drug i s most appropriate for this use?

a. α-tocopherol (vi tamin E)b. Cyanocobalamin (vi tamin B12)

c. Fol ic acidd. Phytonadione (vi tamin K)e. Thiamine (vi tamin B1)

359. A patient has s teatorrhea secondary to cystic fibros is . Which drug usual ly i s cons idered the most reasonable and usual ly effective drug formanaging the fatty s tools?

a. Atorvastatin (or any other HMG Co-A-reductase inhibi tor)b. Cimetidine (or an a l ternative, eg, famotidine)c. Bi le sa l tsd. Metoclopramidee. Pancrel ipase

360. You are meeting with a woman in whom pregnancy has just been confi rmed, and as part of your hol i s tic patient education you give her expl ici tadvice to avoid taking supplements of a particular vi tamin, especia l ly in high doses , because i t i s highly teratogenic. You a lso note the need toavoid any drugs that are derivatives of this nutrient during pregnancy, for the same reason. To which one of the fol lowing vi tamins does thisprecaution apply?

a. Ab. B12

c. Cd. Ee. Fol ic acid

361. A patient being cared for by the gastroenterology service i s being treated with sul fasa lazine. What i s the most l ikely purpose for which i t i sbeing given?

a. Antibiotic-associated pseudomembranous col i ti sb. E. coli-induced diarrheac. Gastric H. pylori infectionsd. Inflammatory bowel diseasee. NSAID-induced gastric ulcer prophylaxis

362. A patient with renal fa i lure i s undergoing periodic hemodia lys is whi le awaiting a transplant. Between dia lys is sess ions we want to reducethe body’s phosphate load by reducing dietary phosphate absorption and removing some phosphate a l ready in the blood. Which drug would bemost sui table for this purpose?

a. Aluminum hydroxideb. Bismuth subsa l icylatec. Magnes ium hydroxide/oxided. Sodium bicarbonatee. Sucra l fate

363. A woman has severe i rri table bowel syndrome (IBS) characterized by frequent, profuse, and symptomatic diarrhea. She has not responded tofi rs t-l ine therapies and i s s tarted on a losetron. What i s the most worrisome adverse effect associated with this drug?

a. Cardiac arrhythmias (serious , eg, ventricular fibri l lation)b. Constipation, bowel impaction, i schemic col i ti sc. Parkinsonian extrapyramidal reactionsd. Pulmonary fibros ise. Renal fa i lure

364. A patient undergoing cancer chemotherapy gets ondansetron for prophylaxis of drug-induced nausea and vomiting. Which best describes thisdrug’s main mechanism of action in this setting?

a. Activates μ-type opioid receptors in the chemoreceptor trigger zone (CTZ)b. Blocks centra l serotonin (5-HT3) receptors

Page 201: Pharmacology pre test self assessment and review (14th ed.)

c. Blocks dopamine receptorsd. Blocks his tamine H1 receptors in the bra instem and inner ear

e. Suppresses gastric moti l i ty and acid secretion via muscarinic blockade

Page 202: Pharmacology pre test self assessment and review (14th ed.)

Gastrointestinal and Urinary Tract Pharmacology, Nutrition (Vitamins)

Answers

339. The answer is c. (Brunton, pp 496, 501-502; Katzung, p 552.) Morphine (which i s often used for short-term postop pa in control ) and fentanyl (whichis often used for intraoperative pa in prevention) cause numerous s ide effects . One of the gravest concerns for most patients i s venti latorydepress ion, but that can be managed eas i ly with such interventions as venti latory support. Even in the absence of any l ingering post-opventi latory problems, constipation from a s trong opioid poses a very common and potentia l ly serious post-op management i ssue. That i s whystool softeners or laxatives take on an important role for such patients . Nei ther ketorolac (a) nor midazolam (b) cause s igni ficant reductions of gutmoti l i ty. Neostigmine (d), the most common acetylchol inesterase inhibi tor used to reverse drug-induced skeleta l muscle para lys is , would notl ikely to require a s tool softener or laxative. Indeed, via neostigmine’s abi l i ty to increase gut moti l i ty (s timulates longi tudina l gut muscle activi ty,relaxes sphincters ) i t i s probably the least l ikely drug to necess i tate use of a s tool softener. Ondansetron (e), qui te effective in managingperioperative nausea and vomiting, has a constipation-inducing ri sk far lower than that associated with s trong opioids .

340. The answer is b. (Brunton, p 845; Katzung, pp 610t, 1083t, 1111, 1155t.) Cimetidine di ffers s igni ficantly from the other H2 blockers , famoti -dine (c),nizatidine (d), and rani tidine (e), in that i t i s a very effective inhibi tor of the hepatic mixed function oxidase (P450) drug-metabol i zing enzymesystems. (Cimetidine inhibi ts CYP1A2, 2C9, 2D6, and 3A4.) The other H2 blockers have no s igni ficant P450 effects , nor do they cause s ide effects asfrequent or as problematic as those caused by cimetidine, especia l ly at high doses . The outcome of P450 inhibi tion, of course, i s reduced hepaticclearance of the interactants , leading to excess ive plasma concentrations and effects i f thei r dosages are not reduced properly. The examplesci ted in the question—phenytoin, warfarin, quinidine, and theophyl l ine—are among the important interactants with cimetidine. They have ratherlow margins of safety (or therapeutic index: LD50/ED50), so even a s l ight increases in plasma levels (reductions of metabol ic clearance) may beenough to cause toxici ty. Given the fact that the a l ternative H2 blockers don’t inhibi t the P450 system, there’s no rational reason for prescribingcimetidine to patients on multiple drug therapy (and in this case, the patient should have been warned about sel f-medicating with i t).

Antacids (a ; particularly those that include an a luminum sa l t, which appl ies to the vast majori ty of OTC antacid combination products ) tend toadsorb other drugs in the gut at or around the same time, and inhibi t thei r absorption (rate, extent, or both). If this patient had taken an antacidwith his other meds , on a regular bas is , you would see reduced blood levels and effects—not increases as described in the question.

Note: Not long after the newer H2 blockers came on the market, and we knew that they caused fewer s ide effects than cimetidine and certa inlydid not inhibi t various CYP enzymes to cause drug-drug interactions , I asked a gastroenterologis t why one would ever prescribe cimetidine. Heranswer? She couldn’t think of a va l id reason except cost. And now, as costs have come down, even that’s not a cons ideration. Fina l ly, i t i simportant to remember: cimetidine i s s ti l l ava i lable OTC, and patients who should not be taking i t (mainly because of interactions with otherdrugs they may be taking) can s ti l l use this drug without your OK or even your knowledge.

341. The correct answer is c. (Brunton, pp 845, 918, 1313t; Katzung, pp 610t, 1083t, 1111, 1155t.) Compared with a l l other drugs or drug groups l i s ted,cimetidine s tands out in terms of i ts abi l i ty to inhibi t s trongly the l iver’s P450 system, which i s important for the metabol ic inactivation of warfarin(and many other drugs). PPIs such as esomeprazole (d) don’t share this property, nor do H2 blockers other than cimetidine (eg, nizatidine).[Rani tidine, not l i s ted as an answer choice, i s an H2 blocker that does have some P-450-inhibi tory activi ty, but i t i s very weak—particularly incomparison with cimetidine—and the consequences of any potentia l drug interaction by this mechanism probably i s not cl inica l ly important.]

There i s an interaction between warfarin (and many other ora l ly adminis tered drugs) with bismuth sa l ts (a ) and, especia l ly, antacids (b, e) orantacid combinations , particularly Mg-Al combinations (e). They a l l tend to reduce the rate and extent of absorption of many ora l drugs taken at ornearly at the same time. In such cases , the bioava i labi l i ty of warfarin would be decreased, resul ting in an INR that i s lower—not higher—thanwould occur otherwise.

342. The correct answer is d. (Brunton, pp 1311-1312; Katzung, pp 1085-1089.) You should look at the “s tem” of the drug’s generic name. It ends in “-prazole,” and so you should rea l i ze that the drug i s class i fied as a proton pump inhibi tor (PPI). Esomeprazole, indeed a l l other “-prazoles ,” haveno antimuscarinic activi ty (a ). Their antisecretory actions occur “downstream” of agonis ts such as ACh, his tamine, and gastrin, and do not involveblockade/antagonism of those l igands on their receptors (b). PPIs have no acid-neutra l i zing activi ty (c; antacids work by that mechanism, andantacids provide the fastest rel ief of acid-related pa in); they s imply, yet effectively and rather profoundly, inhibi t gastric acid secretion. His taminedoes exert pos i tive chronotropic effects on the heart, but that i s not antagonized by any PPI, nor do the PPIs tend to cause bradycardia (e).

343. The answer is a. (Brunton, pp 1315-1316; Katzung, pp 1082-1083, 1150t.) Antacids , at usual recommended doses , rel ieve acid-related pa in fasterthan any other drugs . Their acid-neutra l i zing effects occur instantaneous ly upon contact with gastric (hydrochloric) acid, and pa in rel ief occurswithin minutes of taking a dose. Al l the other drugs inhibi t gastric acid secretion, some more effectively or completely than others , some quickerthan others , but none a l leviates acid-related pa in or discomfort as quickly as antacids do. This includes cimetidine (b) or any other H2 blocker (c),misoprostol (d) and any of the PPIs (e).

Severa l things worth noting here. Misoprostol has two main uses . One i s as an adjunct to drug-induced termination of pregnancy, due to i tss trong uterine smooth muscle s timulating effects . In the context of this question in the GI section, misoprostol i s sometimes used as an adjunct toreduce the incidence or severi ty of NSAID-induced gastric ulcer formation. For that purpose, misoprostol exerts what has been ca l led a mucotropicor gastric cytoprotective effect: i t enhances the formation of mucus in the gastric mucosa, thereby s trengthening the “barrier” between the NSAIDand i ts target of damage.

Remember: No acid, no pa in. This i s a true s tatement. It bas ica l ly means that the way to reduce gastric acid-related pa in i s ei ther to neutra l i zesufficient amounts of acid (with an antacid) and/or inhibi t sufficient amounts of gastric acid secretion, which can be accompl ished with ei ther H2

blocker or, more effectively, a PPI. Note that the rapidi ty of symptom rel ief does NOT para l lel rate of ulcer heal ing. Remember, too, a somewhatrelated s tatement that i s FALSE: No acid means no ulcer. What that means , cl inica l ly, i s that we can neutra l i ze acid, or inhibi t gastric acidsecretion rather quickly, but unless proper therapy i s continued long enough, there s ti l l wi l l be ulcers despi te the lack of the major symptom, pa in.

344. The answer is a. (Brunton, p 1337; Katzung, pp 1090-1091.) Among a l l the medications l i s ted, black tongue and black s tool are uniquelycharacteris tic of the use of bismuth sa l ts , including bismuth subsa l icylate (most widely known, perhaps , as Pepto Bismol ®). These effects are notworrisome (unless the patient i sn’t forewarned about this l ikely occurrence, in which case they can cause much a larm), and otherwise indicate notoxici ty or other potentia l adverse effects . None of the other drugs l i s ted share this property.

Page 203: Pharmacology pre test self assessment and review (14th ed.)

345. The answer is c. (Brunton, pp 1330-1332; Katzung, p 1093.) Lactu-lose i s a synthetic, nonabsorbable disaccharide (ga lactose-fructose). In moderatedoses i t acts as an osmotic laxative. In higher doses i t binds intestina l ammonia and other toxins that accumulate in the intestine in severe l iverdysfunction. These toxins , and perhaps more so the ammonia , contribute to the s igns and symptoms of encephalopathy. None of the other drugsl i s ted provide this benefi t.

Diphenoxylate (a) and loperamide (c) are opioid-l ike antidiarrheal drugs . Esomeprazole (b) i s a proton pump inhibi tor used to suppress gastricacid secretion. And ondansetron (e), a 5-HT3 antagonis t, i s mainly used to manage nausea and vomiting in response to chemotherapy.

346. The answer is b. (Brunton, p 1846; Katzung, pp 609, 772, 781.) Fat-soluble vi tamins , especia l ly A and D (the others being vi tamins E and K), can bestored in mass ive amounts and, hence, have a potentia l for serious toxici ties . (On the bright s ide, this abundant s torage means that relativelybrief periods of inadequate intake are not l ikely to cause cl inica l s igns and symptoms of deficiency.) Water-soluble vi tamins are eas i ly excretedby the kidneys and accumulation to toxic levels i s much less common. However, inadequate dietary intake wi l l lead to mani festations ofdeficiency relatively faster.

On average, most people consume excess ive amounts (a) of fat-soluble vi tamins no more often than they do water-soluble ones . Even very largedoses of fat-soluble vi tamins that might be consumed wi l l not a l ter membrane s tructure or function (c), a l though such effects can bedemonstrated in certa in in vi tro settings that bear l i ttle relevance to intact humans . Di fferences involving greater or more roles in overa l l cel lmetabol i sm (d), or rates of the metabol i sm of the vi tamins themselves (e), are not reasonable explanations for the relatively greater toxicpotentia l of vi tamins A, D, E, and K.

347. The answer is c. (Brunton, pp 206t, 222-224; Katzung, pp 99, 107, 112.) Bethanechol i s a muscarinic agonis t that i s used to manage functional urinaryretention—that i s , urinary retention or a s igni ficant inabi l i ty to void caused by some functional defect in the bladder musculature. It i scontra indicated for patients with bladder wal l s that are damaged or unable to respond safely to drugs that s timulate the bladder musculature,and patients with mechanica l obstruction of the urethra (from, say, prostatism or a prostatic cancer). This was what caused the bethanechol -induced problems for this patient.

Albuterol (a ), predominantly a β2 agonis t used for i ts bronchodi lator effects , has no s igni ficant effects on the heal thy or damaged bladder: β-receptors just aren’t there. Atropine (b) or other antimuscarinics wi l l weaken contraction of the trigone and detrusor, and s timulate contraction ofthe bladder sphincter, caus ing just the oppos i te of what I described. Furosemide (d) and other loop diuretics (torsemide, bumetanide, andethacrynic acid) obvious ly increase the need to urinate and the volume of urine that i s excreted. However, the bladder activation that ensues i smediated by reflexes triggered by a ful l bladder. There i s no di rect s timulation or inhibi tion of bladder musculature. Fina l ly, jus t as β-agonis tshave no appreciable effect on bladder musculature, nei ther do drugs that block β-adrenergic receptors (e).

348. The answer is c. (Brunton, pp 1079-1081; Katzung, pp 581-586.) The description conta ins many of the characteris tics of “i ron-deficiency anemia.”Ora l i ron sa l ts (eg, ferrous sul fate, gluconate, or fumarate) are usual ly the fi rs t choice for management (after rul ing out such causes as bloodloss ). Changes of gut moti l i ty (sometimes diarrhea, sometimes constipation), nausea, and heartburn are common compla ints with ora l i ron sa l ts ,but us ing the ora l drugs i s often preferable to, just as effective as , and safer than us ing parentera l i ron products such as i ron-dextran (ri sk ofanaphylaxis ) or other i ron complexes , which typica l ly involve erythropoietin adminis tration adjunctively.

Megaloblastic anemia (in contrast with the microcytic anemia I described here) would be treated di fferently. If i t i s caused by vi tamin B12

deficiency (vi tamin malabsorption due to deficiency of intrins ic factor; answer a), we would treat with cyanocobalamin. The other cause, folatedeficiency (inadequate dietary intake) i s managed with ora l fol ic acid (b). Deficiencies of vi tamin C or D (answers d and e, respectively) don’ttypica l ly cause anemias .

349. The answer is c. (Brunton, pp 614, 1086; Katzung, pp 484-488, 1157t.) Reca l l that DOPA decarboxylase, an enzyme whose activi ty i s dependent onpyridoxine, i s respons ible for metabol i zing ora l ly adminis tered levodopa to dopamine in the gut; and that only unmetabol i zed levodopa crossesthe blood-bra in barrier to be efficacious in rel ieving parkinsonian s igns and symptoms. Reca l l , too, that levodopa i s often adminis tered withcarbidopa, a drug that inhibi ts the periphera l decarboxylase, sparing levodopa for entry into the bra in and whose actions are antagonized bypyridoxine. Adminis tering supplementa l B6 wi l l reduce the bioava i labi l i ty of levodopa, thereby counteracting i ts antiparkinson effectiveness .None of the other drugs l i s ted, whether used for the s tated purpose or others , has i ts effects antagonized by pyridoxine.

350. The answer is c. (Brunton, pp 639, 780, 1086, 1555-1558; Katzung, pp 840-841.) Pyridoxine deficiencies ari se often during i soniazid therapy becausethe antimycobacteria l drug interferes with metabol ic activation of the vi tamin. The treatment or prophylaxis for at-ri sk patients i s to adminis terrelatively large doses of B6 (pyridoxine).

351. The answer is d. (Brunton, pp 1311-1313; Katzung, pp 1082-1083.) Esomeprazole and related drugs (lansoprazole, omeprazole, rabeprazole,pantoprazole) inhibi t the parieta l cel l H+-K+ATPase—the “proton pump”—that i s the “fina l common pathway” for acid secretion triggered by a l l themajor s timul i of gastric acid secretion. As a resul t, they are the most efficacious , caus ing near complete anti -acid secretory activi ty.

Reca l l the main agonis ts that provoke acid secretion. One i s gastrin, for which there are no cl inica l ly useful antagonis ts at this time. His tamine,aris ing from enterochromaffin-l ike (ECL) cel l s , activates H2 receptors , which then triggers acid secretion. His tamine’s effects can be blocked bycimetidine (c) or other H2 blockers (famotidine, nizatidine, and rani tidine; any of these a l ternatives are preferred to cimetidine, because they lackcimetidine’s P450 inhibi tory effects ). Acetylchol ine increases gastric acid secretion by activating muscarinic receptors on both ECL cel l s and parieta lcel l s . Its effects can be blocked by atropine (a), propanthel ine, pi renzepine (widely taught but never approved for use in the United States ), andsevera l other drugs with antimuscarinic activi ty. However, blocking only the his taminergic influences or only the chol inergic influences—or evenblocking both types of receptors—only partia l ly inhibi ts acid secretion. One additional disadvantage of muscarinic blockade i s the preva lence ofoften dis turbing and usual ly unwanted atropine-l ike s ide effects elsewhere, for example, on the eyes , heart, urinary tract, and exocrine glandsecretions .

Ca lcium carbonate (b), as wel l as a luminum and magnes ium sa l ts and sodium bicarbonate, are antacids . They have no inhibi tory effects onacid secretion. Instead, they neutra l i ze a component of acid that has a l ready been secreted (and a luminum sa l ts a lso adsorb peps ins ). Ca lciumand a luminum sa l ts tend to cause constipation, and magnes ium sa l ts , diarrhea/laxation. Sodium bicarbonate tends to cause no changes of gutmoti l i ty, but the s igni ficant absorption of the sodium poses problems for many patients (eg, those with hypertens ion, heart fa i lure, edema), andbicarbonate absorption causes metabol ic a lka los is .

Misoprostol (e) i s unique in severa l ways . One way i s that this prostaglandin analog mimics the effects of endogenous prostaglandins (mainlyPGE2) on parieta l cel l prostaglandin receptors . Unl ike the agonis ts noted above, one resul t of that i s inhibi tion of acid secretion. Overa l l , the anti -acid secretory effects are relatively weak.

Ultimately, however, regardless of whether gastrin, ACh, or his tamine i s present, parieta l cel l acid secretion ul timately “funnels through” the

Page 204: Pharmacology pre test self assessment and review (14th ed.)

proton pump. Block the receptors for any of the mediators noted above and you wi l l suppress acid secretion caused only by that mediator—butthat i s only a fraction of tota l acid secretion. Block the proton pump and acid secretion wi l l be inhibi ted nearly ful ly, no matter which one or moreagonis ts are present.

352. The answer is b. (Brunton, pp 504f, 507, 1338; Katzung, pp 558, 1094-1095, 1112.) Diphenoxylate i s an opioid that has predominant antidiarrhealactivi ty, and that i s i ts sole approved use. It inhibi ts peris ta ls i s and, hence, increases the passage time of the intestina l bolus . Typica lantidiarrheal doses do not cause venti latory depress ion, analges ia , or the euphoria for which opioids are mainly abused. However, should oneattempt to take high doses to become euphoric, unpleasant effects wi l l appear. That i s because the product conta ins a smal l amount of atropine—usual ly not enough to cause s ide effects when the proper dose of the product i s consumed, but clearly able to cause a l l the typica l andunpleasant anti -muscarinic s ide effects of which you should be aware (see the chapter, “Autonomic Nervous System”) when excess ive doses aretaken. This pharmaceutica l manufacturing “trick” discourages diphenoxylate abuse.

Apomorphine (a) and ipecac (c) are dopaminergic emetics (for parentera l and ora l adminis tration, respectively) that are used in somepoisonings where inducing emes is i s des i red and safe. Magnes ium sul fate (“Epson sa l t”—clearly a foul -tasting substance; answer d) i soccas ional ly and usual ly inappropriately used as a laxative or cathartic. Nal trexone (e) i s an opioid antagonis t (μ and κ receptors , na loxone-l ike,but given ora l ly.)

353. The answer is e. (Brunton, pp 1014, 1315-1316, 1523, 1531; Katzung, pp 1082-1083.) Magnes ium sa l ts (eg, oxide/hydroxide, as found in ordinary mi lkof magnes ia) used a lone tend to cause a laxative effect. (Indeed, at dosages higher than those used for acid neutra l i zation, magnes ium sa l ts areused for thei r laxative or cathartic effects .) Aluminum (and ca lcium) antacids , given a lone, tend to cause constipation. Combining a magnes iumsal t with an a luminum sa l t (and/or ca lcium antacid) i s an often success ful approach to minimizing antacid-induced changes of net gut moti l i ty.

Magnes ium sa l ts do not potentiate the antisecretory actions of Al or any other antacid. Indeed, none of the antacids inhibi t gastric acidsecretion (ie, they don’t have an antisecretory effect to begin with). They merely neutra l i ze acid that has a l ready been secreted (Mg, Ca, sodiumbicarbonate) or adsorb acid and peps ins (most of the a luminum compounds). Al though high concentrations of Mg sa l ts may cause gastrici rri tation, the amounts and concentrations found in antacid products are not sufficient to do that. Remember that in order to minimize the ri sks ofhypermagnesemia, even when otherwise reasonable doses are taken and especia l ly when doses are higher than that, renal function must be“adequate.” As a resul t, many elderly patients and patients of any age who have renal dys function from other causes , are at increased ri sk forhypermagnesemia.

Al sa l ts don’t protect aga inst any such potentia l effect. Magnes ium sa l ts do contribute to a diures is , but i t i s not a s igni ficant effect. Al sa l ts are“non-systemic” antacids : they are not absorbed to any appreciable degree, and therefore they don’t depend on renal processes for thei rel imination. The Al sa l ts are effective in the form in which they are adminis tered, and don’t require any enzymatic (or other type of) “activation” toexert thei r effects (which involves adsorption of acid, peps ins , etc).

354. The answer is a. (Brunton, pp 898-899; Katzung, p 1107.) Ursodeoxychol ic acid (ursodiol ), one of severa l natura l ly occurring bi le acids , i s effective insome patients with cholesterol ga l l s tones . Its main ini tia l action i s reduced hepatic cholesterol synthes is . That, in turn, lowers the cholesterolcontent in the bi le, which favors the dissolution of cholesterol s tones that have formed a l ready and reduces the incidence of new stoneformation.

Ursodeoxychol ic acid tends to inhibi t (not enhance, answer b) intestina l fat digestion and absorption, and may contribute to (not reverse,answer c) impaired absorption of vi tamins A, D, E, and K. The drug does not s timulate gastric acid secretion (d) and i s not indicated for managingachlorhydria . Steatorrhea may develop in response to ursodeoxychol ic acid; the incidence of fatty s tools i s not reduced (e) by the drug.

355. The answer is c. (Brunton, p 1320; Katzung, pp 1087-1088.) Severa l profess ional organizations , including the American Col lege of Gastroenterology,have guidel ines for managing severe, refractory, or recurrent duodenal and/or gastric ulcers , in which H. pylori clearly plays an importantpathophys iologic role. Al though severa l regimens have been suggested, a l l of them include an anti -acid secretory drug (usual ly a proton pumpinhibi tor, sometimes an H2 blocker), two or three antimicrobia ls (eg, amoxici l l in, clari thromycin, or tetracycl ine, usual ly with metronidazole), andbismuth. In many cases , this so-ca l led eradicative therapy can cause a cl inica l cure and prevent recurrences after about 8 weeks of treatment. (Thetreatment i s expens ive, but pa les in comparison with the financia l and personal costs of treating recurrent episodes or potentia l ly seriouscompl ications such as GI bleeding or hemorrhage, ei ther pharmacologica l ly or surgica l ly.)

Antibiotic-associated pseudomembranous col i ti s (a ) i s typica l ly managed with vancomycin or metronidazole, a long with discontinuation of thecausative agent (usual ly cl indamycin). Dietary modi fications (especia l ly increased dietary fiber intake) and various drugs that a l ter gut moti l i ty(laxatives in constipation-predominant IBS; opioid, opioid-l ike, or serotonin antagonis t antidiarrheals when diarrhea predominates ) are oftenchosen to help manage i rri table bowel syndrome (b). Doxycycl ine and/or bismuth sa l ts are often used to prevent or treat E. coli-mediated diarrhea(d). If the goal i s to prevent NSAID-induced gastric ulcers (e), a proton pump inhibi tor (eg, esomeprazole), or misoprostol are reasonable choices .

356. The answer is b. (Brunton, pp 975-976, 981; Katzung, pp 1090-1091.) Bismuth sa l ts such as the subsa l icylate (best known, perhaps , as the brand-name product Pepto Bismol ®) exert antibiotic activi ty aga inst H. pylori and probably E. coli a l so. They are commonly prescribed for the eradicativetherapy of H. pylori-induced refractory gastric ulcers , a long with an antibiotic and metronidazole. The sa l icylate in this formulation contra indicatesi ts use in chi ldren with influenza or any other vi ra l i l lness , owing to the ri sk of Reye syndrome. It i s somewhat controvers ia l what the term“chi ldren” means in this Reye-related context. The ri sk clearly appl ies to chi ldren in thei r adolescent (and earl ier) years of age, but i t may a lsoextend to people younger than about 21 years of age. This i s important to know, s ince chi ldren experiencing such vi ra l i l lnesses as influenza,chick-enpox, or even a common cold may report that thei r “tummy hurts” and they may experience diarrhea for a variety of reasons . The “pinkmeds” described in the question are qui te wel l known to many parents . Since they are ava i lable OTC (that automatica l ly conveys the notion thatthey are “safe”) and effective, i t’s not unl ikely that the unknowing caregiver might give the chi ld precisely the wrong drug. Warnings aga inst theuse of these bismuth-conta ining medications are printed on the labels of these products , but as you probably know by now many people don’t (orcan’t) read product labels .

The sa l icylate component a lso poses ri sks in other s i tuations for which a sa l icylate should be avoided. Aspirin-sens i tive asthma is a primeexample.

Bismuth sa l ts (or the sa l icylate moiety speci fica l ly) lack s igni ficant effects on blood pressure or the effects of antihypertens ive drugs (a). Thereare no wel l -documented beneficia l or adverse effects on s igns and symptoms typica l ly associated with menopause (c). Likewise, there are noknown des i red or unwanted effects on prostate, urinary tract, or ocular function or disease (d), whether in older men or not; no effects on arthri ticdisease (the sa l icylate in bismuth subsa l icylate i s apparently sufficient to warrant the Reye syndrome warning, but not sufficient to favorablyinfluence inflammatory diseases such as arthri ti s ); nor affect s igns and symptoms of seasonal (or other) a l lergic responses (f).

The drug has no H2-blocking or muscarinic receptor-blocking activi ty. In the absence of any contra indications , bismuth subsa l icylate may beuseful for managing some cases of diarrhea, such as “traveler’s diarrhea” that often i s caused by E. coli.

Page 205: Pharmacology pre test self assessment and review (14th ed.)

357. The answer is e. (Brunton, pp 1331-1334; Katzung, p 1083.) You’ve a l l done the experiment: mix vinegar and baking soda (sodium bicarbonate) andone product i s CO2. This gas i s formed when sodium bicarbonate—sti l l used as a lay remedy for “heartburn” (esophageal reflux) and other acid-related GI dis turbances—reacts with HCl .

Normal ly intragastric pressure i s kept in check when the gastroesophageal sphincter opens . However, when pressure can’t be rel ieved quicklyenough, or adequately, the s tomach dis tends l ike a ba l loon. In the presence of ulcers , the les ions are s tretched mechanica l ly, favoring furtherdamage that can lead to acute bleeding. Even in the absence of ulcers , any weakness of the gastric wal l can lead to rupture. (Reports about thisscenario, publ i shed in the lay press a whi le ago and laced with more than a l i ttle hyperbole, s tated that “[the man’s ] s tomach exploded after abig night on the town”, fol lowed by ingesting baking soda for rel ief.)

None of the other antacids l i s ted, whether a lone or in combination, lead to production of CO2 or any other gas that might lead to the outcomedescribed here. Aluminum sa l ts (a ) seldom are used a lone for neutra l i zing gastric acid, but are used occas ional ly to reduce ci rculating phosphatelevels in such patients as those with renal fa i lure. Used a lone, a luminum sa l ts tend to cause constipation. Aluminum-magnes ium combinationproducts (b) are the most commonly prescribed or sel f-prescribed antacids . The rationale for this combination, as ide from overa l l efficacy, i s thatthe laxative effects of the magnes ium sa l ts (c) are counteracted by the constipating effects of the a luminum sa l ts . (Ca lcium sa l ts , particularlyca lcium carbonate, a lso have antacid effects and, when used a lone, tend to cause constipation.) Rani tidine (d; and famotidine, nizatidine and thelesser-used cimetidine) are H2 blockers .

358. The answer is e. (Brunton, pp 642-644, 655; Katzung, pp 389-401, 575.) Thiamine, adminis tered parentera l ly before or at the same time asadminis tering glucose, and often in high dosages , i s used for Wernicke encephalopathy. It dramatica l ly amel iorates the s igns and symptoms.Thiamine must be given i f glucose i s to be given. Thiamine serves as a cofactor for many enzymatic reactions , including those involved incarbohydrate metabol i sm. If glucose i s adminis tered in a thiamine-deficient s tate, what l i ttle vi tamin i s left in the ci rculation wi l l be depleted,aberrant glucose metabol i sm wi l l continue, and the pathology (particularly neurologic) l ikely wi l l be worsened.

Thiamine deficiency i s a l so respons ible for Korsakoff psychos is (or Korsakoff amnestic syndrome), another accompaniment of severe, long-termalcohol consumption (especia l ly without adequately nutri tional diets ). Unfortunately, the s igns and symptoms of Korsakoff (short-term memoryproblems, a tendency to fabricate, polyneuropathies ) are not revers ible.

Vi tamin E (a) i s not indicated for Wernicke encephalopathy. Supplements of that fat-soluble vi tamin are often used (misused?) by the lay publ icfor the vi tamin’s “antioxidant” and “anti -aging” effects . Cyanocobalamin (B12; b) i s often used for macrocytic anemias that involve deficiencies ofintrins ic factor (see Question 348). Fol ic acid (c) supplements have a variety of uses , including reducing the ri sk of teratogenic or s imi lar effects inpregnant women, especia l ly i f they are taking certa in antiepi leptic drugs . The most common use of phytonadione (vi tamin K; d) i s to help reverseexcess ive anticoagulant effects of warfarin.

359. The answer is e. (Brunton, p 1346; Katzung, pp 1106-1107, 1111.) Pancrel ipase i s an a lcohol ic extract of hog pancreas that conta ins l ipase, tryps in,and amylase. (The related drug, pancreatin, i s s imi lar.) The goal in us ing i t here i s not so much to control the symptom, diarrhea, but do so byattacking the cause, which i s endogenous pancreatic enzyme deficiency (l ipases , amylase, chymotryps in, and tryps in) that leads to impaired fatdigestion and absorption.

None of the other drugs mentioned (a s tatin, a ; an H2 blocker, b; bi le sa l ts , c; metoclopramide, d) have actions that would be as effective (i feffective at a l l ) or speci fic as pancrel ipase. Antidiarrheals , for example, would only treat the symptoms, not the underlying cause.

Note: You may find pancrel ipase adminis tered with antacids , or an acid secretion inhibi tor (H2 blocker or proton pump inhibi tor). The purposeof combined therapy i s not related to preventing adverse effects of acid on the gastric mucosa, but rather to ra ise gastric pH and prevent thepancrel ipase from being hydrolyzed and inactivated by acid.

360. The answer is a. (Brunton, pp 1793, 1812; Katzung, pp 1040t, 1070.) Pregnant women should not take more than 25% more than the normal(recommended) da i ly dietary a l lowance for vi tamin A, because i t i s teratogenic, especia l ly in the fi rs t trimester of pregnancy. Note that the vi taminA-l ike drug i sotretinoin, which i s mainly indicated for treating refractory or severe acne vulgaris , i s contra indicated too.

Of course, there are pregnancy-related recommendations for the other nutrients l i s ted in the answer: vi tamins B12 (b); C (c); E (d); and,especia l ly, fol ic acid (e). However, and a l though excess ive intake of these nutrients i s not recommended, and doing so may pose ri sks to themother or the fetus , the consequences are not “highly teratogenic.”

361. The answer is d. (Brunton, pp 944t, 1352t; Katzung, pp 646, 1101.) Sul fasa lazine, which i s a combination of sul fapryidine and 5-aminosa l icyl ic acid(5-ASA) l inked covalently (azo bond), i s qui te effective for managing inflammatory bowel disease (eg, ulcerative col i ti s and Crohn disease). Someof the sul fasa lazine i s absorbed, and a portion of that i s excreted unchanged back into the colon. Colonic bacteria spl i t the azo l inkage, releas ingthe two drugs . The 5-ASA i s respons ible for loca l anti -inflammatory activi ty (suppress ion of inflammatory mediators , but how, i s not wel lunderstood) and symptom rel ief. The other metabol i te, sul fapyridine, i s primari ly respons ible for s ide effects associated with this “two drugs inone” combination: nausea, vomiting, and headaches (dose-dependent); sul fonamide a l lergic reactions in “sul fa-sens i tive” patients ; and rare butpotentia l ly fata l blood reactions including immune-mediated hemolys is and aplastic anemia.

A related drug, mesalamine, conta ins only the active 5-ASA. Lacking any sul fapyridine, the s ide effects , and adverse responses are very lowcompared with sul fasa lazine. An even neater s trategy i s used by olsa lazine: the drug i s comprised of two azo-l inked 5-ASA molecules ; the bond i scleaved by gut bacteria , releas ing two 5-ASA molecules for every molecule of olsa lazine adminis tered.

362. The answer is a. (Brunton, pp 1277, 1288; Katzung, pp 780, 1083, 1111.) Aluminum hydroxide (and a l l other cl inica l ly useful a luminum sa l ts otherthan the phosphate) has a high affini ty for phosphate. In the gut, the a luminum sa l ts bind phosphate and prevent i ts absorption qui te wel l . Theyalso induce a blood-to-gut gradient that favors el imination of ci rculating phosphate. (Used inappropriately, i t may cause hypophosphatemia:susta ined, high-dose use of a luminum-conta ining antacids i s one of the most common causes of hypophosphatemia.) None of the other drugsl i s ted are effective in or used for reducing phosphate absorption or lowering plasma levels .

The main l imitation to us ing an a luminum sa l t by i tsel f i s i ts tendency to cause constipation. This i s usual ly deal t with, when a luminum sa l tsare used as typica l “antacids ,” by coadminis tering a magnes ium sa l t, which a lone tends to cause laxation. (Giving a magnes ium sa l t a lone or incombination with another antacid would be inadvisable for patients with renal fa i lure, such as this dia lys is patient, who are often unable toexcrete the cation at rates sufficient to avoid hypermagnesemia.)

363. The answer is b. (Brunton, p 1340; Katzung, p 1096.) Constipation i s the most common and most worrisome adverse response to a losetron, whichis often used for i rri table bowel syndrome. This drug’s mechanism of des i red action involves selective blockade of serotonin receptors (5-HT3) inthe gut; the main outcomes include s lowing of colonic transport time, increased sodium and water reabsorption from the colon, and reducedsecretion of water and electrolytes into the colon.

Page 206: Pharmacology pre test self assessment and review (14th ed.)

Although constipation (b) might be viewed as a trivia l , minor, or merely annoying compla int with most drugs , with a losetron i t i s the mostworrisome one. Constipation may (and has) progressed to feca l impaction, bowel perforation or obstruction, and i schemic col i ti s . Fata l i ties haveoccurred.

The ri sks are such that the drug was pul led from the market in early 2000 and reapproved two years later with a l imited indication (prolonged,severe diarrhea associated with IBS in women); abundant warnings in the package insert; and a comprehens ive ri sk-management/avoidanceprogram that includes a requirement for (among other things ) a specia l “s ign-off” by both the prescriber and the patient that they understand andacknowledge and can identi fy the ri sks and agree to treatment anyway.

Serious cardiac arrhythmias (a), extrapyramidal reactions (c), pulmonary fibros is (d), or renal fa i lure (e) are not the most l ikely or mostworrisome adverse effects associated with a losetron.

364. The answer is b. (Brunton, pp 1341-1344; Katzung, pp 287, 1097-1099, 1111.) The antiemetic effects of ondansetron—a drug that i s widely andeffectively used to manage nausea and emes is associated with chemotherapy or many of severa l drugs typica l ly adminis tered during surgery—ismainly due to blockade of 5-HT3 receptors in the bra in’s chemoreceptor trigger zone (CTZ) and, probably, in the sol i tary tract nucleus and in thestomach and smal l intestine.

Activation of μ-opioid receptors in the CTZ (a) i s what causes the nausea and emes is that i s so common with such drugs as morphine and otheropioid analges ics .

Blockade of centra l dopamine receptors (c), particularly D2 receptors in the CTZ, i s the main mechanism by which phenothiazines (eg,chlorpromazine, prochlorperazine) cause their antinausea/antiemetic effects . They are useful and effective for some patients with chemotherapy-induced nausea and vomiting, but better sui ted for prophylaxis of these symptoms due to other causes or for prophylaxis of motion s ickness .Centra l antimuscarinic and antihis taminic actions may contribute to the overa l l effects .

Blockade of H1-his tamine receptors (d) in the bra in (bra instem) and vestibular apparatus of the inner ear accounts for the main mechanism ofaction of such drugs as cycl i zine, diphenhydramine (and i ts derivative, dimenhydrinate), and hydroxyzine. Antimuscarinic effects of these anti -his tamines may contribute to the overa l l effect. They are best sui ted for motion s ickness prophylaxis , less effective as “genera l antiemetics ,” andnot very or speci fica l ly effective for chemotherapy-induced symptoms.

Antimuscarinic agents (e; scopolamine can be cons idered the prototype in terms of motion s ickness or emes is control ) do not exert thei r effectsvia actions on the GI musculature, a l though suppress ion of AChmediated GI moti l i ty and tone certa inly occurs . Their effects are primari ly centra l ;thei r cl inica l uti l i ty i s mainly l imited to prophylaxis or treatment (lower efficacy) of motion s ickness .

Page 207: Pharmacology pre test self assessment and review (14th ed.)

Endocrine and Reproductive Pharmacology Adrenal corticosteroidsAnabol ic s teroids , related drugsCalcium-regulating drugsDiabetes mel l i tus and hypoglycemiaErecti le dys functionEstrogens , progestins , contraceptives , ferti l i ty agentsTestosteroneThyroid disordersUterine s timulants (oxytocics ) and relaxants (tocolytics )

Questions

365. A 55-year-old male patient with a 20-year his tory of type 2 diabetes mel l i tus comes to your cl inic for his regular check-up. He looks good, feelswel l . However, over the las t year his HbA1c levels hovered around 9.5% (equiva lent to an average plasma glucose of around 225 mg/dL). Currentdiabetes medications are metformin and glyburide. The patient says he i s doing “the best he can” with recommended diet and exercise plans . Hehas other personal and fami l ia l ri sk factors for coronary heart disease. He takes a s tatin and metformin to control his l ipids , but his LDL andtriglyceride levels are far too high. He i s taking verapami l for Stage 1 hypertens ion, but his pressure i s s ti l l higher than you’d l ike i t to be. Twoyears ago, he had severe angina from atherosclerotic coronary disease and underwent angioplasty and placement of a s tent. He has had noischemic episodes s ince.

You’re thinking about adding niacin to get further control of triglyceride levels , adding another antihypertens ive drug, and adjusting hisdiabetes treatment by adding another hypoglycemic drug (perhaps insul in). Which comorbidi ty or other factor would weigh against your selection ofan angiotens in converting enzyme (ACE) inhibi tor or angiotens in receptor blocker (ARB) as the add-on antihypertens ive drug for this patient?

a. Bi latera l renal artery s tenos is and a lbuminuriab. Bradycardiac. Had two episodes of hyperosmolar hyperglycemic nonketotic syndrome in past 2 yearsd. Has heart fa i luree. Niacin wi l l indeed be prescribed

366. A patient has an inoperable pancreatic i s let cel l carcinoma and i s chronica l ly hypoglycemic. Which drug would be most l ikely chosen forrelatively long-term ora l therapy that attempts to ra ise blood glucose levels into a more acceptable range?

a. Atenolol (or metoprolol )b. Diazoxidec. Glucagond. Octreotide (somatostatin analog)e. Oxytocin

367. A patient with type 2 diabetes mel l i tus i s notorious ly noncompl iant with medication and diet recommendations . However, he thinks he’ssmart enough to fool the phys ician into thinking otherwise: he takes his medication and el iminates nearly a l l carbohydrate intake for a few daysbefore each cl inic vi s i t, knowing he wi l l get a finger s tick for a spot check of plasma glucose levels . What would be the s implest, most cost-effective, and most informative way for the phys ician to assess for past drug and diet compl iance and long-term glycemic control?

a . Glucose concentration in venous blood sampleb. Glucose tolerance test (ora l )c. HbA1c

d. Plasma levels of the antidiabetic druge. Urine ketone levels (in a sample donated at the time of cl inic vi s i t)f. Urine glucose levels

368. A 70-year-old man compla ins of progress ive di ffi cul ty s tarting his urine s tream, and having to get up severa l times during each night tourinate. Recta l examination reveals a genera l ly enlarged, smooth-surfaced prostate. Prostate-speci fic antigen (PSA) ti ters are s igni ficantlyelevated. You decide to s tart finasteride treatment. Eventual ly urine flow increases , noctures is episodes decrease, and overa l l prostate s i zedecreases . Which phrase best summarizes the most l ikely mechanism by which finasteride caused symptom rel ief?

a . Blocks α-adrenergic receptorsb. Blocks testosterone receptorsc. Inhibi ts dihydrotestosterone synthes isd. Inhibi ts testosterone synthes ise. Lowers plasma testosterone levels by increas ing i ts renal clearance

369. A 38-year-old woman presents in the endocrinology and metabol i sm cl inic today. One of the drugs she i s taking i s class i fied as a dipeptidylpeptidase-4 (DPP-4) inhibi tor. Which s tatement best describes the main actions of this drug?

a. Activates hydroxylases required for corti sol and a ldosterone synthes isb. Cleaves active insul in from the proinsul in moleculec. Enzymatica l ly spl i ts levothyroxine from thyroglobul in

Page 208: Pharmacology pre test self assessment and review (14th ed.)

d. Helps normal ize phys iologic release of estrogen and progesteronee. Inhibi ts inactivation of incretins , ra ises glucagon-l ike peptide levels

370. A 54-year-old woman who has not seen a phys ician for severa l years presents with a previous ly undiagnosed thyroid cancer andthyrotoxicos is . One drug that i s adminis tered as part of early management, and may be l i fesaving, i s propranolol . Which of the fol lowing bestsummarizes the objective for adminis tering this drug?

a. Block parenchymal cel l receptors for thyroid hormonesb. Block thyroid hormone release by a di rect effect on the glandc. Inhibi t thyroid hormone synthes isd. Lessen dangerous cardiovascular s igns and symptoms of thyroid hormone excesse. Lower TSH levels

371. A woman deemed at high ri sk of postmenopausa l osteoporos is i s s tarted on a lendronate. What i s this representative bisphosphonate’smain mechanism of action?

a. Activates vi tamin D and so faci l i tates absorption of dietary ca lciumb. Conta ins lots of ca lcium, which supplements dietary ca lcium intakec. Di rectly forms hydroxyapati te crysta ls in boned. Provides supplementa l phosphate, which indirectly elevates plasma Ca 2+e. Reduces the number and activi ty of osteoclasts in bone

372. Some patients who are taking high doses of a bisphosphonate for Paget disease of the bone develop an endocrine-metabol ic disorder. Whatdisorder would that be?

a. Cushing disease (cushingoid symptoms)b. Diabetes ins ipidusc. Diabetes mel l i tusd. Hyperparathyroidisme. Hyperthyroidism

373. Metyrapone i s useful in testing hyper- or hypo-function of certa in endocrine conditions or biologica l processes those glands normal ly control .When we adminis ter this drug for diagnostic purposes , which s tructure or function are we most l ikely assess ing?

a. α cel l s of pancreatic i s letsb. β cel l s of pancreatic i s letsc. Leydig cel l s of the testesd. Pi tui tary-adrenal axise. Thyroid gland’s response to TSH

374. A 60-year-old man on long-term therapy with a drug develops hypertens ion, hyperglycemia, and decreased bone dens i ty. Blood tests indicateanemia. Some of his s tool samples ini tia l ly were pos i tive for occul t blood, and then the s tool developed a “coffee-grounds” appearance. Whichdrug i s most l ikely respons ible for the patient’s symptoms?

a. Beclomethasoneb. Hydrochlorothiazidec. Metformind. Pamidronatee. Prednisone

375. A woman with atria l fibri l lation i s being treated long-term with amiodarone (and warfarin). This antiarrhythmic can cause biochemica lchanges and cl inica l s igns and symptoms that resemble those associated with which endocrine disease/disorder?

a. Addisonian cri s i sb. Cushing syndromec. Diabetes ins ipidusd. Diabetes mel l i tuse. Hypothyroidismf. Ovarian hyperstimulation syndrome

376. A 22-year-old woman has been sexual ly assaul ted and she wishes to have the pregnancy terminated by pharmacologic means . What i sgenera l ly the most appropriate drug, assuming no contra indications?

a. Ergonovine (or methylergonovine)b. Mifepris tonec. Ra loxi fened. Ri todrinee. Tamoxi fen

377. A patient has hyperthyroidism from a thyroid cancer, and the medica l team concludes that ora l radioiodine (sodium iodide 131 [131I]) i s thepreferred treatment. The dosage i s ca lculated correctly, and the drug i s adminis tered. What s tatement about this approach i s a lso correct?

Page 209: Pharmacology pre test self assessment and review (14th ed.)

a. A β-adrenergic blocker should not be used for symptom control i f or when 131I i s usedb. Hyperthyroidism symptoms resolve a lmost completely within 24 to 48 hours after dos ing with 131Ic. Many patients treated with 131I develop metastatic nonthyroid cancers in response to the drugd. Ora l anti thyroid drugs should be adminis tered up to and including the day of 131I adminis tratione. There i s a high incidence of delayed hypothyroidism after us ing 131I for eradication of a thyroid tumor, and so thyroid hormone supplements

may be needed later on

378. A 20-year-old woman, otherwise heal thy, presents with i rregular and occas ional ly missed menstrua l periods , oi ly facia l skin and acne, ands l ight hi rsutism. She was a competi tive runner during high school and now is in tra ining for a triathlon. She i s not taking any medications otherthan an estrogen-progesterone ora l contraceptive. A pelvic ul trasound eva luation reveals numerous immature ovarian fol l i cles (genera l ly butincorrectly referred to as cysts ). The diagnos is i s polycystic ovarian syndrome (PCOS). Which drug would be the most rational ini tia l therapy toprescribe to help provide symptom rel ief without compromis ing the woman’s abi l i ty to conceive?

a. Es trogen (dose higher than in her ora l contraceptive)b. Ketoconazolec. Metformind. Prednisone (or a s imi lar ora l glucocorticoid)e. Testosterone

379. A 50-year-old woman with a high ri sk of breast cancer i s s tarted on tamoxi fen for prophylaxis . What i s the main mechanism of action of thisdrug?

a. Blocks estrogen receptors in breast ti s sueb. Blocks estrogen receptors in the endometriumc. Increases the ri sk of osteoporos isd. Ra ises plasma LDL cholesterol and tota l cholesterol , lowers HDLe. Reduces the ri sk of thromboembol ic disorders

380. A 50-year-old woman is recently diagnosed with type 2 diabetes mel l i tus . Exercise and diet do not provide adequate glycemic control , so drugtherapy needs to s tart. The phys ician contemplates prescribing metformin. Which s tatement about this drug i s correct?

a. Beneficia l and unwanted actions are unaffected by l iver function s tatusb. Lactic acidos is occurs frequently, but i t i s seldom seriousc. Metformin-induced hypoglycemia seldom occursd. Useful for type 1 diabetes a lsoe. Weight ga in i s a common and unwanted s ide effect

381. Most therapeutic insul ins nowadays are modi fications of native human insul in, done by substi tuting some amino acids in the native proteinus ing recombinant DNA technology. For a l l these genetica l ly modi fied insul ins , what i s the one common resul t of such changes?

a. Changes the onsets , durations of actionb. Enables adminis tration by ei ther subcutaneous or intravenous routesc. Prevents cel lular K+ uptake as glucose enters cel l sd. Reactivates endogenous (pancreatic) insul in synthes ise. Selectively affects glucose metabol i sm, l i ttle/no effects on l ipids

382. You have prescribed an ora l agent to help control a patient’s blood glucose levels . He has type 2 diabetes . In expla ining how the drug works ,you describe i t as a “s tarch blocker” that inhibi ts the intestina l uptake of complex carbohydrates in the diet. You advise a lso that flatus or somecramping or “grumbl ing sounds” in the bel ly may develop. Which drug fi ts this description?

a. Acarboseb. Any thiazol idinedione (“gl i tazone”)c. Gl ipizided. Metformine. Tolbutamide

383. A patient with type 1 diabetes i s being treated with insul in glargine. What cl inica l ly important property sets this particular insul in apart, orotherwise di fferentiates i t, from nearly a l l the other insul in formulations that might be used instead?

a. Blood levels , hypoglycemic effects , fol lowing insul in glargine injection are more accurately described as a plateau rather as a defini te “spike”or peak

b. Disul fi ram-l ike reactions (aceta ldehyde accumulation from inhibi ted EtOH metabol i sm) more common, severe, with insul in glarginec. Has an extremely fast onset, useful for immediate postprandia l control of plasma glucose elevationsd. Poses l i ttle or no ri sk of hypoglycemia i f the patient skips severa l meals in a rowe. Sens i ti zes parenchymal cel l s to insul in (eg, the adminis tered insul in i tsel f), not s imply providing or replacing insul in, thereby enhancing

glycemic control

384. A patient presents in the emergency department (ED) with a mass ive overdose of a drug. The most worrisome s igns and symptoms includeexcess ive cardiac s timulation (severe tachycardia , pa lpi tations , angina, etc). The ED phys ician orders IV adminis tration a β-adrenergic blocker,saying (correctly) i t i s the only drug l ikely to normal ize cardiac function quickly and save the patient’s l i fe. What was the most l ikely drug thepatient overdosed on?

Page 210: Pharmacology pre test self assessment and review (14th ed.)

a. A second-generation sul fonylurea (eg, gl ipizide, glyburide)b. Insul inc. Levothyroxined. Prednisone (ora l glucocorticoid)e. Propyl thiouraci l

385. This week finds you accompanying an attending in her outpatient endocrinology cl inic. One patient, a 56-year-old woman, i s taking exena-tide, which i s class i fied as an incretin mimetic. What phrase best describes exenatide’s actions or main cl inica l use?

a. Antagonizes testosterone effects , useful for treating hi rsutism in men or womenb. Is a new adjunct to metformin and/or sul fonylurea therapy of hyperthyroidismc. Often used as an add-on to a bisphosphonate for prophylaxis of postmeno-pausa l osteoporos isd. Preferred s timulant of corti sol production for treating Addison diseasee. Useful adjunct for some patients with poorly control led type 2 diabetes mel l i tus

386. A patient with hypothyroidism fol lowing thyroidectomy wi l l require l i felong hormone replacement therapy. What drug or formulation genera l lywould be most sui table?

a. Levothyroxine (T4)

b. Liothyroninec. Liotrixd. Proti rel ine. Thyroid, des iccated

387. A patient develops marked skeleta l muscle tetany soon after a recent thyroidectomy. The attending confi rms the diagnos is , in part, by l ightlytapping the patient’s cheek in front of the ear. Stimulating the facia l nerves in this manner leads to spasms of the loca l (ori s ) muscle (Chvosteks ign). Which drug i s most l ikely to be chosen to manage this adverse response to surgery?

a. Ca lci toninb. Ca lcium gluconatec. Pl i camycin (mithramycin)d. PTH (parathyroid hormone)e. Vi tamin D

388. A 40-year-old man with a symmetrica l ly enlarged thyroid gland associated with elevated levels of T3 and T4 i s treated with propyl thiouraci l(PTU). Which phrase best summarizes the principa l mechanism of action of PTU?

a. Blocks iodide transport into the thyroidb. Increases hepatic metabol ic inactivation of ci rculating T4 and T3

c. Inhibi ts proteolys is of thyroglobul ind. Inhibi ts thyroida l peroxidasee. Releases T3 and T4 into the blood

389. The attending in the endocrine/metabol ic diseases cl inic prescribes colesevelam as add-on therapy for a 58-year-old male with type 2diabetes who i s not getting adequate glycemic control (based on high levels of HbA1c measured twice in the las t 12 months) from his currentmedications : metformin and glyburide. The man has been a pack-a-day cigarette smoker for 40 years and says he “can’t seem to qui t.” In additionto the diabetes and nicotine addiction he has severa l other comorbidi ties that are not optimal ly managed with his current medications for each,or recommended l i fes tyle modi fications (he largely ignores them). The attending s tates that the cole-sevelam might just help one of thosecomorbidi ties in addition to poss ibly lowering HbA1c levels a fter a few months on the drug. What i s the “other condition” that i s most l ikely to befavorably and rather di rectly influenced by the colesevelam?

a. Chronic bronchi ti s (or emphysema)b. Hyperl ipidemiasc. Hyperuricemia, goutd. Nicotine addictione. Prostatism (ie, benign prostatic hypertrophy)

390. Your patient, who i s taking an ora l contraceptive, has heard about and asks about the ri sk of thromboembol ism as a resul t of taking thesedrugs . To reduce the ri sk of this potentia l ly severe adverse hematologic response, but s ti l l provide reasonably effective contraception, what wouldyou prescribe?

a. A combination product with a higher estrogen doseb. A combination product with a higher progestin dosec. A combination product with a lower estrogen dosed. A combination product with a lower progestin dosee. A product that conta ins only estrogen

391. A 27-year-old woman is diagnosed with hypercorti sm. To determine whether corti sol production i s independent of pi tui tary gland control , youdecide to suppress ACTH production by giving a high-potency glucocorticoid. Which drug would be the best choice for this purpose?

a. Dexamethasoneb. Hydrocorti sone

Page 211: Pharmacology pre test self assessment and review (14th ed.)

c. Methylprednisoloned. Prednisonee. Triamcinolone

392. A patient with Cushing syndrome is being treated by X-i rradiation of the pi tui tary. It may take severa l months of this therapy for adequatesymptomatic and metabol ic improvement. Unti l that time, which drug would be adminis tered to suppress glucocorticoid synthes is?

a. Cimetidineb. Corti sol (mass ive doses)c. Fludrocorti soned. Ketoconazolee. Spi ronolactone

393. A woman who has been taking an ora l contraceptive (estrogen plus progestin) for severa l years i s diagnosed with epi lepsy and s tarted onphenytoin. What i s the most l ikely consequence of adding the phenytoin?

a. Agranulocytos is or aplastic anemia, requiring s topping both drugs immediatelyb. Breakthrough seizures from increased phenytoin clearancec. Phenytoin toxici ty, s igni ficant and of fas t onsetd. Profoundly increased ri sk of craniofacia l abnormal i ties in the fetuse. Reduced contraceptive efficacyf. Thromboembol ism from the estrogen component of the contraceptive

394. A woman wants a prescription for an ora l contraceptive, and your choice i s between an estrogen-progestin combination and a “minipi l l ”(progestin only). Compared with a hormone combination product, what i s the main di fference that would occur when a progestin-only approach i sused?

a. Better contraceptive efficacyb. Di rect spermicida l effectsc. Higher ri sk of thromboembol ismd. More menstrua l i rregulari ties (i rregular cycle length, amenorrhea, spotting, etc)e. Poorer compl iance due to taking the drug on an i rregular cycle, rather than da i ly

395. A woman is taking a combination estrogen-progestin combination ora l contraceptive. She experiences a multi tude of s ide effects . Which s ideeffect i s most l ikely due to what can be described as an “estrogen excess ,” and not l ikely due to the progestin content of the medication?

a. Fatigueb. Hypertens ionc. Hypomenorrhead. Increased appeti tee. Weight ga in

396. A comatose patient with an endocrine disorder undergoes a computed tomography (CT) diagnostic procedure that involves contrast media thathas a tendency to induce lactic acidos is . He i s a lso taking a drug that, in i ts own right, may cause lactic acidos is . Normal ly the drug would bestopped, at least temporari ly, two days before the CT, but due to the patient’s acute and serious i l lness that could not be done. Lactic acidos isindeed develops and the patient nearly dies . Which drug was the most l ikely cause of this severe and potentia l ly fata l metabol ic derangement?

a. Insul in glargine, prescribed for type 1 diabetes mel l i tusb. Levothyroxine, prescribed to mainta in euthyroid s tatus fol lowing thyroidectomyc. Metformin, prescribed for type 2 diabetes mel l i tusd. Propyl thiouraci l , prescribed for hyperthyroidisme. Spi ronolactone, prescribed for an adrenal cortica l tumor

397. A patient with type 2 diabetes mel l i tus begins ga ining weight after severa l months of therapy with an ora l antidiabetic agent. A completework-up indicates edema and other s igns and symptoms of heart failure. Which antidiabetic drug or group was the most l ikely cause?

a. Acarboseb. Biguanidesc. Metformind. Piogl i tazonee. Sul fonylureas , both fi rs t- and second-generation agents (eg, tolbutamide, chlorpropamide, glyburide, gl ipizide)

398. A woman goes into premature labor early enough that there are great concerns about inadequate feta l lung development and the ri sk of feta lrespiratory dis tress syndrome. Sui table uterine-relaxing drug therapy i s s tarted to s low labor, but parturi tion seems imminent. What other adjunctshould be adminis tered prepartum, speci fica l ly for the purpose of reducing the ri sks and compl ications of the newborn’s immature respiratorysystem development?

a. Albuterol (β2 agonis t)

b. Betamethasonec. Ergonovine (or methylergonovine)d. Indomethacine. Magnes ium sul fate

Page 212: Pharmacology pre test self assessment and review (14th ed.)

399. We prescribe etidronate for a postmenopausa l woman who i s at great ri sk for developing for osteoporos is . Which s ide effect or adverseresponse to this drug i s the patient most l ikely to experience?

a. Cholel i thias isb. Esophagi ti sc. Fluid/electrolyte loss from profuse diarrhead. Hepatic necros ise. Renal damage from ca lcium stone formationf. Tetany

400. A 75-year-old man had surgery for prostate carcinoma, and loca l metastases were found intraoperatively. What i s the most appropriatefol low-up drug a imed at treating the metastases?

a. Aminoglutethimideb. Fludrocorti sonec. Leuprol ided. Mifepris tonee. Spi ronolactone

401. A 53-year-old woman with type 2 diabetes mel l i tus i s s tarted on glyburide. Diet, exercise, and usual ly effective doses of metformin have notprovided adequate glycemic control based on periodic measurements of HbA1c. What i s the primary mechanism by which the glyburide i s l ikely toprovide better glycemic control?

a . Decrease insul in res is tance by lowering body weightb. Enhance renal excretion of glucosec. Increase insul in synthes isd. Promote glucose uptake by muscle, l i ver, and adipose ti ssue via an insul in-independent processe. Release insul in from the pancreas

402. A 75-year-old woman with type 2 diabetes i s taking an ora l anti -diabetic drug. One day she goes without eating for 18 hours , but takes herdrug nonetheless . She i s transported to the emergency after pass ing out. Her plasma glucose concentration i s 48 mg/dL (hypoglycemic) uponarriva l at the ED, and she i s in very serious condition. Which drug, that most l ikely aggravated this fas ting hypoglycemia, did she take?

a. Acarboseb. Colesevelamc. Glyburided. Metformine. Piogl i tazone

403. A man with type 2 diabetes i s receiving a combination of ora l drugs to mainta in glycemic control . He becomes hypoglycemic one afternoon andingests some orange juice and two chocolate bars—al l conta ining abundant amounts of complex carbohydrates . Despi te his ingestion of orangejuice and severa l chocolate bars , his blood glucose levels remain low, his symptoms pers is t. Which antidiabetic that he was taking accounted forthe fa i lure of ora l sugars to restore his plasma glucose?

a. Acarboseb. Glyburidec. Metformind. Repagl inidee. Ros igl i tazone

404. A 35-year-old woman has Graves disease, a smal l goi ter, and symptoms that are deemed “mi ld-to-moderate.” Propyl thiouraci l i s prescribed.What i s the most serious adverse response to this drug, for which close monitoring i s required?

a. Agranulocytos isb. Cholestatic jaundicec. Goutd. Renal tubular necros ise. Rhabdomyolys isf. Thyroid cancer

405. A 60-year-old man with type 2 diabetes mel l i tus i s treated with piogl i tazone (in addition to a proper diet and exercise). Which phrasesummarizes best this drug’s main mechanism of action?

a. Blocks intestina l carbohydrate absorptionb. Causes glycosuria (increased renal glucose excretion)c. Increases hepatic gluconeogenes isd. Increases release of endogenous insul ine. Increases target ti s sue sens i tivi ty to insul in

406. A patient with a his tory of type 2 diabetes mel l i tus presents in the ED. His compla ints include nonspeci fic gastrointestina l symptoms,including nausea and vomiting. He s tates he i s bloated and has abdominal pa in. His appeti te has been suppressed for severa l days . He hasmala ise and di ffi cul ty breathing. His l iver i s enlarged and tender; l i ver function tests indicate hepatic damage. Plasma bicarbonate i s low and

Page 213: Pharmacology pre test self assessment and review (14th ed.)

lactate levels are high. Kidney function i s fa l l ing rapidly.

The diagnos is i s lactic acidos is , and the suspicion i s that i t was caused by an antidiabetic drug. Which of the fol lowing drugs i s this patientmost l ikely to be taking?

a. Acarboseb. Gl ipizidec. Glyburided. Metformine. Piogl i tazone

407. A patient i s transported to the ED shortly after taking a mass ive overdose of her levothyroxine in an apparent suicide attempt. Which of thefol lowing drugs should be adminis tered for prompt control of the hormone-related effects that are most l ikely to lead to her death i f not correctlymanaged?

a. Iodine/iodideb. Liothyroninec. Propranolold. Propyl thiouraci le. Radioiodine (131I)

408. There are two main formulations of ora l contraceptives : those that are estrogen-progestin combinations , and those that conta in onlyprogestin (“minipi l l ”). What i s the main mechanism by which these drugs exert thei r des i red contraceptive effects?

a. Acidi fy the cervica l mucus , thereby making the mucus spermicida lb. Displace/detach a ferti l i zed egg from the endometriumc. Inhibi t nidation (implantation of a ferti l i zed ovum)d. Inhibi t ovulatione. Reduce uterine blood flow such that the ferti l i zed ovum becomes hypoxic and dies

409. A 55-year-old postmenopausa l woman develops weakness , polyuria , polydips ia , and s igni ficant increases of plasma creatinineconcentration. A computed tomogram (CT scan) indicates nephroca lcinos is . A drug i s cons idered to be the cause. Which drug was most l ikelyrespons ible?

a. Es trogensb. Etidronatec. Gl ipizided. Prednisonee. Vi tamin D

410. A patient has severe Cushing disease. Surgery cannot be scheduled for severa l months , so the phys ician plans to treat the patient in theinterim with a drug that she describes as a “potent inhibi tor of corticosteroid synthes is .” Which drug best fi ts that description?

a. Dexamethasoneb. Hydrocorti sonec. Ketoconazoled. Prednisonee. Spi ronolactone

411. You prescribe bromocriptine for a woman with primary amenorrhea. Normal menstruation returns about a month after s tarting therapy. Whichstatement best describes the mechanism by which bromocriptine caused i ts des i red effects?

a. Blocked estrogen receptors , enhanced gonadotropin releaseb. Increased fol l i cle-s timulating hormone (FSH) synthes isc. Inhibi ted prolactin released. Stimulated ovarian estrogen and progestin synthes ise. Stimulated gonadotropin-releas ing hormone (GnRH) release

412. A patient has had his parathyroid glands excised during a tota l thyroidectomy. In addition to requiring supplementa l thyroid hormone,interventions a imed at correcting hypoparathyroidism wi l l be necessary. What i s the main phys iologic action or role of parathyroid hormone—onethat necess i tates sui table therapy?

a. Decreases active absorption of Ca from the smal l intestineb. Decreases excretion of phosphatec. Decreases renal tubular reabsorption of ca lciumd. Decreases resorption of phosphate from bonee. Increases mobi l i zation of ca lcium from bone

Page 214: Pharmacology pre test self assessment and review (14th ed.)

Endocrine and Reproductive Pharmacology

Answers

365. The answer is a. (Brunton, pp 731-736, 798; Katzung, pp 183-185, 299.) Pers is tently high plasma glucose and cholesterol levels obvious ly have manyuntoward consequences , one of which i s renovascular disease that may lead to eventual renal fa i lure and death. He a l ready hasmicroalbuminuria , so some renal damage i s l ikely present. When the arteria l supply to both kidneys i s reduced because of s tenos is (or otherfactors ), that weighs aga inst us ing an ACE inhibi tor or an ARB. Angiotens in I I has a cri ti ca l role in mainta ining glomerular fi l tration by increas ingefferent arteriolar pressure. Blocking A-II synthes is , or i ts receptor-mediated vasoconstrictor effects , can s igni ficantly reduce renal perfus ion andGFR and, eventual ly and sometimes precipi tous ly, lead to renal fa i lure. ACE inhibi tors or ARBs do not cause bradycardia (b), whether di rectly or viabaroreceptor-related reflex mechanisms. Prior episodes of hyperosmolar hyperglycemic nonketotic syndrome (HHNKA, c) reflect periods of severe,symptomatic hyper-glycemia (and poor glycemic control ). In the absence of bi latera l renal disease or other renal pathologies (or i f the patient i snot a woman who i s pregnant or of chi ld-bearing potentia l ), an ACE inhibi tor or ARB would be a logica l choice for control l ing hypertens ion; heartfa i lure does not contra indicate (d) the use of these medications . Nei ther hypercholesterolemia or hypertriglyceridemia (or both), nor the use ofniacin or other drugs for dys l ipidemias (e), has any impact on the decis ion to use an ACE inhibi tor or ARB in the absence of othercontra indications .

366. The answer is b. (Brunton, pp 788, 1248, 1272; Katzung, pp 182-183.) Ora l diazoxide i s indicated for the very s i tuation described in the question. Iti s , chemica l ly, a benzothiadiazide (l ike hydrochlorothiazide, many other diuretic/antihypertens ives ), but i t i s more efficacious in terms of reducingparenchymal cel l (skeleta l muscle, adipocytes ) respons iveness , or sens i tivi ty to insul in, thereby helping to ra ise blood glucose levels . (Reca l l thatthiazides tend to cause hyperglycemia as a s ide effect. That i s due to thei r insul in-desens i ti zing effects .) Nei ther diazoxide or a thiazide diuretichas any s igni ficant effects to inhibi t insul in synthes is , release, or blood levels . Indeed, they tend to cause hyperinsul inism in response tohyperglycemia.

Atenolol , metoprolol (a ), and other drugs that block β-adrenergic receptors tend to suppress insul in release, cause hypoglycemia, delay recoveryfrom hypoglycemic episodes by inhibi ting hepatic glycogenolys is and suppress ing insul in release, and block tachycardia (a tip-off to somediabetic patients that thei r blood glucose levels are getting qui te low) that tends to occur in response to hypoglycemia. (The degree or extent towhich β-adrenergic blockers exert thei r effects on glucose depends on the overa l l profi le of the individual agents—nonselective [propranolol ,others ]; atenolol or metoprolol [largely β1 selective]; labeta lol or carvedi lol [nonselective β- plus α1 blockade]; pindolol [nonselective β-blockadewith intrins ic sympathomimetic activi ty]).

Glucagon (c) i s used to manage hypoglycemia (as a supplement to intravenous glucose adminis tration), but i t i s an injectable drug used only asan adjunct (to IV glucose) for short-term, severe, and symptomatic hypoglycemia. Octreotide (d), identi fied as a somatostatin analog, i s used toinhibi t growth hormone (GH) release and the effects of GH on insul in-l ike growth factor (IGF-1) in such cases as acromegaly and carcinoidsyndrome, and i s a lso used to control bleeding from esophageal varices . It i s about 40% to 50% more potent than somatostatin in terms ofsuppress ing GH release, but relatively weak in terms of suppress ing insul in secretion. It would not be a reasonable choice here, given theal ternatives . Oxytocin (e), a posterior pi tui tary hormone used mainly as a uterine s timulant (that’s where we got the terms oxytocic drug or oxytociceffect), has no cl inica l ly useful effects on glycemic control or the responses of target ti s sues to insul in or glucose deficiency or excess .

367. The answer is c. (Brunton, pp 1244-1256; Katzung, pp 761-762, 764-765.) Blood-borne glucose reacts nonenzymatica l ly with hemoglobin to formglycated hemoglobin products (eg, HbA1c). The rate of HbA1c formation i s related to ambient glucose levels , and the amount of HbA1c measured inany given blood sample reflects the average blood glucose levels over the las t 2 to 3 months . Thus , a l though the patient’s plasma glucose levelsmay be acceptable after a couple of days ’ fas t, HbA1c measurements give the big picture about how good glycemic control was on a more long-term(and more important) timel ine. Note that a l though measuring HbA1c gives important information about the long-term, i t does not provide anyinformation about day-to-day fluctuations in glucose levels or what’s happening “right now,” and such information i s important to optimal controlof diabetes and i ts symptoms. Nonetheless , regular, periodic checks of HbA1c should be part of the monitoring for every patient with diabetes(type 1 or 2), not so much as a way to assess for noncompl iance as to make sure that the current treatment plan with which the patient i scomplying i s working. It i s a common and relatively inexpens ive assay.

Having the cl inica l lab measure glucose in a venous blood sample won’t give any additional or meaningful information: handheld glucometers ,used properly, are remarkably accurate. Glucose tolerance tests , even those done with ora l glucose, wi l l provide l i ttle his toric information, andthey are expens ive. Few cl inica l labs are set up to measure plasma concentrations of most ora l antidiabetic drugs , and the cost for thesenonstandard tests would be qui te expens ive. Measuring urine ketone levels are of no benefi t for our purposes . Al l that they might prove i s thatour patient has fasted for severa l days . Urine glucose monitoring i s not very enl ightening ei ther. Reca l l that glucose appears in the urine onlywhen plasma concentrations exceed a renal threshold for reabsorption (around 180 mg/dL or so). A glucose-free urine sample, then, would onlyindicate that plasma glucose levels are below the threshold: they s ti l l could be unacceptably high, or normal , or low, and you would never knowjust by urine testing.

368. The answer is c. (Brunton, pp 307, 1829-1830, 1841; Katzung, pp 738, 1077.) Finasteride competi tively inhibi ts s teroid 5-reductase, the enzymenecessary for synthes is of the active form of testosterone (dihydrotestosterone) in the prostate. Testosterone synthes is (d) and ci rculatingtestosterone levels (e) do not fa l l in response to the drug. PSA ti ters wi l l , however, fa l l .

Reca l l that α1-adrenergic blockers (eg, prazos in as the overa l l prototype and, particularly, tamsulos in in terms of use for BPH) providesymptomatic rel ief in some men with enlarged prostates by relaxing smooth muscle in the urethra , bladder neck, and prostate capsule; moreover,they do so without any effects on testosterone or i ts cel lular actions . Finasteride, however, has no α-adrenergic receptor blocking activi ty (a ).

369. The answer is e. (Brunton, pp 1264-1265; Katzung, pp 760-761.) DPP-4 degrades incretins , which play a crucia l role in regulating insul in andglucagons release, particularly in response to a meal . The DPP-4 inhibitors (ie, s i tagl iptin and other “gl iptins”) thereby indirectly ra ise blood levelsof endogenous glucagon-l ike peptide-1 (GLP-1). The two most important consequences of the now-ra ised GLP-1 levels are an increase of bloodglucose-mediated insul in release, and a s imultaneous reduction of glucagon release. The gl iptins are ora l agents used as adjuncts to thetreatment of type 2 diabetes in certa in patients who fa i l to get adequate glycemic control in response to exercise, proper diet, and moretradi tional ora l antidiabetic agents (ie, metformin, a sul fonylurea, or even piogl i tazone, a lone or in combination).

DPP-4 inhibi tors have no actions on, or cl inica l uti l i ty related to, s teroidogenes is (a ), cleavage of insul in from preinsul in (b), thyroid hormonestorage or release (c), or estrogen or progesterone metabol ic pathways (d).

Page 215: Pharmacology pre test self assessment and review (14th ed.)

370. The answer is d. (Brunton, pp 310-316, 320, 1150; Katzung, pp 687t, 691-694.) In thyrotoxicos is/thyroid s torm, essentia l ly the only effect ofadminis tering a β-adrenergic blocker—and i t i s an important effect, to be sure—is to provide prompt rel ief of both relatively innocuousmani festations of thyroid hormone excess , such as tremor, and those that are much more dangerous , including s igni ficant and potentia l ly l i fe-threatening increases in cardiac rate, contracti l i ty, and automatici ty. Ci rculating thyroid hormone levels modulate the respons iveness of β-adrenergic receptors to thei r agonis ts (eg, epinephrine, norepinephrine). When thyroid hormone levels are excess ive, so i s adrenergic receptorrespons iveness . Therefore, we can reduce the adrenergic consequences of the hormone excess us ing a β-blocker. There are no effects on thyroidhormone levels or thyroid gland function or control .

None of the β-adrenergic blockers block the agonis t effects of thyroid hormones on their receptors (a), nor do they inhibi t thyroid hormonerelease (b) or synthes is (c). They do not a l ter TSH levels (e) di rectly or indi rectly.

371. The answer is e. (Brunton, pp 1294-1296, 1299-1301; Katzung, p 776.) Whether used for osteoporos is (prevention or management, men or women,idiopathic or drug-induced) or Paget disease of the bone, bisphosphonates exert thei r effects on osteoclasts and osteoblasts . The drug i sincorporated into bone. When drug-conta ining bone i s resorbed by the osteoclasts , osteoclast function (and, so, subsequent bone resorption) i sinhibi ted. The bisphosphonates a lso recrui t osteoblasts , which then produce a substance that further inhibi ts osteoclast activi ty.

Bisphosphonates in genera l , or a lendronate in particular, do not activate vi tamin D (a); are not ca lcium-rich drugs (b) nor phosphate-rich drugs(d); and they do not form or s timulate formation of hydroxyapati te crysta ls in bone (c).

372. The answer is d. (Brunton, pp 1278-1280, 1290, 1294-1297; Katzung, pp 775, 780, 783.) You should be able to deduce hyperparathyroidism as theanswer by reca l l ing that the bisphosphonates ul timately affect bone Ca 2+ metabol i sm, and the parathyroid gland i s the main regulator of plasmaCa 2+ levels . When a bisphosphonate i s given to a patient with Paget disease of the bone, a fter about a week the dramatic inhibi tion of boneresorption markedly lowers plasma Ca 2+ levels (because an important portion of plasma Ca 2+ ari ses from bone that i s being resorbed). Whenplasma Ca 2+ fa l l s cons iderably, parathyroid hyperfunction can develop. The eas iest way to prevent this i s to adminis ter dietary ca lciumsupplements a long with the bisphosphonate.

373. The answer is d. (Brunton, pp 1214, 1233-1236; Katzung, p 710.) Metyrapone, because i t decreases plasma levels of corti sol by inhibi ting the 11β-hydroxylation of s teroids in the adrenal , can be used to assess function of the pi tui tary-adrenal cortica l axis . When metyrapone i s given to normalpersons , the adenohypophys is secretes more ACTH. This causes a normal adrenal cortex to synthes ize increased amounts of 17-hydroxylatedsteroids , which can be measured in the urine. However, patients who have disease of the hypothalamic-pi tui tary axis do not produce ACTH inresponse to metyrapone. As a resul t, we find no increased levels of the s teroids in the urine. Before adminis tering metyrapone we need to testrespons iveness of the adrenal cortex to respond to adminis tration of ACTH.

374. The answer is e. (Brunton, pp 1007, 1224-1233, 1668-1669; Katzung, pp 697, 700f, 703, 712, 775.) These findings are characteris tic of what one wouldexpect with long-term (and/or high dose) systemic glucocorticoid therapy (ie, prednisone and many others , but not beclometha-sone, which i sgiven by ora l inhalation and i s not absorbed appreciably). Psychoses , peptic ulceration with hemorrhage (coffee-grounds s tool , indicative ofgastric bleeding) or without (poss ibly caus ing guaiac-pos i tive s tools ), increased susceptibi l i ty to infection, edema, osteoporos is , myopathy, andhypokalemic a lka los is can occur. Other adverse reactions include cataracts , hyperglycemia, s lowed l inea l growth in chi ldren, and iatrogenicCushing syndrome.

Hydrochlorothiazide (and other thiazide and thiazide-l ike diuretics , such as chlortha l idone or metolazone) can increase blood glucose levels .However, they typica l ly lower blood pressure (as evidenced by their widespread use as antihypertens ives ) and tend to ra ise, not lower, plasmacalcium levels (which would be incons is tent with the decreased bone dens i ty described in this man). None of the other drugs l i s ted would causea col lection of findings cons is tent with what I described here.

375. The answer is e. (Brunton, pp 834-837, 1153t; Katzung, pp 242-243, 610t, 693-694.) Amiodarone, an iodine-rich drug, has severa l actions that can leadto cl inica l hypothyroidism—or, more often and mainly in persons with iodine-deficient diets , hyperthyroidism. How can these seemingly oppos i teoutcomes occur? Before we get to that, let’s summarize the other adverse responses associated with amiodarone therapy: the potentia l forpulmonary fibros is (may become serious); abnormal pigmentation of the skin (i s photosens i tive, too); corneal depos i ts (often described as “ha lovis ion”); neuropathies ; and hepatotoxici ty. Now, the thyroid i s sue: amiodarone inhibi ts a deiodinase that removes iodine on both the 5 and 5′pos i tions and that converts thyroxine to tri iodothyronine (T3), mainly in the l iver. This process i s the main contributor to the production ofendogenous (ci rculating) T3 that i s used by most target ti s sues in the body. Inhibi t this enzyme and the periphera l ti s sues have less T3 to uti l i ze,and s igns and symptoms of hypothyroidism can ensue.

The excess iodine derived from metabol i sm of amiodarone may a lso contribute to the hypothyroidism. The mechanism is analogous to the wayin which adminis tering large doses of iodide are cl inica l ly useful for suppress ing thyroid function in hyperthyroid individuals : iodide l imits i tsown transport into fol l i cular cel l s , and, acutely at least, high ci rculating levels of iodide inhibi t thyroid hormone synthes is .

And how might amiodarone cause hyperthyroidism? The iodine cleaved from the drug during i ts metabol i sm can be incorporated, a long withdietary iodine, into thyroid hormone synthetic pathways .

Al though amiodarone can cause cl inica l ly s igni ficant effects on thyroid s tatus , i t does not affect the metabol i sm or responses to otherhormones , including corti sol (a , b), insul in (d, f), and ADH (c).

376. The answer is b. (Brunton, pp 952, 1184-1185, 1837; Katzung, pp 710-711, 731-732.) Mifepris tone (perhaps better known as RU-486) i s a synthetic drugused as an aborti facient. (In some other countries , but not in the United States , i t i s used as a postcoi ta l contraceptive.) Al though the drug blocksglucocorticoid receptors , that effect does not account for i ts use or effects in the context described here. Here the actions ari se from blockinguterine progesterone receptors . They include detachment of the conceptus from the uterine wal l , softening and di lation of the cervix, andincreased myometria l contraction that expels the conceptus . (The latter ari ses from both a drug-induced increase of loca l prostaglandin synthes isand greater myometria l respons iveness to them.) If mi fepris tone fa i l s to induce expuls ion of the fetus , misoprostol i s usual ly given to increaseuterine contractions further. (Es trogens used a lone or in combination with progestins have a lso proven effective in postcoi ta l contraception.)

Ergonovine and methylergonovine (a) are ergot compounds that cause uterine contraction, and are used postpartum to control bleeding, byincreas ing uterine tone, mainly in the face of postpartum uterine atony. They are aborti facient drugs , but not used for postcoi ta l contraception.Ra lxoi fene (c) and tamoxi fen (e) are estrogen receptor agonis ts or antagonis ts (which effect occurs depends on the ti ssue) that are used to treatcerta in estrogen-dependent breast cancers .

Ri todrine (d) i s a β2-adrenergic agonis t that has been used to s low uterine contractions and reduce overa l l uterine tone in premature labor.

377. The answer is e. (Brunton, pp 1154-1157; Katzung, pp 689, 692.) Hypothyroidism is the most common adverse outcome of us ing 131I to treat

Page 216: Pharmacology pre test self assessment and review (14th ed.)

hyperthyroidism. No matter how accurately the dose i s ca lculated, according to some studies about 8 of 10 treated patients develop symptomatichypothyroidism (that needs to be treated) by 10 or more years after treatment. It resul ts , of course, from excess ive thyroid cel l destruction.

Substantive symptom rel ief from 131I takes from severa l weeks to a couple of months to develop (a l though blood chemistries change somewhatfaster). Unti l a euthyroid s tate develops , drugs such as β-adrenergic blockers or ora l anti thyroid drugs may be needed for symptom control . Ora lanti thyroid drugs can be given before 131I treatment, but they should be s topped for a couple of days before radiotherapy so as not to preventradioiodine uptake into the gland. Thyroid and other cancers , metastatic or not, are rare after 131I therapy.

378. The answer is c. (Brunton, pp 1258-1260, 1841, 1845; Katzung, pp 757, 766.) Polycystic ovarian syndrome (PCOS) i s the most common endocrinedisorder in young postpuberta l women, and i s characterized by not only the s igns and symptoms noted in the question, but a lso others that areaccompanied or caused by relative androgen excess (thus el iminating testosterone, e, as an answer). Just as metformin i s often helpful in type 2diabetes , through i ts insul in-sens i ti zing effects on parenchymal cel l s (adipocytes , skeleta l muscle), so i t i s in PCOS; an added benefi t, indi rectly,i s a reduction of ci rculating androgen levels . Higher dosages of estrogen (a) might help the s igns and symptoms of PCOS, but obvious ly thatapproach would not preserve or improve ferti l i ty as long as the drug i s being taken. Ketoconazole (b), a powerful inhibi tor of s teroidogenes is , i stoo nonselective in terms of i ts powerful inhibi tory effects on s teroido-genes is . Prednisone (d) or another glucocorticoid would be i rrational , i f forno other reason than the fact that these drugs wi l l further ra ise blood glucose levels—a particularly unwanted effect cons idering the parenchymalcel l insul in res is tance that i s a component of PCOS. Testosterone (e) i s obvious ly inappropriate given the relative androgen excess a l ready presentin PCOS.

379. The answer is a. (Brunton, pp 1172, 1177-1180, 1299-1300, 1756-1759; Katzung, pp 731, 740.) Tamoxi fen i s often referred to as a selective estrogenreceptor modi fier (SERM). It blocks estrogen receptors in some ti ssues and s timulates them in some others . The drug can be used to prevent ortreat estrogen-dependent breast cancers , and i t works as an estrogen receptor antagonis t there. A receptor-agonis t action a lso accounts for one ofthe drug’s more dis tress ing and common s ide effects , hot flashes . In contrast, the drug activates estrogen receptors in the uterus , increas ing therisk of endometria l cancers . Other estrogen-activating (estrogen-l ike) consequences include a reduced ri sk of osteoporos is , des i rable changes inplasma cholesterol profi les (reduced LDL and tota l cholesterols ; increased HDL), and an increased ri sk of thromboembol ic events . The relateddrug, ra loxi fene, i s largely tamoxi fen-l ike with one main exception: i t does not activate uterine estrogen receptors , and so does not increase therisk of endometria l cancers .

380. The answer is c. (Brunton, pp 1258-1260; Katzung, pp 757, 766.) Metformin, class i fied as a biguanide, “sens i ti zes” periphera l cel l s (skeleta l muscle,adipocytes ) to insul in, thereby faci l i tating glucose uptake and metabol ic use, and suppresses release of glucose from the l iver and into the blood.It i s ineffective in the absence of insul in and so i s approved only for type 2 diabetes (used a lone or in conjunction with such other drugs as asul fonylurea or insul in). Like other ora l agents (and some parentera l drugs such as exenatide), metformin has no beneficia l effect in patients withtype 1 diabetes (d).

Weight loss during metformin therapy i s much more common than weight ga in (e). This i s probably due to an appeti te-suppress ing effect(leading to reduced ca loric intake), rather than because of a speci fic drug-related effect on some metabol ic reaction(s ) or anorexia secondary to GIs ide effects .

Why does metformin seldom—if not rarely—cause hypoglycemia? Rather than actively driving down blood glucose levels (as , say, insul in does),the drug acts as i f i t “puts a l id” on phys iologic ri ses of blood glucose concentrations . (Thus , i t has been described as being an antihyperglycemicdrug, rather than a hypoglycemic agent.)

Metformin i s not metabol i zed by the l iver, but l iver dys function i s one contra indication. That’s mainly because of the ri sks of the drug’s mostimportant adverse effect, lactic acidos is , which i s rare but often fata l (not common and seldom serious , answer b) when i t does occur. Impairedl iver function impairs lactate el imination and favors i ts accumulation to toxic levels (and so answer a i s not correct).

However the main primary cause of the lactic acidos is i s renal insufficiency (plasma creatinine >1.5 mg/dL in men, 1.4 mg/dL in women),whether caused by renal disease or by renal hypoperfus ion (i schemia, as might occur with heart fa i lure and/or hypotens ion).

381. The answer is a. (Brunton, pp 1239f, 1251f, 1252-1253; Katzung, pp 743-753.) Insul in modi fications , whether by rDNA technology to substi tute aminoacids , or by phys ica l means (eg, in the case of NPH insul in), a l ter onsets and durations of action. (Reca l l that regular insul in, given by SC injection,has an onset of about 30 minutes , peaks in about 3 hours , and has a duration of about 7-8 hours . Li spro and aspart insul ins work faster, peakearl ier, and have the shortest durations [rapid/short]; NPH insul ins [“intermediate-acting”] have onsets , times to peak, and durations longer thanregular insul ins ; and insul in glargine has the s lowest onsets [4-6 hours ], wi th durations that are on par with NPH.)

In genera l , the modi fied human insul ins rarely cause a l lergic responses (not the case with the few other insul ins ). Beyond that, however, themodi fications yield cl inica l ly useful pharmacokinetic—not biochemica l—changes .

382. The answer is a. (Brunton, p 1265; Katzung, pp 759-760.) Acarbose and the newer related drug, migl i tol , act in intestina l brush border cel l s toinhibi t monosaccharide formation from complex carbohydrates and ol igosaccharides , and in so doing inhibi t/s low the absorption of monosaccha-rides (eg, glucose) from the gut. The enzymatic targets are α-glucos idases in the brush border of intestina l cel l s . The main consequence in termsof glycemic control i s a blunting of usual postprandia l ri ses of blood glucose. The most common s ide effects of acarbose affect the GI tract, asdescribed in the question. Used a lone, hypoglycemia due to acarbose i s rare; however, the drug i s often used as an adjunct to a sul fonylurea, andtheir propens i ty for caus ing hypoglycemia i s not at a l l reduced by acarbose. It i s important to note that the usual patient-centered approach tomanaging an episode of hypoglycemia, as might occur when certa in ora l antidiabetic drugs are used (eg, sul fonylureas , gl i tazones), i s to ingesttable sugar (sucrose) or some beverage or food that conta ins i t. If the hypoglycemic patient i s taking acarbose, this approach wi l l not correct bloodglucose levels adequately or promptly.

383. The answer is a. (Brunton, pp 1250-1252; Katzung, pp 743-745.) Insul in glargine i s a genetica l ly engineered insul in that i s poorly (but adequately)soluble at phys iologic pH va lues . The drug dissolves and enters the bloodstream s lowly and in such a way that there i s more of a s table plateauin terms of blood levels and hypoglycemic effects . It does not cause a wel l -defined peak in blood levels and effects (a ). The plateau i s rather“flat” over the typica l 24-hour period fol lowing a subcutaneous injection. Thus , the drug i s better able than most other insul ins to mainta in round-the-clock glycemic control , but less able to suppress the postprandia l elevation of glucose that i s necessary for some patients . Insul in glargine’sonset and duration of action are comparable to such preparations as NPH insul in. It i s what happens in between, tempora l ly, that dis tinguishesthis formulation from the rest.

Insul in glargine—indeed a l l the insul in formulations—has no disul fi ram-l ike effects (b). As with a l l the insul ins , insul in glargine poses a ri skof hypoglycemia (and so answer d i s incorrect). Fina l ly, the drug has no parenchymal cel l insul in-sens i ti zing effects (e), and l ike a l l insul ins i t i sgiven solely as hormone replacement therapy.

Page 217: Pharmacology pre test self assessment and review (14th ed.)

384. The answer is c. (Brunton, pp 1150-1151; Katzung, pp 692-693.) Tachycardia , pa lpi tations , tachyarrhythmias , and the poss ibi l i ty of acute myocardia li schemia are among the ha l lmarks of thyrotoxicos is or thyroid hormone overdoses . Thyroid hormone levels regulate the “respons iveness” of β-adrenergic receptors to epinephrine and norepinephrine (or exogenous β-agonis ts ), and this probably contributes to the cl inica l picture. Al thoughexcesses of thyroid hormone are the cause of the problems, management i s symptomatic and a imed at suppress ing β-mediated responses . Hencethe use of propranolol or some other sui table β-adrenergic blocker for immediate and qui te poss ibly l i fesaving symptom control . Sul fonylureas (a ;fi rs t or second generation) would not cause the s igns or symptoms noted above, and β-blocker therapy for overdoses of one of those drugs woulddo more harm than good. The same appl ies to insul in (b); β-blockers might control increases of cardiac rate, but they are more l ikely to furtherlower blood glucose levels and s low or otherwise attenuate ri ses of blood glucose levels when appropriate therapy to render the patienteuglycemic i s s tarted. Overdoses of prednisone (d) or propyl thiouraci l (e) are not l ikely to present as described, and they are not properlymanaged with a β-blocker.

385. The answer is e. (Brunton, pp 1261-1264, 1930t; Katzung, pp 761-762.) Exenatide was the fi rs t in a new class of antidiabetic agents , the incretinmimetics. It i s adminis tered by subcutaneous injection and used adjunctively to improve glycemic control in type 2 diabetes patients a l ready takingmetformin, a sul fonylurea, or both. It i s not indicated for type 1 diabetes .

Exenatide i s a synthetic analog of glucagon-l ike peptide-1 (GLP-1), a peptide hormone in the incretin fami ly. Under phys iologic conditions , GLP-1and other incretins are released from cel l s of the GI tract a fter a meal . Exenatide binds to receptors for GLP-1. In doing so i t s lows gastric emptying,s timulates glucose-dependent insul in release, inhibi ts postprandia l release of glucagon, and suppresses appeti te.

Dose-related hypoglycemia i s common when exenatide i s combined with a sul fonylurea (but not when used with only metformin, which seldomcauses hypoglycemia). Nausea, vomiting, and diarrhea are the most common s ide effects . The drug i s a lso apparently l inked to the developmentof pancreati ti s , and so monitoring for i t i s essentia l .

Some patients taking exenatide develop antibodies di rected aga inst the drug. Antigen-antibody reactions to exenatide do not cause adverseeffects (eg, a l lergic or anaphylactoid reactions), but can reduce the intens i ty of exenatide’s effects . At this time, the drug i s class i fied in FDAPregnancy Risk Category C. Given the years of experience with us ing insul in for diabetes during pregnancy, and the benefi ts and safety i t provides ,there’s l i ttle justi fi cation in us ing exenatide.

Incretin and incretin mimetics have no effects on testosterone (whether antagonism or intens i fication; answer a). Metformin and sul fonylureasare, of course, antidiabetic drugs and are not indicated for thyroid disorders (b). Exenatide has no effects on ca lcium metabol i sm or osteoporos ismanagement (c); and no di rect effects on corticosteroid metabol i sm (d), whether in hyposecretory (Addisonian) or hypersecretory (cushingoid)s tates .

386. The answer is a. (Brunton, pp 1141-1143; Katzung, pp 687t, 690-691.) Levothyroxine i s usual ly cons idered the fi rs t choice for long-term maintenancetherapy of hypothyroidism, such as that which occurs after thyroidectomy or radiation therapy of the thyroid. Once absorbed, we get the s low-onsetand rather s teady, long-lasting effects of the T4. In addi tion, some of the adminis tered T4 i s deiodinated in the ti ssues to T3, “automatica l ly”providing usual ly adequate levels and effects of this more rapidly and shorter-acting hormone without the need to adminis ter T3 (l iothyronine)separately.

If the patient develops profound hypothyroidism (eg, myxedema), some cl inicians may s ti l l prefer T3 for i ts prompt effects . However, in thisinstance too, T4 i s genera l ly regarded as the preferred way to replace both hormones and reestabl i sh a euthyroid s tate (adminis tered withcorticosteroids for myxedema coma).

Liothyronine (b) i s synthetic T3; l iotrix (c) i s a mixture of synthetic T3 and T4; proti rel in (d) i s a synthetic tripeptide, chemica l ly identica l tothyrotropin-releas ing hormone (TRH), that i s used to diagnose some thyroid disorders ; des iccated thyroid products (e) are outmoded for managinghypothyroidism, given the ava i labi l i ty of synthetic hormone preparations .

387. The answer is b. (Brunton, pp 1291, 1301, 1864; Katzung, pp 259, 779-780.) We’re deal ing with hypocalcemia in this patient with tetany post-thyroidectomy, because the parathyroid gland was damaged or removed during the surgery. Adminis tration of intravenous ca lcium gluconatewould immediately correct the tetany. Parathyroid hormone (d) could be cons idered appropriate for long-term management of hypocalcemia, but i thas a s lower onset of action that would not be of much help in a tetanic s tate. We would then probably use vi tamin D (e) and dietarymodi fications for long-term control of plasma ca lcium levels .

Ca lci tonin (a) i s a hypocalcemic antagonis t of parathyroid hormone. (Note that ca lci tonin or ca lci triol wi l l ra ise plasma ca lcium levels fas terthan vi tamin D, but in this emergent s i tuation they wi l l not work fast enough, which i s why we need di rect adminis tration of ionic ca lcium.)Pl icamycin (c; mithramycin) i s used to treat Paget disease and hyperca lcemia. The dose employed i s about one-tenth the amount used forpl icamycin’s cytotoxic action when i t i s used as a chemotherapeutic drug.

388. The answer is d. (Brunton, pp 1146-1157; Katzung, pp 688, 691-692, 695.) Propyl thiouraci l , a thioamide use to manage many patients withhyperthyroidism, has three main actions . It inhibi ts a peroxidase, and in doing so inhibi ts oxidation of inorganic iodide to iodine, and theiodination of tyros ine. It a l so blocks coupl ing of iodotyros ines and inhibi ts deiodination of T4 to T3 in the periphery. Propyl thiouraci l , and thethioamides in genera l , do not: block thyroid uptake of iodide (a); a ffect metabol ic inactivation of T3 and T4 (b); inhibi t thyroglobul in’s proteolys is(c); or di rectly release thyroid hormones into the ci rculation (e).

Note: Most texts and other publ ications refer to this group of drugs as I have: thioamides . However, occas ional ly you wi l l find an a l ternativeterm used: thionamide, thioamine, and even thionamine.

389. The answer is b. (Brunton, pp 898-899; Katzung, pp 629-630, 633, 760.) Colesevelam’s origina l approved indication, and i ts most common use, i s formanaging hypercholesterolemia. It i s class i fied as a bi le acid sequestrant: i t binds the cholesterol -rich bi le acids in the gut, preventing theirenterohepatic reci rculation. The main outcome is lowered LDL levels . In early 2008, i t was approved as an adjunct for some patients with type 2diabetes , such as the gentleman described here. Its antihyperglycemic mechanism of action i s not ful ly understood. There are other bi le acidsequestrants (cholestyramine, colestipol ), but they are not indicated for not apparently effective as an adjunct to therapy of type 2 diabetes . Thereis at least one probable advantage of colesevelam over the a l ternative bi le acid sequestrants . Cholestyramine and colestipol bind to and inhibi tthe absorption (rate, extent, or both) of many other ora l drugs that are in the GI tract at the same time. That i s , they participate in many drug-druginteractions . They a lso tend to bind to and inhibi t absorption of fat-soluble vi tamins (A, D, E, and K), whether from dietary or vi tamin supplementsources . Colesevelam does not s igni ficantly inhibi t the absorption of other ora l medications or fat-soluble vi tamins .

Based on what i s known about colesevelam’s mechanism of action there i s nothing to suggest that i t has any beneficia l effects on thepathophys iol -ogy or cl inica l course of COPD (a), hyperuricemia (c), nicotine (or any other) addiction/dependency (d), or prostate disease (benign ornot).

See Question 256 in the Chapter on Cardiovascular Pharmacology for more information about colesevelam.

Page 218: Pharmacology pre test self assessment and review (14th ed.)

390. The answer is c. (Brunton, pp 1169-1170, 1174-1175, 1669-1670, 1833-1834; Katzung, pp 725t, 726-731.) Thromboembol ism associated with OCadminis tration i s attributed to the estrogen component. If thromboembol ism is a concern, then one approach to reducing the ri sk would beselecting a product with a lower—not higher (e)—estrogen dose. (Us ing estrogen a lone [e] would be i rrational , then; that partia l ly expla ins whythere are no estrogen-only ora l contraceptives .) Even reducing the estrogen dose in a combination contraceptive may not be sufficient, long term,to provide a sui table reduction in thromboembol ic ri sk. Progestins seem to have l i ttle impact on the development of thromboembol ic disordersduring OC therapy, and so increas ing (b) or decreas ing (d) progestin content wi l l do l i ttle more than nothing for most patients .

Smoking i s a major ri sk factor for thromboembol ism, especia l ly for women taking an ora l contraceptive, so i t’s essentia l to encourage thepatient to qui t (or not s tart) smoking. As noted above there are no OCs that conta in estrogen only—unl ike the ava i labi l i ty of progestin-only OCs—the so-ca l led minipi l l s (see Question 394).

391. The answer is a. (Brunton, pp 1228, 1233; Katzung, pp 703t, 712.) Of the drugs l i s ted, dexamethasone i s by far the most potent in terms of relativeglucocorticoid effects . The dexamethasone suppress ion test has severa l uses : i t a l lows not only complete suppress ion of pi tui tary ACTHproduction, but a lso accurate measurement of endogenous corticosteroids , such as 17-ketosteroids , in the urine. The smal l amount ofdexamethasone present contributes minimal ly to this measurement. None of the other drugs l i s ted would be sui table for this test, and a l l areless potent than dexamethasone.

392. The answer is d. (Brunton, pp 1588, 1766-1767, 1841, 1817; Katzung, pp 709f, 710, 712, 737, 853f, 858.) Ketoconazole, long used as an anti fungal agent(see Chapter 10), i s one of the most efficacious inhibi tors of corticosteroid synthes is . It i s not a primary therapy for pi tui tary tumors/Cushingsyndrome, but i s a useful adjunct. The main problem associated with this drug i s the ri sk of hepatotoxici ty and the potentia l for interactions withsome other drugs . An a l ternative to ketoconazole i s aminoglutethimide, which a lso interferes with synthes is of a l l the adrenal s teroids .

Cimetidine, the H2 hi s tamine blocker that i s wel l known as an inhibi tor of P450-mediated metabol i sm of many drugs , would be of no benefi t.Mass ive doses of corti sol , a l though theoretica l ly “reasonable” (feedback suppress ion of pi tui tary function), would be of l i ttle help in terms ofregulating the pi tui tary’s activi ty and clearly would aggravate s igns and symptoms of adrenal corticosteroid excess that we a l ready have.Fludrocorti sone has intense minera locorticoid activi ty and i s the only drug indicated for replacement therapy in chronic minera locorticoiddeficiencies . Spi ronolactone blocks a ldosterone receptors ; i t would counteract only the a ldosterone-related responses to corticosteroid excessand would not have any effects on adrenal corticosteroid synthes is .

393. The answer is e. (Brunton, pp 845, 1012, 1524, 1529; Katzung, pp 407-409, 427, 1159t.) Phenytoin i s one of severa l agents that can enhance thehepatic metabol i sm of ora l contraceptives (especia l ly the estrogen component), leading to reduced contraceptive levels and unintendedpregnancy (contraceptive fa i lure). It i s a l so one that interacts by inducing synthes is of hormone-binding globul ins : more hormone molecules arebound to the protein, and so less free (active) drug i s in the ci rculation. Severa l other common anticonvulsants interact with the same potentia loutcome, especia l ly barbi turates (including phenobarbi ta l , mephobarbi ta l , and primidone), carbamazepine, and oxcarbazepine. Be sure to reca l lthat ri fampin (and ri fabutin) and protease inhibi tors (ri tonavi r, others ) are a lso important interactants with OCs via a metabol i sm-inducingmechanism. Fina l ly, some antibiotics (eg, tetracycl ines ) interact with OCs , but here the mechanism di ffers : the antibiotics suppress gut flora thatparticipate in enterohepatic recycl ing of the OCs . When the bacteria are suppressed, OCs that are secreted into the gut are lost in the feces , ratherthan being reabsorbed.

394. The answer is d. (Brunton, pp 1174-1175, 1182, 1833-1836; Katzung, pp 723-726, 730-731, 740.) Progestin-only ora l contraceptives (“minipi l l s”) areassociated with a higher ri sk of menstrua l i rregulari ties than the more common estrogen-progestin preparations . That i s largely due to the lack ofestrogen. However, the absence of estrogen a lso lowers the ri sk (not increases , c) of thromboembol ic disorders—a major advantage, especia l ly,for women who smoke. Progestin-only formulations are, overa l l , less effective in terms of preventing pregnancy than combination products (and soanswer a i s incorrect); l i ke combination products they lack defini tive spermicida l effects (b) but they thicken cervica l mucus , retard sperm moti l i tyin that way, and reduce the l ikel ihood of nidation, and their adminis tration schedule i s continuous—every day (not answer e), rather than thecycl ic schedule used for combination products .

395. The answer is b. (Brunton, pp 890t, 1174-1175; Katzung, pp 716-723, 740, 1157t.) Hypertens ion, which may occur (and usual ly trans iently) in responseto OCs , i s mainly due to estrogen excess . Fatigue (a), weight ga in (e; usual ly triggered by increased appeti te), and infrequent or missed menses (c)are mainly associated with progestin excess . Hypomenorrhea (c) may a lso be due to inadequate estrogen levels in the combination product.Unusual ly frequent menses—hypermenorrhea—tends to be related to too l i ttle progestin in the product.

396. The answer is c. (Brunton, pp 1258-1260; Katzung, pp 757, 766.) Metformin, a common drug for managing many patients with type 2 diabetesmel l i tus , i s a very wel l -tolerated and effective drug. This biguanide not only lowers ci rculating glucose levels , but suppresses appeti te, therebylowering dietary ca loric intake. One of the main problems associated with metformin i s the development of lactic acidos is—admittedly rare butpotentia l ly fata l adverse response to this drug, and associated with none of the other drugs l i s ted as answer choices (insul in glargine or otherinsul ins (a); thyroid hormones (b); sul fonylureas (d), older or newer; or spi ronolac-tone or the related drug eplerenone (e)).

397. The answer is d. (Brunton, pp 1260-1262; Katzung, pp 758t, 786.) Tolbutamide and the older sul fonylureas (e) have been associated with anincreased ri sk of sudden death. The gl i tazones , however, have been associated with an increased ri sk of heart fa i lure, which i s why patientstaking these drugs should be monitored for weight ga in, edema, and other expected s igns and symptoms of heart fa i lure. The heart fa i lure ri sk i sa lso a major reason why piogl i tazone i s the only gl i tazone to remain on the market: others , such as ros igl i tazone and trogl i tazone, have beenwithdrawn owing to cardiovascular and/or hepatotoxic concerns .

Acarbose (a), an inhibi tor of gastrointestina l carbohydrate uptake, i s relatively free of any serious toxici ties . Biguanides (b), of which metformin(d) i s an example, are widely used antidiabetic drugs . The major toxici ty or adverse response i s a ri sk of lactic acidos is .

398. The answer is b. (Brunton, pp 1046-1052; Katzung, p 703t.) We would give betamethasone to enhance feta l surfactant synthes is and suppressa i rway inflammation, thereby lessening the chance or severi ty of feta l respi ratory dis tress syndrome at bi rth. (The action probably involvesstimulating synthes is of fibroblast pneumocyte factor, which then s timulates surfactant synthes is by pneumocytes in the fetus . Reca l l thatsurfactant lowers a lveolar surface tens ion, thereby reducing the l ikel ihood of a lveolar col lapse and i ts consequences on gas exchange.)

Note: Other corticosteroids , especia l ly dexamethasone, could be used. However, betamethasone i s preferred because i t binds less to plasmaproteins than corti sol and most other glucocorticoids , a l lowing more s teroid to cross the placenta.

Even though a lbuterol i s used as a bronchodi lator (eg, for as thma or COPD), i t i s class i fied, pharmacologica l ly, precisely as we class i fy ri todrine,a relatively selective β2 agonis t. In obstetrics , ergonovine or methylergonovine i s given postpartum only; thei r s trong uterine contracting effects areused to reduce postpartum bleeding, and such intense and prolonged uterine effects would be disastrous for the unborn chi ld because placenta l

Page 219: Pharmacology pre test self assessment and review (14th ed.)

blood flow would be compromised. As ide from being contra-indicated before del ivery, they have no pulmonary effects of the sort we want.Indomethacin (d) i s sometimes used to s low premature labor. It and another nonsteroida l anti -inflammatory drug, ibuprofen, work by blockingsynthes is of prostaglandins that have oxytocic activi ty, but prostaglandin synthes is inhibi tion a lso causes closure of the ductus arteriosus (which,in this setting, would be unwanted). Magnes ium sul fate would not cause the des i red pulmonary effects ; i t i s used in obstetrics to preventseizures in preeclamps ia/eclamps ia . The drug has uterine relaxant activi ty too, but the jury i s out on just how effective and safe the drug i s whenused to suppress premature uterine contractions .

Note: Three pharmaceutica l preparations of surfactant are now ava i lable: ca l factant, beractant, and poractant a l fa . They are given to neonateswith respiratory dis tress syndrome by di rect intratracheal insti l lation.

399. The answer is b. (Brunton, p 1296; Katzung, p 776.) Esophagi ti s , sometimes with esophageal ulcers , i s the most worrisome adverse response tobisphosphonates such as etidronate. It i s not due to any speci fic metabol ic a l teration causes by the drug, but rather by di rect, prolonged contact ofthe drug with the esophagus i f the ora l dose lodges there without pass ing quickly enough to the s tomach. This i s why the drug should not beadminis tered to patients with esophageal disease, di ffi cul ty swal lowing, or an inabi l i ty to s i t or s tand up for at least 30 minutes (to help gravi tybring the tablet to the s tomach) after taking the drug. There i s a lso some concern that long-term therapy with bisphosphonates may causeosteonecros is of the jaw. Nonetheless , overa l l the bisphosphonates cause remarkably few s ide effects in the vast majori ty of patients .

400. The answer is c. (Brunton, pp 1122, 1668-1670, 1765; Katzung, p 678.) Leuprol ide i s a peptide that i s related to GnRH or luteinizing hormone-releas ing hormone (LHRH), and i t i s used to treat metastatic prostate carcinoma. By inhibi ting gonadotropin release i t induces a hypogonadals tate; testosterone levels in the body fa l l s igni ficantly, and this appears to be the mechanism for suppress ion of the cancer. Aminoglutethimide(a) i s an aromatase inhibi tor mainly used to treat Cushing disease (i t inhibi ts synthes is of adrenal corticosteroids ), and some patients with meta-s tatic breast carcinoma. Fludrocorticone (b) i s a minera locorticoid used for chronic adrenal insufficiency (a long with glucocorticoids ) or congenita ladrenal hypoplas ia . Mifepris tone (d) i s an aborti facient/oxytocic drug. Spi ronolactone (e) i s an a ldosterone receptor blocker that i s used mainlyas for patients with primary or secondary hypera ldosteronism, and as an adjunct to the management of severe heart fa i lure. It i s a l so class i fied asa potass ium-sparing diuretic.

401. The answer is e. (Brunton, pp 1256-1258; Katzung, pp 753-762, 755t.) Glyburide i s one of severa l sul fonylurea antidiabetic drugs . Reca l l that thesul fonylureas s impl is tica l ly fa l l into two main classes , based largely on how long they have been used: the so-ca l led fi rs t-generation agents , suchas tolbutamide and chlorpropamide, and the newer second-generation drugs , gl ipizide, glyburide, and gl imepiride. Whether older or newer, thesedrugs mainly lower blood glucose levels by enhancing insul in release from the β cel l s of the pancreas . The mechanism involved binding to andblocking an ATP-sens i tive K+ channel on β-cel l membranes . This depolarizes the membrane, and resul ting Ca 2+ influx triggers insul in release. Themechanis tic dependence on insul in release, of course, expla ins why these drugs are ineffective in type 1 diabetes .

Other probable actions of the sul fonylureas include reduced plasma glucagon levels and increased binding of insul in to parenchymal ti s suecel l s . These drugs have no di rect effects on renal handl ing of glucose (b) or insul in synthes is (c). Unl ike some other ora l antidiabetic drugs , suchas the gl i tazones (thiazol idinediones), they do not increase parenchymal cel l respons iveness to insul in (d) or do so in any other ways that mightbe described as “insul in sens i ti zing.” They do not decrease “insul in res is tance” (a), which i s a common problem for many patients with type 2diabetes mel l i tus , whether by lowering body weight or appeti te or by other mechanisms.

402. The answer is c. (Brunton, pp 75t, 1257-1258; Katzung, pp 753-755.) Of the main groups or chemica l classes of ora l antidiabetic agents ,sul fonylureas are the ones typica l ly associated with caus ing hypoglycemia, whether from overdose or as a rather expected response in somepatients , particularly before meals . As noted in the previous question, sul fonylureas release insul in from pancreas even i f blood glucose levelsa l ready are low. Even in a fast-induced hypoglycemic s tate a sul fonylurea wi l l release insul in and drive blood glucose down even further, with al ikely outcome being hypoglycemia.

The elderly are particularly susceptible to sul fonylurea-induced hypoglycemia. Part of this may relate to diet. However, expected age-relatedfa l l s of renal and/or hepatic function can reduce el imination of these drugs , thereby increas ing their plasma levels and their effects unlessdosages are reduced accordingly.

This propens i ty for preprandia l hypoglycemia i s shared by the megl i tinides—repagl inide and nategl inide—because they, too, increasepancreatic insul in release. However, they are not prescribed nearly as often as a sul fonylurea (or metformin) when ora l therapy i s indicated.Indeed, repagl inide and nategl inide are seldom prescribed for diabetes anymore.

Piogl i tazone (e), the only remaining “gl i tazone” on the market, seldom causes symptomatic hypoglycemia because i t does not release insul in.Rather, i t i s an “insul in sens i ti zer,” acting on targets such as skeleta l muscle cel l s and adipocytes . The low ri sk of hypoglycemia a lso appl ies tometformin (d), the biguanide; and to acarbose (a ; an α-glucos idase inhibi tor, as i s migl i tol ). Colesevelam (b), a l so an ora l agent, i s class i fied as abi le acid sequestrant. It inhibi ts enterohepatic reuptake of cholesterol from the gut. Its origina l approved use was as a cholesterol -loweringadjunct for certa in patients with hyperl ipidemias . It i s a l so approved now adjunctive treatment for certa in patients with type 2 diabetes mel l i tus .Its mechanism of glucose-lowering effects i s not known for certa in.

403. The answer is a. (Brunton, p 1265; Katzung, pp 759-760.) Acarbose, class i fied as an α-glucos idase inhibi tor, s lows the rate and extent of complexcarbohydrate absorption from the intestines . (The related drug, migl i tol , does the same. It i s seldom prescribed.) The consequence of this for thehypoglycemic patient i s that attempts to restore blood glucose levels by consuming sugar or sucrose-conta ining foods or beverages (eg, orangejuice i s a popular choice, as are products such as the one our patient took) wi l l be less effective, i f effective at a l l—and certa inly s lower in termsof symptom rel ief. None of the other ora l diabetes drugs have this mechanism of action and so wi l l not hinder attempts to restore euglycemiawhen complex carbohydrates are consumed. If the patient who i s taking acarbose becomes hypoglycemic, but not to the extent that i t becomessevere or serious , ora l glucose must be used. Severa l glucose-conta ining products are on the market and are made for such patients .

404. The answer is a. (Brunton, pp 1141-1142, 1146-1151; Katzung, pp 688-689, 695.) The incidence of agranulocytos is due to propyl thiouraci l , or therelated thio[n]amide, methimazole, i s not rare. It may develop within the fi rs t few weeks of s tarting therapy, and so a “routine” blood testscheduled for severa l months or more after s tarting the drug may not catch i t. However, i f i t i s detected early (eg, the patient promptly reports asore throat or other flu-l ike symptoms) and the drug i s s topped, i t i s usual ly spontaneous ly revers ible. The most common s ide effect with thethioamides i s urticaria . It may abate spontaneous ly or require symptomatic management with an antihis tamine (ei ther fi rs t or second generation)or with a corticosteroid.

None of the other responses l i s ted are recognized as being associated with thioamides .

405. The answer is e. (Brunton, pp 1259-1261; Katzung, pp 757-759, 766.) Piogl i tazone, l ike a l l the gl i tazone ora l antidiabetic drugs , works by increas ingparenchymal cel l respons iveness to insul in. The mechanism seems to involve activation of peroxisome prol i ferator-activated receptors (PPARs).

Page 220: Pharmacology pre test self assessment and review (14th ed.)

This apparently increases transcription of insul in—respons ive genes that control glucose (and l ipid) metabol i sm and cel lular glucose uptake viaglucose transporters . The net effects include not only lowered plasma levels of glucose, but a lso fatty acids and (indi rectly) of insul in. Given thenecessary involvement of insul in in the drug’s effects , i t wi l l not work (a lone) for patients with type 1 diabetes . When used for patients who havetype 2 diabetes , and who nonetheless require insul in, a thiazol idinedione can be a va luable adjunct that helps lower da i ly insul in requirements .

406. The answer is d. (Brunton, pp 1258-1260; Katzung, pp 757, 761-762.) Metformin (a biguanide) poses the greatest ri sk, of a l l the antidiabetic drugs , ofcaus ing lactic acidos is . Al though the overa l l ri sk of lactic acidos is from this drug i s low, should i t occur, morta l i ty i s qui te common. Alcoholconsumption or renal or hepatic disease increases the ri sk of lactic acidos is associated with metformin.

Acarbose (a), an α-glucos idase inhibi tor (blocks s tarch uptake from the gut), the sul fonylureas (older ones such as tolbutamide or newer onessuch as gl ipizide and glyburide; b and c), and ros igl i tazone (e; and other gl i tazones , eg, piogl i tazone) are not associated with lactic acidos is(unless they are coadminis tered with metformin).

407. The answer is c. (Brunton, pp 211-212, 320, 1150; Katzung, pp 691-694.) Cardiovascular and related hemodynamic changes , mostly ari s ing from orrelated to tachycardia , increased ventricular contracti l i ty, and potentia l ly leading to acute myocardia l i schemia, are among the ha l lmarks ofthyroid hormone excess , whether drug-induced or idiopathic (eg, thyrotoxicos is of other causes). Increased thyroid hormone levels “up-regulate”the β-adrenergic receptors , leading to heightened and potentia l ly letha l responses that involve overactivation of those receptors . Beta-blockadeprovides important and prompt symptom rel ief, and may be l i fesaving in s i tuations such as this . Other interventions a imed at lowering thyroidhormone synthes is or release (iodine/iodides , a ; a thioamide such as propyl thiouraci l , d) ul timately may be important, but saving the patient’sl i fe with adminis tration of propranolol or a sui table a l ternative i s the most important fi rs t s tep.

Liothyronine (b) would be a whol ly i rrational choice for acute thyroid hormone overdoses . Radioiodine (e) i s a l so i rrational for acute andinfrequent episodes of hyperthyroidism such as those that occur with thyroid hormone overdoses . It i s , of course, one option for managinghyperthyroidism due to a thyroid gland cancer in some patients .

408. The answer is d. (Brunton, pp 1185-1186; Katzung, pp 725-731.) There are two main mechanisms by which OCs exert thei r main effects . Arguably themost important i s inhibi tion of ovulation. This prevents (or reduces the ri sk of) ferti l i zation, a process that must come before such others asimpairing nidation (c). The other i s thickening of cervica l mucus . That important effect, mainly caused by the progestin component, provides aphys ica l barrier that s lows or s tops sperm moti l i ty. There are changes of cervica l mucus pH (a), but that does not cause a spermicida l effect per se,nor i s i t the major mechanism by which the contraceptive acts . Detaching a ferti l i zed ovum (b) or reducing uterine or endometria l blood flow (e)are not elements of the mechanisms of action.

409. The answer is e. (Brunton, p 1294; Katzung, pp 772-773, 780.) Over-medication with vi tamin D may lead to a toxic syndrome ca l led hypervi taminos isD. The ini tia l symptoms can include weakness , nausea, weight loss , anemia, and mi ld acidos is . As the excess ive doses are continued, s igns ofnephrotoxici ty can develop, such as polyuria , polydips ia , azotemia, and eventual ly nephroca lcinos is . In adults , osteoporos is can occur. Also, thereis CNS impairment, which can resul t in menta l retardation and seizures .

The s igns and symptoms described in the question are not associated with estrogens (a) or progestins ; etidronate (b) or otherbisphosphonates ; gl ipizide (c) or any other sul fonylureas ; or prednisone (d) or other glucocorticoids .

410. The answer is c. (Brunton, pp 1766-1767; Katzung, pp 709-710, 712, 737, 854.) Ketoconazole i s typica l ly remembered as an azole anti fungal drug, anda common cause of drug interactions due to i ts powerful cyto-chrome P450 inhibi tor effects . At doses higher than those used when the drug i sprescribed as an anti fungal , i t inhibi ts corticosteroid synthes is and i s one of the preeminent drugs for nonoperable Cushing disease. Al ternativesare metyrapone and aminoglutethimide, not l i s ted here. Dexametha-sone (a) i s a synthetic corticosteroid with powerful glucocorticoid effects andl i ttle or no minera locorticoid activi ty. Hydrocorti sone (b) i s corti sol ; an efficacious glucocorticoid with useful (but s l ight) minera locorticoid activi ty.The same appl ies to prednisone (d), predominately a glucocorticoid used as an ora l anti -inflammatory drug for asthma, some arthri tides , andother s teroid-respons ive conditions . Spi ronolactone (e) i s an a ldosterone receptor antagonis t, used mainly for primary or secondaryhypera ldosteronism, for heart fa i lure, and for i ts potass ium-sparing diuretic effects .

411. The answer is c. (Brunton, p 1114; Katzung, pp 290, 672-673.) Primary amenorrhea i s associated with hyperprolactinemia. Women who areamenorrheic from hyperprolactinemia a lso often present with ga lactorrhea and inferti l i ty. Bromocriptine, a dopamine receptor agonis t, inhibi tsprolactin release. The s i te of action i s the pi tui tary, and i t involves the same s i te and ul timate mechanism by which dopamine, released from thehypothalamus, normal ly suppresses prolactin release. Some causes of hyperprolactinemia, in both men and women, include pi tui tary cancer,hypothalamic dys function, and some drugs (eg, antipsychotics , es trogens). Reca l l that bromocriptine, by vi rtue of i ts centra l dopaminergic activi ty,i s sometimes used to help correct the dopamine-ACh imbalance that underl ies Parkinson disease.

Effects on estrogens or progestin (a , d), gonadotropins (a), FSH (b), or GnRH (e) are not respons ible for the therapeutic actions of bromocriptinein the context of managing primary amenorrhea.

412. The answer is e. (Brunton, pp 1278-1280; Katzung, pp 769-771.) Parathyroid hormone’s main role i s to mainta in normal plasma Ca 2+ levels . Whenplasma Ca 2+ concentrations become sufficiently low, parathyroid hormone synthes is and release increase, and i t helps restore a normocalcemicstate by promoting (in concert with vi tamin D) Ca 2+ absorption from the intestines , promotes renal tubular Ca 2+ reabsorption and phosphateexcretion, and increases mobi l i zation of Ca 2+ and phosphate from bone (ie, bone resorption).

Page 221: Pharmacology pre test self assessment and review (14th ed.)

Antimicrobial and Antiviral Pharmacology Antibacteria lsAntimycobacteria lsAnti fungalsAntivi ra ls/Anti retrovi ra lsAntiprotozoals

Questions

413. You are a phys ician and epidemiologis t employed by the US Centers for Disease Control and Prevention. You get an urgent phone ca l l s tatingthat an envelope was opened in the mai l room of a large corporation, and s imi lar ones were del ivered to various government offices . Some sort ofwhite powder fel l out and was blown around the room, expos ing dozens of workers . The suspicion i s anthrax. Which drug would be best, mostproperly indicated, for prophylaxis i f the substance tests pos i tive for Bacillus anthracis?

a. Azi thromycinb. Clari thromycinc. Doxycycl ined. Hydroxychloroquinee. Metronidazole

414. A jaundiced 1-day-old premature infant with an elevated free bi l i -rubin i s seen in the premature baby nursery. The mother had received anantibiotic combination for a urinary tract infection (UTI) 1 week before del ivery. Which antibiotic drug or class was the most l ikely cause of thebaby’s kernicterus?

a. An aminopenici l l in (eg, amoxici l l in)b. Azi thromycinc. Erythromycind. Fourth-generation cephalosporine. Sul famethoxazole plus trimethoprim

415. You are caring for a patient with HIV infection and wi l l s tart drug therapy ini tia l ly with the nucleos ide analog zidovudine (formerly ca l ledazidothymidine, or AZT). Which s tatement most correctly describes zidovudine or the dideoxynucleos ide class to which i t belongs?

a. Levels of active metabol i te in cerebrospina l fluid usual ly are not detectable (ie, zero) because i t cannot cross the blood–bra in barrierb. Res is tance to these antivi ra ls develops rapidly after monotherapy s tarts , involves decreased incorporation of active metabol i te into vi ra l DNAc. The dideoxynucleos ide s tops vi ra l nucleic acid synthes isd. The active metabol i te i s an equal ly effective substrate for vi ra l reverse transcriptase and mammal ian DNA polymerase, which expla ins the high

incidence of host toxici tye. Zidovudine i s dephosphorylated to form the active metabol i te, which i s incorporated into vi ra l nucleic acid via HIV reverse transcriptase

416. A patient on antimicrobia l therapy develops the fol lowing s igns and symptoms that ul timately are found to be drug-induced: cough, dyspnea,and pulmonary infi l trates ; neutropenia and bleeding tendencies ; and paresthes ias . What was the most l ikely cause of this patient’s symptoms?

a. Amoxici l l inb. Azi thromycinc. Ciprofloxacind. Isoniazide. Ni trofurantoin

417. A patient presents with severe, unrelenting diarrhea, and the fluid and electrolyte imbalances one would expect from that. She had beenstarted on an antibiotic recently, and the l ikely diagnos is i s antibiotic-associated pseudomembranous col i ti s (AAPMC). What drug was the mostl ikely cause?

a. Amoxici l l inb. Azi thromycinc. Cl indamycind. Metronidazolee. Trimethoprim plus sul famethoxazole (TMP-SMZ)

418. A 35-year-old woman compla ins of i tching in the vulva l area. Hanging-drop examination of the urine reveals trichomonads . What i s thepreferred treatment for the trichomonias is?

a. Doxycycl ineb. Emetinec. Metronidazoled. Pentamidinee. Pyrazinamide

419. A patient wi l l be s tarted on primaquine to treat active Plasmodium vivax malaria , speci fica l ly to target the hepatic forms of the paras i te. Beforeyou adminis ter the drug you should screen the patient to assess thei r relative ri sk of developing a relatively common and severe adverse

Page 222: Pharmacology pre test self assessment and review (14th ed.)

response to the drug. What i s that primaquine-associated ri sk?

a. Cardiac conduction dis turbancesb. Hemolytic diseasec. Nephrotoxici tyd. Retinopathye. Seizures , convuls ions

420. On morning rounds in the hospi ta l you encounter a patient being treated with l inezol id, the fi rs t approved member of the oxazol idine class ofantimicrobia ls . Which s tatement accurately describes a characteris tic of this drug?

a. Exerts s trong bactericida l effectsb. Mainly used for relatively minor infections with gram-negative organismsc. Preferred a l ternative to amoxici l l in for chi ldren with oti ti s mediad. Preferred a l ternative to ciprofloxacin for B. anthracis infectionse. Sui table for vancomycin-res is tant enterococci

421. A 59-year-old woman is diagnosed with tuberculos is (TB). Before prescribing a multidrug regimen, you take a careful medication his torybecause one of the drugs commonly used to treat TB induces some of the microsomal cytochrome P450 enzymes in the l iver, and i s a commoncause of drug–drug interactions . What i s the most l ikely drug?

a. Ethambutolb. Isoniazidc. Pyrazinamided. Ri fampine. Vi tamin B6

422. A patient with HIV infection i s receiving a combination of protease inhibi tors as part of overa l l antivi ra l therapy. What i s , ordinari ly, the mostl ikely/most common s ide effect(s ) of the protease inhibi tors?

a. Anemia and neutropeniab. Hyperglycemia and hyperl ipidemiac. Lactic acidos isd. Neuropathye. Pancreati ti s

423. A 27-year-old woman has just returned from a trip to Southeast As ia . Over the past 24 hours she has developed shaking, chi l l s , and atemperature of 104°F. A blood smear reveals Plasmodium vivax. What drug would you prescribe to eradicate the extraerythrocytic phase of theorganism?

a. Chloroguanideb. Chloroquinec. Primaquined. Pyrimethaminee. Quinacrine

424. A patient has a severe bacteria l infection that normal ly would respond to an ora l penici l l in or cephalosporin. However, his chart documentsanaphylactoid reactions to both classes of drugs . Given the his tory, what drug would be preferred for treating the infection, and a lso poses theleast ri sk of cross -reactivi ty and an a l lergic response?

a. Clotrimazoleb. Gentamicinc. Metronidazoled. Tetracycl inee. Vancomycin

425. A 40-year-old man i s HIV-pos i tive and has a cluster-of-di fferentiation-4 (CD4) count of 200/mm3. Within 2 months after s tarting drug therapy forhis HIV he develops a periphera l white blood cel l count of 1000/mm3 and a hemoglobin of 9.0 mg/dL. Which drug most l ikely caused thehematologic abnormal i ties?

a. Acyclovi rb. Dideoxycytidinec. Foscarnetd. Rimantadinee. Zidovudine

426. An 86-year-old man compla ins of cough and blood in his sputum for the past 5 days . On admiss ion, his temperature i s 103°F. Phys ica lexamination reveals ra les in his right lung, and x-ray examination shows increased dens i ty in the right middle lobe. A sputum smear shows manygrampos i tive cocci , confi rmed by sputum cul ture as penici l l inase-producing Staphylococcus aureus. Which antibiotic would be best to adminis terfi rs t?

a . Ampici l l inb. Mezloci l l in

Page 223: Pharmacology pre test self assessment and review (14th ed.)

c. Oxaci l l ind. Penici l l in (G or V)e. Ticarci l l in

427. When cons idering a l l the main antibacteria l drugs that work by inhibi ting protein synthes is in one way or another, vi rtua l ly every one exertsbacteriostatic actions . Which drug or drug class di ffers from the rest because the usual consequence of therapeutic plasma levels i s bactericida l ,rather than mere inhibi tion of bacteria l growth and repl ication?

a. Aminoglycos idesb. Cl indamycinc. Erythromycinsd. Linezol ide. Tetracycl ines

428. A patient with HIV infection and cl inica l AIDS i s treated with a combination of agents , one of which i s zidovudine. Which enzyme or repl icativeprocess i s the main target of this antivi ra l drug?

a. Nonnucleos ide reverse transcriptaseb. Nucleos ide reverse transcriptasec. RNA synthes isd. Vi ra l particle assemblye. Vi ra l proteases

429. A patient with HIV/AIDS, being treated with multiple antivi ra l and immunosuppress ive drugs , develops an opportunis tic infection caused by P.jiroveci. Which drug are you most l ikely to use to treat the pulmonary infection caused by this protozoan?

a. Carbenici l l inb. Metronidazolec. Ni furtimoxd. Penici l l in Ge. Pentamidine

430. A 25-year-old woman with an upper respiratory tract infection caused by H. influenzae i s treated with trimethoprim-sul famethoxazole. Sheresponds wel l in a matter of days after s tarting the TMP-SMZ. Which bacteria l process i s inhibi ted by this combination, and accounts for theantibacteria l effects?

a. Cel l -wal l synthes isb. Protein synthes isc. Fol ic acid synthes isd. Topoisomerase II (DNA gyrase)e. DNA polymerase

431. A man who has been at the loca l tavern, drinking a lcohol heavi ly, i s assaul ted. He i s transported to the hospi ta l . Among various findings i s aninfection for which prompt antibiotic therapy i s indicated. Given his high blood a lcohol level , which antibiotic should be avoided because of ahigh potentia l of caus ing a serious disul fi ram-l ike reaction that might provoke venti latory or cardiovascular fa i lure? (Assume that were i t not forthe a lcohol consumption, the antibiotic would be sui table for the infectious organisms that have been detected.)

a . Amoxici l l inb. Cefotetanc. Erythromycin ethylsuccinated. Linezol ide. Penici l l in G

432. A 43-year-old woman is recovering from major surgery, fol lowing discharge from the hospi ta l , in an ass is ted-care faci l i ty. She develops fever,ra les , dyspnea, cough, and purulent sputum. Resul ts of a chest radiograph indicate bi latera l pulmonary infi l trates . We send blood and sputumcultures to the cl inica l pathology lab for cul turing, but now must turn our attention to what we bel ieve i s community-acquired pneumonia causedby antibiotic-res is tant pneumococci . We want to s tart empiric antibiotic therapy unti l cul ture resul ts are ava i lable. Which drug would be best forthis ini tia l therapy?

a. Amoxici l l inb. Cefazol inc. Erythromycind. Levofloxacine. Penici l l in Gf. Vancomycin

433. Blood and sputum cul tures taken in a cri ti ca l ly i l l 26-year-old woman indicate the presence of MRSA—methici l l in-res is tant Staph. aureus.Which drug i s most l ikely to be effective in treating this infection?

a. Amoxici l l in plus clavulanic acidb. Cl indamycinc. Erythromycin

Page 224: Pharmacology pre test self assessment and review (14th ed.)

d. Trimethoprim-sul famethoxazole (TMP-SMZ)e. Vancomycin

434. Compared with most other cephalosporins , the adminis tration of cefmetazole, cefoperazone, or cefotetan i s associated with a higherincidence of an adverse response that i s particularly dangerous for some patients . What i s that rather unique adverse response?

a. Acute heart fa i lureb. Acute renal fa i lurec. Bleeding tendencies in patients taking warfarind. Hypertens ione. Ototoxici ty

435. A patient develops muscle aches and pa ins during the course of antibiotic therapy. A muscle biopsy would clearly show myopathy. Which drugmost l ikely (and rather uniquely) caused this adverse effect on skeleta l muscle?

a. Daptomycinb. Erythromycinc. Gentamicind. Linezol ide. Vancomycin

436. A patient with an infectious disease routinely takes thei r antimicrobia l medication with mi lk or other da i ry products in an attempt to reducestomach upset from the drug. The antibiotic fa i l s to work adequately because ca lcium in the da i ry products chelates the drug and reduces i ts ora lbioava i labi l i ty. Which antimicrobia l drug or drug class was the patient most l ikely taking?

a. Aminoglycos ideb. Antimycobacteria l drug, speci fica l ly i soniazidc. Cephalosporin, fi rs t generationd. Cephalosporin, thi rd generatione. Penici l l inf. Tetracycl ine

437. A patient develops profuse, watery diarrhea, fever, abdominal pa in, and leukocytos is in response to antibiotic therapy. C. difficile infection inthe gut i s confi rmed. What i s the preferred agent for therapy of this antibiotic-associated pseudomembranous col i ti s (AAPMC)?

a. Amphotericin Bb. Cl indamycinc. Gentamicind. Metronidazolee. Trimethoprim plus sul famethoxazole (TMP-SMZ)

438. A patient has been taking warfarin for severa l months and his INR (normal ized prothrombin time) has been kept within the des i redtherapeutic range cons is tently. They develop an infection and are s tarted on antibiotic therapy. Shortly thereafter thei r INR ri ses to eight (veryhigh) and they develop epis taxis and other indicators of excess ive bleeding. Which antibiotic most l ikely interacted with the warfarin, increas ingi ts blood levels and effects , by inhibi ting warfarin’s metabol i sm by the hepatic P-450 system?

a. Azi thromycinb. Erythromycinc. Gentamicind. Penici l l in Ge. Ri fampin

439. Ampici l l in and amoxici l l in are in the same group of penici l l ins (broad spectrum, or aminopenici l l ins ). However, there i s one cl inica l lyimportant di fference. Which phrase best s tates how amoxici l l in differs from ampici l l in?

a. Has better ora l bioava i labi l i ty, particularly when taken with mealsb. Is effective aga inst penici l l inase-producing organismsc. Is a broad-spectrum penici l l ind. Does not cause hypersens i tivi ty reactionse. Has great antipseudomonal activi ty

440. A patient’s his tory notes a documented severe (anaphylactoid) reaction to a penici l l in. What other antibiotic or class i s l ikely to cross -reactand so should be avoided in this patient?

a. Aminoglycos idesb. Azi thromycinc. Cephalosporinsd. Erythromycine. Linezol idf. Tetracycl ines

441. A 30-year-old woman develops a severe P. aeruginosa infection. The phys ician chooses to treat i t wi th amikacin, not with gentamicin. Which

Page 225: Pharmacology pre test self assessment and review (14th ed.)

phrase best describes how amikacin di ffers from gentamicin?

a. Does not require monitoring of blood levels during therapyb. Exerts s igni ficant bactericida l effects aga inst anaerobes tooc. Has broader spectrum against gram-negative baci l l id. Lacks ototoxic potentia le. Protects aga inst typica l aminoglycos ide nephrotoxici ty

442. A 19-year-old being treated for leukemia develops a fever. You give severa l agents that wi l l cover bacteria l , vi ra l , and fungal infections . Twodays later, he develops acute renal fa i lure. Which drug was most l ikely respons ible?

a. Acyclovi rb. Amphotericin Bc. Ceftazidimed. Penici l l in Ge. Vancomycin

443. Penici l l ins , cephalosporins , and amphotericin B are qui te di fferent s tructura l ly, and the antimicrobia l spectrum of amphotericin B i s decidedlydi fferent from those of the other agents . Nonetheless , they a l l share a common property or action. Which s tatement identi fies what that i s?

a . Act, though various mechanisms, on cel l wal l s or membranes of susceptible organismsb. Contra indicated in immunocompromised patientsc. Interact with many drugs by inducing their hepatic metabol i smd. Leukopenia (decreased white cel l counts ) i s a common s ide effecte. Nephrotoxici ty i s common

444. Given the periodic worldwide outbreaks of “swine flu” (particularly influenza A vi rus subtype H1N1) there has been a great need forprophylactic measures in certa in at-ri sk populations who may be susceptible. Which drug i s genera l ly recommended by the US Centers for DiseaseControl and Prevention for this purpose?

a. Acyclovi rb. Amantadinec. Lopinavi rd. Osel tamivi re. Ri tonavir

445. A patient with a P. aeruginosa infection i s receiving intravenous gentamicin. The aminoglycos ide blood levels are wel l above the minimuminhibi tory concentration (MIC), but the cl inica l response i s not sati s factory. A new medication order ca l l s for adding a penici l l in, adminis tered in aseparate IV l ine to avoid a phys ica l incompatibi l i ty. If this order i s carried out, what i s most l ikely to occur?

a. The aminoglycos ide wi l l inactivate the penici l l inb. The aminoglycos ide wi l l chemica l ly neutra l i ze and abol i sh the effects of the penici l l inc. The patient i s l ikely to develop Clostridium difficile col i ti s (superinfection)d. The penici l l in wi l l act synergis tica l ly with the aminoglycos idee. The penici l l in wi l l increase the ri sk of aminoglycos ide nephrotoxici tyf. The ri sk of inducing res is tance to both drugs increases dramatica l ly

446. Narrow spectrum penici l l ins , both penici l l inase-sens i tive and -res is tant, have relatively poor activi ty aga inst gram-negative bacteria . What i sthe main property or characteris tic that expla ins why these micro-organisms do not respond wel l to the penici l l ins?

a. Actively transport any absorbed penici l l in back to the extracel lular spaceb. Have an outer membrane that serves as a phys ica l barrier to the penici l l insc. Lack a surface enzyme necessary to metabol ica l ly activate the penici l l insd. Lack penici l l in-binding proteinse. Metabol ica l ly inactivate these penici l l ins by mechanisms not involving β-lactamase

447. We have a patient with an intraabdominal infection, and Bacteroides fragilis i s the main organism found upon cul ture. Which cephalosporinhas the greatest activi ty aga inst anaerobes such B. fragilis?

a. Cefaclorb. Cefoxi tinc. Cefuroximed. Cephalexine. Cephalothin

448. A patient with a serious infection received intens ive antibiotic therapy that leads to hearing loss . Which antibiotic most l ikely caused theototoxici ty?

a. Aminoglycos ide (eg, gentamicin)b. Cephalosporin, fi rs t-generationc. Cephalosporin, thi rd-generationd. Fluoroquinolone (eg, ciprofloxacin)

Page 226: Pharmacology pre test self assessment and review (14th ed.)

e. Penici l l in

449. A 26-year-old woman with acquired immunodeficiency syndrome (AIDS) develops cryptococca l meningi ti s . She refuses intravenous medication.Which anti fungal agent i s the best choice for ora l therapy of the meningi ti s?

a . Amphotericin Bb. Fluconazolec. Ketoconazoled. Metronidazolee. Nystatin

450. An adult patient i s being treated with a parentera l aminoglycos ide for a serious Pseudomonas aeruginosa infection. He requires immediatesurgery. He i s premedicated with midazolam, fol lowed by adminis tration of pro-pofol for induction. A dose of succinylchol ine i s often given forintubation (due to i ts rapid onset), wi th skeleta l muscle para lys is mainta ined during surgery with vecuronium or another neuromuscular blockerin the same class (nondepolarizing, or competi tive nicotinic receptor blocker). Other components of ba lanced anesthes ia include ni trous oxide,i soflurane, and oxygen. What i s the most l ikely outcome of having the aminoglycos ide “on board” in the perioperative setting a long with a l l theseother drugs?

a. Acute hepatotoxici ty from an aminoglycos ide-isoflurane interactionb. Antagonism of midazolam’s amnestic and sedative effectsc. Enhanced aminoglycos ide toxici ty to host cel l sd. Increased or prolonged response to neuromuscular blockerse. Reduced ri sk of catecholamine-induced cardiac arrhythmias

451. A patient with tuberculos is i s s tarted on i soniazid (INH) as part of a multidrug regimen. The phys ician a lso s tarts therapy with vi tamin B6 a tthe same time. What i s the main reason for giving the vi tamin B6 prophylactica l ly?

a. Faci l i tates INH renal excretion, thereby protecting aga inst nephrotoxici tyb. Inhibi ts metabol i sm of INH, thereby increas ing INH blood levelsc. Is a cofactor required for activation of the INH to i ts antimycobacteria l metabol i ted. Potentiates the anti tubercular activi ty of the INHe. Prevents some adverse effects of INH therapy

452. One antibiotic i s cons idered very effective in treatment of Rickettsia, Mycoplasma, and Chlamydia infections? It i s a l so used to mange somepatients with acne vulgaris les ions . To which drug does this description apply?

a. Baci tracinb. Gentamicinc. Penici l l in Gd. Tetracycl inee. Vancomycin

453. You are s tarting therapy for an establ i shed HIV infection in a 28-year-old man. The drugs are ri tonavi r, lopinavi r, zidovudine, and didanos ine.This involves , of course, us ing two protease inhibi tors and two nucleos ide reverse transcriptase inhibi tors (NRTIs ). What i s the main purpose ofus ing the ri tonavi r?

a . Helps mainta in adequate saquinavi r levels by inhibi ting i ts metabol i smb. Induces the metabol ic activation of the NRTIs , which are prodrugsc. Prevents the l ikely development of hypoglycemiad. Reduces , or hopeful ly el iminates , lopinavi r-mediated host toxici tye. Serves as the main, most active, inhibi tor of vi ra l protease in this combination

454. As part of a multidrug attack on a patient’s infection with Myco bacterium tuberculosis, a phys ician plans to use an aminoglycos ide antibiotic.Which drug i s most active aga inst the tubercle baci l lus and seems to be associated with the fewest problems with res is tance or typica laminoglycos ide-induced adverse effects?

a. Amikacinb. Kanamycinc. Neomycind. Streptomycine. Tobramycin

455. Such agents as clavulanic acid, sulbactam, or tazobactam are often added to some proprietary (manufactured) penici l l in combinationproducts . What i s the main reason for including them, or describes thei r action best?

a. Add antibiotic activi ty aga inst Pseudomonas and many Enterobacter speciesb. Faci l i tate antibiotic penetration into the centra l nervous system and cerebro-spina l fluidc. Inhibi t cel l wal l transpeptidasesd. Inhibi t inactivation of penici l l in by β-lactamase-producing bacteriae. Inhibi t the normal ly s igni ficant hepatic metabol i sm of the penici l l inf. Reduce the ri sk and/or severi ty of a l lergic reactions in susceptible patients

Page 227: Pharmacology pre test self assessment and review (14th ed.)

456. A patient with active tuberculos is i s being treated with i soniazid (INH) and ethambutol as part of the overa l l regimen. What i s the main effectexpected of the ethambutol?

a. Faci l i tated entry of the INH into the mycobacteriab. Faci l i tated penetration of the blood-bra in barrierc. Retarded absorption after intramuscular injectiond. Retarded development of organism res is tancee. Slowed renal excretion of INH to help mainta in effective blood levels

457. A patient has a severe infection caused by anaerobic bacteria . The fi rs t-year house officer wri tes an order for gentamicin. This approach i sdoomed to fa i l because aminoglycos ides have no activi ty aga inst anaerobes . Which best expla ins why anaerobes wi l l be res is tant?

a. Cannot metabol i ze the aminoglycos ides , which are a l l prodrugs , to thei r bactericida l free radica l formsb. Cannot oxidatively metabol i ze aminoglycos ides to moieties that are nontoxic to host cel l sc. Lack molecular oxygen that i s a prerequis i te for drug binding to the 50S subunit of bacteria l ribosomesd. Lack the abi l i ty to transport aminoglycos ides from the extracel lular mi l ieu in the absence of oxygene. Synthes ize more, and more active, res is tance factors than do aerobic bacteria

458. In patients with hepatic coma or porta l -systemic encephalopathy decreas ing the production and absorption of ammonia from thegastrointestina l (GI) tract wi l l be beneficia l . What antibiotic would be most useful in this s i tuation?

a. Cephalothinb. Chloramphenicolc. Neomycind. Penici l l in Ge. Tetracycl ine

459. A 19-year-old gi rl who previous ly was heal thy develops bacteria l meningi ti s . What would you cons ider to be the drug of choice for thiss i tuation?

a. Ceftriaxoneb. Erythromycinc. Penici l l in Gd. Penici l l in Ve. Proca ine penici l l in

460. A patient i s being treated with an antibiotic for a vancomycin-res is tant enterococca l (VRE) infection. They consume an over-the-countermedication conta ining ephedrine and develop a s igni ficant spike of blood pressure that leads to a pounding headache. They are transported tothe hospi ta l . As part of the work-up, blood tests indicate some bone marrow suppress ion. Which antibiotic i s most l ikely associated with thiscl inica l picture?

a. Azi thromycinb. Ciprofloxacinc. Erythromycin estolated. Gentamicine. Linezol id

461. You just s tarted a rotation on the Infectious Disease consul t service. It i s at an academic medica l center that i s in a major hub ci ty forinternational travel , so unusual or uncommon diseases are commonly seen here. One of the patients i s receiving ethambutol . What i s the mostl ikely reason for which this drug i s being given?

a. Amphotericin-res is tant Candida infectionb. E. coli infection, severe, with profound fluid and electrolyte lossc. Entamoeba dispar infection (amebias is )d. M. tuberculosis infectione. Plasmodium (malaria l ) infection

Page 228: Pharmacology pre test self assessment and review (14th ed.)

Antimicrobial and Antiviral Pharmacology

Answers

413. The answer is e. (Brunton, pp 1468-1470, 1817; Katzung, p 832t.) Sul fonamides cross the placenta and enter the fetus in concentrations sufficient toproduce toxic effects . They compete with and displace bi l i rubin from plasma protein binding s i tes , ra is ing free bi l i rubin levels and caus ing thejaundice and other mani festations of kernicterus . For the same reason, sul fonamides should a lso not be given to neonates , especia l ly prematureinfants . This woman should not have been given the sul fonamide, whether a lone or in combination with trimethoprim. Ni trofurantoin usual lywould be preferred for uncompl icated urinary tract infections in pregnant women. None of the other drugs are associated with this neonata lhyperbi l i rubinemia: amoxici l l in or other aminopenici l l ins , a ; azi thromycin, b; erythromycin, c; or cephalosporins , fourth-generation or other, d.

414. The answer is e. (Brunton, pp 1632-1633; Katzung, pp 863, 872, 875-876.) Res is tance develops rapidly when only zidovudine or another nucleos ideanalog i s used to treat HIV. It most l ikely involves decreased incorporation of the active metabol i te into vi ra l DNA, or increased excis ion of i t fromvira l DNA. Zidovudine and other nucleos ide analogs must be phosphorylated (not dephosphorylated, a ) to form the active metabol i te, thedideoxynucleotide triphosphate. It occurs via the actions of severa l kinases in host cel l s . That levels of active metabol i te in CSF are undetectable(a) i s fa lse. In most cases CSF concentrations of active drug reach about 60% of those measured in the plasma at the same time. The ul timateeffect of these drugs i s , indeed, to s top vi ra l nucleic acid synthes is (c), but that occurs only after the adminis tered drug i s phosphorylated (to formthe nucleotide/triphosphate). The active metabol i te i s a good substrate for HIV reverse transcriptase, but a very poor one for DNA polymerases inhost cel l s (and so e i s incorrect).

And the main adverse responses to or s ide effects associated with zidovudine? They include bone marrow toxici ty,leukopenia/granulocytopenia , and anemia; potentia l ly severe or pers is tent insomnia and headache, nausea, and vomiting. The s ide effectsprofi les di ffer somewhat among the other dideoxynucleos ides .

415. The answer is e. (Brunton, pp 1474-1475; Katzung, pp 892-893, 898.) Al though severa l of the antimicrobia l agents l i s ted here can cause one (orperhaps two) of the adverse responses noted here, ni trofurantoin i s the most l ikely cause. GI s ide effects (anorexia , nausea, and vomiting) are themost common s ide effects caused by this drug, which i s s ti l l widely used for managing acute lower urinary tract infections (eg, from many s tra insof E. coli, s taphylococci, streptococci, Neisseria, Bacteroides). However, the drug can a lso cause acute or subacute pulmonary reactions such as thosedescribed, various hematologic reactions (in particular, leukopenia and thrombocytopenia), and periphera l sensory and motor neuropathies .

Amoxici l l in (a )—and most other penici l l ins—may cause centra l neurotoxici ty i f present at extraordinari ly high plasma levels . Beyond that,a l lergic reactions are the most important adverse responses . Pulmonary, hematologic, and periphera l neuropathic adverse responses are notassociated with these drugs .

Azi thromycin’s (b) profi le of adverse effects i s qui te s imi lar to that of erythromycin and other macrol ides , and that profi le does not include thes igns and symptoms noted here (GI upset, mi ld to severe, are the most common compla ints , and azi thromycin may be ototoxic, a property notshared by other macrol ides ).

Ciprofloxacin (c; or other fluoroquinolones) i s not l ikely to cause any of the adverse responses noted. They are qui te wel l tolerated and cause avariety of s ide effects that, in genera l , are mi ld. If one were to reca l l one “unique” toxici ty, i t would be a l terations of col lagen metabol i sm thatmay lead to tendon rupture. There i s a lso evidence that the drug can prolong the QT interva l , leading to arrhythmias , particularly in patients withlong QT syndrome.

Isoniazid (d) can cause periphera l neuropathy (mainly from a drug-induced pyridoxine deficiency) and hepatotoxici ty. Pulmonary and bleedingproblems are not at a l l common in terms of drug-induced problems.

416. The answer is c. (Brunton, pp 1521-1523, 1534-1535; Katzung, p 815.) More so than any other antibiotic, cl indamycin i s associated with the highestri sk of AAPMC (C. difficile superinfection). Thus , i t i s mainly reserved for certa in anaerobic infections located outs ide the CNS (susceptibleanaerobes include B. fragilis, Fusobacterium, and Clostridium perfringens, plus anaerobic streptococci). Tetracycl ine use i s a lso associated with AAPMC.The phenomenon i s not typica l ly attributed to treatment with amoxici l l in (a ), azi thromycin (b), or the trimethoprim-sul famethoxazole combination(e). Metronidazole (d) or vancomycin (not l i s ted) are cons idered preferred treatments for AAPMC—and the offending drug should be s topped assoon as AAPMC is suspected.

417. The answer is c. (Brunton, pp 1428-1430, 1523; Katzung, pp 891-892, 927-929.) Metronidazole i s dis tributed to a l l body fluids and ti ssues . Itsspectrum of activi ty i s l imited largely to anaerobic bacteria—including B. fragilis—and certa in protozoa. It i s cons idered to be the drug of choice fortrichomonias is in females and carrier s tates in males , as wel l as for intestina l infections with Giardia lamblia.

Doxycycl ine (a), emetine (b, an antiprotozoal ), pentamidine (d; used for treatment of P. jirovecii, pneumonia , or prophylactica l ly for that conditionin high-ri sk HIV-pos i tive patients ), and pyrazinamide (e, a bactericida l agent used for M. tuberculosis infections) would not be appropriate for thepatient described in the question.

418. The answer is b. (Brunton, pp 1409-1415; Katzung, pp 923-924.) Hemolys is i s the most common and serious adverse response to prima-quine. Therisk i s clearly highest in patients who have red cel l deficiencies in glucose-6-phosphate dehydrogenase, a heri table tra i t and one that can bescreened for before giving the drug. (This G6PD deficiency i s more common in blacks , and whites with darker skin [eg, some from certa in regions ofthe Middle East or the Mediterranean countries ].) Regardless of the resul ts of pretreatment screening, periodic blood counts should be done, andthe urine checked for unusual darkening (indicating the presence of hemoglobin from lysed red cel l s ), during treatment.

Electrophys iologic changes , whether they affect the conducting ti ssues of the heart (a ) or the bra in (e) are not commonly associated with prima-quine. The same appl ies to renal (c) or retina l (d) pathology, which occur at far lower rates than the hemolytic disease.

Note: If you answered “retinopathy (d),” you were probably thinking about chloroquine, because that adverse response (accompanied by visua lchanges) i s associated with that other commonly used antimalaria l drug.

419. The answer is e. (Brunton, pp 1521-1522, 1537-1538; Katzung, pp 817, 819.) Linezol id i s a bacteriostatic (not cida l , so answer a i s wrong) inhibi tor ofprotein synthes is . Its unique mechanism of action, binding to the 23S portion of the 50S ribosomal subunit, ul timately blocks formation of theini tiator complex that i s necessary for bacteria l repl ication. This unique mechanism means l inezol id i s not susceptible to cross -res is tance thatmay have developed to other antibiotics , but the more the drug i s used the greater the l ikel ihood that res is tance to i t wi l l develop.

Linezol id i s , indeed, the preferred drug for vancomycin-res is tant enterococci . Vancomycin i s often used as a “drug of las t resort” for seriousinfections that cannot be treated with fi rs t-l ine a l ternatives . Another major use i s for methici l l in-res is tant Staph. aureus, for which l inezol id and

Page 229: Pharmacology pre test self assessment and review (14th ed.)

vancomycin seem to be equal ly efficacious . (There are other approved uses for l inezol id, but s ince few bacteria are res is tant to i t—so far—itshould be used sparingly, only when absolutely needed.) It i s not at a l l indicated for relatively minor infections , especia l ly those with gram-negative causes (b). It i s not a preferred a l ternative to amoxici l l in for oti ti s media (c). Doxycycl ine, not l inezol id, i s the preferred a l ternative tociprofloxacin for anthrax prophylaxis or treatment (d).

The most serious adverse effect associated with l inezol id i s revers ible myelosuppress ion. Complete blood counts need to be done at leastonce a week i f this drug i s being adminis tered to myelosuppressed patients (including those taking other myelosuppress ive drugs) or i f l inezol idtherapy las ts more than about 14 days .

420. The answer is d. (Brunton, pp 1012-1013, 1524-1525, 1529, 1534, 1549-1554; Katzung, pp 840-842, 848, 1160t.) Ri fampin induces cytochrome P450enzymes , which causes a s igni ficant increase in el imination rates of many interacting drugs , such as ora l contraceptives , anticoagulants ,ketoconazole, cyclosporine, and chloramphenicol . It a l so promotes urinary excretion of methadone, which may precipi tate withdrawal .

Ethambutol (a ), i soniazid (b), pyrazinamide (c), and vi tamin B6 (e) are not P450 inducers , a l though the metabol i sm of some of these drugs can beinduced by ri fampin.

421. The answer is b. (Brunton, pp 1650-1654; Katzung, pp 870-871, 878-881.) Protease inhibi tors , as a class , tend to cause hyperglycemia (probablyinvolves insul in res is tance in parenchymal cel l s ) and hyperl ipidemia (precise mechanism not known).

The other adverse responses l i s ted are more typica l of those caused by nucleos ide reverse transcriptase inhibi tors (NRTIs ; zidovudine, others ),and i t i s worthwhi le commenting on that here. Anemias and neutropenia (myelosuppress ion, a ) are fa i rly common with NRTIs . The ri sk of NRTI-induced lactic acidos is i s rare, but the ri sks go up s igni ficantly i f the patient receives more than one NRTI. Lactic acidos is tends to develop a longwith hepatomegaly and fatty l iver degeneration (s teatos is ). Periphera l neuropathies (d) tend to be associated with didanos ine and s tavudine butnot with other members of the NRTI class including the prototype, zidovudine. Pancreati ti s (e) i s a l so mainly associated with NRTIs (and amongthe class , mainly didanos ine and s tavudine), not with protease inhibi tors .

422. The answer is c. (Brunton, pp 1409-1410, 1534, 1819; Katzung, pp 919t, 923-924.) Primaquine i s effective aga inst the extraerythrocytic forms of P. vivaxand P. ovale, and i s a lso indicated for some cases of P. falciparum. It helps eradicate plasmodia from the l iver, and in doing so i t not only provides acure but a lso helps prevent relapse.

Chloroguanide (a) i s a very old antimalaria l drug but i s seldom used instead of primaquine. Chloroquine (b) would not be used because i t i seffective only in the erythrocytic phase of the malaria l paras i tes ’ l i fe span: i t wi l l not work aga inst the exoerythrocytic forms of malaria paras i tes ,nor can i t serve as primary prevention. This 4-aminoquinol ine derivative i s a weak base that selectively concentrates in infected red blood cel l s .There i t probably interferes with the abi l i ty of plasmodia to convert heme—a toxin to the paras i te—to nontoxic metabol i tes .

Pyrimethamine (d) and quinacrine (e) are not sui table agents for treating the patient described in the question.

423. The answer is e. (Brunton, pp 1521, 1537, 1539-1542, 1545; Katzung, pp 798t, 802-803, 808.) Vancomycin, which must be given intravenous ly, has anantimicrobia l spectrum closest to those of penici l l ins and cephalosporins , and i t does not cross react in immunologica l ly susceptible patients .Like penici l l ins and cephalosporins , there i s a growing ri sk of res is tance to vancomycin (particularly in hospi ta l settings , where the drug i s mainlyused), but i t nonetheless i s cons idered the best (i f not the las t resort) drug for these patients . Clotrimazole (a) i s an anti fungal drug, mainly usedfor such fungal infections as those involving Candida. Metronidazole (c) i s a l so used mainly for systemic or urinary tract fungal infections , a l thoughit does have some antibacteria l activi ty. Gentamicin (b) would be inappropriate, in part because i ts spectrum of activi ty i s not l ikely to includeorganisms ki l led or inhibi ted by penici l l ins , cephalosporins , or vancomycin. The same appl ies to tetracycl ines (d).

424. The answer is e. (Brunton, pp 1632-1633; Katzung, pp 863f, 872t, 875-876, 889.) One of zidovudine’s major adverse effects i s bone marrow depress ionthat appears to be dose- and length-of-treatment-dependent. The severi ty of the disease and a low CD4 count contribute to the bone marrowdepress ion.

425. The answer is c. (Brunton, pp 1479t, 1488; Katzung, pp 790-797.) Unl ike the other l i s ted drugs , oxaci l l in i s res is tant to penici l l inase (β-lactamase)and i t i s one of a relatively few penici l l ins class i fied as anti -s taphylococca l agents . (The main other ones are cloxaci l l in, dicloxaci l l in, andnafci l l in.) Oxaci l l in and the related drugs are genera l ly speci fic for gram-pos i tive microorganisms. The other drugs l i s ted are broad- (or extended-)spectrum penici l l ins . Penici l l in G (the intravenous dosage form) or V (ora l ) i s indicated for s taph infections provided the s tra ins do have β-lactamase activi ty.

426. The answer is a. (Brunton, pp 1507-1509, 1512-1515, 1543; Katzung, pp 821-827.) Of a l l the protein synthes is inhibi tors , only the aminoglycos idesroutinely cause bacteria l death (bactericida l effect), not just suppress ion of growth or repl ication (bacteriostatic effect). Cl indamycin (b) anderythromycin (c) have bacteriostatic effects , the main mechanism being binding to the 50S subunit of bacteria l ribosomes. Linezol id (d) i s a l somainly a bacteriostatic drug (aga inst staphylococci, and enterococci, for example), a l though in some cases i t can exert bactericida l effects on somestra ins of streptococci. Tetracycl ines (e) are largely bacteriostatic and have a broad spectrum of activi ty. They bind revers ibly to the ribosomal 30Ssubunit. The ul timate effect i s inhibi tion of amino acid incorporation into peptides that otherwise would be formed via RNA activi ty.

427. The answer is b. (Katzung, pp 869-876.) Zidovudine competi tively inhibi ts HIV-1 nucleos ide reverse transcriptase. Zidovudine i s a prodrug: i t mustbe metabol ica l ly activated (phosphorylated) to zidovudine triphosphate. The active metabol i te i s incorporated in the growing vi ra l DNA chain, andin doing so prevents reverse transcriptase from adding more bases . As a resul t, DNA strand growth s tops . Zidovudine decreases the rate of cl inica ldisease progress ion and prolongs surviva l in HIV-infected patients when used as part of a multidrug treatment plan.

An example of a nonnucleos ide reverse transcriptase inhibi tor (a ) i s efavi renz. Unl ike zidovudine, i t i s not a s tructura l ana log of natura l lyoccurring nucleos ides , and does not work by caus ing premature termination of DNA growth. It and other nonnucleos ide reverse transcriptaseinhibi tors work mainly by di rectly binding to, and inhibi ting, reverse transcriptase.

Protease inhibi tors (e) block the fina l s tep in HIV maturation. Reca l l that HIV proteins are synthes ized together as one long peptide that i slargely without biologica l activi ty. Protease cleaves the polyprotein into severa l separate—and functional—proteins . Without this fina l process ingstep HIV cannot form new, and infectious , vi rus particles . Typica l ly, protease inhibi tor therapy involves two drugs , lopinavi r plus ri tonavi r being themost common.

428. The answer is d. (Brunton, pp 1629-1640; Katzung, pp 834-836, 1160t.) Bacteria l DNA gyrase i s composed of four subunits . Ciprofloxacin, levofloxacin,and other quinolones bind to the s trand-cutting subunits and in doing so inhibi ts bacteria l growth and repl ication. (The origina l member of thisgroup was na l idixic acid.) These antibiotics have a broad spectrum of antibiotic activi ty and are relatively free from common or serious s ide effects(some members associated with s igni ficant toxici ties were withdrawn from the market), and tend not to be associated with rapidly developing

Page 230: Pharmacology pre test self assessment and review (14th ed.)

antibiotic tolerance to thei r actions .

429. The answer is e. (Brunton, pp 1434-1435; Katzung, pp 929-932, 935.) Pentamidine i s effective in pneumonia caused by P. jiroveci. (So i s thetrimethoprim-sul famethoxazole combination, not l i s ted.) This protozoal disease usual ly occurs in immunodeficient patients , such as those withAIDS. Ni furtimox (c) i s effective in trypanosomias is , and metronidazole (b) in amebias is and leishmanias is , as wel l as in anaerobic bacteria linfections . Penici l l ins (a or d) are not cons idered drugs of choice for protozoal diseases such as those mentioned here.

430. The answer is c. (Brunton, pp 599, 692, 1086-1088, 1094, 1400, 1410-1411, 1414, 1421, 1468-1470; Katzung, pp 833-834.) Trimethoprim inhibi tsdihydrofol ic acid reductase. Sul famethoxazole inhibi ts p-aminobenzoic acid (PABA) from being incorporated into fol ic acid by competi tiveinhibi tion of dihydropteroate synthase. Ei ther action inhibi ts the synthes is of tetrahydrofol ic acid, but by giving the combination we inhibi t twoessentia l and sequential s teps in the formation of folate-dependent metabol i tes that are necessary for bacteria l viabi l i ty and repl ication.

431. The answer is b. (Brunton, pp 1478-1481, 1494t, 1496-1499; Katzung, pp 797-801, 807.) Cefotetan (second-generation cephalosporin)—as wel l ascefoperazone (thi rd-generation) inhibi t a ldehyde dehydrogenase and cause accumulation of aceta ldehyde (as does disul fi ram, which i s used tocurb a lcohol intake), and so can cause a l l the typica l and potentia l ly serious consequences of a disul fi ram-l ike reaction. Cefmetazole, a second-generation cephalosporin, and metronidazole, a lso cause a s imi lar adverse interaction with a lcohol . (Note that these three cephalosporins area lso the ones that are associated with vi tamin K-related bleeding problems, as addressed in Question 434.)

Erythromycin (whether adminis tered as the base or one of the common sa l ts , eg, ethylsuccinate, estolate, or s tearate) can inhibi t the hepaticP450 system sufficient to cause adverse interactions with (excess ive effects of) such drugs as warfarin, carbamazepine, and theophyl l ine. However,based on current evidence there i s no speci fic inhibi tion of a ldehyde dehydrogenase, nor resul ting accumulation of aceta ldehyde, that wouldcorrectly qual i fy as a disul fi ram-l ike interaction.

Amoxici l l in, penici l l in G, and other penici l l ins do not participate in disul fi ram-l ike reactions .Linezol id inhibi ts monoamine oxidase (MAO), a lbei t weakly, and so can trigger potentia l ly s igni ficant adverse interactions in persons receiving

such sympathomimetics as coca ine, ephedrine, or pseudoephedrine; or those consuming tyramine-rich foods or beverages . However, the drugdoes not inhibi t a lcohol metabol i sm or cause the adverse responses noted in this question.

432. The answer is d. (Brunton, pp 1470-1474, 1532-1533, 1560; Katzung, pp 835-837.) Levofloxacin or another quinolone blocks bacteria l DNA synthes is byinhibi ting bacteria l DNA gyrase (topoisomerase II). Ei ther would be a good choice, not only because of thei r antibiotic spectrum of activi ty, buta lso because the working hypothes is i s that we’re deal ing with a community-acquired respiratory infection caused by res is tant bacteria . Theres is tance i s l ikely to render penici l l ins (a , e), cephalosporins (b), and even erythromycin or other macrol ides (c) ineffective. Res is tance tofluoroquinolones has not become that much of a problem yet. Note that cefazol in (b), a fi rs t-generation cephalosporin, i s ineffective aga instpneumococci , and i s mainly sui table only for infections with S. aureus or s treptococci that are not res is tant to penici l l ins . Vancomycin (f) mightul timately be needed, but s ince i t must be given parentera l ly (there was no mention that the patient was unable to take ora l medications) i twould be premature to use this otherwise “las t resort” antibiotic.

433. The answer is e. (Brunton, pp 1521-1522, 1537, 1539-1542; Katzung, pp 802-803.) The only sui table drug l i s ted for this potentia l ly fata l infection i svancomycin. The finding of MRSA is not uncommon, particularly in hospi ta ls , rendering therapy with penici l l ins (including amoxici l l in withclavulanate, the β-lactamase inhibi tor; a ) ineffective. Macrol ides (including cl indamycin, b; and erythromycin, c) are a lso unsui table, in partbecause of a growing res is tance problem. Res is tance to the trimethoprim-sul fa-methoxazole (d) combination i s a lso a major problem, regardlessof the infections for which this folate reductase inhibi tor therapy i s prescribed.

434. The answer is c. (Brunton, pp 1494t, 1525; Katzung, pp 800-801.) Cefmetazole and cefotetan, both second-generation cephalosporins , andcefoperazone (thi rd generation) can interfere with hepatic vi tamin K metabol i sm, caus ing hypoprothrombinemia and what amounts to a deficiencyof vi tamin K-dependent clotting factor activi ty. Because this i s the genera l mechanism by which warfarin exerts i ts anticoagulant effects , combineduse of one of these cephalosporins can cause further (and potentia l ly dangerous) prolongations of the International Normal ized Ratio (orprothrombin time); the cl inica l consequence can be spontaneous , prolonged, or excess ive bleeding. One should a lso be cautious when thesecephalosporins are given to patients taking aspi rin or other antiplatelet drugs (eg, clopidogrel ) or thrombolytics .

Al though most cephalosporins are excreted unchanged by the kidneys , renal fa i lure (b, especia l ly severe and acute) seldom occurs with theseor other cephalosporins . There i s no l ink between adminis tration of even high doses of these cephalosporins (or others ) with the development ofacute heart fa i lure (a). Hypertens ion (d) or other substantia l changes of blood pressure are not associated with cephalosporins , nor are thesedrugs ototoxic (e).

Al though a his tory of severe a l lergic reactions to penici l l ins requires caution when cons idering a cephalosporin (indeed, cephalosporinsshould be avoided, i f poss ible, in such patients ), there i s nothing unique about cefmetazole, cefoperazone, or cefotetan in this context. None ofthe cephalosporins are contra indicated for patients with mi ld a l lergic reactions due to penici l l ins .

435. The answer is a. (Brunton, pp 1521, 1542-1543, 1545; Katzung, pp 803-804, 808.) Daptomycin, class i fied as a cycl ic l ipopeptide and rapidlybactericida l aga inst nearly a l l gram-pos i tive bacteria , i s uniquely associated (among antimicrobia l agents ) with myopathy and i ts typica l s ignsand symptoms. Dai ly dosages currently recommended pose a low ri sk of myos i ti s and myopathy, but nevertheless creatine kinase levels (markerof muscle damage) should be measured periodica l ly during the course of therapy and the patient should be advised to report any relevantsymptoms at once.

None of the other drugs l i s ted are associated with myopathy. GI s ide effects are qui te common with usual doses of erythromycin (b) and othermacrol ides , and sudden cardiac death (from QT prolongation) has been reported when erythromycin blood levels get too high. The estolate sa l t oferythromycin estolate (sa l t) may cause cholestatic hepati ti s . Gentamicin (c, and most other aminoglycos ides) genera l ly cause few minor s ideeffects ; thei r major adverse effects are ototoxici ty and nephrotoxici ty. The main common s ide effects associated with l inezol id (d) are nausea anddiarrhea, and the potentia l for usual ly revers ible myelosuppress ion and pancytopenia . Vancomycin (e) tends to cause facia l and neck flushing,pruri tus , and urticaria (so-ca l led “red man syndrome”), probably secondary to loca l his tamine release; his tamine release may a lso contribute tothe development of tachycardia and hypotens ion. As with the aminoglycos ides , vancomycin i s ototoxic at high plasma levels .

436. The answer is f. (Brunton, pp 1520-1526, 1528, 1532, 1544, 1817; Katzung, pp 810-813.) Tetracycl ines interact with many polyva lent meta l cations suchthat antibiotic absorption from the gut (ie, bioava i labi l i ty) i s reduced. The extent of this reduction can be cl inica l ly s igni ficant, that i s , leading toinadequate blood levels and effects of the antibiotic.

Ca lcium is , of course, abundant in da i ry products , some OTC antacid products , and in supplements touted for “bone heal th.” Other meta ls thatcan interact with tetracycl ines by this mechanism include i ron, magnes ium, a luminum, and zinc. Note that one or severa l of these interactants are

Page 231: Pharmacology pre test self assessment and review (14th ed.)

typica l ly found in antacid products , multivi tamin/minera l supplements , and even (the trend i s growing) in some (minera l -) forti fied foods , such ascerea ls , and ci trus juices .

Al though foods (in genera l ) may interfere with the ora l absorption of severa l other antibiotics , the cation-antibiotic interaction i s speci fic forand important to the tetracycl ines .

437. The answer is d. (Brunton, pp 1429-1430, 1523; Katzung, pp 891-892, 927-929.) Among the indications for metronidazole i s management of C. difficile(and other clostridia) infections , including AAPMC. Many other obl igate anaerobes , mainly gram-pos i tive baci l l i , wi l l respond. So wi l l various typesof intestina l or systemic amebias is infections (the drug i s genera l ly used adjunctively with iodoquinol for gut infections—symptomaticamebias is ); and for giardias is and Trichomonas vaginalis infections (genera l ly the drug of choice). An a l ternative to metronidazole in the setting ofAAPMC would be vancomycin.

None of the other drugs l i s ted would be sui table. Amphotericin B (a) i s an anti fungal agent. Cl indamycin (b) i s not at a l l the preferredtreatment; indeed, i t i s the most l ikely cause of the superinfection. Gentamicin (c) i s not effective aga inst anaerobes , C. difficile of course included.Trim-ethoprim plus sul famethoxazole (e) i s not indicated for C. difficile infections . The main uses of this broad-spectrum antibiotic combinationare treating most uncompl icated urinary tract infections (especia l ly i f chronic or recurrent), and treating pneumonia and other P. jiroveci infections(mainly in immunocompromised patients with AIDS, cancer, or organ transplants ).

I hope you have learned that the most effective treatment for entera l C. difficile infections , especia l ly those that are severe or refractory toordinari ly preferred drug therapy, i s not some brand new medicine. It i s a fecal transplant, and in essence, i t involves just what the term impl ies .The wonders of modern medicine!

438. The answer is b. (Brunton, pp 1012, 1327-1328, 1529, 1534; Katzung, pp 813-814.) Erythromycin (and cl indamycin) are macrol ide antibiotics that ratherpowerful ly inhibi t metabol i sm of many other drugs , ra is ing blood levels of the interactants . Warfarin i s el iminated a lmost exclus ively bymetabol i sm, and so the excess ive effects described in the question are qui te l ikely. Al though azi thromycin (a) i s a macrol ide, i t i s not a P-450inhibi tor (or inducer). Gentamicin (c) and other aminoglyco-s ides are el iminated solely by renal excretion and have no di rect effects on hepaticdrug metabol i sm or any other important aspects of l iver function. The same appl ies to penici l l in G (d) and other penici l l ins , including thoseclass i fied as penici l l inase-res is tant, broad spectrum (aminopenici l l ins ), and extended spectrum (mainly used aga inst Pseudomonas incombination with an aminoglycos ide). Ri fampin (e) i s a P-450 inducer, not an inhibi tor, and i s obvious ly a cause of many drug-drug interactions asa resul t of i ts s timulatory effect on hepatic metabol i sm.

439. The answer is a. (Brunton, pp 1483, 1487-1490, 1814; Katzung, pp 39t, 795-796.) Compared with ampici l l in, amoxici l l in absorption i s a ffected less bythe presence of food in the gut, so the bioava i labi l i ty i s better and dos ing may be more convenient for many patients . Both drugs are inactivatedby β-lactamases , and so nei ther i s effective aga inst penici l l inase-producing organisms (b). Both drugs are broad spectrum (c) and share the sameantimicrobia l spectrum as penici l l in G, and additional ly are effective, for example, aga inst E. coli, H. influenzae, Salmonella, and Shigella; both a lsocan cross react in patients with a his tory of hypersens i tivi ty reactions (d) to other penici l l ins . Nei ther amoxici l l in nor ampici l l in hasantipseudomonal activi ty. The penici l l ins capable of that are the extended-spectrum penici l l ins , for example, piperaci l l in and ticarci l l in; note thatthe extended-spectrum penici l l ins are susceptible to inactivation by β-lactamase and so are not effective aga inst severa l s tra ins of organisms,notably S. aureus.

440. The answer is c. (Brunton, pp 1477-1493, 1494-1499; Katzung, pp 796, 800.) Unless there are no reasonable a l ternatives , cephalosporins should beavoided for patients with prior severe a l lergic responses to penici l l ins because of thei r cross -reactivi ty. None of the other drugs or drug groupsl i s ted here cross react in penici l l in-sens i tive patients . “Mi ld” a l lergic reactions to penici l l ins do not necessari ly contra indicate cephalosporinuse.

441. The answer is c. (Brunton, pp 1516-1517; Katzung, pp 821-826.) Amikacin s tands out among a l l the aminoglycos ides in two main ways : i t has thebroadest spectrum against gram-negative baci l l i , and i t i s least susceptible to bacteria l enzymes that inactivate aminoglycos ides and lead tores is tance. (Reca l l that among gram-negative bacteria , genetic information that codes for the production of these inactivating enzymes i stransferred via R factors .)

442. The answer is b. (Brunton, pp 1424, 1571-1575; Katzung, pp 849-852, 934.) Amphotericin B, given intravenous ly, often a l ters kidney function. The mostcommon and most eas i ly detected mani festation of this i s decreased creatinine clearance. If this occurs , the dose must be reduced. AmphotericinB a lso commonly increases potass ium (K+) loss , leading to hypokalemia; and can cause anemia and neurologic symptoms. A l iposomalpreparation of amphotericin B may reduce the incidence of renal and neurologic toxici ty (see explanation for the next question). Vancomycin (e)may cause renal damage, but the overa l l incidence i s lower, the severi ty less . Acyclovi r (a ), ceftazidine (c), and penici l l ins (d) rarely are causa l lyrelated to the development of acute renal fa i lure.

443. The answer is a. (Brunton, pp 1478-1481, 1571-1575; Katzung, pp 790-792, 849-852, 857t.) Penici l l ins , cephalosporins , and amphotericin B exert thei rdes i red cl inica l effects by a l tering the s tructure or function of cel l wal l s of susceptible organisms. Penici l l ins interfere with bacteria l cel l wal lsynthes is : thei r β-lactam structures bind to and inhibi t enzymatic function of transpeptidases that normal ly provide susceptible bacteria with cel lwal l s that are capable of mainta ining an osmotica l ly s table intracel lular mi l ieu. Cephalosporins , by vi rtue of thei r β-lactam ring, work inessentia l ly the same way. Amphotericin B (a polyene anti fungal drug) binds to ergosterol in the fungal cel l membrane; the ul timate outcome isincreased cel l permeabi l i ty. Nonetheless , the ul timate effect i s osmotic instabi l i ty of the organism, leading to cel l death.

Neither penici l l ins , cephalosporins , or amphotericin B are contra indicated in patients with immunodeficiencies (b). Indeed, they may play a keyrole in managing opportunis tic infections in such patients .

They do not induce the metabol i sm of other drugs (c), or interact in most of the typica l pharmacokinetic ways .Amphotericin B can cause decreased platelet counts and leukopenia , but this i s rare. The most common hematologic adverse response to this

drug i s a normochromic, normocytic anemia. Penici l l ins and cephalosporins do not share these properties (and so answer d i s incorrect).Fina l ly, amphotericin B i s clearly nephrotoxic (see the note below). When given intravenous ly, renal dys function i s the most serious and most

common long-term mani festation of this anti fungal drug’s toxic spectrum. Penici l l ins and cyclosporins may cause inters ti tia l nephri ti s , butcompared with amphotericin B the incidence and cl inica l consequences are qui te low.

Note: There are new l ipid formulations of amphotericin B—amphotericin B col loida l dispers ion, l iposomal amphotericin B, and a l ipid complexof the drug—apparently cause much less nephrotoxici ty than conventional amphotericin B formulations . That i s probably because these newerformulations a l ter dis tribution of the anti fungal drug such that renal concentrations , and so the nephrotoxic potentia l , are lower.

444. The answer is d. (Brunton, pp 1607-1608; Katzung, pp 886-887.) Osel tamivi r i s usual ly the recommended anti -influenza drug; zanamivi r i s a

Page 232: Pharmacology pre test self assessment and review (14th ed.)

recommended a l ternative. The biochemica l mechanism of action i s inhibi tion of vi ra l neuraminidase; that, in turn, inhibi ts the abi l i ty of newlyformed vi rions to “bud off” of host cel l membranes and spread the vi rus to other cel l s . Osel tamivi r or zanamivi r are a lso used prophylactica l ly foroutbreaks of other influenza A (or B) infections .

Acyclovi r (a ) i s active only aga inst herpesvi ruses (including herpes s implex,[which i s most sens i tive to acyclovi r]; varicel la -zoster; andcytomegalovi rus [but most s tra ins are res is tant]). Acyclovi r must form an active metabol i te, acyclo-GTP, which in turn inhibi ts vi ra l DNA polymerase.Amantadine (b) i s an antivi ra l drug for flu prophylaxis (mainly A s tra ins ) and treatment, but i ts use has decl ined sharply (as osel tamivi r orzanamivi r use has grown) because many vi ra l s tra ins quickly develop res is tance. In addition, amantadine tends to cause dis turbing CNS effects(dizziness , i rri tabi l i ty, insomnia , and impaired menta l concentration) in a large number of patients and cardiovascular depress ion. Amantadine i sa lso used occas ional ly as an adjunct to treatment of Parkinson disease. Lopinavi r (c) and ri tonavi r (e) are protease inhibi tors/anti retrovi ra l drugs .Those two drugs are used in combination, and with a nucleos ide/-tide reverse transcriptase inhibi tor (eg, zidovudine) for HIV infections .

Note: Given the number of H1N1 cases , and worries about a pandemic, in 2009 the FDA gave formal authorization to use ei ther of these drugs forflu-related indications for which they were not previous ly FDA-approved, including chi ldren younger than the preapproved minimum age.

445. The answer is d. (Brunton, pp 1365-1378, 1478-1481, 1505-1518, 1533; Katzung, p 825.) The rationale behind this combination i s that penici l l insessentia l ly weaken the cel l wal l s of susceptible bacteria , which in turn faci l i tates access of the aminoglycos ide to i ts s i te of action, the bacteria lribosomes. This usual ly provides better antibiotic response than with ei ther antibiotic used a lone, and with the aminoglycos ide, plasma levelsaren’t necessari ly so high that they are more l ikely to cause ototoxici ty, nephrotoxici ty, or other adverse responses .

You should a lso reca l l at least two other things : (1) the penici l l in in this combination i s usual ly an extended-spectrum penici l l in, such asticarci l l in; and (2) as I speci fica l ly noted in the question, the adminis tration of these drugs i s by separate IV l ines . That i s because i f the two drugswere mixed together in sufficiently high concentrations , the penici l l in may chemica l ly inactivate the gentamicin.

446. The answer is b. (Brunton, pp 1481-1483, 1529; Katzung, pp 792-793, 796.) Both susceptible and res is tant gram-negative and gram-pos i tive bacteriahave penici l l in-binding proteins (PBPs). Res is tance to the narrow spectrum penici l l ins by gram-negative bacteria ari ses from the presence of anouter membrane with pores that are too smal l to a l low adequate penetration of the drug and access to the PBPs . Thus , we are deal ing with whatamounts to a phys ica l barrier to the drug.

Most penici l l ins (with few exceptions , such as bacampici l l in) are active in the form in which they are adminis tered (ie, they are not prodrugs),and so no subsequent metabol ic activation i s required.

447. The answer is b. (Brunton, pp 1494t, 1496t, 1525; Katzung, pp 798t, 799.) Cefoxi tin, a second-generation cephalosporin, i s sui table for treatingintraabdominal infections caused by many aerobic and anaerobic gramnegative bacteria , including and especia l ly B. fragilis. (Al ternative second-generation cephalosporins that would be sui table are cefmetazole and cefotetan.) Cefoxi tin a lone has been shown to be as effective as thetradi tional therapy of cl indamycin plus gentamicin. The other cephalosporins have much lower antibiotic activi ty aga inst these organisms, and sogenera l ly are not appropriate.

448. The answer is a. (Brunton, pp 1512-1514, 1543; Katzung, p 824.) Aminoglycos ides (gentamicin, tobramycin, etc) are class ic examples of ototoxicdrugs , and they can affect both branches of the eighth crania l nerve.

The ri sks of aminoglycos ide-induced ototoxici ty (and nephrotoxici ty) are among the reasons why i t i s important to keep an eye on peak andtrough drug levels during therapy, adjust dosages accordingly, and avoid concomitant use of other ototoxic drugs . That i s because the hearing lossis blood level -dependent (as opposed to being an idiosyncratic or a l lergic reaction). Aminoglycos ide-induced ototoxici ty i s usual ly i rrevers ible.The ri sk and severi ty of hearing loss from aminoglycos ides are increased i f they are adminis tered with other ototoxic drugs (below).

Reca l l that there are two main forms of drug-induced ototoxici ty. Cochlear toxici ty includes hearing loss , tinni tus (“ringing in the ears”), oroccas ional ly both. Hearing loss may a lso occur with loop diuretics (particularly ethacrynic acid), ci splatin, and the vinca a lka loids (anticancerdrugs). These drugs are intrins ica l ly ototoxic; use one or more of them together or with an aminoglycos ide and the ri sk of ototoxici ty increasesgreatly.

Tinni tus (usual ly revers ible) i s typica l ly associated with such drugs as aspi rin (and, poss ibly, some other NSAIDs) and quinidine.The other main form of ototoxici ty i s vestibular toxici ty, which i s typica l ly mani fest as ba lance and ga i t problems, vertigo, and nausea resul ting

from vestibular apparatus dys function.Nephrotoxici ty may develop during or after the use of an aminoglyco-s ide. It i s genera l ly more common in the elderly when there i s preexis ting

renal dys function. In most patients , renal function gradual ly improves after discontinuation of therapy. Aminoglycos ides rarely causeneuromuscular blockade that can lead to progress ive flaccid para lys is and potentia l fata l respi ratory arrest. Hypersens i tivi ty and dermatologicreactions occas ional ly occur fol lowing use of aminoglycos ides .

None of the other antibiotics l i s ted are l inked to ototoxici ty, whether from excess ive blood levels or due to a hypersens i tivi ty or true a l lergicreaction. Azi thromycin (not an answer choice) i s , however, another antibiotic for which there i s growing evidence of a l ink to sudden onset hearingloss . The mechanism is unknown, and the incidence i s nei ther dose-dependent nor predictable.

449. The answer is b. (Brunton, pp 1579-1580; Katzung, pp 853-855.) Fluconazole, an azole anti fungal , penetrates into cerebrospina l fluid, where i texerts good anti fungal activi ty aga inst Cryptococcus neoformans. When i t i s given ora l ly, blood levels are a lmost as high as when i t i s givenparentera l ly. Amphotericin i s adminis tered intravenous ly and even when given intratheca l ly does not appear to be highly effective in fungalmeningi ti s .

450. The answer is d. (Brunton, p 1543; Katzung, pp 467-469, 475, 824) Aminoglycos ides , at sufficiently high blood levels , can cause skeleta lneuromuscular blockade in thei r own right. This probably ari ses from a combination of effects : inhibi tion of neuronal ACh release and perhapsdirect blockade of nicotinic receptors on skeleta l muscle. This would add to and prolong the effects of both neuromuscular blockers the patienthas received. In addition, i soflurane and other ha logenated hydrocarbon volati le l iquid anesthetics (which are being used less and less becauseof the ava i labi l i ty of effective intravenous anesthetics ) have some neuromuscular blocking effects in thei r own right—but not to a degree that i ssufficient to obviate the need for succinylchol ine and/or nondepolarizing blockers when skeleta l muscle para lys is i s indicated. So here we have acombination of drugs that affect skeleta l muscle activation.

The greatest concern, of course, would be the prolongation of neuromuscular blockade. A “greater degree” of para lys is i s largelyinconsequentia l , so long as venti lation i s supported. It i s the prolonged blockade—and especia l ly the return of skeleta l muscle weakening andventi latory insufficiency after mechanica l venti lation has been discontinued and additional doses of aminoglycos ide are given—which poses thegreatest ri sk i f the patient had a l ready been taken off venti latory support.

(Note, too, in your s tudies , that some other antimicrobia ls seem to have some skeleta l neuromuscular blocking activi ty, including polymyxin B

Page 233: Pharmacology pre test self assessment and review (14th ed.)

and cl indamycin.)You should reca l l that a l though such agents as i soflurane and other ha logenated hydrocarbon volati le l iquid anesthetics may potentiate the

effects of a neuromuscular blocker, once they are el iminated (that occurs fa i rly rapidly when adminis tration s tops) there i s no added ri sk ofprolonged or greater skeleta l muscle weakness or para lys is .

I should add that in settings (eg, hospi ta ls ) where the overa l l incidence of aminoglycos ide res is tance i s low (or in the absence of documentedres is tance to a particular aminoglycos ide in a particular patient), tobramycin or gentamicin i s usual ly the aminoglycos ide of choice. Amikacinshould be reserved for s i tuations where there i s proven res is tance to the a l ternatives .

As with other aminoglycos ides , periodic monitoring of peak and trough blood levels i s essentia l to help insure optimal antibiotic effects whi lereducing the ri sk of ototoxici ty and nephrotoxici ty—both of which can be caused by any aminoglycos ide (a l though with varying relative ri sks ). Noaminoglycos ide has “nephroprotective” effects .

Fina l ly, no aminoglycos ide can ki l l anaerobes .

451. The answer is e. (Brunton, pp 137-138t, 145, 1555-1558; Katzung, pp 840-841, 848.) Isoniazid (INH) inhibi ts cel l -wal l synthes is in mycobacteria .Increas ing vi tamin B6 levels prevents some common and potentia l ly s igni ficant compl ications associated with this inhibi tion, includingperiphera l neuri ti s , insomnia , restlessness , muscle twitching, urinary retention, convuls ions , and psychos is , wi thout affecting theantimycobacteria l activi ty of INH.

452. The answer is d. (Brunton, pp 1520-1526, 1528, 1532; Katzung, pp 810-813.) Tetracycl ines are drugs of choice in the treatment of Rickettsia,Mycoplasma, and Chlamydia infections . The antibiotics that act by inhibi ting cel l -wal l synthes is have no effect on Mycoplasma because the organismdoes not possess a cel l wal l ; penici l l in G (c), vancomycin (e), and baci tracin (a) wi l l be ineffective. Gentamicin (b) a lso has l i ttle or noantimicrobia l activi ty aga inst these organisms.

453. The answer is a. (Brunton, pp 1623-1660; Katzung, pp 871t, 874t, 880, 889.) Ri tonavir i s a powerful inhibi tor of the l iver’s P450 system. Ri tonavir i sused in combination with lopinavi r solely to inhibi t lopinavi r’s metabol i sm and enhance i ts abi l i ty inhibi t the fina l s tep in HIV maturation. Thus ,the ri tonavi r helps keep lopinavi r’s plasma concentration in a therapeutic range longer. Ri tonavir does have protease inhibi tory activation, but i ti s weak (especia l ly at the dosages used in combination with other protease inhibi tors : in this combination the ri tonavi r dose would besubtherapeutic i f i t were the only protease inhibi tor used). In this combination i t i s the lopinavi r, not the ri tonavi r (e) that i s caus ing the maintherapeutic effect. The protease inhibi tors do not cause hypoglycemia (c), nor does ri tonavi r reduce the abi l i ty of lopinavi r to a l ter plasma glucoselevels . In fact, whether used a lone or in combination, protease inhibi tors typica l ly cause hyperglycemia (and may cause cl inica l diabetes mel l i tus ,and qui te often ra ise cholesterol and triglycerides levels too).

The NRTIs (zidovudine, didanos ine) are, indeed, prodrugs that must be metabol ica l ly activated to the triphosphate form in order to serve as asubstrate for reverse transcriptase. However, nei ther lopinavi r nor ri tonavi r faci l i tate that metabol i sm (b).

454. The answer is d. (Brunton, pp 1505-1509, 1514-1515, 1517, 1525; Katzung, pp 822f, 824-825, 828, 840t, 843, 848.) Streptomycin i s bactericida l for thetubercle baci l lus organism. Other aminoglycos ides (eg, gentamicin, tobramycin, neomycin, amikacin, and kanamycin) have activi ty aga inst thisorganism but are seldom used cl inica l ly because of toxici ty or development of res is tance. Streptomycin i s arguably the most ototoxicaminoglycos ide.

455. The answer is d. (Brunton, pp 1489-1490; Katzung, p 801.) These agents are inhibi tors of penici l l inase (β-lactamase) and are used in conjunctionwith β-lactamase-sens i tive penici l l ins to potentiate thei r activi ty. These drugs are found in severa l brand-name fixed-dose penici l l in combinationproducts (amoxici l l in and clavulanic acid; sulbactam with ampici l l in; tazobactam with piperaci l l in). Clavulanic acid i s an i rrevers ible inhibi tor.These agents do not, per se, add activi ty aga inst Pseudomonas or Enterobacter (an activi ty a l ready possessed by piperaci l l in but not by ampici l l inor amoxici l l in). Likewise, they have no intrins ic effect to faci l i tate entry into the CNS or CSF. They do not inhibi t hepatic metabol i sm of thepenici l l ins , and you should reca l l that renal excretion (not metabol i sm) i s the main pathway for penici l l in el imination. Importantly, thepenici l l inase inhibi tors have absolutely no impact on the ri sks or severi ties of a l lergic reactions to penici l l ins .

456. The answer is d. (Brunton, pp 1533, 1558-1559; Katzung, pp 840t, 842.) An important problem in the chemotherapy of TB i s bacteria l drug res is tance.For this reason, concurrent adminis tration of two or more drugs should be employed to delay the development of res is tance. Ethambutol i s oftengiven a long with INH for this purpose. Streptomycin or ri fampin may a lso be added to the regimen to delay even further the development of drugres is tance.

457. The answer is d. (Brunton, pp 1507-1509; Katzung, pp 821-825.) Aminoglycos ides , which are mainly used for parentera l therapy of severe infectionsfrom aerobic gram-negative baci l l i (eg, E. coli, Serratia, Klebsiella), require oxygen in order for the drug to be transported across the bacteria l cel lmembrane. Such incorporation i s necessary for these drugs to exert thei r bactericida l effects , which arise from binding to the 30S subunit ofsusceptible bacteria . Ul timately the aminoglycos ide-ribosomal binding leads to premature termination of bacteria l protein synthes is and theformation of abnormal bacteria l proteins . Such abnormal proteins ul timately insert into the bacteria l cel l membrane, caus ing leakiness and cel ldeath.

The aminoglycos ides are not prodrugs , and so metabol i sm (whether aerobic or otherwise) i s not necessary for activi ty; formation of an oxygenfree radica l or an aminoglycos ide free radica l has nothing to do with thei r antibiotic effects . Moreover, metabol i sm by host cel l s i s not animportant process in the el imination of aminoglycos ides , nor of reducing host cel l toxici ty (eg, to the kidneys or audi tory nerve). Renal excretion i sthe main route of el imination for these drugs , which expla ins why renal function i s such an important cons ideration in dos ing adjustments .

Clearly, both aerobic and anaerobic bacteria can elaborate res is tance factors (or develop res is tance in other ways) to a variety of antibioticclasses . From a cl inica l viewpoint, the presence or absence of molecular oxygen i s not a crucia l or even relevant i s sue in this matter, however.

458. The answer is c. (Brunton, p 1518; Katzung, pp 826-827.) Neomycin, an aminoglycos ide, i s not s igni ficantly absorbed from the GI tract. After ora ladminis tration, the intestina l flora are suppressed or modi fied and the drug i s excreted in the feces . This effect of neomycin i s used in hepaticcoma to decrease col i form flora , thus decreas ing the production of ammonia and reducing levels of free ni trogen in the bloodstream. Otherantimicrobia l agents (eg, tetracycl ine, penici l l in G, chloramphenicol , and cephalothin) do not have the efficacy or potency of neomycin in caus ingthis effect. Note that chloramphenicol i s no longer ava i lable in the United States .

459. The answer is a. (Brunton, pp 1494t, 1496t, 1525, 1538; Katzung, pp 773-780, 781t, 783-784, 888-890.) Penici l l ins were used in the treatment ofmeningi ti s because of thei r abi l i ty to pass across an inflamed blood-bra in barrier. We don’t use them that way anymore, and so answers c, d, ande are incorrect. Erythromycin (b) dis tributes widely throughout the body, but not to the bra in or cerebrospina l fluid. The thi rd-generation

Page 234: Pharmacology pre test self assessment and review (14th ed.)

cephalosporin, ceftriaxone, i s preferred because i t i s effective aga inst β-lactamase producing s tra ins of H. influenzae that may cause meningi ti s inchi ldren.

460. The answer is e. (Brunton, pp 1531, 1533; Katzung, p 817.) There are severa l pieces of information you should l ink together to help arrive at theanswer, for which a relatively new drug i s the correct answer. (1) Al though l inezol id has severa l uses , i t i s best reserved for vancomycin-res is tantenterococci (VRE) and methici l l in-res is tant S. aureus (MRSA) infections . (It’s seldom a fi rs t-l ine antibiotic because of the ri sk of res is tance.) (2)Linezol id i s occas ional ly l inked to bone marrow suppress ion that i s usual ly revers ible upon discontinuation of the drug. (Granted, such otherantibiotics as chloramphenicol [no longer used in the United States ] pose greater ri sks of bone marrow suppress ion, but this property i snonetheless associated with l inezol id.) (3) The thi rd piece of evidence i s the ri se of blood pressure in response to ephedrine, a mixed-actingsympathomimetic that works , in part, by releas ing neuronal norepinephrine. Linezol id has monoamine oxidase inhibi tory activi ty (a lbei t relativelyweak compared with tradi tional MAO inhibi tors ). Piece these three l ines of evidence together and the only reasonable choice i s l inezol id.

461. The answer is d. (Brunton, pp 1558-1159; Katzung, pp 840-842.) Ethambutol , in combination with i soniazid, ri fampin, and pyrazinamide, i scons idered fi rs t-l ine therapy for M. tuberculosis infection. It has largely replaced aminosa l icyl ic acid as a part of the treatment plan, mainlybecause i t i s tolerated better. Ethambutol inhibi ts cel l wal l synthes is ; res is tance develops quickly unless adjuncts (l i s ted above) are givenconcomitantly. (Less common uses for ethambutol include infections with M. avium complex, M. kansasii, and M. marinum.)

Amphotericin-res is tant Candida infections (a) are relatively uncommon (some res is tant s tra ins have been i solated); nonetheless ethambutolhas no anti fungal activi ty. Ethambutol a lso has no activi ty aga inst infections with col i form bacteria (b), or with Entamoeba (c). Infections with P.falciparum or P. vivax (malaria ; e) mainly require management (prophylaxis , or treatment of active disease) with such drugs as chloroquine,mefloquine, doxycycl ine, or severa l other agents—but not ethambutol .

Page 235: Pharmacology pre test self assessment and review (14th ed.)

Cancer and Immune System Pharmacology

QuestionsCel l cycle, cel l cycle speci fici tyAlkylating agentsAnticancer hormones and their antagonis tsAnti tumor antibioticsAntimetabol i tesPlant a lka loidsImmunomodulators

462. Whi le reviewing charts in a genera l medicine cl inic you see that a patient, 55-year-old and with no his tory of cancer at a l l , i s takingmethotrexate (MTX). What i s the most l ikely condition for which this “anticancer drug” i s being given?

a. Asthma or emphysemab. Hyperthyroidismc. Hyperuricemia or cl inica l goutd. Myasthenia gravise. Rheumatoid arthri ti s or psorias is

463. A 47-year-old woman with choriocarcinoma is treated with methotrexate (MTX), given at dosages that you bel ieve may cause s igni ficant hostcel l toxici ty. Which drug would you give to l imit the toxic effects of the drug on normal host cel l s?

a . Deferoxamineb. Leucovorinc. N-acetylcysteined. Penici l laminee. Vi tamin K

464. You order vincris tine, the prototype of the vinca a lka loids , to a patient with a tumor that i s l ikely to be respons ive to this drug. What i s themost l ikely adverse response to this drug?

a. Nephrotoxici ty, renal dys function or fa i lureb. Neutropeniac. Periphera l sensory and motor neuropathyd. Pulmonary damagee. Thrombocytopenia , bleeding

465. A cancer patient develops severe, i rrevers ible cardiomyopathy because the maximum l i fetime dose of an anticancer drug was exceeded. Whatdrug was most l ikely respons ible for this?

a. Asparaginaseb. Bleomycinc. Cisplatind. Cyclophosphamidee. Doxorubicinf. Vincris tine

466. A patient with Wi lms tumor i s receiving a chemotherapeutic agent that i s described as working by interca lating into DNA strands , and that i sefficacious regardless of which s tage of the cel l cycle the tumor cel l s are in. Which drug best fi ts this description?

a. Anastrozoleb. Cytarabine (cytos ine arabinos ide)c. Doxorubicind. Fluorouraci le. Tamoxi fen

467. A 42-year-old woman is diagnosed with metastatic breast cancer. You cons ider use of tamoxi fen, toremifene, or fulvestrant. Why mightfulvestrant be the best choice, a l l other factors being equal?

a. Exerts antiplatelet, rather than thrombotic, effectsb. Lacks abi l i ty to cause hot flashes or other dis turbing s ide effectsc. Lower ri sk of caus ing endometria l cancerd. Provides cl inica l cure, rather than pa l l iation, in a l l patientse. Signi ficantly improves minera l dens i ty in, s trength of, long bones

468. A patient with chronic myelogenous leukemia (CML) i s being treated with imatinib. Which s ide effect or other s i tuation should you anticipatein response to imatinib therapy?

Page 236: Pharmacology pre test self assessment and review (14th ed.)

a. A high rate of therapeutic fa i lure, and the need to switch to interferonsα-2a and -2bb. Hypotens ion and hypovolemia due to s igni ficant drug-induced diures isc. Interactions with other drugs that depend on or affect the cytochrome P450 systemd. Signi ficant toxici ty to normal host cel l s due to profound inhibi tion of tyros ine kinasee. Thrombocytos is , wi th a high ri sk of intravascular clotting

469. As a rule, large (and older) sol id tumors are more di ffi cul t to eradicate when chemotherapy i s s tarted. Which tumor-based property expla insbest the reason for this chemotherapeutic l imitation?

a. Growth fraction s lows, more cel l s enter G0

b. Higher tumor blood flow washes away anticancer drugs fasterc. P-glycoprotein activi ty decreases as tumors get olderd. Their higher metabol ic rate makes them less vulnerable to chemotherapeutic agentse. Topoisomerase activi ty (abi l i ty to sel f-repair DNA strand damage) increases with tumor s i ze

470. A man has prostate cancer that wi l l be treated with leuprol ide. Which drug are you most l ikely to use adjunctively when s tartingchemotherapy?

a. An aromatase inhibi tor (eg, anastrozole)b. Flutamidec. Prednisone or another potent glucocorticoidd. Tamoxi fene. Testosterone

471. As part of the treatment plan for a Hodgkin disease patient we have given mechlorethamine. Which phrase best describes the anticancermechanism of action of this drug?

a. Alkylates DNA, caus ing cross -l inks between para l lel DNA strandsb. Blocks microtubular assembly and mitos is during M-phasec. Inhibi ts topoisomerase, preventing repair of DNA strand breaksd. Interca lates in DNA strands , thereby preventing DNA repl ication by mRNAe. Stabi l i zes microtubular arrays , thereby preventing mitos is

472. The oncology team has treated many patients with acute lymphocytic leukemia us ing a combination of drugs . One drug tends to cause a highincidence of lumbar and abdominal pa in, s igni ficant increases of plasma amylase and transaminase activi ty, and other symptoms of hepaticand/or pancreatic dys function. Some patients developed serious hypersens i tivi ty reactions upon drug adminis tration, and there have beenoccas ional sudden deaths . Which drug best fi ts this description?

a. Asparaginaseb. Azathioprinec. Doxorubicind. Methotrexatee. Vincris tine

473. A 30-year-old woman being treated for ovarian cancer develops high frequency hearing loss and decl ining renal function in response toanticancer drug therapy. Which drug i s the most l ikely cause?

a. Bleomycinb. Cisplatinc. Doxorubicind. 5-Fluorouraci le. Pacl i taxel

474. A 41-year-old woman comes to the outpatient area of the hematology-oncology center for her fi rs t course of adjuvant chemotherapy formetastatic breast cancer fol lowing a left modi fied radica l mastectomy and axi l lary lymph node dissection for infi l trating ducta l carcinoma of thebreast. Two biops ies were pos i tive for cancer.

Fol lowing premedication with dexamethasone and ondansetron, she wi l l receive combination chemotherapy with doxorubicin,cyclophosphamide, and fluorouraci l . Twenty-four hours after the fi rs t course of chemotherapy she wi l l s tart a 10-day regimen with fi lgrastim. Whatis the most l ikely reason for adminis tering the fi lgrastim?

a. Control of nausea and emes isb. Potentiate the anticancer effects of the chemotherapeutic agentsc. Prevent doxorubicin-induced cardiotoxici tyd. Reduce the ri sk/severi ty of chemo-induced neutropenia , and related infectionse. Stimulate the gastric mucosa to repair damage caused by the chemotherapy drugs

475. A cancer patient receives prophylactic a l lopurinol before a course of chemotherapy. What i s the main reason for giving the a l lopurinol?

a . Faci l i tate host cel l detoxi fication of the chemotherapeutic drug, thereby reducing host cel l toxici tiesb. Inhibi t the potentia l for DNA repair, by topoisomerases , that otherwise might lead to chemotherapy fa i lurec. Potentiate the action of a ni trogen mustard or ni trosourea to bind to (cross -l ink) purine moieties in DNA strandsd. Prevent myelosuppress ion and related blood dyscras ias

Page 237: Pharmacology pre test self assessment and review (14th ed.)

e. Reduce the ri sk of hyperuricemia and i ts main consequences (renal damage, gout) that can occur with a mass ive cel l ki l l

476. The FDA requires manufacturers of an anticancer drug to avoid concomitant adminis tration of certa in SSRI antidepressants (fluoxetine,paroxetine, and sertra l ine) to woman receiving a particular anticancer drug. The drug in question i s mainly used to treat certa in types of breastcancers and i s a lso used to prevent breast cancer recurrence. It commonly causes hot flashes and i s associated with a high incidence of nauseaand vomiting. The SSRIs l i s ted above are s trong inhibi tors of CYP2D6 and so they can interfere with the anticancer drug’s metabol ic activation,which i s required for thei r chemotherapeutic effects . Which drug i s the most l ikely target of interactions with these SSRIs?

a. Bleomycinb. Interferon a lphac. Mercaptopurined. Tamoxi fene. Vinblastine

477. Al lopurinol or the newer related drug febuxostat i s commonly adminis tered before ini tiating chemotherapy of leukemias and other blood-based cancers to prevent hyperuricemia and i ts consequences . It i s a l so important in preventing hyperuricemia in response to chemotherapy ofsome sol id tumors . However, i t may potentiate the host toxici ty of certa in anticancer drugs by inhibi ting thei r metabol ic inactivation anddetoxi fication. With which drug should concomitant use of a l lopurinol or febuxostat be avoided?

a. Bleomycinb. Cisplatinc. Cyclophosphamided. Doxorubicine. Mercaptopurine

478. A 48-year-old patient was in renal fa i lure, but fortunately she received a kidney transplant. We s tart her on cyclosporine to reduce the ri sk ofgraft rejection. What are the most common and worrisome adverse responses associated with this immunosuppressant?

a. Cardiotoxici ty and hepatotoxici tyb. Hepatotoxici ty and nephrotoxici tyc. Hypotens ion and pulmonary fibros isd. Nephrotoxici ty and infection ri ske. Thrombos is and pulmonary embol ism or i schemic s troke

Page 238: Pharmacology pre test self assessment and review (14th ed.)

Cancer and Immune System Pharmacology

Answers

462. The answer is e. (Brunton, pp 1018, 1690-1694; Katzung, pp 645, 958, 989, 1104.) The main uses of methotrexate (MTX) for conditions other thanrespons ive cancers are management of rheumatoid arthri ti s (RA) and psorias is that i s usual ly not respons ive to other drugs . Doses and dosageschedules di ffer from those typica l ly used for cancers .

MTX i s one of many disease-modi fying anti rheumatic drugs (DMARDs), which are a lso ca l led s low-acting anti rheumatic drugs (SAARDs) becausetheir onset of symptom rel ief i s much s lower than tradi tional NSAIDs (sa l icylates and other fi rs t-generation COX-1/-2 inhibi tors , or second-generation/COX-2 inhibi tors , ie, the “coxibs”).

Nonetheless , a l though the onset i s cons idered s low, meaningful symptom rel ief usual ly occurs with as l i ttle as 3 to 4 weeks of therapy—fasterthan most of the other DMARDs. Note that other DMARDs, of which there are many, have no cancer-related uses . Al l the potentia l s ide effects ,adverse responses , and contra indications that apply to us ing MTX for cancer apply to the drug’s use for RA or psorias is .

463. The answer is b. (Brunton, pp 1074-1095, 1673, 1690, 1693-1694, 1820; Katzung, pp 645, 958, 959t, 1104.) This essentia l technique to reduce host cel ltoxici ty in response to methotrexate (MTX) therapy i s often referred to as leucovorin (fol inic acid) rescue. Methotrexate, a fol ic acidanalog/antimetabol i te, can be curative for women with choriocarcinoma and i s a lso useful for non-Hodgkin lymphomas and acute lymphocyticleukemias (ALLs ) in chi ldren. The drug ki l l s respons ive cancer cel l s by inhibi ting dihydrofolate, an enzyme necessary for forming tetrahydrofol icacid (FH4). The FH4, in turn, i s cri ti ca l for eventual synthes is of DNA, RNA, and proteins . Inhibi tion of thymidylate synthes is i s probably the s inglemost important consequence in the overa l l reaction scheme.

Some cancer cel l s are res is tant to MTX because they lack adequate mechanisms for transporting the drug intracel lularly. These include somehead and neck cancers and osteogenic sarcomas . In such cases we need to give very large doses of MTX to establ i sh a high concentration gradientthat essentia l ly “drives” i t into the cel l s . Unfortunately, normal host cel l s depend on folate metabol i sm, they take up MTX wel l , and they wi l l beaffected.

To protect normal cel l s we adminis ter leucovorin (a lso ca l led ci trovorum factor or fol inic acid) right after giving the MTX. It i s taken up by thenormal cel l s , bypasses the block induced by the MTX, and so spares normal cel l metabol i sm. The leucovorin does not spare cancer cel l s : jus t asthey cannot take up MTX wel l , they cannot take up the rescue agent and save themselves from cytotoxici ty.

Leucovorin rescue i s not done “automatica l ly” in every case when MTX is given. When low MTX doses are used leucovorin may be withheld unti land unless blood counts show evidence of MTX-induced bone marrow suppress ion. However, i t i s qui te usual ly given a long with MTX when MTXdoses are very high (as in severe or MTX-res is tant cases ), and host toxici ty i s very probable.

There i s another important and growing use of MTX: as a disease-modi fying or s low-acting anti rheumatic drug (DMARD, SAARD, respectively).Owing to toxici ty (see below) i t had been relegated to second- or thi rd-l ine therapy of severe and/or refractory rheumatoid arthri ti s . Now that wehave a better appreciation of the drug’s toxici ty, and how to recognize i t earl ier and minimize i ts occurrences , i t i s being used much more, earl ieron in therapy, for rheumatoid arthri ti s that i s “moderate” and l ikely to progress rapidly.

The main adverse responses to MTX, regardless of the purpose for which i t i s given, include bone marrow suppress ion, pulmonary damage(infi l trates , fibros is ), s tomati ti s , and les ions elsewhere in the GI tract. High doses can be nephrotoxic (ri sk reduced by mainta ining adequatehydration and a lka l inizing the urine). MTX i s a lso teratogenic.

Reca l l that deferoxamine (a) i s used to treat i ron poisoning (i t i s an i ron chelator). N-acetylcysteine (c) i s mainly used ei ther as a mucolytic(mucus-thinning) drug for certa in pulmonary disorders (eg, COPD) or as an antidote for acetaminophen poisoning. Penici l lamine (d) i s mainly acopper chelator, used for copper poisoning or Wi lson disease. Vi tamin K (e) i s used for deficiency s tates , for combating excess ive effects ofwarfarin, or for managing bleeding disorders in newborns of mothers who have been taking certa in drugs (eg, anticonvulsants such as phenytoin)during pregnancy.

464. The answer is c. (Brunton, pp 1705-1707; Katzung, pp 962-964.) Vincris tine i s one of relatively few cytotoxic anticancer drugs , which does not causebone marrow suppress ion (and a l l the potentia l consequences of that) as i ts main toxici ty. Rather, i t causes neuropathies involving both sensoryand motor nerves . Paresthes ias are a common example of the former (hearing loss can a lso occur); muscle weakness and obtunded reflexes areexamples of the latter. Important note: Vincris tine di ffers from the other two vinca a lka loids , vinblastine and vinorelbine, which do cause bonemarrow suppress ion (and not neuropathies ) as thei r main dose-l imiting toxici ty.

465. The answer is e. (Brunton, pp 1713-1715; Katzung, pp 965-966.) Doxorubicin, an anti tumor antibiotic, i s cardiotoxic, and the ri sk for and severi ty ofcardiomyopathy i s dose-related. (There i s a maximum recommended l i fetime [cumulative] dose for this drug, and i f i t i s exceeded the ri sk ofcardiac damage ri ses s igni ficantly.)

Asparaginase (a), used only for acute lymphocytic leukemia, tends to cause mainly pancreati ti s , hepatic dys function, andal lergic/hypersens i tivi ty reactions . The main organ-speci fic toxici ty of bleomycin (b), a l so an anti tumor antibiotic, i s pulmonary damage thatpresents ini tia l ly usual ly as pneumoniti s . It occurs in about 1 of 10 patients treated with this drug. In some cases the pulmonary damage wi l lprogress to pulmonary fibros is that i s , of course, i rrevers ible.

Cisplatin’s (c) main dose-l imiting toxici ty i s renal damage, which can be prevented somewhat by ensuring that the patient i s adequatelyhydrated and producing adequate amounts of urine. Diuretics may be used as adjuncts . The goal i s to minimize accumulation of the nephrotoxicdrug in the renal tubules and urine. (A related drug, oxa l iplatin, tends to cause periphera l sensory neuropathies and does so in most patientswho receive this drug.)

Cyclophosphamide (d) has no particular organ-speci fic toxici ty. Rather, main mani festations of toxici ty involve rapidly growing cel l s such asthose in the bone marrow, intestina l tract mucosae, and ha i r fol l i cles .

Vincris tine’s (f) major dose-l imiting toxici ty i s periphera l nerve damage: motor, sensory, and in some cases autonomic. It probably ari ses in amanner related to the drug’s anticancer effect: inhibi tion of microtubular function—or, in the case of nerves , neurotubules—as a resul t of drugbinding to tubul in.

466. The answer is c. (Brunton, pp 1713-1715; Katzung, pp 965-966.) Anthracycl ine-type anti tumor antibiotics (doxorubicin, daunorubicin, idarubicin, etc)interca late between and eventual ly bind to DNA base pa i rs . This dis tortion of the DNA chains makes the DNA an unsui table template for RNApolymerase, and ul timately RNA and protein synthes is i s inhibi ted. Dactinomycin i s phase-nonspeci fic.

Anastrozole (a) i s a relatively new aromatase inhibi tor. This ora l agent i s used for postmenopausa l women with early or advanced breast

Page 239: Pharmacology pre test self assessment and review (14th ed.)

cancer. In postmenopausa l women, the major source of estrogen (which supports growth and repl ication of estrogen-dependent tumors) i sadrenal androgens . Those androgens are metabol i zed by aromatase to estrogens . As a resul t, anastrozole depletes estrogens and can arresttumor cel l growth.

Cytarabine (b, a lso ca l led cytos ine arabinos ide) i s a pyrimidine analog (antimetabol i te) that i s metabol i zed to the active moiety, ara-CTP. Theara-CTP becomes incorporated into DNA, with the main ul timate effect being suppress ion of DNA synthes is . It i s highly speci fic for cel l s in S-phase.

Fluorouraci l (d), a l so an antimetabol i te, inhibi ts thymidylate synthetase through i ts active metabol i te, 5-fluoro-2′-dexoyuridine-5′-monophosphate (FdUMP). It i s not phase-speci fic, but i ts activi ty depends on cel l s not being in the G0 s tage.

Tamoxi fen (e) i s used for breast cancers . It blocks estrogen receptors on the breast cancer cel l s (for which the main phys iologic agonis t i sestradiol ). Reca l l that tamoxi fen i s class i fied as a selective estrogen receptor modi fier (SERM). Al though i t blocks estrogen receptors onrespons ive breast cancer cel l s and i s therapeutic for them, i t acts as an estrogen receptor agonis t in the uterus . Thus , one of the main ri sks oftherapy with tamoxi fen i s endometria l hyperplas ia that may lead to endometria l cancer. Because the drug acts as an estrogen receptor agonis t insome ti ssues and an antagonis t in others , ri sk-benefi t ratios must be cons idered careful ly. The beneficia l effects in active breast carcinoma mayoutweigh the ri sks of inducing endometria l disease. However, the preventative use in the absence of breast cancer has a much lower benefi t-to-ri sk ratio.

467. The answer is c. (Brunton, pp 1759-1760, 1177-1179, 1756; Katzung, pp 733, 740.) Fulvestrant i s associated with a much lower ri sk of caus ingendometria l pathology, including cancer. It i s a “pure” estrogen antagonis t. That effect, in breast ti s sue, i s what accounts for the drug’s beneficia leffects in some patients with metastatic, es trogen-supported, breast cancer. In contrast, tamoxi fen and toremifene are class i fied as selectiveestrogen receptor modi fiers (SERMs). Al though they block estrogen receptors in breast ti s sue (just as fulvestrant does), they a lso have estrogenic(agonis t) activi ty in some other ti s sues , notably the uterus . There they can cause endometria l prol i feration, hyperplas ia , and (apparently) anincreased ri sk of endometria l cancer.

Because fulvestrant lacks estrogen agonis t activi ty, i t wi l l not enhance bone minera l i zation nor favorably modi fy cholesterol profi les , as theSERMs tend to do. The SERMs s l ightly increase the ri sk of thromboembol ism. Fulvestrant may too, but i t a l so lacks any abi l i ty to prevent plateletaggregation or thromboembol ism. Hot flashes are fa i rly common with any of these drugs .

468. The answer is c. (Brunton, pp 1732-1734; Craig, p 653; Katzung, pp 966-967.) One of severa l problems with imatinib therapy i s that i t i s a substratefor, and rather powerful inhibi tor of, severa l cytochromes (CYP3A4, 2C9, and 2D6) that are important for the metabol i sm of many other drugs—warfarin, theophyl l ine, and many others—whose actions can be increased excess ively i f dosages are not adjusted accordingly. Conversely,imatinib i s a target of interactions by this mechanism. Phenytoin, carbamazepine, barbi turates , and ri fampin are examples of drugs that caninduce imatinib metabol i sm and reduce the cl inica l response to i t; and such drugs as azole anti fungals and erythromycin can reduce imatinib’sclearance and increase the ri sk of toxici ty.

Because of the i ssue of drug interactions , a high frequency of adverse responses , l imited use (see below), and even cost, imatinib i s genera l lyreserved for use after a tria l of interferons has proven inadequate. The reverse—us ing imatinib fi rs t—usual ly i sn’t done.

Hypotens ion and hypovolemia (b) are not what one would expect with this drug. Rather, we see a rather high incidence of fluid retention thatmay not only affect blood pressure, but a lso cause such other problems as asci tes , pericardia l and pleura l effus ions , and poss ibly pulmonaryedema. Likewise, thrombocytos is (e) i s the oppos i te of what typica l ly occurs : thrombocytopenia and bleeding problems, plus neutropenia and anincreased ri sk of infection are fa i rly common.

Recal l that chronic myelogenous leukemia cel l s do synthes ize an abnormal consti tutively active tyros ine kinase (Bcr-Abl ) that i s involved in(abnormal ) protein phosphorylation. It i s that aberrant tyros ine kinase—not ones found in normal host cel l s—that i s a ffected by the drug and thatconfers selectivi ty for the drug’s actions . Thus , tyros ine kinase inhibi tion does not seem to account for the adverse effects of this drug on hostcel l s .

469. The answer is a. (Brunton, pp 1667-1674; Katzung, pp 951-953.) Gompertzian analys is (a plot of the log of the number of cancer cel l s in a tumor vstime) shows that after a tumor has reached a certa in s i ze, the rate of tumor growth (and “overa l l metabol ic rate”) s lows: lower growth fraction or,s tated di fferently, the longer i t takes for the tumor to double in s i ze. This s lowed growth i s partia l ly due to more cel l s entering the G0 (resting)phase of the cel l cycle, where respons iveness to many chemotherapeutic agents i s lower. (One reason for this i s the sheer s i ze of the tumor asrelated to blood flow and the del ivery of nutrients that the rapidly dividing cel l s need. Reduced nutrient and oxygen del ivery not only reduce cel lrepl ication, but a lso del ivery of the chemotherapeutic agents . Therefore answer b i s incorrect.)

P-glycoprotein activi ty (c) does not necessari ly decrease with time or tumor s i ze. However, even i f i t did, that would predict increasedrespons iveness to most anticancer drugs , because i t i s P-glycoprotein that normal ly pumps drugs out of the cancer cel l . Sel f-repair mechanisms, asby topoisomerase (e), i s not a factor in expla ining reduced vulnerabi l i ty of very large tumors .

470. The answer is b. (Brunton, pp 1205, 1766, 1841; Katzung, pp 738-740.) Flutamide, one of a smal l number of androgen receptor blockers used formanaging prostate cancer, i s used as an adjunct to leuprol ide. Leuprol ide acts l ike gonadotropin-releas ing hormone (GnRH; or luteinizinghormone-releas ing hormone). When leuprol ide therapy i s s tarted, i t s timulates release of inters ti tia l cel l -s timulating hormone from the pi tui tary,thereby increas ing testosterone production and supporting tumor growth. It i s only with continued exposure to leuprol ide that GnRH receptorsbecome desens i ti zed, and the eventual inhibi tion of testosterone production (and, thereby, support of tumor growth) occurs . Flutamide, byblocking androgen receptors , prevents the potentia l worsening of the tumor in the early phase of leuprol ide therapy when testosterone levelsri se. Even when leuprol ide’s pi tui tary-desens i ti zing effects occur, androgens that can support prostate tumor growth wi l l come from the adrenalgland. Their effects , too, are blocked by the flutamide.

471. The answer is a. (Brunton, pp 1677-1681, 1683; Katzung, pp 953-955, 956t.) Mechlorethamine, l ike cyclophosphamide (and carmustine and severa lothers ), i s an a lkylating agent. They are ca l led bi functional a lkylating agents because they can covalently bind to DNA in two places (“nucleophi l i cattack”), thereby forming cross -l inks between two adjacent s trands or between two bases in one s trand. This ul timately dis rupts DNA and RNAsynthes is or may cause s trand breakage. Cyclophosphamide (which can be cons idered the prototype of the a lkylating agents ) i s actua l ly a prodrug—it requires metabol ic activation in order for i ts effects to occur. Cyclophosphamides (and other a lkylating agents ) are cel l cycle-nonspeci fic,a l though their efficacy i s greater when cel l s are not in G0.

Bleomycin, dactinomycin, and doxorubicin are good examples of drugs that interca late in DNA strands (d). Thus , the a l tered DNA no longerserves as an adequately precise template for eventual synthes is of more functional DNA and RNA. They are class i fied as anti tumor antibiotics .

Etopos ide and topotecan are examples of drugs that inhibi t topoisom-erase II (c). The consequence i s inhibi ted abi l i ty of a ffected cel l s torepair DNA strand breaks . This s tops the cel l cycle in G2.

The taxoids (eg, pacl i taxel ) impair mitos is (b), but by s tabi l i zing assembled microtubules rather than by exerting a vinca a lka loid-l ike inhibi tion

Page 240: Pharmacology pre test self assessment and review (14th ed.)

of microtubular assembly.

472. The answer is a. (Brunton, pp 1720-1721; Katzung, pp 967-969.) Asparaginase i s an enzyme that cata lyzes the hydrolys is of plasma asparagine toaspartic acid and ammonia . Major toxici ties from asparaginase are related to antigenici ty (i t i s a foreign protein, and some fata l anaphylacticreactions have occurred), pancreati ti s , and a 50% incidence of some hepatic dys function based on the presence of elevated transaminases . Thedrug, which i s largely G1 phase-speci fic, i s not cytotoxic to cel l s other than leukemic lymphoblasts . Host (and other cel l s ) can synthes ize andreplace asparagine that has been hydrolyzed by the drug; the lymphoblasts cannot, and so they are ki l led.

The responses ci ted are not typica l of azathioprine (b), doxorubicin (c) or other antibiotic anticancer agents , methotrexate (d), or the vincaa lka loids (eg, vincris tine, e).

473. The answer is b. (Brunton, pp 685-686, 1541, 1682t, 1688-1689; Katzung, pp 956-958.) Cisplatin, which i s sometimes class i fied a long with tradi tionala lkylating agents , i s unique among a l l the common anticancer agents in terms of the relative incidence of hearing loss (ototoxici ty) andnephrotoxici ty.

You are correct in associating vincris tine with hearing loss , but nephrotoxici ty i s very rare; in contrast, you may have reca l led that methotrexatecan cause nephrotoxici ty, but i t does not cause hearing loss , and i t i s indicated for a variety of cancers , but not ovarian.

Bleomycin’s (a ) main targeted toxici ty i s the lungs (pulmonary infi l trates , fibros is , etc). Doxorubicin (c), as noted elsewhere, i s cardiotoxic. Thedrug i s mainly used for testicular carcinomas , squamous cel l cancers , and lymphomas. 5-FU (d), a pyrimidine antimetabol i te, i s used for a varietyof sol id tumors . However, periphera l neuri ti s or neuropathy (and, especia l ly, hearing loss ) or renal damage are uncommon; rather, there i s arelatively high incidence of bone marrow suppress ion and ora l and GI mucosa l damage. Pacl i taxel (e) i s a microtubular s tabi l i zing drug (and planta lka loid). It i s cons idered fi rs t-l ine for some patients with advanced ovarian cancer or non-smal l -cel l lung cancers , causes dose-dependent bonemarrow suppress ion and periphera l neuropathy, and a fa i rly high incidence of acute infus ion-related hypersens i tivi ty reactions (probably due tothe vehicle in which the drug i s del ivered).

474. The answer is d. (Brunton, pp 1074-1075, 1745, 1750; Katzung, p 598.) Fi lgrastim, a lso known as granulocyte colony-s timulating factor (GCSF),enhances neutrophi l production. One use, therefore, i s to prevent neutropenia and infection associated with bone marrow depress ion from cancerchemotherapy. (Hint: Look at the generic name, fi lgrastim: granu-locyte stimulating.) The drug lacks antiemetic effects (a ), potentiates thechemotherapeutic actions of no drug (b), and has no effect on doxorubicin-mediated cardiotoxici ty (c) or on chemotherapy-related adverse effectson the gastric mucosa (e).

475. The answer is e. (Brunton, pp 997, 1000; Katzung, pp 653-654.) Hyperuricemia i s associated with many cancers and i s a common outcome of mass ivecel l ki l l s induced by chemotherapeutic drugs , especia l ly with blood-involved (disseminated) cancers such as leukemias . The uric acid i s derivedfrom cel lular purine degradation, eventual ly formed from hypoxanthine and xanthine via xanthine oxidase, the enzyme that i s inhibi ted bya l lopurinol . Reca l l that renal damage (and other damage, such as gout) i s due to uric acid’s poor solubi l i ty in body fluids , especia l ly at low pH.

Al lopurinol has no effect on the P450 system or on cel lular trans i tions from one phase of the cel l cycle to another. There i s no effect on DNAsynthes is or repair, or any di rect cytoprotective effect on myeloid or other ti s sues .

Be sure to remember that the metabol ic detoxi fication of azathioprine and i ts active metabol i te 6-mercaptopurine (i tsel f an anticancer drug)depends in part on xanthine oxidase. Inhibi ting that enzyme with a l lopurinol (or the relatively new related xanthine oxidase inhibi tor,febuxostat), therefore, may increase the ri sk of toxici ty to host cel l s .

476. The answer is d. (Brunton, pp 1177-1180, 1299-1300, 1756-1759; Katzung, pp 731, 740.) Of a l l the drugs l i s ted as poss ible answers , only tamoxi fen, aselective estrogen receptor modi fier (SERM), fi ts the description: i t requires metabol ic activation for anticancer effects (the active metabol i te i sabout 100-times more potent than tamoxi fen i tsel f), i t i s indicated for estrogen-sens i tive breast carcinoma (treatment or prophylaxis aga instrecurrence), i s associated with a high incidence of severe nausea and vomiting, and tends to cause hot flashes in many women (typica l ly adelayed response occurring late in, or a fter, treatment with tamoxi fen). Data suggest that women taking one of the l i s ted SSRIs with tamoxi fen,used for breast cancer prophylaxis , have higher rates of recurrence than those taking other SSRIs , or no antidepressants at a l l .

The interaction does not apply to bleomycin (a), interferons (b), mercaptopurine (c; a l though mercaptopurine i s a cri ti ca l target of certa in druginteractions , discussed elsewhere), or vinca a lka loids (eg, vinblastine, e).

Note: It should not be surpris ing that many women who have cancer, or are at ri sk of recurrences , might benefi t from an antidepressant, andSSRIs genera l ly are the preferred drugs . Fluoxetine, paroxetine, and sertra-l ine interact adversely with tamoxi fen, due to s trong CYP2D6 inhibi tion,not a l l SSRIs do. Ci ta lopram, esci ta lopram, and fluvoxamine are not s trong 2D6 inhibi tors , and so they are genera l ly preferred when concomitantSSRI therapy i s warranted.

477. The answer is e. (Brunton, pp 997, 1701-1702; Katzung, pp 653-654.) Mercaptopurine i s a (thio)purine antimetabol i te that i s metabol ica l lyinactivated (detoxi fied) by xanthine oxidase. This purine degradation pathway in which xanthine oxidase participates not only leads to formationof uric acid, but a lso i s important to reducing host cel l toxici ty to the thiopurines . Thus , concomitant use of a l lopurinol increases the ri sk of hostcel l toxici ty. The same appl ies to the newest approved xanthine oxidase inhibi tor, febuxostat. Note that azathioprine (an inhibi tor of B and Tlymphocyte prol i feration, typica l ly used as an immunosuppressant) i s metabol i zed to mercaptopurine. As a resul t, i ts metabol i sm is a lsoinhibi ted by a l lopurinol .

The metabol i sm of the other drugs l i s ted i s not xanthine oxidase-dependent, and so i s not affected by a l lopurinol .

478. The answer is d. (Brunton, pp 1011-1012, 1821; Katzung, pp 644, 986, 999, 1156t.) Nephrotoxici ty, or at least some cl inica l ly s igni ficant degree ofrenal dys function, occurs in about 8 of 10 patients receiving cyclosporine. It i s typica l ly dose-dependent and, particularly in renal transplantpatients , could be due to ei ther the drug (too much) or to rejection. Infection occurs about as often as renal dys function. Cyclosporine can causehepatotoxici ty (a ), but the incidence i s far lower than that of adverse renal responses or infection. Cardiotoxici ty i s not an i ssue. Blood pressurechanges (c) can occur, but with cyclosporine the change more l ikely involves increased pressure. Cardiac or pulmonary toxici ties andthromboembol ism (e) due to the drug i tsel f are extremely uncommon.

Page 241: Pharmacology pre test self assessment and review (14th ed.)

Toxicology, Bioterrorism, and Chemical Warfare Agents Alcohols , ethylene glycolAntidotes for common drugsBioterrorism or chemica l warfare agentsHeavy meta lsPoisonings of unknown causePoisoning syndromesToxic gases

Questions

479. A patient develops s tatus epi lepticus from an unknown poisoning. Which of the fol lowing i s the most appropriate fi rs t IV drug to give?

a. Carbamazepineb. Lorazepamc. Phenobarbi ta ld. Phenytoine. Va lproic acid

480. A fi refighter who i s trying to extinguish a car fi re has a leak in his protective a i r mask. He develops cyanide poisoning from the combustion ofplastic. As ide from rendering symptomatic, supportive care, which might be adminis tered to combat the cyanide poisoning?

a. Ammonium chlorideb. Deferoxaminec. Dimercaprol (BAL; Bri ti sh anti -lewis i te)d. Mannitole. Pra l idoximef. Sodium thiosul fate

481. A 28-year-old male patient i s transported to the emergency department after a mass ive exposure to carbon tetrachloride (CCl 4), a solvent usedat his work. Which one of the fol lowing best describes a characteris tic of poisoning with this toxicant, or i ts cl inica l management?

a. Causes a s low death due to bone marrow suppress ionb. Causes widespread ATP depletion by uncoupl ing oxidative phosphorylationc. Metabol i zed to free radica ls (trichloro radica ls ) that are di rectly cytotoxic to l iver and kidney cel l sd. N-Acetylcysteine effectively scavenges carbon tetrachloride’s cytotoxic metabol i tee. Toxic consequences reduced or prevented by adminis tration of methylene blue

482. A patient who has been on long-term therapy with a drug develops a lupus-l ike syndrome in response to i t. With which drug i s this response,probably mediated by an autoimmune reaction, most l ikely to occur?

a. Dioxinb. Hydra lazinec. Organic mercuryd. Primaquinee. Tricycl ic antidepressants (eg, amitriptyl ine)

483. A patient presents in the emergency department with a drug overdose. Among other things , the phys ician correctly orders IV infus ion ofsodium bicarbonate to a lka l inize the urine, which increases the toxin’s el imination through pH-dependent inhibi tion of i ts tubular reabsorption,and helps correct the combined metabol ic and respiratory acidos is the toxin caused. Which one of the fol lowing drugs most l ikely caused thetoxici ty?

a. Amphetamineb. Aspirin (acetylsa l icyl i c acid)c. Coca ined. Morphinee. Phencycl idine (PCP)

484. A patient has taken a potentia l ly letha l dose of acetaminophen. The emergency department team begins adminis tering repeated doses of N-acetylcysteine, which can be a l i fesaving antidote in many such cases . What i s the main mechanism by which this antidote exerts i ts beneficia leffects?

a. Alka l inizes the urine to faci l i tate acetaminophen excretionb. Causes metabol ic acidos is to counteract metabol ic a lka los is caused by a toxic acetaminophen metabol i tec. Inhibi ts P450 enzymes , thereby inhibi ting formation of acetaminophen’s toxic metabol i ted. Inhibi ts synthes is of superoxide anion radica l and hydrogen peroxidee. Is ri ch in sul fhydryl (–SH) groups that react with and inactivate the toxic acetaminophen metabol i te, sparing hepatocytes from attack by the

metabol i te

Page 242: Pharmacology pre test self assessment and review (14th ed.)

485. Your patient developed acute poisoning as a resul t of inhal ing cyanide gas in an industria l accident. In addition to providing symptomatic,supportive care, and other appropriate interventions , adminis tering which one of the fol lowing would be most l ikely to be effective as anadjunctive drug, in treating the cyanide poisoning?

a. Ammonium chlorideb. Deferoxaminec. Dimercaprol (BAL; Bri ti sh anti -Lewis i te)d. N-acetylcysteinee. Pra l idoximef. Sodium thiosul fate

486. A 5-year-old boy consumed a l iquid from a conta iner in the fami ly garage. He presents with centra l nervous system (CNS) depress ion,obtunded reflexes , and venti latory depress ion. A blood sample indicates profound metabol ic acidos is and an anion gap. A check of the urinereveals crysta ls that are presumed to be oxa late. What i s the most l ikely cause of the poisoning?

a. A ha logenated hydrocarbon from a can of spray pa intb. An insecticidec. Ethylene glycold. Gasol inee. Pa int thinner

487. A farm worker in Ca l i fornia i s brought emergently, and declared dead on arriva l , to a loca l emergency department. He had been exposed to anagricul tura l toxin. A coworker, who was not poisoned but was nearby right after the time of exposure, sa id, “He just s tarted shaking bad, col lapsed,and immediately s topped breathing. I didn’t see anything else happen. He looked perfectly normal one minute, then he was just dead.” What wasthe most l ikely cause of this man’s death?

a. Chol inesterase inhibi tor insecticide, sprayed by a crop duster a i rplaneb. Kerosene from a barn heater, ingestedc. Long-term and cumulative cutaneous exposure to arsenic, today’s exposure proving rapidly fata ld. Petroleum dis ti l lates used to clean grease off farm machinery, inhalede. Strychnine, ingested

Questions 488 to 489

Here i s an excerpt from newspaper article enti tled Weed Users Chase High all the Way to the Hospital. Use i t to answer the next two questions , andnote that the “weed” referred to does not include mari juana or any other plants that conta in tetrahydrocannabinol or an opioid.

“Teenagers seeking a ha l lucinogenic high from the seeds of a poisonous weed that now is in bloom are landing in hospi ta ls across the country,pol ice and heal th officia ls say… “Lunatic, crazy kids ,” says Dodge County, Wis ., Sheri ff Todd Nehls , whose deputies picked up three ha l lucinatingteenagers in October….

“Poison centers las t year recorded 975 incidents involving plants such as [the] weed…” according to the American Association of Poison ControlCenters ’ annual report.

“[The weed has]… pods that conta in seeds that when eaten or brewed in a tea can cause severe ha l lucinations [del i rium] and other reactions ,including dry mouth, overheating [fever], agi tation, [and] urinary retention… Some cannot urinate and need to have a catheter inserted… [Severe]overdoses can lead to seizures , coma, or death… Most people hospi ta l i zed after eating [the] weed have ha l lucinations that make them so erraticthey are a danger to themselves…”

a. Acetylchol ineb. Amphetaminec. Bel ladonna a lka loidsd. Bethanechole. Coca inef. Cyanideg. Fluoxetineh. Neostigminei . Phencycl idine (PCP)j. Physostigminek. Strychnine

488. Which drug l i s ted above causes s igns and symptoms that are most s imi lar to—indeed, vi rtua l ly identica l to—those described in the scenario?

489. Which drug would be most rational and most effective for treating the overdose, assuming the overdose i s severe enough to warrant drugtherapy?

490. A mother ca l l s to report that her 6-year-old chi ld appears to have swal lowed a large amount of an over-the-counter s leep a id about 5 hoursago. The product conta ined only one active drug, and knowing your drugs you suspect the poisoning i s due to diphenhydramine. Assuming yourreasoned guess about the cause of poisoning was correct, which of the fol lowing s igns or symptoms would you expect to find, upon phys ica l exam,to confi rm your hunch?

a. Fever; clear lungs ; absence of bowel sounds ; urinary retention, dry, flushed skin; mydrias is and photophobia ; bizarre behaviorb. Bradycardia and profuse diarrheac. Mios is with l i ttle/no papi l lary response to bright l ights ; spontaneous micturi tion; lack of response to pa inful s timul id. Hypothermia; bounding pulse; hypertens ion

Page 243: Pharmacology pre test self assessment and review (14th ed.)

e. Skeleta l muscle weakness or para lys is ; profound hypermoti l i ty of gut and bladder smooth muscle; bronchospasm

491. A 3-year-old gi rl ingests 30 tablets of aspi rin, 325 mg each. We’ve gotten her to the emergency department within 30 minutes of the poisoning.Which one of the fol lowing drugs would be the most rational and hopeful ly effective to adminis ter as part of the ini tia l treatment plan for whatotherwise could have a fata l outcome?

a. Activated charcoalb. Deferoxaminec. Dimercaprold. N-Acetylcysteinee. Penici l lamine

492. A 50-year-old man has been consuming large amounts of ethanol on an a lmost da i ly bas is for many years . One day, unable to find anyethanol , he ingests a large amount of methanol (wood a lcohol ) that he had bought for his camp lantern. Which of the fol lowing i s the most l ikelyconsequence of his methanol poisoning?

a. Atrioventricular conduction defect (block)b. Bl indnessc. Bronchospasmd. Del i rium tremense. Metabol ic a lka los is

493. A 15-year-old boy attempts suicide with a l iquid that he found in his parents ’ greenhouse. His dad used i t to get rid of “varmints” around theyard. The toxin causes intense abdominal pa in, skeleta l muscle cramps, projecti le vomiting, and severe diarrhea that leads to fluid andelectrolyte imbalances , hypotens ion, and di ffi cul ty swal lowing. On examination he i s found to be volume depleted and i s showing s igns of areduced level of consciousness . His breath smel ls “meta l l i c.” Which of the fol lowing probably accounts for these symptoms?

a. Arsenicb. Cadmiumc. Irond. Leade. Zinc

494. A 22-year-old i s brought to the emergency department by a friend. They had been at a bar for about an hour, and then the patient suddenlybecame drowsy but was s ti l l conscious . She fel l and cut her head, and has l i ttle di ffi cul ty feel ing the pa in from the trauma. Her venti latory rateand depth are depressed, but not to a worrisome degree. Her patel lar reflexes are blunted and she i s ataxic. She responds s lowly to questions butis unable to reca l l anything that happened after arriving at the bar and s ipping her fi rs t (and las t) adul t beverage. Her friend s tated that she hadonly one Cosmopol i tan and hadn’t been drinking before they went out. With what was this patient’s drink most l ikely “spiked?”

a. A barbi turateb. A benzodiazepinec. An opioidd. Chlora l hydratee. Coca inef. Pure (gra in) a lcohol

495. Let’s assume the profoundly CNS-depressed patient in the previous question indeed was overdosed with a benzodiazepine. Which drug i smost l ikely to be effective, indeed would be a preferred pharmacologic antidote, i f that were the case?

a. Amphetamineb. Flumazeni lc. Methylphenidated. Nal trexonee. Physostigmine

496. Recent occupational hea l th s tudies in severa l heavi ly populated urban areas have revealed an astonishingly large number of homes thathave lead-based pa int and chi ldren l iving in them. However, a number of envi ronmenta l poisons that could lead to acute or chronic poisoninghave a lso been found there. Which s igns and symptoms would be cons is tent with chronic exposure to toxic levels of inorganic lead?

a. Anorexia and weight loss ; weakness , especia l ly of extensor muscles (eg, wris t drop); recurrent abdominal pa inb. Gingivi ti s , discolored gums, loosened teeth, or s tomati ti s ; tremor of the extremities ; swol len parotid or other sa l ivary glandsc. Ha l lucinations , insomnia , headache, genera l i zed CNS i rri tabi l i tyd. Hyperventi lation in response to metabol ic acidos is ; hypotens ion; abdominal pa in, diarrhea, brown or bloody vomitus ; pa l lor or cyanos ise. Severe, watery diarrhea; garl i cky or meta l l i c breath; encephalopathy, hypovolemia and hypotens ion

497. Lab tests conducted by the loca l hea l th department are pos i tive for chronic lead exposure in a chi ld. Lead levels are s igni ficantly elevated, butsymptoms fortunately are mi ld and not at a l l imminently l i fe-threatening. What i s the most appropriate antidote for reducing the chi ld’s body loadof lead?

a. Ca-Na 2-EDTA

b. Deferoxaminec. Dimercaprol

Page 244: Pharmacology pre test self assessment and review (14th ed.)

d. N-Acetylcysteinee. Penici l laminef. Succimer

498. Not long ago, severa l patients (and a rather shady heal th care “provider” who i s now incarcerated) seeking “rel ief” from facia l wrinkles nearlydied because they received injections of botul inum toxin that was improperly obta ined and inadequately di luted. Which i s a correct characteris tic,finding, or mechanism associated with this toxin?

a. Complete fa i lure of a l l chol inergic neurotransmiss ionb. Favorable response to adminis tration of pra l idoximec. Impairment of parasympathetic, but not sympathetic, nervous system activationd. Mass ive overstimulation of a l l s tructures having muscarinic chol inergic receptorse. Selective para lys is of skeleta l muscle

499. A terroris t drops a via l of “nerve gas” into a crowded subway at rush hour. The patients are brought to the nearest emergency centers and aregiven atropine. Which effect of the nerve gas wi l l pers is t a fter giving the atropine?

a. Bradycardiab. Bronchospasmc. Excess ive lacrimal , mucus , sweat, and sa l ivary secretionsd. GI hypermoti l i ty, fluid and electrolyte loss from profuse diarrheae. Skeleta l muscle hyperfunction or para lys is

500. A neighbor ca l l s you for advice and help. She knows you are not a veterinary s tudent, but s ince you’re going to be a phys ician soon she figuresyou know everything. Her pet puppy, Pookie, a Pekingese, has eaten a large amount of rodent (rat and mouse) poison. She’s panicked and pleadsto know i f the precious pooch i s poisoned. The rodenticide indeed conta ins a powerful poison intended to ki l l rats and mice, but i t can do thesame to any other animals that have ingested i t too, including humans . What i s the most l ikely toxic ingredient in these rodenticides , andprobably wi l l cause poor Pookie pronounced problems?

a. Amphetamine-coca ine combinationb. Morphinec. Succinylchol ined. Vecuronium (or a s imi lar neuromuscular blocker)e. Warfarin

Page 245: Pharmacology pre test self assessment and review (14th ed.)

Toxicology, Bioterrorism, and Chemical Warfare Agents

Answers

479. The answer is b. (Brunton et al., pp 452-455; Katzung, pp 394, 397-398, 985.) Intravenous ly adminis tered lorazepam is the ini tia l drug of choice fortreatment of s tatus epi lepticus , especia l ly when the cause of this l i fe-threatening seizure (and a speci fic antidote) are unknown. Lorazepam, l ikethe benzodiazepines in genera l , increases the apparent affini ty of the inhibi tory neurotransmitter GABA for binding s i tes on bra in cel lmembranes . The effects of lorazepam (or diazepam) are short-las ting. Immediately after giving i t ei ther phenytoin or fos -phenytoin should begiven to provide longer seizure suppress ion and “coverage” because the effects of lorazepam wear off in a comparatively short time. Other drugsthat can be used for s tatus epi lepticus include phenobarbi ta l (not a drug of choice) and (paradoxica l ly) l idoca ine for refractory seizures (mainly adrug of las t resort). None of the other drugs l i s ted in the question are appropriate for s tatus epi lepticus , despi te thei r widespread use for ora ltherapy of seizure disorders long term.

480. The answer is f. (Brunton, p 86t; Katzung, p 1036.) Cyanide reacts with Fe(II I) in mitochondria l cytochrome oxidase, inhibi ting oxidativephosphorylation. The shi ft in metabol i sm from aerobic to glycolytic soon leads to not only ATP depletion, but a lso severe lactic acidos is .

Cyanide normal ly reacts with endogenous sul fur-conta ining compounds , mainly thiosul fate, and under cata lys is by rhodanese forms relativelyless toxic thiocyanate that i s formed and excreted in the urine. In cyanide poisoning, endogenous sul fur-conta ining substrate s tores are quicklydepleted. We manage this , then, by IV infus ion of an aqueous sodium thiosul fate solution. (This i s the same approach we use for “therapeuticcyanide poisoning,” as can arise when ni tropruss ide doses are too great.)

Ammonium chloride would be ineffective, and may actua l ly make matters worse by exacerbating metabol ic acidos is . Deferoxamine, an i ronchelator, would be of no benefi t in terms of s igns , symptoms, or thei r causes or consequences . Dimercaprol i s a heavy meta l (mainly lead)chelator that would be of no benefi t. Mannitol and osmotic diuretic that i s sometimes used to hasten the renal excretion of some toxins (via“forced diures is”) would not a l leviate or shorten s igns and symptoms. Pra l idoxime i s a chol inesterase reactivator that i s effective only forpoisoning with organophosphate insecticides , nerve gases (sarin, soman), or other drugs that cause profound and largely permanent inactivationof acetylchol inesterase.

481. The answer is c. (Katzung, p 1005.) The primary toxic metabol i te of carbon tetrachloride i s a trichloro radica l , and the l iver and kidneys are theprimary targets . Al though a l l the biochemica l mechanisms involved in CCl 4 poisoning are not known, i t seems clear that uncoupl ing ofmitochondria l oxidative phosphorylation i s not involved (b), as i s clearly the case with, for example, cyanide. There i s no antidote for poisoningwith CCl 4, and so only symptomatic and supportive care are l ikely to help reduce the ri sk of death. Acetaminophen (d) i s , of course, metabol i zed tohepatotoxic radica ls , and prompt and proper adminis tration of N-acetylcysteine can reduce the ri sks and consequences of hepatotoxici ty. However,there i s no evidence that this antidote for acetaminophen has any beneficia l effects on CCl 4 toxici ty. Another organic solvent, benzene, i s theagent that typica l ly causes bone marrow suppress ion (a) and, with chronic exposure, certa in leukemias .

Fina l ly, methylene blue i s of no va lue in managing CCl 4 poisoning. Reca l l , however, that methylene blue does have an important role inmanaging toxici ty from a variety of drugs that tend to oxidize hemoglobin to methemoglobin: ni tri tes (eg, amyl ni tri te), certa in ani l ine dyes ,benzocaine, ni troglycerin, primaquine, and certa in sul fonamides .

482. The answer is b. (Brunton, pp 787-797; Katzung, pp 181-190, 220.) Hydra lazine, mainly used as an afterload-reducing drug for patients with severeheart fa i lure and/or hypertens ion, induces autoantibodies aga inst myeloperoxidase, and more so than many other drugs causes a lupus-l ikesyndrome in some patients . Isoniazid, an element in most therapies of tuberculos is , does the same. The lupus-l ike syndrome from proca in-amide, a now seldom used antiarrhythmic drug, i s a l so an autoimmune phenomenon, but in this case the subcel lular mechanism involvesautoantibodies di rected aga inst DNA. Methyldopa, an antihypertens ive drug, a lso causes an autoimmune-mediated syndrome in some patients ,but in this case the more typica l consequence i s hemolytic anemia, not the lupus-l ike problems.

Al l the other drugs l i s ted participate in important and often serious toxic reactions , but the mechanisms don’t involve an autoimmuneresponse. Dioxin (a), which may be carcinogenic with long-term exposure, interacts with a cel lular aryl hydrocarbon receptor and induces thecytochrome P450 system. Organic mercury compounds (c; eg, methyl or other a lkyl compounds) are di rect neurotoxins that additional ly cause suchresponses as renal fa i lure, muscle tremors , psychos is , and dementia . Primaquine (d), an antimalaria l , oxidizes hemoglobin to methemoglobin.Secondary consequences of this include cyanos is , nausea, cardiac s timulation, and fina l ly convuls ions that precede death. The toxici ty of tricycl icantidepressants (e) i s multi faceted, and includes prolongation of the QRS complex (ie, prolonged cardiac action potentia ls resembl ing thosecaused by quinidine, a sodium channel -blocker), a host of other ventricular arrhythmias , and s trong antimuscarinic (atropine-l ike) effects on theheart and elsewhere.

483. The answer is b. (Katzung, pp 640, 1034-1035.) Alka l inizing the urine interferes with the renal tubular reabsorption of organic acids (such asaspirin [sa l icylate] and phenobarbi ta l ) by increas ing the ionized form of the drug in the urine (per the Henderson-Hasselbach equation). Thisincreases thei r net renal excretion. Conversely, excretion of organic bases (such as amphetamine, coca ine, phencycl idine, and morphine) would bereduced by a lka l inizing the urine.

Note that other consequences of severe aspi rin (sa l icylate) toxici ty are both metabol ic and respiratory acidos is . So in addition to enhancingurinary excretion of sa l icylate, the adminis tration of sodium bicarbonate a lso tends to counteract the fa l l of blood pH. See the answer explanationfor Question 329 (pp 369-373) for more information on the sequence of events that occur with a potentia l ly letha l overdose of aspi rin.

484. The answer is e. (Brunton, pp 86t, 75-76, 982-984; Katzung, p 1032.) N-acetylcysteine i s used and often effective for acetaminophen poisoningbecause i t i s a sul fhydryl -rich drug that, i f given soon enough and properly enough, can prevent hepatic necros is . At safe blood levels , the majorpathways of acetaminophen el imination involve glucuronidation and sul fation. When these pathways are overwhelmed, as occurs withacetaminophen poisoning, a cytochrome P450-dependent pathway attempts to handle the metabol ic load. The active toxic metabol i te, N-acetyl -benzoquinoneimine, i s formed in levels that exceed the abi l i ty of intrins ic sul fhydryl compounds to inactivate i t. So long as ample hepatocytestores of glutathione (a -SH compound) are ava i lable, cytotoxici ty wi l l not occur. However, severe poisoning depletes -SH s tores , and so thehepatotoxic metabol i te attacks key cel lular macromolecules . That leads to hepatic necros is .

N-acetylcysteine acts as a substi tute for endogenous glutathione to react with the toxic metabol i te, thereby sparing -SH groups on keyhepatocyte macromolecules .

Alka l inization of the urine i s of no benefi t with acetaminophen poisoning, as i t can be with severe sa l icylate poisoning (because ra is ing urine

Page 246: Pharmacology pre test self assessment and review (14th ed.)

pH reduces tubular reabsorption of sa l icylate and increases i ts excretion). Moreover, N-acetylcysteine does not di rectly change urine pH.Superoxide anion radica l , or hydrogen peroxide, i s not di rectly involved in the cytotoxici ty (d).

See the answer to Question 317, on pages 362-363, for a summary of the “s tages” of acetaminophen poisoning.

485. The answer is f. (Brunton, p 86t; Katzung, p 1038.) Whether cyanide poisoning occurs from leakage of gas , the combustion of plastics ,ni tropruss ide overdoses , or other causes , management includes many common elements . Cyanide reacts with Fe(II I) in mitochondria l cytochromeoxidase, inhibi ting oxidative phosphorylation. The shi ft in metabol i sm from aerobic metabol i sm to glycolys is soon leads not only to ATPdepletion, but a lso to severe lactic acidos is as the glycolytic end-product accumulates .

We manage cyanide poisoning first by deal ing with the high reactivi ty of CN– with Fe(II) in hemoglobin and the subsequent formation of Fe(II I)hemoglobin. We do that by adminis tering sodium ni tri te (IV) to regenerate active cytochromes and oxidize hemoglobin to the more cyanide-reactive methemoglobin (forming cyanmethemoglobin). (Amyl ni trate can be adminis tered by inhalation unti l venous access i s es tabl i shed andsodium ni tri te and sodium thiosul fate can be given). Once the ni tri te i s on board we adminis ter sodium thiosul fate IV. Cyanide normal ly reactswith endogenous thiosul fate, and the hepatic enzyme rhodanese cata lyzes the formation of relatively less toxic and more eas i ly excretedthiocyanate. In cyanide poisoning endogenous thiosul fate s tores are quickly depleted, so this provides the bas is for infus ing sodium thiosul fate(f, the correct answer) to lower cyanmethemoglobin levels . In cases of severe methemoglobinemia, we can a lso give methylene blueintravenous ly. (Note that large doses of inorganic ni tri tes can be intrins ica l ly toxic as a resul t of methemoglobin formation. However, in thecontext of cyanide poisoning we can capi ta l i ze on the reactivi ty of otherwise toxic inorganic ni tri te, in conjunction with thiosul fate adminis tration,to help treat an otherwise fata l toxic scenario.)

Ammonium chloride (a) would be ineffective and may actua l ly make matters worse by exacerbating metabol ic acidos is . Deferoxamine (b), ani ron chelator, would be of no benefi t, nor would dimercaprol (c), a heavy meta l (mainly lead) chelator. N-acetylcysteine (d), routinely used as anantidote for acetaminophen poisoning, would not a l leviate or shorten s igns and symptoms in cyanide poisoning. Pra l idoxime (e) i s achol inesterase “reactivator” that i s used for poisoning with organophosphate insecticides , nerve gases (sarin, soman), or other drugs that causeprofound and “i rrevers ible” inactivation of acetylchol inesterase.

486. The answer is c. (Brunton, p 86t; Katzung, p 1037.) The tip-off to the correct answer should come from the metabol ic acidos is , the anion gap [=(Na +) – (HCO3

– + Cl –)], and the oxa late crysta ls in the urine. (We’ve omitted serum K+ concentration from the equation, as i s common practice,because i t i s normal ly so low that i t contributes l i ttle to the ca lculated ion gap.) Ethylene glycol (ordinary anti freeze) forms, among othermetabol i tes , glycol ic, hippuric, and oxa l ic acids . It i s the latter that tends to crysta l l i ze in the renal tubules , which may lead to acute renal fa i lure.Ethylene glycol i s bas ica l ly an a lcohol , and as with an overdose of ethanol can cause CNS depress ion and an overa l l s tate of inebriation. None ofthe other agents l i s ted tend to cause a s imi lar poisoning syndrome.

Note: The s igns and symptoms of methanol poisoning reflect, in some ways (eg, CNS depress ion, s tupor) those of ethylene glycol or ethanol .However, methanol ’s main toxici ty ari ses from i ts metabol i sm to formic acid, the consequences of which can lead to permanent bl indness .Whether the poisoning i s from ethylene glycol or methanol , treatment includes adminis tration of ethanol ; i t wi l l be preferentia l ly metabol i zed bya lcohol dehydrogenase, reducing the metabol i sm of the toxin to i ts cytotoxic products .

Fina l ly, the majori ty of hospi ta ls ignore the contribution of K+ in the anion gap “equation,” which then s impl i fies to Na + – (HCO3– + Cl –).

487. The answer is e. (Katzung, p 366t.) Strychnine i s a convulsant, and death from exposure i s just as described in the question. The mechanisminvolves s trychnine’s abi l i ty to antagonize the neuronal -inhibi tory effects of glycine, which i s an important neurotransmitter, particularly in thespina l cord. Those spina l cord receptors are sometimes ca l led s trychnine-sens i tive receptors . They are coupled to a chloride channel , and in thepresence of s trychnine the motor neurons become hyperpolarized. The ul timate effect i s a dis inhibi tion of (primari ly) skeleta l motor neurons ; thatrapidly leads to genera l i zed convuls ions (“he just s tarted shaking bad”) and soon thereafter venti latory fa i lure and death (s topped breathing,didn’t see anything else happening [to him]). There i s no antidote.

Chol inesterase inhibi tors (a ) are the most often used insecticides used in agricul ture. They can cause serious harm, and sometimes death, tohumans exposed to them. However, and depending on which inhibi tor i s involved, and depending on the exposure route (ora l , cutaneous ,pulmonary) the time-course leading to death can be up to severa l (or many) minutes or hours . Exposure i s accompanied, and death i s preceded, bya l l the expected skeleta l muscle-s timulating (NM) and periphera l muscarinic activating effects (heart, gut, a i rways , exocrine secretions) that arecons is tent with what amounts to periphera l “ACh excess .” Depending on the causative agent, CNS/spina l cord mani festations may occur and beprominent too.

As a genera l i zation, kerosene (b) and other petroleum dis ti l lates (d; found in many solvents/cleaners/degreasers , including those forhousehold use) tend to affect multiple organ systems, including: the lungs (pulmonary compl ications are the most common and include a lveolarhemorrhage, a lveolar edema and inflammation, and aspiration that contributes further to pneumoniti s ); heart (cardiomyopathy from chronicexposure, catecholamine-enhanced arrhythmias mainly with acute poisoning); bra in (medul lary s timulation, then depress ion; remember thatmost of these petroleum-based products are highly l ipid soluble and enter the CNS readi ly); gut (typica l ly a pa inful burning sensation andfrequent vomiting i f the toxin i s ingested); l i ver (chronic hepatotoxici ty with centrolobular necros is , usual ly caused by free radica l -mediatedperoxidation of membrane l ipids ); and kidneys (renal tubular necros is and metabol ic acidos is ). Long-term exposure can a lso cause aplasticanemia, myelomas , and leukemias (mainly acute myelogenous).

Arsenic toxici ty (c) and i ts management are described in the answer to Question 493. The cl inica l presentation i s much di fferent than thesudden, convuls ive death described here. You should a lso reca l l that arsenic, too, i s widely used as a pesticide and for other pest- and plant-disease control appl ications .

Note: Before we appreciated the extreme toxici ty of s trychnine, many decades ago, the drug was used “therapeutica l ly,” for humans , as astimulant, a laxative, and a remedy for a host of other common but ordinari ly innocuous GI maladies . Not any more, of course. However, thisdeadly poison i s s ti l l readi ly ava i lable in pesticides (mainly those used to ki l l pesky bi rds ). According to a quick web search, in Ca l i fornia a lonemore than ha l f a ton of s trychnine was used agricul tura l ly a mere 2 to 3 years ago.

488. The answer is c. (Brunton, pp 225-226; Katzung, p 1034.) The weed in question i s Jimson Weed, a lso known as s tinkweed, locoweed, and a fewother names. The s igns and symptoms described in the article are among the class ic ones associated with severe atropine (or, in genera l ,antimuscarinic drug) poisoning—the origina l pharmacologic source being a lka loids i solated from various species of Belladonna. No other drugl i s ted causes responses that even come close to these in terms of the periphera l autonomic responses described above.

489. The answer is k. (Brunton, pp 86t, 239, 242; Katzung, p 1034.) “Atropine poisoning,” more genera l ly ca l led the anticholinergic syndrome, can be causedby a host of drugs and by the plants noted in the question. It i s treated symptomatica l ly and supportively and with adminis tration ofphysostigmine. Physostigmine i s bas ica l ly the only cl inica l ly useful AChE inhibi tor that gets into the bra in, a major target of

Page 247: Pharmacology pre test self assessment and review (14th ed.)

atropine/antimuscarinic poisoning. That i s because i t lacks the quaternary s tructure that nearly a l l the other common a l ternatives possess . Sincei t i s not charged, i t can cross the blood-bra in barrier readi ly to reverse the CNS consequences of the poisoning—not just those in the periphery,where the quaternary chol inesterase inhibi tors work. Of course, as i s true for a l l the other chol inesterase inhibi tors , i t works wel l in the peripherytoo, intens i fying the effects of ACh on both muscarinic and skeleta l muscle nicotinic (NM) receptors and competi tively overcoming the receptorblockade.

Al ternatives such as neostigmine, pyridostigmine, and others wi l l combat periphera l effects of atropine poisoning, just as physostigmine wi l l .Unfortunately, some of the CNS mani festations (eg, severe fever, leading to seizures ) contribute greatly to the morbidi ty and morta l i ty associatedwith high doses of antimuscarinics , and the quaternary agents s imply wi l l not combat them in the CNS.

You won’t encounter too many patients overdosed on atropine i tsel f, because atropine i s seldom used. But you’l l see many people poisonedwith older antihis tamines (eg, diphenhydramine); some of the centra l ly acting antimuscarinics that are used for parkinsonism (eg, benztropineand trihexyphenidyl ); scopolamine (used for motion s ickness ); most of the older phenothiazine antipsychotics (eg, chlorpromazine); and most ofthe older tricycl ic antidepressants such as amitriptyl ine and imipramine. Owing to the often s trong antimuscarinic s ide effects of these drugs ,treating overdoses of most of them probably wi l l involve managing what amounts to “atropine poisoning”—and many other problems too.

490. The answer is a. (Brunton, pp 225-235, 918-926, 1314, 1815; Katzung, pp 123-125, 1034.) Most OTC s leep a ids conta in a fi rs t-generation antihis tamine(sedating agent, a lmost a lways an ethanolamine, usual ly diphenhydramine or the very s imi lar drug, doxylamine). The preponderant s igns andsymptoms of toxici ty ari se not from any his tamine receptor-blocking activi ty, but from intense antimuscarinic (atropine-l ike) effects , plus dose-dependent CNS depress ion that ul timately (and early on, in chi ldren) can lead to seizures . The s igns and symptoms of this “antichol inergicsyndrome” include many, i f not a l l , that you wi l l see in “atropine poisoning.” As ide from symptomatic and supportive care, including the use oftradi tional drugs for s tatus epi lepticus , physostigmine (the nonquaternary, centra l ly and periphera l ly acting acetylchol inesterase inhibi tor) maybe l i fesaving. It wi l l certa inly help reverse many of the centra l and periphera l s igns and symptoms of the overdose.

491. The answer is a. (Brunton, pp 84-85; Katzung, pp 640, 1034-1035.) Activated charcoal , a fine, black, powder with a high adsorptive capaci ty, i scons idered to be a va luable agent in the treatment of many kinds of ora l drug poisonings—primari ly i f adminis tered early on and then removed bygastric lavage. Drugs that are wel l adsorbed by activated charcoal include primaquine, propoxyphene, dextroamphetamine, chlorpheniramine,phenobarbi ta l , carbamazepine, digoxin, and aspirin. Minera l acids , a lka l i s , tolbutamide, and other drugs that are insoluble in acidic aqueoussolution are not wel l adsorbed.

Deferoxamine (b), dimercaprol (c; a l so known as BAL, or Bri ti sh anti -Lewis i te), and penici l lamine (e) are polyva lent cation chelators (i ron, lead,copper) and play no role in managing aspi rin (sa l icylate) poisoning, nor poisoning with any substances other than the meta ls they chelate. N-Acetylcysteine (d) i s the preferred antidote for acetaminophen overdoses (Question 333).

Notes : the term adsorb means “to bind”; i t i s decidedly di fferent from the more common pharmacologic/therapeutic term absorb.

492. The answer is b. (Brunton, pp 86t, 631-632; Katzung, p 1037.) Methanol i s metabol i zed by the same enzymes that metabol i ze ethanol , but theproducts are di fferent: formaldehyde and formic acid in the case of methanol . Headache, vertigo, vomiting, abdominal pa in, dyspnea, and blurredvis ion can occur from accumulation of these metabol ic intermediates . However, the most dangerous (or at least permanently disabl ing)consequence in severe cases i s hyperemia of the optic disc, which can lead to bl indness . The rationale for adminis tering ethanol to treatmethanol poisoning i s fa i rly s imple. Ethanol has a high affini ty for a lcohol and a ldehyde dehydrogenases and competes as a substrate for thoseenzymes , reducing metabol i sm of methanol to i ts more toxic products . Important adjunctive treatments include hemodia lys is to enhance removalof methanol and i ts products ; and adminis tration of systemic a lka l inizing sa l ts (eg, sodium bicarbonate) to counteract metabol ic acidos is .Adminis tration of systemic acidi fying substances such as ascorbic acid would aggravate the condition.

493. The answer is a. (Brunton, pp 1867-1871; Katzung, pp 1014t, 1017-1019.) Arsenic i s a consti tuent of fungicides , herbicides , and pesticides . Symptomsof acute toxici ty include tightness in the throat, di ffi cul ty in swal lowing, and s tomach pa ins . Projecti le vomiting and severe diarrhea can lead tohypovolemic shock, s igni ficant electrolyte derangements , and death. Chronic poisoning may cause periphera l neuri ti s , anemia, skin keratos is , andcapi l lary di lation leading to hypotens ion. Dimercaprol (Bri ti sh anti -Lewis i te [BAL]) i s the main antidote used for arsenic poisoning.

Note: If dimercaprol i s a l so known as anti-Lewis i te, what i s Lewis i te? It i s an obsolete organic arsenica l (As -conta ining substance, speci fica l lychlorovinyl dichloroars ine—don’t memorize that!) developed as a chemica l warfare agent in World War II . Arsenic reacts and forms chelates withsul fhydryl moieties on various substances , including many enzymes involved in essentia l metabol ic reactions . It i s a ves icant (causes bl i s tering)that targets many body ti ssues , but exerts i ts main toxic effects in the lungs when inhaled. Dimercaprol i s a relatively s imple molecule with twosul fhydryl groups in i ts s tructure. It reacts with the Lewis i te, chelating i t and thereby reducing attack on and inactivation of cel lular –SH-richenzymes.

494. The answer is b. (Brunton, pp 458-468; Katzung, pp 373-387, 569t, 575.) Arguably the most important tip-off in this presentation i s the antegradeamnes ia , which (among other things ) i s rather uniquely associated with benzodiazepines . The most l ikely benzodiazepine used in this scenariowas rohypnol (fluni trazepam, better known as “roofies” on the s treet and by those who use i t as a date-rape drug).

Unless this patient were a very atypica l responder, i t i s unl ikely that any of the other CNS depressants—a barbi turate, an opioid, chlora l hydrate—would cause the same responses . She’s had one drink yet s ti l l feels pa in from her head gash. She apparently hasn’t had enough a lcohol to beso obtunded that she doesn’t feel pa in, and a barbi turate i s l ikely to enhance the sensation of pa in (hypera lges ic effect). Chlora l hydrate i s s ti l lused medica l ly, mainly as a sedative for chi ldren, a l though i ts use i s decl ining dramatica l ly. It indeed causes powerful CNS depressants whenused in combination with just a l i ttle a lcohol : chlora l hydrate has been known for years as “knock-out drops” and the combination with a lcoholhas been ca l led a “Mickey Finn.” However, chlora l hydrate i s not nearly as readi ly ava i lable as the i l l i ci t benzodiazepines ; i t does not causeantegrade amnes ia (important to the perpetrator, because he or she anticipates no reca l l of what happened by the victim), but i t i s not readi lyava i lable, and i t s imply doesn’t have the “reputation” as a preferred date-rape drug among those who use such drugs .

495. The answer is b. (Brunton, pp 83t, 86t, 468-469; Katzung, pp 379, 381-382.) Flumazeni l i s the speci fic benzodiazepine receptor antagonis t, and i t i sboth speci fic and a lmost a lways effective for excess ive effects of any benzodiazepine. It’s tempting to use a CNS s timulant (a , amphetamines ; or c,methylphenidate) to overcome the effects of any CNS depressant. It may help, but more l ikely the outcome of giving a s timulant i s excess ive effectsmani fest as seizures and unwanted cardiovascular s timulatory effects . Nal trexone, l ike na loxone, speci fica l ly blocks opioid receptors and wi l l beof no benefi t to this poisoned patient. Some benzodiazepines do cause anti -muscarinic effects , but weakly. For that reason, and others ,physostigmine (or any other AChE inhibi tor) would not at a l l an effective or problem-free drug to give.

496. The answer is a. (Brunton, pp 1861-1864; Katzung, pp 1013-1017.) The presentation of chronic lead exposure, as from being exposed to (or eveneating) older lead-based pa ints , di ffers from the typica l presentation of acute organic lead poisoning (answer c), which usual ly ari ses from

Page 248: Pharmacology pre test self assessment and review (14th ed.)

sni ffing leaded gasol ine (and just about a l l gasol ines nowadays are organic lead-enriched). Answer b, with the predominant gingiva l/head/necks igns and symptoms, i s typica l of chronic or acute mercury intoxication. Answer d, with the hyperventi lation, GI dis turbances (including discoloredvomitus ) and pa l lor, i s what you are l ikely to encounter in acute i ron poisoning (as from a consuming ferrous sul fate supplements in large doses).A characteris tic breath (garl i cky or meta l l i c), profuse diarrhea, encephalopathy, and hypotens ion (answer e) are typica l of acute inorganic arsenicpoisoning. Knowing more about the patient’s his tory and environment wi l l help immensely in sorting out what the “most l ikely” cause ofintoxication i s .

497. The answer is f. (Brunton, p 1874; Katzung, pp 1013-1017.) Succimer (a more polar sa l t of dimercaprol ; Bri ti sh anti -Lewis i te; BAL) would be thechoice, given the proof of lead poisoning, the lack of acute symptoms, and the fact that our patient i s a chi ld. Succimer i s easy to give ora l ly and i stolerated far better than the a l ternatives : Ca-Na 2-EDTA (a), penici l lamine (e; tradi tional ly viewed as a copper chelator, but i t a l so chelates lead),or dimercaprol (c) i tsel f. Al though the heavy meta l chelation profi les for succimer are not drastica l ly di fferent from those of dimercaprol , the factthat succimer i s more polar (and, therefore, less l ikely to enter cel l s ) seems to account for far fewer and mi lder s ide effects than those ofdimercaprol (especia l ly with respect to ri sks of tachycardia and hypertens ion). N-Acetylcysteine (d), as noted elsewhere, i s the drug of choice foracetaminophen poisoning. It has no abi l i ty to chelate lead or other cations .

498. The answer is a. (Brunton, pp 186f, 266, 1792; Katzung, pp 84, 479, 481.) Botul inus (botul inum) toxin prevents release of acetylchol ine (from storageves icles ) by vi rtua l ly a l l chol inergic nerves . Thus , there i s no activation of any chol inergic receptors , whether nicotinic or muscarinic. Noteworthyfindings , then, include an inabi l i ty to activate a l l postgangl ionic neurons (sympathetic and parasympathetic), no phys iologic release ofepinephrine from the adrenal medul la , and flaccid skeleta l muscle para lys is due to fa i lure of ACh release from motor nerves . The cause of deathis venti latory fa i lure because the intercosta l muscles and diaphragm are nonfunctional .

Pra l idoxime is a chol inesterase reactivator, an antidote and adjunct (a long with atropine) for poisonings with “i rrevers ible” chol inesteraseinhibi tors such as soman, sarin, VX (“nerve gases”), and many organo-phosphorus insecticides . Because no ACh i s being released in botul inuspoisoning, “reactivation” of the enzyme that normal ly metabol i zes the neurotransmitter i s i rrelevant (and ineffective).

499. The answer is e. (Brunton, pp 239, 242, 249-250; Katzung, pp 1035-1036.) Most of the adverse responses to nerve gases (i rrevers ible ACh esteraseinhibi tors such as soman and sarin) are due to a bui ld-up of ACh at muscarinic receptors (ie, ACh released from postgangl ionic parasympatheticnerves or sympathetic/chol inergic nerves innervating sweat glands). Those responses wi l l be attenuated by atropine, because i t i s a highlyspeci fic competi tive muscarinic antagonis t. However, skeleta l muscle s timulation (or eventual para lys is ) involves nicotinic receptor activation.That wi l l not be affected by atropine, and unless other supportive measures are provided, the patient i s l ikely to die from venti latoryarrest/apnea.

500. The answer is e. (Brunton, pp 35, 855-857, 862; Katzung, pp 608-611.) No, this i s not a frivolous question. Someday you may be working in anemergency department or a poison center, so you need to know. The cri tters that eat the rodenticide ul timately bleed to death from a warfarinoverdose. When treating such poisonings in humans or poisoned pets , the approach i s the same as with therapeutic warfarin: adminis ter vi taminK (ora l ly or parentera l ly, whichever i s indicated), and provide other symptomatic and supportive measures . Amphetamine or coca ine, a lone or incombination (a), as wel l as morphine (b), are control led substances with high potentia l for dependence and abuse. Such substances s implywouldn’t be permitted in products that vi rtua l ly anyone could obta in at thei r loca l home care or hardware s tore. What about succinylchol ine (c)?It’s ineffective when ingested; i t has to be injected. The same appl ies the nondepolarizing blockers (d), vecuronium, and the other “curoniums” or“curines .” (You’l l remember that some peoples indigenous to the Amazon ki l led their prey with curare-tipped darts . And they ate that curare-lacedmeat with absolutely no i l l effects from the drug.)

Page 249: Pharmacology pre test self assessment and review (14th ed.)

Appendix A

Drug Interactions Involving the P450 System

Page 250: Pharmacology pre test self assessment and review (14th ed.)
Page 251: Pharmacology pre test self assessment and review (14th ed.)
Page 252: Pharmacology pre test self assessment and review (14th ed.)
Page 253: Pharmacology pre test self assessment and review (14th ed.)

Appendix B

Common Serum Chemistries (Normal Values)

Page 254: Pharmacology pre test self assessment and review (14th ed.)
Page 255: Pharmacology pre test self assessment and review (14th ed.)

Appendix C

Web Sites for More InformationMost texts that have been publ i shed in the las t few years , and many that wi l l be publ i shed, have a variety of onl ine resources that are wel l wortha look. These are usual ly “free,” but of course that usual ly means they’re free i f you purchased the book on which the web s i te i s based.

However, there are many other s i tes where you can get good information (and lots more where the information you get i s sketchy or of dubiousaccuracy or objectivi ty, at best). Access to some s i tes i s free, but others require payment. Here are a selected few that I vi s i t frequently, and bel ieveare worth your time and trust. I ’ve l i s ted them a lphabetica l ly.

Drug Facts and Comparisons (http://www.factsandcomparisons .com). This i s usual ly my go-to web s i te for a l l the need-to-know information aboutdrugs and drug products , including drug-drug interactions . The s i te provides excel lent summary tables about a l l sorts of things pharmacologic andtherapeutic, and in a way that wi l l help you glean what’s important. Access to the information i s not free. However, your school should have a s i tel icense for access ; i f they don’t, pester the appropriate adminis trative person to buy one. After a l l , look at how much tui tion you’re paying!

FDA (Food and Drug Adminis tration) (http://www.fda.gov). The FDA has a great (and free!) s i te for a l l things medica l , not just drugs . Some partsare targeted to the publ ic; others are for heal th care profess ionals . You can s tay on top of latest drug or device approvals , concerns , warnings , orreca l l s , and get much more information that ul timately wi l l be of interest and importance to you.

One part of the web s i te, MedWatch (http://www.fda.gov/medwatch) i s a must-see, especia l ly once you hi t the wards . The information there wi l lkeep you up-to-date on the latest drug safety concerns , reca l l s , and so on. You can subscribe, for free, to get personal e-mai l s about the very latestconcerns and warnings . That’s important to you as a future practi tioner and caregiver. (And once you hi t the wards i t wi l l give you current andimportant information that you can casual ly mention to your cl inica l team—or the attending who gives you your clerkship grade—showing thatyou’re rea l ly on top of things .)

Journal Watch (http://www.jwatch.org). Journal Watch, publ i shed by the Massachusetts Medica l Society (the publ i shers of NEJM), has expertreviewers who provide succinct and objective summaries and eva luations of the latest articles from nearly 200 top medica l journals . The topicsrange from genera l internal medicine to specia l ties and subspecia l ties . The reviews are l inked to the origina l publ i shed articles , some of whichcan be accessed for free.

“JWatch” i s a subscription service (your medica l l ibrary l ikely subscribes), and most individual subscribers opt for e-mai l del ivery of thesummaries , which often arrive da i ly.

Journal Watch has a s ibl ing s i te, Fi rs t Watch (fi rs twatch.jwatch.org), that provides free da i ly a lerts , del ivered to your e-mai l inbox, on currentmedica l ly related news.

Clearly, not a l l the summaries on ei ther s i te are drug-related, but many are, and you may find them quite interesting and helpful i f you want aneasy way to keep up with the newest l i terature.

The Medical Letter (http://www.medica l letter.com). This i s another excel lent web s i te that wi l l keep you up to date. One important and ratherunique facet of The Medica l Letter’s updates i s an unbiased look at, for example, whether a new drug i s any better or safer than oldera l ternatives . The publ i sher frequently provides excel lent tables that compare, for example, costs of otherwise s imi lar and a l ternative drugs ;summaries of recommended drugs for a particular condition, and much more.

The Medica l Letter a lso publ i shes “Treatment Guidelines from the Medical Letter,” which col lects and presents in a concise and unbiased way keyinformation usual ly derived from prior Medica l Letter i s sues . Access to “The Medica l Letter” requires a subscription (and you can a lso subscribe tosnai l -mai l updates ). Here, too, see i f your school has a s i te l i cense, and i f they don’t urge them to get one.

Prescriber’s Letter (http://www.prescribers letter.com) tops my l i s t of recommended web-based resources for a variety of reasons , not the least ofwhich i s i ts s ingular focus on drugs and drugs therapy. This i s another web s i te for which you’l l ei ther have to pay, or pester your med ed dean ormedica l l ibrary folks to get a s i te l i cense. Prescriber’s Letter keeps you updated with new and practica l information on the latest drugs and drugtherapies in just about any area you might find of interest. It i s objective and unbiased, and excels (in my opinion) in terms of debunking mythsabout new drugs or therapies , and giving you a decent “heads up” about drug-related i ssues that are l ikely to be important to you. You’l l getrelevant information much sooner with Prescriber’s Letter than with just about any other web s i te I ’ve vis i ted. With a subscription you can accessa l l sorts of good information about drugs , and be assured that you’re not getting anything but current and important information.

RxList (http://www.rxl i s t.com). This i s another comprehens ive s i te where you can find abundant, current, and objective information about drugs .You can a lso get interesting (i f not important) information about, for example, the “most prescribed drugs ,” the latest news, and so on. Al l theinformation on rxl i s t.com is free.

Page 256: Pharmacology pre test self assessment and review (14th ed.)

Index Please note that index links point to page beginnings from the print edition. Locations are approximate in e-readers, and you may need to page down one or moretimes after clicking a link to get to the indexed material.

AAAPMC. See Antibiotic-associated pseudomembranous col i ti s ; Cl indamycinAbciximab 227, 269, 279. See also Antiplatelet drugs

acute coronary syndrome and, 205, 284–285Aborti facients , 270, 431, 442Acarbose, 433, 440, 443–444. See also Antidiabetic drugs

hypoglycemia and, 420, 433s ide effects of, 402, 426–427

ACE inhibi tors . See Angiotens in-converting enzyme inhibi tors ; Antihypertens ive drugsAcebutolol , 15–16. See also β-Adrenergic agonis ts

as partia l agonis t, 42, 67, 92, 107Acetaminophen, 26, 247, 270, 339–340, 350. See also Analges ics ; Antipyretics

l iver fa i lure from overdose of, 50, 362–363N-acetylcysteine for overdose of, 339–340, 355, 363, 370, 375–376, 492stages of fata l poisoning of, 363

Acetazolamide, 296, 299, 301, 313–314, 316–317, 320. See also Carbonic anhydrase inhibi tors ; Diureticsfor a l ti tude s ickness , 302, 316–317as antiepi leptic drug adjunct, 316causes coma, 301, 316for chronic open angle glaucoma, 302, 316hyperammonemia caus ing coma from, 301, 316–317hypokalemia caused by, 309, 319metabol ic acidos is caused by, 309–310, 316–317, 320urate solubi l i ty increased by, 297, 310–311urinary a lka l inization caused by, 310, 317, 326urine electrolyte profi les and, 319

Acetylchol ine, 63, 84, 90, 99, 103, 114, 359, 397, 521airway smooth muscle contraction caused by, 103botul inus toxin prevents release of, 521as endogenous receptor agonis ts , 63endothel ium removal a ffecting responses to, 77, 121, 210, 247–248esterase inhibi tors , 65, 90, 98, 115–116, 122, 173, 192, 268, 281, 283, 286, 332, 343–344, 358, 360, 378, 515, 517, 521gastric acid secretion increased by, 393overstimulation from, 97, 122as periphera l neurotransmitter, 83, 281vasoconstriction from, 77, 122

Acetylchol inesterase (AChE), 65, 83, 90, 98, 110, 115–116, 122, 173, 177, 192, 283, 332, 343, 358, 360, 391, 511, 514, 517pra l idoxime reactivation of, 124, 511, 514, 521

Acetylchol inesterase inhibi torsAlzheimer disease and, 143, 178asthma and adverse effects of, 122, 274myasthenia gravis and, 65, 102, 122, 123, 344poisoning by, 62, 103

ACh. See Acetylchol ineAChE. See Acetylchol inesteraseActivated charcoal in ora l poisonings , 506, 517–518Activated partia l thromboplastin time (APTT), 219, 267Acute coronary syndrome, 151, 188, 205, 227, 229, 284, 297

thrombolytics for, 227, 268–270, 276, 279Acyclovi r, 475–476Adenos ine, 219, 237, 242, 268, 292, 338

as carotid s inus massage a l ternative, 236paroxysmal supraventricular tachycardia and, 204, 230–231, 236, 242

Adenos ine monophosphate (AMP), 6. See also Cycl ic AMPAdenylyl cyclase, 45–46ADH. See Antidiuretic hormoneADHD. See Attention defici t-hyperactivi ty disorderAdrenal cortica l tumor, 312–314. See also Cushing diseaseAdrenal corticosteroids . See also Corticosteroids

Cushing disease, synthes is inhibi ted, 441

Page 257: Pharmacology pre test self assessment and review (14th ed.)

s igns and symptoms of, 438synthes is , 438

Adrenal (suprarenal ) medul la , 74, 84, 88, 107, 118, 124, 339–340, 523α-Adrenergic agonis ts

epinephrine, 19, 42, 58, 60, 63–64, 74, 82, 85, 88, 90, 95, 97, 99–101, 103, 107–108, 118, 120, 124–125, 213, 242–244, 249, 254, 272, 275, 284–285, 291–292,335–336, 339–340, 344, 358, 427, 434, 521

norepinephrine, 39, 42, 53–56, 66, 74, 82–84, 86–88, 92, 99, 103–105, 107–108, 117–120, 124, 126, 155, 164, 170, 172, 181, 186, 196–197, 241, 244, 258, 270–271, 275, 278, 335, 339, 360, 427, 434, 483

phenylephrine, 79–80, 88, 108, 117, 119, 125–126, 159–160, 227, 241, 258, 268, 280, 283β-Adrenergic agonis ts

a lbuterol , 39, 96, 113, 213, 254, 328–329, 335, 338, 341, 345, 395, 440epinephrine, 19, 42, 58, 60, 63–64, 74, 82, 85, 88, 90, 95, 97, 99–101, 103, 107–108, 118, 120, 124–125, 213, 242–244, 249, 254, 272, 275, 284–285, 291–292,

335–336, 339–340, 344, 358, 427, 434, 521isoproterenol , 89, 99–100, 103, 107–108, 113, 116, 119–120, 125, 242, 248, 258, 281, 292, 345norepinephrine, 39, 42, 53–56, 66, 74, 82–84, 86–88, 92, 99, 103–105, 107–108, 117–120, 124, 126, 155, 164, 170, 172, 181, 186, 196–197, 241, 244, 258, 270–

271, 275, 278, 335, 339, 360, 427, 434, 483sa lmeterol , 328, 335–336, 345–346

α-Adrenergic blockers , 89, 101, 107, 119–120, 124, 159, 254, 265, 275, 280–281, 283, 285, 289, 427phenoxybenzamine, 88, 190, 204, 213, 243, 254phentolamine, 65, 88–89, 101–102, 117–118, 159, 241, 289, 360prazos in, 32, 65, 86, 88, 101–102, 107, 119–120, 124–126, 159, 204, 214, 240–241, 243, 248, 257, 258, 265, 267, 270, 275, 280, 283, 427terazos in, 120, 280

β-Adrenergic blockers , 60, 67, 94, 105, 203, 231, 238, 240, 245, 256, 262, 270, 275, 283, 285–286, 358, 360–361, 414, 425, 427, 430, 434. See also Labeta lol ,metoprolol , propranololabrupt discontinuation of, 271–272α-adrenergic blocking properties of, 191, 197, 253angina pectoris and, 202, 229, 240, 244, 414as antihypertens ives , 63, 79, 99, 203, 231asthma and, 67, 105AV nodal conduction and, 105, 268contra indications for, 97, 105–106essentia l hypertens ion and, 58, 96, 203heart fa i lure and, 106, 239hyperthyroidism or thyrotoxicos is and, 67, 105–106, 435, 445pheochromocytomas and, 95–96, 231, 285vasodi lating properties of, 286

Adrenergic bronchodi lators . See Albuterol ; Sa lmeterolAdrenergic nerves and neurotransmiss ion, 53–55, 63, 66, 74, 82, 86, 89–92, 95, 99, 107, 109, 118, 121, 179, 192α-Adrenergic receptors , 54, 56, 82–83, 87–89, 94, 101, 109, 117–118, 120, 124–125, 160, 167, 186, 188, 193, 243–244, 254, 258, 280, 285, 290–291, 345, 360

presynaptic, 56, 87–89, 92, 101–102, 104, 241, 275β-Adrenergic receptors , 16, 42, 54, 56, 58, 85, 90, 98, 105, 107–108, 110, 118, 125, 159, 244, 254, 281, 285–286, 292–293, 335, 339, 345, 361, 395, 425, 427, 434,

445Adrenergic receptors , subtypes and locations of, 82–83, 118Adrenomimetic responses , 55, 89Age affecting drug therapy, 8, 12, 32, 38Agranulocytos is , 153, 187, 193, 443AIDS, drugs for. See Antivi ra l/anti retrovi ra l drugsAirway smooth muscle, adrenergic control of, 39, 85, 103, 109, 113–114, 118–119, 159, 170, 255, 328, 335, 346, 359, 363Akathis ias , 135, 157Albuminuria , 238, 408Albuterol , 39, 96, 113, 213, 254–255, 327–328, 336, 339, 341, 345, 395, 440. See also β-Adrenergic agonis ts ; Asthma

asthma, labeta lol and, 91–92, 94–95, 110, 205, 239–241, 245, 249, 254, 260, 271, 275, 286, 425asthma inappropriately treated with, 327, 334–335skeleta l muscle tremors from, 329, 338

Alcohol . See EthanolAldosterone, 256, 308, 312–314, 316–318, 322–324, 438, 442, 446

receptor blockade by spi ronolactone or eplerenone, 316, 438, 441, 446Alendronate, 410, 428Al lopurinol . See also Antigout/antihyperuricemic drugs

azathioprine toxici ty ri sk with, 488, 499–500cancer and, 348, 360, 489–490, 498–500febuxostat compared to, 246, 348, 359, 366, 369, 490, 499–500mercaptopurine toxici ty ri sk with, 489–490, 499–500uric acid synthes is inhibi tion by, 369, 377

Alosetron, 391, 405–406Alteplase, 220, 263, 268, 276, 279. See also Thrombolytics

Page 258: Pharmacology pre test self assessment and review (14th ed.)

Alti tude s ickness , 302, 316–317Aluminum sa l t antacids , 387, 393, 398, 401–404

magnes ium sa l ts with, 385, 397–399, 401phosphate and, 390, 401, 404

Alzheimer disease, drugs for, 47, 143, 178Amantadine, 174, 184, 199, 476. See also Antiparkinson drugs ; Antivi ra l/anti retrovi ra l drugsAmenorrhea, 135, 167, 181, 199, 423, 446Amikacin, 456, 475, 480–481. See also Antibiotics , aminoglycos idesAmi loride, 257, 271, 304, 317–319, 321–325. See also Diuretics

hypokalemia and, 304, 319identi fying use of, 306, 325

Amine pump, 82, 86–87, 89, 104γ-Aminobutyric acid (GABA), 25, 46, 152, 173, 176, 192, 512Aminocaproic acid, 263–264, 270, 276Aminoglycos ides , 11, 304, 321, 457, 459–461, 468, 472, 474–475, 477–482. See also Antibiotics , individual classes

furosemide with, 216, 221, 224, 230, 240, 249, 255, 259, 264, 271–272, 274, 289, 298–300, 303, 308–312, 317, 319–324, 395neuromuscular blockers and, 459, 479–480ototoxici ty from, 458, 477–480penici l l in and, 451, 456–458, 460, 464, 467–468, 470–472, 474

Amiodarone, 220, 261, 269, 281, 283, 411, 429–431. See also Antiarrhythmic drugsfor atria l fibri l lation, 220, 252–253, 255, 269, 281, 411pulmonary fibros is from, 261, 269, 493thyroid abnormal i ties from, 410, 427–428

Amoxapine, 181–182Amoxici l l in, 400, 456, 464, 466, 470–471, 474, 481. See also Antibiotics ; Penici l l ins

ampici l l in compared to, 456, 474Amphetamines , 39, 57, 86, 89, 107–108, 110, 122, 124, 126, 137, 164, 169–170, 172, 179, 183, 189, 203, 241, 353, 369, 371, 514, 518, 520, 522

toxici ty of, 504, 517Amphotericin B, 457, 473, 475Ampici l l in, 456, 474, 481. See also Antibiotics ; Penici l l insAnalges ics

acetaminophen, 26, 247, 270, 339–340, 350aspirin, other NSAIDs

aspirin, 26, 32, 41, 47, 204, 207, 208, 219–220, 229, 233–234, 239, 242, 246, 249–250, 260, 267, 269, 277, 282–283, 285, 287, 290, 299, 309, 330, 339–340,349–351, 353–355, 358–359, 361–362, 364–366, 368–373, 375, 387, 400, 471, 478, 506, 512, 518

diclofenac, 247, 351, 357, 361ibuprofen, 26, 246–247, 351, 357, 361, 367, 441indomethacin, 349, 351, 355, 357, 359, 361, 367, 369, 374–377, 441ketorolac, 72, 133, 161–163, 391naproxen, 246–247

opioidscodeine, 144, 173, 180, 261, 341, 368fentanyl , 156, 168, 173, 181, 197, 391hydrocodone, 342meperidine, 32, 138, 170–171, 182methadone, 150, 171, 189, 194, 466morphine, 26, 72, 109, 113, 138, 147, 149, 161–162, 171, 179, 184–185, 187–190, 197, 199, 202, 261, 358, 391, 405, 512, 522oxycodone, 144pentazocine, 147, 184–185

Anaphylaxis , 100, 103, 291, 335–336, 359, 396Anesthetics , genera l

inhalation agents , 341halothane, 161–162, 191isoflurane, 191, 459, 479

injectable agents , 136, 161, 177, 425mal ignant hyperthermia and, 46, 161–162, 191–192minimum a lveolar concentration of, 163, 175nitrous oxide as , 141, 175, 181, 459potency of inhaled agents , 177. See also Minimum a lveolar concentration of inhaled anesthetics

Anesthetics , loca l . See also l idoca ine; Proca ineal lergic reactions to, 142, 176–177epinephrine in, 64, 100–101as ester or amide, 177nitrous oxide and, 141, 175recognizing chemica l class by name, xxix–xxx, 176–177

Page 259: Pharmacology pre test self assessment and review (14th ed.)

Angel dust. See Phencycl idineAngina pectoris , 202, 225, 229. See also Antiangina l drugs

β-adrenergic blockers , ni troglycerin and, 202, 225, 229nitrovasodi lators and, 225

Angiotens in I I , 92, 238, 257, 266, 271–272, 277–278, 280, 286–287, 311, 314, 322, 424Angiotens in converting enzyme, 254–255, 271–272, 322, 367, 408Angiotens in receptor blockers , 238, 244, 248, 253, 255–256, 263, 270, 272, 274, 277–278, 280, 409

bi latera l renal artery s tenos is contra indicating, 238, 252, 408losartan, 226, 238, 244, 248, 256, 263, 266, 270, 274, 277–278, 280pregnancy and, 248–249, 253

Angiotens in-converting enzyme (ACE) inhibi torsbi latera l renal artery s tenos is contra indicates , 238, 252, 408blood pressure lowered with, 367captopri l , 205, 221, 223, 230, 240, 255–256, 259, 270–271, 277, 282, 285, 323, 367, 374enalapri l , 224, 274heart fa i lure and, 217, 228, 239, 259, 264, 278, 280, 312, 424hypertens ion and, 190, 217, 228, 238–239, 305, 312, 323, 367l i s inopri l , 225–226, 277–278, 305mechanisms of action of, pregnancy and, 248–249, 253ramipri l , 211, 252, 255, 283thiazides added to, 217, 283

Angle closure glaucoma. See glaucomaAnorexigenic drugs , 57, 108Antacids , 387, 393–394, 397–399, 401, 403–404, 473. See also individual drugsAnthrax, 447, 466Antiangina l drugs

β–adrenergic blockersacebutolol , 15–16, 42, 67, 92, 107atenolol , 92, 105, 109, 126, 221, 244, 254, 275, 286, 345, 425metoprolol , 92, 105, 109, 232, 254, 275, 286, 344, 425nadolol , 62, 92, 97nebivolol , 95, 203, 241, 245, 286pindolol , 37, 92, 107, 254, 426propranolol , 32, 60, 86, 94–95, 99, 105, 110, 114, 116–117, 119, 121, 126, 160, 206, 231, 245, 254, 268, 275, 283–286, 289, 360–361, 409, 425, 434, 445timolol , 92, 98, 125, 240

ca lcium channel blockers , 172, 216, 223, 240, 242, 244–246, 249, 251, 255–256, 259, 266, 270, 273–274, 283, 286, 290, 297, 309, 312nitrates , ni troglycerin, 202, 206, 223–225, 232, 240, 242, 245, 253, 257, 259, 262, 264–267, 270, 272–273, 277, 282, 285, 289, 513

Antiarrhythmic drugsadenos ine, 242, 268, 292, 337amiodarone, 211, 220, 261, 269, 281, 412, 429, 430atropine, 62, 82–84, 89, 97, 99–100, 103, 106–108, 111–112, 115–117, 119–121, 123, 125–127, 159, 167, 171, 173, 178, 199, 214, 258, 281, 289, 292, 338, 341,

343, 360, 374, 378, 395, 397–398, 509, 512, 516–517, 521digoxin, 32, 205, 215–216, 224, 230, 244, 252, 255–257, 259–261, 268, 272, 274, 284, 288, 292, 298, 303, 311, 319–320, 520fleca inide, 216l idoca ine, 32, 177, 240, 242, 281, 292, 512proarrhythmic effects of, 261proca inamide, 254, 269, 512quinidine, 32, 211, 216, 242, 252–254, 260–261, 269, 383, 392, 478, 512Vaughan-Wi l l iams class i fi cation of, 216

Antibiotic-associated pseudomembranous col i ti scl indamycin caus ing, 450metronidazole for treating, 401, 456

Antibioticsaminoglycos ides , 11, 304, 321, 458–461, 468, 472, 474–475, 477–482antimycobacteria ls . See Anti tubercular drugscephalosporins , 195, 451, 454, 457–459, 463, 467, 470–472, 474, 478, 483fluoroquinolones , 464, 471l inezol id, 450, 466, 468, 470, 472, 483macrol ides , 464, 471–472, 474penici l l ins , 254, 451, 456–458, 461, 463–464, 467–468, 470–472, 474–477, 480–483sul fonamides , 254, 403, 463, 511tetracycl ines , 388, 400, 465, 468, 472–473, 480, 482vancomycin, 400, 465–468, 471–473, 475, 480, 483

Anticancer drugsa lkylating agents carmustine, 497

Page 260: Pharmacology pre test self assessment and review (14th ed.)

antimetabol i tescytarabine, 486, 494fluorouraci l , 488, 494

anti tumor antibioticsbleomycin, 493, 497–499dactinomycin, 30, 494, 497daunorubicin, 30, 494doxorubicin, 30, 488, 493–494, 497–498

hormone antagonis tsfulvestrant, 486, 494–495ra loxi fene, 431tamoxi fen, 30, 412, 430–431, 494–495, 499

platinum-based agents ci splatin, 478, 493, 497selective estrogen receptor modi fiers

ra loxi fene, 431tamoxi fen, 30, 412, 430–431, 494–495, 499

taxanes/mitotic inhibi tors , pacl i taxel , 497–498topoisomerase inhibi tors

etopos ide, 30, 497topotecan, 497

vinca a lka loidsvinblastine, 30, 493, 499vincris tine, 486, 493, 497–498

Antichol inergic drugs . See AtropineAnticoagulants

di rect thrombin inhibi tor, biva l i rudin, 249–250, 276heparins , 217, 225, 243–244, 250, 253, 263–264, 267, 276, 279warfarin, 21, 32, 39, 194, 220, 225, 243–244, 247, 249–250, 253, 263–264, 267–269, 272, 276, 288, 375, 382, 392–393, 402, 455, 470–471, 474, 492, 495, 521

Anticonvulsant drugs , 192–193ethosuximide as , 151, 189fosphenytoin as , 176, 189lorazepam as , 32, 160, 176, 189, 197, 370, 510phenytoin as , 136–137, 166, 168–170, 174, 176, 189–190, 194, 197, 307, 368, 370, 382, 392, 417, 438, 492, 495, 510status epi lepticus and, 142, 145, 160, 176, 189, 370, 510, 517

Antidepressantsatypica l , 166, 196

bupropion, 146, 183, 194, 196–198buspirone, 134, 166, 193, 196venlafaxine, 155, 196

monoamine oxidase inhibi tors , 105, 110, 145, 179, 181, 483selective serotonin reuptake inhibi tors (SSRIs ), 105, 110, 145, 179, 181

fluoxetine as , 37, 139, 173, 181, 187, 195, 198, 200, 273, 307, 368, 489, 499sertra l ine as , 181, 187, 195, 200, 273, 368, 489, 499

tricycl ic/heterocycl ic, 73, 82, 84, 86, 104, 109, 116, 119, 122, 135, 166–167, 173, 181–183, 186, 196–197, 200, 512, 517imipramine, 32, 73, 84, 86, 116, 119, 135, 156, 173, 181, 196–198, 517

Antidiabetic drugsamyl in and incretin-related drugs , exenatide, 415, 433, 435–436α-glucos idase inhibi tors , acarbose, 434, 440, 443–444insul ins , 25, 46, 90, 238–239, 289, 307, 408, 413, 425, 427, 430–435, 440, 442–444, 467metformin (biguanides), 211, 251, 289, 408, 416, 420, 428, 431–432, 434–435, 439–440, 443–444sul fonylureas , 195, 289, 427, 432–435, 440, 442–444

gl imepiride, 442gl ipizide, 442, 444–445glyburide, 407, 415, 419, 442, 444tolbutamide, 195, 440, 442, 444, 518

thiazol idinediones (gl i tazones), 443, 445piogl i tazone, 421, 427, 440, 443–444

Antidiuretic hormone, 197, 307, 310, 315Antiepi leptic drugs . See Anticonvulsant drugsAnti freeze poisoning. See Ethylene glycol poisoningAnti fungal drugs

amphotericin B, 457, 473cefmetazole, 454, 470–472, 477esomeprazole, 225, 383, 392–394, 400fluconazole, 458, 479

Page 261: Pharmacology pre test self assessment and review (14th ed.)

ketoconazole, 431, 438, 446, 466metronidazole, 387, 400, 465, 468–470, 473sul famethoxazole, 470, 473

Angle-closure glaucoma. See GlaucomaAntigout/antihyperuricemic drugs

anti -inflammatory drugscolchicine, 356, 359, 364, 369, 376, 377diclofenac, 247, 351, 357, 361ibuprofen, 26, 246–247, 351, 357, 361, 367, 441indomethacin, 349, 351, 355, 357, 359, 361, 367, 369, 374–377, 441

uricosuricsprobenecid, 350, 359, 363–364, 368–369, 377sul finpyrazone, 368

xanthine oxidase inhibi torsa l lopurinol , 246, 359, 369, 377, 489–490, 498–500febuxostat, 246, 348, 359, 366, 369, 490, 499–500

Antihis tamines . See also H1 antagonis ts ; H2 blockers

diphenhydramine, 73, 78–79, 96, 103, 106, 115–116, 123–127, 151, 153, 178–179, 193, 195, 198, 341, 345, 348, 354, 357, 360–361, 368, 374, 378, 405, 505,517

doxylamine, 80, 126–127, 374, 517fexofenadine, 34, 354, 360, 367, 374, 378loratadine, 125, 173, 357, 374, 378

Antihyperl ipidemic drugsbi le acid sequestrants

cholestyramine, 172, 259, 287–288, 437colesevelam, 233, 287–288, 415, 436–437, 443colestipol , 287–288, 437

ezetimibe, 226, 278fibrates , 278HMG Co-A reductase inhibi tors (s tatins )

atorvastatin, 76, 120, 172, 223–224, 260, 285, 297pravastatin, 211, 251rosuvastatin, 247

niacin, 211, 234, 251, 278, 288, 290, 408, 424Antihypertens ive drugs , 63, 79, 87–88, 99, 150, 159, 188, 201, 206–207, 210, 212–214, 222, 224, 227, 230–231, 248, 255, 272, 400, 408, 512Anti -inflammatory drugs , nonsteroida l nonselective COX-1/-2 inhibi tors

aspirin, 26, 32, 41, 47, 204, 207–208, 219–220, 229, 233–234, 239, 242, 246, 249–250, 260, 267, 269, 277, 282–283, 285, 287, 290, 299, 309, 330, 339–340,349–351, 353–355, 358–359, 361–362, 364–366, 368–373, 375, 387, 400, 471, 478, 506, 512, 518

diclofenac, 247, 351, 357, 361ibuprofen, 26, 246–247, 351, 357, 361, 367, 441indomethacin, 349, 351, 355, 357, 359, 361, 367, 369, 374–377, 441ketorolac, 72, 133, 161–163, 391naproxen, 246–247

selective COX-2 inhibi tor, celecoxib, 42, 340, 351, 361, 365–366Antimalaria l drugs

chloroquine, 466, 468, 484hydroxychloroquine, 351, 365, 464primaquine, 449, 466–468, 511–512, 518quinine, 242

Antimuscarinic drugs , 93, 96, 106, 111–112, 120–121, 195, 199, 516Antimuscarinic poisoning, 117, 516. See also Atropine, poisoningAntinuclear antibody ti ters , 220Antiparkinson drugs

antimuscarinics , centra l ly-actingbenztropine, 93, 116, 178, 184–185, 195, 198–199, 517diphenhydramine, 73, 78–79, 96, 103, 106, 115–116, 123–127, 151, 153, 178–179, 193, 195, 198, 341, 345, 348, 354, 357, 360–361, 368, 374, 378, 405,

505, 517trihexyphenidyl , 143, 178, 185, 195, 198–199, 517

dopaminergic drugsamantadine, 174, 184, 199, 478bromocriptine, 141, 164, 174, 176, 182, 184–186carbidopa, 88, 134, 154, 164–165, 174, 184, 195, 199, 396entacapone, 87, 194–195, 199levodopa, 37, 88, 134, 154, 164–165, 174–176, 179, 184, 195, 396ropini role, 142, 175–176

Page 262: Pharmacology pre test self assessment and review (14th ed.)

selegi l ine, 164, 178–179, 184–185, 195, 199Antiplatelet drugs

abciximab, 227, 269, 279aspirin, 26, 32, 41, 47, 204, 207–208, 219–220, 229, 233–234, 239, 242, 246, 249–250, 260, 267, 269, 277, 282–283, 285, 287, 290, 299, 309, 330, 339–340, 349–

351, 353–355, 358–359, 361–362, 364–366, 368–373, 375, 387, 400, 471, 478, 506, 512, 518clopidogrel , 216, 247, 249–250, 267, 269–270, 279, 471

Antipsychotic drugsatypica l

clozapine, 153, 187, 193–194, 200olanzapine, 187, 193, 200risperidone, 200

butyrophenones , ha loperidol , 138, 145, 167–168, 171, 175, 193, 198, 200, 307neuroleptic mal ignant syndrome and, 46, 157, 175, 183, 191–192, 200phenothiazines , chlorpromazine, 73, 116, 138, 146, 151, 167–168, 171, 181, 185–187, 190–191, 194, 198, 200, 406, 519

thioridazine, 524tAntipyretics

acetaminophen, 26, 247, 270, 339–340, 350aspirin, 26, 32, 41, 47, 204, 207–208, 219–220, 229, 233–234, 239, 242, 246, 249–250, 260, 267, 269, 277, 282–283, 285, 287, 290, 299, 309, 330, 339–340, 349–

351, 353–355, 358–359, 361–362, 364–366, 368–373, 375, 387, 400, 471, 478, 506, 512, 518ibuprofen, 26, 246, 247, 351, 357, 361, 367, 441

Anti thyroid drugsiodines/iodides , 411, 421, 429, 431, 436, 445thioamides . See Propyl thiouraci l

Anti tubercular drugsisoniazid, 385, 396, 461, 464, 466, 480, 483, 512ri fampin, 439, 466, 481, 483, 495

Antivi ra l/anti retrovi ra l drugsefavi renz, 469lopinavi r, 460, 469, 476, 480–481ri tonavir, 439, 460, 469, 476, 480–481saquinavi r, 30, 460zidovudine, 276, 448, 451–452, 460, 463–464, 467–469, 476, 481

Anxiolytic drugs , 166, 196, 307Aplastic anemia, 404, 516

petroleum dis ti l lates caus ing, 515sul fasa lazine caus ing, 403

Apomorphine, 199, 386, 398Appeti te suppress ion. See Anorexigenic drugsAPTT. See Activated partia l thromboplastin timeArachidonic acid, 41, 339–340, 358–359, 366. See also Nonsteroida l anti -inflammatory drugsARBs . See Angiotens in receptor blockersArea under the curve, 6, 10, 34–35

bioavai labi l i ty determined with, 34Arrector pi l i muscles , innervation of, 50, 52, 82–84Arsenic, 516, 518–520Asparaginase, 493, 497Aspirin, 26, 260, 368–373, 478, 506, 512, 518. See also Sa l icyl i sm

activated charcoal for poisoning from, 517–518antiplatelet effects of, 47, 246–247, 269–270, 277, 366asthma, problems with, 329–330, 340bleeding time affected by, 218, 267bronchoconstriction/bronchospasm from, 72, 349, 359–360clopidogrel as a l ternative to, 219, 269hemorrhagic s troke from, 229, 283–284myocardia l infarction and, 233, 250NSAIDs antagonizing antiplatelet effects of, 246–247poisoning from, 354, 374Reye syndrome and, 350, 365toxici ty and management, 260, 353, 365–372uricosuric drug effects inhibi ted by, 359, 369

Asthma, 72, 92, 96–97, 105, 108–109, 113–114, 213, 224, 239, 254, 262, 274, 280, 328–330, 332–346albuterol and, 39, 96, 113, 213, 254, 327–328, 336, 339, 341, 345, 395, 440aspirin aggravating, 330, 340chol inesterase inhibi tors provoking, 332, 343–344corticosteroids and, 331, 342

Page 263: Pharmacology pre test self assessment and review (14th ed.)

leukotriene modi fiers and, 339muscarinic agonis ts provoking, 343omal izumab and, 328, 336–337refractory, 329sa lmeterol and, 328, 335–336theophyl l ine for, 329, 337–338venti latory depressants contra indicated in, 161, 331, 341

Atenolol , 92, 105, 109, 126, 221, 244, 254, 275, 286, 344, 425hypertens ion and, 206, 275

Atorvastatin, 76, 120, 172, 223–224, 260, 285, 297. See also Antihyperl ipidemic drugsATP synthes is , 85–86ATPase, 320. See also Sodium-potass ium adenos ine triphosphataseAtria l fibri l lation, 205, 211, 216, 220, 252–253, 255, 269, 281, 411

amiodarone and, 211, 220, 269, 281, 411quinidine and, 211, 216, 252–253

Atrioventricular node, 74Atropine, 62, 82–84, 89, 97, 99–100, 103, 106–108, 111–112, 115–117, 119–121, 123, 125–127, 159, 167, 171, 173, 178, 199, 214, 258, 281, 289, 292, 338, 341,

343, 360, 374, 378, 395, 397–398, 509, 512, 516–517, 521. See also Antimuscarinic drugschol inesterase inhibi tor poisoning treated with, 516–517diphenhydramine s imi lar to, 73, 341, 348diphenoxylate with, 386, 398poisoning, 106–107, 117, 123, 167, 378, 516–517s ide effects , 73, 117for s inus bradycardia , 228, 281–282

Attention defici t-hyperactivi ty disorder, drugs for, 68AUC. See Area under the curve Autonomic nervous system, 49–127AV nodal conduction/block, 221, 228, 242, 255, 264–265, 268, 270–271, 286, 292Azathioprine, 488, 497, 499–500Azi thromycin, 3, 25, 463–465, 474, 478. See also Antibiotics

BBacillus anthracis, 447Bacteroides fragilis, 458, 465, 477BAL. See Bri ti sh anti -Lewis i teBarbi turates

benzodiazepines compared to, 161–162laryngospasm from parentera l , 132, 161lorazepam and withdrawal from, 155, 197margin of safety/therapeutic index of, 166–167

Baroreceptor reflex, 94, 96, 99, 101, 110, 112, 115, 117, 160, 236, 240, 245, 256, 262, 268, 274, 282, 289, 291, 293antimuscarinics and, 117β-blockers and, 107carotid massage inducing, 236–237, 291gangl ionic blockers and, 114vasoconstrictors and, 64, 117, 289, 293vasodi lators and, 94, 112, 117, 160, 245, 270volume depletion and, 289

Beclomethasone, 328–329, 334, 338, 410for asthma, 328in systemic glucocorticoid therapy, 428

Bel ladonna a lka loids , 157, 199, 505, 516Benign prostatic hypertrophy, 76, 228, 280Benzodiazepines , 32, 160–162, 166

alcohol potentiating effects of, 507, 519–520amnestic effects of, 132barbi turates compared to, 161–162diazepam, 32, 140, 173effects blocked by flumazeni l , 146, 167, 169, 173, 182, 507, 519fluni trazepam, 146, 182flurazepam, 194lorazepam, 32, 160, 176, 189, 197, 370, 510midazolam, 68, 109, 132, 136, 145, 160–161, 168, 173, 181, 262, 331, 341, 392, 460seizures treated with, 160–161

Benzothiadiazides . See Hydrochlorothiazide;Thiazide diuretics

Page 264: Pharmacology pre test self assessment and review (14th ed.)

Benztropine, 93, 116, 178, 184–185, 195, 198–199, 517extrapyramidal reactions managed with, 198–199

Betamethasone, 419, 440Bethanechol , 72, 94, 111–113, 115, 122, 126, 145, 344, 360, 395

for functional urinary retention, 344, 395s ide effects of, 72, 112

Bi latera l diabetic nephropathy, 303Bi latera l renal artery s tenos is , angiotens in modi fiers contra indicated in, 238, 252, 408Bi le acid-sequestrants , 287

cholestyramine, 172, 259, 287–288, 437colesevelam, 233, 287–288, 415, 436–437, 443colestipol , 287–288, 437

Bi le sa l ts , 287–288, 402Bioavai labi l i ty, 2, 5, 21, 24, 29–31, 33–39, 43, 154, 172, 189, 393, 396, 455, 473–474

area under the curve to determine, 34–35of drugs , 29, 30–31, 39

Bioterrorism agentsanthrax, 447, 466chol inesterase inhibi tors , 89, 91, 97, 102, 120, 122–124, 199nerve gases , 103, 124, 173, 509, 511, 514, 521radioiodine, 412, 431, 445

Biotransformation reactions . See Drugs , metabol i smBipolar i l lness , 198, 223, 307

antidepressants for, 198, 307l i thium for, 198, 223, 307

Bisphosphonates , 410, 427–428, 441, 445Biva l i rudin, 249–250, 276. See also AnticoagulantsBladder function, drugs and, 71, 89Bleeding time, 219, 267Bleomycin, 493, 497–499β-blockers , 16, 37, 61, 89, 91–97, 99, 105–107, 116, 120, 125–126, 160, 188, 198, 203, 206, 224, 232, 238–241, 243–245, 249, 251–252, 254–255, 259–260, 262,

266, 268, 272, 274–275, 278, 282–283, 285–286, 289, 312, 320, 332, 335, 343–344, 375, 427, 434, 512Blood pressure. See a l so Antihypertens ive drugs

α-adrenergic agonis ts ra is ing, 215, 227β-adrenergic agonis ts ra is ing, 289β-adrenergic blockers lowering, 231, 289angiotens in modi fiers lowering, 238, 246, 249changes activating baroreceptors , 94, 96, 99, 101, 110, 112, 115, 117, 160, 236, 240, 245, 256, 262, 268, 274, 282, 289, 291equation for ca lculating, 94–95ni fedipine, inappropriate use to lower, 223, 273prazos in compared to

phenoxybenzamine for lowering, 243vasodi lators lowering, 222, 234, 275, 290–291

Bone marrow suppress ion, drugs caus ing, 376, 483, 492–493, 498, 511Botul inum toxin and botul i sm, 77, 83, 121, 248, 508, 521BPH. See Benign prostatic hypertrophyBradycardia , 59, 67, 93–94, 105, 115, 120, 123, 228, 257–258, 281, 291, 293, 335, 343, 346, 374, 378, 424

atropine for, 228, 281–282isoproterenol for, 281s inus , 228, 281–282

Bradykinin, 222, 271–272, 278, 352, 367captopri l and, 352, 367

Bradykininase. See Angiotens in converting enzymeBreast cancer

fulvestrant and, 488, 494–495tamoxi fen and, 412, 430–432, 494–495

Bri ti sh anti -Lewis i te, 518–520Bromocriptine, 134, 147–148, 164, 174, 176, 182, 184–186, 190, 199, 293, 423, 446

hyperprolactinemia and, 149, 186, 190, 199, 293, 446neuroleptic mal ignant syndrome and, 157, 200Parkinson disease and, 151, 157, 198–199primary amenorrhea and, 423, 446

Bronchoconstriction, 72, 113–114aspirin caus ing, 349, 361–362

Bronchodi lation, 39, 62, 70, 72, 82, 85, 97, 109, 113–114, 116, 170, 336, 338, 341–342, 346

Page 265: Pharmacology pre test self assessment and review (14th ed.)

Bronchospasm, 47, 92, 103, 105, 113, 170, 175, 254, 278, 328, 330, 334, 340, 343, 349, 358Bupropion, 146, 183, 194, 196–198

epi lepsy/seizures from, 156, 197–198sexual dys function and, 146, 183

Buspirone, 134, 166, 193, 196Butyrophenone antipsychotics . See Antipsychotic drugs ; ha loperidol

CCadmium poisoning, 506Calci tonin, 415, 436Calcium, 244, 250, 255, 259, 272, 397–398, 401, 428–429, 436–437, 455, 473Calcium carbonate, 397, 401Calcium channel blockers , 240

coronary vasospasm and, 259–260dihydropyridines , ni fedipine, 172, 216, 242, 250, 256, 262, 270, 273dihydropyridines compared with nondihydropyridines , 216, 242, 250–251for essentia l hypertens ion and, 240, 249, 297, 312nondihydropyridines

di l tiazem, 216, 223, 239–240, 244, 249–250, 255–256, 259, 262, 270, 273–274, 283verapami l , 25, 34, 174, 216, 239–240, 242, 249–250, 252, 255, 256, 259, 262, 266, 268, 270, 273–274, 282–284, 286, 407

Calcium gluconate, 415, 436cAMP. See Cycl ic AMPCancer chemotherapeutic drugs . See Anticancer drugsCaptopri l , 205, 221, 223, 230, 240, 255–256, 259, 270–271, 277, 282, 285, 323, 367, 374

blood concentrations and, 323–324bradykinin and, 352, 367bradykininase and, 222, 271, 368cough from, 221, 271heart fa i lure and, 221hypertens ion and, 218, 221, 386mechanisms of action, 270, 375NSAIDs affecting, 375–376pregnancy and, 210

Carbamazepine, 439, 470, 495, 518Carbidopa, 88, 134, 154, 164–165, 174, 184, 195, 199, 396

actions in gut, 147, 184–185levodopa used with, 37, 88, 134, 154, 164–165, 174–176, 179, 184, 195, 396pyridoxine counteracting, 385, 396

Carbonic anhydrase inhibi tors , 309, 311, 316–320, 324acetazolamide as , 316–317urine electrolyte profi les and, 314, 317

Cardiac glycos ides . See DigoxinCardiac muscle, adrenergic receptors and, 82, 86, 90, 227, 343Cardiac output

equation for, 94heart fa i lure and, 98, 214, 262pos i tive inotropes and, 303, 320

Cardiomyopathy, 350, 486, 493, 515Cardioselective β-blockers . See β-blockers ; atenolol ; metoprololCarotid s inus massage, arrhythmia termination by, 236–237, 291Carvedi lol , 91, 94–95, 110, 205, 208, 222, 225, 230, 239, 241, 245, 254, 260, 271, 277, 286, 425Catechol O-methyl transferase (COMT), 104, 154, 194

entacapone inhibi ting, 87, 194–195, 199Catecholamine, 42, 55, 58, 86, 88–89, 91, 95–96, 99, 107, 110, 120, 126–127, 164, 179, 186, 190, 239–240, 254, 285, 291, 515CCBs . See Ca lcium channel blockersCefazol in, 453, 471. See also Antibiotics , cephalosporinsCefmetazole, 454, 470–472, 477. See also Antibiotics , cephalosporinsCefoperazone, 454, 470–472. See also Antibiotics , cephalosporinsCefotetan, 455, 470–472, 478. See also Antibiotics , cephalosporinsCefoxi tin, 458, 477–478. See also Antibiotics , cephalosporinsCeftriaxone, 462, 483. See also Antibiotics , cephalosporinsCelecoxib, 42, 340, 351, 361, 365–366. See also Anti -inflammatory drugs , nonsteroida l

actions/properties of, 351, 365Centra l nervous system, 129–200Cephalosporins . See also Antibiotics , individual cephalosporins

Page 266: Pharmacology pre test self assessment and review (14th ed.)

interactions with a lcohol , 470mechanism of action of, 457, 477prolonged INR from, 471–472vi tamin K deficiencies and, 475

Chemoreceptor trigger zone, 163, 185, 391, 406Chemotherapy. See Anticancer drugsChlora l hydrate, 196, 521Chlordiazepoxide, 156, 198Chloroquine, 466–467, 484Chlorothiazide, 296, 320. See also Diuretics , thiazidesChlorpromazine, 30, 116, 138, 148, 157, 167–168, 171, 175, 185–187, 193, 196, 198, 200, 405, 517. See also Antipsychotic drugs

haloperidol compared to, 138, 167–168neuroleptic mal ignant syndrome and, 46, 157, 175, 183, 191–192, 200orthostatic hypotens ion from, 186, 193schizophrenia and, 148, 198schizophrenia treated with benztropine and, 157, 198

Chlortha l idone. See also Diuretics , thiazidesheart fa i lure, furosemide replacing, 289, 298, 300identi fying use of, 307, 325–326renal potass ium loss from, 299, 313

Cholesterol , drugs used to modi fy levels of. See Antihyperl ipidemic drugsCholesterol ga l l s tones , 399Cholesterol synthes is and drugs inhibi ting i t, 226, 399. See also Antihyperl ipidemic drugs , s tatinsCholestyramine, 172, 259, 287–288, 437Chol inergic cri s i s , 65, 78, 102, 122–123

edrophonium for di fferentia l diagnos is of, 78, 122–123myasthenic cri s i s compared to, 122–123

Chol inergic receptors , 82–84, 93, 97, 121, 159, 170, 186, 346, 360, 378subtypes and locations , 82–84

Chol inesterase. See Acetylchol inesteraseChronic myelogenous leukemia, 487, 495–496Chronic obstructive pulmonary disease, 62, 96–97, 327–346

ipratropium for, 96, 329, 337–338N-acetylcysteine and, 330, 339–340

Chronic open angle glaucoma. See GlaucomaCimetidine, 361, 368, 391–393, 397, 401, 438. See also His tamine receptor antagonis tsCinchona a lka loids . See Chloroquine; Hydroxychloroquine; QuinidineCinchonism. See Quinidine, toxici tyCiprofloxacin, 464, 466, 469. See also Antibiotics , fluoroquinolones

long QT syndrome and, 464Cisplatin, 478, 493, 497Cita lopram, 181, 196, 251, 499Clavulanic acid, 460, 481Clearance of drugs , 24, 36Cl indamycin, 400, 465, 468, 471, 473–474, 478–479Clonidine, 87–88, 159, 188, 253, 271–272. See also Antihypertens ive drugs

abrupt discontinuation of, 271–272Clopidogrel , 216, 247, 249–250, 267, 269–270, 279, 471. See also Antiplatelet drugs

as a l ternative to aspi rin for TIAs , 220, 269Closed-head injury, 136, 187

morphine and, 149, 187Clostridium difficile, 455, 465, 473Clostridium toxin, 77Clozapine, 153, 187, 193–194, 200. See also Antipsychotic drugs , atypica l

agranulocytos is from, 153, 187, 193–194Cocaine, 55, 82, 84, 86–87, 89, 100, 104, 109, 119, 121, 148, 177, 186, 470, 512, 522

adverse effects of, 148, 186inhibi tion of norepinephrine reuptake by, 82, 86, 104, 119, 121, 186as loca l anesthetic, 186

Codeine, 144, 173, 180, 261, 341, 368Colchicine, 356, 359, 364, 369, 376–377

acute gout and, 356, 376Colesevelam, 233, 287–288, 415, 436–437, 443

diabetes mel l i tus and, 415, 436–437hyperl ipidemias and, 415, 443

Page 267: Pharmacology pre test self assessment and review (14th ed.)

Colestipol , 287–288, 437Coma, 167, 185, 316–317, 435

due to acetazolamide, 301, 316hepatic, neomycin effect, 461

COMT. See Catechol O-methyl transferaseConivaptan, 315Constipation

alosetron and, 391, 405–406verapami l caus ing, 216, 262–263

Coombs-pos i tive test, drugs caus ing, 254COPD. See Chronic obstructive pulmonary disease Coronary artery disease, 241, 281Coronary s tea l , 246Coronary vasospasm, 105, 151, 174, 186, 190, 202, 206, 211, 215, 229, 245, 250, 259–260, 283Corticosteroids , 187, 213, 328, 331, 334, 336–338, 342, 345, 422, 435, 438, 444, 446

asthma and, 331, 342respiratory acidos is and, 332, 344

Cortisol , 346, 416, 428, 430, 438, 440, 446Cough, 144, 180

captopri l caus ing, 221, 270dextromethorphan treating, 330, 340–341

COX. See CyclooxygenasesCOX-2 inhibi tors , 42, 340, 351, 361, 366, 492. See also Celecoxib; Anti -inflammatory drugs , nonsteroida lCrohn disease, 379, 403Cromolyn, 329, 338Curare. See Neuromuscular blockers ; TubocurarineCushing disease, 314, 422, 441, 446

adrenal corticosteroid synthes is inhibi tion, 441Cyanide

nitropruss ide caus ing poisoning by, 215, 258–259poisoning and treatment, 225, 263, 277, 501, 510–511

Cyanocobalamin, 263, 385, 388, 396, 402Cycl ic AMP (cAMP), 2, 241Cyclooxygenases , 41, 249, 267, 269, 290, 298, 339–340, 351, 358–360, 366–367

See also Nonsteroida l anti -inflammatory drugs ; AntipyreticsCyclosporine

grapefrui t juice affecting bioava i labi l i ty of, 9, 33–34nephrotoxici ty and, 490, 500

Cystic fibros is , pancrel ipase for s teatorrhea in, 389Cytarabine, 486, 494Cytochrome P450 system, 29, 42, 514

genera l reaction schemes involving, 41inducers and inhibi tors of, 29as monooxygenase, 41

DDactinomycin, 30, 494, 497Dantrolene, 141, 157, 191, 200Daptomycin, 454, 472Daunorubicin, 30, 494Depress ion, 37, 78, 94, 100, 103, 109, 134, 145, 155–156, 158, 160, 162, 166–171, 175, 178–180, 182–185, 187, 189–191, 194, 196–198, 200, 211, 270, 341, 370,

378, 391, 398, 468, 476, 498, 503, 514–515, 517. See also Antidepressantsbuspirone and, 134, 196monoamine oxidase inhibi tors and, 179–180

Desens i ti zation of receptors , 39, 344Dexamethasone, 188, 438, 440, 446, 488Dextroamphetamine, 57, 89, 173, 518Dextromethorphan, 144, 180, 330, 340–341Diabetes ins ipidus

l i thium caus ing, 307nephrogenic, 313thiazide diuretics for, 319

Diabetes mel l i tus . See Antidiabetic drugsDiarrhea, 63, 93, 99, 127, 191, 215, 342, 368, 376–377, 383, 387, 395, 397, 400–402, 404, 435, 448, 455, 472, 506, 518, 520Diazepam, 32, 140, 173, 176, 183, 189, 194–195, 197, 262, 307, 341, 368, 510. See also Benzodiazepines

effects antagonized by flumazeni l , 169, 172

Page 268: Pharmacology pre test self assessment and review (14th ed.)

Diazoxide, 234, 289, 307, 424–425. See also Antihypertens ive drugsblood glucose levels ra ised with, 289, 307

Diclofenac, 247, 351, 357, 361Dietary Supplement Heal th and Education Act, 47Digi ta l i s . See DigoxinDigoxin, 29, 32, 205, 215–216, 224, 230, 244, 252, 255–257, 259–261, 268, 272, 274, 284, 288, 292, 298, 303, 311, 319–320, 520

as antiarrhythmic, 216, 252ATPase inhibi tion by, 259AV nodal conduction/block and, 216, 255, 268, 286, 292chromatops ia from, 274, 303coronary artery disease, furosemide with, 215, 255electrocardiographic effects of, 230, 284heart fa i lure and, 205, 215, 230, 259, 284, 319hypokalemia and, 259, 274, 318–319toxici ty of, 216, 224, 259–260, 274, 303, 319

Dihydropyridines , 216, 242, 250, 256, 262, 270, 273. See also Ca lcium channel blockers ; Ni fedipineverapami l/di l tiazem compared to, 270

Di l tiazem, 30, 216, 223, 239–240, 244, 249–250, 255–256, 259, 262, 270, 273–274, 283. See also Ca lcium channel blockersAV nodal conduction/block and, 270–271dihydropyridines compared to, 270use for hypertens ion, 224, 274vasospastic angina and, 211, 244–245

Dimercaprol . See Bri ti sh anti -Lewis i teDipeptidyl phosphate (DPP4) inhibi tors . See Antidiabetic drugs .Diphenhydramine, 73, 78–79, 96, 103, 106, 115–116, 123–127, 151, 153, 178–179, 193, 195, 198, 341, 345, 348, 354, 357, 360–361, 368, 374, 378, 405, 505, 517.

See also Antihis taminesantimuscarinic (atropine-l ike) effects of, 71, 115–116, 379, 517as fi rs t-generation antihis tamine prototype, 73, 115, 123, 126, 360periphera l autonomic actions of, 78, 123physostigmine for overdoses of, 116, 505, 517poisoning, 505, 517promethazine and, 151, 191for seasonal a l lergies , 357second-generation antihis tamines compared with, 354, 374sedation caused by, 126, 360, 374s ide effects of, 73, 78, 348, 357

Diphenoxylate, 384, 386, 394, 398Dipyridamole, 264, 270Disease-modi fying anti rheumatic drugs , 365, 491Dissociative anesthes ia . See KetamineDisul fi ram, 154, 195, 434, 453, 470

antimicrobia ls with actions resembl ing, 470sul fonylureas with actions resembl ing, 195

Diuretics , 139, 172, 188, 240, 246, 249, 253, 255, 295–326loop, 249–250, 255, 264, 271, 289, 308, 310–315, 317–326, 359, 375, 395, 478NSAID effects on actions of, 311, 359osmotic, 187, 318, 321, 511mannitol , 187, 315, 318, 321, 511potass ium-sparing, 257, 281, 311–313, 317, 321, 323, 325, 442, 446

ami loride, 257, 271, 304, 317–319, 321–325eplerenone, 300, 313–314, 316–317, 319, 323, 441spironolactone, 257, 307, 311–314, 316–319, 321–324, 438triamterene, 216, 224, 230, 257, 271, 274, 283, 307, 311, 317, 321–324, 326

pregnancy and, 249thiazides , 240, 244, 246, 253, 296, 319, 325, 425

chlortha l idone, 299, 307–308, 312, 317, 320, 324–325, 429hydrochlorothiazide, 223, 230, 244, 253, 255, 273, 296, 297, 310–313, 317–318, 320, 322, 324–325, 410, 424, 429metolazone, 239, 244, 255, 273, 299, 307–308, 312–313, 316–317, 319–321, 325, 430

DMARDs. See Disease-modi fying anti rheumatic drugsDobutamine, 56, 66, 74, 104, 107, 117–118, 258, 285–286, 308. See also Heart fa i lure, drugs for

a l ternatives to, 74, 117–118blood pressure ra ised by, 232, 286

DOPA decarboxylase, inhibi tors of, 91, 154, 164–165, 195, 396Dopamine, 19, 55–56, 63, 74, 86–88, 91, 93, 98, 104, 107, 117, 148–149, 164–165, 167, 172, 174–176, 178, 181–182, 184–187

high dosages of, 73, 117

Page 269: Pharmacology pre test self assessment and review (14th ed.)

renal effects of, 308Dopamine receptors , blockade of, 142, 157, 164–165, 167, 175, 181–182, 184–187, 199, 406Dose-response curves , 4, 126, 164Down-regulation of receptors , 39Doxorubicin, 30, 486, 488, 493–494, 497–498. See also Anticancer drugs , anti tumor antibioticsDoxycycl ine, 400, 465–466, 484Doxylamine, 80, 126–127, 374, 517. See also DiphenhydramineDPP4 inhibi tors . See Antidiabetic drugs .Dronabinol , 163, 186Droperidol , 145, 181

long QT syndrome and problems with, 181Drugs . See also specific drugs

age affecting therapy with, 32bioavai labi l i ty of, 29, 30–31, 39clearance of, 5–6, 14, 17, 21, 24, 32–33, 35–36, 39–40, 43–44, 169displacement of bound, 39dose-response curves for, 4efficacy vs potency of, 26el imination rate constant of, 10, 34–35, 43–44excess ive doses of, 237extraction ratio of, 25, 36hal f-l i fe of, 8, 10, 18, 39hepatic clearance of, 32ionization of, 23–24loading dose ca lculations for, 14, 40maintenance dose ca lculations for, 5, 18, 205margin of safety of, 167, 343. See also Therapeutic indexmetabol i sm, 19, 38, 475“off-label” use of, 11, 37, 48ora l vs intravenous adminis tration, 6–7OTC regulations , 20, 47as partia l agonis ts , 16, 42recognizing class based on generic name, xxvi i–xl i itherapeutic index of, 167, 338time to reach s teady-s tate concentration of, 5, 10, 18volume of dis tribution of, 1, 11, 14, 17–18, 27, 33

DSHEA. See Dietary Supplement Heal th and Education ActDuctus arteriosus , patent, 353, 367–369, 441

EEchothiophate, 58, 90–91, 98. See also Acetylchol inesterase inhibi torsEdrophonium, 78, 102, 122–123, 268, 281, 283. See also Acetylchol inesterase inhibi tors

for di fferentia l diagnos is of myasthenic, chol inergic cri s i s , 78, 122–123Efavi renz, 469Emes is , drugs for, 161, 186, 199, 399, 406

ondansetron, 133, 163, 186, 391, 394, 405, 488Emetogenic drug. See ApomorphineEnalapri l , 224, 274. See also Angiotens in-converting enzyme inhibi torsEndothel ium removal , ACh and, 210, 247–248Endothel ium-derived relaxing factor (EDRF). See Ni tric oxideEntacapone, 87, 194–195, 199Enterochromaffin-l ike cel l s , 397Ephedrine, 70, 86, 108–110, 117, 164, 179, 241, 462, 470, 483Epi lepsy/seizures , 136, 140, 148, 151–152, 176, 188–189, 198, 385, 417. See also Anticonvulsant drugs

bupropion caus ing, 197–198ethosuximide for, 151, 189phenytoin for, 136, 140, 189therapy of during pregnancy, 152valproic acid for, 140

Epinephrine, 19, 42, 58, 60, 63–64, 74, 82, 85, 88, 90, 95, 97, 99–101, 103, 107–108, 118, 120, 124–125, 213, 242–244, 249, 254, 272, 275, 284–285, 291–292,335–336, 339–340, 344, 358, 427, 434, 521anaphylaxis and, 64, 100, 103loca l anesthetics and, 64, 100–101

Eplerenone, 300, 313–314, 316–317, 319, 323, 441. See also Spi ronolactoneEpoetin a l fa , 225, 276

Page 270: Pharmacology pre test self assessment and review (14th ed.)

Epson sa l t, 398Erecti le dys function

drugs for, 312s i ldenafi l , 218, 264–265tadalafi l , 264vardenafi l , 264

spironolactone caus ing, 312Ergonovine, 293, 411Ergotamine, 189–190, 292–293

myocardia l i schemia from overdose of, 231, 292–293Erythromycin, 463–464, 468, 470–472, 474, 483, 495. See also Antibiotics , macrol ides

long QT syndrome and, 464warfarin and, 455, 474Escherichia coli, 388, 400–401, 462, 464, 474, 482

Esci ta lopram, 196, 200, 211, 251, 501Esmolol , 94, 230, 232, 283, 286. See also β-Adrenergic blockersEsomeprazole, 225, 383, 394–395

for gastroesophageal reflux disease, 383, 394–395Esophagi ti s , 419, 441Essentia l hypertens ion. See Hypertens ionEstrogen, 407, 422

hypertens ion and excess , 417, 440thromboembol ism and, 416, 438

Etanercept, 358, 379Ethacrynic acid, 297, 308, 310–312, 318, 321–323, 395, 478

ototoxici ty and, 321Ethambutol , 450, 461–462, 466, 481, 483–484Ethanol , 155, 196

benzodiazepines in withdrawal from, 155, 193–194, 507, 519–520cephalosporins interacting with, 454, 470disul fi ram interacting with, 154, 195

Ethosuximide, 151, 189Ethylene glycol poisoning, 503, 514–515Etidronate, 419, 441, 445Etopos ide, 30, 497Exenatide, 414, 432, 434–435Extraction ratio, 25, 36Ezetimibe, 226, 278

FFamotidine, 391, 397, 401. See also Antihis tamines , his tamine receptor antagonis tsFDA. See Food and Drug Adminis trationFebuxostat, 246, 348, 359, 366, 369, 490, 499–500. See also Al lopurinol ;Antihyperuricemic drugsFentanyl , 156, 168, 173, 181, 197, 391

naloxone to treat excess ive effects of, 156, 197Ferrous sul fate, 225, 276, 395, 520Festoterodine, 57, 88–89Feta l respi ratory dis tress syndrome, 418, 440Fexofenadine, 314, 360, 367, 374, 378

diphenhydramine compared to, 354, 373–374Fibrates , 278Fibrinolytic drugs . See ThrombolyticsFi lgrastim, 488, 498Finasteride, 409, 426Flavoproteins (FP), 41–42Fleca inide, 216Fluconazole, 458, 479Fludrocorti sone, 416, 438Flumazeni l , 137, 140, 146, 167, 169, 172–173, 182, 197, 507, 519Fluni trazepam, 146, 160, 182, 519Fluoroquinolones . See Antibiotics , fluoroquinolonesFluoxetine, 37, 173, 181, 187, 195, 198, 200, 273, 307, 368, 489, 499

serotonin reuptake and, 196, 200sumatriptan and, 139, 173, 352, 368

Page 271: Pharmacology pre test self assessment and review (14th ed.)

Flurazepam, 194Flutamide, 487, 496Fol ic acid (folate)

deficiency, 396pregnancy and, 402supplements , 152, 192–193, 402synthes is and inhibi tors , 453, 470

Food and Drug Adminis tration (FDA), 11, 20–21, 37, 46–48, 96, 108, 144, 151, 191, 333, 435, 477, 489nutri tional supplements and, 20, 46pregnancy class i fi cations and, 21, 192promethazine warning of, 151, 191sa lmeterol warning of, 333

Fosphenytoin, 176, 189Fulvestrant, 486, 494–495Furosemide, 205, 216, 221, 224, 230, 234, 240, 249, 255, 259, 264, 271–272, 274, 289, 298–300, 308–312, 317, 319–324, 395. See also Diuretics ; Loop diuretics ;

Ototoxici tyaminoglycos ide with, 304, 321coronary artery disease, digoxin with, 215, 259heart fa i lure, thiazide replaced by, 297, 308–309hypokalemia from, 259, 274hyponatremia and, 304, 322hypovolemia from, 298, 311–312identi fying use of, 305

GGABA. See γ-Aminobutyric acidGalactorrhea, 135, 145, 170, 172, 181, 199Gal l bladder disease

gal l s tones dissolved by drugs in, 399opioids in, 171

Gangl ionic blockade, 59, 94, 115Gastric acid-antisecretory drugs

cimetidine, 349, 361, 383, 393, 416, 438esomeprazole, 225, 382–383, 392–394, 396famotidine, 382, 391nizatidine, 382, 391–392omeprazole, 392, 524rani tidine, 382, 388, 391–392, 401

Gastric ulcers , 388, 400

Page 272: Pharmacology pre test self assessment and review (14th ed.)

Gastroesophageal reflux disease (GERD), 383, 386esomeprazole for, 382, 392

Gentamicin, 321, 454, 456–459, 468, 472–474, 478, 480–481. See also Antibiotics , aminoglycos idesamikacin compared to, 460, 475anaerobic bacteria and, 465, 469, 482

GERD. See Gastroesophageal reflux diseaseGlaucoma, drugs and, 58–59, 62, 80, 90–93, 97–98, 103, 127, 202, 240, 301, 309, 316, 318, 332, 343–344, 360, 374Gl imepiride, 442. See also Antidiabetic drugsGl ipizide, 442, 444–445. See also Antidiabetic drugsGl i tazones . See Piogl i tazoneGLP-1. See Glucagon-l ike peptide-1Glucagon, 25, 241, 425, 427, 434–435, 442Glucagon-l ike peptide-1, 427, 434Glucocorticoids , 342, 351, 366, 431, 438, 440. See also specific individual agentsGlucose tolerance tests , 307, 426Glutathione, 362, 375, 513Glyburide

hypoglycemia and, 407, 420insul in release and, 419, 442

Glycopyrrolate, 71, 111–112Glycosuria , 300, 314, 321Gout. See Antigout/antihyperuricemic drugsGram-negative bacteria , 458, 475, 477Grapefrui t juice, 9, 33–34Graves disease, 420Guanadrel , 63, 98–99

HH1 blockers . See Diphenhydramine, his tamine receptor antagonis ts

H1N1 vi rus , 457, 477H2 blockers . See “antihi tamines , his tamine receptor antagonis ts”

Haemophilus influenzae, 453, 474, 483Haloperidol , 147, 181, 184, 186. See also Antipsychotic drugs , butyrophenones

chlorpromazine compared to, 138, 171Halothane, 161–162, 191. See also Anesthetics , genera lHay fever, 357HbA1c. See Hemoglobin A1c

Headaches , 26, 37, 60, 237, 295, 312, 349sumatriptan mimicking serotonin for, 349, 360

Hearing loss . See also Ototoxici ty aminoglycos ides and, 459, 479–480cisplatin and, 488, 498–499loop diuretics and, 318

Heart block, 216, 264verapami l caus ing, 220, 269–270

Heart fa i lure, 64, 66, 71, 102, 105, 112–113angiotens in converting enzyme inhibi tors and, 279β-blockers and, 241, 253, 257, 279, 288chlortha l idone replaced by furosemide for, 295, 307–308digoxin and, 202, 211, 230, 239, 253–254, 286, 319furosemide caus ing hypotonic urine in treating, 299, 313gl i tazones and, 419, 442quinidine and, 214, 259–260

Heart fa i lure, drugs forβ-adrenergic blockersacebutolol , 16, 42atenolol , 70, 109, 206, 244, 425metoprolol , 58, 70, 92, 109, 222, 232, 425nadolol , 58, 62, 92, 97nebivolol , 241pindolol , 58, 67, 92, 107propranolol , 55, 60, 63, 70, 73, 86, 94–95, 99, 105, 110, 114, 119, 126, 224, 228–232, 254, 284, 286, 425timolol , 58, 62, 92, 98afterload reducers . See Antihypertens ive drugs ; Diuretics ; Furosemideangiotens in modi fiers

captopri l , 202, 210, 212, 222, 226–228, 255–256, 270–271, 282, 323, 352, 355, 367, 374

Page 273: Pharmacology pre test self assessment and review (14th ed.)

enalapri l , 224, 275l i s inopri l , 225, 278, 305losartan, 206, 226, 244, 278ramipri l , 211, 230

inotropesdigoxin, 30, 205, 214–215, 222, 224, 230, 240, 255, 257, 259–260, 272dobutamine, 66, 68, 74, 104, 117–118, 232, 258, 285–286dopamine, 55–56, 63, 74, 86–87, 104, 117, 308norepinephrine, 39, 53–55, 63–64, 74, 82–84, 92, 98–99, 103, 119, 124, 186

Helicobacter pylori, 399–400Hemoglobin A1c, 201Hemolytic anemia, drug-induced, 212, 225, 254, 277, 515Hemorrhagic s troke, 222, 233, 272, 283, 346Henderson-Hasselbach equation, 23, 370, 512Heparins , 205, 217, 225, 244

dangers from use with thrombolytics , 279protamine sul fate counteracting, 217, 263

thrombocytopenia induced by, 243, 250, 276Hepatic ci rrhos is , 299, 301, 313Hepatic clearance of drugs , 2, 32, 392Herbal supplements , regulation by FDA of, 20, 46–47Hexamethonium, 110, 114His tamine, 19, 25, 349, 352, 354, 360His tamine receptor antagonis ts . See Antihis taminesHIV/AIDS, 450, 469. See also Antivi ra l/anti retrovi ra l drugs

zidovudine and, 448, 452, 460, 463–464, 469, 476Homatropine, 76, 120–121, 125Hydra lazine

for hypertens ion, 214, 257unwanted cardiovascular responses of, 256vasodi lator effects of, 219, 264

Hydrochlorothiazide, 223, 230, 244, 253, 255, 273, 296–297, 310–313, 317–318, 320, 322, 324–325, 424, 429. See also Thiazide diureticshypertens ion and, 305hyponatremia and, 304, 322–323identi fying use of, 325

Hydrocodone, 342Hydrocorti sone, 416, 446Hydroxychloroquine, 351, 365, 447. See also Antimalaria lsHypera ldosteronism, 312, 319, 442, 446Hyperammonemia, 316Hypercholesterolemia, 76, 218, 223, 226, 231, 251, 288, 297, 300, 436. See also Antihyperl ipidemic drugsHypercorti sm, 416Hyperglycemia, 113, 289, 307, 314, 342, 410, 424–425, 429, 467, 481Hyperka lemia, 152, 191, 221, 248, 257, 271, 281, 283, 312, 320Hyperl ipidemias , 221, 234, 251, 281, 290, 443, 467. See also Drugs for hyperl ipidemiasHyperparathyroidism, 410, 428Hyperprolactinemia, 149, 186, 190, 199, 293Hypertens ion. See also Antihypertens ive drugs

drug induced, 59, 93emergency management of, 60essentia l , 58, 60, 62, 76, 91, 96–97, 203, 207, 213–214, 217–218, 221estrogen and, 417mal ignant, 46methyldopa for, 248, 253–254pheochromocytomas and, 60, 95–96

Hyperthermia. See Mal ignant hyperthermiaHyperthyroidism

β-adrenergic blockers and, 67, 105anti thyroid drugs for, 430–431high dose iodides for, 411, 429, 436radioiodine for, 411, 431thyroid s torm/thyrotoxicos is and, 105

Hyperuricemia, 207, 226, 233, 246, 297, 348, 350, 353, 356, 359, 364, 378, 437, 490. See also Antigout/antihyperuricemic drugsHypervi taminos is D, 445Hypocalcemia, 436

Page 274: Pharmacology pre test self assessment and review (14th ed.)

Hypoglycemiaacarbose and, 421, 444metformin and, 412, 432sul fonylureas caus ing, 432–433thiazol idinediones caus ing, 442

Hypokalemiaacetazolamide caus ing, 304, 323ami loride for, 303, 321digoxin and, 303, 319–320furosemide caus ing, 224, 274

Hyponatremiadi lutional , 322furosemide and, 304, 322–323hydrochlorothiazide and, 304, 322l i thium and, 223, 273

Hypotens ion. See also Orthostatic hypotens ionprazos in inducing, 79, 126propofol caus ing, 132, 160reserpine inducing, 79, 126

Hypothyroidism, 21, 59, 93amiodarone and, 411, 429–430levothyroxine and other thyroid hormones for, 414

Hypotonic urine, 295, 300Hypovolemia, 227, 249, 253, 283, 298, 311, 495

from furosemide, 298, 310–312Hypoxanthine, 348, 360, 498

IIbuprofen, 26, 246–247, 351, 357, 361, 367, 441

aspirin taken with, 208, 246Imatinib, 487, 495Imipramine, 70, 84, 86, 116, 119, 135, 155–156, 173, 181, 196–198, 517

nocturnal enures is and, 156, 197Incretin mimetics , 414, 434–435. See also Antidiabetic drugs .Indomethacin, 350, 352, 355, 357, 359, 361, 367, 369, 374–377, 441. See also Anti -inflammatory drugs , nonsteroida l

angiotens in converting enzyme inhibi tors affected by, 347, 441patent ductus arteriosus closure with, 353, 368ulcer prophylaxis , misoprostol with, 349

Inflammatory bowel disease, 389, 403Inos i tol tri sphosphate (IP3), 2

INR. See International Normal ized Ratio, warfarinInsul in, 90

glargine, 413, 434gl i tazones and receptor sens i tivi ty to, 422, 446glyburide and release of, 420, 444meals and release of, 425molecular modi fication of, 413, 433NPH, 433piogl i tazone and sens i tivi ty to, 421, 444

International Normal ized Ratio, 219, 268cephalosporins prolonging, 456warfarin effects on, 225, 269, 272, 455

Intrins ic sympathomimetic activi ty of β-blockers , 67, 92, 107, 254, 425Iodines/iodides , 411, 429, 431, 436, 445Ipecac, 386, 398Ipratropium, 62, 96, 329, 337

chronic obstructive pulmonary disease and, 62, 329, 337–338Iron, 385, 395Irri table bowel syndrome, 387, 390, 400, 404Isoflurane

halothane compared to, 191mal ignant hyperthermia and, 191

Isoniazid, 480, 483ethambutol with, 461, 483–484pyridoxine and, 385, 396

Page 275: Pharmacology pre test self assessment and review (14th ed.)

tuberculos is , vi tamin B6 and, 459, 480

Isoproterenol , 57, 63–64, 73, 75, 89, 99–100, 103, 107–108, 116, 119, 258, 281, 292, 344. See also β-Adrenergic agonis tseffects of, 73, 116s igns of us ing, 75, 119s inus bradycardia and, 228, 281

Isotretinoin, 403

JJimson weed, 516. See also antimuscarinic drugs , atropine

KKernicterus from sul fonamides , 463Ketamine, 136, 146, 168, 183Ketoconazole, 416, 422, 431, 438, 446, 466

Cushing disease and, 416, 422, 438, 446Ketorolac, 72, 133, 161–163, 391Kininase II . See Angiotens in converting enzymeKorsakoff psychos is , 402

LLabeta lol , 91–92, 94–95, 110, 205, 239–241, 245, 249, 254, 260, 271, 275, 286, 425. See also β-Adrenergic blockersβ-Lactamase (penici l l inase), 458, 468, 471, 474, 481, 483

ceftriaxone res is tant to, 462, 483inhibi tors of, 471oxaci l l in res is tant to, 451, 468

Lactic acidos is , 258, 418, 421, 432, 439–440, 444, 467, 510, 513Lactulose, 384, 394Laryngospasm, 103, 132, 161

barbi turates and, 132, 161Lead poisoning, 520Leucovorin, 485, 491–492Leukemia, 456, 487–491, 493, 495, 498, 511, 516Leukotrienes , 336, 338–339, 345, 347, 358–359, 366

montelukast, 329, 338–339zafi rlukast, 338

Leuprol ide, 419, 441, 487, 496flutamide with, 487, 496

Levodopa. See also Antiparkinson drugs , dopaminergiccarbidopa used with, 134, 154, 164, 195pyridoxine counteracting, 385, 396

Levofloxacin, 453, 469, 471Levothyroxine, 409, 414, 418, 421, 435. See also hypothyroidism.Lewis i te, 518. See also British anti-LewisiteLHRH. See Luteinizing hormone-releas ing hormoneLidoca ine, 32, 177, 202, 228, 237, 240, 242, 281, 292, 510. See also Anesthetics , loca l ; Antiarrhythmic drugsLigands , 2, 25

binding of, 45Linear kinetics , 18Linezol id, 449, 452–453, 456, 466, 468, 470, 472, 483

vancomycin-res is tant enterococca l infections and, 449, 466Liothyronine, 414, 435, 445. See also Thyroid hormonesLiotrix, 414, 435. See also Thyroid hormonesLis inopri l . See also Angiotens in-converting enzyme inhibi tors

losartan compared to, 226, 277–278triamterene with, 296, 305

Li thium, 139, 156, 172, 198, 223, 273, 307hyponatremia and, 223, 273polyuria/polydips ia from, 307

Livedo reticulari s , 174Liver fa i lure, 350, 362Loading dose, 5, 14, 16, 18, 40, 44, 205, 243–244Local anesthetics . See Anesthetics , loca l ; Lidoca ine; Proca ineLong QT syndrome, drugs pos ing ri sks in, 181, 252, 464Loop diuretics , 249, 255–256, 264, 271, 289, 299, 308, 310–315, 317, 319–325, 375, 395, 478. See also Furosemide

urine electrolyte profi les and, 306urine potass ium concentration and, 298, 311

Page 276: Pharmacology pre test self assessment and review (14th ed.)

Loperamide, 384, 394Lopinavi r, 460, 469, 476, 480–481Loratadine, 125, 173, 357, 374, 378Lorazepam, 32, 140, 142, 160, 176, 189. See also Benzodiazepines

barbi turate/a lcohol withdrawal and, 150, 155, 196status epi lepticus and, 142, 176

Losartan, 206–207, 244, 248, 256l i s inopri l compared to, 225, 277–278

Lugol solution. See Strong iodine solutionLuteinizing hormone-releas ing hormone, 441, 496

MMAC. See Minimum a lveolar concentration of inhaled anestheticsMagnes ium sa l ts , 386, 397–398, 401

aluminum sa l ts with, 386, 398renal disease and dangers of, 404

Magnes ium sul fate, 386, 397–399, 401Maintenance dose and i ts ca lculation, 5, 18, 205, 244Malathion, 65, 102Mal ignant hypertens ion, 19, 46Mal ignant hyperthermia, 46, 132, 161–162, 191–192

dantrolene for, 191drugs caus ing, 150, 191

Mannitol , 299, 315cardiac function and, 302, 318fluid el imination increased with, 321

MAO. See monoamine oxidaseMarfan syndrome, 234Margin of safety. See Therapeutic indexMechlorethamine, 487, 497Megaloblastic anemia, 396Meniere disease, 313Meningi ti s , 458, 462, 479, 483Menopause, 400Menstrua l i rregulari ties , 181, 439Meperidine

interactions with monoamine oxidase inhibi tors and, 144, 181morphine compared to, 138, 171

Mercaptopurine, 348, 359, 489, 499–500Mesalamine, 403Metabol ic syndrome, 41, 233Metformin, 211, 251, 289, 407, 412, 415, 418–420, 427, 431–432, 434–435, 439–440, 443–444

appeti te suppressed by, 432diabetes mel l i tus and, 439hypoglycemia and, 412, 432lactic acidos is from, 418, 421, 444polycystic ovarian syndrome and, 412, 431

Methadone, 150, 171, 189, 194, 466Methamphetamine, 89, 144Methanol poisoning, 155, 196–197, 506, 514, 518Methici l l in-res is tant staphylococcus aureus, 454, 466, 483Methohexi ta l , 132, 161Methotrexate, 365, 485, 491, 497–498

cancer and, 485, 491leucovorin rescue and, 492

Methyldopa, 212, 253–254, 512Coombs-pos i tive test and, 254

Methylene blue, 511, 514Methylergonovine, 430, 440Methylphenidate, 86, 107, 124, 138, 170Methylprednisolone, 331, 416Methylxanthines , 329, 337, 342–343, 346. See also Theophyl l ine

overdoses of, 346Metoclopramide, 186, 389, 402Metolazone, 76, 239, 244, 255, 273, 299, 307–308, 312

Page 277: Pharmacology pre test self assessment and review (14th ed.)

hypertens ion and, 299, 311Metoprolol , 58, 70, 92, 105, 109, 286, 408, 425

angina or hypertens ion and, 202, 240Metronidazole, 387, 399–400, 465, 468–470, 473Metyrapone, 410, 428, 446Midazolam, 68, 109, 132, 160–161, 168, 173, 181, 331, 341, 391, 459.

See also Benzodiazepines amnestic effects of, 132Mifepris tone, 30, 369, 419, 430, 441Migl i tol , 433, 443Migra ine headaches , 174, 189, 200, 237, 349, 352Mi lk of magnes ia . See Magnes ium sa l tsMinimum a lveolar concentration of inhaled anesthetics , 141, 163, 175Minipi l l s , 417, 422, 437, 439Misoprostol , 353, 361, 367, 369, 393, 397, 400, 430

abortion with mifepris tone and, 353, 369contra indications for, 369cytoprotective/mucotropic effects of, 361for NSAID-induced ulcer prophylaxis , 349, 361

Monoamine oxidase (MAO), 15, 42, 55–56, 58, 104–105, 110, 134, 145, 179, 181, 183, 200, 470, 483depress ion, inhibi ting, 144, 178–179inhibi tors of, 42, 70, 86, 104, 110, 134, 144, 164, 173, 178–180, 182–184, 195, 199, 200, 483metabol ic inactivation by, 74, 119types of, 135

Monoamine oxidase inhibi tors , 42, 70, 86, 104, 110, 134, 144, 164, 173, 178–180, 182–184, 195, 199–200, 483and depress ion, 144, 178–179drug interactions involving, 167fluoxetine, other SSRIs , and, 172, 182–183and hypertens ion, 179meperidine and, 145, 182pargyl ine as , 55, 86

Montelukast, 329, 338–339, 345, 359Mood s tabi l i zing drugs

antidepressants as , 198l i thium as , 198

Morphine. See also Opioidsclosed-head injury and, 149, 187drugs antagonizing effects of, 185meperidine compared to, 138, 170postoperative pa in control , switching to pentazocine from, 147, 184–185

Motion s ickness , 59, 108, 116, 405, 517MRSA. See Methici l l in-res is tant s taphylococcus aureusMTX. See MethotrexateMultidrug res is tance-associated protein, 29Muscarinic receptors , 51, 62, 71–72, 82–84, 89, 93–94, 97–98, 103, 111, 113–116, 120–126, 149, 159, 171–172, 178, 184, 187, 195, 197, 247–248, 258, 281, 291–

292, 343, 374, 397, 401, 521. See also Chol inergic receptorsMyasthenia gravis , 65, 78, 102, 122–123, 344Myasthenic cri s i s , 65, 78, 122–123

chol inergic cri s i s compared to, 65, 78, 122–123di fferentia l diagnos is of, 65, 123

Mydrias is , 74, 79, 90, 92, 99, 115, 118–120, 124, 228Myelosuppress ion, 467, 472Myocardia l infarction, 217, 220, 227, 233, 239, 250

antiplatelet drugs and, 250aspirin in, 233, 290heparin and, 217, 250nitrates and, 277thrombolytics and, 227warfarin and, 220, 250

Myocardia l i schemia, 105, 141, 151, 174, 190, 206, 229, 231, 250, 265, 277, 284, 293, 434, 444ergotamine overdose caus ing, 237, 293

Myoglobinuria , 208Myopathy, drug-induced

from daptomycin, 454, 472from statins and other l ipid-lowering drugs , 247, 278

Page 278: Pharmacology pre test self assessment and review (14th ed.)

NN-acetylcysteine, 354, 370

acetaminophen overdose and, 362, 518mucolytic effects of, 340, 492

Nadolol , 58, 62, 92, 97. See also β-Adrenergic blockersNADPH. See Nicotinamide adenine dinucleotide phosphateNa +-K+-ATPase. See Sodium-potass ium adenos ine triphosphataseNaloxone, 156–157, 171, 183, 188–189, 197, 398, 520

antagonism of opioid effects by, 156, 197duration of action compared with opioid agonis ts , 197

Naltrexone, 140, 173, 188, 386, 398Naproxen, 246–247Narrow angle glaucoma. See GlaucomaNebivolol , 203, 241, 286. See also β-Adrenergic blockersNeomycin, 460–461, 481–482Neostigmine, 57, 62, 71, 73, 89, 98, 110–112, 114–116, 119Nephropathy, drug-induced, 238, 252, 283, 297, 303, 310, 362Nerve gases , 103, 124, 173, 509, 511, 514, 521Neuroleptic drugs . See Antipsychotic drugsNeuroleptic mal ignant syndrome, 46, 157, 175, 183, 191–192, 200

antipsychotic drugs and, 175, 184, 200bromocriptine for, 157, 199diagnos ing, 157

Neuromuscular blockers , 84, 97, 107, 110, 112, 114, 161–162, 168, 191, 459, 479depolarizing, 84, 93, 111, 168nondepolarizing, 84, 107, 110, 114, 161, 180, 459, 479

Neutropenia , 250, 448Niacin, 211, 234, 251, 278, 288, 290, 408, 424

prostaglandins and, 234, 290Nicotinamide adenine dinucleotide phosphate (NADP), 15, 40Nicotine, 59, 83, 93–94, 198, 415Nicotinic acid. See NiacinNicotinic receptors , 46, 83–84, 94, 97, 99, 116, 122–124, 479, 521. See also Chol inergic receptorsNi fedipine, 139, 172, 216, 223, 239, 242, 250, 256, 259, 262, 266, 270, 272–273. See also Ca lcium channel blockers

abrupt discontinuation of, 271nondihydropyridine ca lcium channel blockers compared to, 242, 270ris ing blood pressure and, 223, 274

Nitric oxide, 93–95, 121, 245, 248, 264, 266, 272, 282, 367and endothel ium, 121, 247, 371nebivolol and, 203, 241synthes is by ni tric oxide synthase, 60and vasodi lators , 256, 264, 266

Nitrofurantoin, 448, 463–464Nitroglycerin, 202, 206, 223, 232, 240, 253, 257, 259, 277, 282, 285

acute coronary syndrome and, 285angina and, 202, 218, 240, 277erecti le dys function drugs interacting with, 312myocardia l infarction and, 231tolerance to, 225, 277transdermal adminis tration of, 225

Nitropruss ideantihypertens ive effect of, 234cyanide poisoning from, 263, 277, 511, 513

Nitrous oxide, 141, 175, 181, 459Nitrovasodi lators . See Ni troglycerinNizatidine, 382, 391–392, 397, 401NMS. See Neuroleptic mal ignant syndromeNNRTIs . See Non-nucleotide/nucleos ide reverse transcriptase inhibi tors ; Efavi renzNocturnal enures is , 156, 197Nondihydropyridine, 216, 240, 242, 244, 250–251, 255, 262, 270, 273–274, 283, 286Non-nucleotide/nucleos ide reverse transcriptase inhibi tors , 469Nonsteroida l anti -inflammatory drugs , 10, 42, 283, 290, 298, 339–340, 441

antagonism of other drugs by, 302, 317aspirin as prototype of tradi tional , 41, 340cyclooxygenases inhibi ted by, 340

Page 279: Pharmacology pre test self assessment and review (14th ed.)

Noradrenergic nerves . See Adrenergic nervesNorepinephrine, 19, 39, 42, 53–56, 63, 66, 74, 82–84, 86–88, 92, 99, 103–105, 107–108, 118–120, 126, 155, 164, 170, 181, 186, 196

as adrenergic agonis t, 328, 335cocaine and tricycl ic antidepressant effects on reuptake of, 82, 84, 86, 104, 109, 119, 122, 186depletion of intraneuronal , 55, 85, 99phentolamine with, 74, 117reuptake of, 51, 55, 66, 74, 82, 84, 86, 104, 109, 119as sympathetic neurotransmitter, 74, 82, 92synthes is of, 54, 85, 87

Normeperidine, 171, 182NPH insul in, 432–434NRTIs . See Nucleos ide reverse transcriptase inhibi tors ; ZidovudineNSAIDs . See Nonsteroida l anti -inflammatory drugsNTG. See Ni troglycerinNucleos ide/nucleotide reverse transcriptase inhibi tors , 460, 467, 469. see also ZidovudineNutri tional supplements/nutriceutica ls , 20, 46–47Nystatin, 458

OOctreotide, 408, 425“Off-label” drug use, 11, 37, 48Olanzapine, 187, 193, 200Omal izumab, 328, 336–337Omeprazole, 396, 524Ondansetron, 30, 133, 163, 186, 390–391, 394, 405, 488

cancer chemotherapy and, 390, 405Opioids . See also specific individual drugs

antagonis ts of, 129, 172, 188, 195naloxone, 171, 183, 188, 197nal trexone, 173, 386, 398codeine, 144, 173, 180, 261, 341fentanyl , 156, 168, 173, 181, 197, 391hydrocodone, 342meperidine, 32, 138, 170–171, 182methadone, 150, 171, 189, 194, 466morphine, 26, 72, 109, 113, 138, 147, 149, 161–162, 171, 179, 184–185, 187–190, 197, 199, 202, 261, 358, 391, 405, 512, 522overdoses of, 173oxycodone, 144pentazocine, 147, 184–185receptors (μ and k) for, 173, 184respiratory depress ion by, 171, 184–185withdrawal from, 188

Orthostatic hypotens ion, 76, 120, 146, 153, 183, 186chlorpromazine and, 153, 193

Osel tamivi r, 457, 476Osteoporos is , 410, 419, 427. See also BisphosphonatesOTC drugs . See Over-the-counter drugsOtotoxici ty, 269, 321, 458

aminoglycos ides and, 455–461, 468, 472loop diuretics and, 321

Over-the-counter drugs , 20, 60, 106Oxaci l l in. See also Antibiotics , penici l l ins

penici l l in and, 451, 468Staphylococcus aureus and, 451

Oxybarbi turates . See Phenobarbi ta lOxycodone, 144Oxytocin, 293, 425

PP450. See Cytochrome P450Pacl i taxel , 488, 497–498, 525Paget disease, 410, 427–428, 436Pancrel ipase, 389, 402–403Pancuronium, 53, 68, 72, 107, 114Parasympathetic nervous system, control of effectors , 114, 281Parathyroid hormone, 415, 423, 436, 446

Page 280: Pharmacology pre test self assessment and review (14th ed.)

Pargyl ine, 55, 86, 110, 179Parkinson disease, 88, 93, 134, 140, 143, 147, 151, 154, 157, 175, 198

bromocriptine and, 149, 157, 190, 199ropini role and, 142, 175selegi l ine and, 147, 199trihexyphenidyl in drug plan for, 143, 178, 185

Parkinsonism, 88, 93, 116, 147–148, 157, 164, 178, 184, 190, 195selegi l ine and, 147, 184–185

Paroxysmal supraventricular tachycardiaadenos ine for, 219, 268verapami l for, 202, 239

Partia l agonis ts , 16, 42, 92, 107, 184, 254Patent ductus arteriosus , 353, 368PCOS. See Polycystic ovarian syndromePCP. See Phencycl idinePenici l lamine, 351, 365, 492Penici l l inase. See β-LactamasePenici l l in-binding proteins , 477Penici l l ins , 32, 54, 451, 456–458, 460, 475–477, 480. See also Antibiotics , penici l l ins

aminoglycos ides and, 458, 478–479mechanism of action of, 466, 477oxaci l l in and, 451, 468vancomycin and, 451, 467–468

Pentamidine, 465, 469Pentazocine, 147, 184–185. See also Analges ics , opioidsPeriphera l nervous systems, 49–127Periphera l vasodi lation, 245, 262, 267Peroxisome prol i ferator-activated receptors , 444P-glycoproteins , 6, 28, 30

inducers and inhibi tors of, 29substrates/inducers/inhibi tors of, 29

Pharmacokinetics , 5, 6, 9–10, 13–14, 29, 32, 71, 111, 160, 162, 169–170, 475Pharmacology, genera l principles of, 1–48Phase I metabol ic reactions , 19, 45Phase II metabol ic reactions , 8Phencycl idine (“angel dust” PCP), 139, 172Phenelzine, 164, 179, 181Phenobarbi ta l , 135, 166, 170, 174, 192, 195, 370, 438, 510, 512, 518Phenothiazine antipsychotics . See ChlorpromazinePhenoxybenzamine, 88, 190, 204, 213, 243, 254

blood pressure control , prazos in compared to, 283pheochromocytomas and, 213, 254

Phentolamine, 65, 88–89, 101–102, 117–118, 159, 241, 275, 281, 289, 360norepinephrine with, 117prazos in compared to, 65, 102

Phenylephrine, 79–80, 88, 117, 119, 125–126, 129, 159–160, 172, 203, 241–242, 258, 268, 280, 283prazos in blocking effects of, 227, 280supraventricular tachycardia and, 230–231, 242

Phenylethanolamine N-methyl transferase, 56, 88Phenytoin, 136–137, 152, 166, 168–169, 176, 189–190, 194. See also Anticonvulsant drugs

epi lepsy/seizures and use of, 135–136, 142, 152, 156, 166ora l contraceptive efficacy reduced by, 417–418, 437, 439–440pharmacokinetics of, 169pregnancy and, 152

Pheochromocytomas , 60, 95–96, 190, 204, 213, 224, 284–285effects of β-blockers in, 95–96, 243, 254, 285phenoxybenzamine and, 213, 254

Phosphate, a luminum sa l ts and, 388, 392, 404Phosphol ipase A2, 357Photophobia , drug-induced, 75, 120, 360Physostigmine, 98, 102, 112, 115–117, 123, 173, 360, 378, 516Phytonadione. See Vi tamin KPi locarpine, 59, 62, 64, 76, 94, 97–98, 121, 125, 332, 343Pindolol , 58, 92, 425

intrins ic sympathomimetic activi ty and, 67, 107

Page 281: Pharmacology pre test self assessment and review (14th ed.)

Piogl i tazone, 420–421, 427, 440, 442–444. See also Antidiabetic drugsinsul in sens i ti zing effects of, 443

Pitui tary tumor, 190, 293, 438Pitui tary-adrenal cortica l axis , 428Plasma concentration. See Steady-s tate concentrationPlasmodium vivax, 449, 450Platelet-activating factor (PAF), 279Pl icamycin, 415, 436Pneumococci , 453, 471Poison syndromes

antichol inergic/antimuscarinic, 106–107, 111, 115–116, 121, 383, 512, 516chol inesterase inhibi tor, 504, 515, 517, 521

Polycystic ovarian syndrome, 412, 431Polydips ia , 307, 422, 445Polyuria , 307, 323, 422, 445Postgangl ionic sympathetic nerves , 51–52, 82, 84, 118Postoperative nausea, 133, 181Postoperative pa in control , 14, 133, 147

morphine, switching to pentazocine for, 147, 184–185Postsynaptic target cel l s , 82–83Potass ium-sparing diuretics

ami loride and triamterene as , 257, 271, 303, 307, 318, 321spironolactone and eplerenone as , 307, 311urine electrolyte profi les and, 306

Potass ium-wasting diuretics . See furosemide, hydrochlorothiazidePra l idoxime, 78, 103, 124, 173, 511Pravastatin, 211, 251Prazos in, 32, 65, 86, 88, 101–102, 107, 119, 240, 270, 275. See also α-Adrenergic blockers .

benign prostatic hypertrophy and, 228, 280blood pressure control , phenoxybenzamine compared to, 243hypotens ion from, 79, 126phentolamine compared to, 65, 102phenylephrine’s effects blocked by, 227, 280

Prednisone, 334, 336, 342, 428, 431, 434, 445–446. See a lso CorticosteroidsPregnancy

angiotens in modi fiers and, 249captopri l and, 210diuretics and, 249fol ic acid supplements and, 152, 192fol ic acid supplements during, 402phenytoin during, 152, 194vi tamin A and, 389, 403warfarin and, 212, 253

Pregnancy class i fi cations , 249Primaquine

hemolytic anemia from, 512Plasmodium vivax and, 449–450Principa l cel l s of nephron, 321

effects of potass ium-sparing diuretics on, 257, 281, 311–312, 317, 321, 323, 325sodium-potass ium exchange by, 299, 313–314

Probenecid, 350, 359, 363–364, 368–369, 377acute gout caused by, 350, 364aspirin impairing actions of, 353, 368

Proca inamide, 254, 269, 512lupus-l ike syndrome from, 257, 269, 502, 512

Proca ine, 177Progestin-only ora l contraceptives , 439Promethazine, 151, 185, 187, 190–191, 341Propafenone, 216, 524Propofol , 109, 132, 160–161, 181, 459Propoxyphene, 171, 518Propranolol , 32, 60, 86, 94, 110, 114, 116–117, 119. See also β-Adrenergic blockers

antihypertens ive mechanism of action of, 63, 99blood pressure control , ni tropruss ide with, 234pheochromocytomas and, 275

Page 282: Pharmacology pre test self assessment and review (14th ed.)

thyrotoxicos is and, 409, 434Propyl thiouraci l , 415, 420, 434, 436, 443, 445

anti thyroid effects of, 415, 421, 436, 445blood dyscras ias from, 186, 194

Prostacycl in, 41, 287, 339, 359–361Prostaglandins , 41, 162, 290, 311, 330, 339–340, 359–361, 367–369, 374, 397, 430, 441Prostate speci fic antigen, 409Prostatic hypertrophy. See also Benign prostatic hypertrophy

drugs contra indicated in, 360, 378drugs for treating, 217, 263, 276

Protamine sul fate, 225, 276heparin antagonized by, 217, 263

Protease inhibi tors , 439, 450, 460, 467, 469. See also Lopinavi r, ri tonavi rProthrombin time, 267–268. See also International normal ized ratioProti rel in, 414, 435Proton pump inhibi tors , 225, 387, 393, 394, 397, 400Pruri tus , 234, 290, 472Pseudoephedrine, 144, 180

as i l lega l source of methamphetamines , 144Pseudomembranous col i ti s , 400, 448, 455. See also Antibiotic-associated

pseudomembranous col i ti s , cl indamycinPseudomonas aeruginosa, 456–457, 459PSVT. See Paroxysmal supraventricular tachycardiaPTU. See Propyl thiouraci lPulmonary edema, 72, 188, 239, 289, 495Pulmonary fibros is , drug-induced, 211, 261, 269, 429, 493Pulmonary function tests , 269, 327, 334, 338, 343Pyridostigmine, 78, 102, 122. See also Acetylchol inesterase inhibi torsPyridoxine (Vi tamin B6), 165, 257, 385, 396, 459, 466, 480

isoniazid and, 385, 396levodopa/carbidopa counteracted by, 385, 396

QQuinidine. See also Antiarrhythmic drugs

adverse effects in long QT syndrome, 252atria l fibri l lation and, 211, 252–253heart fa i lure and, 214, 259reducing renal digoxin clearance, 259sa l icyl i sm resembl ing overdoses of, 242toxici ty, 216, 260, 269

Quinine, 242

RRadioiodine, 411, 431, 445Raloxi fene, 411, 431Ramipri l , 207, 211, 252, 255, 283

hyperka lemia and, 283Ranitidine, 225, 382, 388, 391–392, 397, 401Recognizing drug class by generic name, xxvi i–xl i i“Red man” syndrome and vancomycin, 472Renal artery s tenos is , 408Renal clearance of drugs , 13, 39, 43Renal potass ium loss , 271, 299, 313, 324Renin

β-adrenergic receptors inhibi ting, 58, 286and angiotens in-a ldosterone system, 256, 358release, 83, 91, 95, 118, 286

Repagl inide, 420, 443. See also Antidiabetic drugs .Rescue therapy in asthma

adrenergic bronchodi lators for, 213, 328, 335, 337Reserpine, 86, 98–99, 120, 126

guanadrel compared to, 63, 98–99hypotens ion from, 79–80, 120, 126

Respiratory acidos is , 332, 344, 370, 372, 375, 502, 512Restless leg syndrome, 142

Page 283: Pharmacology pre test self assessment and review (14th ed.)

Reuptake of norepinephrine, 155, 186Reye syndrome, 350, 364, 400–401

aspirin and, 350, 364Rhabdomyolys is , drug-induced, 247, 271, 278Rheumatoid arthri ti s

captopri l a ffected by indomethacin for, 355disease-modi fying anti rheumatic drugs and, 365, 491etanercept and, 357, 379

Rhinorrhea, 79, 357Ri fampin, 30, 439, 450, 466, 481, 483, 495Risperidone, 200Ritonavir, 439, 460, 469, 476, 480–481, 523. See also Protease inhibi torsRodenticides , 509, 521Rohypnol , 146, 160, 182, 519

flumazeni l antagonizing, 146, 182“Roofies .” See RohypnolRopini role, 140, 175, 176Ros igl i tazone. See Piogl i tazoneRosuvastatin, 208, 247. See also Antihyperl ipidemic drugs

SSAARDs. See Disease-modi fying anti rheumatic drugsSa l icyl i sm, 204, 242, 353, 368–369, 371

quinidine and, 242tinni tus and, 204, 242, 353, 368

Salmeterol , 113, 328, 333, 335–336, 345–346asthma and, 328, 335–336FDA warning on, 333

Saquinavi r, 30, 460, 525Schizophrenia . See also Antipsychotic drugs

atypica l antipsychotics for, 153chlorpromazine and, 148, 193chlorpromazine with benztropine for, 155, 198

Scopolamine, 59, 93–94, 107, 405, 417Sedative, hypnotic, anxiolytic drugs

barbi turates , phenobarbi ta l , 135, 166, 354, 370chlordiazepoxide, 155, 196diazepam, 32, 140, 173lorazepam, 140, 142, 160, 176midazolam, 68, 109, 132, 160–161, 181, 262, 341, 391, 459zolpidem, 166, 173, 174, 193

Seizure threshold, reduced by drugs , 176, 185Seizures . See Epi lepsy/seizuresSelective estrogen receptor modi fiers (SERMs), 431, 494, 495, 499Selective serotonin reuptake inhibi tors (SSRIs ), 200, 368

fluoxetine as , 37, 139, 307, 368, 389sertra l ine as , 145, 149, 181, 187tamoxi fen and, 486, 494

Selegi l ine, 146, 164, 178, 184–185monoamine oxidase inhibi tion by, 134, 173, 178–179, 184, 195, 199Parkinson disease and, 134, 167parkinsonism and, 147, 184, 186

SERM. See Selective estrogen receptor modi fiersSerotonin. See also Selective serotonin reuptake inhibi tors

fluoxetine and reuptake of, 155, 196headaches , sumatriptan mimicking, 349, 360

Serotonin syndrome, 172, 182, 349, 368Sertra l ine, 145, 149, 181, 187Sexual dys function

bupropion and, 145, 183s i ldenafi l and, 218, 264

SIADH. See Syndrome of inappropriateantidiuretic hormone secretion

Signal transduction, 45Si ldenafi l , 218, 264

Page 284: Pharmacology pre test self assessment and review (14th ed.)

Sinus bradycardia . See BradycardiaSinus tachycardia . See TachycardiaSkeleta l muscle tremors , a lbuterol caus ing, 329, 338Skeleta l muscles

neuromuscular blockers and, 84para lys is of, 59, 97

Slow-acting anti rheumatic drugs . See Disease-modi fying anti rheumatic drugsSMZ. See Sul famethoxazoleSodium bicarbonate

antacid effects of, 397, 401aspirin poisoning and, 354, 370for hyperuricemia, 356, 377

Sodium thiosul fatecyanide poisoning and, 503, 513for ni tropruss ide toxici ty, 215, 263

Sodium-potass ium adenos inetriphosphatase (Na +-K+-ATPase), inhibi tion by digoxin, 244

Somatic nervous system, 49–127Spironolactone, 257, 298, 307, 311–314, 416, 438

adrenal cortica l adenoma and, 303for adrenal cortica l tumor, 303, 312–314

SSRIs . See Selective serotonin reuptake inhibi torsStaphylococcus aureus, 451Status asthmaticus and i ts treatment, 332, 335, 362Status epi lepticus , drugs for, 142, 160, 176, 179, 370, 501

lorazepam for, 140, 142, 160, 510, 517(fos )phenytoin for, 142, 145, 176, 370, 501, 510

Steady-s tate concentration of drugs , 5, 10, 18, 43, 44Streptokinase. See ThrombolyticsStreptomycin, 321, 481–482Stroke, 57, 64, 88, 94, 99, 104, 129, 159, 160, 186, 233, 244, 271–272, 279, 346Strychnine poisoning, 515–516Succimer, 508, 520Succinylchol ine, 71, 111–112, 133, 161–162, 168, 170, 191, 459, 479, 522

aminoglycos ides prolonging effects of, 479comparison with nondepolarizing neuromuscular blockers , 71, 84mal ignant hyperthermia and, 161, 191phase II block and, 112

Sulbactam, 460, 481Sul famethoxazole

fol ic acid synthes is and, 453, 470kernicterus and, 451, 463TMP with, 448

Sul fapyridine, 403Sul fasa lazine, 389, 403Sul finpyrazone, 368Sul fonamides . See Antibiotics , sul fonamidesSul fonylureas , 434–435, 443–445

gl imepiride, 442gl ipizide, 442glyburide, 407, 419, 421, 442, 444tolbutamide, 195, 413, 418, 440, 442, 444, 518

Sumatriptancerebra l vasoconstrictor effects of, 174fluoxetine and, 139, 173, 352, 368, 489, 499, 524headaches , serotonin mimicked by, 349, 360, 368

Suprarenal medul la . See Adrenal medul laSupraventricular tachycardia , 204, 230–231, 242Sweat glands , innervation of, 50, 52, 82–84, 118, 123“Swine flu” 457Sympathetic nervous system, control of effectors , 15, 53, 85, 92, 114, 120, 124, 136, 179, 265, 270–271, 285–286Syndrome of inappropriate antidiuretic hormone secretion (SIADH)

conivaptan and, 301, 315Systemic vascular res is tance. See Tota l periphera l res is tance

Page 285: Pharmacology pre test self assessment and review (14th ed.)

TTachycardia , 59, 63, 94, 98, 105

adenos ine to terminate, 204, 219, 268β-adrenergic blockers for, 97, 105–106, 231, 288, 414, 434, 436baroreceptor-induced reflex, 112, 262, 283carotid s inus massage to terminate, 236edrophonium for paroxysmal , 268, 283vasopressors for, 291

Tachyphylaxis , 13, 39–40Tacrine, 154, 178, 195, 199Tadalafi l , 264Tamoxi fen, 30, 412, 431, 486, 494–495

breast cancer ri sk from, 412, 431SSRIs and, 489, 499

Tamsulos in, 76, 119–120Tazobactam, 460, 481TB. See Tuberculos isTerazos in, 120, 280. See a lso α-Adrenergic blockers .Terbuta l ine, 68, 108, 113, 328, 335Testosterone, 426, 431, 435Testosterone synthes is , 409, 426Tetany postthyroidectomy, 436Tetraca ine, 142, 176–177Tetracycl ines , 387, 399, 439, 465, 468, 472–473, 480, 482. See also Antibiotics , tetracycl ines

minera l sa l ts inhibi ting ora l absorption of, 473P450 inhibi tion by, 480

Theophyl l ine, 32, 329, 332–333, 335, 337asthma and, 329, 337–338toxici ty, 332–333, 337, 343, 382

Therapeutic index, 167, 337, 392Thiamine (Vi tamin B1), 388, 402

Thiamyla l , 132, 161Thiazide diuretics . See also Hydrochlorothiazide

and angiotens in modi fiers , 246, 249as antihypertens ive drugs , 201, 211, 246, 253for edema, 319urine electrolyte profi les and, 306

Thiazol idinediones (gl i tazones). See piogl i tazoneThioamides (thioamines). See Propyl thiouraci lThiobarbi turates . See Thiamyla l ; Thiopenta lThiopenta l , 132, 161, 168, 181

anesthetic duration of, 132, 145, 161, 168Thioridazine, 524Thrombocytopenia , heparin-induced. See Heparin-induced thrombocytopeniaThromboembol ism, 382, 416, 437, 495Thrombolytics , 227, 264, 268–270, 276, 279, 471

al teplase, 220, 263, 268, 276, 279streptokinase, 227, 269–270, 279

Thrombotic thrombocytopenic purpura, 250Thromboxane A2, 359, 360Thyroid hormones

and adrenergic receptor respons iveness , 427amiodarone and, 429effects of iodine on, 269, 429, 436, 445levothyroxine, 414, 423, 435

Thyroid s torm, 105, 427Ticlopidine, 249–250, 267, 270. See also ClopidogrelTimolol , 92, 98, 125, 240. See also β-Adrenergic blockersTinni tus , 204, 242, 269, 321, 368, 478Tiotropium, 96Tocolytic drugs , 113Tolbutamide, 195, 440, 442, 444, 518Tolerance to, 39–40, 166, 258, 273, 277, 307, 344–345, 469

barbi turates , 166benzodiazepines , 166

Page 286: Pharmacology pre test self assessment and review (14th ed.)

nitrovasodi lators , 225, 265, 277opioids , 150, 171, 184, 187

Topoisomerase II , 453, 471, 497Topotecan, 497Torsades de pointes , drugs inducing, 181, 252Torsemide, 299, 308, 311, 320–323. See also Furosemide; Loop diureticsTota l body water, 25–26Tota l periphera l res is tance

adrenergic control of, 88, 94, 203, 285, 286angiotens in I I and, 214in ca lculation of blood pressure, 94–95, 286vasodi lators and, 60–61

Trans ient i schemic attacks , 220Tranylcypromine, 164, 179, 181, 195. See also Monoamine oxidase inhibi torsTravelers diarrhea, 387, 401Triamcinolone, 416Triamterene, 216, 224, 230, 257, 271, 274, 283, 307, 311, 317, 321–322, 324, 326Trichomonias is , 449, 465Tricycl ic antidepressants , 73, 82, 84, 86, 104, 109, 119, 122, 135, 167, 173, 186, 197, 200

antichol inergic effects of, 135nocturnal enures is and, 197

Trihexyphenidyl , 93, 116, 143, 178, 185, 195, 198–199, 517Trimethaphan, 70, 73, 110, 114–115Trimethoprim

fol ic acid synthes is and, 453, 470sul famethoxazole with, 448, 465

Triptans . See sumatriptanTTP. See Thrombotic thrombocytopenic purpuraTuberculos is , 385, 450, 459–461, 465, 483

isoniazid with vi tamin B6 and, 459, 480

ri fampin for, 450, 466, 483Tubocurarine, 170TXA2. See Thromboxane A2Tyramine, 55, 86, 179Tyros ine hydroxylase, 56, 58, 88, 91

UUrate nephropathy, 310Uric acid, 348, 359Uric acid synthes is , 356, 369, 377–378Uricosuric drugs , 368. See also Antigout/antihyperuricemic drugs

antagonism by aspirin, 368probenecid, 359, 363–364, 369sul finpyrazone, 368

Urinary retention, bethanechol and, 384, 395Urinary tract infections

fluoroquinolones for, 464nitrofurantoin for, 448, 463–464trimethoprim and sul famethoxazole for, 448, 465

Urine, hypotonic, 295, 300, 315Urine glucose monitoring, 408, 426Ursodeoxychol ic acid, 387, 399Ursodiol , 387, 399Urticaria , 72, 103, 179, 220, 340, 444, 472Uterine contractions , drugs affecting, 72, 113, 430, 441Uterine relaxants

as adrenergic bronchodi lator, 335suppress ion of premature uterine contractions , 441

Uterine s timulants . See Oxytocin

VValproic acid, 192–193Vancomycin, 451, 453–454, 465–468, 471–473, 475, 480

MRSA and, 454, 471penici l l in and, 451, 468

Page 287: Pharmacology pre test self assessment and review (14th ed.)

Vancomycin-res is tant enterococci (VRE) infection, 462, 483Vardenafi l , 264Vasoconstrictor, 72, 92, 100–105, 108–109, 121–122, 126, 241, 244–245, 258, 275, 280Vasodi lation, 94–96, 98, 100–105, 112, 115, 117–118, 121, 126, 187, 190, 203, 206, 209, 219, 234, 241, 244–245

from hydra lazine, 219, 266–267periphera l , 267

Vasospastic angina, 67, 105verapami l/di l tiazem for, 211, 244–245

Vaughan-Wi l l iams antiarrhythmics class , 216Venlafaxine, 196Venti latory arrest, 171, 259, 331, 341, 521Ventricular ectopy, 202, 224, 239–240, 274, 292, 303Verapami l , 2, 25, 34, 174, 239–240, 242, 250, 252, 255, 262, 266, 268, 270, 273. See also Ca lcium channel blockers , ni fedipine

AV nodal conduction/block and, 221, 270constipation from, 218, 226dihydropyridines compared to, 242, 259, 262, 270, 273heart block from. See AV nodal conduction/blockfor hypertens ion, 407for PSVT, 204, 242vasospastic angina and, 211

Vigabatrin, 152, 192Vinblastine, 30, 493, 499Vinca a lka loids . See Vinblastine, vincris tineVincris tine, 486, 493Vitamin A, 384, 395, 399, 403, 437Vitamin B1. See Thiamine

Vitamin B6. See Pyridoxine

Vitamin D, 422, 428, 445Vitamin E, 178, 384, 395, 399, 402Vitamin K, 153, 253, 257, 263–264, 268–269, 276, 288, 384, 394–395, 399, 402, 437, 470–471, 492, 521

toxici ty, 384, 437Vitamins . See also specific individual vitamins fat soluble vs water soluble, 384, 395

toxici ty of, 383, 395Volume of dis tribution, 1, 13–14, 29, 32VRE infection. See Vancomycin-res is tant enterococci infection

WWarfarin, 21, 32, 39, 194, 220, 225, 243–244, 247, 249–250, 253, 263–264, 267–269, 272, 276, 288, 375, 382, 392–393, 402, 411, 455, 470–471, 474, 492, 495, 521

aspirin use with, 247, 267, 471dangers of loading doses , 243–244embryopathy from, 21myocardia l infarction and, 220, 250pregnancy and, 21, 212, 253prothrombotic s tate from, 243vi tamin K as antidote for, 263–264, 268–269, 276, 402, 471, 492, 521

Wernicke encephalopathy, 388, 402Wi lms tumor, 486

XXanthine oxidase

al lopurinol or febuxostat inhibi tion of, 369formation of uric acid by, 359–360, 369, 378, 498

Xenobiotic metabol i sm, 28, 42

YYohimbine, 59, 92

ZZafi rlukast, 338, 360

montelukast compared to, 329, 338–339, 345, 359Zero-order el imination of drugs , 18Zidovudine, 276, 448, 452, 460, 463–464, 467–469, 476Zolpidem, 166, 173–174, 193